From owner-obm-l@sucuri.mat.puc-rio.br Sat Jun 1 00:27:57 2002 Return-Path: Received: (from majordom@localhost) by sucuri.mat.puc-rio.br (8.9.3/8.9.3) id AAA13397 for obm-l-list; Sat, 1 Jun 2002 00:27:23 -0300 Received: from seki.bol.com.br (seki.bol.com.br [200.221.24.26]) by sucuri.mat.puc-rio.br (8.9.3/8.9.3) with ESMTP id AAA13393 for ; Sat, 1 Jun 2002 00:27:21 -0300 Received: from homeunean3of2j (200.221.24.192) by seki.bol.com.br (5.1.071) id 3CE51C8C00393AF5 for obm-l@mat.puc-rio.br; Sat, 1 Jun 2002 00:15:32 -0300 Message-ID: <001901c2091a$a55f4880$8403e2c8@homeunean3of2j> From: "Leonardo" To: References: <000d01c2090f$ee0a3140$b591dec8@igor> Subject: Re: [obm-l] Limites?!?! Date: Sat, 1 Jun 2002 00:15:11 -0300 MIME-Version: 1.0 Content-Type: multipart/alternative; boundary="----=_NextPart_000_0014_01C20901.64D5DDB0" X-Priority: 3 X-MSMail-Priority: Normal X-Mailer: Microsoft Outlook Express 6.00.2600.0000 X-MimeOLE: Produced By Microsoft MimeOLE V6.00.2600.0000 X-Sender-IP: 200.226.3.132 Sender: owner-obm-l@sucuri.mat.puc-rio.br Precedence: bulk Reply-To: obm-l@mat.puc-rio.br This is a multi-part message in MIME format. ------=_NextPart_000_0014_01C20901.64D5DDB0 Content-Type: text/plain; charset="iso-8859-1" Content-Transfer-Encoding: quoted-printable Ol=E1 colegas da lista =E9 a 1=AA vez que escrevo, eu tbm gostaria = de aprender mais sobre limites tenho pouca base sobre isso. No entanto a = resposta do limite abaixo seria sqrt(3)/3. =C9 poss=EDvel aplicar L' = Hospital para tirar a indetermina=E7=E3o? Valeu! Leo =20 =20 ----- Original Message -----=20 From: Igor Castro=20 To: obm-l@mat.puc-rio.br=20 Sent: Friday, May 31, 2002 10:59 PM Subject: [obm-l] Limites?!?! -------------------------------------------------------------------------= ----- Quer ter seu pr=F3prio endere=E7o na Internet? Garanta j=E1 o seu e ainda ganhe cinco e-mails personalizados. Dom=EDniosBOL - http://dominios.bol.com.br -------------------------------------------------------------------------= ----- Ol=E1 colegas da lista,=20 estou iniciando ainda neste assunto mas algu=E9m poderia dar uma ajuda = neste limite?=20 LIM [sqrt(x+2) + sqrt(x)] / x x-> -1 n=E3o consigo fugir da indetermina=E7=E3o ou de uma resposta com = "i"(=E9 valido para respostas de limite?) ou talvez o limite nem exista... deixo a analise para vcs.. : ) agrade=E7o desde j=E1... []'s ------=_NextPart_000_0014_01C20901.64D5DDB0 Content-Type: text/html; charset="iso-8859-1" Content-Transfer-Encoding: quoted-printable
    Ol=E1 colegas da = lista =E9 a 1=AA vez=20 que escrevo, eu tbm gostaria de aprender mais sobre limites tenho pouca = base=20 sobre isso. No entanto a resposta do limite abaixo seria  = sqrt(3)/3. =C9 poss=EDvel aplicar L' Hospital  para tirar a=20 indetermina=E7=E3o?
 
    Valeu!
 
    Leo
   
 
   
----- Original Message -----
From:=20 Igor = Castro=20
Sent: Friday, May 31, 2002 = 10:59 PM
Subject: [obm-l] = Limites?!?!


Quer ter seu pr=F3prio endere=E7o na Internet?
Garanta j=E1 o seu e = ainda ganhe=20 cinco e-mails personalizados.
Dom=EDniosBOL - http://dominios.bol.com.br

Ol=E1 colegas da lista,
estou iniciando ainda neste assunto = mas algu=E9m=20 poderia dar uma ajuda neste limite?
 
LIM         = [sqrt(x+2) +=20 sqrt(x)] / x
 x-> -1
 
n=E3o consigo fugir da = indetermina=E7=E3o ou de uma=20 resposta com "i"(=E9 valido para respostas de limite?)
ou talvez o limite nem exista... = deixo a analise=20 para vcs.. : )
agrade=E7o desde j=E1...
[]'s
------=_NextPart_000_0014_01C20901.64D5DDB0-- ========================================================================= Instruções para entrar na lista, sair da lista e usar a lista em http://www.mat.puc-rio.br/~nicolau/olimp/obm-l.html O administrador desta lista é ========================================================================= From owner-obm-l@sucuri.mat.puc-rio.br Sat Jun 1 10:13:31 2002 Return-Path: Received: (from majordom@localhost) by sucuri.mat.puc-rio.br (8.9.3/8.9.3) id KAA16143 for obm-l-list; Sat, 1 Jun 2002 10:13:03 -0300 Received: from gorgo.centroin.com.br (gorgo.centroin.com.br [200.225.63.128]) by sucuri.mat.puc-rio.br (8.9.3/8.9.3) with ESMTP id KAA16139 for ; Sat, 1 Jun 2002 10:13:01 -0300 Received: from centroin.com.br (du115b.rjo.centroin.com.br [200.225.57.115]) (authenticated bits=0) by gorgo.centroin.com.br (8.12.2/8.12.1) with ESMTP id g51D24OA010654 for ; Sat, 1 Jun 2002 10:02:05 -0300 (BRT) Message-ID: <3CF8C614.6040507@centroin.com.br> Date: Sat, 01 Jun 2002 10:03:16 -0300 From: Augusto =?ISO-8859-1?Q?C=E9sar?= Morgado User-Agent: Mozilla/5.0 (Windows; U; Win98; en-US; rv:0.9.4.1) Gecko/20020508 Netscape6/6.2.3 X-Accept-Language: en-us MIME-Version: 1.0 To: obm-l@mat.puc-rio.br Subject: Re: [obm-l] Limites?!?! References: <000d01c2090f$ee0a3140$b591dec8@igor> <001901c2091a$a55f4880$8403e2c8@homeunean3of2j> Content-Type: multipart/alternative; boundary="------------080202060900090003090903" Sender: owner-obm-l@sucuri.mat.puc-rio.br Precedence: bulk Reply-To: obm-l@mat.puc-rio.br --------------080202060900090003090903 Content-Type: text/plain; charset=ISO-8859-1; format=flowed Content-Transfer-Encoding: 8bit Esqueça. Ha um erro no enunciado. Nao se pode falar nesse limite. Morgado Leonardo wrote: > Olá colegas da lista é a 1ª vez que escrevo, eu tbm gostaria de > aprender mais sobre limites tenho pouca base sobre isso. No entanto a > resposta do limite abaixo seria sqrt(3)/3. É possível aplicar L' > Hospital para tirar a indeterminação? > > > > Valeu! > > > > Leo > > > > > > > > ----- Original Message ----- > > From:Igor Castro > > To: obm-l@mat.puc-rio.br > > Sent: Friday, May 31, 2002 10:59 PM > > Subject: [obm-l] Limites?!?! > > > ------------------------------------------------------------------------ > Quer ter seu próprio endereço na Internet? > Garanta já o seu e ainda ganhe cinco e-mails personalizados. > DomíniosBOL - http://dominios.bol.com.br > ------------------------------------------------------------------------ > Olá colegas da lista, > > estou iniciando ainda neste assunto mas alguém poderia dar uma > ajuda neste limite? > > > > LIM [sqrt(x+2) + sqrt(x)] / x > > x-> -1 > > > > não consigo fugir da indeterminação ou de uma resposta com "i"(é > valido para respostas de limite?) > > ou talvez o limite nem exista... deixo a analise para vcs.. : ) > > agradeço desde já... > > []'s > --------------080202060900090003090903 Content-Type: text/html; charset=us-ascii Content-Transfer-Encoding: 7bit Esqueça. Ha um erro no enunciado. Nao se pode falar nesse limite.
Morgado

Leonardo wrote:
    Olá colegas da lista é a 1ª vez que escrevo, eu tbm gostaria de aprender mais sobre limites tenho pouca base sobre isso. No entanto a resposta do limite abaixo seria  sqrt(3)/3. É possível aplicar L' Hospital  para tirar a indeterminação?
 
    Valeu!
 
    Leo
   
 
   
----- Original Message -----
Sent: Friday, May 31, 2002 10:59 PM
Subject: [obm-l] Limites?!?!


Quer ter seu próprio endereço na Internet?
Garanta já o seu e ainda ganhe cinco e-mails personalizados.
DomíniosBOL - http://dominios.bol.com.br

Olá colegas da lista,
estou iniciando ainda neste assunto mas alguém poderia dar uma ajuda neste limite?
 
LIM         [sqrt(x+2) + sqrt(x)] / x
 x-> -1
 
não consigo fugir da indeterminação ou de uma resposta com "i"(é valido para respostas de limite?)
ou talvez o limite nem exista... deixo a analise para vcs.. : )
agradeço desde já...
[]'s

--------------080202060900090003090903-- ========================================================================= Instruções para entrar na lista, sair da lista e usar a lista em http://www.mat.puc-rio.br/~nicolau/olimp/obm-l.html O administrador desta lista é ========================================================================= From owner-obm-l@sucuri.mat.puc-rio.br Sat Jun 1 13:29:00 2002 Return-Path: Received: (from majordom@localhost) by sucuri.mat.puc-rio.br (8.9.3/8.9.3) id NAA17546 for obm-l-list; Sat, 1 Jun 2002 13:24:38 -0300 Received: from www.zipmail.com.br (smtp.zipmail.com.br [200.187.242.10]) by sucuri.mat.puc-rio.br (8.9.3/8.9.3) with ESMTP id NAA17542 for ; Sat, 1 Jun 2002 13:24:36 -0300 From: ghaeser@zipmail.com.br Received: from [200.158.6.12] by www.zipmail.com.br with HTTP; Sat, 1 Jun 2002 13:13:13 -0300 Message-ID: <3CF858CF0000115E@www.zipmail.com.br> Date: Sat, 1 Jun 2002 13:13:13 -0300 In-Reply-To: <3CF8C614.6040507@centroin.com.br> Subject: [obm-l] =?iso-8859-1?Q?Polinomios?= To: obm-l@mat.puc-rio.br MIME-Version: 1.0 Content-Type: text/plain; charset="iso-8859-1" Content-Transfer-Encoding: 8bit X-MIME-Autoconverted: from quoted-printable to 8bit by sucuri.mat.puc-rio.br id NAA17543 Sender: owner-obm-l@sucuri.mat.puc-rio.br Precedence: bulk Reply-To: obm-l@mat.puc-rio.br olá pessoal da lista, um amigo me mostrou uma tal regra de escrever um polinômio em sua forma binomial, a regra era a seguinte: seja P(x) um polinomio de grau n, então faça D1(x)=P(x+1)-P(x), e Dj+1(x)=Dj(x+1)-Dj(x), 1<=j<=n-1 então P(x) pode ser escrito como: P(x)=P(0)(x,0)+D1(0)(x,1)+D2(0)(x,2)+..+Dn(0)(x,n) onde (x,i), é o numero binomial (x escolhe i). fazendo algumas contas encontrei que : P(x)=sumk((x,k)*sumj(k,j)*P(j)), onde sumk = somatório de k=0 até n e sumj = somatório de j=0 até k e será que alguém poderia me ajudar a demonstrar isso ?? .. (ou dar um contra-exemplo, pois ainda não sei se é verdade) "Mathematicus nascitur, non fit" Matemáticos não são feitos, eles nascem --------------------------------------- Gabriel Haeser www.gabas.cjb.net ------------------------------------------ Use o melhor sistema de busca da Internet Radar UOL - http://www.radaruol.com.br ========================================================================= Instruções para entrar na lista, sair da lista e usar a lista em http://www.mat.puc-rio.br/~nicolau/olimp/obm-l.html O administrador desta lista é ========================================================================= From owner-obm-l@sucuri.mat.puc-rio.br Sat Jun 1 16:58:29 2002 Return-Path: Received: (from majordom@localhost) by sucuri.mat.puc-rio.br (8.9.3/8.9.3) id QAA19136 for obm-l-list; Sat, 1 Jun 2002 16:56:40 -0300 Received: from imo-d02.mx.aol.com (imo-d02.mx.aol.com [205.188.157.34]) by sucuri.mat.puc-rio.br (8.9.3/8.9.3) with ESMTP id QAA19132 for ; Sat, 1 Jun 2002 16:56:38 -0300 From: SSayajinGoten@aol.com Received: from SSayajinGoten@aol.com by imo-d02.mx.aol.com (mail_out_v32.5.) id z.113.1256ac21 (4592) for ; Sat, 1 Jun 2002 15:44:58 -0400 (EDT) Message-ID: <113.1256ac21.2a2a7e39@aol.com> Date: Sat, 1 Jun 2002 15:44:57 EDT Subject: [obm-l] Regra d 3 (cinistra) To: obm-l@mat.puc-rio.br MIME-Version: 1.0 Content-Type: text/plain; charset="ISO-8859-1" X-Mailer: AOL 4.0 for Windows sub 108 Content-Transfer-Encoding: 8bit X-MIME-Autoconverted: from quoted-printable to 8bit by sucuri.mat.puc-rio.br id QAA19133 Sender: owner-obm-l@sucuri.mat.puc-rio.br Precedence: bulk Reply-To: obm-l@mat.puc-rio.br Fala Gênios, Tudo blz? Bem, to com um problema de regra de 3 que eu não conseguir fazer, se alguem puder me ajudar..: Após o atentado terrorista às torres gêmeas do WORLD TRADE CENTER, em nova Iorque, os bombeiros iniciaram uma operação de emergência para evacuar os prédios vizinhos. Em um desses prédios, o elevador tinha capacidade para levar 20 adultos ou 24 crianças. Ao parar em um dos andares, havia no interior do elevador 8 crianças e 2 adultos. O número maximo de adultos que ainda poreriam entrar no elevador era de ? ========================================================================= Instruções para entrar na lista, sair da lista e usar a lista em http://www.mat.puc-rio.br/~nicolau/olimp/obm-l.html O administrador desta lista é ========================================================================= From owner-obm-l@sucuri.mat.puc-rio.br Sat Jun 1 17:56:30 2002 Return-Path: Received: (from majordom@localhost) by sucuri.mat.puc-rio.br (8.9.3/8.9.3) id RAA19802 for obm-l-list; Sat, 1 Jun 2002 17:55:11 -0300 Received: from shannon.bol.com.br (shannon.bol.com.br [200.221.24.13]) by sucuri.mat.puc-rio.br (8.9.3/8.9.3) with ESMTP id RAA19798 for ; Sat, 1 Jun 2002 17:55:09 -0300 Received: from bol.com.br (200.221.24.66) by shannon.bol.com.br (5.1.071) id 3CE1A94700441B25 for obm-l@mat.puc-rio.br; Sat, 1 Jun 2002 17:43:02 -0300 Date: Sat, 1 Jun 2002 17:43:03 -0300 Message-Id: Subject: Re:[obm-l] Regra d 3 (cinistra) MIME-Version: 1.0 Content-Type: text/plain;charset="iso-8859-1" From: "rafaelc.l" To: obm-l@mat.puc-rio.br X-XaM3-API-Version: 2.4.3.4.4 X-SenderIP: 200.176.166.8 Content-Transfer-Encoding: 8bit X-MIME-Autoconverted: from quoted-printable to 8bit by sucuri.mat.puc-rio.br id RAA19799 Sender: owner-obm-l@sucuri.mat.puc-rio.br Precedence: bulk Reply-To: obm-l@mat.puc-rio.br - Seja: A-número de adultos C-número de crianças - A/C=20/24=5/6 - Como tinha 8 crianças no elevador, ainda tinha esaço para 24-8=16 crianças, o que equivale a 5/6x16=40/3 adultos. Como havia dois adultos, 40/3-2=34/3 é o numero máximo de adultos que poderiam entrar. O maior inteiro que representa 34/3 é 11. Fala Gênios, Tudo blz? > > Bem, to com um problema de regra de 3 que eu não conseguir fazer, se alguem > puder me ajudar..: > > Após o atentado terrorista às torres gêmeas do WORLD TRADE CENTER, em nova > Iorque, os bombeiros iniciaram uma operação de emergência para evacuar os > prédios vizinhos. Em um desses prédios, o elevador tinha capacidade para > levar 20 adultos ou 24 crianças. Ao parar em um dos andares, havia no > interior do elevador 8 crianças e 2 adultos. O número maximo de adultos que > ainda poreriam entrar no elevador era de ? > > > ========================================================== =============== > Instruções para entrar na lista, sair da lista e usar a lista em > http://www.mat.puc-rio.br/~nicolau/olimp/obm-l.html > O administrador desta lista é > ========================================================== =============== > __________________________________________________________________________ Quer ter seu próprio endereço na Internet? Garanta já o seu e ainda ganhe cinco e-mails personalizados. DomíniosBOL - http://dominios.bol.com.br ========================================================================= Instruções para entrar na lista, sair da lista e usar a lista em http://www.mat.puc-rio.br/~nicolau/olimp/obm-l.html O administrador desta lista é ========================================================================= From owner-obm-l@sucuri.mat.puc-rio.br Sat Jun 1 18:18:41 2002 Return-Path: Received: (from majordom@localhost) by sucuri.mat.puc-rio.br (8.9.3/8.9.3) id SAA20031 for obm-l-list; Sat, 1 Jun 2002 18:17:12 -0300 Received: from hotmail.com (f92.pav1.hotmail.com [64.4.31.92]) by sucuri.mat.puc-rio.br (8.9.3/8.9.3) with ESMTP id SAA20026 for ; Sat, 1 Jun 2002 18:17:09 -0300 Received: from mail pickup service by hotmail.com with Microsoft SMTPSVC; Sat, 1 Jun 2002 14:05:46 -0700 Received: from 200.151.62.52 by pv1fd.pav1.hotmail.msn.com with HTTP; Sat, 01 Jun 2002 21:05:46 GMT X-Originating-IP: [200.151.62.52] From: "Adriano Almeida Faustino" To: obm-l@mat.puc-rio.br Subject: Re: [obm-l] Analise Combinatoria Date: Sat, 01 Jun 2002 21:05:46 +0000 Mime-Version: 1.0 Content-Type: text/plain; charset=iso-8859-1; format=flowed Message-ID: X-OriginalArrivalTime: 01 Jun 2002 21:05:46.0691 (UTC) FILETIME=[19862130:01C209B0] Sender: owner-obm-l@sucuri.mat.puc-rio.br Precedence: bulk Reply-To: obm-l@mat.puc-rio.br E` depois eu saquei a besteira que falei.Mas entao, bastaria ele falar que quer um grupo de tres alunos,que nao tenha alunos designados por numeros consecutivos,ja que onde tem 3 numeros consecutivos, tem 2 ?E se ele falasse uma comissao de 3 alunos,onde nao fazem parte 3 alunos designados por numeros consecutivos,daria na mesma? Por exemplo eu poderia usar Kaplansky aqui Para uma conferencia realizada no auditorio do IME,foram reservados 7 lugares,que serao ocupados por 7 oficiais superiores.Sabendo-se que 3 sao generais,2 almirantes e 2,brigadeiros e que estes lugares estao na primeira fileira,um ao lado do outro,determine de quantos modos podemos acomoda-los,sem que haja sentados juntos oficiais de uma mesma arma. Obrigado. []`s Adriano. >From: Augusto César Morgado >Reply-To: obm-l@mat.puc-rio.br >To: obm-l@mat.puc-rio.br >Subject: Re: [obm-l] Analise Combinatoria >Date: Thu, 30 May 2002 15:40:03 -0300 > >Claro, o que eu fiz foi deduzir localmente o lema de Kaplansky. Mas nao >entendi o final do seu comentario. O que seria dispensavel no enunciado >da questao seria o "tres" e nao o dois. >Morgado > >Adriano Almeida Faustino wrote: > >>O que fez praticamente fez foi o 1ºlema de Kaplansky ( C(n-p+1,p) >>),para p=3 ?E o que adiantou ele falar ``dois` ou tres alunos` ?,o que >>esse `dois` esta influindo? >>[]`s >>Adriano. >> >> >>>From: Augusto Cesar de Oliveira Morgado >>>Reply-To: obm-l@mat.puc-rio.br >>>To: obm-l@mat.puc-rio.br >>>Subject: Re: [obm-l] Analise Combinatoria >>>Date: Mon, 27 May 2002 11:10:13 -0300 (EST) >>> >>> >>>Uma solucao mais elementar seria imaginar os alunos 1 2 ... n e >>>marcar com o sinal de + os escolhidos e com o sinal - os não >>>escolhidos. Formaremos uma fila com 3 sinais + e n-3 sinais -, nao >>>podendo haver dois sinais + consecutivos. Para isso, ponha os n-3 >>>sinais - em fila e vejamos de quantos modos podemos enfiar entre eles >>>(ou antes do primeiro ou depois do ultimo) os sinais +. >>>Sao n-2 espaços dos quais devemos escolher 3 e a resposta eh C(n-2,2). >>> >>>Em Mon, 27 May 2002 00:59:54 -0300, Paulo Rodrigues >>> disse: >>> >>> > : Considere uma turma com n alunos ,numerados de 1 a n. >>> > : Deseja-se organizar uma comissao de 3 alunos.De quantas maneiras >>>pode ser >>> > : formada esta comissao,de modo que nao facam parte da mesma dois >>>ou tres >>> > : alunosdesignados por numeros consecutivos ? >>> > >>> > Seja C={x, y, z} uma comissão satisfazendo às condições do >>>problema, com >>> > x>>elementos pois >>> > z > y +1 > x+2. C1 é necessariamente um subconjunto de >>>[n-2]={1,2,...,n-2} >>> > e prova-se facilmente que essa função que leva C em C1 é uma >>>bijeção do >>> > conjunto considerado no conjunto dos 3-subconjuntos de [n-2]. >>>Portanto, o >>> > número de subconjuntos C é igual ao número de subconjuntos C1, igual a >>> > binomial(n-2,3) = (n-2)(n-3)(n-4)/6. >>> > >>> > >>> > --- >>> > esta mensagem não contém vírus! >>> > Checked by AVG anti-virus system (http://www.grisoft.com). >>> > Version: 6.0.363 / Virus Database: 201 - Release Date: 21/05/2002 >>> > >>> > >>>========================================================================= >>> >>> > Instruções para entrar na lista, sair da lista e usar a lista em >>> > http://www.mat.puc-rio.br/~nicolau/olimp/obm-l.html >>> > O administrador desta lista é >>> > >>>========================================================================= >>> >>> > >>> > >>> >>>========================================================================= >>> >>>Instruções para entrar na lista, sair da lista e usar a lista em >>>http://www.mat.puc-rio.br/~nicolau/olimp/obm-l.html >>>O administrador desta lista é >>>========================================================================= >>> >> >> >> >>_________________________________________________________________ >>Chegou o novo MSN Explorer. Instale já. É gratuito: >>http://explorer.msn.com.br >> >>========================================================================= >>Instruções para entrar na lista, sair da lista e usar a lista em >>http://www.mat.puc-rio.br/~nicolau/olimp/obm-l.html >>O administrador desta lista é >>========================================================================= >> >> > > >========================================================================= >Instruções para entrar na lista, sair da lista e usar a lista em >http://www.mat.puc-rio.br/~nicolau/olimp/obm-l.html >O administrador desta lista é >========================================================================= _________________________________________________________________ Una-se ao maior serviço de email do mundo: o MSN Hotmail. http://www.hotmail.com ========================================================================= Instruções para entrar na lista, sair da lista e usar a lista em http://www.mat.puc-rio.br/~nicolau/olimp/obm-l.html O administrador desta lista é ========================================================================= From owner-obm-l@sucuri.mat.puc-rio.br Sat Jun 1 18:20:29 2002 Return-Path: Received: (from majordom@localhost) by sucuri.mat.puc-rio.br (8.9.3/8.9.3) id SAA20060 for obm-l-list; Sat, 1 Jun 2002 18:19:12 -0300 Received: from hotmail.com (law2-f80.hotmail.com [216.32.181.80]) by sucuri.mat.puc-rio.br (8.9.3/8.9.3) with ESMTP id SAA20056 for ; Sat, 1 Jun 2002 18:19:09 -0300 Received: from mail pickup service by hotmail.com with Microsoft SMTPSVC; Sat, 1 Jun 2002 14:07:47 -0700 Received: from 200.160.246.163 by lw2fd.hotmail.msn.com with HTTP; Sat, 01 Jun 2002 21:07:46 GMT X-Originating-IP: [200.160.246.163] From: "Paulo Santa Rita" To: obm-l@mat.puc-rio.br Subject: Re: [obm-l] Limites?!?! Date: Sat, 01 Jun 2002 21:07:46 +0000 Mime-Version: 1.0 Content-Type: text/plain; charset=iso-8859-1; format=flowed Message-ID: X-OriginalArrivalTime: 01 Jun 2002 21:07:47.0075 (UTC) FILETIME=[61474530:01C209B0] Sender: owner-obm-l@sucuri.mat.puc-rio.br Precedence: bulk Reply-To: obm-l@mat.puc-rio.br Ola Igor e demais colegas desta lista, 1)Voce pode obter como resposta ao calculo de um limite um valor complexo, sem problemas. Basicamente tudo que voce viu valer no dominio real vai valer no dominio dos complexos, com pequenas variacoes. Um bom livro sobre este assunto e : Funcoes de uma variavel complexa Alcides Lins Neto Projeto Euclides - IMPA 2)Voce so pode aplicar a regra de L'Hopital para levantar uma indeterminacao se ela for do tipo 0/0 ou INF/INF. Todavia, muitas outras indeterminacoes que nao se enquadram nesta categoria podem ser reduzidas a elas por uma manipulacam elementar . Exemplo : LIM F(X)*G(X) e tipo 0*INF Voce coloca : LIM [ (1/(1/F(X)))*G(X)] e fica tipo INF/INF Se voce colocar LIM [ F(X)*(1/(1/G(X)))] fica 0/0 Nos dois casos acima, apos a manipulacao elementar, voce pode aplicar a regra de L´Hopital. No livro : Exercicios de Analise Matematica Demidovitch Editora Mir Voce encontra os artificios basicos para lidar com todas as formas basicas de indeterminacao, com exececao do caso : 3) A regra de L´hopital nao e uma panaceia universal ... Ela apenas diz que dois limites tem o mesmo valor, mas pode ser que a razao entre as derivadas seja muito mais complexo e trabalhoso que um artificio inteligente. Exemplo : Calcule : LIM [(A^X - X*Ln(A))/(B^X - X*Ln(B))] Quando X->0. Ln(A) e o logaritmo neperiano de A. Se LIM F(X)/G(X) = 1 e F(X) e G(X) tendem a zero, dizemos que eles sao infesimos equivalentes. Em uma expressao onde ha infinitesimos, eles podem ser trocados que o valor do limite nao se altera. Muitos limites Muito dificeis sao desta natureza : Calcular limites e mais uma arte que uma ciencia. As vezes o caminho mais obvio complica tanto que e mais facil usar um artificio. Exemplo : Calcular o limite : LIM(X->0) [(ARCTG(X)/X)^(1/(X^2))] Para o seu caso talvez seja interessante voce saber que : LIM [((1+X)^A - 1)/X ] = A Quando X tende a zero Alias, os tres exemplos acima sao exercicios interessantes ... Um abraco Paulo Santa Rita 7,1804,010602 >From: "Igor Castro" >Reply-To: obm-l@mat.puc-rio.br >To: >Subject: [obm-l] Limites?!?! >Date: Fri, 31 May 2002 22:59:12 -0300 > >Olá colegas da lista, >estou iniciando ainda neste assunto mas alguém poderia dar uma ajuda neste >limite? > >LIM [sqrt(x+2) + sqrt(x)] / x > x-> -1 > >não consigo fugir da indeterminação ou de uma resposta com "i"(é valido >para respostas de limite?) >ou talvez o limite nem exista... deixo a analise para vcs.. : ) >agradeço desde já... >[]'s _________________________________________________________________ Chegou o novo MSN Explorer. Instale já. É gratuito: http://explorer.msn.com.br ========================================================================= Instruções para entrar na lista, sair da lista e usar a lista em http://www.mat.puc-rio.br/~nicolau/olimp/obm-l.html O administrador desta lista é ========================================================================= From owner-obm-l@sucuri.mat.puc-rio.br Sat Jun 1 18:39:45 2002 Return-Path: Received: (from majordom@localhost) by sucuri.mat.puc-rio.br (8.9.3/8.9.3) id SAA20710 for obm-l-list; Sat, 1 Jun 2002 18:38:26 -0300 Received: from imo-m03.mx.aol.com (imo-m03.mx.aol.com [64.12.136.6]) by sucuri.mat.puc-rio.br (8.9.3/8.9.3) with ESMTP id SAA20705 for ; Sat, 1 Jun 2002 18:38:23 -0300 From: SSayajinGoten@aol.com Received: from SSayajinGoten@aol.com by imo-m03.mx.aol.com (mail_out_v32.5.) id z.171.e6d83f3 (4159) for ; Sat, 1 Jun 2002 17:26:58 -0400 (EDT) Message-ID: <171.e6d83f3.2a2a9622@aol.com> Date: Sat, 1 Jun 2002 17:26:58 EDT Subject: Re: [obm-l] Regra d 3 (cinistra) To: obm-l@mat.puc-rio.br MIME-Version: 1.0 Content-Type: text/plain; charset="ISO-8859-1" X-Mailer: AOL 4.0 for Windows sub 108 Content-Transfer-Encoding: 8bit X-MIME-Autoconverted: from quoted-printable to 8bit by sucuri.mat.puc-rio.br id SAA20706 Sender: owner-obm-l@sucuri.mat.puc-rio.br Precedence: bulk Reply-To: obm-l@mat.puc-rio.br Hum..a sim, valews rapa. Olha eu tenho outra questão que eu tambem não consegui resolver, só que eu não sei como fazer sinal de raiz no pc, então eu vou escrever, e ai voces copiem a questão, fassam, e depois, se possivel, me retornem , Valews! Conseidere a equação do 2º grau em x tal que ax²+bx+c=0(zero),onde, a,b,c são números reais e que "a" diferente de 0(zero), sabendo que 1 e 2 são raízes e que a(0,5 - raiz quadrada de 3)² + b(0,5 - raiz quadrada de 3) + c =1997. O valor de a(2,5 + raiz quadrada de 3)² + b(2,5 + raiz quadrada de 3)+ c, é ? ========================================================================= Instruções para entrar na lista, sair da lista e usar a lista em http://www.mat.puc-rio.br/~nicolau/olimp/obm-l.html O administrador desta lista é ========================================================================= From owner-obm-l@sucuri.mat.puc-rio.br Sat Jun 1 21:17:21 2002 Return-Path: Received: (from majordom@localhost) by sucuri.mat.puc-rio.br (8.9.3/8.9.3) id VAA22321 for obm-l-list; Sat, 1 Jun 2002 21:15:42 -0300 Received: from gorgo.centroin.com.br (gorgo.centroin.com.br [200.225.63.128]) by sucuri.mat.puc-rio.br (8.9.3/8.9.3) with ESMTP id VAA22317 for ; Sat, 1 Jun 2002 21:15:40 -0300 Received: from centroin.com.br (du95b.rjo.centroin.com.br [200.225.57.95]) (authenticated bits=0) by gorgo.centroin.com.br (8.12.2/8.12.1) with ESMTP id g5204UOA004012 for ; Sat, 1 Jun 2002 21:04:30 -0300 (BRT) Message-ID: <3CF96154.1050203@centroin.com.br> Date: Sat, 01 Jun 2002 21:05:40 -0300 From: Augusto =?ISO-8859-1?Q?C=E9sar?= Morgado User-Agent: Mozilla/5.0 (Windows; U; Win98; en-US; rv:0.9.4.1) Gecko/20020508 Netscape6/6.2.3 X-Accept-Language: en-us MIME-Version: 1.0 To: obm-l@mat.puc-rio.br Subject: Re: [obm-l] Polinomios References: <3CF858CF0000115E@www.zipmail.com.br> Content-Type: text/plain; charset=ISO-8859-1; format=flowed Content-Transfer-Encoding: 8bit Sender: owner-obm-l@sucuri.mat.puc-rio.br Precedence: bulk Reply-To: obm-l@mat.puc-rio.br Isso eh verdade. Veja qualquer livro de Calculode Diferenças Finitas ( o do Richardson eh simples, pequeno e otimo) ou um bom livro de Calculo Numerico, no capitulo de interpolaçao. Morgado ghaeser@zipmail.com.br wrote: >olá pessoal da lista, >um amigo me mostrou uma tal regra de escrever um polinômio em sua forma >binomial, a regra era a seguinte: > >seja P(x) um polinomio de grau n, então faça D1(x)=P(x+1)-P(x), e Dj+1(x)=Dj(x+1)-Dj(x), >1<=j<=n-1 >então P(x) pode ser escrito como: > >P(x)=P(0)(x,0)+D1(0)(x,1)+D2(0)(x,2)+..+Dn(0)(x,n) >onde (x,i), é o numero binomial (x escolhe i). > >fazendo algumas contas encontrei que : >P(x)=sumk((x,k)*sumj(k,j)*P(j)), onde >sumk = somatório de k=0 até n e >sumj = somatório de j=0 até k e > >será que alguém poderia me ajudar a demonstrar isso ?? .. (ou dar um contra-exemplo, >pois ainda não sei se é verdade) > > > >"Mathematicus nascitur, non fit" >Matemáticos não são feitos, eles nascem >--------------------------------------- >Gabriel Haeser >www.gabas.cjb.net > > >------------------------------------------ >Use o melhor sistema de busca da Internet >Radar UOL - http://www.radaruol.com.br > > > >========================================================================= >Instruções para entrar na lista, sair da lista e usar a lista em >http://www.mat.puc-rio.br/~nicolau/olimp/obm-l.html >O administrador desta lista é >========================================================================= > > ========================================================================= Instruções para entrar na lista, sair da lista e usar a lista em http://www.mat.puc-rio.br/~nicolau/olimp/obm-l.html O administrador desta lista é ========================================================================= From owner-obm-l@sucuri.mat.puc-rio.br Sat Jun 1 21:19:10 2002 Return-Path: Received: (from majordom@localhost) by sucuri.mat.puc-rio.br (8.9.3/8.9.3) id VAA22349 for obm-l-list; Sat, 1 Jun 2002 21:17:53 -0300 Received: from gorgo.centroin.com.br (gorgo.centroin.com.br [200.225.63.128]) by sucuri.mat.puc-rio.br (8.9.3/8.9.3) with ESMTP id VAA22345 for ; Sat, 1 Jun 2002 21:17:51 -0300 Received: from centroin.com.br (du95b.rjo.centroin.com.br [200.225.57.95]) (authenticated bits=0) by gorgo.centroin.com.br (8.12.2/8.12.1) with ESMTP id g5206vOA004283 for ; Sat, 1 Jun 2002 21:06:58 -0300 (BRT) Message-ID: <3CF961E9.6080507@centroin.com.br> Date: Sat, 01 Jun 2002 21:08:09 -0300 From: Augusto =?ISO-8859-1?Q?C=E9sar?= Morgado User-Agent: Mozilla/5.0 (Windows; U; Win98; en-US; rv:0.9.4.1) Gecko/20020508 Netscape6/6.2.3 X-Accept-Language: en-us MIME-Version: 1.0 To: obm-l@mat.puc-rio.br Subject: Re: [obm-l] Polinomios References: <3CF858CF0000115E@www.zipmail.com.br> Content-Type: text/plain; charset=ISO-8859-1; format=flowed Content-Transfer-Encoding: 8bit Sender: owner-obm-l@sucuri.mat.puc-rio.br Precedence: bulk Reply-To: obm-l@mat.puc-rio.br Fui precipitado no e-mail anterior. O que eu disse estar correto foi a formula que o seu amigo mostrou, formula essa que eh devida a Newton. A sua conclusao ainda vou examinar. Morgado ghaeser@zipmail.com.br wrote: >olá pessoal da lista, >um amigo me mostrou uma tal regra de escrever um polinômio em sua forma >binomial, a regra era a seguinte: > >seja P(x) um polinomio de grau n, então faça D1(x)=P(x+1)-P(x), e Dj+1(x)=Dj(x+1)-Dj(x), >1<=j<=n-1 >então P(x) pode ser escrito como: > >P(x)=P(0)(x,0)+D1(0)(x,1)+D2(0)(x,2)+..+Dn(0)(x,n) >onde (x,i), é o numero binomial (x escolhe i). > >fazendo algumas contas encontrei que : >P(x)=sumk((x,k)*sumj(k,j)*P(j)), onde >sumk = somatório de k=0 até n e >sumj = somatório de j=0 até k e > >será que alguém poderia me ajudar a demonstrar isso ?? .. (ou dar um contra-exemplo, >pois ainda não sei se é verdade) > > > >"Mathematicus nascitur, non fit" >Matemáticos não são feitos, eles nascem >--------------------------------------- >Gabriel Haeser >www.gabas.cjb.net > > >------------------------------------------ >Use o melhor sistema de busca da Internet >Radar UOL - http://www.radaruol.com.br > > > >========================================================================= >Instruções para entrar na lista, sair da lista e usar a lista em >http://www.mat.puc-rio.br/~nicolau/olimp/obm-l.html >O administrador desta lista é >========================================================================= > > ========================================================================= Instruções para entrar na lista, sair da lista e usar a lista em http://www.mat.puc-rio.br/~nicolau/olimp/obm-l.html O administrador desta lista é ========================================================================= From owner-obm-l@sucuri.mat.puc-rio.br Sat Jun 1 21:27:30 2002 Return-Path: Received: (from majordom@localhost) by sucuri.mat.puc-rio.br (8.9.3/8.9.3) id VAA22472 for obm-l-list; Sat, 1 Jun 2002 21:26:11 -0300 Received: from gorgo.centroin.com.br (gorgo.centroin.com.br [200.225.63.128]) by sucuri.mat.puc-rio.br (8.9.3/8.9.3) with ESMTP id VAA22468 for ; Sat, 1 Jun 2002 21:26:09 -0300 Received: from centroin.com.br (du95b.rjo.centroin.com.br [200.225.57.95]) (authenticated bits=0) by gorgo.centroin.com.br (8.12.2/8.12.1) with ESMTP id g520F7OA005024 for ; Sat, 1 Jun 2002 21:15:08 -0300 (BRT) Message-ID: <3CF963D3.8040605@centroin.com.br> Date: Sat, 01 Jun 2002 21:16:19 -0300 From: Augusto =?ISO-8859-1?Q?C=E9sar?= Morgado User-Agent: Mozilla/5.0 (Windows; U; Win98; en-US; rv:0.9.4.1) Gecko/20020508 Netscape6/6.2.3 X-Accept-Language: en-us MIME-Version: 1.0 To: obm-l@mat.puc-rio.br Subject: Re: [obm-l] Polinomios References: <3CF858CF0000115E@www.zipmail.com.br> Content-Type: text/plain; charset=ISO-8859-1; format=flowed Content-Transfer-Encoding: 8bit Sender: owner-obm-l@sucuri.mat.puc-rio.br Precedence: bulk Reply-To: obm-l@mat.puc-rio.br ghaeser@zipmail.com.br wrote: >olá pessoal da lista, >um amigo me mostrou uma tal regra de escrever um polinômio em sua forma >binomial, a regra era a seguinte: > >seja P(x) um polinomio de grau n, então faça D1(x)=P(x+1)-P(x), e Dj+1(x)=Dj(x+1)-Dj(x), >1<=j<=n-1 >então P(x) pode ser escrito como: > >P(x)=P(0)(x,0)+D1(0)(x,1)+D2(0)(x,2)+..+Dn(0)(x,n) >onde (x,i), é o numero binomial (x escolhe i). > >fazendo algumas contas encontrei que : >P(x)=sumk((x,k)*sumj(k,j)*P(j)), > AQUI HA UM ERRO . ONDE ESTA P(j) DEVERIA ESTAR [(-1)^j ]* P(j) >onde >sumk = somatório de k=0 até n e >sumj = somatório de j=0 até k e > >será que alguém poderia me ajudar a demonstrar isso ?? .. (ou dar um contra-exemplo, >pois ainda não sei se é verdade) > > > >"Mathematicus nascitur, non fit" >Matemáticos não são feitos, eles nascem >--------------------------------------- >Gabriel Haeser >www.gabas.cjb.net > > >------------------------------------------ >Use o melhor sistema de busca da Internet >Radar UOL - http://www.radaruol.com.br > > > >========================================================================= >Instruções para entrar na lista, sair da lista e usar a lista em >http://www.mat.puc-rio.br/~nicolau/olimp/obm-l.html >O administrador desta lista é >========================================================================= > > ========================================================================= Instruções para entrar na lista, sair da lista e usar a lista em http://www.mat.puc-rio.br/~nicolau/olimp/obm-l.html O administrador desta lista é ========================================================================= From owner-obm-l@sucuri.mat.puc-rio.br Sun Jun 2 01:41:57 2002 Return-Path: Received: (from majordom@localhost) by sucuri.mat.puc-rio.br (8.9.3/8.9.3) id BAA24492 for obm-l-list; Sun, 2 Jun 2002 01:38:35 -0300 Received: from hotmail.com (f100.pav2.hotmail.com [64.4.37.100]) by sucuri.mat.puc-rio.br (8.9.3/8.9.3) with ESMTP id BAA24488 for ; Sun, 2 Jun 2002 01:38:33 -0300 Received: from mail pickup service by hotmail.com with Microsoft SMTPSVC; Sat, 1 Jun 2002 21:27:11 -0700 Received: from 200.199.29.193 by pv2fd.pav2.hotmail.msn.com with HTTP; Sun, 02 Jun 2002 04:27:11 GMT X-Originating-IP: [200.199.29.193] From: "maurikleber araujo" To: obm-l@mat.puc-rio.br Subject: [obm-l] como se preparar para obm nivel universitario Date: Sun, 02 Jun 2002 01:27:11 -0300 Mime-Version: 1.0 Content-Type: text/plain; charset=iso-8859-1; format=flowed Message-ID: X-OriginalArrivalTime: 02 Jun 2002 04:27:11.0554 (UTC) FILETIME=[C3BB8220:01C209ED] Sender: owner-obm-l@sucuri.mat.puc-rio.br Precedence: bulk Reply-To: obm-l@mat.puc-rio.br oi pessoal nao sou do curso de bacharelato de mat ,faco computacao gostaria de saber a melhor forma de se preparar para as olimpiadas no nivel universitario tipo ,que disciplinas sao mais exigidas ..... _________________________________________________________________ Converse com amigos on-line, conheça o MSN Messenger: http://messenger.msn.com ========================================================================= Instruções para entrar na lista, sair da lista e usar a lista em http://www.mat.puc-rio.br/~nicolau/olimp/obm-l.html O administrador desta lista é ========================================================================= From owner-obm-l@sucuri.mat.puc-rio.br Sun Jun 2 10:03:59 2002 Return-Path: Received: (from majordom@localhost) by sucuri.mat.puc-rio.br (8.9.3/8.9.3) id KAA30170 for obm-l-list; Sun, 2 Jun 2002 10:01:18 -0300 Received: from www.zipmail.com.br (smtp.zipmail.com.br [200.187.242.10]) by sucuri.mat.puc-rio.br (8.9.3/8.9.3) with ESMTP id KAA30166 for ; Sun, 2 Jun 2002 10:01:16 -0300 From: ghaeser@zipmail.com.br Received: from [200.158.6.21] by www.zipmail.com.br with HTTP; Sun, 2 Jun 2002 09:49:56 -0300 Message-ID: <3CF94A7200001FA6@www.zipmail.com.br> Date: Sun, 2 Jun 2002 09:49:56 -0300 In-Reply-To: <3CF963D3.8040605@centroin.com.br> Subject: [obm-l] =?iso-8859-1?Q?Re=3A=20=5Bobm=2Dl=5D=20Polinomios?= To: obm-l@mat.puc-rio.br MIME-Version: 1.0 Content-Type: text/plain; charset="iso-8859-1" Content-Transfer-Encoding: 8bit X-MIME-Autoconverted: from quoted-printable to 8bit by sucuri.mat.puc-rio.br id KAA30167 Sender: owner-obm-l@sucuri.mat.puc-rio.br Precedence: bulk Reply-To: obm-l@mat.puc-rio.br >> >AQUI HA UM ERRO . ONDE ESTA P(j) DEVERIA ESTAR [(-1)^j ]* P(j) vc tem razao .. esqueci de escrever o (-1)^j. P(x)=sumk((x,k)*sumj(k,j)*[(-1)^j]*P(j)) onde sumk = somatório de k=0 até n e sumj = somatório de j=0 até k e mas como demonstro isso agora ? "Mathematicus nascitur, non fit" Matemáticos não são feitos, eles nascem --------------------------------------- Gabriel Haeser www.gabas.cjb.net ------------------------------------------ Use o melhor sistema de busca da Internet Radar UOL - http://www.radaruol.com.br ========================================================================= Instruções para entrar na lista, sair da lista e usar a lista em http://www.mat.puc-rio.br/~nicolau/olimp/obm-l.html O administrador desta lista é ========================================================================= From owner-obm-l@sucuri.mat.puc-rio.br Sun Jun 2 13:28:01 2002 Return-Path: Received: (from majordom@localhost) by sucuri.mat.puc-rio.br (8.9.3/8.9.3) id NAA31578 for obm-l-list; Sun, 2 Jun 2002 13:26:19 -0300 Received: from imo-r04.mx.aol.com (imo-r04.mx.aol.com [152.163.225.100]) by sucuri.mat.puc-rio.br (8.9.3/8.9.3) with ESMTP id NAA31574 for ; Sun, 2 Jun 2002 13:26:16 -0300 From: SSayajinGoten@aol.com Received: from SSayajinGoten@aol.com by imo-r04.mx.aol.com (mail_out_v32.5.) id z.155.ec0b8c3 (4007) for ; Sun, 2 Jun 2002 12:14:53 -0400 (EDT) Message-ID: <155.ec0b8c3.2a2b9e7d@aol.com> Date: Sun, 2 Jun 2002 12:14:53 EDT Subject: [obm-l] =?ISO-8859-1?Q?D=FAvida?= To: obm-l@mat.puc-rio.br MIME-Version: 1.0 Content-Type: text/plain; charset="ISO-8859-1" X-Mailer: AOL 4.0 for Windows sub 108 Content-Transfer-Encoding: 8bit X-MIME-Autoconverted: from quoted-printable to 8bit by sucuri.mat.puc-rio.br id NAA31575 Sender: owner-obm-l@sucuri.mat.puc-rio.br Precedence: bulk Reply-To: obm-l@mat.puc-rio.br Ae Galera, como sou novo,eu tava olhando as menssagens anteriores, e vi um problema interessante que ninguem conseguiu fazer.. queria saber se alguem ja sabe a resposta, ou a resolução, ou se alguem ja conseguiu faze-lo, o problema é o seguinte: * O capim cresce no pasto todo com igual rapidez e espessura. Sabe-se que 70 vacas o comeriam em 24 dias, e 30 em 60. Quantas vacas comeriam todo o capim em 96 dias ? ========================================================================= Instruções para entrar na lista, sair da lista e usar a lista em http://www.mat.puc-rio.br/~nicolau/olimp/obm-l.html O administrador desta lista é ========================================================================= From owner-obm-l@sucuri.mat.puc-rio.br Sun Jun 2 22:41:37 2002 Return-Path: Received: (from majordom@localhost) by sucuri.mat.puc-rio.br (8.9.3/8.9.3) id WAA02655 for obm-l-list; Sun, 2 Jun 2002 22:40:45 -0300 Received: from panther.unisys.com.br (panther.unisys.com.br [200.220.64.10]) by sucuri.mat.puc-rio.br (8.9.3/8.9.3) with ESMTP id WAA02651 for ; Sun, 2 Jun 2002 22:40:42 -0300 Received: from jf (riopm18p27.unisys.com.br [200.220.16.27] (may be forged)) by panther.unisys.com.br (8.12.2/8.12.2) with SMTP id g531TMWa009983 for ; Sun, 2 Jun 2002 22:29:23 -0300 (EST) X-Spam-Filter: check_local@panther.unisys.com.br by digitalanswers.org Message-ID: <005301c20a9d$94d7a960$1b10dcc8@jf> From: "Jose Francisco Guimaraes Costa" To: "obm-l" Subject: [obm-l] problemas de Hilbert Date: Sun, 2 Jun 2002 22:25:42 -0300 MIME-Version: 1.0 Content-Type: multipart/alternative; boundary="----=_NextPart_000_0050_01C20A84.6E4D0F60" X-Priority: 3 X-MSMail-Priority: Normal X-Mailer: Microsoft Outlook Express 5.00.2919.6600 X-MimeOLE: Produced By Microsoft MimeOLE V5.00.2919.6600 Sender: owner-obm-l@sucuri.mat.puc-rio.br Precedence: bulk Reply-To: obm-l@mat.puc-rio.br This is a multi-part message in MIME format. ------=_NextPart_000_0050_01C20A84.6E4D0F60 Content-Type: text/plain; charset="iso-8859-1" Content-Transfer-Encoding: quoted-printable Algu=E9m neste forum poderia relacionar os problemas que David Hilbert = descreveu como "sem solu=E7=E3o" no Congresso Internacional de = Matem=E1ticos de 1900, quais os que foram resolvidos desde ent=E3o, e = fornecer links relacionados sobre o assunto? JF ------=_NextPart_000_0050_01C20A84.6E4D0F60 Content-Type: text/html; charset="iso-8859-1" Content-Transfer-Encoding: quoted-printable
Algu=E9m neste forum poderia relacionar = os problemas=20 que David Hilbert descreveu como "sem solu=E7=E3o" no Congresso = Internacional de=20 Matem=E1ticos de 1900, quais os que foram resolvidos desde ent=E3o, e = fornecer links=20 relacionados sobre o assunto?
 
JF
------=_NextPart_000_0050_01C20A84.6E4D0F60-- ========================================================================= Instruções para entrar na lista, sair da lista e usar a lista em http://www.mat.puc-rio.br/~nicolau/olimp/obm-l.html O administrador desta lista é ========================================================================= From owner-obm-l@sucuri.mat.puc-rio.br Mon Jun 3 06:37:52 2002 Return-Path: Received: (from majordom@localhost) by sucuri.mat.puc-rio.br (8.9.3/8.9.3) id GAA05644 for obm-l-list; Mon, 3 Jun 2002 06:37:16 -0300 Received: from mail.gmx.net (mail.gmx.net [213.165.64.20]) by sucuri.mat.puc-rio.br (8.9.3/8.9.3) with SMTP id GAA05640 for ; Mon, 3 Jun 2002 06:37:13 -0300 Received: (qmail 25697 invoked by uid 0); 3 Jun 2002 09:25:54 -0000 Received: from 200-206-211-169.dsl.telesp.net.br (HELO mentecapto.gmx.de) (200.206.211.169) by mail.gmx.net (mp015-rz3) with SMTP; 3 Jun 2002 09:25:54 -0000 Message-Id: <5.1.0.14.2.20020603062140.00a7b1a0@pop.gmx.net> X-Sender: mentus@gmx.de@pop.gmx.net X-Mailer: QUALCOMM Windows Eudora Version 5.1 Date: Mon, 03 Jun 2002 06:24:47 -0300 To: obm-l@mat.puc-rio.br From: "Fernando Henrique Ferraz P. da Rosa" Subject: Re: [obm-l] problemas de Hilbert In-Reply-To: <005301c20a9d$94d7a960$1b10dcc8@jf> Mime-Version: 1.0 Content-Type: multipart/mixed; x-avg-checked=avg-ok-1EFC200A; boundary="=======593453B7=======" Sender: owner-obm-l@sucuri.mat.puc-rio.br Precedence: bulk Reply-To: obm-l@mat.puc-rio.br --=======593453B7======= Content-Type: text/plain; x-avg-checked=avg-ok-1EFC200A; charset=iso-8859-1; format=flowed Content-Transfer-Encoding: quoted-printable Aqui tem uma explica=E7=E3o geral dos problemas:=20 http://mathworld.wolfram.com/HilbertsProblems.html Em mais detalhes nesse site a apresenta=E7=E3o do pr=F3prio Hilbert:=20 http://aleph0.clarku.edu/~djoyce/hilbert/problems.html S=F3 uma observa=E7= =E3o..=20 ele n=E3o prop=F4s os problemas como 'sem solu=E7=E3o'. A bem verdade boa= parte dos=20 problemas j=E1 foi resolvidos.. A inten=E7=E3o foi fazer uma lista de= problemas=20 que 'ocupariam os matematicos' pelas proximas d=E9cadas. At 22:25 6/2/2002 -0300, you wrote: >Algu=E9m neste forum poderia relacionar os problemas que David Hilbert=20 >descreveu como "sem solu=E7=E3o" no Congresso Internacional de Matem=E1tico= s de=20 >1900, quais os que foram resolvidos desde ent=E3o, e fornecer links=20 >relacionados sobre o assunto? > >JF "Against stupidity, the Gods themselves contend in vain", Friedrich von Schiller's - []'s Fernando Henrique Ferraz Pereira da Rosa mentus@gmx.de --=======593453B7======= Content-Type: text/plain; charset=us-ascii; x-avg=cert; x-avg-checked=avg-ok-1EFC200A Content-Disposition: inline --- Outgoing mail is certified Virus Free. Checked by AVG anti-virus system (http://www.grisoft.com). Version: 6.0.365 / Virus Database: 202 - Release Date: 5/24/2002 --=======593453B7=======-- ========================================================================= Instruções para entrar na lista, sair da lista e usar a lista em http://www.mat.puc-rio.br/~nicolau/olimp/obm-l.html O administrador desta lista é ========================================================================= From owner-obm-l@sucuri.mat.puc-rio.br Mon Jun 3 13:02:34 2002 Return-Path: Received: (from majordom@localhost) by sucuri.mat.puc-rio.br (8.9.3/8.9.3) id NAA09929 for obm-l-list; Mon, 3 Jun 2002 13:00:40 -0300 Received: from smtp.ieg.com.br (194.125.226.200.in-addr.arpa.ig.com.br [200.226.125.194] (may be forged)) by sucuri.mat.puc-rio.br (8.9.3/8.9.3) with ESMTP id NAA09925 for ; Mon, 3 Jun 2002 13:00:25 -0300 Received: from ieg.com.br (hendrix.protocoloweb.com.br [200.185.63.19]) by smtp.ieg.com.br (IeG relay/8.9.3) with SMTP id g53FjD54070308; Mon, 3 Jun 2002 12:45:13 -0300 (BRT) From: "Arnaldo" To: "ReNNeR" , "[OBM]" Date: Mon, 3 Jun 2002 15:41:42 GMT Subject: Re: [obm-l] algu=?ISO-8859-1?B?6W0g?=explica isso? X-Mailer: DMailWeb Web to Mail Gateway 2.7v, http://netwinsite.com/top_mail.htm Message-id: <3cfb8e36.3f3c.0@ieg.com.br> X-User-Info: 146.164.38.49 MIME-Version: 1.0 Content-Type: text/plain; charset="iso-8859-1" Content-Transfer-Encoding: 8bit X-MIME-Autoconverted: from quoted-printable to 8bit by sucuri.mat.puc-rio.br id NAA09926 Sender: owner-obm-l@sucuri.mat.puc-rio.br Precedence: bulk Reply-To: obm-l@mat.puc-rio.br > >1 + 1 = x >(1 + 1)² = x² >1 + 2 + 1 = x² >(1 + 1) + 2 = x² >x + 2 = x² >x² - x - 2 = 0 > >delta = b² - 4ac >delta = (-1)²-4.1.(-2) >delta = 1+8 >delta = 9 >x = [1 +?9]:2 ou x = [1-?9]:2 >x = [1 + 3]:2 ou x = [1-3]:2 >x = 2 ou x = -1. >1+1 = 2 ou; >1+1 = -1 ?? > >Quando vc escreve que 1+1=x tem-se uma equacao do segundo grau que deve ter (neste caso) solucao unica. Quando se eleva ao quadrado vc transforma tal equacao numa quadradica que, a principio, tem duas solucoes. A unica coisa que vc poder afirmar é que o conjunto solucao da equacao do primeiro grau é um subconjunto do conjunto solucao da equacao do segundo grau. Para saber qual a solucao correta vc, obviamente dever fazer a verificacao (na equacao do primeiro grau) das solucoes encontradas. Espero que tenha ajudado, Abraços, Arnaldo. > >========================================================================= >Instruções para entrar na lista, sair da lista e usar a lista em >http://www.mat.puc-rio.br/~nicolau/olimp/obm-l.html >O administrador desta lista é >========================================================================= > http://www.ieg.com.br ========================================================================= Instruções para entrar na lista, sair da lista e usar a lista em http://www.mat.puc-rio.br/~nicolau/olimp/obm-l.html O administrador desta lista é ========================================================================= From owner-obm-l@sucuri.mat.puc-rio.br Mon Jun 3 14:38:37 2002 Return-Path: Received: (from majordom@localhost) by sucuri.mat.puc-rio.br (8.9.3/8.9.3) id OAA11676 for obm-l-list; Mon, 3 Jun 2002 14:36:45 -0300 Received: from fgvrj23.fgv.br (fgvrj23.fgv.br [200.20.164.23]) by sucuri.mat.puc-rio.br (8.9.3/8.9.3) with ESMTP id OAA11670 for ; Mon, 3 Jun 2002 14:36:32 -0300 Received: by FGVRJ23 with Internet Mail Service (5.5.2653.19) id ; Mon, 3 Jun 2002 14:30:35 -0300 Message-ID: <765A72978645D4118B1C0000E229806D05B6B3B6@FGVRJ23> From: Ralph Teixeira To: "Obm (E-mail)" Subject: [obm-l] =?iso-8859-1?Q?Varia=E7=E3o_na_quest=E3o_do_IME=3A_x=3Dsqrt=28?= =?iso-8859-1?Q?a-sqrt=28a-x=29=29?= Date: Mon, 3 Jun 2002 14:30:29 -0300 MIME-Version: 1.0 X-Mailer: Internet Mail Service (5.5.2653.19) Content-Type: text/plain; charset="iso-8859-1" Content-Transfer-Encoding: 8bit X-MIME-Autoconverted: from quoted-printable to 8bit by sucuri.mat.puc-rio.br id OAA11673 Sender: owner-obm-l@sucuri.mat.puc-rio.br Precedence: bulk Reply-To: obm-l@mat.puc-rio.br Olá, galera. Um colega nosso da lista, o Cláudio, destacou que eu havia me enganado quando disse que a equação x=sqrt(5-sqrt(5-x)) tem *DUAS* soluções. Ele tem razão -- apesar de eu ainda defender o fato de que você NÃO PODE SIMPLESMENTE DIZER QUE x=sqrt(5-x), o meu método acaba por gerar apenas uma raiz de qualquer forma (eu havia cometido um erro de álgebra que o Cláudio encontrou). Por exemplo, a equação x=sqrt(0.8-sqrt(0.8-x)) tem 3 raízes reais (você consegue encontrá-las?). Apenas *1* delas é encontrada fazendo x=sqrt(0.8-x). Minha perguntinha para a galera é então: para que valores de a as equações x=sqrt(a-sqrt(a-x)) e x=sqrt(a-x) são equivalentes? Divirtam-se! Abraço, Ralph ========================================================================= Instruções para entrar na lista, sair da lista e usar a lista em http://www.mat.puc-rio.br/~nicolau/olimp/obm-l.html O administrador desta lista é ========================================================================= From owner-obm-l@sucuri.mat.puc-rio.br Mon Jun 3 16:50:11 2002 Return-Path: Received: (from majordom@localhost) by sucuri.mat.puc-rio.br (8.9.3/8.9.3) id QAA14018 for obm-l-list; Mon, 3 Jun 2002 16:49:04 -0300 Received: from web20408.mail.yahoo.com (web20408.mail.yahoo.com [216.136.226.127]) by sucuri.mat.puc-rio.br (8.9.3/8.9.3) with SMTP id QAA14014 for ; Mon, 3 Jun 2002 16:49:01 -0300 Message-ID: <20020603193744.84670.qmail@web20408.mail.yahoo.com> Received: from [200.182.207.235] by web20408.mail.yahoo.com via HTTP; Mon, 03 Jun 2002 16:37:44 ART Date: Mon, 3 Jun 2002 16:37:44 -0300 (ART) From: =?iso-8859-1?q?Helder=20Suzuki?= Subject: Re: [obm-l] Dúvida To: obm-l@mat.puc-rio.br In-Reply-To: <155.ec0b8c3.2a2b9e7d@aol.com> MIME-Version: 1.0 Content-Type: text/plain; charset=iso-8859-1 Content-Transfer-Encoding: 8bit Sender: owner-obm-l@sucuri.mat.puc-rio.br Precedence: bulk Reply-To: obm-l@mat.puc-rio.br > * O capim cresce no pasto todo com igual rapidez e espessura. Sabe-se > que 70 > vacas o comeriam em 24 dias, e 30 em 60. Quantas vacas comeriam todo o > capim > em 96 dias ? Vamos ver se eu consigo isto. pelo enunciado: Vacas*Dias = Capiminicial + Capim*Dias então temos o sistema: { 70*24 = Capim.incial + Capim*24 = 1680 30*60 = Capim.incial + Capim*60 = 1800 } resolvendo o sistema: Capim*36=120 Capim = 10/3 (velocidade de crescimento do capim) Capim.inicial = 1680 - 24*10/3 = 1680 - 80 = 1600 agora: Vacas*96 = 1600 + 10*96/3 = 1600 + 320 = 1920 Vacas = 20 20 Vacas comem o capim em 96 dias :) []'s, Hélder Suzuki _______________________________________________________________________ Yahoo! Encontros O lugar certo para você encontrar aquela pessoa que falta na sua vida. Cadastre-se hoje mesmo! http://br.encontros.yahoo.com/ ========================================================================= Instruções para entrar na lista, sair da lista e usar a lista em http://www.mat.puc-rio.br/~nicolau/olimp/obm-l.html O administrador desta lista é ========================================================================= From owner-obm-l@sucuri.mat.puc-rio.br Mon Jun 3 16:50:34 2002 Return-Path: Received: (from majordom@localhost) by sucuri.mat.puc-rio.br (8.9.3/8.9.3) id QAA14064 for obm-l-list; Mon, 3 Jun 2002 16:50:31 -0300 Received: from shannon.bol.com.br (shannon.bol.com.br [200.221.24.13]) by sucuri.mat.puc-rio.br (8.9.3/8.9.3) with ESMTP id QAA14040 for ; Mon, 3 Jun 2002 16:49:55 -0300 Received: from bol.com.br (200.221.24.80) by shannon.bol.com.br (5.1.071) id 3CE1A947004A071E for obm-l@mat.puc-rio.br; Mon, 3 Jun 2002 16:37:35 -0300 Date: Mon, 3 Jun 2002 16:37:34 -0300 Message-Id: Subject: Re: [obm-l] Regra d 3 (cinistra) MIME-Version: 1.0 Content-Type: text/plain;charset="iso-8859-1" From: "rafaelc.l" To: obm-l@mat.puc-rio.br X-XaM3-API-Version: 2.4.3.4.4 X-SenderIP: 200.176.166.108 Content-Transfer-Encoding: 8bit X-MIME-Autoconverted: from quoted-printable to 8bit by sucuri.mat.puc-rio.br id QAA14061 Sender: owner-obm-l@sucuri.mat.puc-rio.br Precedence: bulk Reply-To: obm-l@mat.puc-rio.br Use a soma e o produto das raízes da equação quadrática e faça um sistema. > Hum..a sim, valews rapa. > > Olha eu tenho outra questão que eu tambem não consegui resolver, só que eu > não sei como fazer sinal de raiz no pc, então eu vou escrever, e ai voces > copiem a questão, fassam, e depois, se possivel, me retornem , Valews! > > Conseidere a equação do 2º grau em x tal que ax²+bx+c=0 (zero),onde, a,b,c são > números reais e que "a" diferente de 0(zero), sabendo que 1 e 2 são raízes e > que > a(0,5 - raiz quadrada de 3)² + b(0,5 - raiz quadrada de 3) + c =1997. > O valor de > a(2,5 + raiz quadrada de 3)² + b(2,5 + raiz quadrada de 3)+ c, é ? > ========================================================== =============== > Instruções para entrar na lista, sair da lista e usar a lista em > http://www.mat.puc-rio.br/~nicolau/olimp/obm-l.html > O administrador desta lista é > ========================================================== =============== > __________________________________________________________________________ Quer ter seu próprio endereço na Internet? Garanta já o seu e ainda ganhe cinco e-mails personalizados. DomíniosBOL - http://dominios.bol.com.br ========================================================================= Instruções para entrar na lista, sair da lista e usar a lista em http://www.mat.puc-rio.br/~nicolau/olimp/obm-l.html O administrador desta lista é ========================================================================= From owner-obm-l@sucuri.mat.puc-rio.br Mon Jun 3 18:58:23 2002 Return-Path: Received: (from majordom@localhost) by sucuri.mat.puc-rio.br (8.9.3/8.9.3) id SAA16517 for obm-l-list; Mon, 3 Jun 2002 18:56:25 -0300 Received: from web13303.mail.yahoo.com (web13303.mail.yahoo.com [216.136.175.39]) by sucuri.mat.puc-rio.br (8.9.3/8.9.3) with SMTP id SAA16512 for ; Mon, 3 Jun 2002 18:56:21 -0300 Message-ID: <20020603214505.82855.qmail@web13303.mail.yahoo.com> Received: from [200.207.158.20] by web13303.mail.yahoo.com via HTTP; Mon, 03 Jun 2002 14:45:05 PDT Date: Mon, 3 Jun 2002 14:45:05 -0700 (PDT) From: Carlos Yuzo Shine Subject: [obm-l] A OPM abriu inscrições To: obm-l@mat.puc-rio.br In-Reply-To: <20020528232652.50585.qmail@web13301.mail.yahoo.com> MIME-Version: 1.0 Content-Type: text/plain; charset=us-ascii Sender: owner-obm-l@sucuri.mat.puc-rio.br Precedence: bulk Reply-To: obm-l@mat.puc-rio.br Olá a todos! O site da OPM está com novidades! Avise o professor de sua escola que as inscrições para a OPM (Olimpíada Paulista de Matemática) estão abertas. Para se inscrever, o seu professor deve acessar o site da OPM: http://www.opm.mat.br Mas atenção: quem deve se inscrever é a *sua escola*, através do seu professor. As inscrições devem ser feitas até 31/07/2002. Caso haja dúvidas, envie um e-mail para info@opm.mat.br. Por favor não mande e-mails para a lista da OBM pedindo informações sobre como participar da OPM. []'s Shine __________________________________________________ Do You Yahoo!? Yahoo! - Official partner of 2002 FIFA World Cup http://fifaworldcup.yahoo.com ========================================================================= Instruções para entrar na lista, sair da lista e usar a lista em http://www.mat.puc-rio.br/~nicolau/olimp/obm-l.html O administrador desta lista é ========================================================================= From owner-obm-l@sucuri.mat.puc-rio.br Mon Jun 3 23:11:07 2002 Return-Path: Received: (from majordom@localhost) by sucuri.mat.puc-rio.br (8.9.3/8.9.3) id XAA19433 for obm-l-list; Mon, 3 Jun 2002 23:10:21 -0300 Received: from wool.vetor.com.br (200.160.244.7.metrored.net.br [200.160.244.7] (may be forged)) by sucuri.mat.puc-rio.br (8.9.3/8.9.3) with ESMTP id XAA19429 for ; Mon, 3 Jun 2002 23:10:19 -0300 Received: (from root@localhost) by wool.vetor.com.br (8.11.4/8.11.4) id g541x4l17963 for obm-l@mat.puc-rio.br; Mon, 3 Jun 2002 22:59:04 -0300 Received: from rodrigo ([200.160.244.107]) by wool.vetor.com.br (8.11.4/8.11.4) with SMTP id g541x2O17947 for ; Mon, 3 Jun 2002 22:59:02 -0300 Message-ID: <001401c20b6b$4df1e5a0$6bf4a0c8@rodrigo> From: "Rodrigo Villard Milet" To: Subject: [obm-l] =?iso-8859-1?Q?Re:_=5Bobm-l=5D_Varia=E7=E3o_na_quest=E3o_do_IME:_x=3Dsq?= =?iso-8859-1?Q?rt=28a-sqrt=28a-x=29=29?= Date: Mon, 3 Jun 2002 22:58:14 -0300 MIME-Version: 1.0 Content-Type: text/plain; charset="iso-8859-1" Content-Transfer-Encoding: 8bit X-Priority: 3 X-MSMail-Priority: Normal X-Mailer: Microsoft Outlook Express 4.72.3110.5 X-MimeOLE: Produced By Microsoft MimeOLE V4.72.3110.3 X-Virus-Scanned: by AMaViS perl-11 Sender: owner-obm-l@sucuri.mat.puc-rio.br Precedence: bulk Reply-To: obm-l@mat.puc-rio.br Fazendo y = sqrt(a-x), temos que x = sqrt(a-y). Daí, segue o sistema : y² = a - x (i) x² = a - y (ii) Subtraindo... (y-x)(y+x) = y-x. Então, ou x=y, ou x+y=1. A pergunta é : quando que x+y=1 é impossível ? x+y=1 ... y = 1-x ... 1-x = sqrt(a-x) ... 1 - 2x + x² = a - x ... x² - x + (1-a) = 0. Essa equação não possui raízes reais se e só se 1 - 4(1-a) < 0, ou seja, a < 3/4. E é fácil ver que se a<0, o problema não tem solução. Logo 0 < a < 3/4. Bem... para achar mais "a" que são solução para o seu problema, eu deveria aceitar o caso em que a equação x² - x + (1-a) = 0 admite apenas soluções negativas, pois é claro que x >= 0 ( Além disso, x <= a ). Seja f(x) = x² - x + (1-a). A soma das raízes é positiva, logo só podemos ter uma negativa. Então o caso que nos interessa é qd uma raiz é <0 e outra >a. Assim, temos que f(0) > 0 e f(a) > 0. f(0) > 0 ... 1-a > 0... a < 1 f(a) > 0 ... a² - 2a + 1... Então não temos problemas com isso... parece que a solução é de fato 0 < a < 3/4. Aguardo comentários. Abraços, Villard -----Mensagem original----- De: Ralph Teixeira Para: Obm (E-mail) Data: Segunda-feira, 3 de Junho de 2002 15:09 Assunto: [obm-l] Variação na questão do IME: x=sqrt(a-sqrt(a-x)) > Olá, galera. > > Um colega nosso da lista, o Cláudio, destacou que eu havia me >enganado quando disse que a equação > > x=sqrt(5-sqrt(5-x)) > > tem *DUAS* soluções. Ele tem razão -- apesar de eu ainda defender o >fato de que você NÃO PODE SIMPLESMENTE DIZER QUE x=sqrt(5-x), o meu método >acaba por gerar apenas uma raiz de qualquer forma (eu havia cometido um erro >de álgebra que o Cláudio encontrou). > > Por exemplo, a equação x=sqrt(0.8-sqrt(0.8-x)) tem 3 raízes reais >(você consegue encontrá-las?). Apenas *1* delas é encontrada fazendo >x=sqrt(0.8-x). > > Minha perguntinha para a galera é então: para que valores de a as >equações >x=sqrt(a-sqrt(a-x)) e x=sqrt(a-x) são equivalentes? > > Divirtam-se! > > Abraço, > Ralph >========================================================================= >Instruções para entrar na lista, sair da lista e usar a lista em >http://www.mat.puc-rio.br/~nicolau/olimp/obm-l.html >O administrador desta lista é >========================================================================= > ========================================================================= Instruções para entrar na lista, sair da lista e usar a lista em http://www.mat.puc-rio.br/~nicolau/olimp/obm-l.html O administrador desta lista é ========================================================================= From owner-obm-l@sucuri.mat.puc-rio.br Tue Jun 4 10:03:25 2002 Return-Path: Received: (from majordom@localhost) by sucuri.mat.puc-rio.br (8.9.3/8.9.3) id KAA23950 for obm-l-list; Tue, 4 Jun 2002 10:00:52 -0300 Received: from Euler.impa.br (euler.impa.br [147.65.1.3]) by sucuri.mat.puc-rio.br (8.9.3/8.9.3) with ESMTP id KAA23946 for ; Tue, 4 Jun 2002 10:00:50 -0300 Received: from obm-01 (obm-01.impa.br [147.65.2.170]) by Euler.impa.br (8.11.6/8.11.6) with SMTP id g54Cna804929 for ; Tue, 4 Jun 2002 09:49:36 -0300 (EST) Message-Id: <3.0.5.32.20020604095057.0080d180@pop.impa.br> X-Sender: obm@pop.impa.br X-Mailer: QUALCOMM Windows Eudora Light Version 3.0.5 (32) Date: Tue, 04 Jun 2002 09:50:57 -0300 To: obm-l@mat.puc-rio.br From: Olimpiada Brasileira de Matematica Subject: [obm-l] Equipe Brasileira XIII Cone Sul Mime-Version: 1.0 Content-Type: text/plain; charset="us-ascii" Sender: owner-obm-l@sucuri.mat.puc-rio.br Precedence: bulk Reply-To: obm-l@mat.puc-rio.br Caros amigos(as) da lista: A equipe que representara' o Brasil na XIII Olimpiada do Cone Sul e' a seguinte: Lider: Prof. Yoshiharu Kohayakawa (Sao Paulo - SP) Vice-Lider: Prof. Luciano Guimaraes Castro (Rio de Janeiro - RJ) Alunos (em ordem alfabetica) BRA1: Alex Correa Abreu (Niteroi - RJ) BRA2: Israel Dourado Carrah (Fortaleza - CE) BRA3: Larissa Cavalcante Q. de Lima (Fortaleza - CE) BRA4: Rafael Daigo Hirama (Campinas - SP) Abracos, Nelly. ========================================================================= Instruções para entrar na lista, sair da lista e usar a lista em http://www.mat.puc-rio.br/~nicolau/olimp/obm-l.html O administrador desta lista é ========================================================================= From owner-obm-l@sucuri.mat.puc-rio.br Tue Jun 4 11:08:49 2002 Return-Path: Received: (from majordom@localhost) by sucuri.mat.puc-rio.br (8.9.3/8.9.3) id LAA25234 for obm-l-list; Tue, 4 Jun 2002 11:07:48 -0300 Received: from matinhos.terra.com.br (matinhos.terra.com.br [200.176.3.21]) by sucuri.mat.puc-rio.br (8.9.3/8.9.3) with ESMTP id LAA25230 for ; Tue, 4 Jun 2002 11:07:44 -0300 Received: from engenho.terra.com.br (engenho.terra.com.br [200.176.3.42]) by matinhos.terra.com.br (Postfix) with ESMTP id 28CE247215 for ; Tue, 4 Jun 2002 10:56:27 -0300 (EST) Received: from terra.com.br (200-206-243-51.dsl.telesp.net.br [200.206.243.51]) (authenticated user lponce) by engenho.terra.com.br (Postfix) with ESMTP id 1E2036809F for ; Tue, 4 Jun 2002 10:56:27 -0300 (EST) Message-ID: <3CFCC6F0.6CA5DC9B@terra.com.br> Date: Tue, 04 Jun 2002 10:56:01 -0300 From: Luiz Antonio Ponce Alonso X-Mailer: Mozilla 4.79 [en] (Windows NT 5.0; U) X-Accept-Language: en MIME-Version: 1.0 To: obm-l@mat.puc-rio.br Subject: Re: [obm-l] Regra d 3 (cinistra) References: <171.e6d83f3.2a2a9622@aol.com> Content-Type: multipart/alternative; boundary="------------120DA7C7641C18C35F72F543" Sender: owner-obm-l@sucuri.mat.puc-rio.br Precedence: bulk Reply-To: obm-l@mat.puc-rio.br --------------120DA7C7641C18C35F72F543 Content-Type: text/plain; charset=iso-8859-1 Content-Transfer-Encoding: 8bit Olá amigos, A resposta para o problema abaixo é 1997. Para isso note que: 1) A abscissa do vertice da parabola obtida pela função f do segundo grau, definida por f(x) =ax²+bx+c é 1,5 e corresponde a média aritmética das raízes ( 1 e 2 ) da equação: ax²+bx+c=0. 2) Os números reais : 1,5 + ( 1+ raiz quadrada de 3) , ou seja 2,5 + raiz quadrada de 3 e 1,5 - ( 1+ raiz quadrada de 3) , ou seja 0,5 - raiz quadrada de 3 são simetricos em relação a abscissa do vértice da parabola obtida por f. 3) De 1 e do enunciado, tem-se também f(0,5 - raiz quadrada de 3)=a(0,5 - raiz quadrada de 3)² + b(0,5 - raiz quadrada de 3) + c =1997. Nestas condições, de (1) , (2), (3) e da propriedade de simetria do gráfico da parabola em relação a reta que passa pelo seu vertice e cuja equação é x = 1,5, tem-se que f(2,5 + raiz quadrada de 3)=f(0,5 - raiz quadrada de 3)=1997. Portanto, a(2,5 + raiz quadrada de 3)² + b(2,5 + raiz quadrada de 3) + c = 1997 Um abraço a todos PONCE SSayajinGoten@aol.com wrote: > Hum..a sim, valews rapa. > > Olha eu tenho outra questão que eu tambem não consegui resolver, só que eu > não sei como fazer sinal de raiz no pc, então eu vou escrever, e ai voces > copiem a questão, fassam, e depois, se possivel, me retornem , Valews! > > Conseidere a equação do 2º grau em x tal que ax²+bx+c=0(zero),onde, a,b,c são > números reais e que "a" diferente de 0(zero), sabendo que 1 e 2 são raízes e > que > a(0,5 - raiz quadrada de 3)² + b(0,5 - raiz quadrada de 3) + c =1997. > O valor de > a(2,5 + raiz quadrada de 3)² + b(2,5 + raiz quadrada de 3)+ c, é ? > ========================================================================= > Instruções para entrar na lista, sair da lista e usar a lista em > http://www.mat.puc-rio.br/~nicolau/olimp/obm-l.html > O administrador desta lista é > ========================================================================= --------------120DA7C7641C18C35F72F543 Content-Type: text/html; charset=us-ascii Content-Transfer-Encoding: 7bit Olá amigos,
A resposta para o problema abaixo é 1997.
Para isso note que:
1) A abscissa do vertice da parabola obtida pela função f do segundo grau,
definida por f(x) =ax²+bx+c é  1,5  e corresponde a  média aritmética das raízes ( 1 e 2 ) da equação:
ax²+bx+c=0.
2) Os números reais :
 1,5 + ( 1+  raiz quadrada de 3) ,        ou       seja    2,5 +  raiz quadrada de 3
e
1,5 - ( 1+  raiz quadrada de 3) ,          ou       seja    0,5 -  raiz quadrada de 3

são simetricos em relação a abscissa do vértice da parabola obtida por f.
3) De 1 e do enunciado, tem-se também
f(0,5 -  raiz quadrada de 3)=a(0,5 - raiz quadrada de 3)² + b(0,5 - raiz quadrada de 3) + c   =1997.

Nestas condições, de (1) , (2), (3) e da propriedade de simetria do gráfico da parabola em relação
a reta que passa pelo seu vertice e cuja equação é x = 1,5, tem-se que

f(2,5 +  raiz quadrada de 3)=f(0,5 -  raiz quadrada de 3)=1997.

Portanto,
a(2,5 +  raiz quadrada de 3)² + b(2,5 +  raiz quadrada de 3) + c = 1997
 

Um abraço a todos
PONCE
 
 

SSayajinGoten@aol.com wrote:

Hum..a sim, valews rapa.

Olha eu tenho outra questão que eu tambem não consegui resolver, só que eu
não sei como fazer sinal de raiz no pc, então eu vou escrever, e ai voces
copiem a questão, fassam, e depois, se possivel, me retornem , Valews!

Conseidere a equação do 2º grau em x tal que ax²+bx+c=0(zero),onde, a,b,c são
números reais e que "a" diferente de 0(zero), sabendo que 1 e 2 são raízes e
que
a(0,5 - raiz quadrada de 3)² + b(0,5 - raiz quadrada de 3) + c =1997.
O valor de
a(2,5 + raiz quadrada de 3)² + b(2,5 + raiz quadrada de 3)+ c, é ?
=========================================================================
Instruções para entrar na lista, sair da lista e usar a lista em
http://www.mat.puc-rio.br/~nicolau/olimp/obm-l.html
O administrador desta lista é <nicolau@mat.puc-rio.br>
=========================================================================

--------------120DA7C7641C18C35F72F543-- ========================================================================= Instruções para entrar na lista, sair da lista e usar a lista em http://www.mat.puc-rio.br/~nicolau/olimp/obm-l.html O administrador desta lista é ========================================================================= From owner-obm-l@sucuri.mat.puc-rio.br Tue Jun 4 15:38:45 2002 Return-Path: Received: (from majordom@localhost) by sucuri.mat.puc-rio.br (8.9.3/8.9.3) id PAA29378 for obm-l-list; Tue, 4 Jun 2002 15:38:25 -0300 Received: from www.zipmail.com.br (smtp.zipmail.com.br [200.187.242.10]) by sucuri.mat.puc-rio.br (8.9.3/8.9.3) with ESMTP id PAA29365; Tue, 4 Jun 2002 15:38:19 -0300 From: PETERDIRICHLET@zipmail.com.br Received: from [200.206.103.3] by www.zipmail.com.br with HTTP; Tue, 4 Jun 2002 15:26:20 -0300 Message-ID: <3CFCFBAE000002F0@www.zipmail.com.br> Date: Tue, 4 Jun 2002 16:26:20 -0200 Subject: [obm-l] =?iso-8859-1?Q?Cl=E1ssicos=20geometricos=2Cguerras=20ideol=F3gico=2Dmatematicas=20e=20?= =?iso-8859-1?Q?assuntos=20afins?= To: =?iso-8859-1?Q?Nicolau?= , =?iso-8859-1?Q?Olimpiada=20Brasileira=20de=20Matem=2E?= , =?iso-8859-1?Q?Paulo=20Rodrigues?= , =?iso-8859-1?Q?Lista=20de=20Discussao?= , =?iso-8859-1?Q?Carlos=20Shine?= MIME-Version: 1.0 Content-Type: text/plain; charset="iso-8859-1" Content-Transfer-Encoding: 8bit X-MIME-Autoconverted: from quoted-printable to 8bit by sucuri.mat.puc-rio.br id PAA29367 Sender: owner-obm-l@sucuri.mat.puc-rio.br Precedence: bulk Reply-To: obm-l@mat.puc-rio.br O MINISTERIO DA SAUDE ADVERTE:LER E-MAILS LONGOS PODE PROVOCAR SONOLENCIA FORTE E PARANOIAS CONSTANTES. Gente,aqui esta uma leva de geometricos.Eles tem uma pequena historia,que escrevi ao final por motivos obvios. 1.(Casey)Considere quatro circunferencias S1,S2,S3,S4(podendo algumas delas ter raio zero) coplanares. Prove que elas sao coinscritiveis(ha uma circunferencia tangente externamente a todas elas)se e so se t12*t34+t14*t23=t13*t24,em que txy e o valor da tangente externa as circunferencias Sx e Sy 2.No triangulo ABC,CA=AB,CAB=20°,CBD=60° e ECB=50°,sendo EC e DB duas cevianas.Calcule BDE. 3.Considere o triangulo retangulo de hipotenusa 4 e cateto 2.Inscreve-se nele um triangulo equilatero tal que um de seus vertices bissecte o cateto de lado 2.Ache o valor do lado desse triangulo inscrito. Quem resolver alguma dessas questoes eu agradeço.E enfim a HISTORIA da guerra Tudo isto começou quando eu estava arrumando meus arquivos de problemas olimpicos resolvidos(por mim mesmo,pelos professores Shine e Tengan nas aulas no "País da OBM"(o nickname que eu dei para a sede regional da OBM em Sampa),pela turma da lista,por uns colegas meus,tirados de sites,etc. e tal).Acabei pegando uma folha com a soluçao do problema 5 da IMO 2001.Eu resolvi-o por Trigonometria(como todo paulista que se preze),como fez o Thiago,e adaptei a soluçao oficial(totalmente Geometria Cearense:pontos magicos,semelhança,coisa e tal).Enfim,propus este problema a um colega de sala,e ele ficou a aula inteira tentando mas nao resolveu.Entao eu mostrei-lhe a minha soluçao.Mal tinha acabado,ele soltou os cachorros em cima de mim,falando que eu tinha apelado,que minha soluçao nao era elegante,e por ai vai.Entao eu resolvi mostrar a ele a soluçao oficial.Ele achou artificial mas elegante(bonitinha mas ordinaria...),e depois ele me disse que tinha asco a Trigonometria.Entao eu lhe propus outros 2 problemas classicos.Ele esta tentando resolve-los,me dara noticias amanha. Enquanto isso,eu resolvi coloca-los nesta lista par que voces os discutissem e me dessem algumas respostas(de todos os tipos,podendo usar desde geometria cearense ate geometria analitica) e me dissessem como posso fazer este meu obstinado amigo mudar de ideia ou pelo menos nao odiar tanto a geometria paulista.E essa e a historia! Assinado:Peterdirichlet TRANSIRE SVVM PECTVS MVNDOQUE POTIRE CONGREGATI EX TOTO ORBE MATHEMATICI OB SCRIPTA INSIGNIA TRIBVERE Medalha Fields(John Charles Fields) ------------------------------------------ Use o melhor sistema de busca da Internet Radar UOL - http://www.radaruol.com.br ========================================================================= Instruções para entrar na lista, sair da lista e usar a lista em http://www.mat.puc-rio.br/~nicolau/olimp/obm-l.html O administrador desta lista é ========================================================================= From owner-obm-l@sucuri.mat.puc-rio.br Tue Jun 4 18:30:10 2002 Return-Path: Received: (from majordom@localhost) by sucuri.mat.puc-rio.br (8.9.3/8.9.3) id SAA32125 for obm-l-list; Tue, 4 Jun 2002 18:29:53 -0300 Received: from sporus.bol.com.br (sporus.bol.com.br [200.221.24.23]) by sucuri.mat.puc-rio.br (8.9.3/8.9.3) with ESMTP id SAA32121 for ; Tue, 4 Jun 2002 18:29:51 -0300 Received: from bol.com.br (200.221.24.84) by sporus.bol.com.br (5.1.071) id 3CDB0F8A006FBCD5 for obm-l@mat.puc-rio.br; Tue, 4 Jun 2002 18:18:14 -0300 Date: Tue, 4 Jun 2002 18:18:14 -0300 Message-Id: Subject: [obm-l] MIME-Version: 1.0 Content-Type: text/plain;charset="iso-8859-1" From: "aleixocarvalho" To: obm-l@mat.puc-rio.br X-XaM3-API-Version: 2.4.3.4.4 X-SenderIP: 143.106.22.138 Content-Transfer-Encoding: 8bit X-MIME-Autoconverted: from quoted-printable to 8bit by sucuri.mat.puc-rio.br id SAA32122 Sender: owner-obm-l@sucuri.mat.puc-rio.br Precedence: bulk Reply-To: obm-l@mat.puc-rio.br por favor alguem me demontre que: " a solucao x^2-2*y^2=1 tem infinitas solucoes dadas por x_k+y_k*2^(1/2)=[3+2*2^(1/2)]^k com k=1,2,3,..." __________________________________________________________________________ Quer ter seu próprio endereço na Internet? Garanta já o seu e ainda ganhe cinco e-mails personalizados. DomíniosBOL - http://dominios.bol.com.br ========================================================================= Instruções para entrar na lista, sair da lista e usar a lista em http://www.mat.puc-rio.br/~nicolau/olimp/obm-l.html O administrador desta lista é ========================================================================= From owner-obm-l@sucuri.mat.puc-rio.br Tue Jun 4 19:29:47 2002 Return-Path: Received: (from majordom@localhost) by sucuri.mat.puc-rio.br (8.9.3/8.9.3) id TAA00742 for obm-l-list; Tue, 4 Jun 2002 19:29:39 -0300 Received: from web10103.mail.yahoo.com (web10103.mail.yahoo.com [216.136.130.53]) by sucuri.mat.puc-rio.br (8.9.3/8.9.3) with SMTP id TAA00738 for ; Tue, 4 Jun 2002 19:29:36 -0300 Message-ID: <20020604221824.55071.qmail@web10103.mail.yahoo.com> Received: from [150.161.4.45] by web10103.mail.yahoo.com via HTTP; Tue, 04 Jun 2002 15:18:24 PDT Date: Tue, 4 Jun 2002 15:18:24 -0700 (PDT) From: Rafael WC Subject: [obm-l] bissetriz interna To: OBM MIME-Version: 1.0 Content-Type: text/plain; charset=us-ascii Sender: owner-obm-l@sucuri.mat.puc-rio.br Precedence: bulk Reply-To: obm-l@mat.puc-rio.br Olá Pessoal! Esta questão não parece difícil, mas não consegui muita coisa além de potência de ponto e o teorema da bissetriz interna. Em um triangulo ABC, a bissetriz interna de A encontra BC em D e o círculo circunscrito em E. Se AB = 8, AC = 6 e DE = 3 calcule o comprimento da bissetriz AD. Se alguém conseguir, agradeço qualquer dica. Obrigado, Rafael. ===== Rafael Werneck Cinoto ICQ# 107011599 rwcinoto@yahoo.com rafael.caixa@gov.com.br matduvidas@yahoo.com.br http://www.rwcinoto.hpg.com.br/ __________________________________________________ Do You Yahoo!? Yahoo! - Official partner of 2002 FIFA World Cup http://fifaworldcup.yahoo.com ========================================================================= Instruções para entrar na lista, sair da lista e usar a lista em http://www.mat.puc-rio.br/~nicolau/olimp/obm-l.html O administrador desta lista é ========================================================================= From owner-obm-l@sucuri.mat.puc-rio.br Tue Jun 4 20:13:26 2002 Return-Path: Received: (from majordom@localhost) by sucuri.mat.puc-rio.br (8.9.3/8.9.3) id UAA01451 for obm-l-list; Tue, 4 Jun 2002 20:12:35 -0300 Received: from web10108.mail.yahoo.com (web10108.mail.yahoo.com [216.136.130.58]) by sucuri.mat.puc-rio.br (8.9.3/8.9.3) with SMTP id UAA01447 for ; Tue, 4 Jun 2002 20:12:32 -0300 Message-ID: <20020604230119.74871.qmail@web10108.mail.yahoo.com> Received: from [150.161.4.45] by web10108.mail.yahoo.com via HTTP; Tue, 04 Jun 2002 16:01:19 PDT Date: Tue, 4 Jun 2002 16:01:19 -0700 (PDT) From: Rafael WC Subject: [obm-l] área do qudrilátero To: obm-l@mat.puc-rio.br In-Reply-To: <3CEE6B3F000005AA@www.zipmail.com.br> MIME-Version: 1.0 Content-Type: text/plain; charset=us-ascii Sender: owner-obm-l@sucuri.mat.puc-rio.br Precedence: bulk Reply-To: obm-l@mat.puc-rio.br Olá Pessoal! Esta aqui também não estou vendo o caminho. Aluma dica? Tem-se um quadrado ABCD de área S. Une-se os vértices A e B ao ponto médio M do lado CD, e une-se os vértices B e C ao ponto médio N do lado AD. O segmento AM intercepta os segmentos BN e CN nos pontos E e F, respectivamente, e o segmento BM intercepta o segmento CN no ponto G. Calcule a área do quadrilátero BEFG em função de S. Resposta: 4S/15 Se aluém puder ajudar... Obrigado, Rafael. ===== Rafael Werneck Cinoto ICQ# 107011599 rwcinoto@yahoo.com rafael.caixa@gov.com.br matduvidas@yahoo.com.br http://www.rwcinoto.hpg.com.br/ __________________________________________________ Do You Yahoo!? Yahoo! - Official partner of 2002 FIFA World Cup http://fifaworldcup.yahoo.com ========================================================================= Instruções para entrar na lista, sair da lista e usar a lista em http://www.mat.puc-rio.br/~nicolau/olimp/obm-l.html O administrador desta lista é ========================================================================= From owner-obm-l@sucuri.mat.puc-rio.br Tue Jun 4 21:21:48 2002 Return-Path: Received: (from majordom@localhost) by sucuri.mat.puc-rio.br (8.9.3/8.9.3) id VAA02379 for obm-l-list; Tue, 4 Jun 2002 21:21:05 -0300 Received: from cairu.terra.com.br (cairu.terra.com.br [200.176.3.19]) by sucuri.mat.puc-rio.br (8.9.3/8.9.3) with ESMTP id VAA02375 for ; Tue, 4 Jun 2002 21:21:03 -0300 Received: from pacuiba.terra.com.br (pacuiba.terra.com.br [200.176.3.40]) by cairu.terra.com.br (Postfix) with ESMTP id DF34147318; Tue, 4 Jun 2002 21:09:46 +0000 (GMT) Received: from nt (RJ231080.user.veloxzone.com.br [200.165.231.80]) (authenticated user ensr) by pacuiba.terra.com.br (Postfix) with ESMTP id 22F098018; Tue, 4 Jun 2002 21:09:51 -0300 (EST) Message-ID: <00a901c20c25$0a2c64c0$5400a8c0@ensrbr> From: "Luis Lopes" To: References: <171.e6d83f3.2a2a9622@aol.com> <3CFCC6F0.6CA5DC9B@terra.com.br> Subject: [obm-l] integral sem fazer a conta Date: Tue, 4 Jun 2002 21:07:53 -0300 MIME-Version: 1.0 Content-Type: multipart/alternative; boundary="----=_NextPart_000_00A6_01C20C0B.E45144A0" X-Priority: 3 X-MSMail-Priority: Normal X-Mailer: Microsoft Outlook Express 5.00.2615.200 X-MimeOLE: Produced By Microsoft MimeOLE V5.00.2615.200 Sender: owner-obm-l@sucuri.mat.puc-rio.br Precedence: bulk Reply-To: obm-l@mat.puc-rio.br This is a multi-part message in MIME format. ------=_NextPart_000_00A6_01C20C0B.E45144A0 Content-Type: text/plain; charset="iso-8859-1" Content-Transfer-Encoding: quoted-printable Sauda,c~oes, Algu=E9m poderia me mostrar por que \int_{-1}^1 \frac{(1+u)du}{u^2 + (1-x^2)/x^2} =3D=20 2\int_0^1 \frac{du}{u^2 + (1-x^2)/x^2} sem fazer as contas? Observe as mudan=E7as nos limites da integral e no numerador do integrando. Ou me dizer um livro de C=E1lculo que mostra isso de maneira geral? Como sempre, \frac{A}{B} =3D A/B. []'s Lu=EDs ------=_NextPart_000_00A6_01C20C0B.E45144A0 Content-Type: text/html; charset="iso-8859-1" Content-Transfer-Encoding: quoted-printable
Sauda,c~oes,
 
Algu=E9m poderia me mostrar por que
 
\int_{-1}^1 \frac{(1+u)du}{u^2 + (1-x^2)/x^2} =  =3D=20
 
2\int_0^1 \frac{du}{u^2 + (1-x^2)/x^2}
 
sem fazer as contas?
 
Observe as mudan=E7as nos limites da integral
e no numerador do integrando.
 
Ou me dizer um livro de C=E1lculo que mostra
isso de maneira geral?
 
Como sempre, \frac{A}{B} =3D A/B.
 
[]'s
Lu=EDs
 
------=_NextPart_000_00A6_01C20C0B.E45144A0-- ========================================================================= Instruções para entrar na lista, sair da lista e usar a lista em http://www.mat.puc-rio.br/~nicolau/olimp/obm-l.html O administrador desta lista é ========================================================================= From owner-obm-l@sucuri.mat.puc-rio.br Tue Jun 4 22:06:28 2002 Return-Path: Received: (from majordom@localhost) by sucuri.mat.puc-rio.br (8.9.3/8.9.3) id WAA03493 for obm-l-list; Tue, 4 Jun 2002 22:05:49 -0300 Received: from candeias.terra.com.br (candeias.terra.com.br [200.176.3.18]) by sucuri.mat.puc-rio.br (8.9.3/8.9.3) with ESMTP id WAA03489 for ; Tue, 4 Jun 2002 22:05:47 -0300 Received: from penha.terra.com.br (penha.terra.com.br [200.176.3.43]) by candeias.terra.com.br (Postfix) with ESMTP id AEC9E43E44 for ; Tue, 4 Jun 2002 21:54:36 -0300 (EST) Received: from Lucelindo (unknown [200.177.64.115]) (authenticated user ldias1) by penha.terra.com.br (Postfix) with ESMTP id D28816809E for ; Tue, 4 Jun 2002 21:54:36 -0300 (EST) Message-ID: <001201c20c2a$e76aefa0$7340b1c8@Lucelindo> From: "Lucelindo D. Ferreira" To: References: <20020604221824.55071.qmail@web10103.mail.yahoo.com> Subject: Re: [obm-l] bissetriz interna Date: Tue, 4 Jun 2002 21:49:49 -0300 MIME-Version: 1.0 Content-Type: text/plain; charset="iso-8859-1" Content-Transfer-Encoding: 8bit X-Priority: 3 X-MSMail-Priority: Normal X-Mailer: Microsoft Outlook Express 5.00.2615.200 X-MimeOLE: Produced By Microsoft MimeOLE V5.00.2615.200 Sender: owner-obm-l@sucuri.mat.puc-rio.br Precedence: bulk Reply-To: obm-l@mat.puc-rio.br Holla! Eu tenho a impressão de q se vc usar o teorema de stewart aliado ao teorema da bissetriz vc vai achar o valor das projeções em função do comprimento da bissetriz. Veja se ñ dá muita conta pq aí a gente pode ver outra maneira de resolver. Um abraço! ----- Original Message ----- From: Rafael WC To: OBM Sent: Tuesday, June 04, 2002 7:18 PM Subject: [obm-l] bissetriz interna > Olá Pessoal! > > Esta questão não parece difícil, mas não consegui > muita coisa além de potência de ponto e o teorema da > bissetriz interna. > > Em um triangulo ABC, a bissetriz interna de A encontra > BC em D e o círculo circunscrito em E. Se AB = 8, AC = > 6 e DE = 3 calcule o comprimento da bissetriz AD. > > Se alguém conseguir, agradeço qualquer dica. > > Obrigado, > > Rafael. > > ===== > Rafael Werneck Cinoto > ICQ# 107011599 > rwcinoto@yahoo.com > rafael.caixa@gov.com.br > matduvidas@yahoo.com.br > http://www.rwcinoto.hpg.com.br/ > > __________________________________________________ > Do You Yahoo!? > Yahoo! - Official partner of 2002 FIFA World Cup > http://fifaworldcup.yahoo.com > ========================================================================= > Instruções para entrar na lista, sair da lista e usar a lista em > http://www.mat.puc-rio.br/~nicolau/olimp/obm-l.html > O administrador desta lista é > ========================================================================= > ========================================================================= Instruções para entrar na lista, sair da lista e usar a lista em http://www.mat.puc-rio.br/~nicolau/olimp/obm-l.html O administrador desta lista é ========================================================================= From owner-obm-l@sucuri.mat.puc-rio.br Wed Jun 5 01:13:49 2002 Return-Path: Received: (from majordom@localhost) by sucuri.mat.puc-rio.br (8.9.3/8.9.3) id BAA05413 for obm-l-list; Wed, 5 Jun 2002 01:13:18 -0300 Received: from hotmail.com (f156.pav1.hotmail.com [64.4.31.156]) by sucuri.mat.puc-rio.br (8.9.3/8.9.3) with ESMTP id BAA05409 for ; Wed, 5 Jun 2002 01:13:16 -0300 Received: from mail pickup service by hotmail.com with Microsoft SMTPSVC; Tue, 4 Jun 2002 21:02:04 -0700 Received: from 200.151.13.60 by pv1fd.pav1.hotmail.msn.com with HTTP; Wed, 05 Jun 2002 04:02:03 GMT X-Originating-IP: [200.151.13.60] From: "Adriano Almeida Faustino" To: obm-l@mat.puc-rio.br Subject: [obm-l] trigonometria Date: Wed, 05 Jun 2002 04:02:03 +0000 Mime-Version: 1.0 Content-Type: text/plain; charset=iso-8859-1; format=flowed Message-ID: X-OriginalArrivalTime: 05 Jun 2002 04:02:04.0026 (UTC) FILETIME=[C06A3DA0:01C20C45] Sender: owner-obm-l@sucuri.mat.puc-rio.br Precedence: bulk Reply-To: obm-l@mat.puc-rio.br Solucione a equacao cos(cos(cos(cos x))) = sen(sen(sen(sen x))) []`s Adriano. _________________________________________________________________ Envie e receba emails com o Hotmail no seu dispositivo móvel: http://mobile.msn.com ========================================================================= Instruções para entrar na lista, sair da lista e usar a lista em http://www.mat.puc-rio.br/~nicolau/olimp/obm-l.html O administrador desta lista é ========================================================================= From owner-obm-l@sucuri.mat.puc-rio.br Wed Jun 5 01:28:25 2002 Return-Path: Received: (from majordom@localhost) by sucuri.mat.puc-rio.br (8.9.3/8.9.3) id BAA05735 for obm-l-list; Wed, 5 Jun 2002 01:28:22 -0300 Received: from web13702.mail.yahoo.com (web13702.mail.yahoo.com [216.136.175.135]) by sucuri.mat.puc-rio.br (8.9.3/8.9.3) with SMTP id BAA05731 for ; Wed, 5 Jun 2002 01:28:20 -0300 Message-ID: <20020605041708.21176.qmail@web13702.mail.yahoo.com> Received: from [200.154.7.53] by web13702.mail.yahoo.com via HTTP; Wed, 05 Jun 2002 01:17:08 ART Date: Wed, 5 Jun 2002 01:17:08 -0300 (ART) From: =?iso-8859-1?q?Andre=20S?= Subject: Re: [obm-l] Variação_na_questão_do_IME:_x=sqrt(a-sqrt(a-x)) To: obm-l@mat.puc-rio.br In-Reply-To: <765A72978645D4118B1C0000E229806D05B6B3B6@FGVRJ23> MIME-Version: 1.0 Content-Type: text/plain; charset=iso-8859-1 Content-Transfer-Encoding: 8bit Sender: owner-obm-l@sucuri.mat.puc-rio.br Precedence: bulk Reply-To: obm-l@mat.puc-rio.br Ralph, em alguma parte do problema eu devo ter cometido algum engano, pq achei 4 raízes para x=sqrt(0,8 - sqrt(0,8-x))... segue meu desenvolvimento... (x^2 - 0,8)^2=0,8 - x (I) x^4 - 1,6x^2 + x - 0,16 = 0 Utilizando-se x=sqrt(a - x), descobre-se 2 das 4 possiveis raízes de x=sqrt(a-sqrt(a-x)) logo, duas raízes de (I) estão em (II) x^2 + x - 0,8 = 0 {(-1+sqrt(4,2))/2;(-1-sqrt(4,2))/2} considerando (I) na forma (x - x1)(x - x2)(x - x3)(x - x4) = 0 , sendo x2, x2, x3, x4 raízes, e (II) na forma (x - x1)(x - x2) = 0, a divisão (I)/(II) deve apresentar uma equação do 2º grau com as duas soluções que faltam. (x^4 - 1,6x^2 + x - 0,16) / (x^2 +x -0,8) = (III) x^2 - x + 0,2 = 0 {(1+sqrt(0,2))/2;1-sqrt(0,2))/2} Dessa forma, seriam soluções da equação inicial S = {(-1+sqrt(4,2))/2;(-1-sqrt(4,2))/2;(1+sqrt(0,2))/2;1-sqrt(0,2))/2} Onde foi meu erro? []'s, A.S. --- Ralph Teixeira escreveu: > Olá, galera. > > Um colega nosso da lista, o Cláudio, destacou que > eu havia me > enganado quando disse que a equação > > x=sqrt(5-sqrt(5-x)) > > tem *DUAS* soluções. Ele tem razão -- apesar de eu > ainda defender o > fato de que você NÃO PODE SIMPLESMENTE DIZER QUE > x=sqrt(5-x), o meu método > acaba por gerar apenas uma raiz de qualquer forma > (eu havia cometido um erro > de álgebra que o Cláudio encontrou). > > Por exemplo, a equação x=sqrt(0.8-sqrt(0.8-x)) tem > 3 raízes reais > (você consegue encontrá-las?). Apenas *1* delas é > encontrada fazendo > x=sqrt(0.8-x). > > Minha perguntinha para a galera é então: para que > valores de a as > equações > x=sqrt(a-sqrt(a-x)) e x=sqrt(a-x) são equivalentes? > > Divirtam-se! > > Abraço, > Ralph > ========================================================================= > Instruções para entrar na lista, sair da lista e > usar a lista em > http://www.mat.puc-rio.br/~nicolau/olimp/obm-l.html > O administrador desta lista é > > ========================================================================= _______________________________________________________________________ Yahoo! Encontros O lugar certo para você encontrar aquela pessoa que falta na sua vida. Cadastre-se hoje mesmo! http://br.encontros.yahoo.com/ ========================================================================= Instruções para entrar na lista, sair da lista e usar a lista em http://www.mat.puc-rio.br/~nicolau/olimp/obm-l.html O administrador desta lista é ========================================================================= From owner-obm-l@sucuri.mat.puc-rio.br Wed Jun 5 02:52:28 2002 Return-Path: Received: (from majordom@localhost) by sucuri.mat.puc-rio.br (8.9.3/8.9.3) id CAA06698 for obm-l-list; Wed, 5 Jun 2002 02:52:14 -0300 Received: from smtp-4.ig.com.br (smtp-4.ig.com.br [200.226.132.153]) by sucuri.mat.puc-rio.br (8.9.3/8.9.3) with SMTP id CAA06694 for ; Wed, 5 Jun 2002 02:52:13 -0300 Received: (qmail 11854 invoked from network); 5 Jun 2002 05:40:46 -0000 Received: from shasta064166.ig.com.br (HELO jat) (200.151.64.166) by smtp-4.ig.com.br with SMTP; 5 Jun 2002 05:40:46 -0000 Message-ID: <004201c20c53$8be5b060$a64097c8@jat> From: "Jose Augusto" To: "Lista de Discussao" Subject: [obm-l] =?iso-8859-1?Q?pq_essa_diferencia=E7=E3o_..._?= Date: Wed, 5 Jun 2002 02:40:30 -0300 MIME-Version: 1.0 Content-Type: multipart/alternative; boundary="----=_NextPart_000_0039_01C20C3A.5B901180" X-Priority: 3 X-MSMail-Priority: Normal X-Mailer: Microsoft Outlook Express 5.50.4133.2400 X-MIMEOLE: Produced By Microsoft MimeOLE V5.50.4133.2400 Sender: owner-obm-l@sucuri.mat.puc-rio.br Precedence: bulk Reply-To: obm-l@mat.puc-rio.br This is a multi-part message in MIME format. ------=_NextPart_000_0039_01C20C3A.5B901180 Content-Type: text/plain; charset="iso-8859-1" Content-Transfer-Encoding: quoted-printable Pq ha essa diferencia=E7ao entre "geometria paulista" e "geometria = cearence"???? Pq todos paulistas que se prezem resolvem problemas nao por geometria = cearence???? Eh para resolvermos problemas desde geometria cearence ate analitica = .. trigronometria tbm mesmo quando nao paulistas??? =20 ------=_NextPart_000_0039_01C20C3A.5B901180 Content-Type: text/html; charset="iso-8859-1" Content-Transfer-Encoding: quoted-printable
    Pq ha essa = diferencia=E7ao entre=20 "geometria paulista" e "geometria cearence"????
   Pq todos paulistas que se = prezem=20 resolvem problemas nao por geometria cearence????
   Eh para resolvermos = problemas desde=20 geometria cearence ate analitica .. trigronometria tbm mesmo quando nao=20 paulistas???
 
------=_NextPart_000_0039_01C20C3A.5B901180-- ========================================================================= Instruções para entrar na lista, sair da lista e usar a lista em http://www.mat.puc-rio.br/~nicolau/olimp/obm-l.html O administrador desta lista é ========================================================================= From owner-obm-l@sucuri.mat.puc-rio.br Wed Jun 5 02:53:01 2002 Return-Path: Received: (from majordom@localhost) by sucuri.mat.puc-rio.br (8.9.3/8.9.3) id CAA06724 for obm-l-list; Wed, 5 Jun 2002 02:52:59 -0300 Received: from smtp-3.ig.com.br (smtp-3.ig.com.br [200.226.132.152]) by sucuri.mat.puc-rio.br (8.9.3/8.9.3) with SMTP id CAA06720 for ; Wed, 5 Jun 2002 02:52:57 -0300 Received: (qmail 15326 invoked from network); 5 Jun 2002 05:41:30 -0000 Received: from shasta064166.ig.com.br (HELO jat) (200.151.64.166) by smtp-3.ig.com.br with SMTP; 5 Jun 2002 05:41:30 -0000 Message-ID: <005a01c20c53$a637f860$a64097c8@jat> From: "Jose Augusto" To: "Lista de Discussao" Subject: [obm-l] =?iso-8859-1?Q?pq_essa_diferencia=E7=E3o_..._?= Date: Wed, 5 Jun 2002 02:41:03 -0300 MIME-Version: 1.0 Content-Type: multipart/alternative; boundary="----=_NextPart_000_0043_01C20C3A.6F5568A0" X-Priority: 3 X-MSMail-Priority: Normal X-Mailer: Microsoft Outlook Express 5.50.4133.2400 X-MIMEOLE: Produced By Microsoft MimeOLE V5.50.4133.2400 Sender: owner-obm-l@sucuri.mat.puc-rio.br Precedence: bulk Reply-To: obm-l@mat.puc-rio.br This is a multi-part message in MIME format. ------=_NextPart_000_0043_01C20C3A.6F5568A0 Content-Type: text/plain; charset="iso-8859-1" Content-Transfer-Encoding: quoted-printable Pq ha essa diferencia=E7ao entre "geometria paulista" e "geometria = cearence"???? Pq todos paulistas que se prezem resolvem problemas nao por geometria = cearence???? Eh para resolvermos problemas desde geometria cearence ate analitica = .. trigronometria tbm mesmo quando nao paulistas??? .. Ah, desculpem -me ,.. sou novo na lista ,,,=20 Jose Augusto Tavares ------=_NextPart_000_0043_01C20C3A.6F5568A0 Content-Type: text/html; charset="iso-8859-1" Content-Transfer-Encoding: quoted-printable
    Pq ha essa = diferencia=E7ao entre=20 "geometria paulista" e "geometria cearence"????
   Pq todos paulistas que se = prezem=20 resolvem problemas nao por geometria cearence????
   Eh para resolvermos = problemas desde=20 geometria cearence ate analitica .. trigronometria tbm mesmo quando nao=20 paulistas???
..
  Ah, desculpem -me ,.. sou novo = na lista ,,,=20
          &nbs= p;       =20 Jose Augusto Tavares
------=_NextPart_000_0043_01C20C3A.6F5568A0-- ========================================================================= Instruções para entrar na lista, sair da lista e usar a lista em http://www.mat.puc-rio.br/~nicolau/olimp/obm-l.html O administrador desta lista é ========================================================================= From owner-obm-l@sucuri.mat.puc-rio.br Wed Jun 5 10:41:25 2002 Return-Path: Received: (from majordom@localhost) by sucuri.mat.puc-rio.br (8.9.3/8.9.3) id KAA11100 for obm-l-list; Wed, 5 Jun 2002 10:40:09 -0300 Received: from saks.bol.com.br (saks.bol.com.br [200.221.24.16]) by sucuri.mat.puc-rio.br (8.9.3/8.9.3) with ESMTP id KAA11096 for ; Wed, 5 Jun 2002 10:40:07 -0300 Received: from bol.com.br (200.221.24.110) by saks.bol.com.br (5.1.071) id 3CDE092B006C436C for obm-l@mat.puc-rio.br; Wed, 5 Jun 2002 10:28:06 -0300 Date: Wed, 5 Jun 2002 10:28:06 -0300 Message-Id: Subject: Re:[obm-l] integral sem fazer a conta MIME-Version: 1.0 Content-Type: text/plain;charset="iso-8859-1" From: "ozorio_loof" To: obm-l@mat.puc-rio.br X-XaM3-API-Version: 2.4.3.4.4 X-SenderIP: 143.107.130.187 Content-Transfer-Encoding: 8bit X-MIME-Autoconverted: from quoted-printable to 8bit by sucuri.mat.puc-rio.br id KAA11097 Sender: owner-obm-l@sucuri.mat.puc-rio.br Precedence: bulk Reply-To: obm-l@mat.puc-rio.br Observe que se vc desmembrar a integral em duas, a primeira será \int_{-1}^1 \frac{du}{u^2 + (1-x^2)/x^2} e a outra será zero (integral de uma função ímpar no limite simétrico), daí é imediato o resultado procurado. \int_{-1}^1 \frac{du}{u^2 + (1-x^2)/x^2} = 2\int_0^1 \frac{du}{u^2 + (1-x^2)/x^2}. []'s Luiz. > Sauda,c~oes, > > Alguém poderia me mostrar por que > > \int_{-1}^1 \frac{(1+u)du}{u^2 + (1-x^2)/x^2} = > > 2\int_0^1 \frac{du}{u^2 + (1-x^2)/x^2} > > sem fazer as contas? > > Observe as mudanças nos limites da integral > e no numerador do integrando. > > Ou me dizer um livro de Cálculo que mostra > isso de maneira geral? > > Como sempre, \frac{A}{B} = A/B. > > []'s > Luís > > __________________________________________________________________________ Quer ter seu próprio endereço na Internet? Garanta já o seu e ainda ganhe cinco e-mails personalizados. DomíniosBOL - http://dominios.bol.com.br ========================================================================= Instruções para entrar na lista, sair da lista e usar a lista em http://www.mat.puc-rio.br/~nicolau/olimp/obm-l.html O administrador desta lista é ========================================================================= From owner-obm-l@sucuri.mat.puc-rio.br Wed Jun 5 14:16:58 2002 Return-Path: Received: (from majordom@localhost) by sucuri.mat.puc-rio.br (8.9.3/8.9.3) id OAA14681 for obm-l-list; Wed, 5 Jun 2002 14:14:45 -0300 Received: from hotmail.com (f14.law15.hotmail.com [64.4.23.14]) by sucuri.mat.puc-rio.br (8.9.3/8.9.3) with ESMTP id OAA14677 for ; Wed, 5 Jun 2002 14:14:42 -0300 Received: from mail pickup service by hotmail.com with Microsoft SMTPSVC; Wed, 5 Jun 2002 10:03:31 -0700 Received: from 200.206.103.3 by lw15fd.law15.hotmail.msn.com with HTTP; Wed, 05 Jun 2002 17:03:31 GMT X-Originating-IP: [200.206.103.3] From: "RICARDO CHAVES" To: obm-l@mat.puc-rio.br Subject: Re: [obm-l] Date: Wed, 05 Jun 2002 17:03:31 +0000 Mime-Version: 1.0 Content-Type: text/html; charset=iso-8859-1 Message-ID: X-OriginalArrivalTime: 05 Jun 2002 17:03:31.0780 (UTC) FILETIME=[EBB1B840:01C20CB2] Sender: owner-obm-l@sucuri.mat.puc-rio.br Precedence: bulk Reply-To: obm-l@mat.puc-rio.br

Use Pell

>From: "aleixocarvalho"
>Reply-To: obm-l@mat.puc-rio.br
>To: obm-l@mat.puc-rio.br
>Subject: [obm-l]
>Date: Tue, 4 Jun 2002 18:18:14 -0300
>
>por favor
>
>alguem me demontre que:
>
>" a solucao x^2-2*y^2=1 tem infinitas solucoes dadas por
>x_k+y_k*2^(1/2)=[3+2*2^(1/2)]^k com k=1,2,3,..."
>
>
>__________________________________________________________________________
>Quer ter seu próprio endereço na Internet?
>Garanta já o seu e ainda ganhe cinco e-mails personalizados.
>DomíniosBOL - http://dominios.bol.com.br
>
>
>=========================================================================
>Instruções para entrar na lista, sair da lista e usar a lista em
>http://www.mat.puc-rio.br/~nicolau/olimp/obm-l.html
>O administrador desta lista é
>=========================================================================


Una-se ao maior serviço de email do mundo: o MSN Hotmail. Clique aqui
========================================================================= Instruções para entrar na lista, sair da lista e usar a lista em http://www.mat.puc-rio.br/~nicolau/olimp/obm-l.html O administrador desta lista é ========================================================================= From owner-obm-l@sucuri.mat.puc-rio.br Wed Jun 5 15:39:18 2002 Return-Path: Received: (from majordom@localhost) by sucuri.mat.puc-rio.br (8.9.3/8.9.3) id PAA16146 for obm-l-list; Wed, 5 Jun 2002 15:36:25 -0300 Received: from www.zipmail.com.br (smtp.zipmail.com.br [200.187.242.10]) by sucuri.mat.puc-rio.br (8.9.3/8.9.3) with ESMTP id PAA16125; Wed, 5 Jun 2002 15:35:42 -0300 From: peterdirichlet@zipmail.com.br Received: from [200.206.103.3] by www.zipmail.com.br with HTTP; Wed, 5 Jun 2002 15:24:24 -0300 Message-ID: <3CFE52570000012C@www.zipmail.com.br> Date: Wed, 5 Jun 2002 16:24:24 -0200 Subject: [obm-l] =?iso-8859-1?Q?URGENTE=21=21=21=21=21=21=21=21=21=21=21=21=21Sobre=3ACl=E1ssicos=20geom=E9tricos=2Cguerras?= =?iso-8859-1?Q?=20ideol=F3gico=2Dmatem=E1ticas=20e=20assuntos=20afins?= To: =?iso-8859-1?Q?Nicolau?= , =?iso-8859-1?Q?Olimpiada=20Brasileira=20de=20Matem=2E?= , =?iso-8859-1?Q?Paulo=20Rodrigues?= , =?iso-8859-1?Q?Lista=20de=20Discussao?= , =?iso-8859-1?Q?Nicolau?= , =?iso-8859-1?Q?Olimpiada=20Brasileira=20de=20Matem=2E?= , =?iso-8859-1?Q?Paulo=20Rodrigues?= , =?iso-8859-1?Q?Lista=20de=20Discussao?= MIME-Version: 1.0 Content-Type: text/plain; charset="iso-8859-1" Content-Transfer-Encoding: 8bit X-MIME-Autoconverted: from quoted-printable to 8bit by sucuri.mat.puc-rio.br id PAA16135 Sender: owner-obm-l@sucuri.mat.puc-rio.br Precedence: bulk Reply-To: obm-l@mat.puc-rio.br O MINISTERIO DA SAUDE ADVERTE:LER E-MAILS LONGOS PODE PROVOCAR SONOLENCIA FORTE E PARANOIAS CONSTANTES. ISTO E URGENTE!!!!!!!!!!!!!!! Gente,aqui esta uma leva de geometricos.Eles tem uma pequena historia,que escrevi ao final por motivos obvios. 1.(Casey)Considere quatro circunferencias S1,S2,S3,S4(podendo algumas delas ter raio zero) coplanares. Prove que elas sao coinscritiveis(ha uma circunferencia tangente externamente a todas elas)se e so se t12*t34+t14*t23=t13*t24,em que txy e o valor da tangente externa as circunferencias Sx e Sy 2.No triangulo ABC,CA=AB,CAB=20°,CBD=60° e ECB=50°,sendo EC e DB duas cevianas.Calcule BDE. 3.Considere o triangulo retangulo de hipotenusa 4 e cateto 2.Inscreve-se nele um triangulo equilatero tal que um de seus vertices bissecte o cateto de lado 2.Ache o valor do lado desse triangulo inscrito. Quem resolver alguma dessas questoes eu agradeço.E enfim a HISTORIA da guerra Tudo isto começou quando eu estava arrumando meus arquivos de problemas olimpicos resolvidos(por mim mesmo,pelos professores Shine e Tengan nas aulas no "País da OBM"(o nickname que eu dei para a sede regional da OBM em Sampa),pela turma da lista,por uns colegas meus,tirados de sites,etc. e tal).Acabei pegando uma folha com a soluçao do problema 5 da IMO 2001.Eu resolvi-o por Trigonometria(como todo paulista que se preze),como fez o Thiago,e adaptei a soluçao oficial(totalmente Geometria Cearense:pontos magicos,semelhança,coisa e tal).Enfim,propus este problema a um colega de sala,e ele ficou a aula inteira tentando mas nao resolveu.Entao eu mostrei-lhe a minha soluçao.Mal tinha acabado,ele soltou os cachorros em cima de mim,falando que eu tinha apelado,que minha soluçao nao era elegante,e por ai vai.Entao eu resolvi mostrar a ele a soluçao oficial.Ele achou artificial mas elegante(bonitinha mas ordinaria...),e depois ele me disse que tinha asco a Trigonometria.Entao eu lhe propus outros 2 problemas classicos.Ele esta tentando resolve-los,me dara noticias amanha. Enquanto isso,eu resolvi coloca-los nesta lista par que voces os discutissem e me dessem algumas respostas(de todos os tipos,podendo usar desde geometria cearense ate geometria analitica) e me dissessem como posso fazer este meu obstinado amigo mudar de ideia ou pelo menos nao odiar tanto a geometria paulista.E essa e a historia! Assinado:Peterdirichlet TRANSIRE SVVM PECTVS MVNDOQUE POTIRE CONGREGATI EX TOTO ORBE MATHEMATICI OB SCRIPTA INSIGNIA TRIBVERE Medalha Fields(John Charles Fields) ------------------------------------------ Use o melhor sistema de busca da Internet Radar UOL - http://www.radaruol.com.br ========================================================================= Instruções para entrar na lista, sair da lista e usar a lista em http://www.mat.puc-rio.br/~nicolau/olimp/obm-l.html O administrador desta lista é ========================================================================= From owner-obm-l@sucuri.mat.puc-rio.br Wed Jun 5 15:39:22 2002 Return-Path: Received: (from majordom@localhost) by sucuri.mat.puc-rio.br (8.9.3/8.9.3) id PAA16117 for obm-l-list; Wed, 5 Jun 2002 15:35:01 -0300 Received: from matinhos.terra.com.br (matinhos.terra.com.br [200.176.3.21]) by sucuri.mat.puc-rio.br (8.9.3/8.9.3) with ESMTP id PAA16113 for ; Wed, 5 Jun 2002 15:34:57 -0300 Received: from pavuna.terra.com.br (pavuna.terra.com.br [200.176.3.41]) by matinhos.terra.com.br (Postfix) with ESMTP id 850FC4738D for ; Wed, 5 Jun 2002 15:23:44 -0300 (EST) Received: from nt (RJ231080.user.veloxzone.com.br [200.165.231.80]) (authenticated user ensr) by pavuna.terra.com.br (Postfix) with ESMTP id 52C67680F6 for ; Wed, 5 Jun 2002 15:23:42 -0300 (EST) Message-ID: <00be01c20cbd$dac08ec0$5400a8c0@ensrbr> From: "Luis Lopes" To: References: Subject: Re: Re:[obm-l] integral sem fazer a conta Date: Wed, 5 Jun 2002 15:21:39 -0300 MIME-Version: 1.0 Content-Type: text/plain; charset="iso-8859-1" Content-Transfer-Encoding: 8bit X-Priority: 3 X-MSMail-Priority: Normal X-Mailer: Microsoft Outlook Express 5.00.2615.200 X-MimeOLE: Produced By Microsoft MimeOLE V5.00.2615.200 Sender: owner-obm-l@sucuri.mat.puc-rio.br Precedence: bulk Reply-To: obm-l@mat.puc-rio.br Sauda,c~oes, É verdade. Bobeei nessa. Obrigado. Essa integral apareceu no cálculo do desenvolvimento em série de (\Arcsin x)^2. []'s Luís -----Mensagem Original----- De: ozorio_loof Para: Enviada em: quarta-feira, 5 de junho de 2002 10:28 Assunto: Re:[obm-l] integral sem fazer a conta > Observe que se vc desmembrar a > integral em duas, > a primeira será \int_{-1}^1 > \frac{du}{u^2 + (1-x^2)/x^2} e a outra > será zero (integral de uma > função ímpar no limite simétrico), daí > é imediato o resultado procurado. > \int_{-1}^1 \frac{du}{u^2 + > (1-x^2)/x^2} = > 2\int_0^1 \frac{du}{u^2 + > (1-x^2)/x^2}. > > []'s > Luiz. ========================================================================= Instruções para entrar na lista, sair da lista e usar a lista em http://www.mat.puc-rio.br/~nicolau/olimp/obm-l.html O administrador desta lista é ========================================================================= From owner-obm-l@sucuri.mat.puc-rio.br Wed Jun 5 16:30:05 2002 Return-Path: Received: (from majordom@localhost) by sucuri.mat.puc-rio.br (8.9.3/8.9.3) id QAA17399 for obm-l-list; Wed, 5 Jun 2002 16:28:29 -0300 Received: from www.zipmail.com.br (smtp.zipmail.com.br [200.187.242.10]) by sucuri.mat.puc-rio.br (8.9.3/8.9.3) with ESMTP id QAA17382 for ; Wed, 5 Jun 2002 16:27:51 -0300 From: peterdirichlet@zipmail.com.br Received: from [200.206.103.3] by www.zipmail.com.br with HTTP; Wed, 5 Jun 2002 16:16:31 -0300 Message-ID: <3CFE525700000464@www.zipmail.com.br> Date: Wed, 5 Jun 2002 17:16:31 -0200 Subject: [obm-l] =?iso-8859-1?Q?Como=20eu=20posso=20achar=20os=20professores=20Elon=20Lima=20e=20Gugu=3FPro?= =?iso-8859-1?Q?blema=206=2CIMO=202001=2DSolu=E7ao=20=20?= To: =?iso-8859-1?Q?Lista=20de=20Discussao?= , =?iso-8859-1?Q?Olimpiada=20Brasileira=20de=20Matem=2E?= MIME-Version: 1.0 Content-Type: text/plain; charset="iso-8859-1" Content-Transfer-Encoding: 8bit X-MIME-Autoconverted: from quoted-printable to 8bit by sucuri.mat.puc-rio.br id QAA17384 Sender: owner-obm-l@sucuri.mat.puc-rio.br Precedence: bulk Reply-To: obm-l@mat.puc-rio.br Ola turma da Lista!!!!!!Alguem sabe como eu posso falar com os professores Elon Lages Lima e Carlos Gustavo Tamm de Araujo Moreira(o Tengan me disse que ele obteve uma soluçao bem legal,mas na porrada,do problema 6 da IMO 2001,EUA.Ai eu queria que ele mostrasse)? ATEEEEEEEEEEEEEEEEEEEEEE!!!!!!!!!!!!!!Ploft!Peterdirichlet. TRANSIRE SVVM PECTVS MVNDOQUE POTIRE CONGREGATI EX TOTO ORBE MATHEMATICI OB SCRIPTA INSIGNIA TRIBVERE Medalha Fields(John Charles Fields) ------------------------------------------ Use o melhor sistema de busca da Internet Radar UOL - http://www.radaruol.com.br ========================================================================= Instruções para entrar na lista, sair da lista e usar a lista em http://www.mat.puc-rio.br/~nicolau/olimp/obm-l.html O administrador desta lista é ========================================================================= From owner-obm-l@sucuri.mat.puc-rio.br Wed Jun 5 18:45:20 2002 Return-Path: Received: (from majordom@localhost) by sucuri.mat.puc-rio.br (8.9.3/8.9.3) id SAA19983 for obm-l-list; Wed, 5 Jun 2002 18:43:27 -0300 Received: from ns-3.idc.dglnet.com.br (ns-3.idc.dglnet.com.br [200.218.161.4]) by sucuri.mat.puc-rio.br (8.9.3/8.9.3) with ESMTP id SAA19979 for ; Wed, 5 Jun 2002 18:43:25 -0300 Received: from user (200-204-37-227.dial-up.telesp.net.br [200.204.37.227]) by ns-3.idc.dglnet.com.br (Postfix) with SMTP id 6705A150010 for ; Wed, 5 Jun 2002 18:32:15 -0300 (BRT) Message-ID: <001901c20cd8$7e2c58e0$e325ccc8@user> From: "Claudio" To: References: <20020604221824.55071.qmail@web10103.mail.yahoo.com> Subject: Re: [obm-l] bissetriz interna Date: Wed, 5 Jun 2002 18:32:27 -0300 MIME-Version: 1.0 Content-Type: text/plain; charset="iso-8859-1" Content-Transfer-Encoding: 8bit X-Priority: 3 X-MSMail-Priority: Normal X-Mailer: Microsoft Outlook Express 5.00.2314.1300 X-MimeOLE: Produced By Microsoft MimeOLE V5.00.2314.1300 Sender: owner-obm-l@sucuri.mat.puc-rio.br Precedence: bulk Reply-To: obm-l@mat.puc-rio.br Há uma semelhança que resolve o problema. Os triângulos ABD e AEC são semelhantes ( nessa ordem ). Chamando AD de b vem: 8 : ( b+3) = b : 6. Daí b=(sqrt(201) - 3)/2. Confira. Saludos. Claudio. ----- Original Message ----- From: Rafael WC To: OBM Sent: Tuesday, June 04, 2002 7:18 PM Subject: [obm-l] bissetriz interna > Olá Pessoal! > > Esta questão não parece difícil, mas não consegui > muita coisa além de potência de ponto e o teorema da > bissetriz interna. > > Em um triangulo ABC, a bissetriz interna de A encontra > BC em D e o círculo circunscrito em E. Se AB = 8, AC = > 6 e DE = 3 calcule o comprimento da bissetriz AD. > > Se alguém conseguir, agradeço qualquer dica. > > Obrigado, > > Rafael. > > ===== > Rafael Werneck Cinoto > ICQ# 107011599 > rwcinoto@yahoo.com > rafael.caixa@gov.com.br > matduvidas@yahoo.com.br > http://www.rwcinoto.hpg.com.br/ > > __________________________________________________ > Do You Yahoo!? > Yahoo! - Official partner of 2002 FIFA World Cup > http://fifaworldcup.yahoo.com > ========================================================================= > Instruções para entrar na lista, sair da lista e usar a lista em > http://www.mat.puc-rio.br/~nicolau/olimp/obm-l.html > O administrador desta lista é > ========================================================================= > ========================================================================= Instruções para entrar na lista, sair da lista e usar a lista em http://www.mat.puc-rio.br/~nicolau/olimp/obm-l.html O administrador desta lista é ========================================================================= From owner-obm-l@sucuri.mat.puc-rio.br Wed Jun 5 19:14:05 2002 Return-Path: Received: (from majordom@localhost) by sucuri.mat.puc-rio.br (8.9.3/8.9.3) id TAA20587 for obm-l-list; Wed, 5 Jun 2002 19:12:22 -0300 Received: from sporus.bol.com.br (sporus.bol.com.br [200.221.24.23]) by sucuri.mat.puc-rio.br (8.9.3/8.9.3) with ESMTP id TAA20582 for ; Wed, 5 Jun 2002 19:12:20 -0300 Received: from bol.com.br (200.221.24.84) by sporus.bol.com.br (5.1.071) id 3CDB0F8A00752E27 for obm-l@mat.puc-rio.br; Wed, 5 Jun 2002 19:00:47 -0300 Date: Wed, 5 Jun 2002 19:00:47 -0300 Message-Id: Subject: Re: [obm-l] MIME-Version: 1.0 Content-Type: text/plain;charset="iso-8859-1" From: "aleixocarvalho" To: obm-l@mat.puc-rio.br X-XaM3-API-Version: 2.4.3.4.4 X-SenderIP: 143.106.2.50 Content-Transfer-Encoding: 8bit X-MIME-Autoconverted: from quoted-printable to 8bit by sucuri.mat.puc-rio.br id TAA20583 Sender: owner-obm-l@sucuri.mat.puc-rio.br Precedence: bulk Reply-To: obm-l@mat.puc-rio.br > meu caro amigo ricardo eu sei q essa equacao eh de pell, eu pedi a demontracao rafael __________________________________________________________________________ Quer ter seu próprio endereço na Internet? Garanta já o seu e ainda ganhe cinco e-mails personalizados. DomíniosBOL - http://dominios.bol.com.br ========================================================================= Instruções para entrar na lista, sair da lista e usar a lista em http://www.mat.puc-rio.br/~nicolau/olimp/obm-l.html O administrador desta lista é ========================================================================= From owner-obm-l@sucuri.mat.puc-rio.br Wed Jun 5 20:38:19 2002 Return-Path: Received: (from majordom@localhost) by sucuri.mat.puc-rio.br (8.9.3/8.9.3) id UAA22049 for obm-l-list; Wed, 5 Jun 2002 20:36:47 -0300 Received: from gorgo.centroin.com.br (gorgo.centroin.com.br [200.225.63.128]) by sucuri.mat.puc-rio.br (8.9.3/8.9.3) with ESMTP id UAA22045 for ; Wed, 5 Jun 2002 20:36:46 -0300 Received: from centroin.com.br (du139c.rjo.centroin.com.br [200.225.58.139]) (authenticated bits=0) by gorgo.centroin.com.br (8.12.2/8.12.1) with ESMTP id g55NQ6ko028421 for ; Wed, 5 Jun 2002 20:26:06 -0300 (BRT) Message-ID: <3CFE9E57.6050600@centroin.com.br> Date: Wed, 05 Jun 2002 20:27:19 -0300 From: Augusto =?ISO-8859-1?Q?C=E9sar?= Morgado User-Agent: Mozilla/5.0 (Windows; U; Win98; en-US; rv:0.9.4.1) Gecko/20020508 Netscape6/6.2.3 X-Accept-Language: en-us MIME-Version: 1.0 To: obm-l@mat.puc-rio.br Subject: Re: [obm-l] integral sem fazer a conta References: Content-Type: multipart/alternative; boundary="------------060103070009080305090900" Sender: owner-obm-l@sucuri.mat.puc-rio.br Precedence: bulk Reply-To: obm-l@mat.puc-rio.br --------------060103070009080305090900 Content-Type: text/plain; charset=ISO-8859-1; format=flowed Content-Transfer-Encoding: 8bit Eu, e creio que muitos outros, quero manifestar minha admiraçao por quem consegue entender alguma coisa escrita em tao exotica notaçao. Morgado ozorio_loof wrote: >Observe que se vc desmembrar a >integral em duas, >a primeira será \int_{-1}^1 >\frac{du}{u^2 + (1-x^2)/x^2} e a outra >será zero (integral de uma >função ímpar no limite simétrico), daí >é imediato o resultado procurado. >\int_{-1}^1 \frac{du}{u^2 + >(1-x^2)/x^2} = >2\int_0^1 \frac{du}{u^2 + >(1-x^2)/x^2}. > >[]'s >Luiz. > >>Sauda,c~oes, >> >>Alguém poderia me mostrar por que >> >>\int_{-1}^1 \frac{(1+u)du}{u^2 + >> >(1-x^2)/x^2} = > >>2\int_0^1 \frac{du}{u^2 + >> >(1-x^2)/x^2} > >>sem fazer as contas? >> >>Observe as mudanças nos limites da >> >integral > >>e no numerador do integrando. >> >>Ou me dizer um livro de Cálculo que >> >mostra > >>isso de maneira geral? >> >>Como sempre, \frac{A}{B} = A/B. >> >>[]'s >>Luís >> >> > > >__________________________________________________________________________ >Quer ter seu próprio endereço na Internet? >Garanta já o seu e ainda ganhe cinco e-mails personalizados. >DomíniosBOL - http://dominios.bol.com.br > > >========================================================================= >Instruções para entrar na lista, sair da lista e usar a lista em >http://www.mat.puc-rio.br/~nicolau/olimp/obm-l.html >O administrador desta lista é >========================================================================= > > --------------060103070009080305090900 Content-Type: text/html; charset=us-ascii Content-Transfer-Encoding: 7bit Eu, e creio que muitos outros, quero manifestar minha admiraçao por quem consegue entender alguma coisa escrita em tao exotica notaçao.
Morgado

ozorio_loof wrote:
Observe que se vc desmembrar a
integral em duas,
a primeira será \int_{-1}^1
\frac{du}{u^2 + (1-x^2)/x^2} e a outra
será zero (integral de uma
função ímpar no limite simétrico), daí
é imediato o resultado procurado.
\int_{-1}^1 \frac{du}{u^2 +
(1-x^2)/x^2} =
2\int_0^1 \frac{du}{u^2 +
(1-x^2)/x^2}.

[]'s
Luiz.

Sauda,c~oes,

Alguém poderia me mostrar por que

\int_{-1}^1 \frac{(1+u)du}{u^2 +
(1-x^2)/x^2}  = 
2\int_0^1 \frac{du}{u^2 +
(1-x^2)/x^2}
sem fazer as contas?

Observe as mudanças nos limites da
integral
e no numerador do integrando.

Ou me dizer um livro de Cálculo que
mostra
isso de maneira geral?

Como sempre, \frac{A}{B} = A/B.

[]'s
Luís




__________________________________________________________________________
Quer ter seu próprio endereço na Internet?
Garanta já o seu e ainda ganhe cinco e-mails personalizados.
DomíniosBOL - http://dominios.bol.com.br


=========================================================================
Instruções para entrar na lista, sair da lista e usar a lista em
http://www.mat.puc-rio.br/~nicolau/olimp/obm-l.html
O administrador desta lista é <nicolau@mat.puc-rio.br>
=========================================================================



--------------060103070009080305090900-- ========================================================================= Instruções para entrar na lista, sair da lista e usar a lista em http://www.mat.puc-rio.br/~nicolau/olimp/obm-l.html O administrador desta lista é ========================================================================= From owner-obm-l@sucuri.mat.puc-rio.br Wed Jun 5 22:22:37 2002 Return-Path: Received: (from majordom@localhost) by sucuri.mat.puc-rio.br (8.9.3/8.9.3) id WAA23582 for obm-l-list; Wed, 5 Jun 2002 22:21:05 -0300 Received: from sporus.bol.com.br (sporus.bol.com.br [200.221.24.23]) by sucuri.mat.puc-rio.br (8.9.3/8.9.3) with ESMTP id WAA23571 for ; Wed, 5 Jun 2002 22:21:03 -0300 Received: from bol.com.br (200.221.24.72) by sporus.bol.com.br (5.1.071) id 3CDB0F8A0075EA41 for obm-l@mat.puc-rio.br; Wed, 5 Jun 2002 22:09:31 -0300 Date: Wed, 5 Jun 2002 22:09:30 -0300 Message-Id: Subject: [obm-l] pai e filho MIME-Version: 1.0 Content-Type: text/plain;charset="iso-8859-1" From: "aleixocarvalho" To: obm-l@mat.puc-rio.br X-XaM3-API-Version: 2.4.3.4.4 X-SenderIP: 143.106.22.144 Content-Transfer-Encoding: 8bit X-MIME-Autoconverted: from quoted-printable to 8bit by sucuri.mat.puc-rio.br id WAA23572 Sender: owner-obm-l@sucuri.mat.puc-rio.br Precedence: bulk Reply-To: obm-l@mat.puc-rio.br em um jogo de fichas,o pai sai com 100 fichas, assim como seu filho, quando o pai perde o filho ganha 6 fichas do pai, e quando o filho perde o pai ganha 4 fichas do pai.Após 20 jogos o filho tem 3 vezes as fichas do pai. Pergunta: quntos jogos o pai ganhou. gostaria de saber a solucao, com resolucao rafael __________________________________________________________________________ Quer ter seu próprio endereço na Internet? Garanta já o seu e ainda ganhe cinco e-mails personalizados. DomíniosBOL - http://dominios.bol.com.br ========================================================================= Instruções para entrar na lista, sair da lista e usar a lista em http://www.mat.puc-rio.br/~nicolau/olimp/obm-l.html O administrador desta lista é ========================================================================= From owner-obm-l@sucuri.mat.puc-rio.br Wed Jun 5 22:43:36 2002 Return-Path: Received: (from majordom@localhost) by sucuri.mat.puc-rio.br (8.9.3/8.9.3) id WAA23992 for obm-l-list; Wed, 5 Jun 2002 22:42:01 -0300 Received: from matinhos.terra.com.br (matinhos.terra.com.br [200.176.3.21]) by sucuri.mat.puc-rio.br (8.9.3/8.9.3) with ESMTP id WAA23987 for ; Wed, 5 Jun 2002 22:41:59 -0300 Received: from engenho.terra.com.br (engenho.terra.com.br [200.176.3.42]) by matinhos.terra.com.br (Postfix) with ESMTP id EAEF246FBC; Wed, 5 Jun 2002 22:30:51 -0300 (EST) Received: from terra.com.br (200-206-243-51.dsl.telesp.net.br [200.206.243.51]) (authenticated user lponce) by engenho.terra.com.br (Postfix) with ESMTP id 6A376680C0; Wed, 5 Jun 2002 22:30:52 -0300 (EST) Message-ID: <3CFEBB11.BC1BC01@terra.com.br> Date: Wed, 05 Jun 2002 22:29:55 -0300 From: Luiz Antonio Ponce Alonso X-Mailer: Mozilla 4.79 [en] (Windows NT 5.0; U) X-Accept-Language: en MIME-Version: 1.0 To: obm-l@mat.puc-rio.br Cc: "Olimpiada Brasileira de Matem." Subject: Re: [obm-l] Como eu posso achar os professores Elon Lima e Gugu?Problema 6,IMO =?iso-8859-1?Q?2001=2DSolu=E7ao?= References: <3CFE525700000464@www.zipmail.com.br> Content-Type: text/plain; charset=iso-8859-1 Content-Transfer-Encoding: 8bit Sender: owner-obm-l@sucuri.mat.puc-rio.br Precedence: bulk Reply-To: obm-l@mat.puc-rio.br Olá amigo, Uma solução legal para este problema foi dada pelo Nicolau. Pergunte a ele e seria bom que pudesse colocar a solução nesta lista. Eu discordo do Tengan com relação à solução oficial. Esta apesar de artificiosa é bonita.Você já analisou-a??? Um abraço PONCE peterdirichlet@zipmail.com.br wrote: > Ola turma da Lista!!!!!!Alguem sabe como eu posso falar com os professores > Elon Lages Lima e Carlos Gustavo Tamm de Araujo Moreira(o Tengan me disse > que ele obteve uma soluçao bem legal,mas na porrada,do problema 6 da IMO > 2001,EUA.Ai eu queria que ele mostrasse)? > ATEEEEEEEEEEEEEEEEEEEEEE!!!!!!!!!!!!!!Ploft!Peterdirichlet. > > TRANSIRE SVVM PECTVS MVNDOQUE POTIRE > CONGREGATI EX TOTO ORBE MATHEMATICI OB SCRIPTA INSIGNIA TRIBVERE > Medalha Fields(John Charles Fields) > > ------------------------------------------ > Use o melhor sistema de busca da Internet > Radar UOL - http://www.radaruol.com.br > > ========================================================================= > Instruções para entrar na lista, sair da lista e usar a lista em > http://www.mat.puc-rio.br/~nicolau/olimp/obm-l.html > O administrador desta lista é > ========================================================================= ========================================================================= Instruções para entrar na lista, sair da lista e usar a lista em http://www.mat.puc-rio.br/~nicolau/olimp/obm-l.html O administrador desta lista é ========================================================================= From owner-obm-l@sucuri.mat.puc-rio.br Thu Jun 6 00:49:21 2002 Return-Path: Received: (from majordom@localhost) by sucuri.mat.puc-rio.br (8.9.3/8.9.3) id AAA25288 for obm-l-list; Thu, 6 Jun 2002 00:47:57 -0300 Received: from ppp1.colband.com.br (pasteur.colband.com.br [200.245.157.15]) by sucuri.mat.puc-rio.br (8.9.3/8.9.3) with ESMTP id AAA25284 for ; Thu, 6 Jun 2002 00:47:56 -0300 Received: from pasteur.colband.com.br (pasteur.colband.com.br [10.1.1.1]) by ppp1.colband.com.br (8.9.2/8.9.2) with ESMTP id AAA32938 for ; Thu, 6 Jun 2002 00:36:18 -0200 Received: (from uucp@localhost) by pasteur.colband.com.br (8.9.3/8.9.3/Debian 8.9.3-21) id AAA01148 for ; Thu, 6 Jun 2002 00:32:01 -0300 X-Authentication-Warning: pasteur.colband.com.br: uucp set sender to using -f Received: from UNKNOWN(200.245.157.42), claiming to be "c2e3u2" via SMTP by pasteur, id smtpdPpEW9W; Thu Jun 6 00:31:13 2002 Message-ID: <005f01c20c42$90e38580$2a9df5c8@c2e3u2> From: "David Daniel Turchick" To: Subject: Re: [obm-l] pai e filho Date: Wed, 5 Jun 2002 00:38:30 -0300 MIME-Version: 1.0 Content-Type: text/plain; charset="iso-8859-1" Content-Transfer-Encoding: 8bit X-Priority: 3 X-MSMail-Priority: Normal X-Mailer: Microsoft Outlook Express 4.72.3110.5 X-MimeOLE: Produced By Microsoft MimeOLE V4.72.3110.3 Sender: owner-obm-l@sucuri.mat.puc-rio.br Precedence: bulk Reply-To: obm-l@mat.puc-rio.br >em um jogo de fichas,o pai sai com 100 fichas, assim >como seu filho, quando o pai perde o filho ganha 6 >fichas do pai, e quando o filho perde o pai ganha 4 >fichas do pai. do filho, neh? Após 20 jogos o filho tem 3 vezes as >fichas do pai. Pergunta: quntos jogos o pai ganhou. No começo, o pai tem 100 fichas. Daí, sendo x o numero de vitorias, ele ganha 4x fichas. Como 20-x eh o num. de derrotas, ele perde 6(20-x) fichas. Como, apos as 20 rodadas, o pai tem um terço das fichas do filho, e o num. total de fichas eh sempre 200, ele terá 50 fichas. Entao vc soh tem q resolver a eq. 100+4x-6(20-x)=50 p/ concluir q o pai venceu em exatamente 7 rodadas. David > > gostaria de saber a solucao, com resolucao > > > rafael > > >__________________________________________________________________________ >Quer ter seu próprio endereço na Internet? >Garanta já o seu e ainda ganhe cinco e-mails personalizados. >DomíniosBOL - http://dominios.bol.com.br > > >========================================================================= >Instruções para entrar na lista, sair da lista e usar a lista em >http://www.mat.puc-rio.br/~nicolau/olimp/obm-l.html >O administrador desta lista é >========================================================================= > ========================================================================= Instruções para entrar na lista, sair da lista e usar a lista em http://www.mat.puc-rio.br/~nicolau/olimp/obm-l.html O administrador desta lista é ========================================================================= From owner-obm-l@sucuri.mat.puc-rio.br Thu Jun 6 01:40:28 2002 Return-Path: Received: (from majordom@localhost) by sucuri.mat.puc-rio.br (8.9.3/8.9.3) id BAA26020 for obm-l-list; Thu, 6 Jun 2002 01:38:56 -0300 Received: from hotmail.com (f40.pav1.hotmail.com [64.4.31.40]) by sucuri.mat.puc-rio.br (8.9.3/8.9.3) with ESMTP id BAA26016 for ; Thu, 6 Jun 2002 01:38:54 -0300 Received: from mail pickup service by hotmail.com with Microsoft SMTPSVC; Wed, 5 Jun 2002 21:27:45 -0700 Received: from 200.151.8.8 by pv1fd.pav1.hotmail.msn.com with HTTP; Thu, 06 Jun 2002 04:27:45 GMT X-Originating-IP: [200.151.8.8] From: "Adriano Almeida Faustino" To: obm-l@mat.puc-rio.br Subject: [obm-l] Analise Combinatoria Date: Thu, 06 Jun 2002 04:27:45 +0000 Mime-Version: 1.0 Content-Type: text/plain; charset=iso-8859-1; format=flowed Message-ID: X-OriginalArrivalTime: 06 Jun 2002 04:27:45.0745 (UTC) FILETIME=[81C3A410:01C20D12] Sender: owner-obm-l@sucuri.mat.puc-rio.br Precedence: bulk Reply-To: obm-l@mat.puc-rio.br estou com duvida nessa questao e queria que alguem me ajudasse Para uma conferencia realizada no auditorio do IME,foram reservados 7 lugares,que serao ocupados por 7 oficiais superiores.Sabendo-se que 3 sao generais,2 almirantes e 2,brigadeiros e que estes lugares estao na primeira fila,um ao lado do outro,determine de quantos modos podemos acomoda-los,sem que haja sentados juntos oficiais de uma mesma arma. []`s Adriano. _________________________________________________________________ Envie e receba emails com o Hotmail no seu dispositivo móvel: http://mobile.msn.com ========================================================================= Instruções para entrar na lista, sair da lista e usar a lista em http://www.mat.puc-rio.br/~nicolau/olimp/obm-l.html O administrador desta lista é ========================================================================= From owner-obm-l@sucuri.mat.puc-rio.br Thu Jun 6 01:50:15 2002 Return-Path: Received: (from majordom@localhost) by sucuri.mat.puc-rio.br (8.9.3/8.9.3) id BAA26144 for obm-l-list; Thu, 6 Jun 2002 01:48:55 -0300 Received: from smtp-6.ig.com.br (smtp-6.ig.com.br [200.226.132.155]) by sucuri.mat.puc-rio.br (8.9.3/8.9.3) with SMTP id BAA26140 for ; Thu, 6 Jun 2002 01:48:53 -0300 Received: (qmail 23832 invoked from network); 6 Jun 2002 04:37:33 -0000 Received: from shasta048048.ig.com.br (HELO jat) (200.151.48.48) by smtp-6.ig.com.br with SMTP; 6 Jun 2002 04:37:33 -0000 Message-ID: <002e01c20d13$df5933a0$303097c8@jat> From: "Jose Augusto" To: References: <3CFE9E57.6050600@centroin.com.br> Subject: [obm-l] sou um adepto .. notacoes realmente exoticas e caoticas .. !!!!!!! Date: Thu, 6 Jun 2002 01:37:25 -0300 MIME-Version: 1.0 Content-Type: multipart/alternative; boundary="----=_NextPart_000_0028_01C20CFA.B647A0A0" X-Priority: 3 X-MSMail-Priority: Normal X-Mailer: Microsoft Outlook Express 5.50.4133.2400 X-MIMEOLE: Produced By Microsoft MimeOLE V5.50.4133.2400 Sender: owner-obm-l@sucuri.mat.puc-rio.br Precedence: bulk Reply-To: obm-l@mat.puc-rio.br This is a multi-part message in MIME format. ------=_NextPart_000_0028_01C20CFA.B647A0A0 Content-Type: text/plain; charset="iso-8859-1" Content-Transfer-Encoding: quoted-printable ----- Original Message -----=20 From: Augusto C=E9sar Morgado=20 To: obm-l@mat.puc-rio.br=20 Sent: Wednesday, June 05, 2002 8:27 PM Subject: Re: [obm-l] integral sem fazer a conta Eu, e creio que muitos outros, quero manifestar minha admira=E7ao por = quem consegue entender alguma coisa escrita em tao exotica nota=E7ao. Morgado ozorio_loof wrote: Observe que se vc desmembrar aintegral em duas,a primeira ser=E1 = \int_{-1}^1\frac{du}{u^2 + (1-x^2)/x^2} e a outraser=E1 zero (integral = de umafun=E7=E3o =EDmpar no limite sim=E9trico), da=ED=E9 imediato o = resultado procurado.\int_{-1}^1 \frac{du}{u^2 +(1-x^2)/x^2} =3D2\int_0^1 = \frac{du}{u^2 +(1-x^2)/x^2}. ------=_NextPart_000_0028_01C20CFA.B647A0A0 Content-Type: text/html; charset="iso-8859-1" Content-Transfer-Encoding: quoted-printable
 
----- Original Message -----
From:=20 Augusto=20 C=E9sar Morgado
Sent: Wednesday, June 05, 2002 = 8:27=20 PM
Subject: Re: [obm-l] integral = sem fazer a=20 conta

Eu, e creio que muitos outros, quero manifestar minha = admira=E7ao=20 por quem consegue entender alguma coisa escrita em tao exotica=20 nota=E7ao.
Morgado

ozorio_loof wrote:
Observe que se vc desmembrar =
a
integral em duas,
a primeira ser=E1 \int_{-1}^1
\frac{du}{u^2 = + (1-x^2)/x^2} e a outra
ser=E1 zero (integral de uma
fun=E7=E3o = =EDmpar no limite sim=E9trico), da=ED
=E9 imediato o resultado = procurado.
\int_{-1}^1 \frac{du}{u^2 +
(1-x^2)/x^2} = =3D
2\int_0^1 \frac{du}{u^2 = +
(1-x^2)/x^2}.
------=_NextPart_000_0028_01C20CFA.B647A0A0-- ========================================================================= Instruções para entrar na lista, sair da lista e usar a lista em http://www.mat.puc-rio.br/~nicolau/olimp/obm-l.html O administrador desta lista é ========================================================================= From owner-obm-l@sucuri.mat.puc-rio.br Thu Jun 6 02:20:18 2002 Return-Path: Received: (from majordom@localhost) by sucuri.mat.puc-rio.br (8.9.3/8.9.3) id CAA26868 for obm-l-list; Thu, 6 Jun 2002 02:18:57 -0300 Received: from ppp1.colband.com.br (pasteur.colband.com.br [200.245.157.15]) by sucuri.mat.puc-rio.br (8.9.3/8.9.3) with ESMTP id CAA26864 for ; Thu, 6 Jun 2002 02:18:55 -0300 Received: from pasteur.colband.com.br (pasteur.colband.com.br [10.1.1.1]) by ppp1.colband.com.br (8.9.2/8.9.2) with ESMTP id CAA03458 for ; Thu, 6 Jun 2002 02:07:19 -0200 Received: (from uucp@localhost) by pasteur.colband.com.br (8.9.3/8.9.3/Debian 8.9.3-21) id CAA04820 for ; Thu, 6 Jun 2002 02:03:02 -0300 X-Authentication-Warning: pasteur.colband.com.br: uucp set sender to using -f Received: from UNKNOWN(200.245.157.27), claiming to be "c2e3u2" via SMTP by pasteur, id smtpdTvb3Aw; Thu Jun 6 02:02:09 2002 Message-ID: <000e01c20c4e$bf444520$1b9df5c8@c2e3u2> From: "David Daniel Turchick" To: Subject: [obm-l] =?iso-8859-1?Q?Re:_=5Bobm-l=5D_=E1rea_do_qudril=E1tero?= Date: Wed, 5 Jun 2002 02:06:26 -0300 MIME-Version: 1.0 Content-Type: text/plain; charset="iso-8859-1" Content-Transfer-Encoding: 8bit X-Priority: 3 X-MSMail-Priority: Normal X-Mailer: Microsoft Outlook Express 4.72.3110.5 X-MimeOLE: Produced By Microsoft MimeOLE V4.72.3110.3 Sender: owner-obm-l@sucuri.mat.puc-rio.br Precedence: bulk Reply-To: obm-l@mat.puc-rio.br Bom, como ninguem ainda respondeu esse de maneira melhor, eu digo que fiz usando coordenadas. Nao deve ter levado mais de 5 minutos, mas uma solucao sintetica eh sempre mais bonita q uma analitica, entao tb gostaria de saber se alguem fez sem "apelar" pro uso de coordenadas. Sejam DC e DA os eixos x e y do plano cartesiano, e "a" o lado do quadrado. O ponto F eh o unico incidente com as retas BD e AM, q sao representadas pelos conjuntos de pontos {(x,y): y=x} e {(x,y): y=-2x+a}; logo F=(a/3,a/3). Do mesmo jeito, E eh o pto incidente com BN e AM, i.e, suas coordenadas satisfazem y=x/2+a/2 e y=-2x+a, donde E=(a/5,3a/5). Vc conhece a formula p/ area dum triangulo, dadas as coordenadas de seus vertices? A area do triangulo ABC, onde A=(xA,yA), B=(xB,yB) e C=(xC,yC) eh igual a 1/2*modulo(det{ [xA,yA,1], [xB,yB,1], [xC,yC,1] }). Entao, como jah temos as coordenadas dos pontos B,E,F, podemos descobrir a area do triangulo BEF fazendo essa continha, onde a^2=S. Chegamos em 2S/15. Por simetria, a area do triangulo BGF eh a mesma (se vc considera "por simetria" um argumento muito vago, veja q a reflexao em torno da reta BD eh uma isometria q leva E em G). Agora eh soh somar essas duas areas. David >Olá Pessoal! > >Esta aqui também não estou vendo o caminho. Aluma >dica? > >Tem-se um quadrado ABCD de área S. Une-se os vértices >A e B ao ponto médio M do lado CD, e une-se os >vértices B e C ao ponto médio N do lado AD. O segmento >AM intercepta os segmentos BN e CN nos pontos E e F, >respectivamente, e o segmento BM intercepta o segmento >CN no ponto G. Calcule a área do quadrilátero BEFG em >função de S. > >Resposta: 4S/15 > >Se aluém puder ajudar... > >Obrigado, > >Rafael. > >===== >Rafael Werneck Cinoto > ICQ# 107011599 > rwcinoto@yahoo.com > rafael.caixa@gov.com.br > matduvidas@yahoo.com.br >http://www.rwcinoto.hpg.com.br/ > >__________________________________________________ >Do You Yahoo!? >Yahoo! - Official partner of 2002 FIFA World Cup >http://fifaworldcup.yahoo.com >========================================================================= >Instruções para entrar na lista, sair da lista e usar a lista em >http://www.mat.puc-rio.br/~nicolau/olimp/obm-l.html >O administrador desta lista é >========================================================================= > ========================================================================= Instruções para entrar na lista, sair da lista e usar a lista em http://www.mat.puc-rio.br/~nicolau/olimp/obm-l.html O administrador desta lista é ========================================================================= From owner-obm-l@sucuri.mat.puc-rio.br Thu Jun 6 03:49:24 2002 Return-Path: Received: (from majordom@localhost) by sucuri.mat.puc-rio.br (8.9.3/8.9.3) id DAA27880 for obm-l-list; Thu, 6 Jun 2002 03:47:54 -0300 Received: from pina.terra.com.br (pina.terra.com.br [200.176.3.17]) by sucuri.mat.puc-rio.br (8.9.3/8.9.3) with ESMTP id DAA27876 for ; Thu, 6 Jun 2002 03:47:52 -0300 Received: from taipe.terra.com.br (taipe.terra.com.br [200.176.3.34]) by pina.terra.com.br (Postfix) with ESMTP id 83E8452EB7 for ; Thu, 6 Jun 2002 03:36:45 -0300 (EST) Received: from xt (200-171-248-227.customer.telesp.net.br [200.171.248.227]) (authenticated user macwad) by taipe.terra.com.br (Postfix) with ESMTP id DC15F1B4089 for ; Thu, 6 Jun 2002 03:36:41 -0300 (EST) Message-ID: <00e901c20d24$880ad430$0401010a@xt> From: =?iso-8859-1?Q?Vinicius_Jos=E9_Fortuna?= To: References: <3CFE9E57.6050600@centroin.com.br> Subject: Re: [obm-l] integral sem fazer a conta Date: Thu, 6 Jun 2002 03:36:41 -0300 MIME-Version: 1.0 Content-Type: multipart/alternative; boundary="----=_NextPart_000_00DE_01C20D0B.5F48E7D0" X-Priority: 3 X-MSMail-Priority: Normal X-Mailer: Microsoft Outlook Express 6.00.2600.0000 X-MimeOLE: Produced By Microsoft MimeOLE V6.00.2600.0000 Sender: owner-obm-l@sucuri.mat.puc-rio.br Precedence: bulk Reply-To: obm-l@mat.puc-rio.br This is a multi-part message in MIME format. ------=_NextPart_000_00DE_01C20D0B.5F48E7D0 Content-Type: text/plain; charset="iso-8859-1" Content-Transfer-Encoding: quoted-printable Oi Augusto, Essa nota=E7=E3o =E9 a mesma utilizada na linguagem LaTeX para = reda=E7=E3o de textos cient=EDficos. Com o uso a gente acaba se = acostumando, mas mesmo assim nem sempre =E9 f=E1cil vizualizar = claramente a express=E3o de primeira. Perceba que termos precedidos de um '\' s=E3o macros especiais. No caso, = \int seria integral e \frac fra=E7=E3o. O que vem seguinto em {} s=E3o = par=E2metros para a macro, no caso, apenas para o \frac. Termos = antecedidos por '_' s=E3o subscritos e precedidos por '^' s=E3o = sobrescritos. Utiliza-se isso para os extremos da integral. Assim podemos ter como exemplo: P(x) =3D \sum_{i=3D0}^n a_i x^i Como a representa=E7=E3o por meio de somat=F3rio de um polin=F4mio de = grau n e coeficientes "'a' =EDndice 'i'" Espero que essa breve explica=E7=E3o facilite o entendimento de outras = f=F3rmulas nesse formato maluco que certamente vir=E3o. At=E9 mais Vinicius Fortuna ----- Original Message -----=20 From: Augusto C=E9sar Morgado=20 To: obm-l@mat.puc-rio.br=20 Sent: Wednesday, June 05, 2002 8:27 PM Subject: Re: [obm-l] integral sem fazer a conta Eu, e creio que muitos outros, quero manifestar minha admira=E7ao por = quem consegue entender alguma coisa escrita em tao exotica nota=E7ao. Morgado ozorio_loof wrote: Observe que se vc desmembrar aintegral em duas,a primeira ser=E1 = \int_{-1}^1\frac{du}{u^2 + (1-x^2)/x^2} e a outraser=E1 zero (integral = de umafun=E7=E3o =EDmpar no limite sim=E9trico), da=ED=E9 imediato o = resultado procurado.\int_{-1}^1 \frac{du}{u^2 +(1-x^2)/x^2} =3D2\int_0^1 = \frac{du}{u^2 +(1-x^2)/x^2}.[]'sLuiz. Sauda,c~oes,Algu=E9m poderia me mostrar por que\int_{-1}^1 = \frac{(1+u)du}{u^2 + (1-x^2)/x^2} =3D=20 2\int_0^1 \frac{du}{u^2 + (1-x^2)/x^2} sem fazer as contas?Observe as mudan=E7as nos limites da integral e no numerador do integrando.Ou me dizer um livro de C=E1lculo que mostra isso de maneira geral?Como sempre, \frac{A}{B} =3D A/B.[]'sLu=EDs ------=_NextPart_000_00DE_01C20D0B.5F48E7D0 Content-Type: text/html; charset="iso-8859-1" Content-Transfer-Encoding: quoted-printable
Oi Augusto,
 
Essa nota=E7=E3o =E9 a mesma utilizada = na linguagem=20 LaTeX para reda=E7=E3o de textos cient=EDficos. Com o uso a gente = acaba se=20 acostumando, mas mesmo assim nem sempre =E9 f=E1cil vizualizar = claramente a=20 express=E3o de primeira.
 
Perceba que termos precedidos de um '\' = s=E3o macros=20 especiais. No caso, \int seria integral e \frac fra=E7=E3o. O que vem = seguinto em {}=20 s=E3o par=E2metros para a macro, no caso, apenas para o \frac. Termos = antecedidos=20 por '_' s=E3o subscritos e precedidos por '^' s=E3o sobrescritos. = Utiliza-se isso=20 para os extremos da integral.
 
Assim podemos ter como = exemplo:
 
P(x) =3D \sum_{i=3D0}^n a_i = x^i
 
Como a representa=E7=E3o por meio de = somat=F3rio de um=20 polin=F4mio de grau n e coeficientes "'a' =EDndice 'i'"
 
Espero que essa breve explica=E7=E3o = facilite o=20 entendimento de outras f=F3rmulas nesse formato maluco que certamente=20 vir=E3o.
 
At=E9 mais
 
Vinicius Fortuna
----- Original Message -----
From:=20 Augusto=20 C=E9sar Morgado
Sent: Wednesday, June 05, 2002 = 8:27=20 PM
Subject: Re: [obm-l] integral = sem fazer a=20 conta

Eu, e creio que = muitos outros,=20 quero manifestar minha admira=E7ao por quem consegue entender alguma = coisa=20 escrita em tao exotica nota=E7ao.
Morgado

ozorio_loof = wrote:
Observe que se vc desmembrar =
a
integral em duas,
a primeira ser=E1 \int_{-1}^1
\frac{du}{u^2 = + (1-x^2)/x^2} e a outra
ser=E1 zero (integral de uma
fun=E7=E3o = =EDmpar no limite sim=E9trico), da=ED
=E9 imediato o resultado = procurado.
\int_{-1}^1 \frac{du}{u^2 +
(1-x^2)/x^2} = =3D
2\int_0^1 \frac{du}{u^2 = +
(1-x^2)/x^2}.

[]'s
Luiz.

Sauda,c~oes,

Algu=E9m poderia me mostrar por = que

\int_{-1}^1 \frac{(1+u)du}{u^2 +
(1-x^2)/x^2}  =3D 
2\int_0^1 \frac{du}{u^2 =
+
(1-x^2)/x^2}
sem fazer as =
contas?

Observe as mudan=E7as nos limites = da
integral
e no numerador do =
integrando.

Ou me dizer um livro de C=E1lculo = que
mostra
isso de maneira =
geral?

Como sempre, \frac{A}{B} =3D = A/B.

[]'s
Lu=EDs
------=_NextPart_000_00DE_01C20D0B.5F48E7D0-- ========================================================================= Instruções para entrar na lista, sair da lista e usar a lista em http://www.mat.puc-rio.br/~nicolau/olimp/obm-l.html O administrador desta lista é ========================================================================= From owner-obm-l@sucuri.mat.puc-rio.br Thu Jun 6 10:23:13 2002 Return-Path: Received: (from majordom@localhost) by sucuri.mat.puc-rio.br (8.9.3/8.9.3) id KAA30731 for obm-l-list; Thu, 6 Jun 2002 10:21:01 -0300 Received: from puma.unisys.com.br (puma.unisys.com.br [200.220.64.7]) by sucuri.mat.puc-rio.br (8.9.3/8.9.3) with ESMTP id KAA30727 for ; Thu, 6 Jun 2002 10:20:54 -0300 Received: from jf ([200.220.2.142]) by puma.unisys.com.br (8.12.3/8.12.3) with SMTP id g56D9WhP003453 for ; Thu, 6 Jun 2002 10:09:37 -0300 (EST) X-Spam-Filter: check_local@puma.unisys.com.br by digitalanswers.org Message-ID: <002701c20d5a$f125cfc0$4110dcc8@jf> From: "Jose Francisco Guimaraes Costa" To: References: Subject: [obm-l] Analise Combinatoria, conceitos militares Date: Thu, 6 Jun 2002 10:04:54 -0300 MIME-Version: 1.0 Content-Type: text/plain; charset="iso-8859-1" Content-Transfer-Encoding: 8bit X-Priority: 3 X-MSMail-Priority: Normal X-Mailer: Microsoft Outlook Express 5.00.2919.6600 X-MimeOLE: Produced By Microsoft MimeOLE V5.00.2919.6600 Sender: owner-obm-l@sucuri.mat.puc-rio.br Precedence: bulk Reply-To: obm-l@mat.puc-rio.br Para evitar problemas que V poderia ter se estivesse na conferência no auditório do IME, duas pequenas correções quanto à parte não matemática do problema: (1) almirantes, generais e brigadeiros são oficiais generais, e não oficiais superiores (na marinha os oficiais superiores são os capitães de corveta, fragata e mar-e-guerra, e no exército e aeronáutica os majores, tenentes coronéis e coronéis); (2) marinha, exército e aeronáutica são "forças" e não "armas" ("armas" - conceito que só se aplica ao exército - são infantaria, cavalaria e artilharia). Se é para levar a frescura um degrau acima, sendo o IME uma "OM" (Organização Militar), os oficiais generais estariam assentados de acordo com sua "antiguidade" (hierarquia): o mais "antigo" ao centro e os mais "modernos" alternadamente à direita e à esquerda dele. Para evitar esta v.... toda, sugiro trocar no enunciado do problema "conferência" por "jogo de futebol", "IME" por "Maracanã"; "auditório" por "geral", "7 oficiais" por "7 torcedores" e "generais, almirantes e brigadeiros" por "flamenguistas, fluminenses e vascaínos". Não sei como V faria para evitar uma p... briga na geral do Maracanã! JF PS: considerem isso uma pausa para recreio, tal como nos tempos de escola. ----- Original Message ----- From: "Adriano Almeida Faustino" To: Sent: Thursday, June 06, 2002 1:27 AM Subject: [obm-l] Analise Combinatoria > estou com duvida nessa questao e queria que alguem me ajudasse > > Para uma conferencia realizada no auditorio do IME,foram reservados 7 > lugares,que serao ocupados por 7 oficiais superiores.Sabendo-se que 3 sao > generais,2 almirantes e 2,brigadeiros e que estes lugares estao na primeira > fila,um ao lado do outro,determine de quantos modos podemos acomoda-los,sem > que haja sentados juntos oficiais de uma mesma arma. > []`s > Adriano. > > > > > > > > > _________________________________________________________________ > Envie e receba emails com o Hotmail no seu dispositivo móvel: > http://mobile.msn.com > > ========================================================================= > Instruções para entrar na lista, sair da lista e usar a lista em > http://www.mat.puc-rio.br/~nicolau/olimp/obm-l.html > O administrador desta lista é > ========================================================================= > ========================================================================= Instruções para entrar na lista, sair da lista e usar a lista em http://www.mat.puc-rio.br/~nicolau/olimp/obm-l.html O administrador desta lista é ========================================================================= From owner-obm-l@sucuri.mat.puc-rio.br Thu Jun 6 15:29:07 2002 Return-Path: Received: (from majordom@localhost) by sucuri.mat.puc-rio.br (8.9.3/8.9.3) id PAA02778 for obm-l-list; Thu, 6 Jun 2002 15:25:46 -0300 Received: from fgvrj23.fgv.br (fgvrj23.fgv.br [200.20.164.23]) by sucuri.mat.puc-rio.br (8.9.3/8.9.3) with ESMTP id PAA02772 for ; Thu, 6 Jun 2002 15:25:43 -0300 Received: by FGVRJ23 with Internet Mail Service (5.5.2653.19) id ; Thu, 6 Jun 2002 15:19:06 -0300 Message-ID: <765A72978645D4118B1C0000E229806D05B6B3C2@FGVRJ23> From: Ralph Teixeira To: "'obm-l@mat.puc-rio.br'" Subject: [obm-l] =?iso-8859-1?Q?RES=3A_=5Bobm-l=5D_=E1rea_do_qudril=E1tero?= Date: Thu, 6 Jun 2002 15:19:01 -0300 MIME-Version: 1.0 X-Mailer: Internet Mail Service (5.5.2653.19) Content-Type: text/plain; charset="iso-8859-1" Content-Transfer-Encoding: 8bit X-MIME-Autoconverted: from quoted-printable to 8bit by sucuri.mat.puc-rio.br id PAA02774 Sender: owner-obm-l@sucuri.mat.puc-rio.br Precedence: bulk Reply-To: obm-l@mat.puc-rio.br Sugestao (complete-a!): A ideia eh calcular soh a area de BGF (metade de S); de fato, use que S(BGF)/S(BMD) = BG/BM . BF/BD onde S(BMD) = S/4 é conhecida. (i) Como calcular BG/BM?? Olhe o triangulo retangulo BCM, mostre que CG é altura. A partir daí, sai via relações métricas no triângulo retângulo. (ii) Como calcular BF/BD ?? Essa é um tiquinho mais difícil... Eu faria pelo Teorema de Menelaus no triangulo BMD com secante CGF... Daí você tira FB/FD e, consequentemente, BF/BD. Esse é o jeito que eu vi mais rápido... Divirta-se!! :) Abraço, Ralph -----Mensagem original----- De: Rafael WC [mailto:rwcinoto@yahoo.com] Enviada em: terça-feira, 4 de junho de 2002 20:01 Para: obm-l@mat.puc-rio.br Assunto: [obm-l] área do qudrilátero Olá Pessoal! Esta aqui também não estou vendo o caminho. Aluma dica? Tem-se um quadrado ABCD de área S. Une-se os vértices A e B ao ponto médio M do lado CD, e une-se os vértices B e C ao ponto médio N do lado AD. O segmento AM intercepta os segmentos BN e CN nos pontos E e F, respectivamente, e o segmento BM intercepta o segmento CN no ponto G. Calcule a área do quadrilátero BEFG em função de S. Resposta: 4S/15 Se aluém puder ajudar... Obrigado, Rafael. ===== Rafael Werneck Cinoto ICQ# 107011599 rwcinoto@yahoo.com rafael.caixa@gov.com.br matduvidas@yahoo.com.br http://www.rwcinoto.hpg.com.br/ __________________________________________________ Do You Yahoo!? Yahoo! - Official partner of 2002 FIFA World Cup http://fifaworldcup.yahoo.com ========================================================================= Instruções para entrar na lista, sair da lista e usar a lista em http://www.mat.puc-rio.br/~nicolau/olimp/obm-l.html O administrador desta lista é ========================================================================= ========================================================================= Instruções para entrar na lista, sair da lista e usar a lista em http://www.mat.puc-rio.br/~nicolau/olimp/obm-l.html O administrador desta lista é ========================================================================= From owner-obm-l@sucuri.mat.puc-rio.br Thu Jun 6 15:44:35 2002 Return-Path: Received: (from majordom@localhost) by sucuri.mat.puc-rio.br (8.9.3/8.9.3) id PAA03073 for obm-l-list; Thu, 6 Jun 2002 15:42:32 -0300 Received: from fgvrj23.fgv.br (fgvrj23.fgv.br [200.20.164.23]) by sucuri.mat.puc-rio.br (8.9.3/8.9.3) with ESMTP id PAA03069 for ; Thu, 6 Jun 2002 15:42:29 -0300 Received: by FGVRJ23 with Internet Mail Service (5.5.2653.19) id ; Thu, 6 Jun 2002 15:31:42 -0300 Message-ID: <765A72978645D4118B1C0000E229806D05B6B3C4@FGVRJ23> From: Ralph Teixeira To: "'obm-l@mat.puc-rio.br'" Subject: [obm-l] =?iso-8859-1?Q?RES=3A_=5Bobm-l=5D_Progress=E1lise=5FCombitm=E9?= =?iso-8859-1?Q?tica_+_Duvidas_sobre_Logica_Matematica?= Date: Thu, 6 Jun 2002 15:31:40 -0300 MIME-Version: 1.0 X-Mailer: Internet Mail Service (5.5.2653.19) Content-Type: text/plain; charset="iso-8859-1" Content-Transfer-Encoding: 8bit X-MIME-Autoconverted: from quoted-printable to 8bit by sucuri.mat.puc-rio.br id PAA03070 Sender: owner-obm-l@sucuri.mat.puc-rio.br Precedence: bulk Reply-To: obm-l@mat.puc-rio.br >1) Determinar o valor lógico da proposicao: "A expressao n^2-n+41 só >produz numeros primos". > >Como eu posso provar que para TODOS os valores de n o resultado será um >numero primo? Experimente n=41. :) ========================================================================= Instruções para entrar na lista, sair da lista e usar a lista em http://www.mat.puc-rio.br/~nicolau/olimp/obm-l.html O administrador desta lista é ========================================================================= From owner-obm-l@sucuri.mat.puc-rio.br Thu Jun 6 16:35:52 2002 Return-Path: Received: (from majordom@localhost) by sucuri.mat.puc-rio.br (8.9.3/8.9.3) id QAA04501 for obm-l-list; Thu, 6 Jun 2002 16:33:14 -0300 Received: from web14810.mail.yahoo.com (web14810.mail.yahoo.com [216.136.224.231]) by sucuri.mat.puc-rio.br (8.9.3/8.9.3) with SMTP id QAA04497 for ; Thu, 6 Jun 2002 16:33:11 -0300 Message-ID: <20020606192204.27571.qmail@web14810.mail.yahoo.com> Received: from [200.17.79.42] by web14810.mail.yahoo.com via HTTP; Thu, 06 Jun 2002 16:22:04 ART Date: Thu, 6 Jun 2002 16:22:04 -0300 (ART) From: =?iso-8859-1?q?Ricardo=20Miranda?= Subject: [obm-l] Livros de Inducao / Analitica To: obm-l@mat.puc-rio.br MIME-Version: 1.0 Content-Type: text/plain; charset=iso-8859-1 Content-Transfer-Encoding: 8bit Sender: owner-obm-l@sucuri.mat.puc-rio.br Precedence: bulk Reply-To: obm-l@mat.puc-rio.br Olá amigos. Alguem poderia me indicar um livro/site que explique o método de Inducao Matematica? Quero um livro que nao seja o "Matematica Elementar". Estou com duvidas principalmente (mas nao somente) em provar que inequacoes sao verdadeiras. Abusando, poderiam me falar um livro de Geometria Analitica (com tratamento de vetores) diferente do do Elon? ===== []s Ricardo Miranda Matematica - UFV ricardomirandabr@yahoo.com.br http://rm2.hpg.ig.com.br/ _______________________________________________________________________ Copa 2002 Yahoo! - Patrocinador oficial da Copa do Mundo da FIFA 2002 http://br.sports.yahoo.com/fifaworldcup/ ========================================================================= Instruções para entrar na lista, sair da lista e usar a lista em http://www.mat.puc-rio.br/~nicolau/olimp/obm-l.html O administrador desta lista é ========================================================================= From owner-obm-l@sucuri.mat.puc-rio.br Thu Jun 6 16:44:44 2002 Return-Path: Received: (from majordom@localhost) by sucuri.mat.puc-rio.br (8.9.3/8.9.3) id QAA04750 for obm-l-list; Thu, 6 Jun 2002 16:43:14 -0300 Received: from www.zipmail.com.br (smtp.zipmail.com.br [200.187.242.10]) by sucuri.mat.puc-rio.br (8.9.3/8.9.3) with ESMTP id QAA04742 for ; Thu, 6 Jun 2002 16:43:11 -0300 From: peterdirichlet@zipmail.com.br Received: from [200.144.43.231] by www.zipmail.com.br with HTTP; Thu, 6 Jun 2002 16:32:04 -0300 Message-ID: <3CFFAD36000002F2@www.zipmail.com.br> Date: Thu, 6 Jun 2002 16:32:04 -0300 In-Reply-To: Subject: [obm-l] =?iso-8859-1?Q?Re=3A=20=5Bobm=2Dl=5DEqua=E7ao=20de=20Pell?= To: obm-l@mat.puc-rio.br MIME-Version: 1.0 Content-Type: text/plain; charset="iso-8859-1" Content-Transfer-Encoding: 8bit X-MIME-Autoconverted: from quoted-printable to 8bit by sucuri.mat.puc-rio.br id QAA04746 Sender: owner-obm-l@sucuri.mat.puc-rio.br Precedence: bulk Reply-To: obm-l@mat.puc-rio.br Eu devo lhe avisar duas coisas: 1)Eu tenho dois provedores para meus e-mails,logo quem esta a falar com voce e o Ricardo. 2)A prova e altamente chata e longa.Se voce quiser,veja a EUREKA! numero 7.O professor Caminha escreveu um artigo com essa e outras coisas de equaçao diofantina.Se voce quiser,eu escreverei algumas linhas sobre oisso.So para começar:veja o Teorema de Kronecker. ATEEEEEEEEEEEEEEEEEEEEEEEEEEE!!!!!!!!Ploft!Peterdirichlet -- Mensagem original -- >> meu caro amigo ricardo > > > eu sei q essa equacao eh de pell, eu pedi a >demontracao > > > rafael > > >__________________________________________________________________________ >Quer ter seu próprio endereço na Internet? >Garanta já o seu e ainda ganhe cinco e-mails personalizados. >DomíniosBOL - http://dominios.bol.com.br > > >========================================================================= >Instruções para entrar na lista, sair da lista e usar a lista em >http://www.mat.puc-rio.br/~nicolau/olimp/obm-l.html >O administrador desta lista é >========================================================================= > TRANSIRE SVVM PECTVS MVNDOQUE POTIRE CONGREGATI EX TOTO ORBE MATHEMATICI OB SCRIPTA INSIGNIA TRIBVERE Medalha Fields(John Charles Fields) ------------------------------------------ Use o melhor sistema de busca da Internet Radar UOL - http://www.radaruol.com.br ========================================================================= Instruções para entrar na lista, sair da lista e usar a lista em http://www.mat.puc-rio.br/~nicolau/olimp/obm-l.html O administrador desta lista é ========================================================================= From owner-obm-l@sucuri.mat.puc-rio.br Thu Jun 6 16:52:05 2002 Return-Path: Received: (from majordom@localhost) by sucuri.mat.puc-rio.br (8.9.3/8.9.3) id QAA04884 for obm-l-list; Thu, 6 Jun 2002 16:50:01 -0300 Received: (from nicolau@localhost) by sucuri.mat.puc-rio.br (8.9.3/8.9.3) id QAA04879 for obm-l@mat.puc-rio.br; Thu, 6 Jun 2002 16:50:01 -0300 Date: Thu, 6 Jun 2002 16:50:01 -0300 From: "Nicolau C. Saldanha" To: obm-l@mat.puc-rio.br Subject: Re: [obm-l] integral sem fazer a conta Message-ID: <20020606165001.B4737@sucuri.mat.puc-rio.br> References: <3CFE9E57.6050600@centroin.com.br> Mime-Version: 1.0 Content-Type: text/plain; charset=iso-8859-1 Content-Disposition: inline Content-Transfer-Encoding: 8bit User-Agent: Mutt/1.2.5i In-Reply-To: <3CFE9E57.6050600@centroin.com.br>; from morgado@centroin.com.br on Wed, Jun 05, 2002 at 08:27:19PM -0300 Sender: owner-obm-l@sucuri.mat.puc-rio.br Precedence: bulk Reply-To: obm-l@mat.puc-rio.br On Wed, Jun 05, 2002 at 08:27:19PM -0300, Augusto César Morgado wrote: > Eu, e creio que muitos outros, quero manifestar minha admiraçao por quem > consegue entender alguma coisa escrita em tao exotica notaçao. > Morgado Esta notação chama-se TeX e não é nada exótica na comunidade matemática. Mesmo assim, acho que devemos tentar usar notações mais autoexplicativas. E principalmente devemos evitar uma notação que para alguns membros da lista é "exótica" quando há alternativas óbvias: pq, por exemplo, escrever \frac{du}{u^2 + (1-x^2)/x^2} ao invés de du/(u^2 + (1-x^2)/x^2)? A idéia aqui é que a lista seja lida e não TeXada ou processada com algum outro programa. Obrigado, []s, N. ========================================================================= Instruções para entrar na lista, sair da lista e usar a lista em http://www.mat.puc-rio.br/~nicolau/olimp/obm-l.html O administrador desta lista é ========================================================================= From owner-obm-l@sucuri.mat.puc-rio.br Thu Jun 6 17:01:54 2002 Return-Path: Received: (from majordom@localhost) by sucuri.mat.puc-rio.br (8.9.3/8.9.3) id RAA05254 for obm-l-list; Thu, 6 Jun 2002 17:00:33 -0300 Received: from hotmail.com (f150.pav1.hotmail.com [64.4.31.150]) by sucuri.mat.puc-rio.br (8.9.3/8.9.3) with ESMTP id RAA05250 for ; Thu, 6 Jun 2002 17:00:30 -0300 Received: from mail pickup service by hotmail.com with Microsoft SMTPSVC; Thu, 6 Jun 2002 12:49:23 -0700 Received: from 200.199.185.240 by pv1fd.pav1.hotmail.msn.com with HTTP; Thu, 06 Jun 2002 19:49:23 GMT X-Originating-IP: [200.199.185.240] From: "Adherbal Rocha Filho" To: obm-l@mat.puc-rio.br Subject: [obm-l] plana ajuda por favor urgente Date: Thu, 06 Jun 2002 19:49:23 +0000 Mime-Version: 1.0 Content-Type: text/plain; charset=iso-8859-1; format=flowed Message-ID: X-OriginalArrivalTime: 06 Jun 2002 19:49:23.0736 (UTC) FILETIME=[41EF7180:01C20D93] Sender: owner-obm-l@sucuri.mat.puc-rio.br Precedence: bulk Reply-To: obm-l@mat.puc-rio.br Olá,gostaria da ajuda de vcs nas seguintes questões: 1.Os pontos P1,P2,... estão sobre uma circunferencia e são tais que o arco q une cada ponto ao seguinte mede 35º.O menor valor de n>1 tal que Pn coincide com P1 é? 2.Para cada ponto pertencente ao interior e aos lados de um triangulo acutangulo ABC,considere a soma das suas distancias aos 3 lados do triangulo.O valor maximo desta soma é? 3.No triangulo ABC,AB=5 e BC=6.Qual a area maxima do triangulo ABC ,sabendo q o angulo C tem a maior medida possivel? Muito obrigado! té + Adherbal _________________________________________________________________ Envie e receba emails com o Hotmail no seu dispositivo móvel: http://mobile.msn.com ========================================================================= Instruções para entrar na lista, sair da lista e usar a lista em http://www.mat.puc-rio.br/~nicolau/olimp/obm-l.html O administrador desta lista é ========================================================================= From owner-obm-l@sucuri.mat.puc-rio.br Thu Jun 6 17:02:23 2002 Return-Path: Received: (from majordom@localhost) by sucuri.mat.puc-rio.br (8.9.3/8.9.3) id RAA05289 for obm-l-list; Thu, 6 Jun 2002 17:00:54 -0300 Received: (from nicolau@localhost) by sucuri.mat.puc-rio.br (8.9.3/8.9.3) id RAA05284 for obm-l@mat.puc-rio.br; Thu, 6 Jun 2002 17:00:54 -0300 Date: Thu, 6 Jun 2002 17:00:54 -0300 From: "Nicolau C. Saldanha" To: obm-l@mat.puc-rio.br Subject: [obm-l] Re: =?iso-8859-1?Q?=5Bobm-l=5D_Como_eu_posso_achar_os_professores_Elon_Lima_?= =?iso-8859-1?Q?e_Gugu=3FProblema_6=2CIMO_2001-Solu=E7ao?= Message-ID: <20020606170054.B5082@sucuri.mat.puc-rio.br> References: <3CFE525700000464@www.zipmail.com.br> <3CFEBB11.BC1BC01@terra.com.br> Mime-Version: 1.0 Content-Type: text/plain; charset=iso-8859-1 Content-Disposition: inline Content-Transfer-Encoding: 8bit User-Agent: Mutt/1.2.5i In-Reply-To: <3CFEBB11.BC1BC01@terra.com.br>; from lponce@terra.com.br on Wed, Jun 05, 2002 at 10:29:55PM -0300 Sender: owner-obm-l@sucuri.mat.puc-rio.br Precedence: bulk Reply-To: obm-l@mat.puc-rio.br On Wed, Jun 05, 2002 at 10:29:55PM -0300, Luiz Antonio Ponce Alonso wrote: > Olá amigo, > Uma solução legal para este problema foi dada pelo Nicolau. > Pergunte a ele e seria bom que pudesse colocar a solução nesta lista. > Eu discordo do Tengan com relação à solução oficial. Esta apesar de > artificiosa > é bonita.Você já analisou-a??? > Um abraço > PONCE Eu j'a mandei para a lista, deve estar no arquivo. []s, N. ========================================================================= Instruções para entrar na lista, sair da lista e usar a lista em http://www.mat.puc-rio.br/~nicolau/olimp/obm-l.html O administrador desta lista é ========================================================================= From owner-obm-l@sucuri.mat.puc-rio.br Thu Jun 6 17:37:09 2002 Return-Path: Received: (from majordom@localhost) by sucuri.mat.puc-rio.br (8.9.3/8.9.3) id RAA06006 for obm-l-list; Thu, 6 Jun 2002 17:13:11 -0300 Received: (from nicolau@localhost) by sucuri.mat.puc-rio.br (8.9.3/8.9.3) id RAA06000 for obm-l@mat.puc-rio.br; Thu, 6 Jun 2002 17:13:10 -0300 Date: Thu, 6 Jun 2002 17:13:10 -0300 From: "Nicolau C. Saldanha" To: obm-l@mat.puc-rio.br Subject: [obm-l] Re: =?iso-8859-1?Q?=5Bobm-l=5D_RES:_=5Bobm-l=5D_Progress=E1lise=5FCombitm=E9?= =?iso-8859-1?Q?tica_+_Duvidas_sobre_Logica_Matematica?= Message-ID: <20020606171310.E5082@sucuri.mat.puc-rio.br> References: <765A72978645D4118B1C0000E229806D05B6B3C4@FGVRJ23> Mime-Version: 1.0 Content-Type: text/plain; charset=iso-8859-1 Content-Disposition: inline Content-Transfer-Encoding: 8bit User-Agent: Mutt/1.2.5i In-Reply-To: <765A72978645D4118B1C0000E229806D05B6B3C4@FGVRJ23>; from RALPH@fgv.br on Thu, Jun 06, 2002 at 03:31:40PM -0300 Sender: owner-obm-l@sucuri.mat.puc-rio.br Precedence: bulk Reply-To: obm-l@mat.puc-rio.br On Thu, Jun 06, 2002 at 03:31:40PM -0300, Ralph Teixeira wrote: > >1) Determinar o valor lógico da proposicao: "A expressao n^2-n+41 só > >produz numeros primos". > > > >Como eu posso provar que para TODOS os valores de n o resultado será um > >numero primo? > > > Experimente n=41. :) Btw, nenhum polin^omio n~ao constante P que assume valores inteiros nos inteiros tem a propriedade de P(n) ser primo para todo n. A prova n~ao 'e dif'icil, fica como exerc'icio. []s, N. ========================================================================= Instruções para entrar na lista, sair da lista e usar a lista em http://www.mat.puc-rio.br/~nicolau/olimp/obm-l.html O administrador desta lista é ========================================================================= From owner-obm-l@sucuri.mat.puc-rio.br Thu Jun 6 17:55:43 2002 Return-Path: Received: (from majordom@localhost) by sucuri.mat.puc-rio.br (8.9.3/8.9.3) id RAA06579 for obm-l-list; Thu, 6 Jun 2002 17:57:31 -0300 Received: from imo-m02.mx.aol.com (imo-m02.mx.aol.com [64.12.136.5]) by sucuri.mat.puc-rio.br (8.9.3/8.9.3) with ESMTP id RAA06575 for ; Thu, 6 Jun 2002 17:57:29 -0300 From: Korshinoi@aol.com Received: from Korshinoi@aol.com by imo-m02.mx.aol.com (mail_out_v32.5.) id z.c1.21d44e47 (4186) for ; Thu, 6 Jun 2002 16:45:06 -0400 (EDT) Message-ID: Date: Thu, 6 Jun 2002 16:45:06 EDT Subject: [obm-l] =?ISO-8859-1?Q?Combina=E7=F5es=20com=20repeti=E7=F5es.?= To: obm-l@mat.puc-rio.br MIME-Version: 1.0 Content-Type: multipart/alternative; boundary="part1_c1.21d44e47.2a3123d2_boundary" X-Mailer: AOL 7.0 for Windows BR sub 10501 Sender: owner-obm-l@sucuri.mat.puc-rio.br Precedence: bulk Reply-To: obm-l@mat.puc-rio.br --part1_c1.21d44e47.2a3123d2_boundary Content-Type: text/plain; charset="ISO-8859-1" Content-Transfer-Encoding: quoted-printable Gostaria de saber se combina=E7=F5es com repeti=E7=F5es s=E3o exploradas no=20= vestibular.=20 Gostaria de saber onde encontro algo sobre isso, haja vista, que nos=20 principais livros que abordam an=E1lise combinat=F3ria no ensino m=E9dio, ou= mesmo=20 em livros intermediarios, nunca vi men=E7=E3o sobre o assunto....aproveito p= ra=20 mandar o problema abaixo ,onde tenho d=FAvidas sobre o que usar. Uma sorveteria tem sorvetes de 10 sabores diferentes. De quantos modos uma=20 pessoa pode escolher seis bolas de sorvete, n=E3o necess=E1riamente de sabor= es=20 diferentes?? Agrade=E7o qualquer ajuda!! Korshin=F3i =20 --part1_c1.21d44e47.2a3123d2_boundary Content-Type: text/html; charset="ISO-8859-1" Content-Transfer-Encoding: quoted-printable Gostaria de saber se combina=E7=F5es com repeti=E7=F5e= s s=E3o exploradas no vestibular. Gostaria de saber onde encontro algo sobre= isso, haja vista, que nos principais livros que abordam an=E1lise combinat= =F3ria no ensino m=E9dio, ou mesmo em livros intermediarios, nunca vi men= =E7=E3o sobre o assunto....aproveito pra mandar o problema abaixo ,onde tenh= o d=FAvidas sobre o que usar.
Uma sorveteria tem sorvetes de 10 sabores diferentes. De quantos modos uma p= essoa pode escolher seis bolas de sorvete, n=E3o necess=E1riamente de sabore= s diferentes??
             Agr= ade=E7o qualquer ajuda!!
            &nbs= p;            Korshin= =F3i
  
--part1_c1.21d44e47.2a3123d2_boundary-- ========================================================================= Instruções para entrar na lista, sair da lista e usar a lista em http://www.mat.puc-rio.br/~nicolau/olimp/obm-l.html O administrador desta lista é ========================================================================= From owner-obm-l@sucuri.mat.puc-rio.br Thu Jun 6 18:27:11 2002 Return-Path: Received: (from majordom@localhost) by sucuri.mat.puc-rio.br (8.9.3/8.9.3) id SAA07867 for obm-l-list; Thu, 6 Jun 2002 18:25:51 -0300 Received: from trex.centroin.com.br (trex.centroin.com.br [200.225.63.134]) by sucuri.mat.puc-rio.br (8.9.3/8.9.3) with ESMTP id SAA07863 for ; Thu, 6 Jun 2002 18:25:48 -0300 Received: from trex.centroin.com.br (localhost [127.0.0.1]) by trex.centroin.com.br (8.12.1/8.12.1) with ESMTP id g56LPelU010856 for ; Thu, 6 Jun 2002 18:25:40 -0300 (EST) Received: by trex.centroin.com.br (8.12.1/8.12.1/Submit) id g56LPdOW010853; Thu, 6 Jun 2002 18:25:39 -0300 (EST) Message-Id: <200206062125.g56LPdOW010853@trex.centroin.com.br> Received: from 200.225.57.107 by trex.centroin.com.br (CIPWM versao 1.4C1) with HTTPS for ; Thu, 6 Jun 2002 18:25:39 -0300 (EST) Date: Thu, 6 Jun 2002 18:25:39 -0300 (EST) From: Augusto Cesar de Oliveira Morgado To: obm-l@mat.puc-rio.br Subject: Re: [obm-l] Livros de Inducao / Analitica MIME-Version: 1.0 X-Mailer: CentroIn Internet Provider WebMail v. 1.4C1 (http://www.centroin.com.br/) Content-Type: text/plain; charset="iso-8859-1" Content-Transfer-Encoding: 8bit X-MIME-Autoconverted: from quoted-printable to 8bit by sucuri.mat.puc-rio.br id SAA07864 Sender: owner-obm-l@sucuri.mat.puc-rio.br Precedence: bulk Reply-To: obm-l@mat.puc-rio.br O Luis Lopes escreveu um livro otimo chamado Manual de Induçao Matemática. Morgado Em Thu, 6 Jun 2002 16:22:04 -0300 (ART), Ricardo Miranda disse: > Olá amigos. > > Alguem poderia me indicar um livro/site que explique o método de Inducao > Matematica? Quero um livro que nao seja o "Matematica Elementar". Estou > com duvidas principalmente (mas nao somente) em provar que inequacoes sao > verdadeiras. > > Abusando, poderiam me falar um livro de Geometria Analitica (com > tratamento de vetores) diferente do do Elon? > > > ===== > []s > Ricardo Miranda > Matematica - UFV > ricardomirandabr@yahoo.com.br > http://rm2.hpg.ig.com.br/ > > _______________________________________________________________________ > Copa 2002 > Yahoo! - Patrocinador oficial da Copa do Mundo da FIFA 2002 > http://br.sports.yahoo.com/fifaworldcup/ > ========================================================================= > Instruções para entrar na lista, sair da lista e usar a lista em > http://www.mat.puc-rio.br/~nicolau/olimp/obm-l.html > O administrador desta lista é > ========================================================================= > > ========================================================================= Instruções para entrar na lista, sair da lista e usar a lista em http://www.mat.puc-rio.br/~nicolau/olimp/obm-l.html O administrador desta lista é ========================================================================= From owner-obm-l@sucuri.mat.puc-rio.br Thu Jun 6 18:33:10 2002 Return-Path: Received: (from majordom@localhost) by sucuri.mat.puc-rio.br (8.9.3/8.9.3) id SAA08092 for obm-l-list; Thu, 6 Jun 2002 18:31:42 -0300 Received: from trex.centroin.com.br (trex.centroin.com.br [200.225.63.134]) by sucuri.mat.puc-rio.br (8.9.3/8.9.3) with ESMTP id SAA08088 for ; Thu, 6 Jun 2002 18:31:40 -0300 Received: from trex.centroin.com.br (localhost [127.0.0.1]) by trex.centroin.com.br (8.12.1/8.12.1) with ESMTP id g56LVVlU013780 for ; Thu, 6 Jun 2002 18:31:31 -0300 (EST) Received: by trex.centroin.com.br (8.12.1/8.12.1/Submit) id g56LVV2s013779; Thu, 6 Jun 2002 18:31:31 -0300 (EST) Message-Id: <200206062131.g56LVV2s013779@trex.centroin.com.br> Received: from 200.225.57.107 by trex.centroin.com.br (CIPWM versao 1.4C1) with HTTPS for ; Thu, 6 Jun 2002 18:31:31 -0300 (EST) Date: Thu, 6 Jun 2002 18:31:31 -0300 (EST) From: Augusto Cesar de Oliveira Morgado To: obm-l@mat.puc-rio.br Subject: =?iso-8859-1?q?Re: [obm-l] Combina=E7=F5es com repeti=E7=F5es.?= MIME-Version: 1.0 X-Mailer: CentroIn Internet Provider WebMail v. 1.4C1 (http://www.centroin.com.br/) Content-Type: text/plain; charset="iso-8859-1" Content-Transfer-Encoding: 8bit X-MIME-Autoconverted: from quoted-printable to 8bit by sucuri.mat.puc-rio.br id SAA08089 Sender: owner-obm-l@sucuri.mat.puc-rio.br Precedence: bulk Reply-To: obm-l@mat.puc-rio.br CR(10,6)=C(10+6-1,6)= C(15,6) Estah no Analise Combinatoria e Probabilidade da SBM Em Thu, 6 Jun 2002 16:45:06 EDT, Korshinoi@aol.com disse: > Gostaria de saber se combinações com repetições são exploradas no vestibular. > Gostaria de saber onde encontro algo sobre isso, haja vista, que nos > principais livros que abordam análise combinatória no ensino médio, ou mesmo > em livros intermediarios, nunca vi menção sobre o assunto....aproveito pra > mandar o problema abaixo ,onde tenho dúvidas sobre o que usar. > Uma sorveteria tem sorvetes de 10 sabores diferentes. De quantos modos uma > pessoa pode escolher seis bolas de sorvete, não necessáriamente de sabores > diferentes?? > Agradeço qualquer ajuda!! > Korshinói > ========================================================================= Instruções para entrar na lista, sair da lista e usar a lista em http://www.mat.puc-rio.br/~nicolau/olimp/obm-l.html O administrador desta lista é ========================================================================= From owner-obm-l@sucuri.mat.puc-rio.br Thu Jun 6 19:31:40 2002 Return-Path: Received: (from majordom@localhost) by sucuri.mat.puc-rio.br (8.9.3/8.9.3) id TAA09981 for obm-l-list; Thu, 6 Jun 2002 19:30:12 -0300 Received: from sr1.terra.com.br (sr1.terra.com.br [200.176.3.16]) by sucuri.mat.puc-rio.br (8.9.3/8.9.3) with ESMTP id TAA09977 for ; Thu, 6 Jun 2002 19:30:10 -0300 Received: from pavuna.terra.com.br (pavuna.terra.com.br [200.176.3.41]) by sr1.terra.com.br (Postfix) with ESMTP id 71DEB6EDE0 for ; Thu, 6 Jun 2002 19:29:40 -0300 (EST) Received: from stabel (dl-nas3-poa-C89A067B.p001.terra.com.br [200.154.6.123]) (authenticated user dudasta) by pavuna.terra.com.br (Postfix) with ESMTP id 5C7EF680B8 for ; Thu, 6 Jun 2002 19:29:38 -0300 (EST) Message-ID: <003f01c20da9$a169e120$7b069ac8@stabel> From: "Eduardo Casagrande Stabel" To: References: Subject: [obm-l] =?iso-8859-1?Q?Re:_=5Bobm-l=5D_Combina=E7=F5es_com_repeti=E7=F5es.?= Date: Thu, 6 Jun 2002 19:28:17 -0300 MIME-Version: 1.0 Content-Type: text/plain; charset="iso-8859-1" Content-Transfer-Encoding: 8bit X-Priority: 3 X-MSMail-Priority: Normal X-Mailer: Microsoft Outlook Express 6.00.2600.0000 X-MimeOLE: Produced By Microsoft MimeOLE V6.00.2600.0000 Sender: owner-obm-l@sucuri.mat.puc-rio.br Precedence: bulk Reply-To: obm-l@mat.puc-rio.br Caro Korshinói, No segundo grau, me ensinaram a dissociar permutação, arranjo e combinação usando uma palavreado de ordem e natureza que em vez de me ajudar só fez uma confusão dos assuntos na minha cabeça. Existe uma fórmula que engloba todas as outras e que considero muito mais simples e esclarecedora. Suponha que temos elementos de vários tipos (possivelmente mais de um elemento de cada tipo). Para simplificar (com uma notação geral) vou chamar os tipos de: tipo 1, tipo 2, tipo 3, ..., tipo K. Vou dizer que temos a_1 elementos do tipo correspondente a 1, a_2 elementos do tipo 2, ..., a_K elementos do tipo K. De forma que dois elementos do mesmo tipo (por exemplo do tipo 3) não podem ser diferenciados um do outro, eles são idênticos. Agora considere uma caixa com todos esses elementos. A caixa tem, portanto, a_1+a_2+...+a_K = N elementos. Faço a seguinte pergunta: - De quantos modos posso retirar dessa caixa todos os N elementos (um por um) de forma que, como disse, dois elementos do mesmo tipo são indistingüiveis? Se você souber responder a essa pergunta, saberá responder a qualquer pergunta sobre combinações, arranjos e permutações. Mostrarei como no final. Primeiro vamos responder a essa pergunta. A idéia que tenho em mente é a seguinte. Inicialmente considere todos os N elementos (lembre-se que N = a_1+a_2+...+a_k) distintos entre si, e numere-os por 1, 2, ...., N. Pergunto: de quantos modos podemos retirar um por um os elementos da caixa? Bem, o primeiro elemento pode ser qualquer um dos N. O segundo pode ser qualquer um dos N-1 restantes (são todos distintos), portanto temos N*(N-1) modos pelo princípio fundamental da contagem. Seguindo assim, concluimos que temos N*(N-1)*(N-2)*...*2*1 = N! modos de retirar todos esse elementos da caixa. A idéia agora, é tranformar os a_1 primeiros elementos numerados (1, 2, ..., a_1) da caixa no tal tipo 1. Ou seja, façamos todos eles iguais. E fazemos a pergunta: de quantos modos podemos retirar os elementos um por um da caixa? Em cada um dos N! modos anteriores, temos os elementos 1, 2, ..., a_k presentes. Como identificamos esses elementos, cada permutação é identificada com muitas outras, para ser preciso, com exatamente a_1! outras. Por que? Suponhamos que em uma das N! permutações os elementos 1, 2,..., a_1 ocupem determinadas posições fixas. Nós podemos alternar esses elementos entre si e ainda teremos a mesma permutação (isso depois de identificar os elementos), e podemos fazer isso de a_1*(a_1-1)*(a_1-2)*...*2*1 = a_1 maneiras, pois em cada uma das posições fixas, em ordem, vamos dispondo os elementos 1, 2, ..., a_1. Portanto separamos as N! permutações anteriores em grupos de a_1! permutações e as identificamos, logo restam N! / a_1! permutações. Faça o mesmo raciocínio para a_2, identificando os elementos a_1+1, a_1+2, ..., a_1+a_2. E restará (N! / a_1!) / a_2! = N! / (a_1!*a_2!). Prossiga esse raciocínio até o tipo K. Restará N! / (a_1! * a_2! * a_3! *...* a_k!) ou, o que dá no mesmo, (a_1 + a_2 +...+ a_k)! / (a_1! * a_2!* a_3! *...* a_k!). Tentarei esclarecer com um exemplo o processo acima. Imagine que tenho 3 letras a, 2 letras b, e 5 letras c. E quero formar palavras de 10 letras utilizando todas elas. A idéia é começar perguntando. Se as dez letras forem diferentes (0, 1, ..., 9), quantos serão as palavras? Um total de 10!. Se idenfiticarmos as 3 primeiras letras (0, 1, 2) como sendo a letra a, quantas serão as pelavras? 10! / 3!. Se identificarmos as 2 letras seguinte (3, 4) como sendo a letra b, quantas palavras poderemos formar? 10! / (3! * 2!). Finalmente, se identificarmos as 5 últimas letras (5, 6, 7, 8, 9) como sendo a letra c, quantas palavras poderemos formar? 10! / (3! * 2! * 5!). E essas são as palavras com 3 a's, 2 b's e 5 c's. COMBINAÇÕES Quantos são os subconjuntos de P elementos de um conjunto com N elementos? Enumeramos os N elementos: 1, 2, ..., P, P+1, ..., N. De quantos modos podemos permutá-los, ou seja, tirá-los um por um de uma caixa? Exatamente N!. Se identificarmos os P primeiros (1, 2, ..., P) por a, restará N! / P! maneiras. Se identificarmos os N-P últimos (P+1, ..., N) por b, restará N! (P! * (N-P)!). Para cada palavra (aabbaba...ba) corresponde o suconjunto dos elementos cujos números são as posições de a nessa palávra. E para cada subconjunto de P elementos dos N números, corresponde uma palavra (abaa...ba) onde os a's estão na posicão dos elementos selecionados e os b's dos elementos deixados de lado. ARRANJOS Quantas são os conjuntos ordenados de P elementos de um conjunto com N elementos? Enumeramos os N elementos: 1, 2, ..., P, P+1, ..., N. De quantos modos podemos permutá-los, ou seja, tirá-los um por um de uma caixa? Exatamente N!. Identificamos os N-P últimos por b, restará N! / (N-P)! permutações. Para cada palavra (1bbPb...23b) corresponde uma subconjunto de P elementos onde o primeiro é a posição onde se encontra o 1, o segundo corresponde à posição onde está o 2, e assim por diante. A cada suconjunto de P elementos ordenados, corresponde uma palavra do mesmo modo. PERMUTAÇÕES Esse termo é que engloba todos os outros. Espero ter esclarecido muito pontos. O assunto combinatória certamente não termina aqui. Existem muitas outras formas de permutar elementos, em círculos, de forma caótica, ... Mas o básico, que é cobrado - acredito - nos vestibulares de todo o país está aí. Um grande abraço a todos! Eduardo Casagrande Stabel. Porto Alegre, RS. >From: Korshinoi@aol.com > >Gostaria de saber se combinações com repetições são exploradas no >vestibular. Gostaria de saber onde encontro algo sobre isso, haja vista, que >nos principais livros que abordam análise combinatória no ensino médio, ou >mesmo em livros intermediarios, nunca vi menção sobre o >assunto....aproveito pra mandar o problema abaixo ,onde tenho dúvidas >sobre o que usar. >Uma sorveteria tem sorvetes de 10 sabores diferentes. De quantos modos >uma pessoa pode escolher seis bolas de sorvete, não necessáriamente de >sabores diferentes?? > Agradeço qualquer ajuda!! > Korshinói > ========================================================================= Instruções para entrar na lista, sair da lista e usar a lista em http://www.mat.puc-rio.br/~nicolau/olimp/obm-l.html O administrador desta lista é ========================================================================= From owner-obm-l@sucuri.mat.puc-rio.br Thu Jun 6 19:36:10 2002 Return-Path: Received: (from majordom@localhost) by sucuri.mat.puc-rio.br (8.9.3/8.9.3) id TAA10037 for obm-l-list; Thu, 6 Jun 2002 19:34:46 -0300 Received: from toole.uol.com.br (toole.uol.com.br [200.231.206.186]) by sucuri.mat.puc-rio.br (8.9.3/8.9.3) with ESMTP id TAA10033 for ; Thu, 6 Jun 2002 19:34:44 -0300 Received: from cabru ([200.158.70.57]) by toole.uol.com.br (8.9.1/8.9.1) with SMTP id TAA28372 for ; Thu, 6 Jun 2002 19:29:36 -0300 (BRT) Message-ID: <000e01c20dab$2bc821a0$39469ec8@cabru> From: "Bruno" To: "OBM-L" Subject: [obm-l] Desafio Date: Thu, 6 Jun 2002 19:40:33 -0300 MIME-Version: 1.0 Content-Type: multipart/alternative; boundary="----=_NextPart_000_000B_01C20D92.05E3D9C0" X-Priority: 3 X-MSMail-Priority: Normal X-Mailer: Microsoft Outlook Express 5.50.4133.2400 X-MimeOLE: Produced By Microsoft MimeOLE V5.50.4133.2400 Sender: owner-obm-l@sucuri.mat.puc-rio.br Precedence: bulk Reply-To: obm-l@mat.puc-rio.br This is a multi-part message in MIME format. ------=_NextPart_000_000B_01C20D92.05E3D9C0 Content-Type: text/plain; charset="iso-8859-1" Content-Transfer-Encoding: quoted-printable Eu n=E3o consegui fazer este exerc=EDcio do ITA e desafio todos dessa = lista: "Suponha a', a'', ....., an s=E3o n=FAmeros reais positivos, com n>2 e = que=20 a'.a''.a'''....an=3D4 Nesta situa=E7=E3o, a repeito do produto: P=3D(1+a')(1+a'').......(1+an) temos: n+3 a.)P>2 n b.)P>5 n+1 c.)P>2 n+1 d.)P>5 e.)n.d.a. ------=_NextPart_000_000B_01C20D92.05E3D9C0 Content-Type: text/html; charset="iso-8859-1" Content-Transfer-Encoding: quoted-printable
Eu n=E3o consegui fazer este exerc=EDcio do ITA e desafio todos = dessa=20 lista:
"Suponha a', a'', ....., an  s=E3o = n=FAmeros reais=20 positivos, com n>2 e que
a'.a''.a'''....an=3D4
Nesta situa=E7=E3o, a repeito do produto:
P=3D(1+a')(1+a'').......(1+an) =20 temos:
         =20 n+3
a.)P>2
          &nbs= p;            = ;   =20 n
          &nbs= p;    =20 b.)P>5
        =    =20             =    =20            =20      n+1
          &nbs= p;            = ;           =20 c.)P>2
          &nbs= p; =20 n+1
d.)P>5
          &nbs= p;    =20 e.)n.d.a.
------=_NextPart_000_000B_01C20D92.05E3D9C0-- ========================================================================= Instruções para entrar na lista, sair da lista e usar a lista em http://www.mat.puc-rio.br/~nicolau/olimp/obm-l.html O administrador desta lista é ========================================================================= From owner-obm-l@sucuri.mat.puc-rio.br Thu Jun 6 20:00:25 2002 Return-Path: Received: (from majordom@localhost) by sucuri.mat.puc-rio.br (8.9.3/8.9.3) id TAA10826 for obm-l-list; Thu, 6 Jun 2002 19:59:06 -0300 Received: from calhau.terra.com.br (calhau.terra.com.br [200.176.3.20]) by sucuri.mat.puc-rio.br (8.9.3/8.9.3) with ESMTP id TAA10822 for ; Thu, 6 Jun 2002 19:59:04 -0300 Received: from penha.terra.com.br (penha.terra.com.br [200.176.3.43]) by calhau.terra.com.br (Postfix) with ESMTP id 8613E4724F for ; Thu, 6 Jun 2002 22:58:34 +0000 (GMT) Received: from raphaelofizs8j (200-163-226-063-gnace7003.dsl.telebrasilia.net.br [200.163.226.63]) (authenticated user gusraan) by penha.terra.com.br (Postfix) with ESMTP id F0C0C6809E for ; Thu, 6 Jun 2002 19:58:32 -0300 (EST) Message-ID: <008301c20dad$ade3b580$01c8a8c0@raphaelofizs8j> From: =?iso-8859-1?Q?Jos=E9_Orlando_Ribeiro?= To: References: <000e01c20dab$2bc821a0$39469ec8@cabru> Subject: [obm-l] Exercicio olimpiada passada.. Date: Thu, 6 Jun 2002 19:58:30 -0300 MIME-Version: 1.0 Content-Type: multipart/alternative; boundary="----=_NextPart_000_0080_01C20D94.87F7A570" X-Priority: 3 X-MSMail-Priority: Normal X-Mailer: Microsoft Outlook Express 6.00.2600.0000 X-MimeOLE: Produced By Microsoft MimeOLE V6.00.2600.0000 Sender: owner-obm-l@sucuri.mat.puc-rio.br Precedence: bulk Reply-To: obm-l@mat.puc-rio.br This is a multi-part message in MIME format. ------=_NextPart_000_0080_01C20D94.87F7A570 Content-Type: text/plain; charset="iso-8859-1" Content-Transfer-Encoding: quoted-printable 22. Seja f uma fun=E7=E3o de Z em Z definida como f(x) =3D = x/10 se x =E9 divis=EDvel por 10 e=20 f(x) =3D x + 1 caso contr=E1rio. Se a0 =3D 2001 e an+1 =3D f(an), qual = =E9 o menor valor de n para o qual an =3D 1? 20. (B) Temos a1 =3D 2002, a2 =3D 2003, ., a9 =3D 2010, a10 =3D 201, = a11 =3D 202, ., a19 =3D 210, a20 =3D 21, a21 =3D 22, ., a29 =3D 30, a30 = =3D 3, a31 =3D 4, ., a37 =3D 10 e finalmente a38 =3D 1. N=E3o entendi , pq a38=3D 1, j=E1 que os numeros s=F3 eram divididos por = 10 quando chegassemos a 10 , 20 , 30.... Alguem poderia explicar? ------=_NextPart_000_0080_01C20D94.87F7A570 Content-Type: text/html; charset="iso-8859-1" Content-Transfer-Encoding: quoted-printable

 

 

22.  Seja  f =20 uma  = fun=E7=E3o  de =20 Z  em  Z =20 definida  = como  f(x) =3D x/10  se  x  =E9 =20 divis=EDvel  = por  10 =20 e

f(x) =3D x + 1 caso contr=E1rio. Se a0 =3D 2001 e an+1 =3D = f(an), qual =E9 o = menor valor de=20 n para o qual an =3D = 1?

 

 

 

20.    = (B) Temos a1 =3D 2002, a2 =3D 2003, =85, = a9 =3D 2010, a10 =3D 201, a11 =3D 202, =85, = a19 =3D 210, a20 =3D 21, a21 =3D 22, =85, a29 =3D 30, a30 =3D 3, a31 =3D 4, =85, a37 =3D 10 e = finalmente a38 =3D=20 1.

 

 

N=E3o entendi , pq a38=3D 1, j=E1 que = os numeros s=F3=20 eram divididos por 10 quando chegassemos a 10 , 20 ,=20 30....

 

Alguem poderia=20 explicar?

 

------=_NextPart_000_0080_01C20D94.87F7A570-- ========================================================================= Instruções para entrar na lista, sair da lista e usar a lista em http://www.mat.puc-rio.br/~nicolau/olimp/obm-l.html O administrador desta lista é ========================================================================= From owner-obm-l@sucuri.mat.puc-rio.br Thu Jun 6 20:18:53 2002 Return-Path: Received: (from majordom@localhost) by sucuri.mat.puc-rio.br (8.9.3/8.9.3) id UAA11528 for obm-l-list; Thu, 6 Jun 2002 20:17:34 -0300 Received: from toole.uol.com.br (toole.uol.com.br [200.231.206.186]) by sucuri.mat.puc-rio.br (8.9.3/8.9.3) with ESMTP id UAA11524 for ; Thu, 6 Jun 2002 20:17:32 -0300 Received: from cabru ([200.158.70.57]) by toole.uol.com.br (8.9.1/8.9.1) with SMTP id UAA09590 for ; Thu, 6 Jun 2002 20:12:25 -0300 (BRT) Message-ID: <007901c20db1$236095a0$39469ec8@cabru> From: "Bruno" To: References: Subject: Re: [obm-l] pai e filho Date: Thu, 6 Jun 2002 20:23:16 -0300 MIME-Version: 1.0 Content-Type: text/plain; charset="iso-8859-1" Content-Transfer-Encoding: 8bit X-Priority: 3 X-MSMail-Priority: Normal X-Mailer: Microsoft Outlook Express 5.50.4133.2400 X-MimeOLE: Produced By Microsoft MimeOLE V5.50.4133.2400 Sender: owner-obm-l@sucuri.mat.puc-rio.br Precedence: bulk Reply-To: obm-l@mat.puc-rio.br Eu acredito que essa resolução é boa, mas provavelmente tem outras(sem usar muitas letras) que eu ainda não consegui pensar: Sendo: n=número de jogos p=fichas do pai f=fichas do filho v=vitórias do pai v'=vitórias do filho as fichas do pai é dada pela equação: p=100 +4v -6v' as fichas do filho é dada pela equação: f=100 -4v +6v' e o número de jogos é dado por: n= v + v' como após vinte jogos: n=20 e como n= v + v' então v' = 20 - v e f=3p portanto: 100-4v+6v'=3(100+4v-6v') 16v - 24v' = 200 dividindo tudo por 8 temos: 2v - 3v'=25 ecomo v'= 20-v temos v=17 Resposta: o pai venceu 17 jogos. ----- Original Message ----- From: "aleixocarvalho" To: Sent: Wednesday, June 05, 2002 10:09 PM Subject: [obm-l] pai e filho > em um jogo de fichas,o pai sai com 100 fichas, assim > como seu filho, quando o pai perde o filho ganha 6 > fichas do pai, e quando o filho perde o pai ganha 4 > fichas do pai.Após 20 jogos o filho tem 3 vezes as > fichas do pai. Pergunta: quntos jogos o pai ganhou. > > > gostaria de saber a solucao, com resolucao > > > rafael > > > __________________________________________________________________________ > Quer ter seu próprio endereço na Internet? > Garanta já o seu e ainda ganhe cinco e-mails personalizados. > DomíniosBOL - http://dominios.bol.com.br > > > ========================================================================= > Instruções para entrar na lista, sair da lista e usar a lista em > http://www.mat.puc-rio.br/~nicolau/olimp/obm-l.html > O administrador desta lista é > ========================================================================= > ========================================================================= Instruções para entrar na lista, sair da lista e usar a lista em http://www.mat.puc-rio.br/~nicolau/olimp/obm-l.html O administrador desta lista é ========================================================================= From owner-obm-l@sucuri.mat.puc-rio.br Thu Jun 6 22:48:13 2002 Return-Path: Received: (from majordom@localhost) by sucuri.mat.puc-rio.br (8.9.3/8.9.3) id WAA13654 for obm-l-list; Thu, 6 Jun 2002 22:45:20 -0300 Received: from puma.unisys.com.br (puma.unisys.com.br [200.220.64.7]) by sucuri.mat.puc-rio.br (8.9.3/8.9.3) with ESMTP id WAA13650 for ; Thu, 6 Jun 2002 22:45:18 -0300 Received: from jf (riopm18p135.unisys.com.br [200.220.16.135] (may be forged)) by puma.unisys.com.br (8.12.3/8.12.3) with SMTP id g571ilwQ017733 for ; Thu, 6 Jun 2002 22:44:48 -0300 (EST) X-Spam-Filter: check_local@puma.unisys.com.br by digitalanswers.org Message-ID: <006601c20dc4$71e3c9a0$8710dcc8@jf> From: "Jose Francisco Guimaraes Costa" To: References: <765A72978645D4118B1C0000E229806D05B6B3C4@FGVRJ23> <20020606171310.E5082@sucuri.mat.puc-rio.br> Subject: [obm-l] =?iso-8859-1?Q?Progress=E1lise=5FCombitm=E9tica_+_Duvidas_sobre_Logica_Ma?= =?iso-8859-1?Q?tematica?= Date: Thu, 6 Jun 2002 22:31:18 -0300 MIME-Version: 1.0 Content-Type: text/plain; charset="iso-8859-1" Content-Transfer-Encoding: 8bit X-Priority: 3 X-MSMail-Priority: Normal X-Mailer: Microsoft Outlook Express 5.00.2919.6600 X-MimeOLE: Produced By Microsoft MimeOLE V5.00.2919.6600 Sender: owner-obm-l@sucuri.mat.puc-rio.br Precedence: bulk Reply-To: obm-l@mat.puc-rio.br Com o pedido de desculpas antecipado pela possibilidade de ser uma pergunta boba, o que quer dizer "... que assume valores inteiros nos inteiros ...", no teorema mencionado pelo N abaixo? JF ----- Original Message ----- From: "Nicolau C. Saldanha" To: Sent: Thursday, June 06, 2002 5:13 PM Subject: [obm-l] Re: [obm-l] RES: [obm-l] Progressálise_Combitmética + Duvidas sobre Logica Matematica > On Thu, Jun 06, 2002 at 03:31:40PM -0300, Ralph Teixeira wrote: > > >1) Determinar o valor lógico da proposicao: "A expressao n^2-n+41 só > > >produz numeros primos". > > > > > >Como eu posso provar que para TODOS os valores de n o resultado será um > > >numero primo? > > > > > > Experimente n=41. :) > > Btw, nenhum polin^omio n~ao constante P que assume valores inteiros > nos inteiros tem a propriedade de P(n) ser primo para todo n. > A prova n~ao 'e dif'icil, fica como exerc'icio. > > []s, N. > ========================================================================= > Instruções para entrar na lista, sair da lista e usar a lista em > http://www.mat.puc-rio.br/~nicolau/olimp/obm-l.html > O administrador desta lista é > ========================================================================= > ========================================================================= Instruções para entrar na lista, sair da lista e usar a lista em http://www.mat.puc-rio.br/~nicolau/olimp/obm-l.html O administrador desta lista é ========================================================================= From owner-obm-l@sucuri.mat.puc-rio.br Thu Jun 6 22:48:13 2002 Return-Path: Received: (from majordom@localhost) by sucuri.mat.puc-rio.br (8.9.3/8.9.3) id WAA13666 for obm-l-list; Thu, 6 Jun 2002 22:45:26 -0300 Received: from puma.unisys.com.br (puma.unisys.com.br [200.220.64.7]) by sucuri.mat.puc-rio.br (8.9.3/8.9.3) with ESMTP id WAA13661 for ; Thu, 6 Jun 2002 22:45:24 -0300 Received: from jf (riopm18p135.unisys.com.br [200.220.16.135] (may be forged)) by puma.unisys.com.br (8.12.3/8.12.3) with SMTP id g571ilwS017733 for ; Thu, 6 Jun 2002 22:44:53 -0300 (EST) X-Spam-Filter: check_local@puma.unisys.com.br by digitalanswers.org Message-ID: <006701c20dc4$753a47a0$8710dcc8@jf> From: "Jose Francisco Guimaraes Costa" To: References: <200206062125.g56LPdOW010853@trex.centroin.com.br> Subject: [obm-l] =?iso-8859-1?Q?manual_de_indu=E7=E3o_matem=E1tica?= Date: Thu, 6 Jun 2002 22:33:27 -0300 MIME-Version: 1.0 Content-Type: text/plain; charset="iso-8859-1" Content-Transfer-Encoding: 8bit X-Priority: 3 X-MSMail-Priority: Normal X-Mailer: Microsoft Outlook Express 5.00.2919.6600 X-MimeOLE: Produced By Microsoft MimeOLE V5.00.2919.6600 Sender: owner-obm-l@sucuri.mat.puc-rio.br Precedence: bulk Reply-To: obm-l@mat.puc-rio.br Qual a editora? Onde pode ser comprado? JF ----- Original Message ----- From: "Augusto Cesar de Oliveira Morgado" To: Sent: Thursday, June 06, 2002 6:25 PM Subject: Re: [obm-l] Livros de Inducao / Analitica > O Luis Lopes escreveu um livro otimo chamado Manual de Induçao Matemática. > Morgado > > > Em Thu, 6 Jun 2002 16:22:04 -0300 (ART), Ricardo Miranda disse: > > > Olá amigos. > > > > Alguem poderia me indicar um livro/site que explique o método de Inducao > > Matematica? Quero um livro que nao seja o "Matematica Elementar". Estou > > com duvidas principalmente (mas nao somente) em provar que inequacoes sao > > verdadeiras. > > > > Abusando, poderiam me falar um livro de Geometria Analitica (com > > tratamento de vetores) diferente do do Elon? > > > > > > ===== > > []s > > Ricardo Miranda > > Matematica - UFV > > ricardomirandabr@yahoo.com.br > > http://rm2.hpg.ig.com.br/ > > > > _______________________________________________________________________ > > Copa 2002 > > Yahoo! - Patrocinador oficial da Copa do Mundo da FIFA 2002 > > http://br.sports.yahoo.com/fifaworldcup/ > > ========================================================================= > > Instruções para entrar na lista, sair da lista e usar a lista em > > http://www.mat.puc-rio.br/~nicolau/olimp/obm-l.html > > O administrador desta lista é > > ========================================================================= > > > > > > ========================================================================= > Instruções para entrar na lista, sair da lista e usar a lista em > http://www.mat.puc-rio.br/~nicolau/olimp/obm-l.html > O administrador desta lista é > ========================================================================= > ========================================================================= Instruções para entrar na lista, sair da lista e usar a lista em http://www.mat.puc-rio.br/~nicolau/olimp/obm-l.html O administrador desta lista é ========================================================================= From owner-obm-l@sucuri.mat.puc-rio.br Thu Jun 6 22:48:36 2002 Return-Path: Received: (from majordom@localhost) by sucuri.mat.puc-rio.br (8.9.3/8.9.3) id WAA13684 for obm-l-list; Thu, 6 Jun 2002 22:46:04 -0300 Received: from sr1.terra.com.br (sr1.terra.com.br [200.176.3.16]) by sucuri.mat.puc-rio.br (8.9.3/8.9.3) with ESMTP id WAA13680 for ; Thu, 6 Jun 2002 22:46:02 -0300 Received: from mucuri.terra.com.br (mucuri.terra.com.br [200.176.3.39]) by sr1.terra.com.br (Postfix) with ESMTP id BD8296ECBB for ; Thu, 6 Jun 2002 22:45:32 -0300 (EST) Received: from stabel (dl-nas1-poa-C89A010A.p001.terra.com.br [200.154.1.10]) (authenticated user dudasta) by mucuri.terra.com.br (Postfix) with ESMTP id 435EEBE8DD for ; Thu, 6 Jun 2002 22:45:31 -0300 (EST) Message-ID: <003b01c20dc4$fe5e4fe0$18069ac8@stabel> From: "Eduardo Casagrande Stabel" To: References: <000e01c20dab$2bc821a0$39469ec8@cabru> Subject: Re: [obm-l] Desafio Date: Thu, 6 Jun 2002 22:45:22 -0300 MIME-Version: 1.0 Content-Type: text/plain; charset="iso-8859-1" Content-Transfer-Encoding: 8bit X-Priority: 3 X-MSMail-Priority: Normal X-Mailer: Microsoft Outlook Express 6.00.2600.0000 X-MimeOLE: Produced By Microsoft MimeOLE V6.00.2600.0000 Sender: owner-obm-l@sucuri.mat.puc-rio.br Precedence: bulk Reply-To: obm-l@mat.puc-rio.br Caro Bruno, a notação que você usou não está muito legível. Seria melhor adotar índices para os a's, por exemplo: a_1, a_2, a_3, ..., a_n. Para fazer exponenciação geralmente se usa "^", aí as alternativas seriam P>2^(n+3), P>5^n, e assim por diante. Quanto ao problema. Existe uma desigualdade, que aprendi a demonstrar por indução (e talvez você já conheça ou queira provar como exercício) que diz que se a_1, a_2, ..., a_n são números não-negativos então (1 + a_1)*(1 + a_2)*...*(1 + a_n) >= 1 + (a_1*a_2*...*a_n) com a igualdade se e so se todos os a_i's forem iguais a zero. No caso do seu problema. Temos P = (1 + a_1)*(1 + a_2)*...*(1 + a_n) > 1 + (a_1*a_2*...*a_n) = 5. Isso claramente não resolve o problema. Uma estratégia mais interessante me parece procurar pelo valor mínimo de P, após fixado o n. Fazendo a multiplicação, temos P = (1 + a_1)*(1 + a_2)*...*(1 + a_n) = 1 + [a_1+a_2+...+a_n] + [a_1a_2+a_1a_3+...+a_(n-1)a_n] + ... + [a_1a_2...a_n] No primeiro colchetes temos os n termos a's solitários. No segundo colchetes temos os produtos de pares de a's. No terceiro, o produto de trincas. E assim por diante. Vamos aplicar a desigualdade: média aritmética >= média geométrica em cada um dos colchetes. P >= 1 + n*[RAIZ_n {a_1a_2...a_n}] + n(n-1)/2*[RAIZ_n(n-1)/2 {(a_1a_2...a_n)^(n-1)}] + ... + [a_1a_2...a_n] De forma mais compacta P >= 1 + SOMATÓRIO{ k=1...n : C(n,k) * RAIZ_C(n,k) { (a_1a_2...a_n)^(C(n-1,k-1)) } } = 1 + SOMATÓRIO{ k=1...n : C(n,k) * (RAIZ_n (4^k) } = (1 + RAIZ_n(4))^n ((Revisem as contas, fiz de modo simplificado)) Basta mostrar que a igualdade ocorre se e somente se a_1=a_2=...a_n, mas isso é claro por termos usado a desigualdade média aritmética e geométrica. Portanto P >= (1 + RAIZ_n(4))^n e a igualdade pode ocorrer para cada n. Com isso fica fácil de ver que qualquer exponencial do tipo a^n (onde a>1) vai acabar superando P para algum n suficientemente grande, repare que 1 + RAIZ_n(4), a "base" da nossa exponencial se aproxima de 1 a medida que n cresce. De modo que nem a) nem b) nem c) nem d) são verdadeiras. Logo a alternativa correta é e). Um abraço! Eduardo Casagrande Stabel. Porto Alegre, RS. >From: Bruno > >Eu não consegui fazer este exercício do ITA e desafio todos dessa lista: >"Suponha a', a'', ....., an são números reais positivos, com n>2 e que >a'.a''.a'''....an=4 >Nesta situação, a repeito do produto: >P=(1+a')(1+a'').......(1+an) temos: > n+3 >a.)P>2 > n > b.)P>5 > n+1 > c.)P>2 > n+1 >d.)P>5 > e.)n.d.a. > ========================================================================= Instruções para entrar na lista, sair da lista e usar a lista em http://www.mat.puc-rio.br/~nicolau/olimp/obm-l.html O administrador desta lista é ========================================================================= From owner-obm-l@sucuri.mat.puc-rio.br Thu Jun 6 22:57:58 2002 Return-Path: Received: (from majordom@localhost) by sucuri.mat.puc-rio.br (8.9.3/8.9.3) id WAA13951 for obm-l-list; Thu, 6 Jun 2002 22:56:40 -0300 Received: from gorgo.centroin.com.br (gorgo.centroin.com.br [200.225.63.128]) by sucuri.mat.puc-rio.br (8.9.3/8.9.3) with ESMTP id WAA13937 for ; Thu, 6 Jun 2002 22:56:37 -0300 Received: from centroin.com.br (du173c.rjo.centroin.com.br [200.225.58.173]) (authenticated bits=0) by gorgo.centroin.com.br (8.12.2/8.12.1) with ESMTP id g571uako012832 for ; Thu, 6 Jun 2002 22:56:36 -0300 (BRT) Message-ID: <3D00131F.2090304@centroin.com.br> Date: Thu, 06 Jun 2002 22:57:51 -0300 From: Augusto =?ISO-8859-1?Q?C=E9sar?= Morgado User-Agent: Mozilla/5.0 (Windows; U; Win98; en-US; rv:0.9.4.1) Gecko/20020508 Netscape6/6.2.3 X-Accept-Language: en-us MIME-Version: 1.0 To: obm-l@mat.puc-rio.br Subject: Re: [obm-l] =?ISO-8859-1?Q?Progress=E1lise=5FCombitm=E9tica?= + Duvidas sobre Logica Matematica References: <765A72978645D4118B1C0000E229806D05B6B3C4@FGVRJ23> <20020606171310.E5082@sucuri.mat.puc-rio.br> <006601c20dc4$71e3c9a0$8710dcc8@jf> Content-Type: multipart/alternative; boundary="------------000702020309060608030600" Sender: owner-obm-l@sucuri.mat.puc-rio.br Precedence: bulk Reply-To: obm-l@mat.puc-rio.br --------------000702020309060608030600 Content-Type: text/plain; charset=ISO-8859-1; format=flowed Content-Transfer-Encoding: 8bit Para x inteiro, P(x) eh inteiro Jose Francisco Guimaraes Costa wrote: >Com o pedido de desculpas antecipado pela possibilidade de ser uma pergunta >boba, o que quer dizer "... que assume valores inteiros nos inteiros ...", >no teorema mencionado pelo N abaixo? > >JF > >----- Original Message ----- >From: "Nicolau C. Saldanha" >To: >Sent: Thursday, June 06, 2002 5:13 PM >Subject: [obm-l] Re: [obm-l] RES: [obm-l] Progressálise_Combitmética + >Duvidas sobre Logica Matematica > > >>On Thu, Jun 06, 2002 at 03:31:40PM -0300, Ralph Teixeira wrote: >> >>>>1) Determinar o valor lógico da proposicao: "A expressao n^2-n+41 só >>>>produz numeros primos". >>>> >>>>Como eu posso provar que para TODOS os valores de n o resultado será um >>>>numero primo? >>>> >>> >>>Experimente n=41. :) >>> >>Btw, nenhum polin^omio n~ao constante P que assume valores inteiros >>nos inteiros tem a propriedade de P(n) ser primo para todo n. >>A prova n~ao 'e dif'icil, fica como exerc'icio. >> >>[]s, N. >>========================================================================= >>Instruções para entrar na lista, sair da lista e usar a lista em >>http://www.mat.puc-rio.br/~nicolau/olimp/obm-l.html >>O administrador desta lista é >>========================================================================= >> > >========================================================================= >Instruções para entrar na lista, sair da lista e usar a lista em >http://www.mat.puc-rio.br/~nicolau/olimp/obm-l.html >O administrador desta lista é >========================================================================= > > --------------000702020309060608030600 Content-Type: text/html; charset=us-ascii Content-Transfer-Encoding: 7bit Para x inteiro, P(x) eh inteiro

Jose Francisco Guimaraes Costa wrote:
Com o pedido de desculpas antecipado pela possibilidade de ser uma pergunta
boba, o que quer dizer "... que assume valores inteiros nos inteiros ...",
no teorema mencionado pelo N abaixo?

JF

----- Original Message -----
From: "Nicolau C. Saldanha" <nicolau@sucuri.mat.puc-rio.br>
To: <obm-l@mat.puc-rio.br>
Sent: Thursday, June 06, 2002 5:13 PM
Subject: [obm-l] Re: [obm-l] RES: [obm-l] Progressálise_Combitmética +
Duvidas sobre Logica Matematica


On Thu, Jun 06, 2002 at 03:31:40PM -0300, Ralph Teixeira wrote:
1) Determinar o valor lógico da proposicao: "A expressao n^2-n+41 só
produz numeros primos".

Como eu posso provar que para TODOS os valores de n o resultado será um
numero primo?

Experimente n=41. :)
Btw, nenhum polin^omio n~ao constante P que assume valores inteiros
nos inteiros tem a propriedade de P(n) ser primo para todo n.
A prova n~ao 'e dif'icil, fica como exerc'icio.

[]s, N.
=========================================================================
Instruções para entrar na lista, sair da lista e usar a lista em
http://www.mat.puc-rio.br/~nicolau/olimp/obm-l.html
O administrador desta lista é <nicolau@mat.puc-rio.br>
=========================================================================


=========================================================================
Instruções para entrar na lista, sair da lista e usar a lista em
http://www.mat.puc-rio.br/~nicolau/olimp/obm-l.html
O administrador desta lista é <nicolau@mat.puc-rio.br>
=========================================================================



--------------000702020309060608030600-- ========================================================================= Instruções para entrar na lista, sair da lista e usar a lista em http://www.mat.puc-rio.br/~nicolau/olimp/obm-l.html O administrador desta lista é ========================================================================= From owner-obm-l@sucuri.mat.puc-rio.br Thu Jun 6 23:01:55 2002 Return-Path: Received: (from majordom@localhost) by sucuri.mat.puc-rio.br (8.9.3/8.9.3) id XAA14194 for obm-l-list; Thu, 6 Jun 2002 23:00:38 -0300 Received: from candeias.terra.com.br (candeias.terra.com.br [200.176.3.18]) by sucuri.mat.puc-rio.br (8.9.3/8.9.3) with ESMTP id XAA14190 for ; Thu, 6 Jun 2002 23:00:36 -0300 Received: from pacuiba.terra.com.br (pacuiba.terra.com.br [200.176.3.40]) by candeias.terra.com.br (Postfix) with ESMTP id 7969A43D83 for ; Thu, 6 Jun 2002 23:00:06 -0300 (EST) Received: from stabel (dl-nas1-poa-C89A010A.p001.terra.com.br [200.154.1.10]) (authenticated user dudasta) by pacuiba.terra.com.br (Postfix) with ESMTP id BFEDE8015 for ; Thu, 6 Jun 2002 23:00:04 -0300 (EST) Message-ID: <004f01c20dc7$06f70460$18069ac8@stabel> From: "Eduardo Casagrande Stabel" To: Subject: [obm-l] =?iso-8859-1?Q?Re:_=5Bobm-l=5D_Desafio_=5Bcorre=E7=E3o=5D?= Date: Thu, 6 Jun 2002 22:59:49 -0300 MIME-Version: 1.0 Content-Type: text/plain; charset="iso-8859-1" Content-Transfer-Encoding: 8bit X-Priority: 3 X-MSMail-Priority: Normal X-Mailer: Microsoft Outlook Express 6.00.2600.0000 X-MimeOLE: Produced By Microsoft MimeOLE V6.00.2600.0000 Sender: owner-obm-l@sucuri.mat.puc-rio.br Precedence: bulk Reply-To: obm-l@mat.puc-rio.br Caro Bruno e colegas, minha conclusão foi errônea, como muitas de outras mensagens minhas. O fato de P >= (1 + RAIZ_n(4))^n implica, por exemplo que: P >= 2^n, já que 1 + RAIZ_n(4) > 2 O meu erro foi achar que RAIZ_n(4) tende a zero quando n cresce, isso não é verdade. Essa seqüência tende a 1, de modo descrescente. Mesmo assim *acho* que a alternativa correta é e). Pois para n=1, podemos fazer P=5. De modo que a) não pode valer. As alternativas b) e d) certamente não valem. Não sei quanto a c). Vou pensar mais a respeito. Eduardo Casagrande Stabel. Porto Alegre, RS. From: "Eduardo Casagrande Stabel" > Caro Bruno, > > a notação que você usou não está muito legível. Seria melhor adotar índices > para os a's, por exemplo: a_1, a_2, a_3, ..., a_n. Para fazer exponenciação > geralmente se usa "^", aí as alternativas seriam P>2^(n+3), P>5^n, e assim > por diante. > > Quanto ao problema. Existe uma desigualdade, que aprendi a demonstrar por > indução (e talvez você já conheça ou queira provar como exercício) que diz > que se a_1, a_2, ..., a_n são números não-negativos então > > (1 + a_1)*(1 + a_2)*...*(1 + a_n) >= 1 + (a_1*a_2*...*a_n) > > com a igualdade se e so se todos os a_i's forem iguais a zero. > > No caso do seu problema. Temos > > P = (1 + a_1)*(1 + a_2)*...*(1 + a_n) > 1 + (a_1*a_2*...*a_n) = 5. > > Isso claramente não resolve o problema. Uma estratégia mais interessante me > parece procurar pelo valor mínimo de P, após fixado o n. > > Fazendo a multiplicação, temos > > P = (1 + a_1)*(1 + a_2)*...*(1 + a_n) = 1 + [a_1+a_2+...+a_n] + > [a_1a_2+a_1a_3+...+a_(n-1)a_n] + ... + [a_1a_2...a_n] > > No primeiro colchetes temos os n termos a's solitários. > No segundo colchetes temos os produtos de pares de a's. > No terceiro, o produto de trincas. E assim por diante. > Vamos aplicar a desigualdade: média aritmética >= média geométrica em cada > um dos colchetes. > > P >= 1 + n*[RAIZ_n {a_1a_2...a_n}] + n(n-1)/2*[RAIZ_n(n-1)/2 > {(a_1a_2...a_n)^(n-1)}] + ... + [a_1a_2...a_n] > > De forma mais compacta > > P >= 1 + SOMATÓRIO{ k=1...n : C(n,k) * RAIZ_C(n,k) { > (a_1a_2...a_n)^(C(n-1,k-1)) } } = > 1 + SOMATÓRIO{ k=1...n : C(n,k) * (RAIZ_n (4^k) } > = (1 + RAIZ_n(4))^n > > ((Revisem as contas, fiz de modo simplificado)) > > Basta mostrar que a igualdade ocorre se e somente se a_1=a_2=...a_n, mas > isso é claro por termos usado a desigualdade média aritmética e geométrica. > > Portanto P >= (1 + RAIZ_n(4))^n e a igualdade pode ocorrer para cada n. > > Com isso fica fácil de ver que qualquer exponencial do tipo a^n (onde a>1) > vai acabar superando P para algum n suficientemente grande, repare que 1 + > RAIZ_n(4), a "base" da nossa exponencial se aproxima de 1 a medida que n > cresce. De modo que nem a) nem b) nem c) nem d) são verdadeiras. Logo a > alternativa correta é e). > > Um abraço! > > Eduardo Casagrande Stabel. Porto Alegre, RS. > > > >From: Bruno > > > >Eu não consegui fazer este exercício do ITA e desafio todos dessa lista: > >"Suponha a', a'', ....., an são números reais positivos, com n>2 e que > >a'.a''.a'''....an=4 > >Nesta situação, a repeito do produto: > >P=(1+a')(1+a'').......(1+an) temos: > > n+3 > >a.)P>2 > > n > > b.)P>5 > > n+1 > > c.)P>2 > > n+1 > >d.)P>5 > > e.)n.d.a. > > > > ========================================================================= Instruções para entrar na lista, sair da lista e usar a lista em http://www.mat.puc-rio.br/~nicolau/olimp/obm-l.html O administrador desta lista é ========================================================================= From owner-obm-l@sucuri.mat.puc-rio.br Thu Jun 6 23:22:29 2002 Return-Path: Received: (from majordom@localhost) by sucuri.mat.puc-rio.br (8.9.3/8.9.3) id XAA15473 for obm-l-list; Thu, 6 Jun 2002 23:21:09 -0300 Received: from calhau.terra.com.br (calhau.terra.com.br [200.176.3.20]) by sucuri.mat.puc-rio.br (8.9.3/8.9.3) with ESMTP id XAA15469 for ; Thu, 6 Jun 2002 23:21:07 -0300 Received: from taipe.terra.com.br (taipe.terra.com.br [200.176.3.34]) by calhau.terra.com.br (Postfix) with ESMTP id 3870347188 for ; Fri, 7 Jun 2002 02:20:38 +0000 (GMT) Received: from terra.com.br (200-206-243-51.dsl.telesp.net.br [200.206.243.51]) (authenticated user lponce) by taipe.terra.com.br (Postfix) with ESMTP id E1C4D1B4085 for ; Thu, 6 Jun 2002 23:20:37 -0300 (EST) Message-ID: <3D00185D.E2826B1@terra.com.br> Date: Thu, 06 Jun 2002 23:20:13 -0300 From: Luiz Antonio Ponce Alonso X-Mailer: Mozilla 4.79 [en] (Windows NT 5.0; U) X-Accept-Language: en MIME-Version: 1.0 To: obm-l@mat.puc-rio.br Subject: [obm-l] ajuda: colegio naval Content-Type: text/plain; charset=iso-8859-1 Content-Transfer-Encoding: 8bit Sender: owner-obm-l@sucuri.mat.puc-rio.br Precedence: bulk Reply-To: obm-l@mat.puc-rio.br Caros amigos, Estou precisando de provas de matemática do colegio naval para ajudar na preparação do filho de meu amigo. Caso alguém tenha alguma ou um site em que posso adquiri-las ficaria muito grato. Desde já fico agradecido por qualquer ajuda futura Um abraço De seu amigo PONCE ========================================================================= Instruções para entrar na lista, sair da lista e usar a lista em http://www.mat.puc-rio.br/~nicolau/olimp/obm-l.html O administrador desta lista é ========================================================================= From owner-obm-l@sucuri.mat.puc-rio.br Fri Jun 7 07:02:36 2002 Return-Path: Received: (from majordom@localhost) by sucuri.mat.puc-rio.br (8.9.3/8.9.3) id HAA19435 for obm-l-list; Fri, 7 Jun 2002 07:00:37 -0300 Received: from web10108.mail.yahoo.com (web10108.mail.yahoo.com [216.136.130.58]) by sucuri.mat.puc-rio.br (8.9.3/8.9.3) with SMTP id HAA19431 for ; Fri, 7 Jun 2002 07:00:34 -0300 Message-ID: <20020607100004.84920.qmail@web10108.mail.yahoo.com> Received: from [200.199.32.71] by web10108.mail.yahoo.com via HTTP; Fri, 07 Jun 2002 03:00:04 PDT Date: Fri, 7 Jun 2002 03:00:04 -0700 (PDT) From: Rafael WC Subject: [obm-l] diferença de raízes To: obm-l@mat.puc-rio.br In-Reply-To: <765A72978645D4118B1C0000E229806D05B6B3C2@FGVRJ23> MIME-Version: 1.0 Content-Type: text/plain; charset=us-ascii Sender: owner-obm-l@sucuri.mat.puc-rio.br Precedence: bulk Reply-To: obm-l@mat.puc-rio.br Olá Pessoal! Já tem um mês que eu tento resolver esse exercício sem sucesso. Se alguém conseguir algum avanço, por favor escreva! Qual a diferença entre a maior e a menor raiz da equação: x^2 + (9x²)/(x+3)² = 27 Obrigado. Rafael. ===== Rafael Werneck Cinoto ICQ# 107011599 rwcinoto@yahoo.com rafael.caixa@gov.com.br matduvidas@yahoo.com.br http://www.rwcinoto.hpg.com.br/ __________________________________________________ Do You Yahoo!? Yahoo! - Official partner of 2002 FIFA World Cup http://fifaworldcup.yahoo.com ========================================================================= Instruções para entrar na lista, sair da lista e usar a lista em http://www.mat.puc-rio.br/~nicolau/olimp/obm-l.html O administrador desta lista é ========================================================================= From owner-obm-l@sucuri.mat.puc-rio.br Fri Jun 7 07:02:36 2002 Return-Path: Received: (from majordom@localhost) by sucuri.mat.puc-rio.br (8.9.3/8.9.3) id HAA19446 for obm-l-list; Fri, 7 Jun 2002 07:00:52 -0300 Received: from web10101.mail.yahoo.com (web10101.mail.yahoo.com [216.136.130.51]) by sucuri.mat.puc-rio.br (8.9.3/8.9.3) with SMTP id HAA19442 for ; Fri, 7 Jun 2002 07:00:49 -0300 Message-ID: <20020607100019.67538.qmail@web10101.mail.yahoo.com> Received: from [200.199.32.71] by web10101.mail.yahoo.com via HTTP; Fri, 07 Jun 2002 03:00:19 PDT Date: Fri, 7 Jun 2002 03:00:19 -0700 (PDT) From: Rafael WC Subject: [obm-l] diferença de raízes To: obm-l@mat.puc-rio.br In-Reply-To: <765A72978645D4118B1C0000E229806D05B6B3C2@FGVRJ23> MIME-Version: 1.0 Content-Type: text/plain; charset=us-ascii Sender: owner-obm-l@sucuri.mat.puc-rio.br Precedence: bulk Reply-To: obm-l@mat.puc-rio.br Olá Pessoal! Já tem um mês que eu tento resolver esse exercício sem sucesso. Se alguém conseguir algum avanço, por favor escreva! Qual a diferença entre a maior e a menor raiz da equação: x^2 + (9x²)/(x+3)² = 27 Obrigado. Rafael. ===== Rafael Werneck Cinoto ICQ# 107011599 rwcinoto@yahoo.com rafael.caixa@gov.com.br matduvidas@yahoo.com.br http://www.rwcinoto.hpg.com.br/ __________________________________________________ Do You Yahoo!? Yahoo! - Official partner of 2002 FIFA World Cup http://fifaworldcup.yahoo.com ========================================================================= Instruções para entrar na lista, sair da lista e usar a lista em http://www.mat.puc-rio.br/~nicolau/olimp/obm-l.html O administrador desta lista é ========================================================================= From owner-obm-l@sucuri.mat.puc-rio.br Fri Jun 7 07:58:13 2002 Return-Path: Received: (from majordom@localhost) by sucuri.mat.puc-rio.br (8.9.3/8.9.3) id HAA20631 for obm-l-list; Fri, 7 Jun 2002 07:56:54 -0300 Received: from hotmail.com (f19.pav1.hotmail.com [64.4.31.19]) by sucuri.mat.puc-rio.br (8.9.3/8.9.3) with ESMTP id HAA20627 for ; Fri, 7 Jun 2002 07:56:51 -0300 Received: from mail pickup service by hotmail.com with Microsoft SMTPSVC; Fri, 7 Jun 2002 03:56:21 -0700 Received: from 200.199.179.61 by pv1fd.pav1.hotmail.msn.com with HTTP; Fri, 07 Jun 2002 10:56:21 GMT X-Originating-IP: [200.199.179.61] From: "Adherbal Rocha Filho" To: obm-l@mat.puc-rio.br Subject: [obm-l] plana ajuda por favor urgente Date: Fri, 07 Jun 2002 10:56:21 +0000 Mime-Version: 1.0 Content-Type: text/plain; charset=iso-8859-1; format=flowed Message-ID: X-OriginalArrivalTime: 07 Jun 2002 10:56:21.0781 (UTC) FILETIME=[F59E2450:01C20E11] Sender: owner-obm-l@sucuri.mat.puc-rio.br Precedence: bulk Reply-To: obm-l@mat.puc-rio.br >Olá,gostaria da ajuda de vcs nas seguintes questões: >1.Os pontos P1,P2,... estão sobre uma circunferencia e são tais que o arco >q >une cada ponto ao seguinte mede 35º.O menor valor de n>1 tal que Pn >coincide >com P1 é? > >2.Para cada ponto pertencente ao interior e aos lados de um triangulo >acutangulo ABC,considere a soma das suas distancias aos 3 lados do >triangulo.O valor maximo desta soma é? > >3.No triangulo ABC,AB=5 e BC=6.Qual a area maxima do triangulo ABC ,sabendo >q o angulo C tem a maior medida possivel? > >Muito obrigado! >té + >Adherbal _________________________________________________________________ Chegou o novo MSN Explorer. Instale já. É gratuito: http://explorer.msn.com.br ========================================================================= Instruções para entrar na lista, sair da lista e usar a lista em http://www.mat.puc-rio.br/~nicolau/olimp/obm-l.html O administrador desta lista é ========================================================================= From owner-obm-l@sucuri.mat.puc-rio.br Fri Jun 7 10:31:28 2002 Return-Path: Received: (from majordom@localhost) by sucuri.mat.puc-rio.br (8.9.3/8.9.3) id KAA23238 for obm-l-list; Fri, 7 Jun 2002 10:30:30 -0300 Received: (from nicolau@localhost) by sucuri.mat.puc-rio.br (8.9.3/8.9.3) id KAA23233 for obm-l@mat.puc-rio.br; Fri, 7 Jun 2002 10:30:29 -0300 Date: Fri, 7 Jun 2002 10:30:29 -0300 From: "Nicolau C. Saldanha" To: obm-l@mat.puc-rio.br Subject: [obm-l] Re: =?iso-8859-1?Q?=5Bobm-l=5D_Progress=E1lise=5FCombitm=E9tica_+_Duvidas_so?= =?iso-8859-1?Q?bre_Logica_Matematica?= Message-ID: <20020607103029.C22934@sucuri.mat.puc-rio.br> References: <765A72978645D4118B1C0000E229806D05B6B3C4@FGVRJ23> <20020606171310.E5082@sucuri.mat.puc-rio.br> <006601c20dc4$71e3c9a0$8710dcc8@jf> Mime-Version: 1.0 Content-Type: text/plain; charset=iso-8859-1 Content-Disposition: inline Content-Transfer-Encoding: 8bit User-Agent: Mutt/1.2.5i In-Reply-To: <006601c20dc4$71e3c9a0$8710dcc8@jf>; from jfgcosta@unisys.com.br on Thu, Jun 06, 2002 at 10:31:18PM -0300 Sender: owner-obm-l@sucuri.mat.puc-rio.br Precedence: bulk Reply-To: obm-l@mat.puc-rio.br On Thu, Jun 06, 2002 at 10:31:18PM -0300, Jose Francisco Guimaraes Costa wrote: > Com o pedido de desculpas antecipado pela possibilidade de ser uma pergunta > boba, o que quer dizer "... que assume valores inteiros nos inteiros ...", > no teorema mencionado pelo N abaixo? Significa que P(n) é inteiro para todo inteiro n. Assim, o polinômio P(n) = n(n+1)/2 assume valores inteiros nos inteiros apesar de seus coeficientes não serem todos inteiros. []s, N. ========================================================================= Instruções para entrar na lista, sair da lista e usar a lista em http://www.mat.puc-rio.br/~nicolau/olimp/obm-l.html O administrador desta lista é ========================================================================= From owner-obm-l@sucuri.mat.puc-rio.br Fri Jun 7 13:02:24 2002 Return-Path: Received: (from majordom@localhost) by sucuri.mat.puc-rio.br (8.9.3/8.9.3) id NAA26302 for obm-l-list; Fri, 7 Jun 2002 13:01:31 -0300 Received: from wool.vetor.com.br (200.160.244.7.metrored.net.br [200.160.244.7] (may be forged)) by sucuri.mat.puc-rio.br (8.9.3/8.9.3) with ESMTP id NAA26298 for ; Fri, 7 Jun 2002 13:01:28 -0300 Received: (from root@localhost) by wool.vetor.com.br (8.11.4/8.11.4) id g57G0lF16507 for obm-l@mat.puc-rio.br; Fri, 7 Jun 2002 13:00:47 -0300 Received: (from root@localhost) by wool.vetor.com.br (8.11.4/8.11.4) id g57F0V402745 for obm-l@mat.puc-rio.br; Fri, 7 Jun 2002 12:00:31 -0300 Received: (from root@localhost) by wool.vetor.com.br (8.11.4/8.11.4) id g57E22m21108 for obm-l@mat.puc-rio.br; Fri, 7 Jun 2002 11:02:02 -0300 Received: from rodrigo ([200.160.244.83]) by wool.vetor.com.br (8.11.4/8.11.4) with SMTP id g57E1wJ21051 for ; Fri, 7 Jun 2002 11:01:59 -0300 Message-ID: <004001c20e2b$d1cdc020$53f4a0c8@rodrigo> From: "Rodrigo Villard Milet" To: Subject: Re: [obm-l] Desafio Date: Fri, 7 Jun 2002 11:01:21 -0300 MIME-Version: 1.0 Content-Type: text/plain; charset="iso-8859-1" Content-Transfer-Encoding: 8bit X-Priority: 3 X-MSMail-Priority: Normal X-Mailer: Microsoft Outlook Express 4.72.3110.5 X-MimeOLE: Produced By Microsoft MimeOLE V4.72.3110.3 X-Virus-Scanned: by AMaViS perl-11 Sender: owner-obm-l@sucuri.mat.puc-rio.br Precedence: bulk Reply-To: obm-l@mat.puc-rio.br Use que 1+a(i) >=2sqrt[a(i)]. Fazendo o produto dessas n equações, temos que P >=2^n * sqrt[ produto a(i) ] = 2^n * 2 = 2^(n+1). RESPOSTA : C. Villard -----Mensagem original----- De: Eduardo Casagrande Stabel Para: obm-l@mat.puc-rio.br Data: Quinta-feira, 6 de Junho de 2002 23:09 Assunto: Re: [obm-l] Desafio >Caro Bruno, > >a notação que você usou não está muito legível. Seria melhor adotar índices >para os a's, por exemplo: a_1, a_2, a_3, ..., a_n. Para fazer exponenciação >geralmente se usa "^", aí as alternativas seriam P>2^(n+3), P>5^n, e assim >por diante. > >Quanto ao problema. Existe uma desigualdade, que aprendi a demonstrar por >indução (e talvez você já conheça ou queira provar como exercício) que diz >que se a_1, a_2, ..., a_n são números não-negativos então > >(1 + a_1)*(1 + a_2)*...*(1 + a_n) >= 1 + (a_1*a_2*...*a_n) > >com a igualdade se e so se todos os a_i's forem iguais a zero. > >No caso do seu problema. Temos > >P = (1 + a_1)*(1 + a_2)*...*(1 + a_n) > 1 + (a_1*a_2*...*a_n) = 5. > >Isso claramente não resolve o problema. Uma estratégia mais interessante me >parece procurar pelo valor mínimo de P, após fixado o n. > >Fazendo a multiplicação, temos > >P = (1 + a_1)*(1 + a_2)*...*(1 + a_n) = 1 + [a_1+a_2+...+a_n] + >[a_1a_2+a_1a_3+...+a_(n-1)a_n] + ... + [a_1a_2...a_n] > >No primeiro colchetes temos os n termos a's solitários. >No segundo colchetes temos os produtos de pares de a's. >No terceiro, o produto de trincas. E assim por diante. >Vamos aplicar a desigualdade: média aritmética >= média geométrica em cada >um dos colchetes. > >P >= 1 + n*[RAIZ_n {a_1a_2...a_n}] + n(n-1)/2*[RAIZ_n(n-1)/2 >{(a_1a_2...a_n)^(n-1)}] + ... + [a_1a_2...a_n] > >De forma mais compacta > >P >= 1 + SOMATÓRIO{ k=1...n : C(n,k) * RAIZ_C(n,k) { >(a_1a_2...a_n)^(C(n-1,k-1)) } } = >1 + SOMATÓRIO{ k=1...n : C(n,k) * (RAIZ_n (4^k) } >= (1 + RAIZ_n(4))^n > >((Revisem as contas, fiz de modo simplificado)) > >Basta mostrar que a igualdade ocorre se e somente se a_1=a_2=...a_n, mas >isso é claro por termos usado a desigualdade média aritmética e geométrica. > >Portanto P >= (1 + RAIZ_n(4))^n e a igualdade pode ocorrer para cada n. > >Com isso fica fácil de ver que qualquer exponencial do tipo a^n (onde a>1) >vai acabar superando P para algum n suficientemente grande, repare que 1 + >RAIZ_n(4), a "base" da nossa exponencial se aproxima de 1 a medida que n >cresce. De modo que nem a) nem b) nem c) nem d) são verdadeiras. Logo a >alternativa correta é e). > >Um abraço! > >Eduardo Casagrande Stabel. Porto Alegre, RS. > > >>From: Bruno >> >>Eu não consegui fazer este exercício do ITA e desafio todos dessa lista: >>"Suponha a', a'', ....., an são números reais positivos, com n>2 e que >>a'.a''.a'''....an=4 >>Nesta situação, a repeito do produto: >>P=(1+a')(1+a'').......(1+an) temos: >> n+3 >>a.)P>2 >> n >> b.)P>5 >> n+1 >> c.)P>2 >> n+1 >>d.)P>5 >> e.)n.d.a. >> > > >========================================================================= >Instruções para entrar na lista, sair da lista e usar a lista em >http://www.mat.puc-rio.br/~nicolau/olimp/obm-l.html >O administrador desta lista é >========================================================================= > ========================================================================= Instruções para entrar na lista, sair da lista e usar a lista em http://www.mat.puc-rio.br/~nicolau/olimp/obm-l.html O administrador desta lista é ========================================================================= From owner-obm-l@sucuri.mat.puc-rio.br Fri Jun 7 13:24:17 2002 Return-Path: Received: (from majordom@localhost) by sucuri.mat.puc-rio.br (8.9.3/8.9.3) id NAA26751 for obm-l-list; Fri, 7 Jun 2002 13:23:51 -0300 Received: from web2.poli.usp.br (web2.poli.usp.br [143.107.106.102]) by sucuri.mat.puc-rio.br (8.9.3/8.9.3) with SMTP id NAA26747 for ; Fri, 7 Jun 2002 13:23:49 -0300 Received: from apl03.poli.usp.br ([143.107.106.15]) by web2.poli.usp.br with Microsoft SMTPSVC(5.0.2195.4453); Fri, 7 Jun 2002 13:23:18 -0300 X-MimeOLE: Produced By Microsoft Exchange V6.0.5762.3 content-class: urn:content-classes:message MIME-Version: 1.0 Content-Type: text/plain; charset="iso-8859-1" Subject: RES: [obm-l] integral sem fazer a conta Date: Fri, 7 Jun 2002 13:23:18 -0300 Message-ID: <2B184DFE97456744924ACF58987D941D01186768@apl03.poli.usp.br> X-MS-Has-Attach: X-MS-TNEF-Correlator: Thread-Topic: [obm-l] integral sem fazer a conta Thread-Index: AcINnvY23iB+G1tPSwyriiv28nB/KwAoDHNc From: "Diego Alonso Teixeira" To: , X-OriginalArrivalTime: 07 Jun 2002 16:23:18.0622 (UTC) FILETIME=[A22A87E0:01C20E3F] Content-Transfer-Encoding: 8bit X-MIME-Autoconverted: from quoted-printable to 8bit by sucuri.mat.puc-rio.br id NAA26748 Sender: owner-obm-l@sucuri.mat.puc-rio.br Precedence: bulk Reply-To: obm-l@mat.puc-rio.br ola,sou novo na lista, e estou aprendendo integral na faculdade, gostaria de saber se existe algum truque para calcular a integral de sen x elevado a um numero par grande. -----Mensagem original----- De: Nicolau C. Saldanha [mailto:nicolau@sucuri.mat.puc-rio.br] Enviada: qui 6/6/2002 16:50 Para: obm-l@mat.puc-rio.br Cc: Assunto: Re: [obm-l] integral sem fazer a conta On Wed, Jun 05, 2002 at 08:27:19PM -0300, Augusto César Morgado wrote: > Eu, e creio que muitos outros, quero manifestar minha admiraçao por quem > consegue entender alguma coisa escrita em tao exotica notaçao. > Morgado Esta notação chama-se TeX e não é nada exótica na comunidade matemática. Mesmo assim, acho que devemos tentar usar notações mais autoexplicativas. E principalmente devemos evitar uma notação que para alguns membros da lista é "exótica" quando há alternativas óbvias: pq, por exemplo, escrever \frac{du}{u^2 + (1-x^2)/x^2} ao invés de du/(u^2 + (1-x^2)/x^2)? A idéia aqui é que a lista seja lida e não TeXada ou processada com algum outro programa. Obrigado, []s, N. ========================================================================= Instruções para entrar na lista, sair da lista e usar a lista em http://www.mat.puc-rio.br/~nicolau/olimp/obm-l.html O administrador desta lista é ========================================================================= ola,sou novo na lista, e estou aprendendo integral na faculdade, gostaria de saber se existe algum truque para calcular a integral de sen x elevado a um numero par grande. ========================================================================= Instruções para entrar na lista, sair da lista e usar a lista em http://www.mat.puc-rio.br/~nicolau/olimp/obm-l.html O administrador desta lista é ========================================================================= From owner-obm-l@sucuri.mat.puc-rio.br Fri Jun 7 14:11:31 2002 Return-Path: Received: (from majordom@localhost) by sucuri.mat.puc-rio.br (8.9.3/8.9.3) id OAA27719 for obm-l-list; Fri, 7 Jun 2002 14:09:04 -0300 Received: from fgvrj23.fgv.br (fgvrj23.fgv.br [200.20.164.23]) by sucuri.mat.puc-rio.br (8.9.3/8.9.3) with ESMTP id OAA27715 for ; Fri, 7 Jun 2002 14:08:57 -0300 Received: by FGVRJ23 with Internet Mail Service (5.5.2653.19) id ; Fri, 7 Jun 2002 14:14:00 -0300 Message-ID: <765A72978645D4118B1C0000E229806D05B6B3C9@FGVRJ23> From: Ralph Teixeira To: "'obm-l@mat.puc-rio.br'" Subject: [obm-l] =?iso-8859-1?Q?RES=3A_=5Bobm-l=5D_diferen=E7a_de_ra=EDzes?= Date: Fri, 7 Jun 2002 14:13:56 -0300 MIME-Version: 1.0 X-Mailer: Internet Mail Service (5.5.2653.19) Content-Type: text/plain; charset="iso-8859-1" Content-Transfer-Encoding: 8bit X-MIME-Autoconverted: from quoted-printable to 8bit by sucuri.mat.puc-rio.br id OAA27716 Sender: owner-obm-l@sucuri.mat.puc-rio.br Precedence: bulk Reply-To: obm-l@mat.puc-rio.br > Qual a diferença entre a maior e a menor raiz da > equação: > x^2 + (9x²)/(x+3)² = 27 Eu tentei fazer y=(x+3)/3 para facilitar um pouco as contas (x=3y-3) e cheguei a: 9(y-1)^2 + 9.9(y-1)^2/9y^2 = 27 (y-1)^2 + (y-1)^2/y^2 = 3 y^2 -2y +1 +1 -2/y +1/y^2 -3=0 y^2 -2y -1 -2/y +1/y^2 = 0 Essa aqui é uma daquelas equações "clássicas" que sai fazendo z=y-1/y. Não terminei as contas, mas agora tem de sair.... eu espero. :) Abraço, Ralph ========================================================================= Instruções para entrar na lista, sair da lista e usar a lista em http://www.mat.puc-rio.br/~nicolau/olimp/obm-l.html O administrador desta lista é ========================================================================= From owner-obm-l@sucuri.mat.puc-rio.br Fri Jun 7 14:20:42 2002 Return-Path: Received: (from majordom@localhost) by sucuri.mat.puc-rio.br (8.9.3/8.9.3) id OAA28083 for obm-l-list; Fri, 7 Jun 2002 14:19:03 -0300 Received: from fgvrj23.fgv.br (fgvrj23.fgv.br [200.20.164.23]) by sucuri.mat.puc-rio.br (8.9.3/8.9.3) with ESMTP id OAA28070 for ; Fri, 7 Jun 2002 14:18:47 -0300 Received: by FGVRJ23 with Internet Mail Service (5.5.2653.19) id ; Fri, 7 Jun 2002 14:23:23 -0300 Message-ID: <765A72978645D4118B1C0000E229806D05B6B3CA@FGVRJ23> From: Ralph Teixeira To: "'obm-l@mat.puc-rio.br'" Subject: [obm-l] =?iso-8859-1?Q?RES=3A_=5Bobm-l=5D_Varia=E7=E3o=5Fna=5Fquest=E3?= =?iso-8859-1?Q?o=5Fdo=5FIME=3A=5Fx=3Dsqrt=28a-sqrt=28a-x=29=29?= Date: Fri, 7 Jun 2002 14:23:18 -0300 MIME-Version: 1.0 X-Mailer: Internet Mail Service (5.5.2653.19) Content-Type: text/plain; charset="iso-8859-1" Content-Transfer-Encoding: 8bit X-MIME-Autoconverted: from quoted-printable to 8bit by sucuri.mat.puc-rio.br id OAA28072 Sender: owner-obm-l@sucuri.mat.puc-rio.br Precedence: bulk Reply-To: obm-l@mat.puc-rio.br O erro está no fato de que estas soluções que você achou serem apenas CANDIDATAS a soluções. Quando você eleva ao quadrado, pode introduzir "raízes estranhas" à equação. Tipo, digamos que temos de resolver: x-1=2 Sim, a solução é x=3... Mas alguém poderia elevar ao quadrado: x^2-2x+1=4 x^2-2x-3=0 x=-1 ou x=3 O fato de que chegamos a duas soluções não significa que ambas sejam válidas! (Neste caso, por exemplo, x=-1 é estranha). Você chegou a 4 soluções elevando a equação duas vezes ao quadrado. Mas será que todas são soluções da equação original? Um pouco mais de trabalho mostrará que uma delas não é raiz da original (x negativo, não pode ser, pois x=sqrt(alguma coisa) tem de ser não-negativo). Abraço, Ralph -----Mensagem original----- De: Andre S [mailto:andreluiz_sch@yahoo.com.br] Enviada em: quarta-feira, 5 de junho de 2002 01:17 Para: obm-l@mat.puc-rio.br Assunto: Re: [obm-l] Variação_na_questão_do_IME:_x=sqrt(a-sqrt(a-x)) Ralph, em alguma parte do problema eu devo ter cometido algum engano, pq achei 4 raízes para x=sqrt(0,8 - sqrt(0,8-x))... segue meu desenvolvimento... (x^2 - 0,8)^2=0,8 - x (I) x^4 - 1,6x^2 + x - 0,16 = 0 Utilizando-se x=sqrt(a - x), descobre-se 2 das 4 possiveis raízes de x=sqrt(a-sqrt(a-x)) logo, duas raízes de (I) estão em (II) x^2 + x - 0,8 = 0 {(-1+sqrt(4,2))/2;(-1-sqrt(4,2))/2} considerando (I) na forma (x - x1)(x - x2)(x - x3)(x - x4) = 0 , sendo x2, x2, x3, x4 raízes, e (II) na forma (x - x1)(x - x2) = 0, a divisão (I)/(II) deve apresentar uma equação do 2º grau com as duas soluções que faltam. (x^4 - 1,6x^2 + x - 0,16) / (x^2 +x -0,8) = (III) x^2 - x + 0,2 = 0 {(1+sqrt(0,2))/2;1-sqrt(0,2))/2} Dessa forma, seriam soluções da equação inicial S = {(-1+sqrt(4,2))/2;(-1-sqrt(4,2))/2;(1+sqrt(0,2))/2;1-sqrt(0,2))/2} Onde foi meu erro? []'s, A.S. --- Ralph Teixeira escreveu: > Olá, galera. > > Um colega nosso da lista, o Cláudio, destacou que > eu havia me > enganado quando disse que a equação > > x=sqrt(5-sqrt(5-x)) > > tem *DUAS* soluções. Ele tem razão -- apesar de eu > ainda defender o > fato de que você NÃO PODE SIMPLESMENTE DIZER QUE > x=sqrt(5-x), o meu método > acaba por gerar apenas uma raiz de qualquer forma > (eu havia cometido um erro > de álgebra que o Cláudio encontrou). > > Por exemplo, a equação x=sqrt(0.8-sqrt(0.8-x)) tem > 3 raízes reais > (você consegue encontrá-las?). Apenas *1* delas é > encontrada fazendo > x=sqrt(0.8-x). > > Minha perguntinha para a galera é então: para que > valores de a as > equações > x=sqrt(a-sqrt(a-x)) e x=sqrt(a-x) são equivalentes? > > Divirtam-se! > > Abraço, > Ralph > ========================================================================= > Instruções para entrar na lista, sair da lista e > usar a lista em > http://www.mat.puc-rio.br/~nicolau/olimp/obm-l.html > O administrador desta lista é > > ========================================================================= _______________________________________________________________________ Yahoo! Encontros O lugar certo para você encontrar aquela pessoa que falta na sua vida. Cadastre-se hoje mesmo! http://br.encontros.yahoo.com/ ========================================================================= Instruções para entrar na lista, sair da lista e usar a lista em http://www.mat.puc-rio.br/~nicolau/olimp/obm-l.html O administrador desta lista é ========================================================================= ========================================================================= Instruções para entrar na lista, sair da lista e usar a lista em http://www.mat.puc-rio.br/~nicolau/olimp/obm-l.html O administrador desta lista é ========================================================================= From owner-obm-l@sucuri.mat.puc-rio.br Fri Jun 7 14:48:29 2002 Return-Path: Received: (from majordom@localhost) by sucuri.mat.puc-rio.br (8.9.3/8.9.3) id OAA29082 for obm-l-list; Fri, 7 Jun 2002 14:48:13 -0300 Received: from www.zipmail.com.br (smtp.zipmail.com.br [200.187.242.10]) by sucuri.mat.puc-rio.br (8.9.3/8.9.3) with ESMTP id OAA29078 for ; Fri, 7 Jun 2002 14:48:06 -0300 From: peterdirichlet@zipmail.com.br Received: from [200.206.103.3] by www.zipmail.com.br with HTTP; Fri, 7 Jun 2002 14:26:54 -0300 Message-ID: <3D00E2D200000266@www.zipmail.com.br> Date: Fri, 7 Jun 2002 15:26:54 -0200 Subject: [obm-l] =?iso-8859-1?Q?Re=3A=20=5Bobm=2Dl=5D=20Re=3A=20=5Bobm=2Dl=5D=20Como=20eu=20posso=20achar=20os=20professo?= =?iso-8859-1?Q?res=20Elon=20Lima=20e=20Gugu=3FProblema=206=2CIMO=202001=2DSolu=E7oes=20=28ofic?= =?iso-8859-1?Q?ial=20e=20outras=29?= To: obm-l@mat.puc-rio.br MIME-Version: 1.0 Content-Type: text/plain; charset="iso-8859-1" Content-Transfer-Encoding: 8bit X-MIME-Autoconverted: from quoted-printable to 8bit by sucuri.mat.puc-rio.br id OAA29079 Sender: owner-obm-l@sucuri.mat.puc-rio.br Precedence: bulk Reply-To: obm-l@mat.puc-rio.br Genti,voces nao entenderam...O que eu quero e a soluçao do Gugu.O Tengan me disse que ele fez bem na porrada:escreveu o d em funçao de a,b e c,obteve uma equaçao de grau 2,escreveu as condiçoes de delta quadrado,e por ai vai...Enfim,era esta e a outra soluçao que eu precisava(a por EISENSTEIN).Por falar nisso,VALEU SALDANHA!!!!!!! Quanto ao Ponce,de onde tu tirou essa de soluçao oficial?Eu tenho essa oficial e uma outra,esta ultima usando a ideia do Caminha,mas com outras expressoes.Depois eu mostro,blz? Peterdirichlet -- Mensagem original -- >On Wed, Jun 05, 2002 at 10:29:55PM -0300, Luiz Antonio Ponce Alonso wrote: >> Olá amigo, >> Uma solução legal para este problema foi dada pelo Nicolau. >> Pergunte a ele e seria bom que pudesse colocar a solução nesta lista. >> Eu discordo do Tengan com relação à solução oficial. Esta apesar de >> artificiosa >> é bonita.Você já analisou-a??? >> Um abraço >> PONCE > >Eu j'a mandei para a lista, deve estar no arquivo. []s, N. >========================================================================= >Instruções para entrar na lista, sair da lista e usar a lista em >http://www.mat.puc-rio.br/~nicolau/olimp/obm-l.html >O administrador desta lista é >========================================================================= > TRANSIRE SVVM PECTVS MVNDOQUE POTIRE CONGREGATI EX TOTO ORBE MATHEMATICI OB SCRIPTA INSIGNIA TRIBVERE Medalha Fields(John Charles Fields) ------------------------------------------ Use o melhor sistema de busca da Internet Radar UOL - http://www.radaruol.com.br ========================================================================= Instruções para entrar na lista, sair da lista e usar a lista em http://www.mat.puc-rio.br/~nicolau/olimp/obm-l.html O administrador desta lista é ========================================================================= From owner-obm-l@sucuri.mat.puc-rio.br Fri Jun 7 15:18:06 2002 Return-Path: Received: (from majordom@localhost) by sucuri.mat.puc-rio.br (8.9.3/8.9.3) id PAA29948 for obm-l-list; Fri, 7 Jun 2002 15:16:51 -0300 Received: from candeias.terra.com.br (candeias.terra.com.br [200.176.3.18]) by sucuri.mat.puc-rio.br (8.9.3/8.9.3) with ESMTP id PAA29943 for ; Fri, 7 Jun 2002 15:16:45 -0300 Received: from engenho.terra.com.br (engenho.terra.com.br [200.176.3.42]) by candeias.terra.com.br (Postfix) with ESMTP id E75F343D7C for ; Fri, 7 Jun 2002 15:16:06 -0300 (EST) Received: from Itautec.terra.com.br (200-158-61-80.dsl.telesp.net.br [200.158.61.80]) (authenticated user bruleite) by engenho.terra.com.br (Postfix) with ESMTP id 42E64680D2 for ; Fri, 7 Jun 2002 15:16:06 -0300 (EST) Message-Id: <5.1.0.14.0.20020607145547.00a22060@pop.sao.terra.com.br> X-Sender: bruleite@pop.sao.terra.com.br X-Mailer: QUALCOMM Windows Eudora Version 5.1 Date: Fri, 07 Jun 2002 15:14:50 -0300 To: obm-l@mat.puc-rio.br From: "Bruno F. C. Leite" Subject: Re: [obm-l] Livros de Inducao / Analitica In-Reply-To: <20020606192204.27571.qmail@web14810.mail.yahoo.com> Mime-Version: 1.0 Content-Type: text/plain; charset="iso-8859-1"; format=flowed Content-Transfer-Encoding: 8bit X-MIME-Autoconverted: from quoted-printable to 8bit by sucuri.mat.puc-rio.br id PAA29945 Sender: owner-obm-l@sucuri.mat.puc-rio.br Precedence: bulk Reply-To: obm-l@mat.puc-rio.br At 16:22 06/06/02 -0300, you wrote: >Olá amigos. > >Alguem poderia me indicar um livro/site que explique o método de Inducao >Matematica? Quero um livro que nao seja o "Matematica Elementar". Estou >com duvidas principalmente (mas nao somente) em provar que inequacoes sao >verdadeiras. Há um artigo, acho que na eureka1, sobre isso. além disso, no site http://www.maths.uwa.edu.au/~gregg/Academy/ tem umas coisas sobre indução >Abusando, poderiam me falar um livro de Geometria Analitica (com >tratamento de vetores) diferente do do Elon? No IME-USP usam o "Geometria Analítica - um tratamento vetorial", de Boulos e Camargo Bruno Leite http://www.ime.usp.br/~brleite >===== >[]s >Ricardo Miranda >Matematica - UFV >ricardomirandabr@yahoo.com.br >http://rm2.hpg.ig.com.br/ > >_______________________________________________________________________ >Copa 2002 >Yahoo! - Patrocinador oficial da Copa do Mundo da FIFA 2002 >http://br.sports.yahoo.com/fifaworldcup/ >========================================================================= >Instruções para entrar na lista, sair da lista e usar a lista em >http://www.mat.puc-rio.br/~nicolau/olimp/obm-l.html >O administrador desta lista é >========================================================================= ========================================================================= Instruções para entrar na lista, sair da lista e usar a lista em http://www.mat.puc-rio.br/~nicolau/olimp/obm-l.html O administrador desta lista é ========================================================================= From owner-obm-l@sucuri.mat.puc-rio.br Fri Jun 7 15:22:13 2002 Return-Path: Received: (from majordom@localhost) by sucuri.mat.puc-rio.br (8.9.3/8.9.3) id PAA30089 for obm-l-list; Fri, 7 Jun 2002 15:22:03 -0300 Received: from wool.vetor.com.br (200.160.244.7.metrored.net.br [200.160.244.7] (may be forged)) by sucuri.mat.puc-rio.br (8.9.3/8.9.3) with ESMTP id PAA30085 for ; Fri, 7 Jun 2002 15:21:47 -0300 Received: (from root@localhost) by wool.vetor.com.br (8.11.4/8.11.4) id g57IKvQ17963 for obm-l@mat.puc-rio.br; Fri, 7 Jun 2002 15:20:57 -0300 Received: from rodrigo ([200.160.244.105]) by wool.vetor.com.br (8.11.4/8.11.4) with SMTP id g57IKu317942 for ; Fri, 7 Jun 2002 15:20:56 -0300 Message-ID: <003201c20e4f$f9a153e0$69f4a0c8@rodrigo> From: "Rodrigo Villard Milet" To: Subject: [obm-l] =?iso-8859-1?Q?Re:_=5Bobm-l=5D_diferen=E7a_de_ra=EDzes?= Date: Fri, 7 Jun 2002 15:20:15 -0300 MIME-Version: 1.0 Content-Type: text/plain; charset="iso-8859-1" Content-Transfer-Encoding: 8bit X-Priority: 3 X-MSMail-Priority: Normal X-Mailer: Microsoft Outlook Express 4.72.3110.5 X-MimeOLE: Produced By Microsoft MimeOLE V4.72.3110.3 X-Virus-Scanned: by AMaViS perl-11 Sender: owner-obm-l@sucuri.mat.puc-rio.br Precedence: bulk Reply-To: obm-l@mat.puc-rio.br Só complementando a msg anterior... a resposta então é 3*sqrt(5). -----Mensagem original----- De: Rafael WC Para: obm-l@mat.puc-rio.br Data: Sexta-feira, 7 de Junho de 2002 07:19 Assunto: [obm-l] diferença de raízes >Olá Pessoal! > >Já tem um mês que eu tento resolver esse exercício sem >sucesso. Se alguém conseguir algum avanço, por favor >escreva! > >Qual a diferença entre a maior e a menor raiz da >equação: >x^2 + (9x²)/(x+3)² = 27 > >Obrigado. > >Rafael. > > >===== >Rafael Werneck Cinoto > ICQ# 107011599 > rwcinoto@yahoo.com > rafael.caixa@gov.com.br > matduvidas@yahoo.com.br >http://www.rwcinoto.hpg.com.br/ > >__________________________________________________ >Do You Yahoo!? >Yahoo! - Official partner of 2002 FIFA World Cup >http://fifaworldcup.yahoo.com >========================================================================= >Instruções para entrar na lista, sair da lista e usar a lista em >http://www.mat.puc-rio.br/~nicolau/olimp/obm-l.html >O administrador desta lista é >========================================================================= > ========================================================================= Instruções para entrar na lista, sair da lista e usar a lista em http://www.mat.puc-rio.br/~nicolau/olimp/obm-l.html O administrador desta lista é ========================================================================= From owner-obm-l@sucuri.mat.puc-rio.br Fri Jun 7 15:49:12 2002 Return-Path: Received: (from majordom@localhost) by sucuri.mat.puc-rio.br (8.9.3/8.9.3) id PAA31395 for obm-l-list; Fri, 7 Jun 2002 15:48:50 -0300 Received: from wool.vetor.com.br (200.160.244.7.metrored.net.br [200.160.244.7] (may be forged)) by sucuri.mat.puc-rio.br (8.9.3/8.9.3) with ESMTP id PAA31384 for ; Fri, 7 Jun 2002 15:47:23 -0300 Received: (from root@localhost) by wool.vetor.com.br (8.11.4/8.11.4) id g57IkXH24006 for obm-l@mat.puc-rio.br; Fri, 7 Jun 2002 15:46:33 -0300 Received: (from root@localhost) by wool.vetor.com.br (8.11.4/8.11.4) id g57IIt917432 for obm-l@mat.puc-rio.br; Fri, 7 Jun 2002 15:18:55 -0300 Received: from rodrigo ([200.160.244.105]) by wool.vetor.com.br (8.11.4/8.11.4) with SMTP id g57IIr317425 for ; Fri, 7 Jun 2002 15:18:53 -0300 Message-ID: <002101c20e4f$b06c7ba0$69f4a0c8@rodrigo> From: "Rodrigo Villard Milet" To: Subject: [obm-l] =?iso-8859-1?Q?Re:_=5Bobm-l=5D_diferen=E7a_de_ra=EDzes?= Date: Fri, 7 Jun 2002 15:18:12 -0300 MIME-Version: 1.0 Content-Type: text/plain; charset="iso-8859-1" Content-Transfer-Encoding: 8bit X-Priority: 3 X-MSMail-Priority: Normal X-Mailer: Microsoft Outlook Express 4.72.3110.5 X-MimeOLE: Produced By Microsoft MimeOLE V4.72.3110.3 X-Virus-Scanned: by AMaViS perl-11 Sender: owner-obm-l@sucuri.mat.puc-rio.br Precedence: bulk Reply-To: obm-l@mat.puc-rio.br A equação dada é equivalente a : [x - 3x/(x+3)]² = 27 - 6x²/(x+3), ou seja, [x²/(x+3)]² = 27 - 6x²/(x+3). Agora faça x²/(x+3) = y. Temos que y² + 6y - 27 = 0 e segue que y = -9 ou y = 3. (i) x²/(x+3) = -9 ... x² + 9x + 27 = 0, que ñ dá raízes reais... (ii) x²/(x+3) = 3 ... x² - 3x - 9 = 0 ... que dá x = [3+-3 sqrt(5)]/2 Talvez tenha alguma conta errada... confira ! Abraços, Villard -----Mensagem original----- De: Rafael WC Para: obm-l@mat.puc-rio.br Data: Sexta-feira, 7 de Junho de 2002 07:19 Assunto: [obm-l] diferença de raízes >Olá Pessoal! > >Já tem um mês que eu tento resolver esse exercício sem >sucesso. Se alguém conseguir algum avanço, por favor >escreva! > >Qual a diferença entre a maior e a menor raiz da >equação: >x^2 + (9x²)/(x+3)² = 27 > >Obrigado. > >Rafael. > > >===== >Rafael Werneck Cinoto > ICQ# 107011599 > rwcinoto@yahoo.com > rafael.caixa@gov.com.br > matduvidas@yahoo.com.br >http://www.rwcinoto.hpg.com.br/ > >__________________________________________________ >Do You Yahoo!? >Yahoo! - Official partner of 2002 FIFA World Cup >http://fifaworldcup.yahoo.com >========================================================================= >Instruções para entrar na lista, sair da lista e usar a lista em >http://www.mat.puc-rio.br/~nicolau/olimp/obm-l.html >O administrador desta lista é >========================================================================= > ========================================================================= Instruções para entrar na lista, sair da lista e usar a lista em http://www.mat.puc-rio.br/~nicolau/olimp/obm-l.html O administrador desta lista é ========================================================================= From owner-obm-l@sucuri.mat.puc-rio.br Fri Jun 7 16:25:11 2002 Return-Path: Received: (from majordom@localhost) by sucuri.mat.puc-rio.br (8.9.3/8.9.3) id QAA00388 for obm-l-list; Fri, 7 Jun 2002 16:23:59 -0300 Received: from candeias.terra.com.br (candeias.terra.com.br [200.176.3.18]) by sucuri.mat.puc-rio.br (8.9.3/8.9.3) with ESMTP id QAA00372 for ; Fri, 7 Jun 2002 16:23:42 -0300 Received: from taipe.terra.com.br (taipe.terra.com.br [200.176.3.34]) by candeias.terra.com.br (Postfix) with ESMTP id 7FB7D43DD1 for ; Fri, 7 Jun 2002 16:23:10 -0300 (EST) Received: from niski.com (dl-adsl-C8D44CF7.sao.terra.com.br [200.212.76.247]) (authenticated user fniski) by taipe.terra.com.br (Postfix) with ESMTP id 7995F1B408C for ; Fri, 7 Jun 2002 16:23:09 -0300 (EST) Message-ID: <3D01081B.2020600@niski.com> Date: Fri, 07 Jun 2002 16:23:07 -0300 From: niski User-Agent: Mozilla/5.0 (Windows; U; Windows NT 5.1; en-US; rv:0.9.4.1) Gecko/20020508 Netscape6/6.2.3 X-Accept-Language: en-us MIME-Version: 1.0 To: obm-l@mat.puc-rio.br Subject: Re: [obm-l] ajuda: colegio naval References: <3D00185D.E2826B1@terra.com.br> Content-Type: text/plain; charset=ISO-8859-1; format=flowed Content-Transfer-Encoding: 8bit Sender: owner-obm-l@sucuri.mat.puc-rio.br Precedence: bulk Reply-To: obm-l@mat.puc-rio.br Olá Ponce! Tenho em pdf a apenas a prova de 1999/2000. Para qual e-mail seu voce quer que eu mande?! Um grande abraço , do aluno Fabio Luiz Antonio Ponce Alonso wrote: >Caros amigos, >Estou precisando de provas de matemática do colegio naval para ajudar na >preparação >do filho de meu amigo. Caso alguém tenha alguma ou um site em que posso >adquiri-las >ficaria muito grato. >Desde já fico agradecido por qualquer ajuda futura >Um abraço >De seu amigo >PONCE > > >========================================================================= >Instruções para entrar na lista, sair da lista e usar a lista em >http://www.mat.puc-rio.br/~nicolau/olimp/obm-l.html >O administrador desta lista é >========================================================================= > > ========================================================================= Instruções para entrar na lista, sair da lista e usar a lista em http://www.mat.puc-rio.br/~nicolau/olimp/obm-l.html O administrador desta lista é ========================================================================= From owner-obm-l@sucuri.mat.puc-rio.br Fri Jun 7 16:54:41 2002 Return-Path: Received: (from majordom@localhost) by sucuri.mat.puc-rio.br (8.9.3/8.9.3) id QAA01220 for obm-l-list; Fri, 7 Jun 2002 16:54:08 -0300 Received: from studer.bol.com.br (studer.bol.com.br [200.221.24.21]) by sucuri.mat.puc-rio.br (8.9.3/8.9.3) with ESMTP id QAA01216 for ; Fri, 7 Jun 2002 16:54:06 -0300 Received: from h15ag (200.221.24.99) by studer.bol.com.br (5.1.071) id 3CFE81FC000C17D3 for obm-l@mat.puc-rio.br; Fri, 7 Jun 2002 16:52:44 -0300 Message-ID: <000901c20e5d$118017f0$c866010a@valor> From: "leo" To: References: <2B184DFE97456744924ACF58987D941D01186768@apl03.poli.usp.br> Subject: Re: [obm-l] integral sem fazer a conta Date: Fri, 7 Jun 2002 16:53:59 -0300 MIME-Version: 1.0 Content-Type: text/plain; charset="iso-8859-1" Content-Transfer-Encoding: 8bit X-Priority: 3 X-MSMail-Priority: Normal X-Mailer: Microsoft Outlook Express 5.00.2314.1300 X-MimeOLE: Produced By Microsoft MimeOLE V5.00.2314.1300 X-Sender-IP: 200.189.244.67 Sender: owner-obm-l@sucuri.mat.puc-rio.br Precedence: bulk Reply-To: obm-l@mat.puc-rio.br Eu tbm sou novo na lista e tbm tenho duvidas em integrais . Para fazer a integral abaixo seria algo parecido com isso para uma caso genérico: Int senx^2n = (-cosx^2n+1)/2n+1 ? Alguem poderia me indicar bons livros sobre o tema? E mais uma duvida : Onde esbarram os esforços de matematicos em resolver as equações de navier-strokes? []s ----- Original Message ----- From: Diego Alonso Teixeira To: ; Sent: Friday, June 07, 2002 1:23 PM Subject: RES: [obm-l] integral sem fazer a conta > Quer ter seu próprio endereço na Internet? > Garanta já o seu e ainda ganhe cinco e-mails personalizados. > DomíniosBOL - http://dominios.bol.com.br > > > > > > ola,sou novo na lista, e estou aprendendo integral na faculdade, gostaria de saber se existe algum truque para calcular a integral de sen x elevado a um numero par grande. > > -----Mensagem original----- > De: Nicolau C. Saldanha [mailto:nicolau@sucuri.mat.puc-rio.br] > Enviada: qui 6/6/2002 16:50 > Para: obm-l@mat.puc-rio.br > Cc: > Assunto: Re: [obm-l] integral sem fazer a conta > > > > On Wed, Jun 05, 2002 at 08:27:19PM -0300, Augusto César Morgado wrote: > > Eu, e creio que muitos outros, quero manifestar minha admiraçao por quem > > consegue entender alguma coisa escrita em tao exotica notaçao. > > Morgado > > Esta notação chama-se TeX e não é nada exótica na comunidade matemática. > Mesmo assim, acho que devemos tentar usar notações mais autoexplicativas. > E principalmente devemos evitar uma notação que para alguns membros da > lista é "exótica" quando há alternativas óbvias: pq, por exemplo, > escrever \frac{du}{u^2 + (1-x^2)/x^2} ao invés de du/(u^2 + (1-x^2)/x^2)? > A idéia aqui é que a lista seja lida e não TeXada ou processada com algum > outro programa. > > Obrigado, []s, N. > > ========================================================================= > Instruções para entrar na lista, sair da lista e usar a lista em > http://www.mat.puc-rio.br/~nicolau/olimp/obm-l.html > O administrador desta lista é > ========================================================================= > ola,sou novo na lista, e estou aprendendo integral na faculdade, gostaria de saber se existe algum truque para calcular a integral de sen x elevado a um numero par grande. > > ========================================================================= > Instruções para entrar na lista, sair da lista e usar a lista em > http://www.mat.puc-rio.br/~nicolau/olimp/obm-l.html > O administrador desta lista é > ========================================================================= > ========================================================================= Instruções para entrar na lista, sair da lista e usar a lista em http://www.mat.puc-rio.br/~nicolau/olimp/obm-l.html O administrador desta lista é ========================================================================= From owner-obm-l@sucuri.mat.puc-rio.br Fri Jun 7 17:10:38 2002 Return-Path: Received: (from majordom@localhost) by sucuri.mat.puc-rio.br (8.9.3/8.9.3) id RAA01570 for obm-l-list; Fri, 7 Jun 2002 17:10:03 -0300 Received: from candeias.terra.com.br (candeias.terra.com.br [200.176.3.18]) by sucuri.mat.puc-rio.br (8.9.3/8.9.3) with ESMTP id RAA01563 for ; Fri, 7 Jun 2002 17:09:59 -0300 Received: from mucuri.terra.com.br (mucuri.terra.com.br [200.176.3.39]) by candeias.terra.com.br (Postfix) with ESMTP id 4942943E1A for ; Fri, 7 Jun 2002 17:09:32 -0300 (EST) Received: from stabel (dl-nas3-poa-C89A0610.p001.terra.com.br [200.154.6.16]) (authenticated user dudasta) by mucuri.terra.com.br (Postfix) with ESMTP id 19A3EBE924 for ; Fri, 7 Jun 2002 17:09:30 -0300 (EST) Message-ID: <007e01c20e5f$378043b0$d2019ac8@stabel> From: "Eduardo Casagrande Stabel" To: References: <004001c20e2b$d1cdc020$53f4a0c8@rodrigo> Subject: Re: [obm-l] Desafio Date: Fri, 7 Jun 2002 17:07:11 -0300 MIME-Version: 1.0 Content-Type: text/plain; charset="iso-8859-1" Content-Transfer-Encoding: 8bit X-Priority: 3 X-MSMail-Priority: Normal X-Mailer: Microsoft Outlook Express 6.00.2600.0000 X-MimeOLE: Produced By Microsoft MimeOLE V6.00.2600.0000 Sender: owner-obm-l@sucuri.mat.puc-rio.br Precedence: bulk Reply-To: obm-l@mat.puc-rio.br Caro Rodrigo Villard, em verdade faltou um detalhe bobo: P>=2^(n+1) não implica que P>2^(n+1) mesmo assim podemos consertar facilmente pois nem todas as desigualdades 1 + a_i >= 2 RAIZ( a_i ) podem ser igualdades simultaneamente, ja que para isso ocorrer teriamos de ter todos os a_i's iguais a 1. Portanto a conclusao pode ser P>2^(n+1) e o problema esta todo resolvido pelo colega. Eduardo Casagrande Stabel. Porto Alegre, RS. PS. só um outro detalhe, a desigualdade P >= (1 + RAIZ_n( 4 ))^n é verdadeira! O que não é verdadeira é minha conclusão final. Mas é claro que para os propósitos da questão a solução do Villard satisfaz muito mais pois é direta ao resultado, mais curta e utiliza menos recursos. É muito mais objetiva. From: "Rodrigo Villard Milet" > Use que 1+a(i) >=2sqrt[a(i)]. Fazendo o produto dessas n equações, temos que > P >=2^n * sqrt[ produto a(i) ] = 2^n * 2 = 2^(n+1). RESPOSTA : C. > Villard > -----Mensagem original----- > De: Eduardo Casagrande Stabel > Para: obm-l@mat.puc-rio.br > Data: Quinta-feira, 6 de Junho de 2002 23:09 > Assunto: Re: [obm-l] Desafio > > > >Caro Bruno, > > > >a notação que você usou não está muito legível. Seria melhor adotar índices > >para os a's, por exemplo: a_1, a_2, a_3, ..., a_n. Para fazer exponenciação > >geralmente se usa "^", aí as alternativas seriam P>2^(n+3), P>5^n, e assim > >por diante. > > > >Quanto ao problema. Existe uma desigualdade, que aprendi a demonstrar por > >indução (e talvez você já conheça ou queira provar como exercício) que diz > >que se a_1, a_2, ..., a_n são números não-negativos então > > > >(1 + a_1)*(1 + a_2)*...*(1 + a_n) >= 1 + (a_1*a_2*...*a_n) > > > >com a igualdade se e so se todos os a_i's forem iguais a zero. > > > >No caso do seu problema. Temos > > > >P = (1 + a_1)*(1 + a_2)*...*(1 + a_n) > 1 + (a_1*a_2*...*a_n) = 5. > > > >Isso claramente não resolve o problema. Uma estratégia mais interessante me > >parece procurar pelo valor mínimo de P, após fixado o n. > > > >Fazendo a multiplicação, temos > > > >P = (1 + a_1)*(1 + a_2)*...*(1 + a_n) = 1 + [a_1+a_2+...+a_n] + > >[a_1a_2+a_1a_3+...+a_(n-1)a_n] + ... + [a_1a_2...a_n] > > > >No primeiro colchetes temos os n termos a's solitários. > >No segundo colchetes temos os produtos de pares de a's. > >No terceiro, o produto de trincas. E assim por diante. > >Vamos aplicar a desigualdade: média aritmética >= média geométrica em cada > >um dos colchetes. > > > >P >= 1 + n*[RAIZ_n {a_1a_2...a_n}] + n(n-1)/2*[RAIZ_n(n-1)/2 > >{(a_1a_2...a_n)^(n-1)}] + ... + [a_1a_2...a_n] > > > >De forma mais compacta > > > >P >= 1 + SOMATÓRIO{ k=1...n : C(n,k) * RAIZ_C(n,k) { > >(a_1a_2...a_n)^(C(n-1,k-1)) } } = > >1 + SOMATÓRIO{ k=1...n : C(n,k) * (RAIZ_n (4^k) } > >= (1 + RAIZ_n(4))^n > > > >((Revisem as contas, fiz de modo simplificado)) > > > >Basta mostrar que a igualdade ocorre se e somente se a_1=a_2=...a_n, mas > >isso é claro por termos usado a desigualdade média aritmética e geométrica. > > > >Portanto P >= (1 + RAIZ_n(4))^n e a igualdade pode ocorrer para cada n. > > > >Com isso fica fácil de ver que qualquer exponencial do tipo a^n (onde a>1) > >vai acabar superando P para algum n suficientemente grande, repare que 1 + > >RAIZ_n(4), a "base" da nossa exponencial se aproxima de 1 a medida que n > >cresce. De modo que nem a) nem b) nem c) nem d) são verdadeiras. Logo a > >alternativa correta é e). > > > >Um abraço! > > > >Eduardo Casagrande Stabel. Porto Alegre, RS. > > > > > >>From: Bruno > >> > >>Eu não consegui fazer este exercício do ITA e desafio todos dessa lista: > >>"Suponha a', a'', ....., an são números reais positivos, com n>2 e que > >>a'.a''.a'''....an=4 > >>Nesta situação, a repeito do produto: > >>P=(1+a')(1+a'').......(1+an) temos: > >> n+3 > >>a.)P>2 > >> n > >> b.)P>5 > >> n+1 > >> c.)P>2 > >> n+1 > >>d.)P>5 > >> e.)n.d.a. > >> > > > > > >========================================================================= > >Instruções para entrar na lista, sair da lista e usar a lista em > >http://www.mat.puc-rio.br/~nicolau/olimp/obm-l.html > >O administrador desta lista é > >========================================================================= > > > > ========================================================================= > Instruções para entrar na lista, sair da lista e usar a lista em > http://www.mat.puc-rio.br/~nicolau/olimp/obm-l.html > O administrador desta lista é > ========================================================================= > > ========================================================================= Instruções para entrar na lista, sair da lista e usar a lista em http://www.mat.puc-rio.br/~nicolau/olimp/obm-l.html O administrador desta lista é ========================================================================= From owner-obm-l@sucuri.mat.puc-rio.br Fri Jun 7 18:31:11 2002 Return-Path: Received: (from majordom@localhost) by sucuri.mat.puc-rio.br (8.9.3/8.9.3) id SAA03684 for obm-l-list; Fri, 7 Jun 2002 18:30:22 -0300 Received: from traven10.uol.com.br (traven10.uol.com.br [200.231.206.211]) by sucuri.mat.puc-rio.br (8.9.3/8.9.3) with ESMTP id SAA03680 for ; Fri, 7 Jun 2002 18:30:20 -0300 Received: from cabru ([200.158.70.241]) by traven10.uol.com.br (8.9.1/8.9.1) with SMTP id SAA18783 for ; Fri, 7 Jun 2002 18:30:04 -0300 (BRT) Message-ID: <005701c20e6b$5d422260$f1469ec8@cabru> From: "Bruno" To: References: <004001c20e2b$d1cdc020$53f4a0c8@rodrigo> Subject: Re: [obm-l] Desafio Date: Fri, 7 Jun 2002 18:36:19 -0300 MIME-Version: 1.0 Content-Type: text/plain; charset="iso-8859-1" Content-Transfer-Encoding: 8bit X-Priority: 3 X-MSMail-Priority: Normal X-Mailer: Microsoft Outlook Express 5.50.4133.2400 X-MimeOLE: Produced By Microsoft MimeOLE V5.50.4133.2400 Sender: owner-obm-l@sucuri.mat.puc-rio.br Precedence: bulk Reply-To: obm-l@mat.puc-rio.br obrigado pela solução mas o que é: "sqrt"? abraços, Bruno ----- Original Message ----- From: "Rodrigo Villard Milet" To: Sent: Friday, June 07, 2002 11:01 AM Subject: Re: [obm-l] Desafio > Use que 1+a(i) >=2sqrt[a(i)]. Fazendo o produto dessas n equações, temos que > P >=2^n * sqrt[ produto a(i) ] = 2^n * 2 = 2^(n+1). RESPOSTA : C. > Villard > -----Mensagem original----- > De: Eduardo Casagrande Stabel > Para: obm-l@mat.puc-rio.br > Data: Quinta-feira, 6 de Junho de 2002 23:09 > Assunto: Re: [obm-l] Desafio > > > >Caro Bruno, > > > >a notação que você usou não está muito legível. Seria melhor adotar índices > >para os a's, por exemplo: a_1, a_2, a_3, ..., a_n. Para fazer exponenciação > >geralmente se usa "^", aí as alternativas seriam P>2^(n+3), P>5^n, e assim > >por diante. > > > >Quanto ao problema. Existe uma desigualdade, que aprendi a demonstrar por > >indução (e talvez você já conheça ou queira provar como exercício) que diz > >que se a_1, a_2, ..., a_n são números não-negativos então > > > >(1 + a_1)*(1 + a_2)*...*(1 + a_n) >= 1 + (a_1*a_2*...*a_n) > > > >com a igualdade se e so se todos os a_i's forem iguais a zero. > > > >No caso do seu problema. Temos > > > >P = (1 + a_1)*(1 + a_2)*...*(1 + a_n) > 1 + (a_1*a_2*...*a_n) = 5. > > > >Isso claramente não resolve o problema. Uma estratégia mais interessante me > >parece procurar pelo valor mínimo de P, após fixado o n. > > > >Fazendo a multiplicação, temos > > > >P = (1 + a_1)*(1 + a_2)*...*(1 + a_n) = 1 + [a_1+a_2+...+a_n] + > >[a_1a_2+a_1a_3+...+a_(n-1)a_n] + ... + [a_1a_2...a_n] > > > >No primeiro colchetes temos os n termos a's solitários. > >No segundo colchetes temos os produtos de pares de a's. > >No terceiro, o produto de trincas. E assim por diante. > >Vamos aplicar a desigualdade: média aritmética >= média geométrica em cada > >um dos colchetes. > > > >P >= 1 + n*[RAIZ_n {a_1a_2...a_n}] + n(n-1)/2*[RAIZ_n(n-1)/2 > >{(a_1a_2...a_n)^(n-1)}] + ... + [a_1a_2...a_n] > > > >De forma mais compacta > > > >P >= 1 + SOMATÓRIO{ k=1...n : C(n,k) * RAIZ_C(n,k) { > >(a_1a_2...a_n)^(C(n-1,k-1)) } } = > >1 + SOMATÓRIO{ k=1...n : C(n,k) * (RAIZ_n (4^k) } > >= (1 + RAIZ_n(4))^n > > > >((Revisem as contas, fiz de modo simplificado)) > > > >Basta mostrar que a igualdade ocorre se e somente se a_1=a_2=...a_n, mas > >isso é claro por termos usado a desigualdade média aritmética e geométrica. > > > >Portanto P >= (1 + RAIZ_n(4))^n e a igualdade pode ocorrer para cada n. > > > >Com isso fica fácil de ver que qualquer exponencial do tipo a^n (onde a>1) > >vai acabar superando P para algum n suficientemente grande, repare que 1 + > >RAIZ_n(4), a "base" da nossa exponencial se aproxima de 1 a medida que n > >cresce. De modo que nem a) nem b) nem c) nem d) são verdadeiras. Logo a > >alternativa correta é e). > > > >Um abraço! > > > >Eduardo Casagrande Stabel. Porto Alegre, RS. > > > > > >>From: Bruno > >> > >>Eu não consegui fazer este exercício do ITA e desafio todos dessa lista: > >>"Suponha a', a'', ....., an são números reais positivos, com n>2 e que > >>a'.a''.a'''....an=4 > >>Nesta situação, a repeito do produto: > >>P=(1+a')(1+a'').......(1+an) temos: > >> n+3 > >>a.)P>2 > >> n > >> b.)P>5 > >> n+1 > >> c.)P>2 > >> n+1 > >>d.)P>5 > >> e.)n.d.a. > >> > > > > > >========================================================================= > >Instruções para entrar na lista, sair da lista e usar a lista em > >http://www.mat.puc-rio.br/~nicolau/olimp/obm-l.html > >O administrador desta lista é > >========================================================================= > > > > ========================================================================= > Instruções para entrar na lista, sair da lista e usar a lista em > http://www.mat.puc-rio.br/~nicolau/olimp/obm-l.html > O administrador desta lista é > ========================================================================= > ========================================================================= Instruções para entrar na lista, sair da lista e usar a lista em http://www.mat.puc-rio.br/~nicolau/olimp/obm-l.html O administrador desta lista é ========================================================================= From owner-obm-l@sucuri.mat.puc-rio.br Fri Jun 7 18:44:36 2002 Return-Path: Received: (from majordom@localhost) by sucuri.mat.puc-rio.br (8.9.3/8.9.3) id SAA03877 for obm-l-list; Fri, 7 Jun 2002 18:44:32 -0300 Received: from gorgo.centroin.com.br (gorgo.centroin.com.br [200.225.63.128]) by sucuri.mat.puc-rio.br (8.9.3/8.9.3) with ESMTP id SAA03873 for ; Fri, 7 Jun 2002 18:44:29 -0300 Received: from centroin.com.br (du78c.rjo.centroin.com.br [200.225.58.78]) (authenticated bits=0) by gorgo.centroin.com.br (8.12.2/8.12.1) with ESMTP id g57LiLko011414 for ; Fri, 7 Jun 2002 18:44:25 -0300 (BRT) Message-ID: <3D01297F.7080501@centroin.com.br> Date: Fri, 07 Jun 2002 18:45:35 -0300 From: Augusto =?ISO-8859-1?Q?C=E9sar?= Morgado User-Agent: Mozilla/5.0 (Windows; U; Win98; en-US; rv:0.9.4.1) Gecko/20020508 Netscape6/6.2.3 X-Accept-Language: en-us MIME-Version: 1.0 To: obm-l@mat.puc-rio.br Subject: Re: [obm-l] plana ajuda por favor urgente References: Content-Type: text/plain; charset=ISO-8859-1; format=flowed Content-Transfer-Encoding: 8bit Sender: owner-obm-l@sucuri.mat.puc-rio.br Precedence: bulk Reply-To: obm-l@mat.puc-rio.br 1)P1Pn deve ser múltiplo de 360 graus. (n-1)35 = k360 (n-1)7=k72 Como 7 eh primo com 72, n-1 deve ser multiplo de 72. O menor n eh 73. Adherbal Rocha Filho wrote: > > > >> Olá,gostaria da ajuda de vcs nas seguintes questões: >> 1.Os pontos P1,P2,... estão sobre uma circunferencia e são tais que o >> arco q >> une cada ponto ao seguinte mede 35º.O menor valor de n>1 tal que Pn >> coincide >> com P1 é? >> >> 2.Para cada ponto pertencente ao interior e aos lados de um triangulo >> acutangulo ABC,considere a soma das suas distancias aos 3 lados do >> triangulo.O valor maximo desta soma é? >> >> 3.No triangulo ABC,AB=5 e BC=6.Qual a area maxima do triangulo ABC >> ,sabendo >> q o angulo C tem a maior medida possivel? >> >> Muito obrigado! >> té + >> Adherbal > > > > _________________________________________________________________ > Chegou o novo MSN Explorer. Instale já. É gratuito: > http://explorer.msn.com.br > > ========================================================================= > Instruções para entrar na lista, sair da lista e usar a lista em > http://www.mat.puc-rio.br/~nicolau/olimp/obm-l.html > O administrador desta lista é > ========================================================================= > > ========================================================================= Instruções para entrar na lista, sair da lista e usar a lista em http://www.mat.puc-rio.br/~nicolau/olimp/obm-l.html O administrador desta lista é ========================================================================= From owner-obm-l@sucuri.mat.puc-rio.br Fri Jun 7 18:47:04 2002 Return-Path: Received: (from majordom@localhost) by sucuri.mat.puc-rio.br (8.9.3/8.9.3) id SAA03988 for obm-l-list; Fri, 7 Jun 2002 18:47:02 -0300 Received: from gorgo.centroin.com.br (gorgo.centroin.com.br [200.225.63.128]) by sucuri.mat.puc-rio.br (8.9.3/8.9.3) with ESMTP id SAA03984 for ; Fri, 7 Jun 2002 18:46:59 -0300 Received: from centroin.com.br (du39b.nit.centroin.com.br [200.225.56.39]) (authenticated bits=0) by gorgo.centroin.com.br (8.12.2/8.12.1) with ESMTP id g57Ll0ko011597 for ; Fri, 7 Jun 2002 18:47:01 -0300 (BRT) Message-ID: <3D012A1F.5080804@centroin.com.br> Date: Fri, 07 Jun 2002 18:48:15 -0300 From: Augusto =?ISO-8859-1?Q?C=E9sar?= Morgado User-Agent: Mozilla/5.0 (Windows; U; Win98; en-US; rv:0.9.4.1) Gecko/20020508 Netscape6/6.2.3 X-Accept-Language: en-us MIME-Version: 1.0 To: obm-l@mat.puc-rio.br Subject: Re: [obm-l] plana ajuda por favor urgente References: Content-Type: text/plain; charset=ISO-8859-1; format=flowed Content-Transfer-Encoding: 8bit Sender: owner-obm-l@sucuri.mat.puc-rio.br Precedence: bulk Reply-To: obm-l@mat.puc-rio.br Nao tem maximo demais no problema 3? Adherbal Rocha Filho wrote: > > > >> Olá,gostaria da ajuda de vcs nas seguintes questões: >> 1.Os pontos P1,P2,... estão sobre uma circunferencia e são tais que o >> arco q >> une cada ponto ao seguinte mede 35º.O menor valor de n>1 tal que Pn >> coincide >> com P1 é? >> >> 2.Para cada ponto pertencente ao interior e aos lados de um triangulo >> acutangulo ABC,considere a soma das suas distancias aos 3 lados do >> triangulo.O valor maximo desta soma é? >> >> 3.No triangulo ABC,AB=5 e BC=6.Qual a area maxima do triangulo ABC >> ,sabendo >> q o angulo C tem a maior medida possivel? >> >> Muito obrigado! >> té + >> Adherbal > > > > _________________________________________________________________ > Chegou o novo MSN Explorer. Instale já. É gratuito: > http://explorer.msn.com.br > > ========================================================================= > Instruções para entrar na lista, sair da lista e usar a lista em > http://www.mat.puc-rio.br/~nicolau/olimp/obm-l.html > O administrador desta lista é > ========================================================================= > > ========================================================================= Instruções para entrar na lista, sair da lista e usar a lista em http://www.mat.puc-rio.br/~nicolau/olimp/obm-l.html O administrador desta lista é ========================================================================= From owner-obm-l@sucuri.mat.puc-rio.br Fri Jun 7 18:54:21 2002 Return-Path: Received: (from majordom@localhost) by sucuri.mat.puc-rio.br (8.9.3/8.9.3) id SAA04316 for obm-l-list; Fri, 7 Jun 2002 18:54:19 -0300 Received: from traven10.uol.com.br (traven10.uol.com.br [200.231.206.211]) by sucuri.mat.puc-rio.br (8.9.3/8.9.3) with ESMTP id SAA04312 for ; Fri, 7 Jun 2002 18:54:16 -0300 Received: from cabru ([200.158.70.241]) by traven10.uol.com.br (8.9.1/8.9.1) with SMTP id SAA09179; Fri, 7 Jun 2002 18:54:02 -0300 (BRT) Message-ID: <012201c20e6e$b3e33ac0$f1469ec8@cabru> From: "Bruno" To: "OBM-L" Cc: "OBM-L" Subject: [obm-l] Desafio o retorno!! Date: Fri, 7 Jun 2002 19:00:13 -0300 MIME-Version: 1.0 Content-Type: multipart/alternative; boundary="----=_NextPart_000_011F_01C20E55.8E2825C0" X-Priority: 3 X-MSMail-Priority: Normal X-Mailer: Microsoft Outlook Express 5.50.4133.2400 X-MimeOLE: Produced By Microsoft MimeOLE V5.50.4133.2400 Sender: owner-obm-l@sucuri.mat.puc-rio.br Precedence: bulk Reply-To: obm-l@mat.puc-rio.br This is a multi-part message in MIME format. ------=_NextPart_000_011F_01C20E55.8E2825C0 Content-Type: text/plain; charset="iso-8859-1" Content-Transfer-Encoding: quoted-printable Ol=E1 amigos, Estou de volta com outro exerc=EDcio(+dif=EDcil na minha opini=E3o) Meu professor disse que nem ele consegue fatorar a seguinte express=E3o = em IR: x^6 + (xy)^3 + y^6 Abra=E7os, Bruno ------=_NextPart_000_011F_01C20E55.8E2825C0 Content-Type: text/html; charset="iso-8859-1" Content-Transfer-Encoding: quoted-printable
Ol=E1 amigos,
Estou de volta com outro exerc=EDcio(+dif=EDcil na = minha=20 opini=E3o)
Meu professor disse que nem ele consegue fatorar a = seguinte=20 express=E3o em IR:
x^6 + (xy)^3 + y^6
 
Abra=E7os,
Bruno
------=_NextPart_000_011F_01C20E55.8E2825C0-- ========================================================================= Instruções para entrar na lista, sair da lista e usar a lista em http://www.mat.puc-rio.br/~nicolau/olimp/obm-l.html O administrador desta lista é ========================================================================= From owner-obm-l@sucuri.mat.puc-rio.br Fri Jun 7 19:05:22 2002 Return-Path: Received: (from majordom@localhost) by sucuri.mat.puc-rio.br (8.9.3/8.9.3) id TAA04860 for obm-l-list; Fri, 7 Jun 2002 19:03:40 -0300 Received: from candeias.terra.com.br (candeias.terra.com.br [200.176.3.18]) by sucuri.mat.puc-rio.br (8.9.3/8.9.3) with ESMTP id TAA04855 for ; Fri, 7 Jun 2002 19:03:37 -0300 Received: from mucuri.terra.com.br (mucuri.terra.com.br [200.176.3.39]) by candeias.terra.com.br (Postfix) with ESMTP id 3E72543CEB for ; Fri, 7 Jun 2002 19:03:10 -0300 (EST) Received: from nt (RJ231080.user.veloxzone.com.br [200.165.231.80]) (authenticated user ensr) by mucuri.terra.com.br (Postfix) with ESMTP id A7CDBBE8C3 for ; Fri, 7 Jun 2002 19:03:09 -0300 (EST) Message-ID: <00e101c20e6e$d5de43e0$5400a8c0@ensrbr> From: "Luis Lopes" To: References: <3CFE9E57.6050600@centroin.com.br> <20020606165001.B4737@sucuri.mat.puc-rio.br> Subject: [obm-l] Uso do TeX [era: integral sem fazer a conta] Date: Fri, 7 Jun 2002 19:01:10 -0300 MIME-Version: 1.0 Content-Type: text/plain; charset="iso-8859-1" Content-Transfer-Encoding: 8bit X-Priority: 3 X-MSMail-Priority: Normal X-Mailer: Microsoft Outlook Express 5.00.2615.200 X-MimeOLE: Produced By Microsoft MimeOLE V5.00.2615.200 Sender: owner-obm-l@sucuri.mat.puc-rio.br Precedence: bulk Reply-To: obm-l@mat.puc-rio.br Sauda,c~oes, Não se trata de complicar. Naturalmente ou não a lista já usa notação TeX em diversas msgs. Veja essa por exemplo: 1+a(i) >=2sqrt[a(i)]. Fazendo o produto dessas n equações, temos que P >=2^n * sqrt[ produto a(i) ] = 2^n * 2 = 2^(n+1). Isso é quase TeX. Além disso não vejo nada melhor para integral, somatório, produtório, índices e expoentes do que \int, \sum, \prod, ^ _ Assim \int_a^b f(x)dx = F(b) - F(a) é bem apropriado. Por outro lado, escrever \frac{A}{B} para representar A/B é um pouco demais. Escrevo \frac quando A ou B é longo para chamar a atenção de que uma fração vai seguir. Não acho todavia que devemos considerar um empecilho uma mensagem escrita em TeX (com parcimônia) e inibir aqueles que se sentem à vontade em lê-lo e escrevê-lo. Até porque i) é a linguagem da comunicação na comunidade matemática; ii) pode sempre haver esclarecimentos da parte de quem mandou a mensagem. Espero não incomodar a lista com outras msgs do gênero. []'s Luís -----Mensagem Original----- De: Nicolau C. Saldanha Para: Enviada em: quinta-feira, 6 de junho de 2002 16:50 Assunto: Re: [obm-l] integral sem fazer a conta > On Wed, Jun 05, 2002 at 08:27:19PM -0300, Augusto César Morgado wrote: > > Eu, e creio que muitos outros, quero manifestar minha admiraçao por quem > > consegue entender alguma coisa escrita em tao exotica notaçao. > > Morgado > > Esta notação chama-se TeX e não é nada exótica na comunidade matemática. > Mesmo assim, acho que devemos tentar usar notações mais autoexplicativas. > E principalmente devemos evitar uma notação que para alguns membros da > lista é "exótica" quando há alternativas óbvias: pq, por exemplo, > escrever \frac{du}{u^2 + (1-x^2)/x^2} ao invés de du/(u^2 + (1-x^2)/x^2)? > A idéia aqui é que a lista seja lida e não TeXada ou processada com algum > outro programa. > > Obrigado, []s, N. > > ========================================================================= > Instruções para entrar na lista, sair da lista e usar a lista em > http://www.mat.puc-rio.br/~nicolau/olimp/obm-l.html > O administrador desta lista é > ========================================================================= > > ========================================================================= Instruções para entrar na lista, sair da lista e usar a lista em http://www.mat.puc-rio.br/~nicolau/olimp/obm-l.html O administrador desta lista é ========================================================================= From owner-obm-l@sucuri.mat.puc-rio.br Fri Jun 7 19:10:20 2002 Return-Path: Received: (from majordom@localhost) by sucuri.mat.puc-rio.br (8.9.3/8.9.3) id TAA05211 for obm-l-list; Fri, 7 Jun 2002 19:08:56 -0300 Received: from gorgo.centroin.com.br (gorgo.centroin.com.br [200.225.63.128]) by sucuri.mat.puc-rio.br (8.9.3/8.9.3) with ESMTP id TAA05206 for ; Fri, 7 Jun 2002 19:08:53 -0300 Received: from centroin.com.br (du171c.rjo.centroin.com.br [200.225.58.171]) (authenticated bits=0) by gorgo.centroin.com.br (8.12.2/8.12.1) with ESMTP id g57M8sko013479 for ; Fri, 7 Jun 2002 19:08:54 -0300 (BRT) Message-ID: <3D012F3E.1000703@centroin.com.br> Date: Fri, 07 Jun 2002 19:10:06 -0300 From: Augusto =?ISO-8859-1?Q?C=E9sar?= Morgado User-Agent: Mozilla/5.0 (Windows; U; Win98; en-US; rv:0.9.4.1) Gecko/20020508 Netscape6/6.2.3 X-Accept-Language: en-us MIME-Version: 1.0 To: obm-l@mat.puc-rio.br Subject: Re: [obm-l] integral sem fazer a conta References: <2B184DFE97456744924ACF58987D941D01186768@apl03.poli.usp.br> <000901c20e5d$118017f0$c866010a@valor> Content-Type: multipart/alternative; boundary="------------030001070407070402090108" Sender: owner-obm-l@sucuri.mat.puc-rio.br Precedence: bulk Reply-To: obm-l@mat.puc-rio.br --------------030001070407070402090108 Content-Type: text/plain; charset=ISO-8859-1; format=flowed Content-Transfer-Encoding: 8bit 1) O que voce escreveu nao esta correto. 2) Essas integrais se calculam com auxilio de (senx)^2 = (1-cos2x)/2, que reduz o "grau" do integrando ah metade. 3) Pode-se tambem fazer uma formula de recorrencia In= integral de (senx)^2n dx = integral de (senx)^(n-1) * senxdx Fazendo integraçao por partes, I(n) = (senx)^(n-1) * (-cosx) - integral de (-cosx)* (n-1)*(senx)^(n-2)*cosx dx= = (senx)^(n-1) * (-cosx) +(n-1) integral de(senx)^(n-2) * (cosx)^2 dx= = (senx)^(n-1) * (-cosx) +(n-1) integral de(senx)^(n-2) dx -(n-1) integral de (senx)^n dx nI(n) = (senx)^(n-1) * (-cosx) + (n-1) I(n-2) 4) No livro de calculo do Thomas voce encontra isso. leo wrote: > Eu tbm sou novo na lista e tbm tenho duvidas em integrais . Para fazer a >integral abaixo seria algo parecido com isso para uma caso genérico: > > Int senx^2n = (-cosx^2n+1)/2n+1 ? > > Alguem poderia me indicar bons livros sobre o tema? E mais uma duvida : >Onde esbarram os esforços de matematicos em resolver as equações de >navier-strokes? > > []s > >----- Original Message ----- >From: Diego Alonso Teixeira >To: ; >Sent: Friday, June 07, 2002 1:23 PM >Subject: RES: [obm-l] integral sem fazer a conta > > >>Quer ter seu próprio endereço na Internet? >>Garanta já o seu e ainda ganhe cinco e-mails personalizados. >>DomíniosBOL - http://dominios.bol.com.br >> >> >> >> >> >>ola,sou novo na lista, e estou aprendendo integral na faculdade, gostaria >> >de saber se existe algum truque para calcular a integral de sen x elevado a >um numero par grande. > >>-----Mensagem original----- >>De: Nicolau C. Saldanha [mailto:nicolau@sucuri.mat.puc-rio.br] >>Enviada: qui 6/6/2002 16:50 >>Para: obm-l@mat.puc-rio.br >>Cc: >>Assunto: Re: [obm-l] integral sem fazer a conta >> >> >> >>On Wed, Jun 05, 2002 at 08:27:19PM -0300, Augusto César Morgado wrote: >> >>>Eu, e creio que muitos outros, quero manifestar minha admiraçao por quem >>>consegue entender alguma coisa escrita em tao exotica notaçao. >>>Morgado >>> >>Esta notação chama-se TeX e não é nada exótica na comunidade matemática. >>Mesmo assim, acho que devemos tentar usar notações mais autoexplicativas. >>E principalmente devemos evitar uma notação que para alguns membros da >>lista é "exótica" quando há alternativas óbvias: pq, por exemplo, >>escrever \frac{du}{u^2 + (1-x^2)/x^2} ao invés de du/(u^2 + (1-x^2)/x^2)? >>A idéia aqui é que a lista seja lida e não TeXada ou processada com algum >>outro programa. >> >>Obrigado, []s, N. >> >>========================================================================= >>Instruções para entrar na lista, sair da lista e usar a lista em >>http://www.mat.puc-rio.br/~nicolau/olimp/obm-l.html >>O administrador desta lista é >>========================================================================= >>ola,sou novo na lista, e estou aprendendo integral na faculdade, gostaria >> >de saber se existe algum truque para calcular a integral de sen x elevado a >um numero par grande. > >>========================================================================= >>Instruções para entrar na lista, sair da lista e usar a lista em >>http://www.mat.puc-rio.br/~nicolau/olimp/obm-l.html >>O administrador desta lista é >>========================================================================= >> > >========================================================================= >Instruções para entrar na lista, sair da lista e usar a lista em >http://www.mat.puc-rio.br/~nicolau/olimp/obm-l.html >O administrador desta lista é >========================================================================= > > --------------030001070407070402090108 Content-Type: text/html; charset=us-ascii Content-Transfer-Encoding: 7bit 1) O que voce escreveu nao esta correto.
2) Essas integrais se calculam com auxilio de (senx)^2 = (1-cos2x)/2, que reduz o "grau" do integrando ah metade.
3) Pode-se tambem fazer uma formula de recorrencia
In= integral de (senx)^2n dx = integral de (senx)^(n-1) * senxdx
Fazendo integraçao por partes,
I(n) = (senx)^(n-1) * (-cosx) - integral de (-cosx)* (n-1)*(senx)^(n-2)*cosx dx=
= (senx)^(n-1) * (-cosx) +(n-1) integral de(senx)^(n-2) * (cosx)^2 dx=
= (senx)^(n-1) * (-cosx) +(n-1) integral de(senx)^(n-2) dx -(n-1) integral de (senx)^n dx
nI(n) = (senx)^(n-1) * (-cosx) + (n-1) I(n-2)
4) No livro de calculo do Thomas voce encontra isso.

leo wrote:
    Eu tbm sou novo na lista e tbm tenho duvidas em integrais . Para fazer a
integral abaixo seria algo parecido com isso para uma caso genérico:

Int senx^2n = (-cosx^2n+1)/2n+1 ?

Alguem poderia me indicar bons livros sobre o tema? E mais uma duvida :
Onde esbarram os esforços de matematicos em resolver as equações de
navier-strokes?

[]s

----- Original Message -----
From: Diego Alonso Teixeira <diego.teixeira@poli.usp.br>
To: <obm-l@mat.puc-rio.br>; <obm-l@mat.puc-rio.br>
Sent: Friday, June 07, 2002 1:23 PM
Subject: RES: [obm-l] integral sem fazer a conta


Quer ter seu próprio endereço na Internet?
Garanta já o seu e ainda ganhe cinco e-mails personalizados.
DomíniosBOL - http://dominios.bol.com.br





ola,sou novo na lista, e estou aprendendo integral na faculdade, gostaria
de saber se existe algum truque para calcular a integral de sen x elevado a
um numero par grande.
-----Mensagem original-----
De: Nicolau C. Saldanha [mailto:nicolau@sucuri.mat.puc-rio.br]
Enviada: qui 6/6/2002 16:50
Para: obm-l@mat.puc-rio.br
Cc:
Assunto: Re: [obm-l] integral sem fazer a conta



On Wed, Jun 05, 2002 at 08:27:19PM -0300, Augusto César Morgado wrote:
Eu, e creio que muitos outros, quero manifestar minha admiraçao por quem
consegue entender alguma coisa escrita em tao exotica notaçao.
Morgado
Esta notação chama-se TeX e não é nada exótica na comunidade matemática.
Mesmo assim, acho que devemos tentar usar notações mais autoexplicativas.
E principalmente devemos evitar uma notação que para alguns membros da
lista é "exótica" quando há alternativas óbvias: pq, por exemplo,
escrever \frac{du}{u^2 + (1-x^2)/x^2} ao invés de du/(u^2 + (1-x^2)/x^2)?
A idéia aqui é que a lista seja lida e não TeXada ou processada com algum
outro programa.

Obrigado, []s, N.

=========================================================================
Instruções para entrar na lista, sair da lista e usar a lista em
http://www.mat.puc-rio.br/~nicolau/olimp/obm-l.html
O administrador desta lista &eacut e; <nicolau@mat.puc-rio.br>
=========================================================================
ola,sou novo na lista, e estou aprendendo integral na faculdade, gostaria
de saber se existe algum truque para calcular a integral de sen x elevado a
um numero par grande.
=========================================================================
Instruções para entrar na lista, sair da lista e usar a lista em
http://www.mat.puc-rio.br/~nicolau/olimp/obm-l.html
O administrador desta lista é <nicolau@mat.puc-rio.br>
=========================================================================


=========================================================================
Instruções para entrar na lista, sair da lista e usar a lista em
http://www.mat.puc-rio.br/~nicolau/olimp/obm-l.html
O administrador desta lista é <nicolau@mat.puc-rio.br>
=========================================================================



--------------030001070407070402090108-- ========================================================================= Instruções para entrar na lista, sair da lista e usar a lista em http://www.mat.puc-rio.br/~nicolau/olimp/obm-l.html O administrador desta lista é ========================================================================= From owner-obm-l@sucuri.mat.puc-rio.br Fri Jun 7 19:15:26 2002 Return-Path: Received: (from majordom@localhost) by sucuri.mat.puc-rio.br (8.9.3/8.9.3) id TAA05742 for obm-l-list; Fri, 7 Jun 2002 19:14:09 -0300 Received: from sr1.terra.com.br (sr1.terra.com.br [200.176.3.16]) by sucuri.mat.puc-rio.br (8.9.3/8.9.3) with ESMTP id TAA05737 for ; Fri, 7 Jun 2002 19:14:06 -0300 Received: from pavuna.terra.com.br (pavuna.terra.com.br [200.176.3.41]) by sr1.terra.com.br (Postfix) with ESMTP id CA9D56ECF1 for ; Fri, 7 Jun 2002 19:13:39 -0300 (EST) Received: from nt (RJ231080.user.veloxzone.com.br [200.165.231.80]) (authenticated user ensr) by pavuna.terra.com.br (Postfix) with ESMTP id 4311668091 for ; Fri, 7 Jun 2002 19:13:39 -0300 (EST) Message-ID: <00f301c20e70$4d2cc1a0$5400a8c0@ensrbr> From: "Luis Lopes" To: References: <004001c20e2b$d1cdc020$53f4a0c8@rodrigo> <005701c20e6b$5d422260$f1469ec8@cabru> Subject: Re: [obm-l] Desafio Date: Fri, 7 Jun 2002 19:11:40 -0300 MIME-Version: 1.0 Content-Type: text/plain; charset="iso-8859-1" Content-Transfer-Encoding: 8bit X-Priority: 3 X-MSMail-Priority: Normal X-Mailer: Microsoft Outlook Express 5.00.2615.200 X-MimeOLE: Produced By Microsoft MimeOLE V5.00.2615.200 Sender: owner-obm-l@sucuri.mat.puc-rio.br Precedence: bulk Reply-To: obm-l@mat.puc-rio.br Sauda,c~oes, Para ser coerente com o que acabo de escrever: a lista deve prestar um serviço nesse tipo de dúvida também. sqrt é a forma abreviada de "square root" ou raiz quadrada. No TeX, \sqrt{x} é a raiz quadrada de x. Assim c = \sqrt{a^2+b^2} é a hipotenusa de um triângulo retângulo de catetos a e b, por exemplo. []'s Luís -----Mensagem Original----- De: Bruno Para: Enviada em: sexta-feira, 7 de junho de 2002 18:36 Assunto: Re: [obm-l] Desafio > obrigado pela solução mas o que é: > "sqrt"? > > abraços, > Bruno > ----- Original Message ----- > From: "Rodrigo Villard Milet" > To: > Sent: Friday, June 07, 2002 11:01 AM > Subject: Re: [obm-l] Desafio > > > > Use que 1+a(i) >=2sqrt[a(i)]. Fazendo o produto dessas n equações, temos > que > > P >=2^n * sqrt[ produto a(i) ] = 2^n * 2 = 2^(n+1). RESPOSTA : C. > > Villard ========================================================================= Instruções para entrar na lista, sair da lista e usar a lista em http://www.mat.puc-rio.br/~nicolau/olimp/obm-l.html O administrador desta lista é ========================================================================= From owner-obm-l@sucuri.mat.puc-rio.br Fri Jun 7 19:26:02 2002 Return-Path: Received: (from majordom@localhost) by sucuri.mat.puc-rio.br (8.9.3/8.9.3) id TAA06399 for obm-l-list; Fri, 7 Jun 2002 19:24:43 -0300 Received: from calhau.terra.com.br (calhau.terra.com.br [200.176.3.20]) by sucuri.mat.puc-rio.br (8.9.3/8.9.3) with ESMTP id TAA06386 for ; Fri, 7 Jun 2002 19:24:39 -0300 Received: from smtp4-poa.terra.com.br (smtp4-poa.terra.com.br [200.176.3.35]) by calhau.terra.com.br (Postfix) with ESMTP id 5BB3447128 for ; Fri, 7 Jun 2002 22:24:12 +0000 (GMT) Received: from nt (RJ231080.user.veloxzone.com.br [200.165.231.80]) (authenticated user ensr) by smtp4-poa.terra.com.br (Postfix) with ESMTP id AA91EAC59B for ; Fri, 7 Jun 2002 19:24:11 -0300 (EST) Message-ID: <010701c20e71$c6312cc0$5400a8c0@ensrbr> From: "Luis Lopes" To: References: <200206062125.g56LPdOW010853@trex.centroin.com.br> <006701c20dc4$753a47a0$8710dcc8@jf> Subject: [obm-l] =?iso-8859-1?Q?Re:_=5Bobm-l=5D_manual_de_indu=E7=E3o_matem=E1tica?= Date: Fri, 7 Jun 2002 19:22:13 -0300 MIME-Version: 1.0 Content-Type: text/plain; charset="iso-8859-1" Content-Transfer-Encoding: 8bit X-Priority: 3 X-MSMail-Priority: Normal X-Mailer: Microsoft Outlook Express 5.00.2615.200 X-MimeOLE: Produced By Microsoft MimeOLE V5.00.2615.200 Sender: owner-obm-l@sucuri.mat.puc-rio.br Precedence: bulk Reply-To: obm-l@mat.puc-rio.br Sauda,c~oes, Foi publicado pela Interciência, Rio de Janeiro. No site www.escolademestres.com/qedtexte há uma amostra do livro num arquivo em pdf. Se quiser comprá-lo e as livrarias listadas lá não forem convenientes para você, escreva diretamente pra mim que poderei lhe dar o nome de outras. []'s Luís -----Mensagem Original----- De: Jose Francisco Guimaraes Costa Para: Enviada em: quinta-feira, 6 de junho de 2002 22:33 Assunto: [obm-l] manual de indução matemática > Qual a editora? Onde pode ser comprado? > > JF > > ----- Original Message ----- > From: "Augusto Cesar de Oliveira Morgado" > To: > Sent: Thursday, June 06, 2002 6:25 PM > Subject: Re: [obm-l] Livros de Inducao / Analitica > > > > O Luis Lopes escreveu um livro otimo chamado Manual de Induçao Matemática. > > Morgado > > > > > > Em Thu, 6 Jun 2002 16:22:04 -0300 (ART), Ricardo Miranda > disse: > > > > > Olá amigos. > > > > > > Alguem poderia me indicar um livro/site que explique o método de Inducao > > > Matematica? Quero um livro que nao seja o "Matematica Elementar". Estou > > > com duvidas principalmente (mas nao somente) em provar que inequacoes > sao > > > verdadeiras. > > > > > > Abusando, poderiam me falar um livro de Geometria Analitica (com > > > tratamento de vetores) diferente do do Elon? > > > > > > > > > ===== > > > []s > > > Ricardo Miranda > > > Matematica - UFV > > > ricardomirandabr@yahoo.com.br > > > http://rm2.hpg.ig.com.br/ > > > ========================================================================= Instruções para entrar na lista, sair da lista e usar a lista em http://www.mat.puc-rio.br/~nicolau/olimp/obm-l.html O administrador desta lista é ========================================================================= From owner-obm-l@sucuri.mat.puc-rio.br Fri Jun 7 19:27:23 2002 Return-Path: Received: (from majordom@localhost) by sucuri.mat.puc-rio.br (8.9.3/8.9.3) id TAA06586 for obm-l-list; Fri, 7 Jun 2002 19:27:17 -0300 Received: from sporus.bol.com.br (sporus.bol.com.br [200.221.24.23]) by sucuri.mat.puc-rio.br (8.9.3/8.9.3) with ESMTP id TAA06573 for ; Fri, 7 Jun 2002 19:27:14 -0300 Received: from bol.com.br (200.221.24.84) by sporus.bol.com.br (5.1.071) id 3CFFBEE50004EDD2 for obm-l@mat.puc-rio.br; Fri, 7 Jun 2002 19:26:22 -0300 Date: Fri, 7 Jun 2002 19:26:22 -0300 Message-Id: Subject: Re: [obm-l] pai e filho MIME-Version: 1.0 Content-Type: text/plain;charset="iso-8859-1" From: "aleixocarvalho" To: obm-l@mat.puc-rio.br X-XaM3-API-Version: 2.4.3.4.4 X-SenderIP: 143.106.2.50 Content-Transfer-Encoding: 8bit X-MIME-Autoconverted: from quoted-printable to 8bit by sucuri.mat.puc-rio.br id TAA06574 Sender: owner-obm-l@sucuri.mat.puc-rio.br Precedence: bulk Reply-To: obm-l@mat.puc-rio.br sinto muito, tente outra solucao,pois esta esta errada.Como o pai vence 17 partidas e o filho tem 3 vezes as fichas do pai seu raciocinio esta correto, mas vc errou contas no final rafael > Eu acredito que essa resolução é boa, mas provavelmente tem outras(sem usar > muitas letras) que eu ainda não consegui pensar: > Sendo: n=número de jogos > p=fichas do pai > f=fichas do filho > v=vitórias do pai > v'=vitórias do filho > > as fichas do pai é dada pela equação: p=100 +4v -6v' > as fichas do filho é dada pela equação: f=100 -4v +6v' > e o número de jogos é dado por: n= v + v' > > como após vinte jogos: > n=20 e como n= v + v' então v' = 20 - v > > e f=3p > portanto: 100-4v+6v'=3(100+4v-6v') > 16v - 24v' = 200 > dividindo tudo por 8 temos: 2v - 3v'=25 > ecomo v'= 20-v temos > v=17 > Resposta: o pai venceu 17 jogos. > > > > ----- Original Message ----- > From: "aleixocarvalho" > To: > Sent: Wednesday, June 05, 2002 10:09 PM > Subject: [obm-l] pai e filho > > > > em um jogo de fichas,o pai sai com 100 fichas, assim > > como seu filho, quando o pai perde o filho ganha 6 > > fichas do pai, e quando o filho perde o pai ganha 4 > > fichas do pai.Após 20 jogos o filho tem 3 vezes as > > fichas do pai. Pergunta: quntos jogos o pai ganhou. > > > > > > gostaria de saber a solucao, com resolucao > > > > > > rafael > > > > > > _________________________________________________________ _________________ > > Quer ter seu próprio endereço na Internet? > > Garanta já o seu e ainda ganhe cinco e-mails personalizados. > > DomíniosBOL - http://dominios.bol.com.br > > > > > > ========================================================= ================ > > Instruções para entrar na lista, sair da lista e usar a lista em > > http://www.mat.puc-rio.br/~nicolau/olimp/obm-l.html > > O administrador desta lista é > > ========================================================= ================ > > > > ========================================================= ================ > Instruções para entrar na lista, sair da lista e usar a lista em > http://www.mat.puc-rio.br/~nicolau/olimp/obm-l.html > O administrador desta lista é > ========================================================= ================ > __________________________________________________________________________ Quer ter seu próprio endereço na Internet? Garanta já o seu e ainda ganhe cinco e-mails personalizados. DomíniosBOL - http://dominios.bol.com.br ========================================================================= Instruções para entrar na lista, sair da lista e usar a lista em http://www.mat.puc-rio.br/~nicolau/olimp/obm-l.html O administrador desta lista é ========================================================================= From owner-obm-l@sucuri.mat.puc-rio.br Fri Jun 7 21:39:19 2002 Return-Path: Received: (from majordom@localhost) by sucuri.mat.puc-rio.br (8.9.3/8.9.3) id VAA09493 for obm-l-list; Fri, 7 Jun 2002 21:39:09 -0300 Received: from Euler.impa.br (euler.impa.br [147.65.1.3]) by sucuri.mat.puc-rio.br (8.9.3/8.9.3) with ESMTP id VAA09489 for ; Fri, 7 Jun 2002 21:39:06 -0300 Received: from [147.65.11.3] (dial03.impa.br [147.65.11.3]) by Euler.impa.br (8.11.6/8.11.6) with ESMTP id g580ccf01655 for ; Fri, 7 Jun 2002 21:38:38 -0300 (EST) Message-Id: <200206080038.g580ccf01655@Euler.impa.br> X-Mailer: Microsoft Outlook Express Macintosh Edition - 4.5 (0410) Date: Sun, 09 Jun 2002 21:36:25 -0300 Subject: Re: [obm-l] plana ajuda por favor urgente From: "Eduardo Wagner" To: obm-l@mat.puc-rio.br Mime-version: 1.0 X-Priority: 3 Content-type: text/plain; charset="ISO-8859-1" Content-Transfer-Encoding: 8bit X-MIME-Autoconverted: from quoted-printable to 8bit by sucuri.mat.puc-rio.br id VAA09490 Sender: owner-obm-l@sucuri.mat.puc-rio.br Precedence: bulk Reply-To: obm-l@mat.puc-rio.br ---------- >From: Augusto César Morgado >To: obm-l@mat.puc-rio.br >Subject: Re: [obm-l] plana ajuda por favor urgente >Date: Fri, Jun 7, 2002, 6:48 PM > > Nao tem maximo demais no problema 3? > Claro que tem. A pergunta deve ser: qual eh a area do triangulo sabendo que o angulo ABC tem a maior medida possivel. > >> >> >> >>> Olá,gostaria da ajuda de vcs nas seguintes questões: >>> 1.Os pontos P1,P2,... estão sobre uma circunferencia e são tais que o >>> arco q >>> une cada ponto ao seguinte mede 35º.O menor valor de n>1 tal que Pn >>> coincide >>> com P1 é? >>> >>> 2.Para cada ponto pertencente ao interior e aos lados de um triangulo >>> acutangulo ABC,considere a soma das suas distancias aos 3 lados do >>> triangulo.O valor maximo desta soma é? >>> >>> 3.No triangulo ABC,AB=5 e BC=6.Qual a area maxima do triangulo ABC >>> ,sabendo >>> q o angulo C tem a maior medida possivel? >>> >>> Muito obrigado! >>> té + >>> Adherbal >> >> >> >> _________________________________________________________________ >> Chegou o novo MSN Explorer. Instale já. É gratuito: >> http://explorer.msn.com.br >> >> ========================================================================= >> Instruções para entrar na lista, sair da lista e usar a lista em >> http://www.mat.puc-rio.br/~nicolau/olimp/obm-l.html >> O administrador desta lista é >> ========================================================================= >> >> > > > ========================================================================= > Instruções para entrar na lista, sair da lista e usar a lista em > http://www.mat.puc-rio.br/~nicolau/olimp/obm-l.html > O administrador desta lista é > ========================================================================= ========================================================================= Instruções para entrar na lista, sair da lista e usar a lista em http://www.mat.puc-rio.br/~nicolau/olimp/obm-l.html O administrador desta lista é ========================================================================= From owner-obm-l@sucuri.mat.puc-rio.br Fri Jun 7 21:45:01 2002 Return-Path: Received: (from majordom@localhost) by sucuri.mat.puc-rio.br (8.9.3/8.9.3) id VAA09581 for obm-l-list; Fri, 7 Jun 2002 21:44:59 -0300 Received: from hotmail.com (f127.pav1.hotmail.com [64.4.31.127]) by sucuri.mat.puc-rio.br (8.9.3/8.9.3) with ESMTP id VAA09577 for ; Fri, 7 Jun 2002 21:44:56 -0300 Received: from mail pickup service by hotmail.com with Microsoft SMTPSVC; Fri, 7 Jun 2002 17:44:29 -0700 Received: from 200.199.179.22 by pv1fd.pav1.hotmail.msn.com with HTTP; Sat, 08 Jun 2002 00:44:28 GMT X-Originating-IP: [200.199.179.22] From: "Adherbal Rocha Filho" To: obm-l@mat.puc-rio.br Subject: Re: [obm-l] plana ajuda por favor urgente Date: Sat, 08 Jun 2002 00:44:28 +0000 Mime-Version: 1.0 Content-Type: text/plain; charset=iso-8859-1; format=flowed Message-ID: X-OriginalArrivalTime: 08 Jun 2002 00:44:29.0048 (UTC) FILETIME=[A5896780:01C20E85] Sender: owner-obm-l@sucuri.mat.puc-rio.br Precedence: bulk Reply-To: obm-l@mat.puc-rio.br > >Nao tem maximo demais no problema 3? > Tem razão :P Segue o enunciado correto( e obrigado pela ajuda no primeiro...) >>>2.Para cada ponto pertencente ao interior e aos lados de um triangulo >>>acutangulo ABC,considere a soma das suas distancias aos 3 lados do >>>triangulo.O valor maximo desta soma é? >>> 3.No triangulo ABC,AB=5 e BC=6.Qual a area do triangulo ABC ,sabendo q o angulo C tem a maior medida possivel? _________________________________________________________________ Envie e receba emails com o Hotmail no seu dispositivo móvel: http://mobile.msn.com ========================================================================= Instruções para entrar na lista, sair da lista e usar a lista em http://www.mat.puc-rio.br/~nicolau/olimp/obm-l.html O administrador desta lista é ========================================================================= From owner-obm-l@sucuri.mat.puc-rio.br Sat Jun 8 01:00:05 2002 Return-Path: Received: (from majordom@localhost) by sucuri.mat.puc-rio.br (8.9.3/8.9.3) id AAA11771 for obm-l-list; Sat, 8 Jun 2002 00:58:13 -0300 Received: from mail.gmx.net (mail.gmx.net [213.165.64.20]) by sucuri.mat.puc-rio.br (8.9.3/8.9.3) with SMTP id AAA11767 for ; Sat, 8 Jun 2002 00:58:09 -0300 Received: (qmail 3630 invoked by uid 0); 8 Jun 2002 03:57:40 -0000 Received: from unknown (HELO gomes) (200.216.104.48) by mail.gmx.net (mp015-rz3) with SMTP; 8 Jun 2002 03:57:40 -0000 Date: Sat, 8 Jun 2002 00:59:08 -0300 From: Igor GomeZZ X-Mailer: The Bat! (v1.60c) Organization: -- X-Priority: 3 (Normal) Message-ID: <5748448395.20020608005908@gmx.net> To: Luiz Antonio Ponce Alonso Subject: Re: [obm-l] ajuda: colegio naval In-Reply-To: <3D00185D.E2826B1@terra.com.br> References: <3D00185D.E2826B1@terra.com.br> MIME-Version: 1.0 Content-Type: text/plain; charset=ISO-8859-1 Content-Transfer-Encoding: 8bit Sender: owner-obm-l@sucuri.mat.puc-rio.br Precedence: bulk Reply-To: obm-l@mat.puc-rio.br Em 6/6/2002, 23:20, Luiz (lponce@terra.com.br) disse: > Caros amigos, > Estou precisando de provas de matemática do colegio naval para ajudar na > preparação > do filho de meu amigo. Caso alguém tenha alguma ou um site em que posso > adquiri-las > ficaria muito grato. www.estudemais.com.br tem muitas provas, inclusive as do Colégio Naval, gabaritadas... > Desde já fico agradecido por qualquer ajuda futura > Um abraço Flws! Fui! ####### Igor GomeZZ ######## UIN: 29249895 Vitória, Espírito Santo, Brasil Criação: 8/6/2002 (00:57) #################################### Pare para pensar: Algo é só impossível até que alguém duvide e acabe provando o contrário. (Albert Einstein) #################################### ========================================================================= Instruções para entrar na lista, sair da lista e usar a lista em http://www.mat.puc-rio.br/~nicolau/olimp/obm-l.html O administrador desta lista é ========================================================================= From owner-obm-l@sucuri.mat.puc-rio.br Sat Jun 8 02:40:14 2002 Return-Path: Received: (from majordom@localhost) by sucuri.mat.puc-rio.br (8.9.3/8.9.3) id CAA12679 for obm-l-list; Sat, 8 Jun 2002 02:40:02 -0300 Received: from web10103.mail.yahoo.com (web10103.mail.yahoo.com [216.136.130.53]) by sucuri.mat.puc-rio.br (8.9.3/8.9.3) with SMTP id CAA12672 for ; Sat, 8 Jun 2002 02:39:58 -0300 Message-ID: <20020608053931.50203.qmail@web10103.mail.yahoo.com> Received: from [200.151.204.226] by web10103.mail.yahoo.com via HTTP; Fri, 07 Jun 2002 22:39:31 PDT Date: Fri, 7 Jun 2002 22:39:31 -0700 (PDT) From: Rafael WC Subject: [obm-l] x² + y² To: OBM MIME-Version: 1.0 Content-Type: text/plain; charset=us-ascii Sender: owner-obm-l@sucuri.mat.puc-rio.br Precedence: bulk Reply-To: obm-l@mat.puc-rio.br Olá Pessoal! Esse exercício eu resolvi, mas não foi do jeito que eu queria. Eu acabei fazendo o jeito convencional de isolar o x da primeira equação e colocar na segunda. Mas deve haver um jeito de se chegar a resposta manipulando as equações dadas sem que precisemos encontrar os valores de x e y. Vejam se vocês conseguem. Sejam x e y inteiros positivos tais que: xy + x + y = 71 x²y + xy² = 880. Determine x² + y². Talvez facilite saberem que se trocarmos as variáveis, escrevendo x no lugar de y as equações não mudam. Isso quer dizer que as respostas são (x, y) e (y, x). Além disso, os valores que encontrei foram x = 11 e y = 5 (ou vice-versa). Um abraço, Rafael. ===== Rafael Werneck Cinoto ICQ# 107011599 rwcinoto@yahoo.com rafael.caixa@gov.com.br matduvidas@yahoo.com.br http://www.rwcinoto.hpg.com.br/ __________________________________________________ Do You Yahoo!? Yahoo! - Official partner of 2002 FIFA World Cup http://fifaworldcup.yahoo.com ========================================================================= Instruções para entrar na lista, sair da lista e usar a lista em http://www.mat.puc-rio.br/~nicolau/olimp/obm-l.html O administrador desta lista é ========================================================================= From owner-obm-l@sucuri.mat.puc-rio.br Sat Jun 8 10:41:57 2002 Return-Path: Received: (from majordom@localhost) by sucuri.mat.puc-rio.br (8.9.3/8.9.3) id KAA15127 for obm-l-list; Sat, 8 Jun 2002 10:36:29 -0300 Received: from gorgo.centroin.com.br ([200.225.63.128]) by sucuri.mat.puc-rio.br (8.9.3/8.9.3) with ESMTP id KAA15123 for ; Sat, 8 Jun 2002 10:36:27 -0300 Received: from centroin.com.br (du138c.rjo.centroin.com.br [200.225.58.138]) (authenticated bits=0) by gorgo.centroin.com.br (8.12.2/8.12.1) with ESMTP id g58DZvko002711 for ; Sat, 8 Jun 2002 10:35:57 -0300 (BRT) Message-ID: <3D020889.806@centroin.com.br> Date: Sat, 08 Jun 2002 10:37:13 -0300 From: Augusto =?ISO-8859-1?Q?C=E9sar?= Morgado User-Agent: Mozilla/5.0 (Windows; U; Win98; en-US; rv:0.9.4.1) Gecko/20020508 Netscape6/6.2.3 X-Accept-Language: en-us MIME-Version: 1.0 To: obm-l@mat.puc-rio.br Subject: Re: [obm-l] =?ISO-8859-1?Q?x=B2?= + =?ISO-8859-1?Q?y=B2?= References: <20020608053931.50203.qmail@web10103.mail.yahoo.com> Content-Type: text/plain; charset=ISO-8859-1; format=flowed Content-Transfer-Encoding: 8bit Sender: owner-obm-l@sucuri.mat.puc-rio.br Precedence: bulk Reply-To: obm-l@mat.puc-rio.br O usual nesses casos eh por S=x+y e P=xy Ficaria S+P=71 e SP=880. ...... Rafael WC wrote: >Olá Pessoal! > >Esse exercício eu resolvi, mas não foi do jeito que eu >queria. Eu acabei fazendo o jeito convencional de >isolar o x da primeira equação e colocar na segunda. >Mas deve haver um jeito de se chegar a resposta >manipulando as equações dadas sem que precisemos >encontrar os valores de x e y. Vejam se vocês >conseguem. > >Sejam x e y inteiros positivos tais que: >xy + x + y = 71 >x²y + xy² = 880. > >Determine x² + y². > >Talvez facilite saberem que se trocarmos as variáveis, >escrevendo x no lugar de y as equações não mudam. Isso >quer dizer que as respostas são (x, y) e (y, x). Além >disso, os valores que encontrei foram x = 11 e y = 5 >(ou vice-versa). > >Um abraço, > >Rafael. > >===== >Rafael Werneck Cinoto > ICQ# 107011599 > rwcinoto@yahoo.com > rafael.caixa@gov.com.br > matduvidas@yahoo.com.br >http://www.rwcinoto.hpg.com.br/ > >__________________________________________________ >Do You Yahoo!? >Yahoo! - Official partner of 2002 FIFA World Cup >http://fifaworldcup.yahoo.com >========================================================================= >Instruções para entrar na lista, sair da lista e usar a lista em >http://www.mat.puc-rio.br/~nicolau/olimp/obm-l.html >O administrador desta lista é >========================================================================= > > ========================================================================= Instruções para entrar na lista, sair da lista e usar a lista em http://www.mat.puc-rio.br/~nicolau/olimp/obm-l.html O administrador desta lista é ========================================================================= From owner-obm-l@sucuri.mat.puc-rio.br Sat Jun 8 17:47:00 2002 Return-Path: Received: (from majordom@localhost) by sucuri.mat.puc-rio.br (8.9.3/8.9.3) id RAA00816 for obm-l-list; Sat, 8 Jun 2002 17:43:39 -0300 Received: from caravelas.terra.com.br (caravelas.terra.com.br [200.176.3.26]) by sucuri.mat.puc-rio.br (8.9.3/8.9.3) with ESMTP id RAA00813 for ; Sat, 8 Jun 2002 17:43:36 -0300 Received: from candeias.terra.com.br (candeias.terra.com.br [200.176.3.18]) by caravelas.terra.com.br (Postfix) with ESMTP id 760D2147409 for ; Sat, 8 Jun 2002 16:39:00 -0300 (EST) Received: from smtp4-poa.terra.com.br (smtp4-poa.terra.com.br [200.176.3.35]) by candeias.terra.com.br (Postfix) with ESMTP id 675D443D19 for ; Sat, 8 Jun 2002 16:39:00 -0300 (EST) Received: from stabel (dl-nas1-poa-C89A0027.p001.terra.com.br [200.154.0.39]) (authenticated user dudasta) by smtp4-poa.terra.com.br (Postfix) with ESMTP id 9EF21AC59F for ; Sat, 8 Jun 2002 16:38:58 -0300 (EST) Message-ID: <004301c20f24$1f4a2f70$27009ac8@stabel> From: "Eduardo Casagrande Stabel" To: References: <20020608053931.50203.qmail@web10103.mail.yahoo.com> <3D020889.806@centroin.com.br> Subject: [obm-l] =?iso-8859-1?Q?Re:_=5Bobm-l=5D_x=B2_+_y=B2?= Date: Sat, 8 Jun 2002 16:38:51 -0300 MIME-Version: 1.0 Content-Type: text/plain; charset="iso-8859-1" Content-Transfer-Encoding: 8bit X-Priority: 3 X-MSMail-Priority: Normal X-Mailer: Microsoft Outlook Express 6.00.2600.0000 X-MIMEOLE: Produced By Microsoft MimeOLE V6.00.2600.0000 Sender: owner-obm-l@sucuri.mat.puc-rio.br Precedence: bulk Reply-To: obm-l@mat.puc-rio.br Ola pessoal! Se temos dois números, x e y, é conhecida sua soma S=x + y e seu produto P=xy, então podemos determinar os dois números resolvendo uma equação de segundo grau. Repare que y = S - x e daí substituindo x em P temos P = x(S - x) P - Sx + x^2 = 0 x^2 - Sx + P = 0 O mesmo raciocínio para x = S - y e aí temos y^2 - Sy + P = 0 Essa equação polinomial "bonita" se repete em casos muito mais gerais. Por exemplo, se três números, x, y e z, são tais que se conhece sua soma S = x + y + z, a soma de pares Q = xy + xz + yz e seu produto P = xyz dá pra mostrar que (não é difícil): x^3 - Sx^2 + Qx - P = 0 E a mesma expressão vale para y e z no lugar de x. Esse procedimento também fornece uma fatoração desse polinômio k^3 - Sk^2 + Qk - P = (k - x)(k - y)(k - z) E esse resultado pode ser aumentado para uma grande quantidade de números. Talvez esse assunto seja de conhecimento de muita gente. Mas acho que alguns da lista não conhecem muita coisa sobre ele. Um abraço! Eduardo Casagrande Stabel. Porto Alegre, RS. From: "Augusto César Morgado" > O usual nesses casos eh por S=x+y e P=xy > Ficaria S+P=71 e SP=880. > ...... > > Rafael WC wrote: > > >Olá Pessoal! > > > >Esse exercício eu resolvi, mas não foi do jeito que eu > >queria. Eu acabei fazendo o jeito convencional de > >isolar o x da primeira equação e colocar na segunda. > >Mas deve haver um jeito de se chegar a resposta > >manipulando as equações dadas sem que precisemos > >encontrar os valores de x e y. Vejam se vocês > >conseguem. > > > >Sejam x e y inteiros positivos tais que: > >xy + x + y = 71 > >x²y + xy² = 880. > > > >Determine x² + y². > > > >Talvez facilite saberem que se trocarmos as variáveis, > >escrevendo x no lugar de y as equações não mudam. Isso > >quer dizer que as respostas são (x, y) e (y, x). Além > >disso, os valores que encontrei foram x = 11 e y = 5 > >(ou vice-versa). > > > >Um abraço, > > > >Rafael. > > > >===== > >Rafael Werneck Cinoto > > ICQ# 107011599 > > rwcinoto@yahoo.com > > rafael.caixa@gov.com.br > > matduvidas@yahoo.com.br > >http://www.rwcinoto.hpg.com.br/ > > > >__________________________________________________ > >Do You Yahoo!? > >Yahoo! - Official partner of 2002 FIFA World Cup > >http://fifaworldcup.yahoo.com > >========================================================================= > >Instruções para entrar na lista, sair da lista e usar a lista em > >http://www.mat.puc-rio.br/~nicolau/olimp/obm-l.html > >O administrador desta lista é > >========================================================================= > > > > > > > ========================================================================= > Instruções para entrar na lista, sair da lista e usar a lista em > http://www.mat.puc-rio.br/~nicolau/olimp/obm-l.html > O administrador desta lista é > ========================================================================= > > ========================================================================= Instruções para entrar na lista, sair da lista e usar a lista em http://www.mat.puc-rio.br/~nicolau/olimp/obm-l.html O administrador desta lista é ========================================================================= From owner-obm-l@sucuri.mat.puc-rio.br Sat Jun 8 19:19:14 2002 Return-Path: Received: (from majordom@localhost) by sucuri.mat.puc-rio.br (8.9.3/8.9.3) id TAA02202 for obm-l-list; Sat, 8 Jun 2002 19:16:13 -0300 Received: from traven9.uol.com.br (traven9.uol.com.br [200.231.206.210]) by sucuri.mat.puc-rio.br (8.9.3/8.9.3) with ESMTP id TAA02199 for ; Sat, 8 Jun 2002 19:16:11 -0300 Received: from cabru ([200.158.70.90]) by traven9.uol.com.br (8.9.1/8.9.1) with SMTP id MAA14846 for ; Sat, 8 Jun 2002 12:50:34 -0300 (BRT) Message-ID: <001501c20f04$f0b9f600$5a469ec8@cabru> From: "Bruno" To: References: Subject: Re: [obm-l] pai e filho Date: Sat, 8 Jun 2002 12:55:40 -0300 MIME-Version: 1.0 Content-Type: text/plain; charset="iso-8859-1" Content-Transfer-Encoding: 8bit X-Priority: 3 X-MSMail-Priority: Normal X-Mailer: Microsoft Outlook Express 5.50.4133.2400 X-MimeOLE: Produced By Microsoft MimeOLE V5.50.4133.2400 Sender: owner-obm-l@sucuri.mat.puc-rio.br Precedence: bulk Reply-To: obm-l@mat.puc-rio.br Vc tem razão eu errei as contas no final abraços Bruno ----- Original Message ----- From: "aleixocarvalho" To: Sent: Friday, June 07, 2002 7:26 PM Subject: Re: [obm-l] pai e filho > > sinto muito, tente outra solucao,pois esta esta > errada.Como o pai vence 17 partidas e o filho tem 3 > vezes as fichas do pai seu raciocinio esta correto, mas > vc errou contas no final > > rafael > > > > > > > > Eu acredito que essa resolução é boa, mas > provavelmente tem outras(sem usar > > muitas letras) que eu ainda não consegui pensar: > > Sendo: n=número de jogos > > p=fichas do pai > > f=fichas do filho > > v=vitórias do pai > > v'=vitórias do filho > > > > as fichas do pai é dada pela equação: p=100 +4v -6v' > > as fichas do filho é dada pela equação: f=100 -4v +6v' > > e o número de jogos é dado por: n= v + v' > > > > como após vinte jogos: > > n=20 e como n= v + v' então v' = 20 - v > > > > e f=3p > > portanto: 100-4v+6v'=3(100+4v-6v') > > 16v - 24v' = 200 > > dividindo tudo por 8 temos: 2v - 3v'=25 > > ecomo v'= 20-v temos > > v=17 > > Resposta: o pai venceu 17 jogos. > > > > > > > > ----- Original Message ----- > > From: "aleixocarvalho" > > To: > > Sent: Wednesday, June 05, 2002 10:09 PM > > Subject: [obm-l] pai e filho > > > > > > > em um jogo de fichas,o pai sai com 100 fichas, assim > > > como seu filho, quando o pai perde o filho ganha 6 > > > fichas do pai, e quando o filho perde o pai ganha 4 > > > fichas do pai.Após 20 jogos o filho tem 3 vezes as > > > fichas do pai. Pergunta: quntos jogos o pai ganhou. > > > > > > > > > gostaria de saber a solucao, com resolucao > > > > > > > > > rafael > > > > > > > > > > _________________________________________________________ > _________________ > > > Quer ter seu próprio endereço na Internet? > > > Garanta já o seu e ainda ganhe cinco e-mails > personalizados. > > > DomíniosBOL - http://dominios.bol.com.br > > > > > > > > > > ========================================================= > ================ > > > Instruções para entrar na lista, sair da lista e > usar a lista em > > > http://www.mat.puc-rio.br/~nicolau/olimp/obm-l.html > > > O administrador desta lista é rio.br> > > > > ========================================================= > ================ > > > > > > > > ========================================================= > ================ > > Instruções para entrar na lista, sair da lista e usar > a lista em > > http://www.mat.puc-rio.br/~nicolau/olimp/obm-l.html > > O administrador desta lista é > > > ========================================================= > ================ > > > > > __________________________________________________________________________ > Quer ter seu próprio endereço na Internet? > Garanta já o seu e ainda ganhe cinco e-mails personalizados. > DomíniosBOL - http://dominios.bol.com.br > > > ========================================================================= > Instruções para entrar na lista, sair da lista e usar a lista em > http://www.mat.puc-rio.br/~nicolau/olimp/obm-l.html > O administrador desta lista é > ========================================================================= > ========================================================================= Instruções para entrar na lista, sair da lista e usar a lista em http://www.mat.puc-rio.br/~nicolau/olimp/obm-l.html O administrador desta lista é ========================================================================= From owner-obm-l@sucuri.mat.puc-rio.br Sat Jun 8 19:19:54 2002 Return-Path: Received: (from majordom@localhost) by sucuri.mat.puc-rio.br (8.9.3/8.9.3) id TAA02208 for obm-l-list; Sat, 8 Jun 2002 19:17:24 -0300 Received: from smtp-6.ig.com.br (smtp-6.ig.com.br [200.226.132.155]) by sucuri.mat.puc-rio.br (8.9.3/8.9.3) with SMTP id TAA02205 for ; Sat, 8 Jun 2002 19:17:21 -0300 Received: (qmail 1532 invoked from network); 8 Jun 2002 14:57:41 -0000 Received: from shasta058195.ig.com.br (HELO jat) (200.151.58.195) by smtp-6.ig.com.br with SMTP; 8 Jun 2002 14:57:41 -0000 Message-ID: <000f01c20efc$d8db66c0$c33a97c8@jat> From: "Jose Augusto" To: Subject: [obm-l] Rafael Santos ;;;; Date: Sat, 8 Jun 2002 11:57:43 -0300 MIME-Version: 1.0 Content-Type: multipart/alternative; boundary="----=_NextPart_000_000C_01C20EE3.B2586DE0" X-Priority: 3 X-MSMail-Priority: Normal X-Mailer: Microsoft Outlook Express 5.50.4133.2400 X-MimeOLE: Produced By Microsoft MimeOLE V5.50.4133.2400 Sender: owner-obm-l@sucuri.mat.puc-rio.br Precedence: bulk Reply-To: obm-l@mat.puc-rio.br This is a multi-part message in MIME format. ------=_NextPart_000_000C_01C20EE3.B2586DE0 Content-Type: text/plain; charset="iso-8859-1" Content-Transfer-Encoding: quoted-printable Serve achando o XY e consequentemente X + Y e o quadrado deles ... = ???? ------=_NextPart_000_000C_01C20EE3.B2586DE0 Content-Type: text/html; charset="iso-8859-1" Content-Transfer-Encoding: quoted-printable
  Serve achando o XY e = consequentemente X +=20 Y  e o quadrado deles ... ????
------=_NextPart_000_000C_01C20EE3.B2586DE0-- ========================================================================= Instruções para entrar na lista, sair da lista e usar a lista em http://www.mat.puc-rio.br/~nicolau/olimp/obm-l.html O administrador desta lista é ========================================================================= From owner-obm-l@sucuri.mat.puc-rio.br Sat Jun 8 23:40:16 2002 Return-Path: Received: (from majordom@localhost) by sucuri.mat.puc-rio.br (8.9.3/8.9.3) id XAA03666 for obm-l-list; Sat, 8 Jun 2002 23:37:12 -0300 Received: from wool.vetor.com.br (200.160.244.7.metrored.net.br [200.160.244.7] (may be forged)) by sucuri.mat.puc-rio.br (8.9.3/8.9.3) with ESMTP id XAA03663 for ; Sat, 8 Jun 2002 23:37:10 -0300 Received: (from root@localhost) by wool.vetor.com.br (8.11.4/8.11.4) id g592OE429912 for obm-l@mat.puc-rio.br; Sat, 8 Jun 2002 23:24:14 -0300 Received: from rodrigo ([200.160.244.116]) by wool.vetor.com.br (8.11.4/8.11.4) with SMTP id g592OCT29903 for ; Sat, 8 Jun 2002 23:24:12 -0300 Message-ID: <002801c20f5c$aea12740$74f4a0c8@rodrigo> From: "Rodrigo Villard Milet" To: "Obm" Subject: [obm-l] =?iso-8859-1?B?UXVlc3TjbyA6IHPpcmllL3NlcXXqbmNpYQ==?= Date: Sat, 8 Jun 2002 23:23:44 -0300 MIME-Version: 1.0 Content-Type: multipart/alternative; boundary="----=_NextPart_000_0025_01C20F43.885FCB40" X-Priority: 3 X-MSMail-Priority: Normal X-Mailer: Microsoft Outlook Express 4.72.3110.5 X-MIMEOLE: Produced By Microsoft MimeOLE V4.72.3110.3 X-Virus-Scanned: by AMaViS perl-11 Sender: owner-obm-l@sucuri.mat.puc-rio.br Precedence: bulk Reply-To: obm-l@mat.puc-rio.br This is a multi-part message in MIME format. ------=_NextPart_000_0025_01C20F43.885FCB40 Content-Type: text/plain; charset="iso-8859-1" Content-Transfer-Encoding: quoted-printable Talvez a quest=E3o que estou enviando seja f=E1cil... mas quero ver se = algu=E9m d=E1 alguma solu=E7=E3o elegante pra ela... l=E1 vai : Sabe-se que somat=F3rio { a(n) } converge. Calcular lim = [(1/n)*somat=F3rio(k*a(k))], onde o somat=F3rio vai de 1 at=E9 n e o = limite =E9 qd n-> +oo. Abra=E7os, Villard ------=_NextPart_000_0025_01C20F43.885FCB40 Content-Type: text/html; charset="iso-8859-1" Content-Transfer-Encoding: quoted-printable
Talvez a questão = que estou=20 enviando seja fácil... mas quero ver se alguém dá = alguma=20 solução elegante pra ela... lá vai :
Sabe-se que = somatório { a(n) }=20 converge. Calcular lim [(1/n)*somatório(k*a(k))], onde o = somatório=20 vai de 1 até n e o limite é qd n-> +oo.
Abraços,
 =20 Villard
------=_NextPart_000_0025_01C20F43.885FCB40-- ========================================================================= Instruções para entrar na lista, sair da lista e usar a lista em http://www.mat.puc-rio.br/~nicolau/olimp/obm-l.html O administrador desta lista é ========================================================================= From owner-obm-l@sucuri.mat.puc-rio.br Sun Jun 9 01:24:27 2002 Return-Path: Received: (from majordom@localhost) by sucuri.mat.puc-rio.br (8.9.3/8.9.3) id BAA04271 for obm-l-list; Sun, 9 Jun 2002 01:09:29 -0300 Received: from sr1.terra.com.br (sr1.terra.com.br [200.176.3.16]) by sucuri.mat.puc-rio.br (8.9.3/8.9.3) with ESMTP id BAA04268 for ; Sun, 9 Jun 2002 01:09:27 -0300 Received: from mucuri.terra.com.br (mucuri.terra.com.br [200.176.3.39]) by sr1.terra.com.br (Postfix) with ESMTP id A29A96ED53 for ; Sun, 9 Jun 2002 00:56:48 -0300 (EST) Received: from stabel (dl-nas3-poa-C89A06A6.p001.terra.com.br [200.154.6.166]) (authenticated user dudasta) by mucuri.terra.com.br (Postfix) with ESMTP id 33120BE8BB for ; Sun, 9 Jun 2002 00:56:46 -0300 (EST) Message-ID: <005c01c20f69$a9fbcf30$a6069ac8@stabel> From: "Eduardo Casagrande Stabel" To: References: <002801c20f5c$aea12740$74f4a0c8@rodrigo> Subject: [obm-l] =?iso-8859-1?Q?Re:_=5Bobm-l=5D_Quest=E3o_:_s=E9rie/sequ=EAncia?= Date: Sun, 9 Jun 2002 00:56:37 -0300 MIME-Version: 1.0 Content-Type: text/plain; charset="iso-8859-1" Content-Transfer-Encoding: 8bit X-Priority: 3 X-MSMail-Priority: Normal X-Mailer: Microsoft Outlook Express 6.00.2600.0000 X-MimeOLE: Produced By Microsoft MimeOLE V6.00.2600.0000 Sender: owner-obm-l@sucuri.mat.puc-rio.br Precedence: bulk Reply-To: obm-l@mat.puc-rio.br E aí, Villard? Espero que essa solução seja suficientemente elegante. :) Fixe um E > 0. (esse é o epsilon dos livros de análise) Suponha S = SOMATÓRIO{k=1...infinito : a_k} e s_n = SOMATÓRIO{k=1...n : a_k} Pela definição, existe um n_1 tal que n > n_1 implica |S - s_n | < 2E ou ainda | SOMATÓRIO{k=n+1...infinito : a_k}| < 2E Mantenha esse n_1 fixado. Essa última implica que existe um n_2 tal que se n > n_2 então | SOMATÓRIO{k=n_1+1...n : a_k}| < E Agora escolha um n_3 > n_2 tal que se n > n_3 então (n_1 / n) < E Repito os passos: escolhemos o n_1 tal que S_n fosse perto do limite para todo n > n_1 Aí escolhemos um n_2 para que |s_n - s_(n_1+1)| fosse pequeno para todo n > n_2 Por fim, escolhemos um n_3 maior que n_1 e n_2 tal que (1/n) * n_1 < E Suponha que n > n_3 então | SOMATÓRIO{k=1...n : (k*a_k)/n }| < | SOMATÓRIO{k=1...n_1 : (k*a_k)/n } | + | SOMATÓRIO{k=n_1+1...n : (k*a_k)/n } | < | (n_1/n)*SOMATÓRIO{k=1...n_1 : a_k }| + |SOMATÓRIO{k=n_1+1...n : a_k}| < (E) * |(S+E)| + E = E(|S + E| + 1) O primeiro (E) é por que n > n_3. O segundo |S+E| é por que n > n_1. E o terceiro E é por que n > n_2. Claro que tomando o E muito pequeno tornamos E(|S + E| + 1) o quao pequeno quanto quisermos. Portanto o limite que tu perguntou é zero. Está certo isso? Que outra demonstração você tinha em mente? Um abraço! Eduardo Casagrande Stabel. Porto Alegre, RS. PS. Villard, e outros interessados: o que você acha de fazermos uma lista para discutirmos problemas universitários? >From: Rodrigo Villard Milet > > >Talvez a questão que estou enviando seja fácil... mas quero ver se alguém dá >alguma solução elegante pra ela... lá vai : >Sabe-se que somatório { a(n) } converge. Calcular lim [(1/n)*somatório(k*a>(k))], onde o somatório vai de 1 até n e o limite é qd n-> +oo. >Abraços, > Villard > ========================================================================= Instruções para entrar na lista, sair da lista e usar a lista em http://www.mat.puc-rio.br/~nicolau/olimp/obm-l.html O administrador desta lista é ========================================================================= From owner-obm-l@sucuri.mat.puc-rio.br Sun Jun 9 11:03:43 2002 Return-Path: Received: (from majordom@localhost) by sucuri.mat.puc-rio.br (8.9.3/8.9.3) id LAA09050 for obm-l-list; Sun, 9 Jun 2002 11:00:28 -0300 Received: (from nicolau@localhost) by sucuri.mat.puc-rio.br (8.9.3/8.9.3) id LAA09046 for obm-l@mat.puc-rio.br; Sun, 9 Jun 2002 11:00:27 -0300 Date: Sun, 9 Jun 2002 11:00:27 -0300 From: "Nicolau C. Saldanha" To: obm-l@mat.puc-rio.br Subject: [obm-l] OBM, 1a fase Message-ID: <20020609110027.B9016@sucuri.mat.puc-rio.br> References: <002801c20f5c$aea12740$74f4a0c8@rodrigo> <005c01c20f69$a9fbcf30$a6069ac8@stabel> Mime-Version: 1.0 Content-Type: text/plain; charset=iso-8859-1 Content-Disposition: inline Content-Transfer-Encoding: 8bit User-Agent: Mutt/1.2.5i In-Reply-To: <005c01c20f69$a9fbcf30$a6069ac8@stabel>; from dudasta@terra.com.br on Sun, Jun 09, 2002 at 12:56:37AM -0300 Sender: owner-obm-l@sucuri.mat.puc-rio.br Precedence: bulk Reply-To: obm-l@mat.puc-rio.br Peço a todos aqui que não usem a lista para falar da 1a fase da OBM até a Nelly publicar o gabarito na home page. Isto deve ocorrer 3a feira durante o dia. []s, N. ========================================================================= Instruções para entrar na lista, sair da lista e usar a lista em http://www.mat.puc-rio.br/~nicolau/olimp/obm-l.html O administrador desta lista é ========================================================================= From owner-obm-l@sucuri.mat.puc-rio.br Sun Jun 9 16:16:49 2002 Return-Path: Received: (from majordom@localhost) by sucuri.mat.puc-rio.br (8.9.3/8.9.3) id QAA10591 for obm-l-list; Sun, 9 Jun 2002 16:13:48 -0300 Received: from matinhos.terra.com.br (matinhos.terra.com.br [200.176.3.21]) by sucuri.mat.puc-rio.br (8.9.3/8.9.3) with ESMTP id QAA10588 for ; Sun, 9 Jun 2002 16:13:46 -0300 Received: from pavuna.terra.com.br (pavuna.terra.com.br [200.176.3.41]) by matinhos.terra.com.br (Postfix) with ESMTP id EA2AF4705C for ; Sun, 9 Jun 2002 16:01:09 -0300 (EST) Received: from stabel (dl-nas3-poa-C89A0670.p001.terra.com.br [200.154.6.112]) (authenticated user dudasta) by pavuna.terra.com.br (Postfix) with ESMTP id 62801680AB for ; Sun, 9 Jun 2002 16:01:08 -0300 (EST) Message-ID: <001a01c20fe8$016e2460$70069ac8@stabel> From: "Eduardo Casagrande Stabel" To: References: <002801c20f5c$aea12740$74f4a0c8@rodrigo> <005c01c20f69$a9fbcf30$a6069ac8@stabel> Subject: [obm-l] =?iso-8859-1?Q?Re:_=5Bobm-l=5D_Re:_=5Bobm-l=5D_Quest=E3o_:_s=E9rie/sequ?= =?iso-8859-1?Q?=EAncia?= Date: Sun, 9 Jun 2002 16:01:02 -0300 MIME-Version: 1.0 Content-Type: text/plain; charset="iso-8859-1" Content-Transfer-Encoding: 8bit X-Priority: 3 X-MSMail-Priority: Normal X-Mailer: Microsoft Outlook Express 6.00.2600.0000 X-MimeOLE: Produced By Microsoft MimeOLE V6.00.2600.0000 Sender: owner-obm-l@sucuri.mat.puc-rio.br Precedence: bulk Reply-To: obm-l@mat.puc-rio.br Eu acho que essa solução está difícil de ler. Vou tentar ser mais breve. Sejam s(n) = \sum_{k=1...n}{a(k)} e s = lim{s(k)}. Escolha arbitrariamente um e > 0. Existem dois inteiros p < q tais que |s(n) - s(p)| < e sempre que n > q. Dessa forma temos que | \sum_{k=1...n}{ ( ka(k) )/n } | <= | \sum_{k=1...p}{ ( ka(k) )/n } | + | \sum_{k=p+1...n}{ ( ka(k) )/n } | <= | \sum_{k=1...p}{ ( pa(k) )/n } | + | \sum_{k=p+1...n}{ ( na(k) )/n } | = |p/n| * |s(p)| + |s(n) - s(p)| <= |p/n| * (|S| + e) + e Essa última expressão pode ser feita muito pequena, desde que n seja grande. Acho que agora está melhor. Eduardo Casagrande Stabel. Porto Alegre, RS. From: "Eduardo Casagrande Stabel" > E aí, Villard? > > Espero que essa solução seja suficientemente elegante. :) > > Fixe um E > 0. (esse é o epsilon dos livros de análise) > Suponha S = SOMATÓRIO{k=1...infinito : a_k} e s_n = SOMATÓRIO{k=1...n : a_k} > > Pela definição, existe um n_1 tal que n > n_1 implica > |S - s_n | < 2E ou ainda > | SOMATÓRIO{k=n+1...infinito : a_k}| < 2E > > Mantenha esse n_1 fixado. > Essa última implica que existe um n_2 tal que se n > n_2 então > | SOMATÓRIO{k=n_1+1...n : a_k}| < E > > Agora escolha um n_3 > n_2 tal que se n > n_3 então > (n_1 / n) < E > > Repito os passos: escolhemos o n_1 tal que S_n fosse perto do limite para > todo n > n_1 > Aí escolhemos um n_2 para que |s_n - s_(n_1+1)| fosse pequeno para todo n > > n_2 > Por fim, escolhemos um n_3 maior que n_1 e n_2 tal que (1/n) * n_1 < E > > Suponha que n > n_3 então > | SOMATÓRIO{k=1...n : (k*a_k)/n }| < > | SOMATÓRIO{k=1...n_1 : (k*a_k)/n } | + | SOMATÓRIO{k=n_1+1...n : > (k*a_k)/n } | < > | (n_1/n)*SOMATÓRIO{k=1...n_1 : a_k }| + |SOMATÓRIO{k=n_1+1...n : a_k}| < > (E) * |(S+E)| + E = E(|S + E| + 1) > > O primeiro (E) é por que n > n_3. O segundo |S+E| é por que n > n_1. E o > terceiro E é por que n > n_2. > > Claro que tomando o E muito pequeno tornamos E(|S + E| + 1) o quao pequeno > quanto quisermos. Portanto o limite que tu perguntou é zero. > > Está certo isso? > > Que outra demonstração você tinha em mente? > > Um abraço! > > Eduardo Casagrande Stabel. Porto Alegre, RS. > > PS. Villard, e outros interessados: o que você acha de fazermos uma lista > para discutirmos problemas universitários? > > > >From: Rodrigo Villard Milet > > > > > >Talvez a questão que estou enviando seja fácil... mas quero ver se alguém > dá >alguma solução elegante pra ela... lá vai : > >Sabe-se que somatório { a(n) } converge. Calcular lim > [(1/n)*somatório(k*a>(k))], onde o somatório vai de 1 até n e o limite é qd > n-> +oo. > >Abraços, > > Villard > > > > ========================================================================= > Instruções para entrar na lista, sair da lista e usar a lista em > http://www.mat.puc-rio.br/~nicolau/olimp/obm-l.html > O administrador desta lista é > ========================================================================= > > ========================================================================= Instruções para entrar na lista, sair da lista e usar a lista em http://www.mat.puc-rio.br/~nicolau/olimp/obm-l.html O administrador desta lista é ========================================================================= From owner-obm-l@sucuri.mat.puc-rio.br Sun Jun 9 19:36:23 2002 Return-Path: Received: (from majordom@localhost) by sucuri.mat.puc-rio.br (8.9.3/8.9.3) id TAA11606 for obm-l-list; Sun, 9 Jun 2002 19:33:26 -0300 Received: from web10103.mail.yahoo.com (web10103.mail.yahoo.com [216.136.130.53]) by sucuri.mat.puc-rio.br (8.9.3/8.9.3) with SMTP id TAA11603 for ; Sun, 9 Jun 2002 19:33:23 -0300 Message-ID: <20020609222046.52881.qmail@web10103.mail.yahoo.com> Received: from [200.151.221.70] by web10103.mail.yahoo.com via HTTP; Sun, 09 Jun 2002 15:20:46 PDT Date: Sun, 9 Jun 2002 15:20:46 -0700 (PDT) From: Rafael WC Subject: [obm-l] Fatoração To: OBM MIME-Version: 1.0 Content-Type: text/plain; charset=us-ascii Sender: owner-obm-l@sucuri.mat.puc-rio.br Precedence: bulk Reply-To: obm-l@mat.puc-rio.br Oi Pessoal! Eu queria saber como posso achar um fator entre 1000 e 5000 do nº 2^33 - 2^19 - 2^17 - 1 A resposta que tenho é 1983, mas acho vou precisar de conhecimentos que vão além do nível médio, não? Abraços, Rafael. ===== Rafael Werneck Cinoto ICQ# 107011599 rwcinoto@yahoo.com rafael.caixa@gov.com.br matduvidas@yahoo.com.br http://www.rwcinoto.hpg.com.br/ __________________________________________________ Do You Yahoo!? Yahoo! - Official partner of 2002 FIFA World Cup http://fifaworldcup.yahoo.com ========================================================================= Instruções para entrar na lista, sair da lista e usar a lista em http://www.mat.puc-rio.br/~nicolau/olimp/obm-l.html O administrador desta lista é ========================================================================= From owner-obm-l@sucuri.mat.puc-rio.br Sun Jun 9 19:40:38 2002 Return-Path: Received: (from majordom@localhost) by sucuri.mat.puc-rio.br (8.9.3/8.9.3) id TAA11652 for obm-l-list; Sun, 9 Jun 2002 19:38:33 -0300 Received: from traven.uol.com.br (traven.uol.com.br [200.231.206.184]) by sucuri.mat.puc-rio.br (8.9.3/8.9.3) with ESMTP id TAA11649 for ; Sun, 9 Jun 2002 19:38:32 -0300 Received: from computador ([200.213.211.193]) by traven.uol.com.br (8.9.1/8.9.1) with SMTP id TAA21501 for ; Sun, 9 Jun 2002 19:14:28 -0300 (BRT) From: =?iso-8859-1?Q?Moacyr_Rodrigues_J=FAnior?= To: Subject: RES: [obm-l] plana ajuda por favor urgente Date: Sun, 9 Jun 2002 19:24:51 -0300 Message-ID: MIME-Version: 1.0 Content-Type: text/plain; charset="iso-8859-1" Content-Transfer-Encoding: 8bit X-Priority: 3 (Normal) X-MSMail-Priority: Normal X-Mailer: Microsoft Outlook IMO, Build 9.0.2416 (9.0.2910.0) X-MimeOLE: Produced By Microsoft MimeOLE V6.00.2600.0000 In-Reply-To: Importance: Normal Sender: owner-obm-l@sucuri.mat.puc-rio.br Precedence: bulk Reply-To: obm-l@mat.puc-rio.br 3.No triangulo ABC,AB=5 e BC=6.Qual a area do triangulo ABC,sabendo q o angulo C tem a maior medida possivel? Será 5 vezes a raiz quadrada de 11, dividido por 2. Monte uma circunferência de centro B e raio 6. Coloque um ponto A no interior desta circunferência, a uma distância 5 do centro B. O vértice C deve estar na circunferência, veja que esta construção satisfaz o enunciado da questão. Para calcularmos o ângulo basta traçarmos as cordas CP que contém o ponto A e CQ que contém o ponto B, sendo esta última um diâmetro desta circunferência. Assim o triângulo CPQ será sempre retângulo em P. A partir daqui podemos provar de várias maneiras que o ângulo C é máximo quando A for 90º. Uma forma é traçando a altura relativa ao vértice B, no triângulo ABC, encontramos o ponto H do lado CA. O seno do ângulo C pode ser obtido por BH/CB como CB é fixo, este seno será máximo quando BH for máximo, o que ocorrerá para A=90º. Com A=90º temos a hipotenusa de medida 6 e um cateto de medida 5, o outro cateto terá medida igual à raiz quadrada de 11, logo sua área será 5 vezes a raiz quadrada de 11, dividido por 2. Abraços, Moacyr Rodrigues Júnior. ========================================================================= Instruções para entrar na lista, sair da lista e usar a lista em http://www.mat.puc-rio.br/~nicolau/olimp/obm-l.html O administrador desta lista é ========================================================================= From owner-obm-l@sucuri.mat.puc-rio.br Sun Jun 9 20:05:10 2002 Return-Path: Received: (from majordom@localhost) by sucuri.mat.puc-rio.br (8.9.3/8.9.3) id UAA12098 for obm-l-list; Sun, 9 Jun 2002 20:02:36 -0300 Received: from web21305.mail.yahoo.com (web21305.mail.yahoo.com [216.136.129.141]) by sucuri.mat.puc-rio.br (8.9.3/8.9.3) with SMTP id UAA12095 for ; Sun, 9 Jun 2002 20:02:34 -0300 Message-ID: <20020609224957.50446.qmail@web21305.mail.yahoo.com> Received: from [200.227.209.233] by web21305.mail.yahoo.com via HTTP; Sun, 09 Jun 2002 19:49:57 ART Date: Sun, 9 Jun 2002 19:49:57 -0300 (ART) From: =?iso-8859-1?q?Marcos=20Reynaldo?= Subject: Re: [obm-l] Rafael Santos ;;;; To: obm-l@mat.puc-rio.br In-Reply-To: <000f01c20efc$d8db66c0$c33a97c8@jat> MIME-Version: 1.0 Content-Type: text/plain; charset=iso-8859-1 Content-Transfer-Encoding: 8bit Sender: owner-obm-l@sucuri.mat.puc-rio.br Precedence: bulk Reply-To: obm-l@mat.puc-rio.br --- Jose Augusto escreveu: > Serve achando o XY e consequentemente X + Y e o > quadrado deles ... ???? > Lembre-se que (x+y)^2 - 2xy = x^2 + y^2. _______________________________________________________________________ Copa 2002 Yahoo! - Patrocinador oficial da Copa do Mundo da FIFA 2002 http://br.sports.yahoo.com/fifaworldcup/ ========================================================================= Instruções para entrar na lista, sair da lista e usar a lista em http://www.mat.puc-rio.br/~nicolau/olimp/obm-l.html O administrador desta lista é ========================================================================= From owner-obm-l@sucuri.mat.puc-rio.br Sun Jun 9 21:01:36 2002 Return-Path: Received: (from majordom@localhost) by sucuri.mat.puc-rio.br (8.9.3/8.9.3) id UAA12765 for obm-l-list; Sun, 9 Jun 2002 20:59:24 -0300 Received: from salvatore2.bol.com.br (salvatore2.bol.com.br [200.221.24.87]) by sucuri.mat.puc-rio.br (8.9.3/8.9.3) with ESMTP id UAA12762 for ; Sun, 9 Jun 2002 20:59:23 -0300 Received: from bol.com.br (200.221.24.84) by salvatore2.bol.com.br (5.1.071) id 3CFE36F1000D9EEB for obm-l@mat.puc-rio.br; Sun, 9 Jun 2002 20:46:22 -0300 Date: Sun, 9 Jun 2002 20:46:22 -0300 Message-Id: Subject: [obm-l] =?iso-8859-1?q?pergunta_sobre_uma_quest=E3o=2E=2E=2E?= MIME-Version: 1.0 Content-Type: text/plain;charset="iso-8859-1" From: "leon-17" To: obm-l@mat.puc-rio.br X-XaM3-API-Version: 2.4.3.4.4 X-SenderIP: 200.254.248.244 Content-Transfer-Encoding: 8bit X-MIME-Autoconverted: from quoted-printable to 8bit by sucuri.mat.puc-rio.br id UAA12763 Sender: owner-obm-l@sucuri.mat.puc-rio.br Precedence: bulk Reply-To: obm-l@mat.puc-rio.br Gostaria de saber como se faz a questão 9 dos problemas do nivel 1, 1ª fase da XXIII Olimpíada de Matemática. Sobre o serralheiro que possuia 10 pedaços de 3 elos de ferro cada um. Ele quer fazer uma corrente única de 30 elos. Para abrir e soltar um elo ele leva 5 minutos. Quantos minutos no mínimo ele levará para fazer a corrente? A resposta é 35 min, mas eu queria saber porque. Obrigado Thiago Lima __________________________________________________________________________ Quer ter seu próprio endereço na Internet? Garanta já o seu e ainda ganhe cinco e-mails personalizados. DomíniosBOL - http://dominios.bol.com.br ========================================================================= Instruções para entrar na lista, sair da lista e usar a lista em http://www.mat.puc-rio.br/~nicolau/olimp/obm-l.html O administrador desta lista é ========================================================================= From owner-obm-l@sucuri.mat.puc-rio.br Sun Jun 9 21:47:20 2002 Return-Path: Received: (from majordom@localhost) by sucuri.mat.puc-rio.br (8.9.3/8.9.3) id VAA13176 for obm-l-list; Sun, 9 Jun 2002 21:45:32 -0300 Received: from smtp.estaminas.com.br (smtp.estaminas.com.br [200.188.191.51]) by sucuri.mat.puc-rio.br (8.9.3/8.9.3) with ESMTP id VAA13173 for ; Sun, 9 Jun 2002 21:45:29 -0300 Received: from a0x0t6 (dial-tm-ip-4-11.estaminas.com.br [200.216.168.139]) by smtp.estaminas.com.br (8.11.0/8.11.0) with SMTP id g5A0bB728598 for ; Sun, 9 Jun 2002 21:37:13 -0300 Message-ID: <000301c21016$ad374280$8ba8d8c8@a0x0t6> From: "Douglas Carvalho" To: References: Subject: [obm-l] =?iso-8859-1?Q?Re:_=5Bobm-l=5D_pergunta_sobre_uma_quest=E3o...?= Date: Sun, 9 Jun 2002 21:34:50 -0300 MIME-Version: 1.0 Content-Type: text/plain; charset="iso-8859-1" Content-Transfer-Encoding: 8bit X-Priority: 3 X-MSMail-Priority: Normal X-Mailer: Microsoft Outlook Express 5.00.2615.200 X-MimeOLE: Produced By Microsoft MimeOLE V5.00.2615.200 Sender: owner-obm-l@sucuri.mat.puc-rio.br Precedence: bulk Reply-To: obm-l@mat.puc-rio.br > Gostaria de saber como se faz a questão 9 dos problemas > do nivel 1, 1ª fase da XXIII Olimpíada de Matemática. > Sobre o serralheiro que possuia 10 pedaços de 3 elos de > ferro cada um. Ele quer fazer uma corrente única de 30 > elos. Para abrir e soltar um elo ele leva 5 minutos. > Quantos minutos no mínimo ele levará para fazer a > corrente? A resposta é 35 min, mas eu queria saber porque. > Quando ele abre um dos conjuntos de 3 elos, ele gasta 15 minutos e pode ligar mais 4 conjuntos de 3 elos: ooo O ooo O ooo O ooo (os "o"'s são os tres elos unidos e os "O" 's sao os que foram abertos) abrindo outro conjunto de 3 elos temos ooo O ooo O ooo O ooo Entao gastamos 30 minutos e temos 2 correntes de 15 elos. Para unir as duas correntes (representados pelo hífen abaixo) necessitamos de abrir mais um elo: mais 5 minutos. Total 35 minutos oooOoooOoooOooo-oooOoooOoooOooo []'s Douglas Carvalho ========================================================================= Instruções para entrar na lista, sair da lista e usar a lista em http://www.mat.puc-rio.br/~nicolau/olimp/obm-l.html O administrador desta lista é ========================================================================= From owner-obm-l@sucuri.mat.puc-rio.br Sun Jun 9 22:50:51 2002 Return-Path: Received: (from majordom@localhost) by sucuri.mat.puc-rio.br (8.9.3/8.9.3) id WAA13600 for obm-l-list; Sun, 9 Jun 2002 22:47:17 -0300 Received: from hotmail.com (f69.law4.hotmail.com [216.33.149.69]) by sucuri.mat.puc-rio.br (8.9.3/8.9.3) with ESMTP id WAA13597 for ; Sun, 9 Jun 2002 22:47:14 -0300 Received: from mail pickup service by hotmail.com with Microsoft SMTPSVC; Sun, 9 Jun 2002 18:34:37 -0700 Received: from 200.191.230.63 by lw4fd.law4.hotmail.msn.com with HTTP; Mon, 10 Jun 2002 01:34:37 GMT X-Originating-IP: [200.191.230.63] From: "rafael dowsley" To: obm-l@mat.puc-rio.br Subject: [obm-l] =?iso-8859-1?B?RGVtb250cmHn9WVz?= Date: Mon, 10 Jun 2002 01:34:37 +0000 Mime-Version: 1.0 Content-Type: text/plain; charset=iso-8859-1; format=flowed Message-ID: X-OriginalArrivalTime: 10 Jun 2002 01:34:37.0783 (UTC) FILETIME=[FBB56E70:01C2101E] Sender: owner-obm-l@sucuri.mat.puc-rio.br Precedence: bulk Reply-To: obm-l@mat.puc-rio.br Uma coisa que eu sempre quis saber e nunca em encontrei em nenhum lugar é a demonstração daquelas regras simples para saber se um número é divísivel por 7, 11... Alguem poderia me passar ou então me dizer onde posso encontra-las Rafael Baião Dowsley _________________________________________________________________ Una-se ao maior serviço de email do mundo: o MSN Hotmail. http://www.hotmail.com ========================================================================= Instruções para entrar na lista, sair da lista e usar a lista em http://www.mat.puc-rio.br/~nicolau/olimp/obm-l.html O administrador desta lista é ========================================================================= From owner-obm-l@sucuri.mat.puc-rio.br Sun Jun 9 22:59:39 2002 Return-Path: Received: (from majordom@localhost) by sucuri.mat.puc-rio.br (8.9.3/8.9.3) id WAB13666 for obm-l-list; Sun, 9 Jun 2002 22:57:36 -0300 Received: from hotmail.com (f21.law4.hotmail.com [216.33.149.21]) by sucuri.mat.puc-rio.br (8.9.3/8.9.3) with ESMTP id WAA13663 for ; Sun, 9 Jun 2002 22:57:28 -0300 Received: from mail pickup service by hotmail.com with Microsoft SMTPSVC; Sun, 9 Jun 2002 18:44:23 -0700 Received: from 200.191.230.63 by lw4fd.law4.hotmail.msn.com with HTTP; Mon, 10 Jun 2002 01:44:22 GMT X-Originating-IP: [200.191.230.63] From: "rafael dowsley" To: obm-l@mat.puc-rio.br Subject: [obm-l] Ajuda Geometria Date: Mon, 10 Jun 2002 01:44:22 +0000 Mime-Version: 1.0 Content-Type: text/plain; charset=iso-8859-1; format=flowed Message-ID: X-OriginalArrivalTime: 10 Jun 2002 01:44:23.0041 (UTC) FILETIME=[588CB710:01C21020] Sender: owner-obm-l@sucuri.mat.puc-rio.br Precedence: bulk Reply-To: obm-l@mat.puc-rio.br Alguem poderia me explicar a resolução da questão 18 da ollimpíada do nível 3? Rafael Baião Dowsley _________________________________________________________________ Una-se ao maior serviço de email do mundo: o MSN Hotmail. http://www.hotmail.com ========================================================================= Instruções para entrar na lista, sair da lista e usar a lista em http://www.mat.puc-rio.br/~nicolau/olimp/obm-l.html O administrador desta lista é ========================================================================= From owner-obm-l@sucuri.mat.puc-rio.br Mon Jun 10 00:21:33 2002 Return-Path: Received: (from majordom@localhost) by sucuri.mat.puc-rio.br (8.9.3/8.9.3) id AAA14314 for obm-l-list; Mon, 10 Jun 2002 00:18:51 -0300 Received: from web10108.mail.yahoo.com (web10108.mail.yahoo.com [216.136.130.58]) by sucuri.mat.puc-rio.br (8.9.3/8.9.3) with SMTP id AAA14311 for ; Mon, 10 Jun 2002 00:18:48 -0300 Message-ID: <20020610030611.73923.qmail@web10108.mail.yahoo.com> Received: from [200.151.211.171] by web10108.mail.yahoo.com via HTTP; Sun, 09 Jun 2002 20:06:11 PDT Date: Sun, 9 Jun 2002 20:06:11 -0700 (PDT) From: Rafael WC Subject: Re: [obm-l] Demontrações To: obm-l@mat.puc-rio.br In-Reply-To: MIME-Version: 1.0 Content-Type: text/plain; charset=us-ascii Sender: owner-obm-l@sucuri.mat.puc-rio.br Precedence: bulk Reply-To: obm-l@mat.puc-rio.br --- rafael dowsley wrote: > > Uma coisa que eu sempre quis saber e nunca em > encontrei em nenhum lugar é a > demonstração daquelas regras simples para saber se > um número é divísivel por > 7, 11... > Alguem poderia me passar ou então me dizer onde > posso encontra-las > > > Rafael Baião Dowsley Oi Rafael! Experimente essa página que tem bastante coisa: http://pessoal.sercomtel.com.br/matematica/fundam/103/mod103b.htm Abraços, Refael. ===== Rafael Werneck Cinoto ICQ# 107011599 rwcinoto@yahoo.com rafael.caixa@gov.com.br matduvidas@yahoo.com.br http://www.rwcinoto.hpg.com.br/ __________________________________________________ Do You Yahoo!? Yahoo! - Official partner of 2002 FIFA World Cup http://fifaworldcup.yahoo.com ========================================================================= Instruções para entrar na lista, sair da lista e usar a lista em http://www.mat.puc-rio.br/~nicolau/olimp/obm-l.html O administrador desta lista é ========================================================================= From owner-obm-l@sucuri.mat.puc-rio.br Mon Jun 10 00:29:04 2002 Return-Path: Received: (from majordom@localhost) by sucuri.mat.puc-rio.br (8.9.3/8.9.3) id AAA14438 for obm-l-list; Mon, 10 Jun 2002 00:26:33 -0300 Received: from mail.gmx.net (mail.gmx.net [213.165.64.20]) by sucuri.mat.puc-rio.br (8.9.3/8.9.3) with SMTP id AAA14435 for ; Mon, 10 Jun 2002 00:26:29 -0300 Received: (qmail 5918 invoked by uid 0); 10 Jun 2002 03:13:50 -0000 Received: from unknown (HELO gomes) (200.216.104.107) by mail.gmx.net (mp006-rz3) with SMTP; 10 Jun 2002 03:13:50 -0000 Date: Mon, 10 Jun 2002 00:15:48 -0300 From: Igor GomeZZ X-Mailer: The Bat! (v1.60c) Organization: -- X-Priority: 3 (Normal) Message-ID: <11845250586.20020610001548@gmx.net> To: OBM Subject: [obm-l] Provas da OBM MIME-Version: 1.0 Content-Type: text/plain; charset=ISO-8859-1 Content-Transfer-Encoding: 8bit Sender: owner-obm-l@sucuri.mat.puc-rio.br Precedence: bulk Reply-To: obm-l@mat.puc-rio.br Existe algum site que tenha a resolução de todas as provas das olimpíadas? (ou semelhante) Fui! ####### Igor GomeZZ ######## UIN: 29249895 Vitória, Espírito Santo, Brasil Criação: 10/6/2002 (00:14) #################################### Pare para pensar: Aquele que não conhece a verdade é simplesmente um ignorante, mas aquele que a conhece e diz que é mentira, este é um criminoso. (Brecht) #################################### ========================================================================= Instruções para entrar na lista, sair da lista e usar a lista em http://www.mat.puc-rio.br/~nicolau/olimp/obm-l.html O administrador desta lista é ========================================================================= From owner-obm-l@sucuri.mat.puc-rio.br Mon Jun 10 13:15:31 2002 Return-Path: Received: (from majordom@localhost) by sucuri.mat.puc-rio.br (8.9.3/8.9.3) id NAA19113 for obm-l-list; Mon, 10 Jun 2002 13:12:58 -0300 Received: from www.zipmail.com.br (smtp.zipmail.com.br [200.187.242.10]) by sucuri.mat.puc-rio.br (8.9.3/8.9.3) with ESMTP id NAA19110 for ; Mon, 10 Jun 2002 13:12:54 -0300 From: ghaeser@zipmail.com.br Received: from [143.106.22.156] by www.zipmail.com.br with HTTP; Mon, 10 Jun 2002 13:00:17 -0300 Message-ID: <3D00E61A00003E82@www.zipmail.com.br> Date: Mon, 10 Jun 2002 17:00:17 +0100 Subject: [obm-l] =?iso-8859-1?Q?Polinomio?= To: obm-l@mat.puc-rio.br MIME-Version: 1.0 Content-Type: text/plain; charset="iso-8859-1" Content-Transfer-Encoding: 8bit X-MIME-Autoconverted: from quoted-printable to 8bit by sucuri.mat.puc-rio.br id NAA19111 Sender: owner-obm-l@sucuri.mat.puc-rio.br Precedence: bulk Reply-To: obm-l@mat.puc-rio.br Prove ou dê um contra-exemplo: Seja P um polinômio de grau n entao P pode ser escrito como: P(x)=sum((x,k)*sum((k,j)*(-1)^j*P(j))), onde a soma interna é de j=0,..,k e a externa de k=0,..,n. obrigado!! Gabriel Haeser www.gabas.cjb.net "Mathematicus nascitur, non fit" Matemáticos não são feitos, eles nascem --------------------------------------- Gabriel Haeser www.gabas.cjb.net ------------------------------------------ Use o melhor sistema de busca da Internet Radar UOL - http://www.radaruol.com.br ========================================================================= Instruções para entrar na lista, sair da lista e usar a lista em http://www.mat.puc-rio.br/~nicolau/olimp/obm-l.html O administrador desta lista é ========================================================================= From owner-obm-l@sucuri.mat.puc-rio.br Mon Jun 10 13:31:34 2002 Return-Path: Received: (from majordom@localhost) by sucuri.mat.puc-rio.br (8.9.3/8.9.3) id NAA19335 for obm-l-list; Mon, 10 Jun 2002 13:31:21 -0300 Received: from smtp.brturbo.com (smtp1.brturbo.com [200.199.201.21]) by sucuri.mat.puc-rio.br (8.9.3/8.9.3) with ESMTP id NAA19331 for ; Mon, 10 Jun 2002 13:31:11 -0300 Received: from Jeremias (unknown [200.181.84.28]) by smtp.brturbo.com (Postfix) with SMTP id E3A7E17E009 for ; Mon, 10 Jun 2002 13:18:32 -0300 (BRT) Message-ID: <002001c2109a$3cf21020$02c8a8c0@dummy.net> From: "Jeremias de Paula Eduardo" To: References: <20020610030611.73923.qmail@web10108.mail.yahoo.com> Subject: [obm-l] =?iso-8859-1?Q?Re:_=5Bobm-l=5D_Demontra=E7=F5es?= Date: Mon, 10 Jun 2002 13:16:55 -0300 MIME-Version: 1.0 Content-Type: text/plain; charset="iso-8859-1" Content-Transfer-Encoding: 8bit X-Priority: 3 X-MSMail-Priority: Normal X-Mailer: Microsoft Outlook Express 6.00.2600.0000 X-MimeOLE: Produced By Microsoft MimeOLE V6.00.2600.0000 Sender: owner-obm-l@sucuri.mat.puc-rio.br Precedence: bulk Reply-To: obm-l@mat.puc-rio.br A quanto tempo vc participa dessa lista? É o Jeremias Eu acabei de entrar. > --- rafael dowsley wrote: > > Uma coisa que eu sempre quis saber e nunca em > encontrei em nenhum lugar é a > demonstração daquelas regras simples para saber se > um número é divísivel por > 7, 11... > Alguem poderia me passar ou então me dizer onde > posso encontra-las > > > Rafael Baião Dowsley ========================================================================= Instruções para entrar na lista, sair da lista e usar a lista em http://www.mat.puc-rio.br/~nicolau/olimp/obm-l.html O administrador desta lista é ========================================================================= From owner-obm-l@sucuri.mat.puc-rio.br Mon Jun 10 14:08:52 2002 Return-Path: Received: (from majordom@localhost) by sucuri.mat.puc-rio.br (8.9.3/8.9.3) id OAA20060 for obm-l-list; Mon, 10 Jun 2002 14:08:28 -0300 Received: from www.zipmail.com.br (smtp.zipmail.com.br [200.187.242.10]) by sucuri.mat.puc-rio.br (8.9.3/8.9.3) with ESMTP id OAA20057 for ; Mon, 10 Jun 2002 14:08:24 -0300 From: peterdirichlet@zipmail.com.br Received: from [200.206.103.3] by www.zipmail.com.br with HTTP; Mon, 10 Jun 2002 13:55:26 -0300 Message-ID: <3D04D69F000000C3@www.zipmail.com.br> Date: Mon, 10 Jun 2002 14:55:26 -0200 In-Reply-To: <20020608053931.50203.qmail@web10103.mail.yahoo.com> Subject: [obm-l] =?iso-8859-1?Q?Re=3A=20=5Bobm=2Dl=5D=20x=B2=20=2B=20y=B2?= To: obm-l@mat.puc-rio.br MIME-Version: 1.0 Content-Type: text/plain; charset="iso-8859-1" Content-Transfer-Encoding: 8bit X-MIME-Autoconverted: from quoted-printable to 8bit by sucuri.mat.puc-rio.br id OAA20058 Sender: owner-obm-l@sucuri.mat.puc-rio.br Precedence: bulk Reply-To: obm-l@mat.puc-rio.br Essa de trocar x e y ajuda mesmo!!!!!!!!Vamos substituir pelos polinomios simetricos elementares(soma,soma dos produtos 2 a 2,soma dos produtos 3 a 3...,produto de todos).No nosso caso, xy=P,x+y=S. Entao S+P=71,e SP=880.Ai o resto ce continua... -- Mensagem original -- >Olá Pessoal! > >Esse exercício eu resolvi, mas não foi do jeito que eu >queria. Eu acabei fazendo o jeito convencional de >isolar o x da primeira equação e colocar na segunda. >Mas deve haver um jeito de se chegar a resposta >manipulando as equações dadas sem que precisemos >encontrar os valores de x e y. Vejam se vocês >conseguem. > >Sejam x e y inteiros positivos tais que: >xy + x + y = 71 >x²y + xy² = 880. > >Determine x² + y². > >Talvez facilite saberem que se trocarmos as variáveis, >escrevendo x no lugar de y as equações não mudam. Isso >quer dizer que as respostas são (x, y) e (y, x). Além >disso, os valores que encontrei foram x = 11 e y = 5 >(ou vice-versa). > >Um abraço, > >Rafael. > >===== >Rafael Werneck Cinoto > ICQ# 107011599 > rwcinoto@yahoo.com > rafael.caixa@gov.com.br > matduvidas@yahoo.com.br >http://www.rwcinoto.hpg.com.br/ > >__________________________________________________ >Do You Yahoo!? >Yahoo! - Official partner of 2002 FIFA World Cup >http://fifaworldcup.yahoo.com >========================================================================= >Instruções para entrar na lista, sair da lista e usar a lista em >http://www.mat.puc-rio.br/~nicolau/olimp/obm-l.html >O administrador desta lista é >========================================================================= > TRANSIRE SVVM PECTVS MVNDOQUE POTIRE CONGREGATI EX TOTO ORBE MATHEMATICI OB SCRIPTA INSIGNIA TRIBVERE Medalha Fields(John Charles Fields) ------------------------------------------ Use o melhor sistema de busca da Internet Radar UOL - http://www.radaruol.com.br ========================================================================= Instruções para entrar na lista, sair da lista e usar a lista em http://www.mat.puc-rio.br/~nicolau/olimp/obm-l.html O administrador desta lista é ========================================================================= From owner-obm-l@sucuri.mat.puc-rio.br Mon Jun 10 15:34:10 2002 Return-Path: Received: (from majordom@localhost) by sucuri.mat.puc-rio.br (8.9.3/8.9.3) id PAA21615 for obm-l-list; Mon, 10 Jun 2002 15:32:17 -0300 Received: from salvatore2.bol.com.br (salvatore2.bol.com.br [200.221.24.87]) by sucuri.mat.puc-rio.br (8.9.3/8.9.3) with ESMTP id PAA21612 for ; Mon, 10 Jun 2002 15:32:14 -0300 Received: from bol.com.br (200.221.24.80) by salvatore2.bol.com.br (5.1.071) id 3CFE36F100102606 for obm-l@mat.puc-rio.br; Mon, 10 Jun 2002 15:19:11 -0300 Date: Mon, 10 Jun 2002 15:19:11 -0300 Message-Id: Subject: [obm-l] =?iso-8859-1?q?D=FAvida=2E=2E?= MIME-Version: 1.0 Content-Type: text/plain;charset="iso-8859-1" From: "leon-17" To: obm-l@mat.puc-rio.br X-XaM3-API-Version: 2.4.3.4.4 X-SenderIP: 200.254.248.25 Content-Transfer-Encoding: 8bit X-MIME-Autoconverted: from quoted-printable to 8bit by sucuri.mat.puc-rio.br id PAA21613 Sender: owner-obm-l@sucuri.mat.puc-rio.br Precedence: bulk Reply-To: obm-l@mat.puc-rio.br OBRIGADO DOUGLAS PELA RESPOSTA. Tenho outra dúvida (minha vida é repleta dela). Os pontos P1, P2, P3, ... estão nesta ordem sobre uma circunferência e são tais que o arco que une cada ponto ao seguinte mede 35°. O menor valor de n>1 tal que Pn coincide com P1 é... Alguém por favor me livre desta ignorância sobre circunferências e me ajude nesta questão. Obrigado antecipado.. Thiago Lima __________________________________________________________________________ Quer ter seu próprio endereço na Internet? Garanta já o seu e ainda ganhe cinco e-mails personalizados. DomíniosBOL - http://dominios.bol.com.br ========================================================================= Instruções para entrar na lista, sair da lista e usar a lista em http://www.mat.puc-rio.br/~nicolau/olimp/obm-l.html O administrador desta lista é ========================================================================= From owner-obm-l@sucuri.mat.puc-rio.br Mon Jun 10 16:34:12 2002 Return-Path: Received: (from majordom@localhost) by sucuri.mat.puc-rio.br (8.9.3/8.9.3) id QAA22630 for obm-l-list; Mon, 10 Jun 2002 16:33:36 -0300 Received: from smtp-30.ig.com.br (smtp-30.ig.com.br [200.226.132.180]) by sucuri.mat.puc-rio.br (8.9.3/8.9.3) with SMTP id QAA22627 for ; Mon, 10 Jun 2002 16:33:33 -0300 From: alvie@ig.com.br Message-Id: <200206101933.QAA22627@sucuri.mat.puc-rio.br> Received: (qmail 19489 invoked from network); 10 Jun 2002 19:20:53 -0000 Received: from 120.133.226.200.in-addr.arpa.ig.com.br (HELO localhost) (200.226.133.120) by smtp-30.ig.com.br with SMTP; 10 Jun 2002 19:20:53 -0000 Date: Mon, 10 Jun 2002 16:07:46 -0300 To: obm-l@mat.puc-rio.br Cc: X-Originating-IP: [200.226.133.121]200.144.48.210 X-Mailer: InMail by Insite - www.insite.com.br X-user: alvie@ig.com.br MIME-Version: 1.0 Content-type: text/plain Subject: Re: [obm-l] Dúvida.. Sender: owner-obm-l@sucuri.mat.puc-rio.br Precedence: bulk Reply-To: obm-l@mat.puc-rio.br Bom, acho que é assim: Repare que um ponto é marcado na circunferencia, dai, "espaçados" de 35 graus, novos pontos sao acrescentados... Deseja-se saber qual desses novos pontos que ao ser colocado coincide com o primeiro... A circunferencia tem 360 graus, logo: mmc(360,35) = 2520 (em graus) 2520 /35 = 72 Assim, o menor n é 72... Acho que é assim... Desculpem se errei... Em 10 Jun 2002, obm-l@mat.puc-rio.br escreveu: >OBRIGADO DOUGLAS PELA RESPOSTA. > >Tenho outra dúvida (minha vida é repleta dela). >Os pontos P1, P2, P3, ... estão nesta ordem sobre uma >circunferência e são tais que o arco que une cada ponto >ao seguinte mede 35°. O menor valor de n>1 tal que Pn >coincide com P1 é... > >Alguém por favor me livre desta ignorância sobre >circunferências e me ajude nesta questão. Obrigado >antecipado.. > >Thiago Lima > >__________________________________________________________________________ >Quer ter seu próprio endereço na Internet? >Garanta já o seu e ainda ganhe cinco e-mails personalizados. >DomíniosBOL - http://dominios.bol.com.br > >========================================================================= >Instruções para entrar na lista, sair da lista e usar a lista em >http://www.mat.puc-rio.br/~nicolau/olimp/obm-l.html >O administrador desta lista é >========================================================================= > >---------- _________________________________________________________ Oi! Você quer um iG-mail gratuito? Então clique aqui: http://registro.ig.com.br/ ========================================================================= Instruções para entrar na lista, sair da lista e usar a lista em http://www.mat.puc-rio.br/~nicolau/olimp/obm-l.html O administrador desta lista é ========================================================================= From owner-obm-l@sucuri.mat.puc-rio.br Mon Jun 10 16:47:20 2002 Return-Path: Received: (from majordom@localhost) by sucuri.mat.puc-rio.br (8.9.3/8.9.3) id QAA22858 for obm-l-list; Mon, 10 Jun 2002 16:46:39 -0300 Received: from www.zipmail.com.br (smtp.zipmail.com.br [200.187.242.10]) by sucuri.mat.puc-rio.br (8.9.3/8.9.3) with ESMTP id QAA22854 for ; Mon, 10 Jun 2002 16:46:35 -0300 From: peterdirichlet@zipmail.com.br Received: from [200.206.103.3] by www.zipmail.com.br with HTTP; Mon, 10 Jun 2002 16:33:58 -0300 Message-ID: <3D04E6CB000005CC@www.zipmail.com.br> Date: Mon, 10 Jun 2002 17:33:58 -0200 In-Reply-To: <012201c20e6e$b3e33ac0$f1469ec8@cabru> Subject: [obm-l] =?iso-8859-1?Q?Re=3A=20=5Bobm=2Dl=5D=20Desafio=20o=20retorno=21=21?= To: obm-l@mat.puc-rio.br MIME-Version: 1.0 Content-Type: text/plain; charset="iso-8859-1" Content-Transfer-Encoding: 8bit X-MIME-Autoconverted: from quoted-printable to 8bit by sucuri.mat.puc-rio.br id QAA22855 Sender: owner-obm-l@sucuri.mat.puc-rio.br Precedence: bulk Reply-To: obm-l@mat.puc-rio.br Vou usar complexos(a paixao de JP): Seja a expressao f(x)=x^2+x+1.Vamos fatora-la em R+Ri,i^2+1=0. Defina cis x=sen x+i*cos x=e^(ix). Entao w=(cis(2*pi/3)) e wbarra=(cis(4*pi)/3) sao zeros de f. Para as raizes de x^6+x^3+1,ache as raizes cubicas de w e wbarra. Te mais!!!!!!!!!!!!!!!! -- Mensagem original -- >Olá amigos, >Estou de volta com outro exercício(+difícil na minha opinião) >Meu professor disse que nem ele consegue fatorar a seguinte expressão em >IR: >x^6 + (xy)^3 + y^6 > >Abraços, >Bruno > TRANSIRE SVVM PECTVS MVNDOQUE POTIRE CONGREGATI EX TOTO ORBE MATHEMATICI OB SCRIPTA INSIGNIA TRIBVERE Medalha Fields(John Charles Fields) ------------------------------------------ Use o melhor sistema de busca da Internet Radar UOL - http://www.radaruol.com.br ========================================================================= Instruções para entrar na lista, sair da lista e usar a lista em http://www.mat.puc-rio.br/~nicolau/olimp/obm-l.html O administrador desta lista é ========================================================================= From owner-obm-l@sucuri.mat.puc-rio.br Mon Jun 10 16:50:20 2002 Return-Path: Received: (from majordom@localhost) by sucuri.mat.puc-rio.br (8.9.3/8.9.3) id QAA22918 for obm-l-list; Mon, 10 Jun 2002 16:50:03 -0300 Received: from candeias.terra.com.br (candeias.terra.com.br [200.176.3.18]) by sucuri.mat.puc-rio.br (8.9.3/8.9.3) with ESMTP id QAA22911 for ; Mon, 10 Jun 2002 16:49:57 -0300 Received: from pacuiba.terra.com.br (pacuiba.terra.com.br [200.176.3.40]) by candeias.terra.com.br (Postfix) with ESMTP id 3455743DE4 for ; Mon, 10 Jun 2002 16:37:07 -0300 (EST) Received: from stabel (dl-nas3-poa-C89A0615.p001.terra.com.br [200.154.6.21]) (authenticated user dudasta) by pacuiba.terra.com.br (Postfix) with ESMTP id A1449800A for ; Mon, 10 Jun 2002 16:37:04 -0300 (EST) Message-ID: <002001c210b6$32b796e0$15069ac8@stabel> From: "Eduardo Casagrande Stabel" To: References: Subject: [obm-l] =?iso-8859-1?Q?Re:_=5Bobm-l=5D_D=FAvida..?= Date: Mon, 10 Jun 2002 16:34:16 -0300 MIME-Version: 1.0 Content-Type: text/plain; charset="iso-8859-1" Content-Transfer-Encoding: 8bit X-Priority: 3 X-MSMail-Priority: Normal X-Mailer: Microsoft Outlook Express 6.00.2600.0000 X-MimeOLE: Produced By Microsoft MimeOLE V6.00.2600.0000 Sender: owner-obm-l@sucuri.mat.puc-rio.br Precedence: bulk Reply-To: obm-l@mat.puc-rio.br Caro Thiago Lima, comece em P1 e percorra (como o ponteiro do relogio) todo o circulo. Voce vai passar por P2, P3, ... Se um certo Pn coincidir com P1, vai ter dado uma quantidade k de voltas completas, ou seja 35*(n-1) = 360*k 5*7*(n-1) = 72*5*k 7*(n-1)=72*k eh preciso, portanto, que k seja multiplo de 7, k=7*q 7*(n-1)=72*7*q (n-1)=72*q o menor valor possivel de q=1, dai k=7 e (n-1)=72*1=72 portanto P73 é o primeiro ponto que coincide com P1. Eduardo Casagrande Stabel. Porto Alegre, RS. From: "leon-17" > OBRIGADO DOUGLAS PELA RESPOSTA. > > Tenho outra dúvida (minha vida é repleta dela). > Os pontos P1, P2, P3, ... estão nesta ordem sobre uma > circunferência e são tais que o arco que une cada ponto > ao seguinte mede 35°. O menor valor de n>1 tal que Pn > coincide com P1 é... > > Alguém por favor me livre desta ignorância sobre > circunferências e me ajude nesta questão. Obrigado > antecipado.. > > Thiago Lima > > > __________________________________________________________________________ > Quer ter seu próprio endereço na Internet? > Garanta já o seu e ainda ganhe cinco e-mails personalizados. > DomíniosBOL - http://dominios.bol.com.br > > > ========================================================================= > Instruções para entrar na lista, sair da lista e usar a lista em > http://www.mat.puc-rio.br/~nicolau/olimp/obm-l.html > O administrador desta lista é > ========================================================================= > > ========================================================================= Instruções para entrar na lista, sair da lista e usar a lista em http://www.mat.puc-rio.br/~nicolau/olimp/obm-l.html O administrador desta lista é ========================================================================= From owner-obm-l@sucuri.mat.puc-rio.br Mon Jun 10 17:34:27 2002 Return-Path: Received: (from majordom@localhost) by sucuri.mat.puc-rio.br (8.9.3/8.9.3) id RAA23768 for obm-l-list; Mon, 10 Jun 2002 17:33:41 -0300 Received: from regusnet.centri.net (regusnet.centri.net [194.73.67.42]) by sucuri.mat.puc-rio.br (8.9.3/8.9.3) with ESMTP id RAA23764 for ; Mon, 10 Jun 2002 17:33:37 -0300 Received: from jjunior (unknown [200.198.83.126]) by regusnet.centri.net (Postfix) with ESMTP id 2B3DA2727D for ; Mon, 10 Jun 2002 22:21:06 +0100 (BST) From: "Jose Jayme Moraes Junior" To: Subject: [obm-l] =?iso-8859-1?Q?RE:_=5Bobm-l=5D_D=FAvida..?= Date: Mon, 10 Jun 2002 17:25:54 -0300 Organization: M13 Message-ID: <004e01c210bd$078698c0$0479a8c0@jjunior> MIME-Version: 1.0 Content-Type: text/plain; charset="iso-8859-1" X-Priority: 3 (Normal) X-MSMail-Priority: Normal X-Mailer: Microsoft Outlook, Build 10.0.2616 Importance: Normal In-Reply-To: X-MimeOLE: Produced By Microsoft MimeOLE V6.00.2600.0000 Content-Transfer-Encoding: 8bit X-MIME-Autoconverted: from quoted-printable to 8bit by sucuri.mat.puc-rio.br id RAA23766 Sender: owner-obm-l@sucuri.mat.puc-rio.br Precedence: bulk Reply-To: obm-l@mat.puc-rio.br P2 = P1 + 35 graus P3 = P1 + 2 * 35 P4 = P1 + 3 * 35 ...... Pn = P1 + (n-1) * 35 Para que Pn coincida com P1 Pn = P1 + x * 360 P1 + x * 360 = P1 + (n-1) * 35 n = x * (360/35) + 1 Como n é inteiro temos que x deve ser um divisor de 35 (1, 5, 7, 35). Além disso, o resultado da divisão de 35/x deve ser divisor de 360. Opções: x = 35 , n = 361 x = 7 , n = (360/5) + 1 = 73 x = 5, n = (360/7) + 1 = 52,428.. (não é inteiro) x = 1, n = (360/35) + 1 = 11,285.. (não é inteiro) Resposta: n = 73. -----Original Message----- From: owner-obm-l@sucuri.mat.puc-rio.br [mailto:owner-obm-l@sucuri.mat.puc-rio.br] On Behalf Of leon-17 Sent: segunda-feira, 10 de junho de 2002 15:19 To: obm-l@mat.puc-rio.br Subject: [obm-l] Dúvida.. OBRIGADO DOUGLAS PELA RESPOSTA. Tenho outra dúvida (minha vida é repleta dela). Os pontos P1, P2, P3, ... estão nesta ordem sobre uma circunferência e são tais que o arco que une cada ponto ao seguinte mede 35°. O menor valor de n>1 tal que Pn coincide com P1 é... Alguém por favor me livre desta ignorância sobre circunferências e me ajude nesta questão. Obrigado antecipado.. Thiago Lima ________________________________________________________________________ __ Quer ter seu próprio endereço na Internet? Garanta já o seu e ainda ganhe cinco e-mails personalizados. DomíniosBOL - http://dominios.bol.com.br ======================================================================== = Instruções para entrar na lista, sair da lista e usar a lista em http://www.mat.puc-rio.br/~nicolau/olimp/obm-l.html O administrador desta lista é ======================================================================== = ========================================================================= Instruções para entrar na lista, sair da lista e usar a lista em http://www.mat.puc-rio.br/~nicolau/olimp/obm-l.html O administrador desta lista é ========================================================================= From owner-obm-l@sucuri.mat.puc-rio.br Mon Jun 10 17:44:55 2002 Return-Path: Received: (from majordom@localhost) by sucuri.mat.puc-rio.br (8.9.3/8.9.3) id RAA23979 for obm-l-list; Mon, 10 Jun 2002 17:44:32 -0300 Received: from www.zipmail.com.br (smtp.zipmail.com.br [200.187.242.10]) by sucuri.mat.puc-rio.br (8.9.3/8.9.3) with ESMTP id RAA23965 for ; Mon, 10 Jun 2002 17:44:28 -0300 From: ghaeser@zipmail.com.br Received: from [143.106.22.157] by www.zipmail.com.br with HTTP; Mon, 10 Jun 2002 17:31:49 -0300 Message-ID: <3D04E07700000709@www.zipmail.com.br> Date: Mon, 10 Jun 2002 21:31:49 +0100 In-Reply-To: <3D00E61A00003E82@www.zipmail.com.br> Subject: [obm-l] =?iso-8859-1?Q?Polinomio?= To: obm-l@mat.puc-rio.br MIME-Version: 1.0 Content-Type: text/plain; charset="iso-8859-1" Content-Transfer-Encoding: 8bit X-MIME-Autoconverted: from quoted-printable to 8bit by sucuri.mat.puc-rio.br id RAA23971 Sender: owner-obm-l@sucuri.mat.puc-rio.br Precedence: bulk Reply-To: obm-l@mat.puc-rio.br Prove ou dê um contra-exemplo: Seja P um polinômio de grau n entao P pode ser escrito como: P(x)=sum((x,k)*sum((k,j)*(-1)^j*P(j))), onde a soma interna é de j=0,..,k e a externa de k=0,..,n. e (z,y) é o numero binomial z!/[y!*(z-y)!] obrigado!! "Mathematicus nascitur, non fit" Matemáticos não são feitos, eles nascem --------------------------------------- Gabriel Haeser www.gabas.cjb.net ------------------------------------------ Use o melhor sistema de busca da Internet Radar UOL - http://www.radaruol.com.br ========================================================================= Instruções para entrar na lista, sair da lista e usar a lista em http://www.mat.puc-rio.br/~nicolau/olimp/obm-l.html O administrador desta lista é ========================================================================= From owner-obm-l@sucuri.mat.puc-rio.br Mon Jun 10 21:23:00 2002 Return-Path: Received: (from majordom@localhost) by sucuri.mat.puc-rio.br (8.9.3/8.9.3) id VAA25876 for obm-l-list; Mon, 10 Jun 2002 21:22:00 -0300 Received: from gorgo.centroin.com.br (gorgo.centroin.com.br [200.225.63.128]) by sucuri.mat.puc-rio.br (8.9.3/8.9.3) with ESMTP id VAA25873 for ; Mon, 10 Jun 2002 21:21:58 -0300 Received: from centroin.com.br (du97b.rjo.centroin.com.br [200.225.57.97]) (authenticated bits=0) by gorgo.centroin.com.br (8.12.2/8.12.1) with ESMTP id g5B09rko018536 for ; Mon, 10 Jun 2002 21:09:54 -0300 (BRT) Message-ID: <3D05401C.5060507@centroin.com.br> Date: Mon, 10 Jun 2002 21:11:08 -0300 From: Augusto =?ISO-8859-1?Q?C=E9sar?= Morgado User-Agent: Mozilla/5.0 (Windows; U; Win98; en-US; rv:0.9.4.1) Gecko/20020508 Netscape6/6.2.3 X-Accept-Language: en-us MIME-Version: 1.0 To: obm-l@mat.puc-rio.br Subject: Re: [obm-l] =?ISO-8859-1?Q?D=FAvida=2E=2E?= References: <200206101933.QAA22627@sucuri.mat.puc-rio.br> Content-Type: multipart/alternative; boundary="------------020006030300090807030304" Sender: owner-obm-l@sucuri.mat.puc-rio.br Precedence: bulk Reply-To: obm-l@mat.puc-rio.br --------------020006030300090807030304 Content-Type: text/plain; charset=ISO-8859-1; format=flowed Content-Transfer-Encoding: 8bit Este problema passou pela lista ha uns dois dias atras. A resposta nao eh 72 e sim 73. alvie@ig.com.br wrote: >Bom, acho que é assim: > >Repare que um ponto é marcado na circunferencia, dai, "espaçados" de 35 >graus, novos pontos sao acrescentados... Deseja-se saber qual desses novos >pontos que ao ser colocado coincide com o primeiro... > >A circunferencia tem 360 graus, logo: > >mmc(360,35) = 2520 (em graus) > >2520 /35 = 72 > >Assim, o menor n é 72... > >Acho que é assim... >Desculpem se errei... > > > > >Em 10 Jun 2002, obm-l@mat.puc-rio.br escreveu: > >>OBRIGADO DOUGLAS PELA RESPOSTA. >> >>Tenho outra dúvida (minha vida é repleta dela). >>Os pontos P1, P2, P3, ... estão nesta ordem sobre uma >>circunferência e são tais que o arco que une cada ponto >>ao seguinte mede 35°. O menor valor de n>1 tal que Pn >>coincide com P1 é... >> >>Alguém por favor me livre desta ignorância sobre >>circunferências e me ajude nesta questão. Obrigado >>antecipado.. >> >>Thiago Lima >> >>__________________________________________________________________________ >>Quer ter seu próprio endereço na Internet? >>Garanta já o seu e ainda ganhe cinco e-mails personalizados. >>DomíniosBOL - http://dominios.bol.com.br >> >>========================================================================= >>Instruções para entrar na lista, sair da lista e usar a lista em >>http://www.mat.puc-rio.br/~nicolau/olimp/obm-l.html >>O administrador desta lista é >>========================================================================= >> >>---------- >> > >_________________________________________________________ >Oi! Você quer um iG-mail gratuito? >Então clique aqui: http://registro.ig.com.br/ > >========================================================================= >Instruções para entrar na lista, sair da lista e usar a lista em >http://www.mat.puc-rio.br/~nicolau/olimp/obm-l.html >O administrador desta lista é >========================================================================= > > --------------020006030300090807030304 Content-Type: text/html; charset=us-ascii Content-Transfer-Encoding: 7bit Este problema passou pela lista ha uns dois dias atras. A resposta nao eh 72 e sim 73.

alvie@ig.com.br wrote:
Bom, acho que é assim: 

Repare que um ponto é marcado na circunferencia, dai, "espaçados" de 35
graus, novos pontos sao acrescentados... Deseja-se saber qual desses novos
pontos que ao ser colocado coincide com o primeiro...

A circunferencia tem 360 graus, logo:

mmc(360,35) = 2520 (em graus)

2520 /35 = 72

Assim, o menor n é 72...

Acho que é assim...
Desculpem se errei...




Em 10 Jun 2002, obm-l@mat.puc-rio.br escreveu:

OBRIGADO DOUGLAS PELA RESPOSTA. 

Tenho outra dúvida (minha vida é repleta dela).
Os pontos P1, P2, P3, ... estão nesta ordem sobre uma
circunferência e são tais que o arco que une cada ponto
ao seguinte mede 35°. O menor valor de n>1 tal que Pn
coincide com P1 é...

Alguém por favor me livre desta ignorância sobre
circunferências e me ajude nesta questão. Obrigado
antecipado..

Thiago Lima

__________________________________________________________________________
Quer ter seu próprio endereço na Internet?
Garanta já o seu e ainda ganhe cinco e-mails personalizados.
DomíniosBOL - http://dominios.bol.com.br

=========================================================================
Instruções para entra r na lista, sair da lista e usar a lista em
http://www.mat.puc-rio.br/~nicolau/olimp/obm-l.html
O administrador desta lista é
=========================================================================

----------

_________________________________________________________
Oi! Você quer um iG-mail gratuito?
Então clique aqui: http://registro.ig.com.br/

=========================================================================
Instruções para entrar na lista, sair da lista e usar a lista em
http://www.mat.puc-rio.br/~nicolau/olimp/obm-l.html
O administrador desta lista é <nicolau@mat.puc-rio.br>
=========================================================================



--------------020006030300090807030304-- ========================================================================= Instruções para entrar na lista, sair da lista e usar a lista em http://www.mat.puc-rio.br/~nicolau/olimp/obm-l.html O administrador desta lista é ========================================================================= From owner-obm-l@sucuri.mat.puc-rio.br Mon Jun 10 22:40:39 2002 Return-Path: Received: (from majordom@localhost) by sucuri.mat.puc-rio.br (8.9.3/8.9.3) id WAA26396 for obm-l-list; Mon, 10 Jun 2002 22:40:18 -0300 Received: from smtp.estaminas.com.br (smtp.estaminas.com.br [200.188.191.51]) by sucuri.mat.puc-rio.br (8.9.3/8.9.3) with ESMTP id WAA26393 for ; Mon, 10 Jun 2002 22:40:16 -0300 Received: from dfabiano (dial-tm-ip-1-82.estaminas.com.br [200.195.86.83]) by smtp.estaminas.com.br (8.11.0/8.11.0) with SMTP id g5B1W2703002 for ; Mon, 10 Jun 2002 22:32:04 -0300 Message-ID: <009c01c210e7$8109fb40$5356c3c8@dfabiano> From: "Douglas Carvalho" To: References: Subject: [obm-l] =?iso-8859-1?Q?Re:_=5Bobm-l=5D_D=FAvida..?= Date: Mon, 10 Jun 2002 22:28:19 -0300 MIME-Version: 1.0 Content-Type: text/plain; charset="iso-8859-1" Content-Transfer-Encoding: 8bit X-Priority: 3 X-MSMail-Priority: Normal X-Mailer: Microsoft Outlook Express 5.00.2615.200 X-MimeOLE: Produced By Microsoft MimeOLE V5.00.2615.200 Sender: owner-obm-l@sucuri.mat.puc-rio.br Precedence: bulk Reply-To: obm-l@mat.puc-rio.br > Tenho outra dúvida (minha vida é repleta dela). > Os pontos P1, P2, P3, ... estão nesta ordem sobre uma > circunferência e são tais que o arco que une cada ponto > ao seguinte mede 35°. O menor valor de n>1 tal que Pn > coincide com P1 é... 35º * nº de arcos = 360º * nº de voltas dadas Sendo x o nº de arcos percorridos e y o nº de voltas dadas P1 = Pn 35x = 360y 7x = 72y x = 72, portanto n = 73 Após 72 arcos (sendo q este ultimo arco é determinado pelos pontos P72 e P73) 7 voltas sao completadas, coincidindo o ponto P73 com P1. []'s Douglas Carvalho ========================================================================= Instruções para entrar na lista, sair da lista e usar a lista em http://www.mat.puc-rio.br/~nicolau/olimp/obm-l.html O administrador desta lista é ========================================================================= From owner-obm-l@sucuri.mat.puc-rio.br Tue Jun 11 01:56:59 2002 Return-Path: Received: (from majordom@localhost) by sucuri.mat.puc-rio.br (8.9.3/8.9.3) id BAA27663 for obm-l-list; Tue, 11 Jun 2002 01:56:18 -0300 Received: from hotmail.com (f178.pav1.hotmail.com [64.4.31.178]) by sucuri.mat.puc-rio.br (8.9.3/8.9.3) with ESMTP id BAA27660 for ; Tue, 11 Jun 2002 01:56:15 -0300 Received: from mail pickup service by hotmail.com with Microsoft SMTPSVC; Mon, 10 Jun 2002 21:43:42 -0700 Received: from 200.199.179.175 by pv1fd.pav1.hotmail.msn.com with HTTP; Tue, 11 Jun 2002 04:43:41 GMT X-Originating-IP: [200.199.179.175] From: "Adherbal Rocha Filho" To: obm-l@mat.puc-rio.br Subject: [obm-l] =?iso-8859-1?B?dC4gZG9zIG66cw==?= Date: Tue, 11 Jun 2002 04:43:41 +0000 Mime-Version: 1.0 Content-Type: text/plain; charset=iso-8859-1; format=flowed Message-ID: X-OriginalArrivalTime: 11 Jun 2002 04:43:42.0007 (UTC) FILETIME=[8FCE6470:01C21102] Sender: owner-obm-l@sucuri.mat.puc-rio.br Precedence: bulk Reply-To: obm-l@mat.puc-rio.br ajuda: Mostrar q se o primo p é tal q p==3(mod4), então a equação p^2= a^2 +b^2 possui solução inteira mostre q todo quadrado perfeito pode ser representado como soma dos quadrados de racionais ,naum inteiros, r e s. valeu! _________________________________________________________________ Chegou o novo MSN Explorer. Instale já. É gratuito: http://explorer.msn.com.br ========================================================================= Instruções para entrar na lista, sair da lista e usar a lista em http://www.mat.puc-rio.br/~nicolau/olimp/obm-l.html O administrador desta lista é ========================================================================= From owner-obm-l@sucuri.mat.puc-rio.br Tue Jun 11 11:07:40 2002 Return-Path: Received: (from majordom@localhost) by sucuri.mat.puc-rio.br (8.9.3/8.9.3) id LAA30575 for obm-l-list; Tue, 11 Jun 2002 11:05:46 -0300 Received: from bidu.ime.usp.br (bidu.ime.usp.br [143.107.45.12]) by sucuri.mat.puc-rio.br (8.9.3/8.9.3) with SMTP id LAA30567 for ; Tue, 11 Jun 2002 11:05:38 -0300 Received: (qmail 28676 invoked from network); 11 Jun 2002 13:53:01 -0000 Received: from fradim.ime.usp.br (143.107.45.37) by bidu.ime.usp.br with SMTP; 11 Jun 2002 13:53:01 -0000 Received: (qmail 24958 invoked by uid 1604); 11 Jun 2002 13:50:25 -0000 Date: Tue, 11 Jun 2002 10:50:25 -0300 (EST) From: Salvador Addas Zanata X-Sender: sazanata@fradim To: obm-l@mat.puc-rio.br Subject: Re: [obm-l] =?iso-8859-1?B?dC4gZG9zIG66cw==?= In-Reply-To: Message-ID: MIME-Version: 1.0 Content-Type: TEXT/PLAIN; charset=ISO-8859-1 Content-Transfer-Encoding: 8bit X-MIME-Autoconverted: from QUOTED-PRINTABLE to 8bit by sucuri.mat.puc-rio.br id LAA30572 Sender: owner-obm-l@sucuri.mat.puc-rio.br Precedence: bulk Reply-To: obm-l@mat.puc-rio.br O primeiro problema so pode ter solucao se p=4n+1. Para ver isso, observe que a deve ser par e b impar. Logo a^2+b^2 e da forma: 4c^2+4d^2+4c+1, que e da forma 4n+1. De fato todo primo da forma 4n+1 se escreve de um unico jeito como a soma de 2 quadrados. Tem um livro chamado "100 great elementary problems: Their history and solutions" Heinrich Dorrie, que tem essa prova e muitas outras bacanas. Alias esse livro apresenta as "melhores" provas de cada problema. E da Dover e nao e dificil de achar. Abraco, Salvador On Tue, 11 Jun 2002, Adherbal Rocha Filho wrote: > > ajuda: > > Mostrar q se o primo p é tal q p==3(mod4), então a equação p^2= a^2 +b^2 > possui solução inteira > > mostre q todo quadrado perfeito pode ser representado como soma dos > quadrados de racionais ,naum inteiros, r e s. > > valeu! > > _________________________________________________________________ > Chegou o novo MSN Explorer. Instale já. É gratuito: > http://explorer.msn.com.br > > ========================================================================= > Instruções para entrar na lista, sair da lista e usar a lista em > http://www.mat.puc-rio.br/~nicolau/olimp/obm-l.html > O administrador desta lista é > ========================================================================= > ========================================================================= Instruções para entrar na lista, sair da lista e usar a lista em http://www.mat.puc-rio.br/~nicolau/olimp/obm-l.html O administrador desta lista é ========================================================================= From owner-obm-l@sucuri.mat.puc-rio.br Tue Jun 11 13:22:12 2002 Return-Path: Received: (from majordom@localhost) by sucuri.mat.puc-rio.br (8.9.3/8.9.3) id NAA31954 for obm-l-list; Tue, 11 Jun 2002 13:19:27 -0300 Received: from www.zipmail.com.br (smtp.zipmail.com.br [200.187.242.10]) by sucuri.mat.puc-rio.br (8.9.3/8.9.3) with ESMTP id NAA31951 for ; Tue, 11 Jun 2002 13:19:24 -0300 From: yurigomes@zipmail.com.br Received: from [200.151.158.20] by www.zipmail.com.br with HTTP; Tue, 11 Jun 2002 13:06:52 -0300 Message-ID: <3CFCCE890000430C@www.zipmail.com.br> Date: Tue, 11 Jun 2002 13:06:52 -0300 In-Reply-To: Subject: [obm-l] =?iso-8859-1?Q?Re=3A=20=5Bobm=2Dl=5D=20t=2E=20dos=20n=BAs?= To: obm-l@mat.puc-rio.br MIME-Version: 1.0 Content-Type: text/plain; charset="iso-8859-1" Content-Transfer-Encoding: 8bit X-MIME-Autoconverted: from quoted-printable to 8bit by sucuri.mat.puc-rio.br id NAA31952 Sender: owner-obm-l@sucuri.mat.puc-rio.br Precedence: bulk Reply-To: obm-l@mat.puc-rio.br Oi Salvador, Vc confundiu o problema. A equação é p^2= a^2 = b^2 e não p= a^2 = b^2 De fato, no livro Introdução à Teoria dos Números, capítulo 7, existe um teorema que diz que um inteiro n é representado como soma de dois quadrados se e somente se os expoentes dos primos congruentes a 3 mod 4 que dividem n são pares. Logo, p^2 pode ser representado dessa forma Ateh mais -- Mensagem original -- > > >O primeiro problema so pode ter solucao se p=4n+1. > >Para ver isso, observe que a deve ser par e b impar. Logo a^2+b^2 e da >forma: 4c^2+4d^2+4c+1, que e da forma 4n+1. > >De fato todo primo da forma 4n+1 se escreve de um unico jeito como a soma >de 2 quadrados. Tem um livro chamado "100 great elementary problems: Their >history and solutions" Heinrich Dorrie, que tem essa prova e muitas outras >bacanas. Alias esse livro apresenta as "melhores" provas de cada >problema. E da Dover e nao e dificil de achar. > > >Abraco, > >Salvador > > >On Tue, 11 Jun 2002, Adherbal Rocha Filho wrote: > >> >> ajuda: >> >> Mostrar q se o primo p é tal q p==3(mod4), então a equação p^2= a^2 +b^2 > >> possui solução inteira >> >> mostre q todo quadrado perfeito pode ser representado como soma dos >> quadrados de racionais ,naum inteiros, r e s. >> >> valeu! >> >> _________________________________________________________________ >> Chegou o novo MSN Explorer. Instale já. É gratuito: >> http://explorer.msn.com.br >> >> ========================================================================= >> Instruções para entrar na lista, sair da lista e usar a lista em >> http://www.mat.puc-rio.br/~nicolau/olimp/obm-l.html >> O administrador desta lista é >> ========================================================================= >> > >========================================================================= >Instruções para entrar na lista, sair da lista e usar a lista em >http://www.mat.puc-rio.br/~nicolau/olimp/obm-l.html >O administrador desta lista é >========================================================================= > []'s, Yuri ICQ: 64992515 ------------------------------------------ Use o melhor sistema de busca da Internet Radar UOL - http://www.radaruol.com.br ========================================================================= Instruções para entrar na lista, sair da lista e usar a lista em http://www.mat.puc-rio.br/~nicolau/olimp/obm-l.html O administrador desta lista é ========================================================================= From owner-obm-l@sucuri.mat.puc-rio.br Tue Jun 11 13:49:16 2002 Return-Path: Received: (from majordom@localhost) by sucuri.mat.puc-rio.br (8.9.3/8.9.3) id NAA32409 for obm-l-list; Tue, 11 Jun 2002 13:48:21 -0300 Received: from www.zipmail.com.br (smtp.zipmail.com.br [200.187.242.10]) by sucuri.mat.puc-rio.br (8.9.3/8.9.3) with ESMTP id NAA32406 for ; Tue, 11 Jun 2002 13:48:17 -0300 From: yurigomes@zipmail.com.br Received: from [200.151.158.189] by www.zipmail.com.br with HTTP; Tue, 11 Jun 2002 13:35:43 -0300 Message-ID: <3CFCCE890000439A@www.zipmail.com.br> Date: Tue, 11 Jun 2002 13:35:43 -0300 In-Reply-To: Subject: [obm-l] =?iso-8859-1?Q?Re=3A=20=5Bobm=2Dl=5D=20t=2E=20dos=20n=BAs?= To: obm-l@mat.puc-rio.br MIME-Version: 1.0 Content-Type: text/plain; charset="iso-8859-1" Content-Transfer-Encoding: 8bit X-MIME-Autoconverted: from quoted-printable to 8bit by sucuri.mat.puc-rio.br id NAA32407 Sender: owner-obm-l@sucuri.mat.puc-rio.br Precedence: bulk Reply-To: obm-l@mat.puc-rio.br Obs: o teorema anterior afirma que existem INTEIROS a e b. No problema p^2= a^2 + b^2 tem (0, p) como soluções inteiras. Se formos procurar soluções naturais, deveremos ter p|a^2 + b^2 . Suponha que p não divide a. Então seja c o inverso de a mod. p ( que existe, pois (a, p) ). Daí, p|(ac)^2 + (bc)^2, donde (bc)^2 = -1 ( mod p ). Mas o símbolo de Legendre (-1/p) é igual a (-1)^[(p-1)/2], que é -1 ( pois (p-1)/2 é ímpar ), absurdo!! Logo, p|a e p|b e assim, se a e b são maiores que zero, temos a^2 + b^2 > p^2. -- Mensagem original -- > > >O primeiro problema so pode ter solucao se p=4n+1. > >Para ver isso, observe que a deve ser par e b impar. Logo a^2+b^2 e da >forma: 4c^2+4d^2+4c+1, que e da forma 4n+1. > >De fato todo primo da forma 4n+1 se escreve de um unico jeito como a soma >de 2 quadrados. Tem um livro chamado "100 great elementary problems: Their >history and solutions" Heinrich Dorrie, que tem essa prova e muitas outras >bacanas. Alias esse livro apresenta as "melhores" provas de cada >problema. E da Dover e nao e dificil de achar. > > >Abraco, > >Salvador > > >On Tue, 11 Jun 2002, Adherbal Rocha Filho wrote: > >> >> ajuda: >> >> Mostrar q se o primo p é tal q p==3(mod4), então a equação p^2= a^2 +b^2 > >> possui solução inteira >> >> mostre q todo quadrado perfeito pode ser representado como soma dos >> quadrados de racionais ,naum inteiros, r e s. >> >> valeu! >> >> _________________________________________________________________ >> Chegou o novo MSN Explorer. Instale já. É gratuito: >> http://explorer.msn.com.br >> >> ========================================================================= >> Instruções para entrar na lista, sair da lista e usar a lista em >> http://www.mat.puc-rio.br/~nicolau/olimp/obm-l.html >> O administrador desta lista é >> ========================================================================= >> > >========================================================================= >Instruções para entrar na lista, sair da lista e usar a lista em >http://www.mat.puc-rio.br/~nicolau/olimp/obm-l.html >O administrador desta lista é >========================================================================= > []'s, Yuri ICQ: 64992515 ------------------------------------------ Use o melhor sistema de busca da Internet Radar UOL - http://www.radaruol.com.br ========================================================================= Instruções para entrar na lista, sair da lista e usar a lista em http://www.mat.puc-rio.br/~nicolau/olimp/obm-l.html O administrador desta lista é ========================================================================= From owner-obm-l@sucuri.mat.puc-rio.br Tue Jun 11 14:02:52 2002 Return-Path: Received: (from majordom@localhost) by sucuri.mat.puc-rio.br (8.9.3/8.9.3) id OAA32689 for obm-l-list; Tue, 11 Jun 2002 14:01:56 -0300 Received: (from nicolau@localhost) by sucuri.mat.puc-rio.br (8.9.3/8.9.3) id OAA32685 for obm-l@mat.puc-rio.br; Tue, 11 Jun 2002 14:01:56 -0300 Date: Tue, 11 Jun 2002 14:01:56 -0300 From: "Nicolau C. Saldanha" To: obm-l@mat.puc-rio.br Subject: [obm-l] Re: =?iso-8859-1?Q?=5Bobm-l=5D_t=2E_dos_n=BAs?= Message-ID: <20020611140156.B32557@sucuri.mat.puc-rio.br> References: Mime-Version: 1.0 Content-Type: text/plain; charset=iso-8859-1 Content-Disposition: inline Content-Transfer-Encoding: 8bit User-Agent: Mutt/1.2.5i In-Reply-To: ; from adherbalmat@hotmail.com on Tue, Jun 11, 2002 at 04:43:41AM +0000 Sender: owner-obm-l@sucuri.mat.puc-rio.br Precedence: bulk Reply-To: obm-l@mat.puc-rio.br On Tue, Jun 11, 2002 at 04:43:41AM +0000, Adherbal Rocha Filho wrote: > > ajuda: > > Mostrar q se o primo p é tal q p==3(mod4), então a equação p^2= a^2 +b^2 > possui solução inteira Deve haver um engano, vale o contrário: se p é da forma 4k+1 (e não 4k+3) então p pode ser escrito da forma p = a^2 + b^2. Isto segue de um estudo dos inteiros de Gauss. Note que a equação p^2 = a^2 + b^2 sempre admite a solução trivial a = p, b = 0. []s, N. ========================================================================= Instruções para entrar na lista, sair da lista e usar a lista em http://www.mat.puc-rio.br/~nicolau/olimp/obm-l.html O administrador desta lista é ========================================================================= From owner-obm-l@sucuri.mat.puc-rio.br Tue Jun 11 14:41:56 2002 Return-Path: Received: (from majordom@localhost) by sucuri.mat.puc-rio.br (8.9.3/8.9.3) id OAA01252 for obm-l-list; Tue, 11 Jun 2002 14:40:06 -0300 Received: from bidu.ime.usp.br (bidu.ime.usp.br [143.107.45.12]) by sucuri.mat.puc-rio.br (8.9.3/8.9.3) with SMTP id OAA01247 for ; Tue, 11 Jun 2002 14:39:57 -0300 Received: (qmail 25411 invoked from network); 11 Jun 2002 17:27:22 -0000 Received: from mafalda.ime.usp.br (143.107.45.13) by bidu.ime.usp.br with SMTP; 11 Jun 2002 17:27:22 -0000 Received: (qmail 26233 invoked by uid 1604); 11 Jun 2002 17:27:16 -0000 Date: Tue, 11 Jun 2002 14:27:16 -0300 (EST) From: Salvador Addas Zanata X-Sender: sazanata@mafalda To: obm-l@mat.puc-rio.br Subject: Re: [obm-l] =?iso-8859-1?Q?Re=3A=20=5Bobm=2Dl=5D=20t=2E=20dos=20n=BAs?= In-Reply-To: <3CFCCE890000430C@www.zipmail.com.br> Message-ID: MIME-Version: 1.0 Content-Type: TEXT/PLAIN; charset=ISO-8859-1 Content-Transfer-Encoding: 8bit X-MIME-Autoconverted: from QUOTED-PRINTABLE to 8bit by sucuri.mat.puc-rio.br id OAA01250 Sender: owner-obm-l@sucuri.mat.puc-rio.br Precedence: bulk Reply-To: obm-l@mat.puc-rio.br Foi mal, nao vi que p ia ao quadrado... Desculpem, Salvador On Tue, 11 Jun 2002 yurigomes@zipmail.com.br wrote: > Oi Salvador, > Vc confundiu o problema. A equação é > p^2= a^2 = b^2 e não p= a^2 = b^2 > De fato, no livro Introdução à Teoria dos Números, capítulo 7, existe um > teorema que diz que um inteiro n é representado como soma de dois quadrados > se e somente se os expoentes dos primos congruentes a 3 mod 4 que dividem > n são pares. Logo, p^2 pode ser representado dessa forma > > Ateh mais > -- Mensagem original -- > > > > > > >O primeiro problema so pode ter solucao se p=4n+1. > > > >Para ver isso, observe que a deve ser par e b impar. Logo a^2+b^2 e da > >forma: 4c^2+4d^2+4c+1, que e da forma 4n+1. > > > >De fato todo primo da forma 4n+1 se escreve de um unico jeito como a soma > >de 2 quadrados. Tem um livro chamado "100 great elementary problems: Their > >history and solutions" Heinrich Dorrie, que tem essa prova e muitas outras > >bacanas. Alias esse livro apresenta as "melhores" provas de cada > >problema. E da Dover e nao e dificil de achar. > > > > > >Abraco, > > > >Salvador > > > > > >On Tue, 11 Jun 2002, Adherbal Rocha Filho wrote: > > > >> > >> ajuda: > >> > >> Mostrar q se o primo p é tal q p==3(mod4), então a equação p^2= a^2 +b^2 > > > >> possui solução inteira > >> > >> mostre q todo quadrado perfeito pode ser representado como soma dos > >> quadrados de racionais ,naum inteiros, r e s. > >> > >> valeu! > >> > >> _________________________________________________________________ > >> Chegou o novo MSN Explorer. Instale já. É gratuito: > >> http://explorer.msn.com.br > >> > >> ========================================================================= > >> Instruções para entrar na lista, sair da lista e usar a lista em > >> http://www.mat.puc-rio.br/~nicolau/olimp/obm-l.html > >> O administrador desta lista é > >> ========================================================================= > >> > > > >========================================================================= > >Instruções para entrar na lista, sair da lista e usar a lista em > >http://www.mat.puc-rio.br/~nicolau/olimp/obm-l.html > >O administrador desta lista é > >========================================================================= > > > > []'s, Yuri > ICQ: 64992515 > > > ------------------------------------------ > Use o melhor sistema de busca da Internet > Radar UOL - http://www.radaruol.com.br > > > > ========================================================================= > Instruções para entrar na lista, sair da lista e usar a lista em > http://www.mat.puc-rio.br/~nicolau/olimp/obm-l.html > O administrador desta lista é > ========================================================================= > ========================================================================= Instruções para entrar na lista, sair da lista e usar a lista em http://www.mat.puc-rio.br/~nicolau/olimp/obm-l.html O administrador desta lista é ========================================================================= From owner-obm-l@sucuri.mat.puc-rio.br Tue Jun 11 16:33:53 2002 Return-Path: Received: (from majordom@localhost) by sucuri.mat.puc-rio.br (8.9.3/8.9.3) id QAA03076 for obm-l-list; Tue, 11 Jun 2002 16:32:42 -0300 Received: from smtp-31.ig.com.br (smtp-31.ig.com.br [200.226.132.181]) by sucuri.mat.puc-rio.br (8.9.3/8.9.3) with SMTP id QAA03068 for ; Tue, 11 Jun 2002 16:32:24 -0300 Message-Id: <200206111932.QAA03068@sucuri.mat.puc-rio.br> Received: (qmail 16603 invoked from network); 11 Jun 2002 19:19:30 -0000 Received: from unknown (HELO localhost) (200.226.132.27) by smtp-31.ig.com.br with SMTP; 11 Jun 2002 19:19:30 -0000 Date: Tue, 11 Jun 2002 16:19:31 -0300 To: obm-l@mat.puc-rio.br From: "J. A. Tavares" Cc: X-Originating-IP: [200.226.133.253]200.17.67.2 X-Mailer: InMail by Insite - www.insite.com.br X-user: frolstty@ig.com.br MIME-Version: 1.0 Content-type: text/plain Subject: Re: [obm-l] Rafael Santos ;;;; Sender: owner-obm-l@sucuri.mat.puc-rio.br Precedence: bulk Reply-To: obm-l@mat.puc-rio.br Em 09 Jun 2002, obm-l@mat.puc-rio.br escreveu: >--- Jose Augusto escreveu: > >Serve achando o XY e consequentemente X + Y e o >> quadrado deles ... ???? >> > >Lembre-se que (x+y)^2 - 2xy = x^2 + y^2. > >_______________________________________________________________________ Quando eu disse o quadrado deles eu me referia a x^2 + y^2 desculpe a ambiguidade.........! _________________________________________________________ Oi! Você quer um iG-mail gratuito? Então clique aqui: http://registro.ig.com.br/ ========================================================================= Instruções para entrar na lista, sair da lista e usar a lista em http://www.mat.puc-rio.br/~nicolau/olimp/obm-l.html O administrador desta lista é ========================================================================= From owner-obm-l@sucuri.mat.puc-rio.br Tue Jun 11 16:41:57 2002 Return-Path: Received: (from majordom@localhost) by sucuri.mat.puc-rio.br (8.9.3/8.9.3) id QAA03140 for obm-l-list; Tue, 11 Jun 2002 16:38:51 -0300 Received: from web14808.mail.yahoo.com (web14808.mail.yahoo.com [216.136.224.224]) by sucuri.mat.puc-rio.br (8.9.3/8.9.3) with SMTP id QAA03137 for ; Tue, 11 Jun 2002 16:38:48 -0300 Message-ID: <20020611192617.35792.qmail@web14808.mail.yahoo.com> Received: from [200.17.79.42] by web14808.mail.yahoo.com via HTTP; Tue, 11 Jun 2002 16:26:17 ART Date: Tue, 11 Jun 2002 16:26:17 -0300 (ART) From: =?iso-8859-1?q?Ricardo=20Miranda?= Subject: Re: [obm-l] manual_de_indução_matemática To: obm-l@mat.puc-rio.br In-Reply-To: <006701c20dc4$753a47a0$8710dcc8@jf> MIME-Version: 1.0 Content-Type: text/plain; charset=iso-8859-1 Content-Transfer-Encoding: 8bit Sender: owner-obm-l@sucuri.mat.puc-rio.br Precedence: bulk Reply-To: obm-l@mat.puc-rio.br > From: "Augusto Cesar de Oliveira Morgado" > To: > Sent: Thursday, June 06, 2002 6:25 PM > Subject: Re: [obm-l] Livros de Inducao / Analitica > > > > O Luis Lopes escreveu um livro otimo chamado Manual de Induçao > Matemática. > > Morgado Alguem que tenha o livro pode entrar em contato comigo pelo meu e-mail pessoal? Tenho interesse no xerox de algumas partes do livro. ===== []s Ricardo Miranda Matematica - UFV ricardomirandabr@yahoo.com.br http://rm2.hpg.ig.com.br/ _______________________________________________________________________ Copa 2002 Yahoo! - Patrocinador oficial da Copa do Mundo da FIFA 2002 http://br.sports.yahoo.com/fifaworldcup/ ========================================================================= Instruções para entrar na lista, sair da lista e usar a lista em http://www.mat.puc-rio.br/~nicolau/olimp/obm-l.html O administrador desta lista é ========================================================================= From owner-obm-l@sucuri.mat.puc-rio.br Tue Jun 11 16:46:13 2002 Return-Path: Received: (from majordom@localhost) by sucuri.mat.puc-rio.br (8.9.3/8.9.3) id QAA03249 for obm-l-list; Tue, 11 Jun 2002 16:45:49 -0300 Received: from web14803.mail.yahoo.com (web14803.mail.yahoo.com [216.136.224.219]) by sucuri.mat.puc-rio.br (8.9.3/8.9.3) with SMTP id QAA03246 for ; Tue, 11 Jun 2002 16:45:46 -0300 Message-ID: <20020611193314.39492.qmail@web14803.mail.yahoo.com> Received: from [200.17.79.42] by web14803.mail.yahoo.com via HTTP; Tue, 11 Jun 2002 16:33:14 ART Date: Tue, 11 Jun 2002 16:33:14 -0300 (ART) From: =?iso-8859-1?q?Ricardo=20Miranda?= Subject: [obm-l] Duvidas de analitica. To: obm-l@mat.puc-rio.br MIME-Version: 1.0 Content-Type: text/plain; charset=iso-8859-1 Content-Transfer-Encoding: 8bit Sender: owner-obm-l@sucuri.mat.puc-rio.br Precedence: bulk Reply-To: obm-l@mat.puc-rio.br No livro do Elon, no segundo capitulo (Coordenadas no plano), exercicio 4, temos: 4) Para cada uma das equacoes abaixo, descreva o conjunto dos pontos (x,y) cujas coordenadas satisfazem essa equacao: b)y^2-6*x+9=0 c)x^2+y^2+1=0 Nao sei como fazer essas duas letras. A letra b me parece uma curva no R3 (no maple fica algo como um meio copo). A letra c é uma "circunferencia de raio -1", o que para mim eh incoerente. 5) Ql é o ponto da diagonal DELTA (y=x) mais proximo de P(x,y)? Pelo que eu fiz, encontrei que é (x,y-x). Está correto? Alguem pode me ajudar? Ajudaria MUITO se alguem tivesse o gabarito desse livro e pudesse me enviar (assim como outros da lista tb podem querer). Obrigado, como sempre. ===== []s Ricardo Miranda Matematica - UFV ricardomirandabr@yahoo.com.br http://rm2.hpg.ig.com.br/ _______________________________________________________________________ Copa 2002 Yahoo! - Patrocinador oficial da Copa do Mundo da FIFA 2002 http://br.sports.yahoo.com/fifaworldcup/ ========================================================================= Instruções para entrar na lista, sair da lista e usar a lista em http://www.mat.puc-rio.br/~nicolau/olimp/obm-l.html O administrador desta lista é ========================================================================= From owner-obm-l@sucuri.mat.puc-rio.br Tue Jun 11 17:16:24 2002 Return-Path: Received: (from majordom@localhost) by sucuri.mat.puc-rio.br (8.9.3/8.9.3) id RAA04079 for obm-l-list; Tue, 11 Jun 2002 17:16:12 -0300 Received: from smtp-32.ig.com.br (smtp-32.ig.com.br [200.226.132.182]) by sucuri.mat.puc-rio.br (8.9.3/8.9.3) with SMTP id RAA04076 for ; Tue, 11 Jun 2002 17:16:07 -0300 Message-Id: <200206112016.RAA04076@sucuri.mat.puc-rio.br> Received: (qmail 23197 invoked from network); 11 Jun 2002 20:03:55 -0000 Received: from webmail-4.ig.com.br (HELO localhost) (200.226.133.91) by smtp-32.ig.com.br with SMTP; 11 Jun 2002 20:03:55 -0000 Date: Tue, 11 Jun 2002 16:59:41 -0300 To: obm-l@mat.puc-rio.br From: "J. A. Tavares" Cc: X-Originating-IP: [200.226.133.109]200.17.67.2 X-Mailer: InMail by Insite - www.insite.com.br X-user: frolstty@ig.com.br MIME-Version: 1.0 Content-type: text/plain Subject: [obm-l] Olimpiada universitaria ......... Sender: owner-obm-l@sucuri.mat.puc-rio.br Precedence: bulk Reply-To: obm-l@mat.puc-rio.br Mandei um e-mal para O IMPA e nao responderam sobre como cadastrar minha faculdade para a obm e como vai ser realizada , estilo obm niveis 1,2,3????????? Obrigado. _________________________________________________________ Oi! Você quer um iG-mail gratuito? Então clique aqui: http://registro.ig.com.br/ ========================================================================= Instruções para entrar na lista, sair da lista e usar a lista em http://www.mat.puc-rio.br/~nicolau/olimp/obm-l.html O administrador desta lista é ========================================================================= From owner-obm-l@sucuri.mat.puc-rio.br Tue Jun 11 17:56:22 2002 Return-Path: Received: (from majordom@localhost) by sucuri.mat.puc-rio.br (8.9.3/8.9.3) id RAA05010 for obm-l-list; Tue, 11 Jun 2002 17:55:47 -0300 Received: (from nicolau@localhost) by sucuri.mat.puc-rio.br (8.9.3/8.9.3) id RAA05006 for obm-l@mat.puc-rio.br; Tue, 11 Jun 2002 17:55:47 -0300 Date: Tue, 11 Jun 2002 17:55:47 -0300 From: "Nicolau C. Saldanha" To: obm-l@mat.puc-rio.br Subject: Re: [obm-l] Duvidas de analitica. Message-ID: <20020611175547.A4872@sucuri.mat.puc-rio.br> References: <20020611193314.39492.qmail@web14803.mail.yahoo.com> Mime-Version: 1.0 Content-Type: text/plain; charset=iso-8859-1 Content-Disposition: inline Content-Transfer-Encoding: 8bit User-Agent: Mutt/1.2.5i In-Reply-To: <20020611193314.39492.qmail@web14803.mail.yahoo.com>; from ricardomirandabr@yahoo.com.br on Tue, Jun 11, 2002 at 04:33:14PM -0300 Sender: owner-obm-l@sucuri.mat.puc-rio.br Precedence: bulk Reply-To: obm-l@mat.puc-rio.br On Tue, Jun 11, 2002 at 04:33:14PM -0300, Ricardo Miranda wrote: > No livro do Elon, no segundo capitulo (Coordenadas no plano), exercicio 4, > temos: > 4) Para cada uma das equacoes abaixo, descreva o conjunto dos pontos (x,y) > cujas coordenadas satisfazem essa equacao: > > b)y^2-6*x+9=0 Escreva como x = 1/6 y^2 + 3/2, uma parábola. > c)x^2+y^2+1=0 Conjunto vazio. > > Nao sei como fazer essas duas letras. > A letra b me parece uma curva no R3 (no maple fica algo como um meio > copo). R3?!? Só há duas letras, x e y. > A letra c é uma "circunferencia de raio -1", o que para mim eh incoerente. O raio seria sqrt(-1). Isto significa que *nunca* vale x^2 + y^2 + 1 = 0... []s, N. ========================================================================= Instruções para entrar na lista, sair da lista e usar a lista em http://www.mat.puc-rio.br/~nicolau/olimp/obm-l.html O administrador desta lista é ========================================================================= From owner-obm-l@sucuri.mat.puc-rio.br Tue Jun 11 18:02:48 2002 Return-Path: Received: (from majordom@localhost) by sucuri.mat.puc-rio.br (8.9.3/8.9.3) id SAA05229 for obm-l-list; Tue, 11 Jun 2002 18:02:23 -0300 Received: (from nicolau@localhost) by sucuri.mat.puc-rio.br (8.9.3/8.9.3) id SAA05225 for obm-l@mat.puc-rio.br; Tue, 11 Jun 2002 18:02:23 -0300 Date: Tue, 11 Jun 2002 18:02:22 -0300 From: "Nicolau C. Saldanha" To: obm-l@mat.puc-rio.br Subject: Re: [obm-l] Olimpiada universitaria ......... Message-ID: <20020611180222.B4872@sucuri.mat.puc-rio.br> References: <200206112016.RAA04076@sucuri.mat.puc-rio.br> Mime-Version: 1.0 Content-Type: text/plain; charset=iso-8859-1 Content-Disposition: inline Content-Transfer-Encoding: 8bit User-Agent: Mutt/1.2.5i In-Reply-To: <200206112016.RAA04076@sucuri.mat.puc-rio.br>; from frolstty@ig.com.br on Tue, Jun 11, 2002 at 04:59:41PM -0300 Sender: owner-obm-l@sucuri.mat.puc-rio.br Precedence: bulk Reply-To: obm-l@mat.puc-rio.br On Tue, Jun 11, 2002 at 04:59:41PM -0300, J. A. Tavares wrote: > Mandei um e-mal para O IMPA e nao responderam sobre como cadastrar > minha faculdade para a obm e como vai ser realizada , estilo obm niveis > 1,2,3????????? > Obrigado. O nível universitário tem duas fases com datas coincidindo com a 2a e 3a fases dos outros níveis. Aliás, a nossa secretária Nelly trabalha no prédio do Impa e tem endereço . Mas ela não é bem funcionária do Impa, é funcionária da OBM. []s, N. ========================================================================= Instruções para entrar na lista, sair da lista e usar a lista em http://www.mat.puc-rio.br/~nicolau/olimp/obm-l.html O administrador desta lista é ========================================================================= From owner-obm-l@sucuri.mat.puc-rio.br Tue Jun 11 19:03:00 2002 Return-Path: Received: (from majordom@localhost) by sucuri.mat.puc-rio.br (8.9.3/8.9.3) id TAA06331 for obm-l-list; Tue, 11 Jun 2002 19:01:48 -0300 Received: from matinhos.terra.com.br (matinhos.terra.com.br [200.176.3.21]) by sucuri.mat.puc-rio.br (8.9.3/8.9.3) with ESMTP id TAA06327 for ; Tue, 11 Jun 2002 19:01:46 -0300 Received: from pacuiba.terra.com.br (pacuiba.terra.com.br [200.176.3.40]) by matinhos.terra.com.br (Postfix) with ESMTP id 10FA04705F for ; Tue, 11 Jun 2002 18:49:16 -0300 (EST) Received: from nt (RJ231080.user.veloxzone.com.br [200.165.231.80]) (authenticated user ensr) by pacuiba.terra.com.br (Postfix) with ESMTP id A0BB78056 for ; Tue, 11 Jun 2002 18:49:15 -0300 (EST) Message-ID: <00d701c21191$8bf7d400$5400a8c0@ensrbr> From: "Luis Lopes" To: References: <20020611193314.39492.qmail@web14803.mail.yahoo.com> <20020611175547.A4872@sucuri.mat.puc-rio.br> Subject: Re: [obm-l] Duvidas de analitica. Date: Tue, 11 Jun 2002 18:47:12 -0300 MIME-Version: 1.0 Content-Type: text/plain; charset="iso-8859-1" Content-Transfer-Encoding: 8bit X-Priority: 3 X-MSMail-Priority: Normal X-Mailer: Microsoft Outlook Express 5.00.2615.200 X-MimeOLE: Produced By Microsoft MimeOLE V5.00.2615.200 Sender: owner-obm-l@sucuri.mat.puc-rio.br Precedence: bulk Reply-To: obm-l@mat.puc-rio.br Sauda,c~oes, > > b)y^2-6*x+9=0 > > Escreva como x = 1/6 y^2 + 3/2, uma parábola. E quais seriam o foco, diretriz e eixo? x - x_0 = p(y - y_0)^2. []'s Luis -----Mensagem Original----- De: Nicolau C. Saldanha Para: Enviada em: terça-feira, 11 de junho de 2002 17:55 Assunto: Re: [obm-l] Duvidas de analitica. > On Tue, Jun 11, 2002 at 04:33:14PM -0300, Ricardo Miranda wrote: > > No livro do Elon, no segundo capitulo (Coordenadas no plano), exercicio 4, > > temos: > > 4) Para cada uma das equacoes abaixo, descreva o conjunto dos pontos (x,y) > > cujas coordenadas satisfazem essa equacao: > > > > b)y^2-6*x+9=0 > > Escreva como x = 1/6 y^2 + 3/2, uma parábola. > > > c)x^2+y^2+1=0 > > Conjunto vazio. > > ========================================================================= Instruções para entrar na lista, sair da lista e usar a lista em http://www.mat.puc-rio.br/~nicolau/olimp/obm-l.html O administrador desta lista é ========================================================================= From owner-obm-l@sucuri.mat.puc-rio.br Tue Jun 11 23:05:16 2002 Return-Path: Received: (from majordom@localhost) by sucuri.mat.puc-rio.br (8.9.3/8.9.3) id XAA07931 for obm-l-list; Tue, 11 Jun 2002 23:04:57 -0300 Received: from sr1.terra.com.br (sr1.terra.com.br [200.176.3.16]) by sucuri.mat.puc-rio.br (8.9.3/8.9.3) with ESMTP id XAA07928 for ; Tue, 11 Jun 2002 23:04:55 -0300 Received: from smtp4-poa.terra.com.br (smtp4-poa.terra.com.br [200.176.3.35]) by sr1.terra.com.br (Postfix) with ESMTP id 645AA6EF0C for ; Tue, 11 Jun 2002 22:52:26 -0300 (EST) Received: from terra.com.br (una.terra.com.br [200.176.3.184]) by smtp4-poa.terra.com.br (Postfix) with ESMTP id 55DA1AC5BC for ; Tue, 11 Jun 2002 22:52:26 -0300 (EST) Date: Wed, 12 Jun 2002 01:52:26 +0000 Message-Id: Subject: [obm-l] =?iso-8859-1?q?Fatora=E7=E3o?= MIME-Version: 1.0 Content-Type: text/plain;charset="iso-8859-1" From: "hilhend" To: "obm-l" X-XaM3-API-Version: 2.4.3.2.9 X-SenderIP: 200.161.14.239 Content-Transfer-Encoding: 8bit X-MIME-Autoconverted: from quoted-printable to 8bit by sucuri.mat.puc-rio.br id XAA07929 Sender: owner-obm-l@sucuri.mat.puc-rio.br Precedence: bulk Reply-To: obm-l@mat.puc-rio.br Estou tendo probleminhas pra fatorar em reais x^6 + (x^3)(y^3) + y^6 Ouvi falar que a solução foi proposta por Gauss, mas estou sem nada e não faço idéia onde possa achar. Sou muito grato pela demonstração e/ou pela indicação de bibliografias. ========================================================================= Instruções para entrar na lista, sair da lista e usar a lista em http://www.mat.puc-rio.br/~nicolau/olimp/obm-l.html O administrador desta lista é ========================================================================= From owner-obm-l@sucuri.mat.puc-rio.br Wed Jun 12 00:43:19 2002 Return-Path: Received: (from majordom@localhost) by sucuri.mat.puc-rio.br (8.9.3/8.9.3) id AAA08797 for obm-l-list; Wed, 12 Jun 2002 00:42:53 -0300 Received: from hotmail.com (f83.law8.hotmail.com [216.33.241.83]) by sucuri.mat.puc-rio.br (8.9.3/8.9.3) with ESMTP id AAA08789 for ; Wed, 12 Jun 2002 00:42:35 -0300 Received: from mail pickup service by hotmail.com with Microsoft SMTPSVC; Tue, 11 Jun 2002 20:30:05 -0700 Received: from 200.202.96.10 by lw8fd.law8.hotmail.msn.com with HTTP; Wed, 12 Jun 2002 03:30:05 GMT X-Originating-IP: [200.202.96.10] From: "Antonio Neto" To: obm-l@mat.puc-rio.br Subject: Re: [obm-l] t. dos nºs Date: Wed, 12 Jun 2002 03:30:05 +0000 Mime-Version: 1.0 Content-Type: text/plain; format=flowed Message-ID: X-OriginalArrivalTime: 12 Jun 2002 03:30:05.0670 (UTC) FILETIME=[71E09860:01C211C1] Sender: owner-obm-l@sucuri.mat.puc-rio.br Precedence: bulk Reply-To: obm-l@mat.puc-rio.br Lindissimo livro!!! Abracos, olavo. >From: Salvador Addas Zanata >Reply-To: obm-l@mat.puc-rio.br >To: obm-l@mat.puc-rio.br >Subject: Re: [obm-l] t. dos nºs >Date: Tue, 11 Jun 2002 10:50:25 -0300 (EST) > > > >O primeiro problema so pode ter solucao se p=4n+1. > >Para ver isso, observe que a deve ser par e b impar. Logo a^2+b^2 e da >forma: 4c^2+4d^2+4c+1, que e da forma 4n+1. > >De fato todo primo da forma 4n+1 se escreve de um unico jeito como a soma >de 2 quadrados. Tem um livro chamado "100 great elementary problems: Their >history and solutions" Heinrich Dorrie, que tem essa prova e muitas outras >bacanas. Alias esse livro apresenta as "melhores" provas de cada >problema. E da Dover e nao e dificil de achar. > > >Abraco, > >Salvador > > >On Tue, 11 Jun 2002, Adherbal Rocha Filho wrote: > > > > > ajuda: > > > > Mostrar q se o primo p é tal q p==3(mod4), então a equação p^2= a^2 +b^2 > > possui solução inteira > > > > mostre q todo quadrado perfeito pode ser representado como soma dos > > quadrados de racionais ,naum inteiros, r e s. > > > > valeu! > > > > _________________________________________________________________ > > Chegou o novo MSN Explorer. Instale já. É gratuito: > > http://explorer.msn.com.br > > > > >========================================================================= > > Instruções para entrar na lista, sair da lista e usar a lista em > > http://www.mat.puc-rio.br/~nicolau/olimp/obm-l.html > > O administrador desta lista é > > >========================================================================= > > > >========================================================================= >Instruções para entrar na lista, sair da lista e usar a lista em >http://www.mat.puc-rio.br/~nicolau/olimp/obm-l.html >O administrador desta lista é >========================================================================= _________________________________________________________________ Chat with friends online, try MSN Messenger: http://messenger.msn.com ========================================================================= Instruções para entrar na lista, sair da lista e usar a lista em http://www.mat.puc-rio.br/~nicolau/olimp/obm-l.html O administrador desta lista é ========================================================================= From owner-obm-l@sucuri.mat.puc-rio.br Wed Jun 12 08:55:00 2002 Return-Path: Received: (from majordom@localhost) by sucuri.mat.puc-rio.br (8.9.3/8.9.3) id IAA10755 for obm-l-list; Wed, 12 Jun 2002 08:54:41 -0300 Received: from smtp-31.ig.com.br (smtp-31.ig.com.br [200.226.132.181]) by sucuri.mat.puc-rio.br (8.9.3/8.9.3) with SMTP id IAA10752 for ; Wed, 12 Jun 2002 08:54:37 -0300 Message-Id: <200206121154.IAA10752@sucuri.mat.puc-rio.br> Received: (qmail 13278 invoked from network); 12 Jun 2002 11:42:05 -0000 Received: from unknown (HELO localhost) (200.226.132.30) by smtp-31.ig.com.br with SMTP; 12 Jun 2002 11:42:05 -0000 Date: Wed, 12 Jun 2002 08:42:07 -0300 To: obm-l@mat.puc-rio.br From: Michele Calefe Cc: X-Originating-IP: [200.226.133.51]143.106.47.148 X-Mailer: InMail by Insite - www.insite.com.br X-user: m.calefe@ig.com.br MIME-Version: 1.0 Content-type: text/plain Subject: [obm-l] Variáveis Complexas Sender: owner-obm-l@sucuri.mat.puc-rio.br Precedence: bulk Reply-To: obm-l@mat.puc-rio.br Olá, pessoal, tudo bem? Estou com dúvida num problema de 'Variáveis Complexas: Expanda a função f(z)=1/[(2z-1)(z-2)^2] numa série de Laurent em torno dos pontos z=2 e z=1/2. Em torno do ponto z=2 eu consegui, porém não consigo expandi-la em torno de z=1/2. Obrigada desde já, michele calefe "Let us eat and drink, for tomorrow we die" _________________________________________________________ Oi! Você quer um iG-mail gratuito? Então clique aqui: http://registro.ig.com.br/ ========================================================================= Instruções para entrar na lista, sair da lista e usar a lista em http://www.mat.puc-rio.br/~nicolau/olimp/obm-l.html O administrador desta lista é ========================================================================= From owner-obm-l@sucuri.mat.puc-rio.br Wed Jun 12 10:16:32 2002 Return-Path: Received: (from majordom@localhost) by sucuri.mat.puc-rio.br (8.9.3/8.9.3) id KAA11582 for obm-l-list; Wed, 12 Jun 2002 10:15:52 -0300 Received: from web14801.mail.yahoo.com (web14801.mail.yahoo.com [216.136.224.217]) by sucuri.mat.puc-rio.br (8.9.3/8.9.3) with SMTP id KAA11579 for ; Wed, 12 Jun 2002 10:15:49 -0300 Message-ID: <20020612130319.56530.qmail@web14801.mail.yahoo.com> Received: from [200.17.79.42] by web14801.mail.yahoo.com via HTTP; Wed, 12 Jun 2002 10:03:19 ART Date: Wed, 12 Jun 2002 10:03:19 -0300 (ART) From: =?iso-8859-1?q?Ricardo=20Miranda?= Subject: Re: [obm-l] Duvidas de analitica. To: obm-l@mat.puc-rio.br In-Reply-To: <00d701c21191$8bf7d400$5400a8c0@ensrbr> MIME-Version: 1.0 Content-Type: text/plain; charset=iso-8859-1 Content-Transfer-Encoding: 8bit Sender: owner-obm-l@sucuri.mat.puc-rio.br Precedence: bulk Reply-To: obm-l@mat.puc-rio.br > -----Mensagem Original----- > De: Nicolau C. Saldanha > Para: > Enviada em: terça-feira, 11 de junho de 2002 17:55 > Assunto: Re: [obm-l] Duvidas de analitica. > > > b)y^2-6*x+9=0 > > > > Escreva como x = 1/6 y^2 + 3/2, uma parábola. No Maple.. > plot(y^2-6*x+9, x=1..10, y=1..10); Plotting error, empty plot Só consigo usando plot3d. Nao era pra dar certo do modo normal (usando somente plot)? ===== []s Ricardo Miranda Matematica - UFV ricardomirandabr@yahoo.com.br http://rm2.hpg.ig.com.br/ _______________________________________________________________________ Copa 2002 Yahoo! - Patrocinador oficial da Copa do Mundo da FIFA 2002 http://br.sports.yahoo.com/fifaworldcup/ ========================================================================= Instruções para entrar na lista, sair da lista e usar a lista em http://www.mat.puc-rio.br/~nicolau/olimp/obm-l.html O administrador desta lista é ========================================================================= From owner-obm-l@sucuri.mat.puc-rio.br Wed Jun 12 21:04:13 2002 Return-Path: Received: (from majordom@localhost) by sucuri.mat.puc-rio.br (8.9.3/8.9.3) id VAA19972 for obm-l-list; Wed, 12 Jun 2002 21:03:38 -0300 Received: from gorgo.centroin.com.br (gorgo.centroin.com.br [200.225.63.128]) by sucuri.mat.puc-rio.br (8.9.3/8.9.3) with ESMTP id VAA19968 for ; Wed, 12 Jun 2002 21:03:37 -0300 Received: from centroin.com.br (du124c.rjo.centroin.com.br [200.225.58.124]) (authenticated bits=0) by gorgo.centroin.com.br (8.12.2/8.12.1) with ESMTP id g5CNpdko021425 for ; Wed, 12 Jun 2002 20:51:41 -0300 (BRT) Message-ID: <3D07DED9.2080602@centroin.com.br> Date: Wed, 12 Jun 2002 20:52:57 -0300 From: Augusto =?ISO-8859-1?Q?C=E9sar?= Morgado User-Agent: Mozilla/5.0 (Windows; U; Win98; en-US; rv:0.9.4.1) Gecko/20020508 Netscape6/6.2.3 X-Accept-Language: en-us MIME-Version: 1.0 To: obm-l@mat.puc-rio.br Subject: Re: [obm-l] =?ISO-8859-1?Q?Vari=E1veis?= Complexas References: <200206121154.IAA10752@sucuri.mat.puc-rio.br> Content-Type: text/plain; charset=ISO-8859-1; format=flowed Content-Transfer-Encoding: 8bit Sender: owner-obm-l@sucuri.mat.puc-rio.br Precedence: bulk Reply-To: obm-l@mat.puc-rio.br Faça z-(1/2)=t e decomponha em fraçoes parciais. Obtem-se uma soma de tres parcelas: A primeira eh 2/9 * 1/t A segunda eh 4/27 * [1-(2t/3)] A terceira eh 4/27 * [1-(2t/3)]^2 (Espero nao ter errado as contas) Lembre-se que 1/(1-x) = 1 + x + x^2 + x^3 +.... para Modulo de x <1 e que 1/[(1-x)^2] = 1 + 2x + 3x^2 + 4x^3 =... para Modulo de x <1. Michele Calefe wrote: >Olá, pessoal, tudo bem? > >Estou com dúvida num problema de 'Variáveis Complexas: > >Expanda a função f(z)=1/[(2z-1)(z-2)^2] numa série de Laurent em torno dos >pontos z=2 e z=1/2. > >Em torno do ponto z=2 eu consegui, porém não consigo expandi-la em torno de >z=1/2. > >Obrigada desde já, > >michele calefe > >"Let us eat and drink, for tomorrow we die" > >_________________________________________________________ >Oi! Você quer um iG-mail gratuito? >Então clique aqui: http://registro.ig.com.br/ > >========================================================================= >Instruções para entrar na lista, sair da lista e usar a lista em >http://www.mat.puc-rio.br/~nicolau/olimp/obm-l.html >O administrador desta lista é >========================================================================= > > ========================================================================= Instruções para entrar na lista, sair da lista e usar a lista em http://www.mat.puc-rio.br/~nicolau/olimp/obm-l.html O administrador desta lista é ========================================================================= From owner-obm-l@sucuri.mat.puc-rio.br Thu Jun 13 04:00:40 2002 Return-Path: Received: (from majordom@localhost) by sucuri.mat.puc-rio.br (8.9.3/8.9.3) id EAA23087 for obm-l-list; Thu, 13 Jun 2002 04:00:15 -0300 Received: from web10106.mail.yahoo.com (web10106.mail.yahoo.com [216.136.130.56]) by sucuri.mat.puc-rio.br (8.9.3/8.9.3) with SMTP id EAA23083 for ; Thu, 13 Jun 2002 04:00:12 -0300 Message-ID: <20020613064745.90169.qmail@web10106.mail.yahoo.com> Received: from [200.151.202.98] by web10106.mail.yahoo.com via HTTP; Wed, 12 Jun 2002 23:47:45 PDT Date: Wed, 12 Jun 2002 23:47:45 -0700 (PDT) From: Rafael WC Subject: [obm-l] abc + p = 0 To: OBM MIME-Version: 1.0 Content-Type: text/plain; charset=us-ascii Sender: owner-obm-l@sucuri.mat.puc-rio.br Precedence: bulk Reply-To: obm-l@mat.puc-rio.br Oi Pessoal! Essa aqui já me incomoda há muito tempo e não consigo resolver: "Sejam a, b, c, p quatro números reais dados tais que a, b e c não sejam simultaneamente iguais e: a + 1/b = b + 1/c = c + 1/a = p Qual o valor de (abc + p)?" Até a resposta eu tenho, mas mesmo assim não sai. A resposta é zero. Se alguém conseguir, agradeço. Abraços, Rafael. ===== Rafael Werneck Cinoto ICQ# 107011599 rwcinoto@yahoo.com rafael.caixa@gov.com.br matduvidas@yahoo.com.br http://www.rwcinoto.hpg.com.br/ __________________________________________________ Do You Yahoo!? Yahoo! - Official partner of 2002 FIFA World Cup http://fifaworldcup.yahoo.com ========================================================================= Instruções para entrar na lista, sair da lista e usar a lista em http://www.mat.puc-rio.br/~nicolau/olimp/obm-l.html O administrador desta lista é ========================================================================= From owner-obm-l@sucuri.mat.puc-rio.br Thu Jun 13 14:46:51 2002 Return-Path: Received: (from majordom@localhost) by sucuri.mat.puc-rio.br (8.9.3/8.9.3) id OAA29897 for obm-l-list; Thu, 13 Jun 2002 14:43:49 -0300 Received: from candeias.terra.com.br (candeias.terra.com.br [200.176.3.18]) by sucuri.mat.puc-rio.br (8.9.3/8.9.3) with ESMTP id OAA29893 for ; Thu, 13 Jun 2002 14:43:46 -0300 Received: from engenho.terra.com.br (engenho.terra.com.br [200.176.3.42]) by candeias.terra.com.br (Postfix) with ESMTP id BB20043D9B for ; Thu, 13 Jun 2002 14:31:20 -0300 (EST) Received: from xt (200-171-248-158.customer.telesp.net.br [200.171.248.158]) (authenticated user macwad) by engenho.terra.com.br (Postfix) with ESMTP id B420A680C1 for ; Thu, 13 Jun 2002 14:31:19 -0300 (EST) Message-ID: <002e01c21300$322f65d0$0401010a@xt> From: =?iso-8859-1?Q?Vinicius_Jos=E9_Fortuna?= To: References: <20020613064745.90169.qmail@web10106.mail.yahoo.com> Subject: [obm-l] =?iso-8859-1?Q?Iberoamericana_Universit=E1ria?= Date: Thu, 13 Jun 2002 14:31:46 -0300 MIME-Version: 1.0 Content-Type: text/plain; charset="iso-8859-1" Content-Transfer-Encoding: 8bit X-Priority: 3 X-MSMail-Priority: Normal X-Mailer: Microsoft Outlook Express 6.00.2600.0000 X-MimeOLE: Produced By Microsoft MimeOLE V6.00.2600.0000 Sender: owner-obm-l@sucuri.mat.puc-rio.br Precedence: bulk Reply-To: obm-l@mat.puc-rio.br Pessoal, Como é que se faz para participar da Olimpíada Iberoamericana de Matemática? Quando vai ser? Mandei um e-mail lá para a OBM perguntando, mas não me responderam. :-( Obrigado Vinicius Fortuna ========================================================================= Instruções para entrar na lista, sair da lista e usar a lista em http://www.mat.puc-rio.br/~nicolau/olimp/obm-l.html O administrador desta lista é ========================================================================= From owner-obm-l@sucuri.mat.puc-rio.br Thu Jun 13 14:56:22 2002 Return-Path: Received: (from majordom@localhost) by sucuri.mat.puc-rio.br (8.9.3/8.9.3) id OAA30036 for obm-l-list; Thu, 13 Jun 2002 14:55:50 -0300 Received: from smtp-5.ig.com.br (smtp-5.ig.com.br [200.226.132.154]) by sucuri.mat.puc-rio.br (8.9.3/8.9.3) with SMTP id OAA30032 for ; Thu, 13 Jun 2002 14:55:43 -0300 Received: (qmail 15346 invoked from network); 13 Jun 2002 17:43:02 -0000 Received: from shasta002202.ig.com.br (HELO windows9) (200.151.2.202) by smtp-5.ig.com.br with SMTP; 13 Jun 2002 17:43:02 -0000 Message-ID: <001b01c21301$ffb45aa0$ca0297c8@windows9> From: "Eric Campos Bastos Guedes" To: References: <20020613064745.90169.qmail@web10106.mail.yahoo.com> Subject: Re: [obm-l] abc + p = 0 Date: Thu, 13 Jun 2002 14:41:25 -0300 MIME-Version: 1.0 Content-Type: text/plain; charset="iso-8859-1" Content-Transfer-Encoding: 8bit X-Priority: 3 X-MSMail-Priority: Normal X-Mailer: Microsoft Outlook Express 5.00.2615.200 X-MIMEOLE: Produced By Microsoft MimeOLE V5.00.2615.200 Sender: owner-obm-l@sucuri.mat.puc-rio.br Precedence: bulk Reply-To: obm-l@mat.puc-rio.br > Oi Pessoal! > > "Sejam a, b, c, p quatro numeros reais dados tais que > a, b e c não sejam simultaneamente iguais e: > a + 1/b = b + 1/c = c + 1/a = p > > Qual o valor de (abc + p)?" > a + 1/b = b + 1/c acarreta (a - b) = 1/c - 1/b = (b - c)/bc logo [1] (a - b) = (b - c)/bc analogamente [2] (b - c) = (c - a)/ac [3] (c - a) = (a - b)/ab note que se a = b ou b = c ou c = a então a = b = c. Logo a, b, c sao dois a dois distintos. Multiplicando [1], [2] e [3] tem-se (a-b)(b-c)(c-a) = (b-c)(c-a)(a-b)/(abc^2) donde, cancelando 1 = 1/(abc^2) e abc = 1 ou abc = -1 (i) Suponha que abc = -1. Entao p = a + 1/b = a - (abc)/b = a - ac, entao p = a - ac e analogamente p = b - ba p = c - cb logo p + abc = (a-ac) + abc = a(1-c+bc) = a(1 - (c-bc)) = = a(1 - p), isto eh p+abc = a(1-p), entao p + abc = a(1-p) = b(1-p) = c(1-p) como a,b,c sao distintos, p = 1 logo p + abc = 1 - 1 = 0 (ii) Suponha que abc=1 p = a + 1/b = a + abc/b = a + ac, logo p = a + ac e analogamente p = b + ab p = c + bc entao p + abc = (a+ac) + abc = a(1+c+bc) = a(1 + p), isto eh p+abc = a(1 + p), analogamente p+abc = b(1 + p) p+abc = c(1 + p) logo a(1+p)=b(1+p)=c(1+p) como a,b,c sao distintos, entao p = -1 (e abc =1) donde p + abc = -1 + 1 = 0 Abrac,os, Eric. ========================================================================= Instruções para entrar na lista, sair da lista e usar a lista em http://www.mat.puc-rio.br/~nicolau/olimp/obm-l.html O administrador desta lista é ========================================================================= From owner-obm-l@sucuri.mat.puc-rio.br Thu Jun 13 15:34:59 2002 Return-Path: Received: (from majordom@localhost) by sucuri.mat.puc-rio.br (8.9.3/8.9.3) id PAA31118 for obm-l-list; Thu, 13 Jun 2002 15:34:00 -0300 Received: from salvatore2.bol.com.br (salvatore2.bol.com.br [200.221.24.87]) by sucuri.mat.puc-rio.br (8.9.3/8.9.3) with ESMTP id PAA31114 for ; Thu, 13 Jun 2002 15:33:44 -0300 Received: from bol.com.br (200.221.24.72) by salvatore2.bol.com.br (5.1.071) id 3CFE36F1001B1CF8 for obm-l@mat.puc-rio.br; Thu, 13 Jun 2002 15:20:40 -0300 Date: Thu, 13 Jun 2002 15:20:40 -0300 Message-Id: Subject: [obm-l] Outra pegunta.. MIME-Version: 1.0 Content-Type: text/plain;charset="iso-8859-1" From: "leon-17" To: obm-l@mat.puc-rio.br X-XaM3-API-Version: 2.4.3.4.4 X-SenderIP: 200.254.248.142 Content-Transfer-Encoding: 8bit X-MIME-Autoconverted: from quoted-printable to 8bit by sucuri.mat.puc-rio.br id PAA31115 Sender: owner-obm-l@sucuri.mat.puc-rio.br Precedence: bulk Reply-To: obm-l@mat.puc-rio.br Obrigado Douglas e os demais que me responderam a mesma questão de tão variadas formas. Aí vai outra pergunta: Seja f uma função de Z em Z definida como f(x)=x/10 se x é divisível por 10 e f(x)=x+1 caso contrário. Se a0=2001 e an+1=f(an), qual é o menor valor de n para o qual an=1? __________________________________________________________________________ Quer ter seu próprio endereço na Internet? Garanta já o seu e ainda ganhe cinco e-mails personalizados. DomíniosBOL - http://dominios.bol.com.br ========================================================================= Instruções para entrar na lista, sair da lista e usar a lista em http://www.mat.puc-rio.br/~nicolau/olimp/obm-l.html O administrador desta lista é ========================================================================= From owner-obm-l@sucuri.mat.puc-rio.br Thu Jun 13 16:09:32 2002 Return-Path: Received: (from majordom@localhost) by sucuri.mat.puc-rio.br (8.9.3/8.9.3) id QAA32229 for obm-l-list; Thu, 13 Jun 2002 16:08:39 -0300 Received: (from nicolau@localhost) by sucuri.mat.puc-rio.br (8.9.3/8.9.3) id QAA32224 for obm-l@mat.puc-rio.br; Thu, 13 Jun 2002 16:08:39 -0300 Date: Thu, 13 Jun 2002 16:08:39 -0300 From: "Nicolau C. Saldanha" To: obm-l@mat.puc-rio.br Subject: [obm-l] Re: =?iso-8859-1?Q?=5Bobm-l=5D_Iberoamericana_Universit=E1ria?= Message-ID: <20020613160839.A32093@sucuri.mat.puc-rio.br> References: <20020613064745.90169.qmail@web10106.mail.yahoo.com> <002e01c21300$322f65d0$0401010a@xt> Mime-Version: 1.0 Content-Type: text/plain; charset=iso-8859-1 Content-Disposition: inline Content-Transfer-Encoding: 8bit User-Agent: Mutt/1.2.5i In-Reply-To: <002e01c21300$322f65d0$0401010a@xt>; from vinicius.fortuna@ic.unicamp.br on Thu, Jun 13, 2002 at 02:31:46PM -0300 Sender: owner-obm-l@sucuri.mat.puc-rio.br Precedence: bulk Reply-To: obm-l@mat.puc-rio.br On Thu, Jun 13, 2002 at 02:31:46PM -0300, Vinicius José Fortuna wrote: > Pessoal, > > Como é que se faz para participar da Olimpíada Iberoamericana de Matemática? Pelo subject, deduzo que você está falando da Iberoamericana *Universitária*. Esta ocorre mais para o fim do ano, depois da 1a fase da OBM nível univ e só é recomendável para quem tem uma certa experiência com olimpíadas de matemática, ou seja, para quem fez pelo menos a 1a fase da OBM e teve um bom resultado. []s, N. ========================================================================= Instruções para entrar na lista, sair da lista e usar a lista em http://www.mat.puc-rio.br/~nicolau/olimp/obm-l.html O administrador desta lista é ========================================================================= From owner-obm-l@sucuri.mat.puc-rio.br Thu Jun 13 16:15:58 2002 Return-Path: Received: (from majordom@localhost) by sucuri.mat.puc-rio.br (8.9.3/8.9.3) id QAA32403 for obm-l-list; Thu, 13 Jun 2002 16:15:09 -0300 Received: from salvatore4.bol.com.br ([200.221.24.52]) by sucuri.mat.puc-rio.br (8.9.3/8.9.3) with ESMTP id QAA32399 for ; Thu, 13 Jun 2002 16:15:06 -0300 Received: from bol.com.br (200.221.24.69) by salvatore4.bol.com.br (5.1.071) id 3C436908016142BB for obm-l@mat.puc-rio.br; Thu, 13 Jun 2002 16:01:43 -0300 Date: Thu, 13 Jun 2002 16:01:43 -0300 Message-Id: Subject: Re:[obm-l] Outra pegunta.. MIME-Version: 1.0 Content-Type: text/plain;charset="iso-8859-1" From: "Thiago Sobral" To: obm-l@mat.puc-rio.br X-XaM3-API-Version: 2.4.3.4.4 X-SenderIP: 200.194.102.154 Content-Transfer-Encoding: 8bit X-MIME-Autoconverted: from quoted-printable to 8bit by sucuri.mat.puc-rio.br id QAA32400 Sender: owner-obm-l@sucuri.mat.puc-rio.br Precedence: bulk Reply-To: obm-l@mat.puc-rio.br f(x)=x/10 se x eh mult de 10, f(x)=x+1, caso contrario. vejamos: observando a funcao, veja q a9=2010 (vai soh somando 1...) e a10=201 (pois a9 eh mult. de 10) daih, do mesmo modo, a19=210 => a20=21 => a29=30 => a30=3 => a37=10 => a38=1. Assim o menor n/ an=1 eh 38. []s, Thiago Sobral icq:115100259 > Obrigado Douglas e os demais que me responderam a mesma > questão de tão variadas formas. > Aí vai outra pergunta: > Seja f uma função de Z em Z definida como f(x)=x/10 se x > é divisível por 10 e f(x)=x+1 caso contrário. Se a0=2001 > e an+1=f(an), qual é o menor valor de n para o qual an=1? > __________________________________________________________________________ Quer ter seu próprio endereço na Internet? Garanta já o seu e ainda ganhe cinco e-mails personalizados. DomíniosBOL - http://dominios.bol.com.br ========================================================================= Instruções para entrar na lista, sair da lista e usar a lista em http://www.mat.puc-rio.br/~nicolau/olimp/obm-l.html O administrador desta lista é ========================================================================= From owner-obm-l@sucuri.mat.puc-rio.br Thu Jun 13 17:01:46 2002 Return-Path: Received: (from majordom@localhost) by sucuri.mat.puc-rio.br (8.9.3/8.9.3) id RAA01517 for obm-l-list; Thu, 13 Jun 2002 17:00:49 -0300 Received: from regusnet.centri.net (regusnet.centri.net [194.73.67.42]) by sucuri.mat.puc-rio.br (8.9.3/8.9.3) with ESMTP id RAA01511 for ; Thu, 13 Jun 2002 17:00:46 -0300 Received: from jjunior (unknown [200.198.83.126]) by regusnet.centri.net (Postfix) with ESMTP id 1309227148 for ; Thu, 13 Jun 2002 21:48:51 +0100 (BST) From: "Jose Jayme Moraes Junior" To: Subject: RE: [obm-l] Outra pegunta.. Date: Thu, 13 Jun 2002 16:53:43 -0300 Organization: M13 Message-ID: <003501c21314$0664faf0$0479a8c0@jjunior> MIME-Version: 1.0 Content-Type: text/plain; charset="iso-8859-1" X-Priority: 3 (Normal) X-MSMail-Priority: Normal X-Mailer: Microsoft Outlook, Build 10.0.2616 Importance: Normal X-MimeOLE: Produced By Microsoft MimeOLE V6.00.2600.0000 In-Reply-To: Content-Transfer-Encoding: 8bit X-MIME-Autoconverted: from quoted-printable to 8bit by sucuri.mat.puc-rio.br id RAA01512 Sender: owner-obm-l@sucuri.mat.puc-rio.br Precedence: bulk Reply-To: obm-l@mat.puc-rio.br Seja f uma função de Z em Z definida como f(x)=x/10 se x é divisível por 10 e f(x)=x+1 caso contrário. Se a0=2001 e an+1=f(an), qual é o menor valor de n para o qual an=1? a1 = f(a0) = 2002 a2 = f(a1) = 2003 a3 = f(a2) = 2004 . . . a9 = f(a8) = 2010 a10 = f(a9) = 201 a11 = f(a10) = 202 . . . a20 = f(a19) = 21 a30 = f (a29) = 3 a37 = 3 + 7 = 10 a38 = 1 Resposta: a38 -----Original Message----- From: owner-obm-l@sucuri.mat.puc-rio.br [mailto:owner-obm-l@sucuri.mat.puc-rio.br] On Behalf Of leon-17 Sent: quinta-feira, 13 de junho de 2002 15:21 To: obm-l@mat.puc-rio.br Subject: [obm-l] Outra pegunta.. Obrigado Douglas e os demais que me responderam a mesma questão de tão variadas formas. Aí vai outra pergunta: Seja f uma função de Z em Z definida como f(x)=x/10 se x é divisível por 10 e f(x)=x+1 caso contrário. Se a0=2001 e an+1=f(an), qual é o menor valor de n para o qual an=1? ________________________________________________________________________ __ Quer ter seu próprio endereço na Internet? Garanta já o seu e ainda ganhe cinco e-mails personalizados. DomíniosBOL - http://dominios.bol.com.br ======================================================================== = Instruções para entrar na lista, sair da lista e usar a lista em http://www.mat.puc-rio.br/~nicolau/olimp/obm-l.html O administrador desta lista é ======================================================================== = ========================================================================= Instruções para entrar na lista, sair da lista e usar a lista em http://www.mat.puc-rio.br/~nicolau/olimp/obm-l.html O administrador desta lista é ========================================================================= From owner-obm-l@sucuri.mat.puc-rio.br Thu Jun 13 17:02:09 2002 Return-Path: Received: (from majordom@localhost) by sucuri.mat.puc-rio.br (8.9.3/8.9.3) id RAA01573 for obm-l-list; Thu, 13 Jun 2002 17:02:05 -0300 Received: from traven10.uol.com.br (traven10.uol.com.br [200.231.206.211]) by sucuri.mat.puc-rio.br (8.9.3/8.9.3) with ESMTP id RAA01562 for ; Thu, 13 Jun 2002 17:02:01 -0300 Received: from Eder ([200.227.70.29]) by traven10.uol.com.br (8.9.1/8.9.1) with SMTP id QAA28128 for ; Thu, 13 Jun 2002 16:49:32 -0300 (BRT) Message-ID: <001a01c21314$4add83a0$6672fea9@Eder> From: "Eder" To: Subject: [obm-l] ???????? Date: Thu, 13 Jun 2002 16:53:33 -0300 MIME-Version: 1.0 Content-Type: multipart/alternative; boundary="----=_NextPart_000_0013_01C212FA.DAA89340" X-Priority: 3 X-MSMail-Priority: Normal X-Mailer: Microsoft Outlook Express 5.00.2314.1300 X-MimeOLE: Produced By Microsoft MimeOLE V5.00.2314.1300 Sender: owner-obm-l@sucuri.mat.puc-rio.br Precedence: bulk Reply-To: obm-l@mat.puc-rio.br This is a multi-part message in MIME format. ------=_NextPart_000_0013_01C212FA.DAA89340 Content-Type: text/plain; charset="iso-8859-1" Content-Transfer-Encoding: quoted-printable Ol=E1 colegas de lista, Eu gostaria de ajuda no seguinte problema: Prove que n=E3o existe n natural tal que (2^n + 1) seja divis=EDvel por = 7. Ah!Eu resolvi este outro utilizando congru=EAncias: Encontre todos os valores de n para os quais (2^n - 1)seja divis=EDvel = por 7. Ser=E1 que h=E1 outra maneira?=20 S=E3o parecidos...Mas eu ainda n=E3o "matei" o =FAltimo.Desde = j=E1,obrigado por qualquer coment=E1rio. Eder ------=_NextPart_000_0013_01C212FA.DAA89340 Content-Type: text/html; charset="iso-8859-1" Content-Transfer-Encoding: quoted-printable
Ol=E1 colegas de lista,
 
Eu gostaria de ajuda no seguinte=20 problema:
 
 
Prove que n=E3o existe n natural = tal que (2^n +=20 1) seja divis=EDvel por 7.
 
Ah!Eu resolvi este = outro utilizando=20 congru=EAncias:
 
Encontre todos os valores de n para = os quais=20 (2^n - 1)seja divis=EDvel por 7.
 
Ser=E1 que h=E1 outra = maneira? 
 
S=E3o parecidos...Mas eu ainda n=E3o = "matei" o=20 =FAltimo.Desde j=E1,obrigado por qualquer coment=E1rio.
 
 
Eder
------=_NextPart_000_0013_01C212FA.DAA89340-- ========================================================================= Instruções para entrar na lista, sair da lista e usar a lista em http://www.mat.puc-rio.br/~nicolau/olimp/obm-l.html O administrador desta lista é ========================================================================= From owner-obm-l@sucuri.mat.puc-rio.br Thu Jun 13 17:12:37 2002 Return-Path: Received: (from majordom@localhost) by sucuri.mat.puc-rio.br (8.9.3/8.9.3) id RAA02243 for obm-l-list; Thu, 13 Jun 2002 17:11:57 -0300 Received: from hotmail.com (law2-f127.hotmail.com [216.32.181.127]) by sucuri.mat.puc-rio.br (8.9.3/8.9.3) with ESMTP id RAA02237 for ; Thu, 13 Jun 2002 17:11:54 -0300 Received: from mail pickup service by hotmail.com with Microsoft SMTPSVC; Thu, 13 Jun 2002 13:11:18 -0700 Received: from 32.94.119.253 by lw2fd.hotmail.msn.com with HTTP; Thu, 13 Jun 2002 20:11:17 GMT X-Originating-IP: [32.94.119.253] From: "Paulo Santa Rita" To: obm-l@mat.puc-rio.br Subject: [obm-l] Geometry Problem Date: Thu, 13 Jun 2002 20:11:17 +0000 Mime-Version: 1.0 Content-Type: text/plain; charset=iso-8859-1; format=flowed Message-ID: X-OriginalArrivalTime: 13 Jun 2002 20:11:18.0206 (UTC) FILETIME=[7A4FE5E0:01C21316] Sender: owner-obm-l@sucuri.mat.puc-rio.br Precedence: bulk Reply-To: obm-l@mat.puc-rio.br >Hi. Firstly, this is not homework, I actually want this for a bit of >software I am writing. >Secondly, while I know it would be better if I worked it out myself, I'd >rather get the solution than not, even at the price of missing the >opportunity to brush up my geometry skills. > >I am attempting to lay out points on the Poincare projection of the >hyperbolic plane. I have done up a diagram at >http://www.users.bigpond.com/pmurray/Math1.gif > >I have a unit circle C, with a centre at O. >I have a radius of that circle r. >I have another circle D, that intersects C at right angles at a point P and >that intersects the radius R at point Q. > >Angle POr we shall call theta. >Angle Dr we shall call phi. (that is, the angle between r and the tangent >to >D at Q). >Distance OQ we shall call d. > >1 - given phi and d, what is theta? >2 - given phi and theta, what is d? > >That's all I think I need, although if you feel like working out phi from >theta and d, feel free. > >Please reply to me at pmurray@bigpond.com. > >.. _________________________________________________________________ Chegou o novo MSN Explorer. Instale já. É gratuito: http://explorer.msn.com.br ========================================================================= Instruções para entrar na lista, sair da lista e usar a lista em http://www.mat.puc-rio.br/~nicolau/olimp/obm-l.html O administrador desta lista é ========================================================================= From owner-obm-l@sucuri.mat.puc-rio.br Thu Jun 13 18:43:59 2002 Return-Path: Received: (from majordom@localhost) by sucuri.mat.puc-rio.br (8.9.3/8.9.3) id SAA07157 for obm-l-list; Thu, 13 Jun 2002 18:43:20 -0300 Received: from hotmail.com (law2-f84.hotmail.com [216.32.181.84]) by sucuri.mat.puc-rio.br (8.9.3/8.9.3) with ESMTP id SAA07153 for ; Thu, 13 Jun 2002 18:43:18 -0300 Received: from mail pickup service by hotmail.com with Microsoft SMTPSVC; Thu, 13 Jun 2002 14:42:46 -0700 Received: from 32.94.119.254 by lw2fd.hotmail.msn.com with HTTP; Thu, 13 Jun 2002 21:42:46 GMT X-Originating-IP: [32.94.119.254] From: "Paulo Santa Rita" To: obm-l@mat.puc-rio.br Subject: Re: [obm-l] Geometry Problem Date: Thu, 13 Jun 2002 21:42:46 +0000 Mime-Version: 1.0 Content-Type: text/plain; charset=iso-8859-1; format=flowed Message-ID: X-OriginalArrivalTime: 13 Jun 2002 21:42:46.0494 (UTC) FILETIME=[41962FE0:01C21323] Sender: owner-obm-l@sucuri.mat.puc-rio.br Precedence: bulk Reply-To: obm-l@mat.puc-rio.br Ola Pessoal, Saudações a Todos ! A Mensagem abaixo seguiu para a LISTA OBM-L sem nenhum comentario e em outro idioma por falha minha no momento de encaminha-la para outras pessoas. Peço desculpas a todos. Paulo Santa Rita 5,1840,130602 >From: "Paulo Santa Rita" >Reply-To: obm-l@mat.puc-rio.br >To: obm-l@mat.puc-rio.br >Subject: [obm-l] Geometry Problem >Date: Thu, 13 Jun 2002 20:11:17 +0000 > > > >Hi. Firstly, this is not homework, I actually want this for a bit of >software I am writing. >Secondly, while I know it would be better if I worked it out myself, I'd >rather get the solution than not, even at the price of missing the >opportunity to brush up my geometry skills. > >I am attempting to lay out points on the Poincare projection of the >hyperbolic plane. I have done up a diagram at >http://www.users.bigpond.com/pmurray/Math1.gif > >I have a unit circle C, with a centre at O. >I have a radius of that circle r. >I have another circle D, that intersects C at right angles at a point P and >that intersects the radius R at point Q. > >Angle POr we shall call theta. >Angle Dr we shall call phi. (that is, the angle between r and the tangent >to >D at Q). >Distance OQ we shall call d. > >1 - given phi and d, what is theta? >2 - given phi and theta, what is d? > >That's all I think I need, although if you feel like working out phi from >theta and d, feel free. > >Please reply to me at pmurray@bigpond.com. _________________________________________________________________ Envie e receba emails com o Hotmail no seu dispositivo móvel: http://mobile.msn.com ========================================================================= Instruções para entrar na lista, sair da lista e usar a lista em http://www.mat.puc-rio.br/~nicolau/olimp/obm-l.html O administrador desta lista é ========================================================================= From owner-obm-l@sucuri.mat.puc-rio.br Thu Jun 13 18:52:35 2002 Return-Path: Received: (from majordom@localhost) by sucuri.mat.puc-rio.br (8.9.3/8.9.3) id SAA07321 for obm-l-list; Thu, 13 Jun 2002 18:52:30 -0300 Received: from bidu.ime.usp.br (bidu.ime.usp.br [143.107.45.12]) by sucuri.mat.puc-rio.br (8.9.3/8.9.3) with SMTP id SAA07317 for ; Thu, 13 Jun 2002 18:52:28 -0300 Received: (qmail 23775 invoked from network); 13 Jun 2002 21:51:56 -0000 Received: from fradim.ime.usp.br (143.107.45.37) by bidu.ime.usp.br with SMTP; 13 Jun 2002 21:51:56 -0000 Received: (qmail 7703 invoked by uid 1604); 13 Jun 2002 21:49:21 -0000 Date: Thu, 13 Jun 2002 18:49:21 -0300 (EST) From: Salvador Addas Zanata X-Sender: sazanata@fradim To: obm-l@mat.puc-rio.br Subject: Re: [obm-l] ???????? In-Reply-To: <001a01c21314$4add83a0$6672fea9@Eder> Message-ID: MIME-Version: 1.0 Content-Type: TEXT/PLAIN; charset=ISO-8859-1 Content-Transfer-Encoding: 8bit X-MIME-Autoconverted: from QUOTED-PRINTABLE to 8bit by sucuri.mat.puc-rio.br id SAA07318 Sender: owner-obm-l@sucuri.mat.puc-rio.br Precedence: bulk Reply-To: obm-l@mat.puc-rio.br Oi, Se valesse o que voce escreveu, entao 2^n == 6 mod 7. Como 2^3-1=7, dividindo n por 3 temos n=3m+r. 2^3 == 1 mod 7 => 2^n == 2^r mod 7, que e <> 6 para r=0,1,2. Abraco, Salvador On Thu, 13 Jun 2002, Eder wrote: > Olá colegas de lista, > > Eu gostaria de ajuda no seguinte problema: > > > Prove que não existe n natural tal que (2^n + 1) seja divisível por 7. > > Ah!Eu resolvi este outro utilizando congruências: > > Encontre todos os valores de n para os quais (2^n - 1)seja divisível por 7. > > Será que há outra maneira? > > São parecidos...Mas eu ainda não "matei" o último.Desde já,obrigado por qualquer comentário. > > > Eder > ========================================================================= Instruções para entrar na lista, sair da lista e usar a lista em http://www.mat.puc-rio.br/~nicolau/olimp/obm-l.html O administrador desta lista é ========================================================================= From owner-obm-l@sucuri.mat.puc-rio.br Thu Jun 13 18:59:21 2002 Return-Path: Received: (from majordom@localhost) by sucuri.mat.puc-rio.br (8.9.3/8.9.3) id SAA07624 for obm-l-list; Thu, 13 Jun 2002 18:59:15 -0300 Received: from bidu.ime.usp.br (bidu.ime.usp.br [143.107.45.12]) by sucuri.mat.puc-rio.br (8.9.3/8.9.3) with SMTP id SAA07620 for ; Thu, 13 Jun 2002 18:59:12 -0300 Received: (qmail 24574 invoked from network); 13 Jun 2002 21:58:41 -0000 Received: from fradim.ime.usp.br (143.107.45.37) by bidu.ime.usp.br with SMTP; 13 Jun 2002 21:58:41 -0000 Received: (qmail 8023 invoked by uid 216); 13 Jun 2002 21:56:05 -0000 Date: Thu, 13 Jun 2002 18:56:05 -0300 (EST) From: Angelo Barone Netto X-Sender: barone@fradim To: obm-l@mat.puc-rio.br Subject: Re: [obm-l] ???????? In-Reply-To: <001a01c21314$4add83a0$6672fea9@Eder> Message-ID: MIME-Version: 1.0 Content-Type: TEXT/PLAIN; charset=US-ASCII Sender: owner-obm-l@sucuri.mat.puc-rio.br Precedence: bulk Reply-To: obm-l@mat.puc-rio.br Caro Eder (2^{n+3}+1)-(2^n+1)=7*2^n, assim, os restos (por 7, de 2^n+1) repetem-se, com periodo 3. Basta entao calcular os 3 primeiros e ver que sao nao nulos: 2^0+1=2, 2^1+1=3, 2^2+1=5. Angelo Barone{\ --\ }Netto Universidade de Sao Paulo Departamento de Matematica Aplicada Instituto de Matematica e Estatistica Rua do Matao, 1010 Butanta - Cidade Universitaria Caixa Postal 66 281 phone +55-11-3091-6162/6224/6136 05311-970 - Sao Paulo - SP fax +55-11-3091-6131 Agencia Cidade de Sao Paulo . ========================================================================= Instruções para entrar na lista, sair da lista e usar a lista em http://www.mat.puc-rio.br/~nicolau/olimp/obm-l.html O administrador desta lista é ========================================================================= From owner-obm-l@sucuri.mat.puc-rio.br Thu Jun 13 19:52:00 2002 Return-Path: Received: (from majordom@localhost) by sucuri.mat.puc-rio.br (8.9.3/8.9.3) id TAA09214 for obm-l-list; Thu, 13 Jun 2002 19:51:58 -0300 Received: from cairu.terra.com.br (cairu.terra.com.br [200.176.3.19]) by sucuri.mat.puc-rio.br (8.9.3/8.9.3) with ESMTP id TAA09209 for ; Thu, 13 Jun 2002 19:51:56 -0300 Received: from mucuri.terra.com.br (mucuri.terra.com.br [200.176.3.39]) by cairu.terra.com.br (Postfix) with ESMTP id 3705D4702F for ; Thu, 13 Jun 2002 19:51:19 +0000 (GMT) Received: from nt (RJ231080.user.veloxzone.com.br [200.165.231.80]) (authenticated user ensr) by mucuri.terra.com.br (Postfix) with ESMTP id 2DC11BE8C0 for ; Thu, 13 Jun 2002 19:51:25 -0300 (EST) Message-ID: <01f401c2132c$901b6b40$5400a8c0@ensrbr> From: "Luis Lopes" To: References: <193.7362b7f.2a181e78@aol.com> Subject: Re: [obm-l] (nenhum assunto) Date: Thu, 13 Jun 2002 19:49:22 -0300 MIME-Version: 1.0 Content-Type: multipart/alternative; boundary="----=_NextPart_000_01F1_01C21313.6A2F8240" X-Priority: 3 X-MSMail-Priority: Normal X-Mailer: Microsoft Outlook Express 5.00.2615.200 X-MimeOLE: Produced By Microsoft MimeOLE V5.00.2615.200 Sender: owner-obm-l@sucuri.mat.puc-rio.br Precedence: bulk Reply-To: obm-l@mat.puc-rio.br This is a multi-part message in MIME format. ------=_NextPart_000_01F1_01C21313.6A2F8240 Content-Type: text/plain; charset="iso-8859-1" Content-Transfer-Encoding: quoted-printable Sauda,c~oes, Calcule S_n =3D \sum_{k=3D1}^n cos(k alpha) para n >=3D 1 e ache F(n+1) - F(1), onde F(k) =E9 uma antidiferen=E7a para cos(k alpha). Ent=E3o F(k) =3D {sen[k-1/2]alpha} / {2sen(alpha/2)} . Colocando alpha=3D2pi/(2n+1), obtemos S_n =3D -1/2. Para n=3D3, S_3 =3D cos(2pi/7) + cos(4pi/7) + cos(6pi/7) =3D -1/2. Conclua que=20 cos (pi/7) - cos (2.pi/7) + cos (3.pi/7) =3D 1/2 []'s Lu=EDs -----Mensagem Original-----=20 De: Marcelo Rufino de Oliveira=20 Para: obm-l@mat.puc-rio.br=20 Enviada em: segunda-feira, 20 de maio de 2002 08:03 Assunto: Re: [obm-l] (nenhum assunto) Considere o podlin=F4mio P(x) =3D x^7 - 1, que possui as 7 seguintes = ra=EDzes complexas: z(k) =3D cos (2.k.pi/7) + i.sen (2.k.pi/7), k =3D 0, 1, 2, 3, 4, 5, = 6 Como o coeficiente de x^6 em P(x) =E9 0 ent=E3o a soma das ra=EDzes de = P(x) =E9 0, implicando que: cos 0 + cos (2.pi/7) + cos (4.pi/7) + cos (6.pi/7) + cos (8.pi/7) + = cos (10.pi/7) + cos (12.pi/7) =3D 0 =20 Como 2.pi/7 + 12.pi/7 =3D 2.pi =3D> cos (12.pi/7) =3D cos = (2.pi/7) Como 4.pi/7 + 10.pi/7 =3D 2.pi =3D> cos (10.pi/7) =3D cos = (4.pi/7) =3D - cos (3.pi/7)=20 Como 6.pi/7 + 8.pi/7 =3D 2.pi =3D> cos (8.pi/7) =3D cos (6.pi/7) = =3D - cos (pi/7) Portanto: 1 + cos (2.pi/7) - cos (3.pi/7) - cos (pi/7) - cos (pi/7) - cos = (3.pi/7) + cos (2.pi/7) =3D 0 =3D> cos (pi/7) - cos (2.pi/7) + cos (3.pi/7) =3D 1/2 At=E9 mais, Marcelo Rufino de Oliveira ----- Original Message -----=20 From: DEOLIVEIRASOU@aol.com=20 To: obm-l@mat.puc-rio.br=20 Sent: Saturday, May 18, 2002 6:15 PM Subject: [obm-l] (nenhum assunto) (IMO-1963) PROVE QUE COS(PI/7)-COS(2PI/7)+COS(3PI/7)=3D1/2.COMECEI A = FAZER E FOI FICANDO GRANDE...CADA VEZ MAIOR...RISOS...ALGUEM CONSEGUE = ACHAR UM TRUQUIINHO AI?? VALEU! CROM=20 ------=_NextPart_000_01F1_01C21313.6A2F8240 Content-Type: text/html; charset="iso-8859-1" Content-Transfer-Encoding: quoted-printable
Sauda,c~oes,
 
Calcule S_n =3D \sum_{k=3D1}^n cos(k alpha) para n = >=3D=20 1
e ache F(n+1) - F(1), onde F(k) =E9 uma = antidiferen=E7a=20 para
cos(k alpha). Ent=E3o
 
F(k) =3D {sen[k-1/2]alpha} / {2sen(alpha/2)} .=20 Colocando
alpha=3D2pi/(2n+1), obtemos S_n =3D = -1/2.
 
Para n=3D3, S_3 =3D cos(2pi/7) + cos(4pi/7) + = cos(6pi/7) =3D=20 -1/2.
 
Conclua que
 
cos (pi/7) - cos (2.pi/7) + cos = (3.pi/7) =3D=20 1/2
 
[]'s
Lu=EDs
-----Mensagem Original-----
De: Marcelo Rufino de = Oliveira
Enviada em: segunda-feira, 20 = de maio de=20 2002 08:03
Assunto: Re: [obm-l] (nenhum=20 assunto)

Considere o podlin=F4mio P(x) = =3D x^7 - 1, =20 que possui as 7 seguintes ra=EDzes complexas:
z(k) =3D cos (2.k.pi/7) + i.sen=20 (2.k.pi/7),   k =3D 0, 1, 2, 3, 4, 5, 6
 
Como o coeficiente de x^6 em P(x) = =E9 0 ent=E3o=20 a soma das ra=EDzes de P(x) =E9 0, implicando que:
 
cos 0 + cos (2.pi/7) + cos (4.pi/7) + = cos=20 (6.pi/7) + cos (8.pi/7) + cos (10.pi/7) + cos (12.pi/7) =3D = 0  =20
 
Como  2.pi/7 + 12.pi/7 =3D = 2.pi  =20 =3D>   cos (12.pi/7) =3D cos (2.pi/7)
Como  4.pi/7 + 10.pi/7 =3D = 2.pi  =20 =3D>   cos (10.pi/7) =3D cos (4.pi/7) =3D - cos = (3.pi/7)=20
Como  6.pi/7 + 8.pi/7 =3D = 2.pi  =20 =3D>   cos (8.pi/7) =3D cos (6.pi/7) =3D - cos=20 (pi/7)
Portanto:
1 + cos (2.pi/7) - cos (3.pi/7) = - cos (pi/7)=20 - cos (pi/7) - cos (3.pi/7) + cos (2.pi/7) =3D 0   = =3D>
cos (pi/7) - cos (2.pi/7) + cos = (3.pi/7) =3D=20 1/2
 
 
At=E9 mais,
Marcelo Rufino de = Oliveira
 
----- Original Message -----
From:=20 DEOLIVEIRASOU@aol.com =
Sent: Saturday, May 18, 2002 = 6:15=20 PM
Subject: [obm-l] (nenhum = assunto)

(IMO-1963) PROVE QUE=20 COS(PI/7)-COS(2PI/7)+COS(3PI/7)=3D1/2.COMECEI A FAZER E FOI FICANDO=20 GRANDE...CADA VEZ MAIOR...RISOS...ALGUEM CONSEGUE ACHAR UM = TRUQUIINHO=20 = AI??
           = ;            =              = = VALEU!
          &nb= sp;           &nbs= p;            = ;          =20 CROM
------=_NextPart_000_01F1_01C21313.6A2F8240-- ========================================================================= Instruções para entrar na lista, sair da lista e usar a lista em http://www.mat.puc-rio.br/~nicolau/olimp/obm-l.html O administrador desta lista é ========================================================================= From owner-obm-l@sucuri.mat.puc-rio.br Thu Jun 13 21:13:54 2002 Return-Path: Received: (from majordom@localhost) by sucuri.mat.puc-rio.br (8.9.3/8.9.3) id VAA10662 for obm-l-list; Thu, 13 Jun 2002 21:13:19 -0300 Received: from smtp-4.ig.com.br (smtp-4.ig.com.br [200.226.132.153]) by sucuri.mat.puc-rio.br (8.9.3/8.9.3) with SMTP id VAA10658 for ; Thu, 13 Jun 2002 21:13:17 -0300 Received: (qmail 16082 invoked from network); 14 Jun 2002 00:12:32 -0000 Received: from shasta004200.ig.com.br (HELO windows9) (200.151.4.200) by smtp-4.ig.com.br with SMTP; 14 Jun 2002 00:12:32 -0000 Message-ID: <001501c21338$6b857580$c80497c8@windows9> From: "Eric Campos Bastos Guedes" To: References: <20020613064745.90169.qmail@web10106.mail.yahoo.com><002e01c21300$322f65d0$0401010a@xt> <20020613160839.A32093@sucuri.mat.puc-rio.br> Subject: [obm-l] =?iso-8859-1?Q?Re:_=5Bobm-l=5D_Re:_=5Bobm-l=5D_Iberoamericana_Universit?= =?iso-8859-1?Q?=E1ria?= Date: Thu, 13 Jun 2002 21:14:01 -0300 MIME-Version: 1.0 Content-Type: text/plain; charset="iso-8859-1" Content-Transfer-Encoding: 8bit X-Priority: 3 X-MSMail-Priority: Normal X-Mailer: Microsoft Outlook Express 5.00.2615.200 X-MimeOLE: Produced By Microsoft MimeOLE V5.00.2615.200 Sender: owner-obm-l@sucuri.mat.puc-rio.br Precedence: bulk Reply-To: obm-l@mat.puc-rio.br > On Thu, Jun 13, 2002 at 02:31:46PM -0300, Vinicius José Fortuna wrote: > > Pessoal, > > > > Como é que se faz para participar da Olimpíada Iberoamericana de Matemática? > > Pelo subject, deduzo que você está falando da Iberoamericana *Universitária*. > Esta ocorre mais para o fim do ano, depois da 1a fase da OBM nível univ > e só é recomendável para quem tem uma certa experiência com olimpíadas > de matemática, ou seja, para quem fez pelo menos a 1a fase da OBM e teve > um bom resultado. > > []s, N. Tudo bem, mas como proceder para participar? Abrac,os, Eric. ========================================================================= Instruções para entrar na lista, sair da lista e usar a lista em http://www.mat.puc-rio.br/~nicolau/olimp/obm-l.html O administrador desta lista é ========================================================================= From owner-obm-l@sucuri.mat.puc-rio.br Fri Jun 14 01:26:42 2002 Return-Path: Received: (from majordom@localhost) by sucuri.mat.puc-rio.br (8.9.3/8.9.3) id BAA13270 for obm-l-list; Fri, 14 Jun 2002 01:26:23 -0300 Received: from hotmail.com (f108.pav2.hotmail.com [64.4.37.108]) by sucuri.mat.puc-rio.br (8.9.3/8.9.3) with ESMTP id BAA13266 for ; Fri, 14 Jun 2002 01:26:21 -0300 Received: from mail pickup service by hotmail.com with Microsoft SMTPSVC; Thu, 13 Jun 2002 21:25:50 -0700 Received: from 200.199.178.63 by pv2fd.pav2.hotmail.msn.com with HTTP; Fri, 14 Jun 2002 04:25:50 GMT X-Originating-IP: [200.199.178.63] From: "Fernanda Medeiros" To: obm-l@mat.puc-rio.br Subject: Re: [obm-l] ???????? Date: Fri, 14 Jun 2002 04:25:50 +0000 Mime-Version: 1.0 Content-Type: text/plain; charset=iso-8859-1; format=flowed Message-ID: X-OriginalArrivalTime: 14 Jun 2002 04:25:50.0783 (UTC) FILETIME=[908BD4F0:01C2135B] Sender: owner-obm-l@sucuri.mat.puc-rio.br Precedence: bulk Reply-To: obm-l@mat.puc-rio.br Oi! (IMO-63) Se vc resolveu o de baixo entaum viu q soh eh divisivel por 7 qnd n=3k k inteiro (2^n==1(mod7) ); dae sabemos q n soh pode ser da forma 3k,3k+1 ou 3k+2, logo: 2^3k -1=7x => 2^3k +1=7x +2 2^(3k+1) -1 =7y +1 => 2^(3k+1) +1=7y+3 2^(3k+2) -1=7z+3 => 2^(3k+2)+1=7z+5 logo, 2^n==/ -1(mod7) pra todo n inteiro :) []´s Fê >Olá colegas de lista, > >Eu gostaria de ajuda no seguinte problema: > > >Prove que não existe n natural tal que (2^n + 1) seja divisível por 7. > >Ah!Eu resolvi este outro utilizando congruências: > >Encontre todos os valores de n para os quais (2^n - 1)seja divisível por 7. > >Será que há outra maneira? > >São parecidos...Mas eu ainda não "matei" o último.Desde já,obrigado por >qualquer comentário. > > >Eder _________________________________________________________________ O MSN Photos é o modo mais fácil de compartilhar e imprimir suas fotos: http://photos.msn.com/support/worldwide.aspx ========================================================================= Instruções para entrar na lista, sair da lista e usar a lista em http://www.mat.puc-rio.br/~nicolau/olimp/obm-l.html O administrador desta lista é ========================================================================= From owner-obm-l@sucuri.mat.puc-rio.br Fri Jun 14 01:37:46 2002 Return-Path: Received: (from majordom@localhost) by sucuri.mat.puc-rio.br (8.9.3/8.9.3) id BAA13445 for obm-l-list; Fri, 14 Jun 2002 01:37:43 -0300 Received: from web10101.mail.yahoo.com (web10101.mail.yahoo.com [216.136.130.51]) by sucuri.mat.puc-rio.br (8.9.3/8.9.3) with SMTP id BAA13441 for ; Fri, 14 Jun 2002 01:37:40 -0300 Message-ID: <20020614043710.34539.qmail@web10101.mail.yahoo.com> Received: from [150.161.199.16] by web10101.mail.yahoo.com via HTTP; Thu, 13 Jun 2002 21:37:10 PDT Date: Thu, 13 Jun 2002 21:37:10 -0700 (PDT) From: Rafael WC Subject: [obm-l] difícil To: OBM MIME-Version: 1.0 Content-Type: text/plain; charset=us-ascii Sender: owner-obm-l@sucuri.mat.puc-rio.br Precedence: bulk Reply-To: obm-l@mat.puc-rio.br Essa é de Olimpíada! Se a, b, c são números reais tais que: (bc - a²)^(-1) + (ca - b²)^(-1) + (ab - c²)^(-1) = 0 Então: a.(bc - a²)^(-2) + b.(ac - b²)^(-2) + c.(ab -c²)^(-2) é igual a... A resposta é zero, mas nunca consegui resolver esse exercício e nunca vi a resolução. Se alguém conseguir fazer... Abraços, Rafael. ===== Rafael Werneck Cinoto ICQ# 107011599 rwcinoto@yahoo.com rafael.caixa@gov.com.br matduvidas@yahoo.com.br http://www.rwcinoto.hpg.com.br/ __________________________________________________ Do You Yahoo!? Yahoo! - Official partner of 2002 FIFA World Cup http://fifaworldcup.yahoo.com ========================================================================= Instruções para entrar na lista, sair da lista e usar a lista em http://www.mat.puc-rio.br/~nicolau/olimp/obm-l.html O administrador desta lista é ========================================================================= From owner-obm-l@sucuri.mat.puc-rio.br Fri Jun 14 13:18:32 2002 Return-Path: Received: (from majordom@localhost) by sucuri.mat.puc-rio.br (8.9.3/8.9.3) id NAA19257 for obm-l-list; Fri, 14 Jun 2002 13:16:52 -0300 Received: from www.zipmail.com.br (smtp.zipmail.com.br [200.187.242.10]) by sucuri.mat.puc-rio.br (8.9.3/8.9.3) with ESMTP id NAA19239 for ; Fri, 14 Jun 2002 13:16:44 -0300 From: peterdirichlet@zipmail.com.br Received: from [200.206.103.3] by www.zipmail.com.br with HTTP; Fri, 14 Jun 2002 13:15:58 -0300 Message-ID: <3D08872500002094@www.zipmail.com.br> Date: Fri, 14 Jun 2002 13:15:58 -0300 In-Reply-To: <200206131525.g5DFPI809255@Gauss.impa.br> Subject: [obm-l] =?iso-8859-1?Q?Re=3A=20IMO=202001=2DProblema=206=28valeu=21=21=21=21=29?= To: =?iso-8859-1?Q?Carlos=20Gustavo=20Tamm=20de=20Araujo=20Moreira?= , =?iso-8859-1?Q?Lista=20de=20Discussao?= MIME-Version: 1.0 Content-Type: text/plain; charset="iso-8859-1" Content-Transfer-Encoding: 8bit X-MIME-Autoconverted: from quoted-printable to 8bit by sucuri.mat.puc-rio.br id NAA19245 Sender: owner-obm-l@sucuri.mat.puc-rio.br Precedence: bulk Reply-To: obm-l@mat.puc-rio.br Bem,eu ja tenho essa soluçao baseada na oficial(e a oficial,que usava trigonometria(?))e a do Antonio Caminha(que "inteirizou" os inteiros de Eisenstein)e uma que esta no site oficial da IMO. De qualquer jeito,valeu por tudo!!!!! Ass.:Peterdirichlet. -- Mensagem original -- > Caro Peterdirichlet, > Minha solucao na epoca ficou bastante enrolada:era baseada em achar todas >as solucoes racionais (x,y,z)=(a/d,b/d,c/d) de xz+y=(x+y+1-z)(1+y-x+z), que >e' uma equacao do segundo grau da qual conhecemos solucoes (por exemplo >(x,y,z)=(0,0,1)).Para achar todas as solucoes e' so' ver onde retas >parametrizadas do tipo {(0,0,1)+t(u,v,w)} com u,v,w inteiros intersectam >nossa superficie.Vai dar uma equacao do segundo grau em t da qual ja' >sabemos que 0 e' solucao,donde a outra solucao vai ser uma funcao racional >de u,v,w.A partir dai pode-se construir uma fatoracao explicita de ab+cd,se >eu nao me engano. > De qualquer jeito a solucao do Nicolau usando inteiros de Eisenstein e' > >bem melhor... > Abracos, > Gugu >P.S.:Voce ja' viu a solucao da pagina do John Scholes >(http://www.kalva.demon.co.uk/imo/isoln/isoln016.html) ? > >> >>OI Gugu!Eu sou um participante da lista de problemas,e queria saber qual >>foi sua soluçao para o problema 6 da IMO 2001(consegui juntar varias respostas) >>Falow,Peterdirichlet >> >>TRANSIRE SVVM PECTVS MVNDOQUE POTIRE >>CONGREGATI EX TOTO ORBE MATHEMATICI OB SCRIPTA INSIGNIA TRIBVERE >>Medalha Fields(John Charles Fields) >> >> >>------------------------------------------ >>Use o melhor sistema de busca da Internet >>Radar UOL - http://www.radaruol.com.br >> >> >> > > TRANSIRE SVVM PECTVS MVNDOQUE POTIRE CONGREGATI EX TOTO ORBE MATHEMATICI OB SCRIPTA INSIGNIA TRIBVERE Medalha Fields(John Charles Fields) ------------------------------------------ Use o melhor sistema de busca da Internet Radar UOL - http://www.radaruol.com.br ========================================================================= Instruções para entrar na lista, sair da lista e usar a lista em http://www.mat.puc-rio.br/~nicolau/olimp/obm-l.html O administrador desta lista é ========================================================================= From owner-obm-l@sucuri.mat.puc-rio.br Fri Jun 14 13:18:36 2002 Return-Path: Received: (from majordom@localhost) by sucuri.mat.puc-rio.br (8.9.3/8.9.3) id NAA19248 for obm-l-list; Fri, 14 Jun 2002 13:16:51 -0300 Received: from www.zipmail.com.br (smtp.zipmail.com.br [200.187.242.10]) by sucuri.mat.puc-rio.br (8.9.3/8.9.3) with ESMTP id NAA19242 for ; Fri, 14 Jun 2002 13:16:47 -0300 From: peterdirichlet@zipmail.com.br Received: from [200.206.103.3] by www.zipmail.com.br with HTTP; Fri, 14 Jun 2002 13:15:56 -0300 Message-ID: <3D08872500002093@www.zipmail.com.br> Date: Fri, 14 Jun 2002 13:15:56 -0300 In-Reply-To: <200206131525.g5DFPI809255@Gauss.impa.br> Subject: [obm-l] =?iso-8859-1?Q?Re=3A=20IMO=202001=2DProblema=206=28valeu=21=21=21=21=29?= To: =?iso-8859-1?Q?Carlos=20Gustavo=20Tamm=20de=20Araujo=20Moreira?= , =?iso-8859-1?Q?Lista=20de=20Discussao?= MIME-Version: 1.0 Content-Type: text/plain; charset="iso-8859-1" Content-Transfer-Encoding: 8bit X-MIME-Autoconverted: from quoted-printable to 8bit by sucuri.mat.puc-rio.br id NAA19243 Sender: owner-obm-l@sucuri.mat.puc-rio.br Precedence: bulk Reply-To: obm-l@mat.puc-rio.br Bem,eu ja tenho essa soluçao baseada na oficial(e a oficial,que usava trigonometria(?))e a do Antonio Caminha(que "inteirizou" os inteiros de Eisenstein)e uma que esta no site oficial da IMO. De qualquer jeito,valeu por tudo!!!!! Ass.:Peterdirichlet. -- Mensagem original -- > Caro Peterdirichlet, > Minha solucao na epoca ficou bastante enrolada:era baseada em achar todas >as solucoes racionais (x,y,z)=(a/d,b/d,c/d) de xz+y=(x+y+1-z)(1+y-x+z), que >e' uma equacao do segundo grau da qual conhecemos solucoes (por exemplo >(x,y,z)=(0,0,1)).Para achar todas as solucoes e' so' ver onde retas >parametrizadas do tipo {(0,0,1)+t(u,v,w)} com u,v,w inteiros intersectam >nossa superficie.Vai dar uma equacao do segundo grau em t da qual ja' >sabemos que 0 e' solucao,donde a outra solucao vai ser uma funcao racional >de u,v,w.A partir dai pode-se construir uma fatoracao explicita de ab+cd,se >eu nao me engano. > De qualquer jeito a solucao do Nicolau usando inteiros de Eisenstein e' > >bem melhor... > Abracos, > Gugu >P.S.:Voce ja' viu a solucao da pagina do John Scholes >(http://www.kalva.demon.co.uk/imo/isoln/isoln016.html) ? > >> >>OI Gugu!Eu sou um participante da lista de problemas,e queria saber qual >>foi sua soluçao para o problema 6 da IMO 2001(consegui juntar varias respostas) >>Falow,Peterdirichlet >> >>TRANSIRE SVVM PECTVS MVNDOQUE POTIRE >>CONGREGATI EX TOTO ORBE MATHEMATICI OB SCRIPTA INSIGNIA TRIBVERE >>Medalha Fields(John Charles Fields) >> >> >>------------------------------------------ >>Use o melhor sistema de busca da Internet >>Radar UOL - http://www.radaruol.com.br >> >> >> > > TRANSIRE SVVM PECTVS MVNDOQUE POTIRE CONGREGATI EX TOTO ORBE MATHEMATICI OB SCRIPTA INSIGNIA TRIBVERE Medalha Fields(John Charles Fields) ------------------------------------------ Use o melhor sistema de busca da Internet Radar UOL - http://www.radaruol.com.br ========================================================================= Instruções para entrar na lista, sair da lista e usar a lista em http://www.mat.puc-rio.br/~nicolau/olimp/obm-l.html O administrador desta lista é ========================================================================= From owner-obm-l@sucuri.mat.puc-rio.br Fri Jun 14 14:02:46 2002 Return-Path: Received: (from majordom@localhost) by sucuri.mat.puc-rio.br (8.9.3/8.9.3) id OAA20322 for obm-l-list; Fri, 14 Jun 2002 14:00:37 -0300 Received: from web14809.mail.yahoo.com (web14809.mail.yahoo.com [216.136.224.230]) by sucuri.mat.puc-rio.br (8.9.3/8.9.3) with SMTP id OAA20317 for ; Fri, 14 Jun 2002 14:00:34 -0300 Message-ID: <20020614170004.53346.qmail@web14809.mail.yahoo.com> Received: from [200.17.79.42] by web14809.mail.yahoo.com via HTTP; Fri, 14 Jun 2002 14:00:04 ART Date: Fri, 14 Jun 2002 14:00:04 -0300 (ART) From: =?iso-8859-1?q?Ricardo=20Miranda?= Subject: [obm-l] Mais duvidas de analitica/geo plana To: obm-l@mat.puc-rio.br MIME-Version: 1.0 Content-Type: text/plain; charset=iso-8859-1 Content-Transfer-Encoding: 8bit Sender: owner-obm-l@sucuri.mat.puc-rio.br Precedence: bulk Reply-To: obm-l@mat.puc-rio.br Eu, de novo, com meus problemas de analitica. Tendo dois pontos A(a,b) B(c,d), eu consigo achar a equacao da reta que passa pelos dois pontos multiplicando a matriz {a, b | c, d} por {x,y}. Como eu posso provar que isso é verdade? Outra coisa que eu fiz, mas acho que a resposta nao está conferindo. Como eu provo que as diagonais de um paralelogramo se cortam ao meio? Pode ser por analitica ou por plana. ===== []s Ricardo Miranda Matematica - UFV ricardomirandabr@yahoo.com.br http://rm2.hpg.ig.com.br/ _______________________________________________________________________ Copa 2002 Yahoo! - Patrocinador oficial da Copa do Mundo da FIFA 2002 http://br.sports.yahoo.com/fifaworldcup/ ========================================================================= Instruções para entrar na lista, sair da lista e usar a lista em http://www.mat.puc-rio.br/~nicolau/olimp/obm-l.html O administrador desta lista é ========================================================================= From owner-obm-l@sucuri.mat.puc-rio.br Fri Jun 14 15:10:19 2002 Return-Path: Received: (from majordom@localhost) by sucuri.mat.puc-rio.br (8.9.3/8.9.3) id PAA21674 for obm-l-list; Fri, 14 Jun 2002 15:08:15 -0300 Received: (from nicolau@localhost) by sucuri.mat.puc-rio.br (8.9.3/8.9.3) id PAA21669 for obm-l@mat.puc-rio.br; Fri, 14 Jun 2002 15:08:15 -0300 Date: Fri, 14 Jun 2002 15:08:15 -0300 From: "Nicolau C. Saldanha" To: obm-l@mat.puc-rio.br Subject: [obm-l] Re: =?iso-8859-1?Q?=5Bobm-l=5D_Re:_=5Bobm-l=5D_Re:_=5Bobm-l=5D_Iberoamerican?= =?iso-8859-1?Q?a_Universit=E1ria?= Message-ID: <20020614150815.A21663@sucuri.mat.puc-rio.br> References: <20020613064745.90169.qmail@web10106.mail.yahoo.com><002e01c21300$322f65d0$0401010a@xt> <20020613160839.A32093@sucuri.mat.puc-rio.br> <001501c21338$6b857580$c80497c8@windows9> Mime-Version: 1.0 Content-Type: text/plain; charset=iso-8859-1 Content-Disposition: inline Content-Transfer-Encoding: 8bit User-Agent: Mutt/1.2.5i In-Reply-To: <001501c21338$6b857580$c80497c8@windows9>; from mathfire@ig.com.br on Thu, Jun 13, 2002 at 09:14:01PM -0300 Sender: owner-obm-l@sucuri.mat.puc-rio.br Precedence: bulk Reply-To: obm-l@mat.puc-rio.br On Thu, Jun 13, 2002 at 09:14:01PM -0300, Eric Campos Bastos Guedes wrote: > > On Thu, Jun 13, 2002 at 02:31:46PM -0300, Vinicius José Fortuna wrote: > > > Pessoal, > > > > > > Como é que se faz para participar da Olimpíada Iberoamericana de > Matemática? > > > > Pelo subject, deduzo que você está falando da Iberoamericana > *Universitária*. > > Esta ocorre mais para o fim do ano, depois da 1a fase da OBM nível univ > > e só é recomendável para quem tem uma certa experiência com olimpíadas > > de matemática, ou seja, para quem fez pelo menos a 1a fase da OBM e teve > > um bom resultado. > > > > []s, N. > > Tudo bem, mas como proceder para participar? Fale com seu coordenador universitário (ou de outra universidade), ele deverá receber a prova no devido tempo. []s, N. ========================================================================= Instruções para entrar na lista, sair da lista e usar a lista em http://www.mat.puc-rio.br/~nicolau/olimp/obm-l.html O administrador desta lista é ========================================================================= From owner-obm-l@sucuri.mat.puc-rio.br Fri Jun 14 17:52:11 2002 Return-Path: Received: (from majordom@localhost) by sucuri.mat.puc-rio.br (8.9.3/8.9.3) id RAA24693 for obm-l-list; Fri, 14 Jun 2002 17:51:06 -0300 Received: from mail.gmx.net (mail.gmx.net [213.165.64.20]) by sucuri.mat.puc-rio.br (8.9.3/8.9.3) with SMTP id RAA24689 for ; Fri, 14 Jun 2002 17:51:03 -0300 Received: (qmail 27093 invoked by uid 0); 14 Jun 2002 20:50:33 -0000 Received: from 200-206-211-169.dsl.telesp.net.br (HELO mentecapto.gmx.de) (200.206.211.169) by mail.gmx.net (mp007-rz3) with SMTP; 14 Jun 2002 20:50:33 -0000 Message-Id: <5.1.0.14.2.20020614171511.00aa8618@pop.gmx.net> X-Sender: mentus@gmx.de@pop.gmx.net X-Mailer: QUALCOMM Windows Eudora Version 5.1 Date: Fri, 14 Jun 2002 17:49:18 -0300 To: obm-l@mat.puc-rio.br From: "Fernando Henrique Ferraz P. da Rosa" Subject: Re: [obm-l] Mais duvidas de analitica/geo plana In-Reply-To: <20020614170004.53346.qmail@web14809.mail.yahoo.com> Mime-Version: 1.0 Content-Type: multipart/mixed; x-avg-checked=avg-ok-24685364; boundary="=====================_338183562==_" Sender: owner-obm-l@sucuri.mat.puc-rio.br Precedence: bulk Reply-To: obm-l@mat.puc-rio.br --=====================_338183562==_ Content-Type: text/plain; x-avg-checked=avg-ok-24685364; charset=iso-8859-1; format=flowed Content-Transfer-Encoding: quoted-printable At 14:00 6/14/2002 -0300, you wrote: >Eu, de novo, com meus problemas de analitica. > >Tendo dois pontos A(a,b) B(c,d), eu consigo achar a equacao da reta que >passa pelos dois pontos multiplicando a matriz {a, b | c, d} por {x,y}. >Como eu posso provar que isso =E9 verdade? >Outra coisa que eu fiz, mas acho que a resposta nao est=E1 conferindo. >Como eu provo que as diagonais de um paralelogramo se cortam ao meio? >Pode ser por analitica ou por plana. Hum.. vou comentar primeiro o segundo que tenho mais certeza. Temos o=20 paralelogramo ABCD (segue figura). Seja M o ponto m=E9dio do segmento AC, ou= =20 seja, AM[vetor] =3D MC[vetor]. Queremos mostrar que M tamb=E9m =E9 ponto= m=E9dio do=20 segumento BC, ou seja, que BM[vetor] =3D MD[vetor]. BM[vetor] =3D BC[vetor] + CM[vetor] =3D MA[vetor] + AD[vetor] =3D -DM[vetor]= =3D=20 MC[vetor]. Logo as duas diagonais AC e BD se cortam ao meio. O primeiro problema =E9 algo que vem me atormentando h=E1 tempos mas eu= conhe=E7o=20 de uma maneira levemente diferente. Sejam os pontos A=3D(a,b) e B=3D(c,d)=20 montando a seguinte matriz e igualando o determinante a 0, tambem chegamos= =20 na equa=E7=E3o geral da reta que passa por esses dois pontos: | x y 1 | | a b 1 | =3D 0 | c d 1 | matriz [1] Essa matriz tamb=E9m tem outras propriedades misteriosas... Tomemos um=20 triangulo em E=B2 com v=E9rtices A=3D(a,b), B=3D(c,d) e C=3D(e,f), sendo D o= =20 determinante da matriz abaixo: | a b 1 | | c d 1 | | e f 1 | matriz[2] A =E1rea do tri=E2ngulo desse triangulo =E9 dada por |D|/2. A partir dessa propriedade fica natural 'zerarmos' o determinante que=20 usamos para achar a equa=E7=E3o da reta... j=E1 que se os pontos sao= colineares=20 eles nao podem formar um triangulo e portanto a area deve ser nula. Ainda assim.. s=F3 disfar=E7amos um pouco o problema... fica a quest=E3o.= porque=20 cargas d'=E1gua ao montarmos a matriz [2] acima, calcularmos o determinante,= =20 pegarmos seu valor absoluto e dividirmos por dois temos a area do= triangulo... Quando trabalhamos em E=B3, quando temos tres vetores u=3D(x1,y1,z1)=20 v=3D(x2,y2,z2) e w=3D(x3,y3,z3). e calculamos o determinante da matriz= formada=20 pelos valores de x1,y1,z1...etc. se os tres vetores forem linearmente=20 independentes, obtemos o volume do paralelepipedo dos quais os tres sao=20 vertices. Se eles forem linearmente dependentes esse determinante =E9 0. Nao sei at=E9 que ponto essas propriedades misteriosas da matrizes se= aplicam=20 ou podem ser provadas em E=B2. Voltando para a matriz[1] podemos imaginar=20 tres vetores, v=3D(x,y,1), u=3D(c,d,1) e w=3D(e,f,1). Ao impormos= D(matriz[1]) =3D=20 0, queremos que esses tres vetores sejam linearmente dependentes, ou seja=20 paralelos ao mesmo plano. Novamente.. ainda =E9 um mist=E9rio pra mim como= isso=20 se relaciona as retas em E=B2. Bom, acho que s=F3 atrapalhei e inventei mais duvidas do que solucionei seu= =20 problema de fato... mas como j=E1 comentei era algo que vinha me= atormentando=20 e com sorte outros membros da lista vao jogar uma luz nisso :P "... a perfect formulation of a problem is already half its solution." David Hilbert. - []'s Fernando Henrique Ferraz Pereira da Rosa USP, IME, Estat=EDstica http://www.linux.ime.usp.br/~feferraz --=====================_338183562==_ Content-Type: image/gif; x-avg-checked=avg-ok-24685364; name=paralel.gif; x-mac-type=47494666; x-mac-creator=4A565752 Content-Disposition: attachment; filename="paralel.gif" Content-Transfer-Encoding: base64 R0lGODlh2wBsAJEAAB8aFwB8w////wAAACwAAAAA2wBsAAAI/gAFCBxIsKDBgwgTKhwYoGGAhRAj SpxIsaLFixgzaqz4UGDHjSBDihwJ0uFHkiRPnkTJsqXLjSpfhowps6bNmwVX4sxIc6fPnyJ1AqXY c6jRoxGFIl34UenSp0idQjVocqrVqUWvat1qtSrXgQDCih1LtqzZs2jTql3Ltq3bt3ABfEUoN+zc uy3tCpCLlyDfvX0Da9QLWHBhsIYTRyR8OPBfv4ojFxRr8HFfy4glK2YM2TDmzpovf84seDTp0Fw5 V05s2m9r1EApQ3x9lTZo2EdVJ7Rtlfdp3D5172Z90TdwkLIpGn+6/PdxlsJnE8/Y/Pni6s7vYs9u HWN0iduH/oYHO/558sGeSZaH/b3i+p3vuXc/2N79dJTxHde3mN9mf/nW7VfcfXnN55pM/yFYU4LM MQjgVg4eCNx5L0WIX3DsWXjbZT9p6JKAInk4koiTsQbiiARiKJpRJIbU4mpznQhdirGl9iKMpUEl Y4U7Fphjb0v1+CGNQbJ4o0JHUpdacEIqmN6S/iUp3ZNfRdjkTVIOqF1eWS5GJITvUVgllY5tdGWH X8Z40Eo6nYkmmXAK0BBDH4nJYZw/0umRnHt1qSSeZD4kqJx+ogcolYMGAIBUd+aJml2KzsmnZoVO VGmIHWUa2qVeTljXcZxO+ShjoXqXpn50sXcqXu2Vqtyq/1Xy5mqnjuq3nJtv1spqeLOmeqhWuPZa 4q9T2fknq7AihSt9yBKrrIXCCotjbSLOKu2wvS1raajXbhhklpx2+yB8hVYqrkB+GsuijsnyOKay 7XLZ7LrOylumePGGqC1O+9J6L7/nMgtfvoMFPByWBHtn8EIeBmxlv8UumPC2Bnqr3sSzLWyqj7qa CfGuF9ZLccX+IodxZRrP6OLJEpJsn8ki0/UxbuM5TF3KAx/7r3I45/pyx0jOTLKMNo/sspPgtSv0 0dgynOzSTE97MNDoQh21wFPvPFnPWjc979ZXM+drreqG3aFlRVdtNoQWq9nn2u/abDXc6J0b1914 56333j58nxUYo3QHDhPggmvl0FeDFi6Ypobvqfjfjl/V1ON9Hb6VSYRTHlXkWBGUueZGMd5VTqAj znnpqCtk+VUBAQA7 --=====================_338183562==_ Content-Type: text/plain; charset=us-ascii; x-avg=cert; x-avg-checked=avg-ok-24685364 Content-Disposition: inline --- Outgoing mail is certified Virus Free. Checked by AVG anti-virus system (http://www.grisoft.com). Version: 6.0.370 / Virus Database: 205 - Release Date: 6/5/2002 --=====================_338183562==_-- ========================================================================= Instruções para entrar na lista, sair da lista e usar a lista em http://www.mat.puc-rio.br/~nicolau/olimp/obm-l.html O administrador desta lista é ========================================================================= From owner-obm-l@sucuri.mat.puc-rio.br Sat Jun 15 00:15:05 2002 Return-Path: Received: (from majordom@localhost) by sucuri.mat.puc-rio.br (8.9.3/8.9.3) id AAA27561 for obm-l-list; Sat, 15 Jun 2002 00:14:38 -0300 Received: from ginsberg.uol.com.br (ginsberg.uol.com.br [200.231.206.26]) by sucuri.mat.puc-rio.br (8.9.3/8.9.3) with ESMTP id AAA27557 for ; Sat, 15 Jun 2002 00:14:36 -0300 Received: from xxx ([200.191.161.163]) by ginsberg.uol.com.br (8.9.1/8.9.1) with SMTP id AAA19377 for ; Sat, 15 Jun 2002 00:12:57 -0300 (BRT) Message-ID: <000e01c21424$22694e20$a3a1bfc8@xxx> From: "haroldo" To: Subject: Re: [obm-l] Mais duvidas de analitica/geo plana Date: Sat, 15 Jun 2002 01:21:32 -0300 MIME-Version: 1.0 Content-Type: text/plain; charset="iso-8859-1" Content-Transfer-Encoding: 8bit X-Priority: 3 X-MSMail-Priority: Normal X-Mailer: Microsoft Outlook Express 4.72.3110.5 X-MimeOLE: Produced By Microsoft MimeOLE V4.72.3110.3 Sender: owner-obm-l@sucuri.mat.puc-rio.br Precedence: bulk Reply-To: obm-l@mat.puc-rio.br -----Mensagem original----- De: Ricardo Miranda Para: obm-l@mat.puc-rio.br Data: Sexta-feira, 14 de Junho de 2002 14:41 Assunto: [obm-l] Mais duvidas de analitica/geo plana >Eu, de novo, com meus problemas de analitica. > >Tendo dois pontos A(a,b) B(c,d), eu consigo achar a equacao da reta que >passa pelos dois pontos multiplicando a matriz {a, b | c, d} por {x,y}. >Como eu posso provar que isso é verdade? > >Outra coisa que eu fiz, mas acho que a resposta nao está conferindo. >Como eu provo que as diagonais de um paralelogramo se cortam ao meio? >Pode ser por analitica ou por plana. > >===== >[]s >Ricardo Miranda >Matematica - UFV >ricardomirandabr@yahoo.com.br >http://rm2.hpg.ig.com.br/ > >_______________________________________________________________________ >Copa 2002 >Yahoo! - Patrocinador oficial da Copa do Mundo da FIFA 2002 >http://br.sports.yahoo.com/fifaworldcup/ >========================================================================= >Instruções para entrar na lista, sair da lista e usar a lista em >http://www.mat.puc-rio.br/~nicolau/olimp/obm-l.html >O administrador desta lista é >========================================================================= >Seja ABCD paralelogramo , AC INTERSECÇÃO BD = M ----> AM = CM e BM = DM DEMO: ABCD É PARALELOGRAMO ----> AB = CD e AB// CD ----> ANGULOS BAC = DCA E ABD =CDB LOGO TRIANGULOS ABM E CDM CONGRUENTES (ALA) ---> AM=CM e BM=DM. ========================================================================= Instruções para entrar na lista, sair da lista e usar a lista em http://www.mat.puc-rio.br/~nicolau/olimp/obm-l.html O administrador desta lista é ========================================================================= From owner-obm-l@sucuri.mat.puc-rio.br Sat Jun 15 04:13:54 2002 Return-Path: Received: (from majordom@localhost) by sucuri.mat.puc-rio.br (8.9.3/8.9.3) id EAA29240 for obm-l-list; Sat, 15 Jun 2002 04:13:29 -0300 Received: from pina.terra.com.br (pina.terra.com.br [200.176.3.17]) by sucuri.mat.puc-rio.br (8.9.3/8.9.3) with ESMTP id EAA29237 for ; Sat, 15 Jun 2002 04:13:27 -0300 Received: from pacuiba.terra.com.br (pacuiba.terra.com.br [200.176.3.40]) by pina.terra.com.br (Postfix) with ESMTP id C212A52E90 for ; Sat, 15 Jun 2002 04:13:01 -0300 (EST) Received: from stabel (dl-nas3-poa-C89A0674.p001.terra.com.br [200.154.6.116]) (authenticated user dudasta) by pacuiba.terra.com.br (Postfix) with ESMTP id D37337FA5 for ; Sat, 15 Jun 2002 04:12:59 -0300 (EST) Message-ID: <003701c2143c$15a085b0$74069ac8@stabel> From: "Eduardo Casagrande Stabel" To: References: <5.1.0.14.2.20020614171511.00aa8618@pop.gmx.net> Subject: Re: [obm-l] Mais duvidas de analitica/geo plana Date: Sat, 15 Jun 2002 04:12:58 -0300 MIME-Version: 1.0 Content-Type: text/plain; charset="iso-8859-1" Content-Transfer-Encoding: 8bit X-Priority: 3 X-MSMail-Priority: Normal X-Mailer: Microsoft Outlook Express 6.00.2600.0000 X-MimeOLE: Produced By Microsoft MimeOLE V6.00.2600.0000 Sender: owner-obm-l@sucuri.mat.puc-rio.br Precedence: bulk Reply-To: obm-l@mat.puc-rio.br Ola! Um jeito de esclarever sua dúvida é fazer o seguinte. Sejam (a,b) e (c,d) dois pontos distintos do plano. Prove o seguinte: Se o ponto (x,y) pertence à reta que passa pelos dois pontos então existe um real t tal que t*(a,b) + (1-t)*(c,d) = (x,y) Repare que o grafico da função t -> t*(a,b) + (1-t)*(c,d) é uma reta, e calcule t=0 e t=1 para ver que ela passa pelos pontos (a,b) e (c,d). Depois de provar isso, use as propriedades do determinante. Se o determinante for | x y 1 | | a b 1 | = 0 | c d 1 | então as linhas são linearmente dependentes. Como as duas últimas são linearmente independentes segue que a primeira é combinação das duas últimas (x,y,1) = q*(a,b,1) + p*(c,d,1) Temos q + p = 1, substitui q = t e p = 1 - t (x,y,1) = t*(a,b,1) + (1-t)*(c,d,1) o que implica (x,y) = t*(a,b) + (1-t)*(c,d) e daí (x,y) pertence à reta que passa por (a,b) e (c,d). Reciprocamente, se (x,y) pertence à essa reta, existe t tal que (x,y) = t*(a,b) + (1-t)*(c,d) e daí (x,y,1) = t*(a,b,1) + (1-t)*(c,d,1) o que implica que (x,y,1) é combinação linear de (a,b,1) e (c,d,1) daí o determinante | x y 1 | | a b 1 | = 0 | c d 1 | o que completa a prova. Em relação à primeira observação, a saber, que todos os pontos da reta são obtidos multiplicando-se | a b | | x | | c d |.| y | tenho que dizer que ela não é verdadeira. Por que? Ponha (a,b) = (1,0) e (c,d) = (0,1), todos que estudaram um pouco de matrizes sabem que todos os pontos do plano podem ser obtidos fazendo a multiplicação matricial aí de cima, portanto não se trate de uma reta. Uma possibilidade, em termos de multiplicação seria | a c | | t | | b d |.|1-t| o que é igual à primeira observação do e-mail. Um abraço! Eduardo Casagrande Stabel. Porto Alegre, RS. From: "Fernando Henrique Ferraz P. da Rosa" >Eu, de novo, com meus problemas de analitica. > >Tendo dois pontos A(a,b) B(c,d), eu consigo achar a equacao da reta que >passa pelos dois pontos multiplicando a matriz {a, b | c, d} por {x,y}. >Como eu posso provar que isso é verdade? >Outra coisa que eu fiz, mas acho que a resposta nao está conferindo. >Como eu provo que as diagonais de um paralelogramo se cortam ao meio? >Pode ser por analitica ou por plana. Hum.. vou comentar primeiro o segundo que tenho mais certeza. Temos o paralelogramo ABCD (segue figura). Seja M o ponto médio do segmento AC, ou seja, AM[vetor] = MC[vetor]. Queremos mostrar que M também é ponto médio do segumento BC, ou seja, que BM[vetor] = MD[vetor]. BM[vetor] = BC[vetor] + CM[vetor] = MA[vetor] + AD[vetor] = -DM[vetor] = MC[vetor]. Logo as duas diagonais AC e BD se cortam ao meio. O primeiro problema é algo que vem me atormentando há tempos mas eu conheço de uma maneira levemente diferente. Sejam os pontos A=(a,b) e B=(c,d) montando a seguinte matriz e igualando o determinante a 0, tambem chegamos na equação geral da reta que passa por esses dois pontos: | x y 1 | | a b 1 | = 0 | c d 1 | matriz [1] Essa matriz também tem outras propriedades misteriosas... Tomemos um triangulo em E² com vértices A=(a,b), B=(c,d) e C=(e,f), sendo D o determinante da matriz abaixo: | a b 1 | | c d 1 | | e f 1 | matriz[2] A área do triângulo desse triangulo é dada por |D|/2. A partir dessa propriedade fica natural 'zerarmos' o determinante que usamos para achar a equação da reta... já que se os pontos sao colineares eles nao podem formar um triangulo e portanto a area deve ser nula. Ainda assim.. só disfarçamos um pouco o problema... fica a questão. porque cargas d'água ao montarmos a matriz [2] acima, calcularmos o determinante, pegarmos seu valor absoluto e dividirmos por dois temos a area do triangulo... Quando trabalhamos em E³, quando temos tres vetores u=(x1,y1,z1) v=(x2,y2,z2) e w=(x3,y3,z3). e calculamos o determinante da matriz formada pelos valores de x1,y1,z1...etc. se os tres vetores forem linearmente independentes, obtemos o volume do paralelepipedo dos quais os tres sao vertices. Se eles forem linearmente dependentes esse determinante é 0. Nao sei até que ponto essas propriedades misteriosas da matrizes se aplicam ou podem ser provadas em E². Voltando para a matriz[1] podemos imaginar tres vetores, v=(x,y,1), u=(c,d,1) e w=(e,f,1). Ao impormos D(matriz[1]) = 0, queremos que esses tres vetores sejam linearmente dependentes, ou seja paralelos ao mesmo plano. Novamente.. ainda é um mistério pra mim como isso se relaciona as retas em E². Bom, acho que só atrapalhei e inventei mais duvidas do que solucionei seu problema de fato... mas como já comentei era algo que vinha me atormentando e com sorte outros membros da lista vao jogar uma luz nisso :P "... a perfect formulation of a problem is already half its solution." David Hilbert. - []'s Fernando Henrique Ferraz Pereira da Rosa USP, IME, Estatística http://www.linux.ime.usp.br/~feferraz ---------------------------------------------------------------------------- ---- > > --- > Outgoing mail is certified Virus Free. > Checked by AVG anti-virus system (http://www.grisoft.com). > Version: 6.0.370 / Virus Database: 205 - Release Date: 6/5/2002 > ========================================================================= Instruções para entrar na lista, sair da lista e usar a lista em http://www.mat.puc-rio.br/~nicolau/olimp/obm-l.html O administrador desta lista é ========================================================================= From owner-obm-l@sucuri.mat.puc-rio.br Sat Jun 15 10:14:15 2002 Return-Path: Received: (from majordom@localhost) by sucuri.mat.puc-rio.br (8.9.3/8.9.3) id KAA31042 for obm-l-list; Sat, 15 Jun 2002 10:14:07 -0300 Received: from www.zipmail.com.br (smtp.zipmail.com.br [200.187.242.10]) by sucuri.mat.puc-rio.br (8.9.3/8.9.3) with ESMTP id KAA31038 for ; Sat, 15 Jun 2002 10:14:05 -0300 From: luizhenriquerick@zipmail.com.br Received: from [200.165.185.178] by www.zipmail.com.br with HTTP; Sat, 15 Jun 2002 10:13:40 -0300 Message-ID: <3D0ABA96000009F4@www.zipmail.com.br> Date: Sat, 15 Jun 2002 10:13:40 -0300 Subject: [obm-l] =?iso-8859-1?Q?Aritm=E9tica=28D=FAvida=29=20=2D=20Rick=2DC=2ER=2EB=2E?= To: obm-l@mat.puc-rio.br MIME-Version: 1.0 Content-Type: text/plain; charset="iso-8859-1" Content-Transfer-Encoding: 8bit X-MIME-Autoconverted: from quoted-printable to 8bit by sucuri.mat.puc-rio.br id KAA31039 Sender: owner-obm-l@sucuri.mat.puc-rio.br Precedence: bulk Reply-To: obm-l@mat.puc-rio.br Olá amigos .. Estava resolvendo um exercício , consegui resolver na boa , mas o verificar o resultado , vi que não estava batendo.. Vejam só.. Seja N = xyzzyx um número natural escrito na base dez , onde x , y e z são algarismos distintos .Se N1 e N2 são os dois maiores números divisíveis por 3 e 25 , obtido a partir de N pela substituição de x , y e z , então N1 + N2 é igual a : Fiz assim : Para que o número seja divisível por 3 e por 25 , tem que ser divisível então por 75. Fazendo primeiros 12 múltiplos de 75 , temos: 2 x 75 = 150 3 x 75 = 225 4 x 75 = 300 5 x 75 = 375 6 x 75 = 450 7 x 75 = 525 8 x 75 = 600 9 x 75 = 675 10 x 75 = 750 11 x 75 = 825 12 x 75 = 900 13 x 75 = 975 14 x 75 = 1050 15 x 75 = 1125 . . . . . . Então , fazendo N1 > N2 N1 = xyzzyx , verificando x (algarismo das unidades) , o maior valor que pode assumir é 5 , ficamos com... N1 = 5yzzy5 ,verificando y ( algarismo das dezenas ) , o maior valor que pode assumir é 7 , ficamos com ... N1 = 57zz75 , verificando z (algarismo das centenas ) , o maior valor que pode assumir é 9 , ficamos com... N1 = 579975 Agora N2 N2 = xyzzyx , verificando x (algarismo das unidades ) , o maior valor que pode assumir é 5 , ficamos com... N2 = 5yzzy5 ,verificando y ( algarismo das dezenas ) , o maior valor que pode assumir é 5, mas o número terminado em 55 não é divisível por 75, ficando então o 2 , ficamos com ... N2 = 52zz25 , verificando z (algarismo das centenas ) , o maior valor que pode assumir é 8 , ficamos com... N2 = 528825 Então.. N1 + N2 = 1108800 Esta resposta tem entre umas das alternativas , porém a correta é 1157000 Será que alguém poderia me ajudar ? Grato... Rick - C.R.B. ---------------------------------------- |-=Rick-C.R.B.=- | |ICQ 124805654 | |e-mail luizhenriquerick@zipmail.com.br | ---------------------------------------- ------------------------------------------ Use o melhor sistema de busca da Internet Radar UOL - http://www.radaruol.com.br ========================================================================= Instruções para entrar na lista, sair da lista e usar a lista em http://www.mat.puc-rio.br/~nicolau/olimp/obm-l.html O administrador desta lista é ========================================================================= From owner-obm-l@sucuri.mat.puc-rio.br Sat Jun 15 10:17:37 2002 Return-Path: Received: (from majordom@localhost) by sucuri.mat.puc-rio.br (8.9.3/8.9.3) id KAA31116 for obm-l-list; Sat, 15 Jun 2002 10:17:29 -0300 Received: from www.zipmail.com.br (smtp.zipmail.com.br [200.187.242.10]) by sucuri.mat.puc-rio.br (8.9.3/8.9.3) with ESMTP id KAA31112 for ; Sat, 15 Jun 2002 10:17:28 -0300 From: luizhenriquerick@zipmail.com.br Received: from [200.165.185.178] by www.zipmail.com.br with HTTP; Sat, 15 Jun 2002 10:17:01 -0300 Message-ID: <3D0ABA9600000A1F@www.zipmail.com.br> Date: Sat, 15 Jun 2002 10:17:01 -0300 Subject: [obm-l] =?iso-8859-1?Q?Algebra=28ajuda=29?= To: obm-l@mat.puc-rio.br MIME-Version: 1.0 Content-Type: text/plain; charset="iso-8859-1" Content-Transfer-Encoding: 8bit X-MIME-Autoconverted: from quoted-printable to 8bit by sucuri.mat.puc-rio.br id KAA31113 Sender: owner-obm-l@sucuri.mat.puc-rio.br Precedence: bulk Reply-To: obm-l@mat.puc-rio.br Olá amigos .. Será que poderiam me ajudar com estes 2 exercícios ? 1- Se (5² + 9²)(12² + 17²) for escrito sob a forma a² + b² então a + b é igual a : 2- Se x² + y² = 9797 onde x e y são inteiros positivos tais que x > y , existem exatamente dois pares ordenados de inteiros (x,y) que satisfazem tal equação . A soma das coordenadas destes dois pares é: ---------------------------------------- |-=Rick-C.R.B.=- | |ICQ 124805654 | |e-mail luizhenriquerick@zipmail.com.br | ---------------------------------------- ------------------------------------------ Use o melhor sistema de busca da Internet Radar UOL - http://www.radaruol.com.br ========================================================================= Instruções para entrar na lista, sair da lista e usar a lista em http://www.mat.puc-rio.br/~nicolau/olimp/obm-l.html O administrador desta lista é ========================================================================= From owner-obm-l@sucuri.mat.puc-rio.br Sat Jun 15 10:21:32 2002 Return-Path: Received: (from majordom@localhost) by sucuri.mat.puc-rio.br (8.9.3/8.9.3) id KAA31191 for obm-l-list; Sat, 15 Jun 2002 10:21:30 -0300 Received: from www.zipmail.com.br (smtp.zipmail.com.br [200.187.242.10]) by sucuri.mat.puc-rio.br (8.9.3/8.9.3) with ESMTP id KAA31186 for ; Sat, 15 Jun 2002 10:21:28 -0300 From: luizhenriquerick@zipmail.com.br Received: from [200.165.185.178] by www.zipmail.com.br with HTTP; Sat, 15 Jun 2002 10:21:03 -0300 Message-ID: <3D0ABA9600000A51@www.zipmail.com.br> Date: Sat, 15 Jun 2002 10:21:03 -0300 Subject: [obm-l] =?iso-8859-1?Q?Fatora=E7=E3o=2D=20=20Rick=2DC=2ER=2EB?= To: obm-l@mat.puc-rio.br MIME-Version: 1.0 Content-Type: multipart/mixed; boundary="=========3D0ABA9600000A51/www.zipmail.com.br" Sender: owner-obm-l@sucuri.mat.puc-rio.br Precedence: bulk Reply-To: obm-l@mat.puc-rio.br --=========3D0ABA9600000A51/www.zipmail.com.br Content-Type: text/plain; charset="iso-8859-1" Content-Transfer-Encoding: quoted-printable Ol=E1 amigos.. Se puderem me dar uma luz nessa quest=E3o.. Obs: Nicolau , eu sei que vc ja me disse isso mais de mil vezes ..rsrs Mais qual o tamanho ideal para um arquivo ser mandado para a lista ? Grato.. Rick-C.R.B. ---------------------------------------- |-=3DRick-C.R.B.=3D- | |ICQ 124805654 | |e-mail luizhenriquerick@zipmail.com.br | ---------------------------------------- ------------------------------------------ Use o melhor sistema de busca da Internet Radar UOL - http://www.radaruol.com.br --=========3D0ABA9600000A51/www.zipmail.com.br Content-Type: application/octet-stream Content-Transfer-Encoding: base64 Content-Disposition: attachment; filename="Fatoração.wps" 0M8R4KGxGuEAAAAAAAAAAAAAAAAAAAAAPgADAP7/CQAGAAAAAAAAAAAAAAABAAAAAQAAAAAAAAAA EAAAAgAAAAEAAAD+////AAAAAAAAAAD///////////////////////////////////////////// //////////////////////////////////////////////////////////////////////////// //////////////////////////////////////////////////////////////////////////// //////////////////////////////////////////////////////////////////////////// //////////////////////////////////////////////////////////////////////////// //////////////////////////////////////////////////////////////////////////// //////////////////////////////////////////////////////////////////////////// ///////////////////////////////////////////////////////////////////////////9 /////v////7////+/////v///////////////////wkAAAAKAAAACwAAAAwAAAANAAAADgAAAA8A AAAQAAAAEQAAABIAAAATAAAAAwAAAP////////////////////////////////////////////// //////////////////////////////////////////////////////////////////////////// //////////////////////////////////////////////////////////////////////////// //////////////////////////////////////////////////////////////////////////// //////////////////////////////////////////////////////////////////////////// //////////////////////////////////////////////////////////////////////////// //////////////////////////////////////////////////////////////////////////// /////////////////////////////////////////////////////////////////////////1IA bwBvAHQAIABFAG4AdAByAHkAAAAAAAAAAAAAAAAAAAAAAAAAAAAAAAAAAAAAAAAAAAAAAAAAAAAA AAAAAAAWAAUA//////////8BAAAAslqkDgqe0RGkBwDAT7kyugAAAAAAAAAAAAAAAOBcH+luFMIB BAAAAIAAAAAAAAAAQwBPAE4AVABFAE4AVABTAAAAAAAAAAAAAAAAAAAAAAAAAAAAAAAAAAAAAAAA AAAAAAAAAAAAAAAAAAAAAAAAABIAAgECAAAA//////////8AAAAAAAAAAAAAAAAAAAAAAAAAAAAA AAAAAAAAAAAAAAAAAAAIAAAAABoAAAAAAAABAEMAbwBtAHAATwBiAGoAAAAAAAAAAAAAAAAAAAAA AAAAAAAAAAAAAAAAAAAAAAAAAAAAAAAAAAAAAAAAAAAAEgACAP///////////////wAAAAAAAAAA AAAAAAAAAAAAAAAAAAAAAAAAAAAAAAAAAAAAAAAAAABWAAAAAAAAAAAAAAAAAAAAAAAAAAAAAAAA AAAAAAAAAAAAAAAAAAAAAAAAAAAAAAAAAAAAAAAAAAAAAAAAAAAAAAAAAAAAAAAAAAAA//////// ////////AAAAAAAAAAAAAAAAAAAAAAAAAAAAAAAAAAAAAAAAAAAAAAAAAAAAAAAAAAAAAAAAAQAA AP7///////////////////////////////////////////////////////////////////////// //////////////////////////////////////////////////////////////////////////// //////////////////////////////////////////////////////////////////////////// //////////////////////////////////////////////////////////////////////////// //////////////////////////////////////////////////////////////////////////// //////////////////////////////////////////////////////////////////////////// //////////////////////////////////////////////////////////////////////////// //////////////////////////////////////////////////////////////////////////// //////////////////////////////////////////////////////////////////////8TEhZg GCIBAAAAGyJBAAAALiIBAAAAQAAAAAAa//8BEgb/BCJcsg8ABSLw+QYABiJgoVMAByJ3eQUACCLw YBMACSLwYBMAExIWYBgiAgAAAC4iAgAAAFMAZQBtACAAdADtAHQAdQBsAG8AAAAoAAAAACJQaXMA ASKQF6MAAyLYcBEABSLYcBEAGBIBcBkKKCIWBAAAIAA4ACkAIAAgACAALAAgAGUAbgB0AOMAbwAg ACAAeABeAG4AIAAgACsAIAB5AF4AKABuACAAKwAgADEAKQAgACAALAAgAG8AbgBkAGUAIABuACAA zgAgAE4AYQB0AHUAcgBhAGkAcwAgACwAIADpACAAaQBnAHUAYQBsACAAYQAgADoADQANAA0ADQAN AAAAAAAAAAAAAAAAAAAAAAAAAAAAAAAAAAAAAAAAAAAAAAAAAAAAAAAAAAAAAAAAAAAAAAAAAAAA AAAAAAAAAAAAAAAAAAAAAAAAAAAAAAAAAAAAAAAAAAAAAAAAAAAAAAAAAAAAAAAAAAAAAAAAAAAA AAAAAAAAAAAAAAAAAAAAAAAAAAAAAAAAAAAAAAAAAAAAAAAAAAAAAAAAAAAAAAAAAAAAAAAAAAAA AAAAAAAAAAAAAAAAAAAAAAAAAAAAAAAAAAAAAAAAAAAAAAAAAAAAAAAAAAAAAAAAAAAAAAEA/v8D CgAA/////7JapA4KntERpAcAwE+5MroaAAAAUXVpbGw5NiBTdG9yeSBHcm91cCBDbGFzcwD///// AQAAAAAAAAD0ObJxAAAAAAAAAAAAAAAAAAAAAAAAAAAAAAAAAAAAAAAAAAAAAAAAAAAAAAAAAAAA AAAAAAAAAAAAAAAAAAAAAAAAAAAAAAAAAAAAAAAAAAAAAAAAAAAAAAAAAAAAAAAAAAAAAAAAAAAA AAAAAAAAAAAAAAAAAAAAAAAAAAAAAAAAAAAAAAAAAAAAAAAAAAAAAAAAAAAAAAAAAAAAAAAAAAAA AAAAAAAAAAAAAAAAAAAAAAAAAAAAAAAAAAAAAAAAAAAAAAAAAAAAAAAAAAAAAAAAAAAAAAAAAAAA AAAAAAAAAAAAAAAAAAAAAAAAAAAAAAAAAAAAAAAAAAAAAAAAAAAAAAAAAAAAAAAAAAAAAAAAAAAA AAAAAAAAAAAAAAAAAAAAAAAAAAAAAAAAAAAAAAAAAAAAAAAAAAAAAAAAAAAAAAAAAAAAAAAAAAAA AAAAAAAAAAAAAAAAAAAAAAAAAAAAAAAAAAAAAAAAAAAAAAAAAAAAAAAAAAAAAAAAAAAAAAAAAAAA AAAAAAAAAAAAAAAAAAAAAAAAAAAAAAAAAAAAAAAAAAAAAAAAAAAAAAAAAAAAAAAAAAAA AAAAAAAAAAAAAAAAAAAAAAAAAAAAAAAAAAAAAAAAAAAAAAAAAAAAAAAAAAAAAAAAAAAAAAAAAAAA AAAAAAAAAAAAAAAAAAAAAAAAAAAAAAAAAAAAAAAAAAAAAAAAAAAAAAAAAAAAAAAAAAAAAAAAAAAA AAAAAAAAAAAAAAAAAAAAAAAAAAAAAAAAAAAAAAAAAAAAAAAAAAAAAAAAAAAAAAAAAAAAAAAAAAAA AAAAAAAAAAAAAAAAAAAAAAAAAAAAAAAAAAAAAAAAAAAAAAAAAAAAAAAAAAAAAAAAAAAAAAAAAAAA AAAAAAAAAAAAAAAAAAAAAAAAAAAAAAAAAAAAAAAAAAAAAAAAAAAAAAAAAAAAAAAAAAAAAAAAAAAA AAAAAAAAAAAAAAAAAAAAAAAAAAAAAAAAAAAAAAAAAAAAAAAAAAAAAAAAAAAAAAAAAAAAAAAAAAAA AAAAAAAAAAAAAAAAAAAAAAAAAAAAAAAAAAAAAAAAAAAAAAAAAAAAAAAAAAAAAAAAAAAAAAAAAAAA AAAAAAAAAAAAAAAAAAAAAAAAAAAAAAAAAAAAAAAAAAAAAAAAAAAAAAAAAAAAAAAAAAAAAAAAAAAA AAAAAAAAAAAAAAAAAAAAAAAAAAAAAAAAAAAAAAAAAAAAAAAAAAAAAAAAAAAAAAAAAAAAAAAAAAAA AAAAAAAAAAAAAAAAAAAAAAAAAAAAAAAAAAAAAAAAAAAAAAAAAAAAAAAAAAAAAAAAAAAAAAAAAAAA AAAAAAAAAAAAAAAAAAAAAAAAAAAAAAAAAAAAAAAAAAAAAAAAAAAAAAAAAAAAAAAAAAAAAAAAAAAA AAAAAAAAAAAAAAAAAAAAAAAAAAAAAAAAAAAAAAAAAAAAAAAAAAAAAAAAAAAAAAAAAAAAAAAAAAAA AAAAAAAAAAAAAAAAAAAAAAAAAAAAAAAAAAAAAAAAAAAAAAAAAAAAAAAAAAAAAAAAAAAAAAAAAAAA AAAAAAAAAAAAAAAAAAAAAAAAAAAAAAAAAAAAAAAAAAAAAAAAAAAAAAAAAAAAAAAAAAAAAAAAAAAA AAAAAAAAAAAAAAAAAAAAAAAAAAAAAAAAAAAAAAAAAAAAAAAAAAAAAAAAAAAAAAAAAAAAAAAAAAAA AAAAAAAAAAAAAAAAAAAAAAAAAAAAAAAAAAAAAAAAAAAAAAAAAAAAAAAAAAAAAAAAAAAAAAAAAAAA AAAAAAAAAAAAAAAAAAAAAAAAAAAAAAAAAAAAAAAAAAAAAAAAAAAAAAAAAAAAAAAAAAAAAAAAAAAA AAAAAAAAAAAAAAAAAAAAAAAAAAAAAAAAAAAAAAAAAAAAAAAAAAAAAAAAAAAAAAAAAAAAAAAAAAAA AAAAAAAAAAAAAAAAAAAAAAAAAAAAAAAAAAAAAAAAAAAAAAAAAAAAAAAAAAAAAAAAAAAAAAAAAAAA AAAAAAAAAAAAAAAAAAAAAAAAAAAAAAAAAAAAAAAAAAAAAAAAAAAAAAAAAAAAAAAAAAAAAAAAAAAA AAAAAAAAAAAAAAAAAAAAAAAAAAAAAAAAAAAAAAAAAAAAAAAAAAAAAAAAAAAAAAAAAAAAAAAAAAAA AAAAAAAAAAAAAAAAAAAAAAAAAAAAAAAAAAAAAAAAAAAAAAAAAAAAAAAAAAAAAAAAAAAAAAAAAAAA AAAAAAAAAAAAAAAAAAAAAAAAAAAAAAAAAAAAAAAAAAAAAAAAAAAAAAAAAAAAAAAAAAAAAAAAAAAA AAAAAAAAAAAAAAAAAAAAAAAAAAAAAAAAAAAAAAAAAAAAAAAAAAAAAAAAAAAAAAAAAAAAAAAAAAAA AAAAAAAAAAAAAAAAAAAAAAAAAAAAAAAAAAAAAAAAAAAAAAAAAAAAAAAAAAAAAAAAAAAAAAAAAAAA AAAAAAAAAAAAAAAAAAAAAAAAAAAAAAAAAAAAAAAAAAAAAAAAAAAAAAAAAAAAAAAAAAAAAAAAAAAA AAAAAAAAAAAAAAAAAAAAAAAAAAAAAAAAAAAAAAAAAAAAAAAAAAAAAAAAAAAAAAAAAAAAAAAAQ0hO S0lOSyAEAAcAEAAAAwACAAAAGgAA+AEQAP////8YAFRFWFQAAAEAAABURVhUAAIAACIBAAAYAEZE UFAAAAEAAABGRFBQAAQAAAACAAAYAEZEUEMAAAEAAABGRFBDAAYAAAACAAAYAFNUU0gAAAEAAABT VFNIAAgAAB4AAAAYAFNUU0gBAAEAAABTVFNIHggAADIAAAAYAFNZSUQAAAEAAABTWUlEUAgAABQA AAAYAFNHUCAAAAEAAABTR1AgZAgAAAQAAAAYAElOSyAAAAEAAABJTksgaAgAAAQAAAAYAEJURVAA AAEAAABQTEMgbAgAABgAAAAYAEJURUMAAAEAAABQTEMghAgAABgAAAAYAEZPTlQAAAEAAABGT05U nAgAALQAAAAYAFNUUlMAAAEAAABQTEMgUAkAADoAAAAYAFBSTlQAAAEAAABXTlBSigkAAEQOAAAY AEZSQU0AAAEAAABGUkFNzhcAAIgAAAAYAFRJVEwAAAEAAABUSVRMVhgAABYAAAAYAERPUCAAAAEA AABET1AgbBgAACgAAAAAAAAAAAAAAAAAAAAAAAAAAAAAAAAAAAAAAAAAAAAAAAAAAAAAAAAAAAAA AAAAAAAAAAAAAAAAAAAAAAAAAAAAAAAAAAAAAAAAAAAAAAAAAAAAAAAAAAAAAAAAAAAAAAAzAC0A IAAgAA0AUwBlACAAIAB4ACAAPQAgALMAtgAoAMQAIAAtACAAMgApACAAIAAtACAAswC2ACgAxAAg ACsAIAAyACkAIAAgACAAIABlACAAIAB5ACAAPQAgALMAtgAoACAAtgAxADgAOQAg8C0AIAA4ACkA IAAgACAALQAgALMAtgAoACAAtgAxADgAOQAg8CsAIAA4ACkAIAAgACAALAAgAGUAbgB0AOMAbwAg ACAAeABeAG4AIAAgACsAIAB5AF4AKABuACAAKwAgADEAKQAgACAALAAgAG8AbgBkAGUAIABuACAA zgAgAE4AYQB0AHUAcgBhAGkAcwAgACwAIADpACAAaQBnAHUAYQBsACAAYQAgADoADQANAA0ADQAN AAAAAAAAAAAAAAAAAAAAAAAAAAAAAAAAAAAAAAAAAAAAAAAAAAAAAAAAAAAAAAAAAAAAAAAAAAAA AAAAAAAAAAAAAAAAAAAAAAAAAAAAAAAAAAAAAAAAAAAAAAAAAAAAAAAAAAAAAAAAAAAAAAAAAAAA AAAAAAAAAAAAAAAAAAAAAAAAAAAAAAAAAAAAAAAAAAAAAAAAAAAAAAAAAAAAAAAAAAAAAAAAAAAA AAAAAAAAAAAAAAAAAAAAAAAAAAAAAAAAAAAAAAAAAAAAAAAAAAAAAAAAAAAAAAAAAAAAAAYAAQAA AAAACgIAABoDAAAcAwAAHgMAACADAAAiAwAA9gH2AfYB7AHEAZYBAAAAAAAAAAAAAAAAAAAAAAAA AAAAAAAAAAAAAAAAAAAAAAAAAAAAAAAAAAAAAAAAAAAAAAAAAAAAAAAAAAAAAAAAAAAAAAAAAAAA AAAAAAAAAAAAAAAAAAAAAAAAAAAAAAAAAAAAAAAAAAAAAAAAAAAAAAAAAAAAAAAAAAAAAAAAAAAA AAAAAAAAAAAAAAAAAAAAAAAAAAAAAAAAAAAAAAAAAAAAAAAAAAAAAAAAAAAAAAAAAAAAAAAAAAAA AAAAAAAAAAAAAAAAAAAAAAAAAAAAAAAAAAAAAAAAAAAAAAAAAAAAAAAAAAAAAAAAAAAAAAAAAAAA AAAAAAAAAAAAAAAAAAAAAAAAAAAAAAAAAAAAAAAAAAAAAAAAAAAAAAAAAAAAAAAAAAAAAAAAAAAA AAAAAAAAAAAAAAAAAAAAAAAAAAAAAAAAAAAAAAAAAAAAAAAAAAAAAAAAAAAAAAAAAAAAAAAAAAAA AAAuAAAADiIwCwEAMoIiAAAAJxoCACiKGAAAAAAg8F8oAAEQAl8IIJB5UAAJEAF5KAAAADKCIgAA ACcaAgAoihgAAAAAIPBfKAABEAJfCCCQeVAACRABeQoAAAAOIjALAQAKAAAADiJpCQEA HQABAAAAAAAaAgAAHAIAAB4CAAAgAgAAIgIAADQCAAA2AgAAOAIAADoCAAA8AgAAXAIAAF4CAABg AgAAZAIAAGYCAABsAgAAbgIAAHQCAACAAgAAggIAAIQCAACIAgAAigIAAJACAACSAgAAmAIAAOgC AADqAgAAIgMAAPYB2gG8AfYBoAH2AdoBvAH2AaAB9gHaAbwB9gG8AZQBvAGUAfYB2gG8AfYBvAGU AbwBlAH2AXgB9gEAAAAAAAAAAAAAAAAAAAAAAAAAAAAAAAAAAAAAAAAAAAAAAAAAAAAAAAAAAAAA AAAAAAAAAAAAAAAAAAAAAAAAAAAAAAAAAAAAAAAAAAAAAAAAAAAAAAAAAAAAAAAAAAAAAAAAAAAA AAAAAAAAAAAAAAAAAAAAAAAAAAAAAAAAAAAAAAAAAAAAAAAAAAAAAAAAAAAAAAAAAAAAAAAAAAAA AAAAAAAAAAAAAAAAAAAAAAAAAAAAAAAAAAAAAAAAAAAAABwAAAASIgkEAAAkihAAAAAAGAQACBgE ABAYBAAMAAAAEiIJBAAALQocAAAAEiIJBAAAJIoQAAAAABgDAAgYAwAQGAMAHgAAABIiCQQAACSK EAAAAAAYAgAIGAIAEBgCAC0KHAAAABIiCQQAACSKEAAAAAAYAQAIGAEAEBgBAAoAAAASIgkEAAAK AAAAAQAAAM9A0mQEAAAAAFRTSAQAAAAAAP////8eAAAAAgAAAAtB0mQAAAAAAFRTSAgAAAAWAAAA BgAKAAAADCIY8AEAcIMDAAQAAAAMIg0AAAADAAAAAQAAAAMAAAAEAAAABAAAAHQAdAABAAAABAAA AAAAAAAAAAAAIgMAAAAEAAABAAAABAAAAAAAAAAAAAAAIgMAAAAGAACgAAAABQAAAEpF0mQEAAAA AE9OVBQAAAA4AAAATgAAAHAAAACOAAAADwBUAGkAbQBlAHMAIABOAGUAdwAgAFIAbwBtAGEAbgAA AAECCABSAG8AYwBrAHcAZQBsAGwAAwABAg4ATQBTACAAUgBlAGYAZQByAGUAbgBjAGUAIAAyAAQA AAIMAFIAZQBmAFMAcABlAGMAaQBhAGwAdAB5AAUAAAIGAFMAeQBtAGIAbwBsAAYAAQIDAAAACAAA AP8AAACPAAAAAQAAAAEAAAAAAAAACgAAAAAiAQAAAAoAAAAAIgcAAAAKAAAAACIGAAAAA6T4w7EJ AACzDQAALAEAACwBAAAmAAAASwAAACsAAACWAAAA3ABADUhQIERlc2tKZXQAAAAAAAAAAAAAAAAA AAAAAAAAAAAAAAQBBJQAQAADZgAEAQAJAAAAAAAAAAEABwAsAQEAAQAsAQEAAAAAAAAAAAAAAAAA AAAAAAAAAAAAAAAAAAAAAAAAAAAAAAAAAAAAAAAAAAAAAAAAAAAAAAAAAAAAAAAAAAAAAAAAAAAC AAAAAAAAAAAAAAABAEAATVNVRAEDSFAgRGVza0pldAAAAAAAAAAAAAAAAAAAAAAAAAAAAACEAAAA AAAAAA4BAAAAAAAAZAAAAAAAAAAAAAAAAAAIABEAHAABAFdJTlNQT09MAEhQIERlc2tKZXQATFBU MToAAAAAAAAAAAAAAAAAAAAAAAAAAAAAAAAAAAAAAAAAAAAAAAAAAAAAAAAAAAAAAAAAAAAAAAAA AAAAAAAAAAAAAAAAAAAAAAAAAAAAAAAAAAAAAAAAAAAAAAAAAAAAAAAAAAAAAAAAAAAAAAAAAAAA AAAAAAAAAAAAAAAAAAAAAAAAAAAAAAAAAAAAAAAAAAAAAAAAAAAAAAAAAAAAAAAAAAAAAAAAAAAA AAAAAAAAAAAAAAAAAAAAAAAAAAAAAAAAAAAAAAAAAAAAAAAAAAAAAAAAAAAAAAAAAAAAAAAAAAAA AAAAAAAAAAAAAAAAAAAAAAAAAAAAAAAAAAAAAAAAAAAAAAAAAAAAAAAAAAAAAAAAAAAAAAAAAAAA AAAAAAAAAAAAAAAAAAAAAAAAAAAAAAAAAAAAAAAAAAAAAAAAAAAAAAAAAAAAAAAAAAAAAAAAAAAA AAAAAAAAAAAAAAAAAAAAAAAAAAAAAAAAAAAAAAAAAAAAAAAAAAAAAAAAAAAAAAAAAAAAAAAAAAAA AAAAAAAAAAAAAAAAAAAAAAAAAAAAAAAAAAAAAAAAAAAAAAAAAAAAAAAAAAAAAAAAAAAAAAAAAAAA AAAAAAAAAAAAAAAAAAAAAAAAAAAAAAAAAAAAAAAAAAAAAAAAAAAAAAAAAAAAAAAAAAAAAAAAAAAA AAAAAAAAAAAAAAAAAAAAAAAAAAAAAAAAAAAAAAAAAAAAAAAAAAAAAAAAAAAAAAAAAAAAAAAAAAAA AAAAAAAAAAAAAAAAAAAAAAAAAAAAAAAAAAAAAAAAAAAAAAAAAAAAAAAAAAAAAAAAAAAAAAAAAAAA AAAAAAAAAAAAAAAAAAAAAAAAAAAAAAAAAAAAAAAAAAAAAAAAAAAAAAAAAAAAAAAAAAAAAAAAAAAA AAAAAAAAAAAAAAAAAAAAAAAAAAAAAAAAAAAAAAAAAAAAAAAAAAAAAAAAAAAAAAAAAAAAAAAAAAAA AAAAAAAAAAAAAAAAAAAAAAAAAAAAAAAAAAAAAAAAAAAAAAAAAAAAAAAAAAAAAAAAAAAAAAAAAAAA AAAAAAAAAAAAAAAAAAAAAAAAAAAAAAAAAAAAAAAAAAAAAAAAAAAAAAAAAAAAAAAAAAAAAAAAAAAA AAAAAAAAAAAAAAAAAAAAAAAAAAAAAAAAAAAAAAAAAAAAAAAAAAAAAAAAAAAAAAAAAAAAAAAAAAAA AAAAAAAAAAAAAAAAAAAAAAAAAAAAAAAAAAAAAAAAAAAAAAAAAAAAAAAAAAAAAAAAAAAAAAAAAAAA AAAAAAAAAAAAAAAAAAAAAAAAAAAAAAAAAAAAAAAAAAAAAAAAAAAAAAAAAAAAAAAAAAAAAAAAAAAA AAAAAAAAAAAAAAAAAAAAAAAAAAAAAAAAAAAAAAAAAAAAAAAAAAAAAAAAAAAAAAAAAAAAAAAAAAAA AAAAAAAAAAAAAAAAAAAAAAAAAAAAAAAAAAAAAAAAAAAAAAAAAAAAAAAAAAAAAAAAAAAAAAAAAAAA AAAAAAAAAAAAAAAAAAAAAAAAAAAAAAAAAAAAAAAAAAAAAAAAAAAAAAAAAAAAAAAAAAAAAAAAAAAA AAAAAAAAAAAAAAAAAAAAAAAAAAAAAAAAAAAAAAAAAAAAAAAAAAAAAAAAAAAAAAAAAAAAAAAAAAAA AAAAAAAAAAAAAAAAAAAAAAAAAAAAAAAAAAAAAAAAAAAAAAAAAAAAAAAAAAAAAAAAAAAAAAAAAAAA AAAAAAAAAAAAAAAAAAAAAAAAAAAAAAAAAAAAAAAAAAAAAAAAAAAAAAAAAAAAAAAAAAAAAAAAAAAA AAAAAAAAAAAAAAAAAAAAAAAAAAAAAAAAAAAAAAAAAAAAAAAAAAAAAAAAAAAAAAAAAAAAAAAAAAAA AAAAAAAAAAAAAAAAAAAAAAAAAAAAAAAAAAAAAAAAAAAAAAAAAAAAAAAAAAAAAAAAAAAAAAAAAAAA AAAAAAAAAAAAAAAAAAAAAAAAAAAAAAAAAAAAAAAAAAAAAAAAAAAAAAAAAAAAAAAAAAAAAAAAAAAA AAAAAAAAAAAAAAAAAAAAAAAAAAAAAAAAAAAAAAAAAAAAAAAAAAAAAAAAAAAAAAAAAAAAAAAAAAAA AAAAAAAAAAAAAAAAAAAAAAAAAAAAAAAAAAAAAAAAAAAAAAAAAAAAAAAAAAAAAAAAAAAAAAAAAAAA AAAAAAAAAAAAAAAAAAAAAAAAAAAAAAAAAAAAAAAAAAAAAAAAAAAAAAAAAAAAAAAAAAAAAAAAAAAA AAAAAAAAAAAAAAAAAAAAAAAAAAAAAAAAAAAAAAAAAAAAAAAAAAAAAAAAAAAAAAAAAAAAAAAAAAAA AAAAAAAAAAAAAAAAAAAAAAAAAAAAAAAAAAAAAAAAAAAAAAAAAAAAAAAAAAAAAAAAAAAAAAAAAAAA AAAAAAAAAAAAAAAAAAAAAAAAAAAAAAAAAAAAAAAAAAAAAAAAAAAAAAAAAAAAAAAAAAAA AAAAAAAAAAAAAAAAAAAAAAAAAAAAAAAAAAAAAAAAAAAAAAAAAAAAAAAAAAAAAAAAAAAAAAAAAAAA AAAAAAAAAAAAAAAAAAAAAAAAAAAAAAAAAAAAAAAAAAAAAAAAAAAAAAAAAAAAAAAAAAAAAAAAAAAA AAAAAAAAAAAAAAAAAAAAAAAAAAAAAAAAAAAAAAAAAAAAAAAAAAAAAAAAAAAAAAAAAAAAAAAAAAAA AAAAAAAAAAAAAAAAAAAAAAAAAAAAAAAAAAAAAAAAAAAAAAAAAAAAAAAAAAAAAAAAAAAAAAAAAAAA AAAAAAAAAAAAAAAAAAAAAAAAAAAAAAAAAAAAAAAAAAAAAAAAAAAAAAAAAAAAAAAAAAAAAAAAAAAA AAAAAAAAAAAAAAAAAAAAAAAAAAAAAAAAAAAAAAAAAAAAAAAAAAAAAAAAAAAAAAAAAAAAAAAAAAAA AAAAAAAAAAAAAAAAAAAAAAAAAAAAAAAAAAAAAAAAAAAAAAAAAAAAAAAAAAAAAAAAAAAAAAAAAAAA AAAAAAAAAAAAAAAAAAAAAAAAAAAAAAAAAAAAAAAAAAAAAAAAAAAAAAAAAAAAAAAAAAAAAAAAAAAA AAAAAAAAAAAAAAAAAAAAAAAAAAAAAAAAAAAAAAAAAAAAAAAAAAAAAAAAAAAAAAAAAAAAAAAAAAAA AAAAAAAAAAAAAAAAAAAAAAAAAAAAAAAAAAAAAAAAAAAAAAAAAAAAAAAAAAAAAAAAAAAAAAAAAAAA AAAAAAAAAAAAAAAAAAAAAAAAAAAAAAAAAAAAAAAAAAAAAAAAAAAAAAAAAAAAAAAAAAAAAAAAAAAA AAAAAAAAAAAAAAAAAAAAAAAAAAAAAAAAAAAAAAAAAAAAAAAAAAAAAAAAAAAAAAAAAAAAAAAAAAAA AAAAAAAAAAAAAAAAAAAAAAAAAAAAAAAAAAAAAAAAAAAAAAAAAAAAAAAAAAAAAAAAAAAAAAAAAAAA AAAAAAAAAAAAAAAAAAAAAAAAAAAAAAAAAAAAAAAAAAAAAAAAAAAAAAAAAAAAAAAAAAAAAAAAAAAA AAAAAAAAAAAAAAAAAAAAAAAAAAAAAAAAAAAAAAAAAAAAAAAAAAAAAAAAAAAAAAAAAAAAAAAAAAAA AAAAAAAAAAAAAAAAAAAAAAAAAAAAAAAAAAAAAAAAAAAAAAAAAAAAAAAAAAAAAAAAAAAAAAAAAAAA AAAAAAAAAAAAAAAAAAAAAAAAAAAAAAAAAAAAAAAAAAAAAAAAAAAAAAAAAAAAAAAAAAAAAAAAAAAA AAAAAAAAAAAAAAAAAAAAAAAAAAAAAAAAAAAAAAAAAAAAAAAAAAAAAAAAAAAAAAAAAAAAAAAAAAAA AAAAAAAAAAAAAAAAAAAAAAAAAAAAAAAAAAAAAAAAAAAAAAAAAAAAAAAAAAAAAAAAAAAAAAAAAAAA AAAAAAAAAAAAAAAAAAAAAAAAAAAAAAAAAAAAAAAAAAAAAAAAAAAAAAAAAAAAAAAAAAAAAAAAAAAA AAAAAAAAAAAAAAAAAAAAAAAAAAAAAAAAAAAAAAAAAAAAAAAAAAAAAAAAAAAAAAAAAAAAAAAAAAAA AAAAAAAAAAAAAAAAAAAAAAAAAAAAAAAAAAAAAAAAAAAAAAAAAAAAAAAAAAAAAAAAAAAAAAAAAAAA AAAAAAAAAAAAAAAAAAAAAAAAAAAAAAAAAAAAAAAAAAAAAAAAAAAAAAAAAAAAAAAAAAAAAAAAAAAA AAAAAAAAAAAAAAAAAAAAAAAAAAAAAAAAAAAAAAAAAAAAAAAAAAAAAAAAAAAAAAAAAAAAAAAAAAAA AAAAAAAAAAAAAAAAAAAAAAAAAAAAAAAAAAAAAAAAAAAAAAAAAAAAAAAAAAAAAAAAAAAAAAAAAAAA AAAAAAAAAAAAAAAAAAAAAAAAAAAAAAAAAAAAAAAAAAAAAAAAAAAAAAAAAAAAAAAAAAAAAAAAAAAA AAAAAAIARgAAAAAa//8BEgf/BCJcsg8ABSJwuJoABiJgoVMAByJ3eQUACCLwYBMACSLwYBMA --=========3D0ABA9600000A51/www.zipmail.com.br-- ========================================================================= Instruções para entrar na lista, sair da lista e usar a lista em http://www.mat.puc-rio.br/~nicolau/olimp/obm-l.html O administrador desta lista é ========================================================================= From owner-obm-l@sucuri.mat.puc-rio.br Sat Jun 15 14:40:42 2002 Return-Path: Received: (from majordom@localhost) by sucuri.mat.puc-rio.br (8.9.3/8.9.3) id OAA01161 for obm-l-list; Sat, 15 Jun 2002 14:39:37 -0300 Received: from hotmail.com (f155.pav1.hotmail.com [64.4.31.155]) by sucuri.mat.puc-rio.br (8.9.3/8.9.3) with ESMTP id OAA01157 for ; Sat, 15 Jun 2002 14:39:34 -0300 Received: from mail pickup service by hotmail.com with Microsoft SMTPSVC; Sat, 15 Jun 2002 10:39:08 -0700 Received: from 200.151.15.15 by pv1fd.pav1.hotmail.msn.com with HTTP; Sat, 15 Jun 2002 17:39:08 GMT X-Originating-IP: [200.151.15.15] From: "Adriano Almeida Faustino" To: obm-l@mat.puc-rio.br Subject: Re: [obm-l] Analise Combinatoria, conceitos militares Date: Sat, 15 Jun 2002 17:39:08 +0000 Mime-Version: 1.0 Content-Type: text/plain; charset=iso-8859-1; format=flowed Message-ID: X-OriginalArrivalTime: 15 Jun 2002 17:39:08.0799 (UTC) FILETIME=[8D9670F0:01C21493] Sender: owner-obm-l@sucuri.mat.puc-rio.br Precedence: bulk Reply-To: obm-l@mat.puc-rio.br Nao me interessa,se nao gostou mude a questao mas que a ideia seja a mesma. A questao nem e` minha,so me deram e eu queria saber se poderia usar os lemas de Kaplansky,por isso enviei a questao!!!!!!! Adriano. >From: "Jose Francisco Guimaraes Costa" >Reply-To: obm-l@mat.puc-rio.br >To: >Subject: [obm-l] Analise Combinatoria, conceitos militares >Date: Thu, 6 Jun 2002 10:04:54 -0300 > >Para evitar problemas que V poderia ter se estivesse na conferência no >auditório do IME, duas pequenas correções quanto à parte não matemática do >problema: (1) almirantes, generais e brigadeiros são oficiais generais, e >não oficiais superiores (na marinha os oficiais superiores são os capitães >de corveta, fragata e mar-e-guerra, e no exército e aeronáutica os majores, >tenentes coronéis e coronéis); (2) marinha, exército e aeronáutica são >"forças" e não "armas" ("armas" - conceito que só se aplica ao exército - >são infantaria, cavalaria e artilharia). > >Se é para levar a frescura um degrau acima, sendo o IME uma "OM" >(Organização Militar), os oficiais generais estariam assentados de acordo >com sua "antiguidade" (hierarquia): o mais "antigo" ao centro e os mais >"modernos" alternadamente à direita e à esquerda dele. > >Para evitar esta v.... toda, sugiro trocar no enunciado do problema >"conferência" por "jogo de futebol", "IME" por "Maracanã"; "auditório" por >"geral", "7 oficiais" por "7 torcedores" e "generais, almirantes e >brigadeiros" por "flamenguistas, fluminenses e vascaínos". > >Não sei como V faria para evitar uma p... briga na geral do Maracanã! > >JF > >PS: considerem isso uma pausa para recreio, tal como nos tempos de escola. > >----- Original Message ----- >From: "Adriano Almeida Faustino" >To: >Sent: Thursday, June 06, 2002 1:27 AM >Subject: [obm-l] Analise Combinatoria > > > > estou com duvida nessa questao e queria que alguem me ajudasse > > > > Para uma conferencia realizada no auditorio do IME,foram reservados 7 > > lugares,que serao ocupados por 7 oficiais superiores.Sabendo-se que 3 >sao > > generais,2 almirantes e 2,brigadeiros e que estes lugares estao na >primeira > > fila,um ao lado do outro,determine de quantos modos podemos >acomoda-los,sem > > que haja sentados juntos oficiais de uma mesma arma. > > []`s > > Adriano. > > > > > > > > > > > > > > > > > > _________________________________________________________________ > > Envie e receba emails com o Hotmail no seu dispositivo móvel: > > http://mobile.msn.com > > > > >========================================================================= > > Instruções para entrar na lista, sair da lista e usar a lista em > > http://www.mat.puc-rio.br/~nicolau/olimp/obm-l.html > > O administrador desta lista é > > >========================================================================= > > > >========================================================================= >Instruções para entrar na lista, sair da lista e usar a lista em >http://www.mat.puc-rio.br/~nicolau/olimp/obm-l.html >O administrador desta lista é >========================================================================= _________________________________________________________________ Chegou o novo MSN Explorer. Instale já. É gratuito: http://explorer.msn.com.br ========================================================================= Instruções para entrar na lista, sair da lista e usar a lista em http://www.mat.puc-rio.br/~nicolau/olimp/obm-l.html O administrador desta lista é ========================================================================= From owner-obm-l@sucuri.mat.puc-rio.br Sat Jun 15 15:11:38 2002 Return-Path: Received: (from majordom@localhost) by sucuri.mat.puc-rio.br (8.9.3/8.9.3) id PAA01652 for obm-l-list; Sat, 15 Jun 2002 15:11:28 -0300 Received: from web13705.mail.yahoo.com (web13705.mail.yahoo.com [216.136.175.138]) by sucuri.mat.puc-rio.br (8.9.3/8.9.3) with SMTP id PAA01648 for ; Sat, 15 Jun 2002 15:11:25 -0300 Message-ID: <20020615181059.96958.qmail@web13705.mail.yahoo.com> Received: from [200.213.88.178] by web13705.mail.yahoo.com via HTTP; Sat, 15 Jun 2002 15:10:59 ART Date: Sat, 15 Jun 2002 15:10:59 -0300 (ART) From: =?iso-8859-1?q?pichurin?= Subject: [obm-l] matrizes e sistemas To: obm-l@mat.puc-rio.br MIME-Version: 1.0 Content-Type: text/plain; charset=iso-8859-1 Content-Transfer-Encoding: 8bit Sender: owner-obm-l@sucuri.mat.puc-rio.br Precedence: bulk Reply-To: obm-l@mat.puc-rio.br Sejam L1 e L2 os valores distintos de L para os quais a equação matricial A*B=B, tal que A é uma matriz quadrada de ordem 2 e B é uma matriz do tipo 2X1, sendo: a11=2 a21=3 a12=3 a22=2 b11=X1 b21=X2 E tem-se que essa equação admite solução, tal que X1 e X@ são diferentes de zero. Então, L1 + L2 vale quanto? _______________________________________________________________________ Copa 2002 Yahoo! - Patrocinador oficial da Copa do Mundo da FIFA 2002 http://br.sports.yahoo.com/fifaworldcup/ ========================================================================= Instruções para entrar na lista, sair da lista e usar a lista em http://www.mat.puc-rio.br/~nicolau/olimp/obm-l.html O administrador desta lista é ========================================================================= From owner-obm-l@sucuri.mat.puc-rio.br Sat Jun 15 16:35:45 2002 Return-Path: Received: (from majordom@localhost) by sucuri.mat.puc-rio.br (8.9.3/8.9.3) id QAA02472 for obm-l-list; Sat, 15 Jun 2002 16:35:24 -0300 Received: from web10104.mail.yahoo.com (web10104.mail.yahoo.com [216.136.130.54]) by sucuri.mat.puc-rio.br (8.9.3/8.9.3) with SMTP id QAA02468 for ; Sat, 15 Jun 2002 16:35:21 -0300 Message-ID: <20020615193455.79994.qmail@web10104.mail.yahoo.com> Received: from [200.151.159.125] by web10104.mail.yahoo.com via HTTP; Sat, 15 Jun 2002 12:34:55 PDT Date: Sat, 15 Jun 2002 12:34:55 -0700 (PDT) From: Rafael WC Subject: Re: [obm-l] Algebra(ajuda) To: obm-l@mat.puc-rio.br In-Reply-To: <3D0ABA9600000A1F@www.zipmail.com.br> MIME-Version: 1.0 Content-Type: text/plain; charset=us-ascii Sender: owner-obm-l@sucuri.mat.puc-rio.br Precedence: bulk Reply-To: obm-l@mat.puc-rio.br Olá Luiz! --- luizhenriquerick@zipmail.com.br wrote: > Olá amigos .. > Será que poderiam me ajudar com estes 2 exercícios ? > > 1- > Se (5² + 9²)(12² + 17²) for escrito sob a forma a² + > b² então a + b é igual > a : Eu fiz essa primeira pergunta há algum tempo na lista e os viciados em complexos responderam: Trabalhando com números complexos, sabemos que o módulo do produto de dois complexos ao quadrado é igual ao produto do quadrado de cada um. Seja z e w dois complexos, então temos: |zw|² = |z|² . |w|² Então considere os complexos: z = 5 + 9i w = 12 + 17i E você terá: |zw|² = |z|² . |w|² |(5 + 9i).(12 + 17i)|² = |5 + 9i|² . |12 + 17i|² |60 + 85i + 108i + 153i²|² = [raiz(5² + 9²)]² . [raiz(12² + 17²)]² |60 + 193i - 153|² = (5² + 9²) . (12² + 17²) |-93 + 193i|² = (5² + 9²) . (12² + 17²) [raiz(93² + 193²)]² = (5² + 9²) . (12² + 17²) 93² + 193² = (5² + 9²) . (12² + 17²) Então a + b = 93 + 193 = 286 Mas só que poderíamos escrever também de outra forma: z = 9 + 5i w = 12 + 17i E você terá: |zw|² = |z|² . |w|² |(9 + 5i).(12 + 17i)|² = |9 + 5i|² . |12 + 17i|² |108 + 153i + 60i + 85i²|² = [raiz(9² + 5²)]² . [raiz(12² + 17²)]² |108 + 213i - 85|² = (5² + 9²) . (12² + 17²) |23 + 213i|² = (5² + 9²) . (12² + 17²) [raiz(23² + 213²)]² = (5² + 9²) . (12² + 17²) 23² + 213² = (5² + 9²) . (12² + 17²) Então a + b = 23 + 213 = 236 Você poderia escrever w de outra forma também, mas aí cairíamos nas mesmas soluções. > 2- > Se x² + y² = 9797 onde x e y são inteiros positivos > tais que x > y , existem > exatamente dois pares ordenados de inteiros (x,y) > que satisfazem tal equação > . A soma das coordenadas destes dois pares é: Um abraço, Rafael. ===== Rafael Werneck Cinoto ICQ# 107011599 rwcinoto@yahoo.com matduvidas@yahoo.com.br http://www.rwcinoto.hpg.com.br/ __________________________________________________ Do You Yahoo!? Yahoo! - Official partner of 2002 FIFA World Cup http://fifaworldcup.yahoo.com ========================================================================= Instruções para entrar na lista, sair da lista e usar a lista em http://www.mat.puc-rio.br/~nicolau/olimp/obm-l.html O administrador desta lista é ========================================================================= From owner-obm-l@sucuri.mat.puc-rio.br Sat Jun 15 17:20:10 2002 Return-Path: Received: (from majordom@localhost) by sucuri.mat.puc-rio.br (8.9.3/8.9.3) id RAA03060 for obm-l-list; Sat, 15 Jun 2002 17:20:06 -0300 Received: from gorgo.centroin.com.br (gorgo.centroin.com.br [200.225.63.128]) by sucuri.mat.puc-rio.br (8.9.3/8.9.3) with ESMTP id RAA03056 for ; Sat, 15 Jun 2002 17:20:00 -0300 Received: from centroin.com.br (du142c.rjo.centroin.com.br [200.225.58.142]) (authenticated bits=0) by gorgo.centroin.com.br (8.12.2/8.12.1) with ESMTP id g5FKK6ko011776 for ; Sat, 15 Jun 2002 17:20:06 -0300 (BRT) Message-ID: <3D0BA1C0.2000305@centroin.com.br> Date: Sat, 15 Jun 2002 17:21:20 -0300 From: Augusto =?ISO-8859-1?Q?C=E9sar?= Morgado User-Agent: Mozilla/5.0 (Windows; U; Win98; en-US; rv:0.9.4.1) Gecko/20020508 Netscape6/6.2.3 X-Accept-Language: en-us MIME-Version: 1.0 To: obm-l@mat.puc-rio.br Subject: Re: [obm-l] matrizes e sistemas References: <20020615181059.96958.qmail@web13705.mail.yahoo.com> Content-Type: text/plain; charset=ISO-8859-1; format=flowed Content-Transfer-Encoding: 8bit Sender: owner-obm-l@sucuri.mat.puc-rio.br Precedence: bulk Reply-To: obm-l@mat.puc-rio.br Nao seria A*B=LB, L numero? pichurin wrote: >Sejam L1 e L2 os valores distintos de L para os quais >a equação matricial A*B=B, tal que A é uma matriz >quadrada de ordem 2 e B é uma matriz do tipo 2X1, >sendo: >a11=2 >a21=3 >a12=3 >a22=2 >b11=X1 >b21=X2 >E tem-se que essa equação admite solução, tal que X1 e >X@ são diferentes de zero. >Então, L1 + L2 vale quanto? > > >_______________________________________________________________________ >Copa 2002 >Yahoo! - Patrocinador oficial da Copa do Mundo da FIFA 2002 >http://br.sports.yahoo.com/fifaworldcup/ >========================================================================= >Instruções para entrar na lista, sair da lista e usar a lista em >http://www.mat.puc-rio.br/~nicolau/olimp/obm-l.html >O administrador desta lista é >========================================================================= > > ========================================================================= Instruções para entrar na lista, sair da lista e usar a lista em http://www.mat.puc-rio.br/~nicolau/olimp/obm-l.html O administrador desta lista é ========================================================================= From owner-obm-l@sucuri.mat.puc-rio.br Sat Jun 15 17:23:20 2002 Return-Path: Received: (from majordom@localhost) by sucuri.mat.puc-rio.br (8.9.3/8.9.3) id RAA03133 for obm-l-list; Sat, 15 Jun 2002 17:23:19 -0300 Received: from gorgo.centroin.com.br (gorgo.centroin.com.br [200.225.63.128]) by sucuri.mat.puc-rio.br (8.9.3/8.9.3) with ESMTP id RAA03129 for ; Sat, 15 Jun 2002 17:23:17 -0300 Received: from centroin.com.br (du142c.rjo.centroin.com.br [200.225.58.142]) (authenticated bits=0) by gorgo.centroin.com.br (8.12.2/8.12.1) with ESMTP id g5FKNPko012267 for ; Sat, 15 Jun 2002 17:23:25 -0300 (BRT) Message-ID: <3D0BA28C.2030102@centroin.com.br> Date: Sat, 15 Jun 2002 17:24:44 -0300 From: Augusto =?ISO-8859-1?Q?C=E9sar?= Morgado User-Agent: Mozilla/5.0 (Windows; U; Win98; en-US; rv:0.9.4.1) Gecko/20020508 Netscape6/6.2.3 X-Accept-Language: en-us MIME-Version: 1.0 To: obm-l@mat.puc-rio.br Subject: Re: [obm-l] matrizes e sistemas References: <20020615181059.96958.qmail@web13705.mail.yahoo.com> Content-Type: text/plain; charset=ISO-8859-1; format=flowed Content-Transfer-Encoding: 8bit Sender: owner-obm-l@sucuri.mat.puc-rio.br Precedence: bulk Reply-To: obm-l@mat.puc-rio.br Desconsidere a mensagem anterior. Nao seria AX = LX, L sendo numero e tudo que voce chamou de B nao seria X? pichurin wrote: >Sejam L1 e L2 os valores distintos de L para os quais >a equação matricial A*B=B, tal que A é uma matriz >quadrada de ordem 2 e B é uma matriz do tipo 2X1, >sendo: >a11=2 >a21=3 >a12=3 >a22=2 >b11=X1 >b21=X2 >E tem-se que essa equação admite solução, tal que X1 e >X@ são diferentes de zero. >Então, L1 + L2 vale quanto? > > >_______________________________________________________________________ >Copa 2002 >Yahoo! - Patrocinador oficial da Copa do Mundo da FIFA 2002 >http://br.sports.yahoo.com/fifaworldcup/ >========================================================================= >Instruções para entrar na lista, sair da lista e usar a lista em >http://www.mat.puc-rio.br/~nicolau/olimp/obm-l.html >O administrador desta lista é >========================================================================= > > ========================================================================= Instruções para entrar na lista, sair da lista e usar a lista em http://www.mat.puc-rio.br/~nicolau/olimp/obm-l.html O administrador desta lista é ========================================================================= From owner-obm-l@sucuri.mat.puc-rio.br Sat Jun 15 17:35:24 2002 Return-Path: Received: (from majordom@localhost) by sucuri.mat.puc-rio.br (8.9.3/8.9.3) id RAA03477 for obm-l-list; Sat, 15 Jun 2002 17:35:21 -0300 Received: from gorgo.centroin.com.br (gorgo.centroin.com.br [200.225.63.128]) by sucuri.mat.puc-rio.br (8.9.3/8.9.3) with ESMTP id RAA03473 for ; Sat, 15 Jun 2002 17:35:19 -0300 Received: from centroin.com.br (du142c.rjo.centroin.com.br [200.225.58.142]) (authenticated bits=0) by gorgo.centroin.com.br (8.12.2/8.12.1) with ESMTP id g5FKZMko013088 for ; Sat, 15 Jun 2002 17:35:23 -0300 (BRT) Message-ID: <3D0BA559.4040400@centroin.com.br> Date: Sat, 15 Jun 2002 17:36:41 -0300 From: Augusto =?ISO-8859-1?Q?C=E9sar?= Morgado User-Agent: Mozilla/5.0 (Windows; U; Win98; en-US; rv:0.9.4.1) Gecko/20020508 Netscape6/6.2.3 X-Accept-Language: en-us MIME-Version: 1.0 To: obm-l@mat.puc-rio.br Subject: Re: [obm-l] =?ISO-8859-1?Q?Aritm=E9tica?=(=?ISO-8859-1?Q?D=FAvida?=) - Rick-C.R.B. References: <3D0ABA96000009F4@www.zipmail.com.br> Content-Type: text/plain; charset=ISO-8859-1; format=flowed Content-Transfer-Encoding: 8bit Sender: owner-obm-l@sucuri.mat.puc-rio.br Precedence: bulk Reply-To: obm-l@mat.puc-rio.br Para ser multiplo de 25, o final yx so pode ser 25, 50 ou 75 (00 esta excluido pois x, y, z sao distintos). Para ser divisivel por 3 a soma dos algarismos deve ser divisivel por 3. As soluçoes sao y=2, x=5, z=8 y=5, x=0, z=1 ou 4 ou 7 (acho que x=0 nao vale) y=7, x=5, z=0, 3, 6 ou 9 Os dois maiores numeros sao 579 975 e 576 675, cuja soma vale 1 156 650 luizhenriquerick@zipmail.com.br wrote: >Olá amigos .. >Estava resolvendo um exercício , consegui resolver na boa , mas o verificar >o resultado , vi que não estava batendo.. >Vejam só.. > >Seja N = xyzzyx um número natural escrito na base dez , onde x , y e z >são algarismos distintos .Se N1 e N2 são os dois maiores números divisíveis >por 3 e 25 , obtido a partir de N pela substituição de x , y e z , então >N1 + N2 é igual a : > >Fiz assim : >Para que o número seja divisível por 3 e por 25 , tem que ser divisível >então por 75. >Fazendo primeiros 12 múltiplos de 75 , temos: >2 x 75 = 150 >3 x 75 = 225 >4 x 75 = 300 >5 x 75 = 375 >6 x 75 = 450 >7 x 75 = 525 >8 x 75 = 600 >9 x 75 = 675 >10 x 75 = 750 >11 x 75 = 825 >12 x 75 = 900 >13 x 75 = 975 >14 x 75 = 1050 >15 x 75 = 1125 > . . > . . > . . > >Então , fazendo N1 > N2 > > >N1 = xyzzyx , verificando x (algarismo das unidades) , o maior valor que >pode assumir é 5 , ficamos com... >N1 = 5yzzy5 ,verificando y ( algarismo das dezenas ) , o maior valor que >pode assumir é 7 , ficamos com ... >N1 = 57zz75 , verificando z (algarismo das centenas ) , o maior valor que >pode assumir é 9 , ficamos com... >N1 = 579975 > >Agora N2 > >N2 = xyzzyx , verificando x (algarismo das unidades ) , o maior valor >que pode assumir é 5 , ficamos com... >N2 = 5yzzy5 ,verificando y ( algarismo das dezenas ) , o maior valor que >pode assumir é 5, mas o número terminado em 55 não é divisível por 75, ficando >então o 2 , ficamos com ... >N2 = 52zz25 , verificando z (algarismo das centenas ) , o maior valor que >pode assumir é 8 , ficamos com... >N2 = 528825 > >Então.. >N1 + N2 = 1108800 >Esta resposta tem entre umas das alternativas , porém a correta é 1157000 > >Será que alguém poderia me ajudar ? >Grato... >Rick - C.R.B. > > > > ---------------------------------------- > |-=Rick-C.R.B.=- | > |ICQ 124805654 | > |e-mail luizhenriquerick@zipmail.com.br | > ---------------------------------------- > > >------------------------------------------ >Use o melhor sistema de busca da Internet >Radar UOL - http://www.radaruol.com.br > > > >========================================================================= >Instruções para entrar na lista, sair da lista e usar a lista em >http://www.mat.puc-rio.br/~nicolau/olimp/obm-l.html >O administrador desta lista é >========================================================================= > > ========================================================================= Instruções para entrar na lista, sair da lista e usar a lista em http://www.mat.puc-rio.br/~nicolau/olimp/obm-l.html O administrador desta lista é ========================================================================= From owner-obm-l@sucuri.mat.puc-rio.br Sat Jun 15 17:47:20 2002 Return-Path: Received: (from majordom@localhost) by sucuri.mat.puc-rio.br (8.9.3/8.9.3) id RAA04019 for obm-l-list; Sat, 15 Jun 2002 17:47:14 -0300 Received: from gorgo.centroin.com.br (gorgo.centroin.com.br [200.225.63.128]) by sucuri.mat.puc-rio.br (8.9.3/8.9.3) with ESMTP id RAA04015 for ; Sat, 15 Jun 2002 17:47:12 -0300 Received: from centroin.com.br (du142c.rjo.centroin.com.br [200.225.58.142]) (authenticated bits=0) by gorgo.centroin.com.br (8.12.2/8.12.1) with ESMTP id g5FKkxko014017 for ; Sat, 15 Jun 2002 17:47:00 -0300 (BRT) Message-ID: <3D0BA813.50809@centroin.com.br> Date: Sat, 15 Jun 2002 17:48:19 -0300 From: Augusto =?ISO-8859-1?Q?C=E9sar?= Morgado User-Agent: Mozilla/5.0 (Windows; U; Win98; en-US; rv:0.9.4.1) Gecko/20020508 Netscape6/6.2.3 X-Accept-Language: en-us MIME-Version: 1.0 To: obm-l@mat.puc-rio.br Subject: Re: [obm-l] Analise Combinatoria, conceitos militares References: Content-Type: text/plain; charset=ISO-8859-1; format=flowed Content-Transfer-Encoding: 8bit Sender: owner-obm-l@sucuri.mat.puc-rio.br Precedence: bulk Reply-To: obm-l@mat.puc-rio.br Essas manifestaçoes de mau humor, como ja disse o Nicolau, nao devem ser mandadas para a lista e sim diretamente para o alvo visado. Tais manifestaçoes ja tiveram como resultado a auto-exclusao de um membro proeminente desta lista, o Prof. Dr. Jose Paulo Quinhoes Carneiro. Morgado Adriano Almeida Faustino wrote: > > Nao me interessa,se nao gostou mude a questao mas que a ideia seja a > mesma. > A questao nem e` minha,so me deram e eu queria saber se poderia usar > os lemas de Kaplansky,por isso enviei a questao!!!!!!! > > Adriano. > >> From: "Jose Francisco Guimaraes Costa" >> Reply-To: obm-l@mat.puc-rio.br >> To: >> Subject: [obm-l] Analise Combinatoria, conceitos militares >> Date: Thu, 6 Jun 2002 10:04:54 -0300 >> >> Para evitar problemas que V poderia ter se estivesse na conferência no >> auditório do IME, duas pequenas correções quanto à parte não >> matemática do >> problema: (1) almirantes, generais e brigadeiros são oficiais >> generais, e >> não oficiais superiores (na marinha os oficiais superiores são os >> capitães >> de corveta, fragata e mar-e-guerra, e no exército e aeronáutica os >> majores, >> tenentes coronéis e coronéis); (2) marinha, exército e aeronáutica são >> "forças" e não "armas" ("armas" - conceito que só se aplica ao >> exército - >> são infantaria, cavalaria e artilharia). >> >> Se é para levar a frescura um degrau acima, sendo o IME uma "OM" >> (Organização Militar), os oficiais generais estariam assentados de >> acordo >> com sua "antiguidade" (hierarquia): o mais "antigo" ao centro e os mais >> "modernos" alternadamente à direita e à esquerda dele. >> >> Para evitar esta v.... toda, sugiro trocar no enunciado do problema >> "conferência" por "jogo de futebol", "IME" por "Maracanã"; >> "auditório" por >> "geral", "7 oficiais" por "7 torcedores" e "generais, almirantes e >> brigadeiros" por "flamenguistas, fluminenses e vascaínos". >> >> Não sei como V faria para evitar uma p... briga na geral do Maracanã! >> >> JF >> >> PS: considerem isso uma pausa para recreio, tal como nos tempos de >> escola. >> >> ----- Original Message ----- >> From: "Adriano Almeida Faustino" >> To: >> Sent: Thursday, June 06, 2002 1:27 AM >> Subject: [obm-l] Analise Combinatoria >> >> >> > estou com duvida nessa questao e queria que alguem me ajudasse >> > >> > Para uma conferencia realizada no auditorio do IME,foram reservados 7 >> > lugares,que serao ocupados por 7 oficiais superiores.Sabendo-se que >> 3 sao >> > generais,2 almirantes e 2,brigadeiros e que estes lugares estao na >> primeira >> > fila,um ao lado do outro,determine de quantos modos podemos >> acomoda-los,sem >> > que haja sentados juntos oficiais de uma mesma arma. >> > []`s >> > Adriano. >> > >> > >> > >> > >> > >> > >> > >> > >> > _________________________________________________________________ >> > Envie e receba emails com o Hotmail no seu dispositivo móvel: >> > http://mobile.msn.com >> > >> > >> ========================================================================= >> >> > Instruções para entrar na lista, sair da lista e usar a lista em >> > http://www.mat.puc-rio.br/~nicolau/olimp/obm-l.html >> > O administrador desta lista é >> > >> ========================================================================= >> >> > >> >> ========================================================================= >> >> Instruções para entrar na lista, sair da lista e usar a lista em >> http://www.mat.puc-rio.br/~nicolau/olimp/obm-l.html >> O administrador desta lista é >> ========================================================================= >> > > > > _________________________________________________________________ > Chegou o novo MSN Explorer. Instale já. É gratuito: > http://explorer.msn.com.br > > ========================================================================= > Instruções para entrar na lista, sair da lista e usar a lista em > http://www.mat.puc-rio.br/~nicolau/olimp/obm-l.html > O administrador desta lista é > ========================================================================= > > ========================================================================= Instruções para entrar na lista, sair da lista e usar a lista em http://www.mat.puc-rio.br/~nicolau/olimp/obm-l.html O administrador desta lista é ========================================================================= From owner-obm-l@sucuri.mat.puc-rio.br Sat Jun 15 21:31:01 2002 Return-Path: Received: (from majordom@localhost) by sucuri.mat.puc-rio.br (8.9.3/8.9.3) id VAA06353 for obm-l-list; Sat, 15 Jun 2002 21:30:34 -0300 Received: from puma.unisys.com.br (puma.unisys.com.br [200.220.64.7]) by sucuri.mat.puc-rio.br (8.9.3/8.9.3) with ESMTP id VAA06349 for ; Sat, 15 Jun 2002 21:30:32 -0300 Received: from jf (riopm18p178.unisys.com.br [200.220.16.178] (may be forged)) by puma.unisys.com.br (8.12.3/8.12.3) with SMTP id g5G0U38C019894 for ; Sat, 15 Jun 2002 21:30:04 -0300 (EST) X-Spam-Filter: check_local@puma.unisys.com.br by digitalanswers.org Message-ID: <006801c214cc$9a2b2d40$b210dcc8@jf> From: "Jose Francisco Guimaraes Costa" To: "obm-l" References: Subject: Re: [obm-l] Analise Combinatoria, conceitos militares Date: Sat, 15 Jun 2002 21:24:33 -0300 MIME-Version: 1.0 Content-Type: text/plain; charset="iso-8859-1" Content-Transfer-Encoding: 8bit X-Priority: 3 X-MSMail-Priority: Normal X-Mailer: Microsoft Outlook Express 5.00.2919.6600 X-MimeOLE: Produced By Microsoft MimeOLE V5.00.2919.6600 Sender: owner-obm-l@sucuri.mat.puc-rio.br Precedence: bulk Reply-To: obm-l@mat.puc-rio.br No meio de tanta coisa séria que é tratada aqui, eu quis fazer "uma pausa para recreio, tal como nos tempos de escola", conforme procurei deixar bem claro na minha mensagem ao usar expressões pouco acadêmicas como "Se é para levar a frescura um degrau acima", "Para evitar esta v.... toda" e "Não sei como V faria para evitar uma p... briga na geral do Maracanã!". Infelizmente fui mal compreendido. Isto não se repetirá. JF ----- Original Message ----- From: "Adriano Almeida Faustino" To: Sent: Saturday, June 15, 2002 2:39 PM Subject: Re: [obm-l] Analise Combinatoria, conceitos militares > > Nao me interessa,se nao gostou mude a questao mas que a ideia seja a mesma. > A questao nem e` minha,so me deram e eu queria saber se poderia usar os > lemas de Kaplansky,por isso enviei a questao!!!!!!! > > Adriano. > > >From: "Jose Francisco Guimaraes Costa" > >Reply-To: obm-l@mat.puc-rio.br > >To: > >Subject: [obm-l] Analise Combinatoria, conceitos militares > >Date: Thu, 6 Jun 2002 10:04:54 -0300 > > > >Para evitar problemas que V poderia ter se estivesse na conferência no > >auditório do IME, duas pequenas correções quanto à parte não matemática do > >problema: (1) almirantes, generais e brigadeiros são oficiais generais, e > >não oficiais superiores (na marinha os oficiais superiores são os capitães > >de corveta, fragata e mar-e-guerra, e no exército e aeronáutica os majores, > >tenentes coronéis e coronéis); (2) marinha, exército e aeronáutica são > >"forças" e não "armas" ("armas" - conceito que só se aplica ao exército - > >são infantaria, cavalaria e artilharia). > > > >Se é para levar a frescura um degrau acima, sendo o IME uma "OM" > >(Organização Militar), os oficiais generais estariam assentados de acordo > >com sua "antiguidade" (hierarquia): o mais "antigo" ao centro e os mais > >"modernos" alternadamente à direita e à esquerda dele. > > > >Para evitar esta v.... toda, sugiro trocar no enunciado do problema > >"conferência" por "jogo de futebol", "IME" por "Maracanã"; "auditório" por > >"geral", "7 oficiais" por "7 torcedores" e "generais, almirantes e > >brigadeiros" por "flamenguistas, fluminenses e vascaínos". > > > >Não sei como V faria para evitar uma p... briga na geral do Maracanã! > > > >JF > > > >PS: considerem isso uma pausa para recreio, tal como nos tempos de escola. > > > >----- Original Message ----- > >From: "Adriano Almeida Faustino" > >To: > >Sent: Thursday, June 06, 2002 1:27 AM > >Subject: [obm-l] Analise Combinatoria > > > > > > > estou com duvida nessa questao e queria que alguem me ajudasse > > > > > > Para uma conferencia realizada no auditorio do IME,foram reservados 7 > > > lugares,que serao ocupados por 7 oficiais superiores.Sabendo-se que 3 > >sao > > > generais,2 almirantes e 2,brigadeiros e que estes lugares estao na > >primeira > > > fila,um ao lado do outro,determine de quantos modos podemos > >acomoda-los,sem > > > que haja sentados juntos oficiais de uma mesma arma. > > > []`s > > > Adriano. > > > > > > > > > > > > > > > > > > > > > > > > > > > _________________________________________________________________ > > > Envie e receba emails com o Hotmail no seu dispositivo móvel: > > > http://mobile.msn.com > > > > > > > >========================================================================= > > > Instruções para entrar na lista, sair da lista e usar a lista em > > > http://www.mat.puc-rio.br/~nicolau/olimp/obm-l.html > > > O administrador desta lista é > > > > >========================================================================= > > > > > > >========================================================================= > >Instruções para entrar na lista, sair da lista e usar a lista em > >http://www.mat.puc-rio.br/~nicolau/olimp/obm-l.html > >O administrador desta lista é > >========================================================================= > > > _________________________________________________________________ > Chegou o novo MSN Explorer. Instale já. É gratuito: > http://explorer.msn.com.br > > ========================================================================= > Instruções para entrar na lista, sair da lista e usar a lista em > http://www.mat.puc-rio.br/~nicolau/olimp/obm-l.html > O administrador desta lista é > ========================================================================= > ========================================================================= Instruções para entrar na lista, sair da lista e usar a lista em http://www.mat.puc-rio.br/~nicolau/olimp/obm-l.html O administrador desta lista é ========================================================================= From owner-obm-l@sucuri.mat.puc-rio.br Sat Jun 15 22:20:28 2002 Return-Path: Received: (from majordom@localhost) by sucuri.mat.puc-rio.br (8.9.3/8.9.3) id WAA06992 for obm-l-list; Sat, 15 Jun 2002 22:20:19 -0300 Received: from smtp02.ibest.com.br ([200.181.68.79]) by sucuri.mat.puc-rio.br (8.9.3/8.9.3) with SMTP id WAA06988 for ; Sat, 15 Jun 2002 22:20:17 -0300 From: hugoiver@ibest.com.br Message-Id: <200206160120.WAA06988@sucuri.mat.puc-rio.br> Received: (qmail 13020 invoked from network); 16 Jun 2002 01:19:53 -0000 Received: from unknown (HELO www5.mailbr.com) ([200.181.68.90]) (envelope-sender ) by mail.ibest.com.br (qmail-ldap-1.03) with SMTP for ; 16 Jun 2002 01:19:53 -0000 Content-Type: text/plain Content-Disposition: inline Content-Transfer-Encoding: binary X-Mailer: iBestMail 3.0 X-Originating-Ip: 200.181.68.6 X-Webmail-User: hugoiver@ibest.com.br To: obm-l@mat.puc-rio.br X-Priority: 3 (Baixa) MIME-Version: 1.0 Subject: [obm-l] (sem assunto) Date: Sun, 16 Jun 2002 1:19:53 GMT Sender: owner-obm-l@sucuri.mat.puc-rio.br Precedence: bulk Reply-To: obm-l@mat.puc-rio.br olá, alguém poderia resolver a seguinte integral? int( sin(x/(x+1))) dx iBEST - Acesso grátis à internet com alta qualidade de conexão. Experimente agora: baixe o Ultradiscador* iBEST em www.ibest.com.br ========================================================================= Instruções para entrar na lista, sair da lista e usar a lista em http://www.mat.puc-rio.br/~nicolau/olimp/obm-l.html O administrador desta lista é ========================================================================= From owner-obm-l@sucuri.mat.puc-rio.br Sat Jun 15 23:58:44 2002 Return-Path: Received: (from majordom@localhost) by sucuri.mat.puc-rio.br (8.9.3/8.9.3) id XAA07804 for obm-l-list; Sat, 15 Jun 2002 23:58:29 -0300 Received: from salvatore2.bol.com.br (salvatore2.bol.com.br [200.221.24.87]) by sucuri.mat.puc-rio.br (8.9.3/8.9.3) with ESMTP id XAA07800 for ; Sat, 15 Jun 2002 23:58:27 -0300 Received: from bol.com.br (200.221.24.84) by salvatore2.bol.com.br (5.1.071) id 3CFE36F100230D24 for obm-l@mat.puc-rio.br; Sat, 15 Jun 2002 23:57:39 -0300 Date: Sat, 15 Jun 2002 23:57:39 -0300 Message-Id: Subject: [obm-l] =?iso-8859-1?q?O_de_sempre=2C_outra_d=FAvida=2E=2E?= MIME-Version: 1.0 Content-Type: text/plain;charset="iso-8859-1" From: "leon-17" To: obm-l@mat.puc-rio.br X-XaM3-API-Version: 2.4.3.4.4 X-SenderIP: 200.254.252.200 Content-Transfer-Encoding: 8bit X-MIME-Autoconverted: from quoted-printable to 8bit by sucuri.mat.puc-rio.br id XAA07801 Sender: owner-obm-l@sucuri.mat.puc-rio.br Precedence: bulk Reply-To: obm-l@mat.puc-rio.br Obrigado José pela resposta. Aí vai outra que eu naum consegui... A soma dos valores reais de x tais que x²+x+1=156/(x²+x) é? Obrigado antecipado.. Thiago Lima __________________________________________________________________________ Quer ter seu próprio endereço na Internet? Garanta já o seu e ainda ganhe cinco e-mails personalizados. DomíniosBOL - http://dominios.bol.com.br ========================================================================= Instruções para entrar na lista, sair da lista e usar a lista em http://www.mat.puc-rio.br/~nicolau/olimp/obm-l.html O administrador desta lista é ========================================================================= From owner-obm-l@sucuri.mat.puc-rio.br Sun Jun 16 00:50:21 2002 Return-Path: Received: (from majordom@localhost) by sucuri.mat.puc-rio.br (8.9.3/8.9.3) id AAA08396 for obm-l-list; Sun, 16 Jun 2002 00:50:06 -0300 Received: from web21302.mail.yahoo.com (web21302.mail.yahoo.com [216.136.173.210]) by sucuri.mat.puc-rio.br (8.9.3/8.9.3) with SMTP id AAA08392 for ; Sun, 16 Jun 2002 00:50:03 -0300 Message-ID: <20020616034939.3329.qmail@web21302.mail.yahoo.com> Received: from [200.227.209.107] by web21302.mail.yahoo.com via HTTP; Sun, 16 Jun 2002 00:49:39 ART Date: Sun, 16 Jun 2002 00:49:39 -0300 (ART) From: =?iso-8859-1?q?Marcos=20Reynaldo?= Subject: Re: [obm-l] (sem assunto) To: obm-l@mat.puc-rio.br In-Reply-To: <200206160120.WAA06988@sucuri.mat.puc-rio.br> MIME-Version: 1.0 Content-Type: text/plain; charset=iso-8859-1 Content-Transfer-Encoding: 8bit Sender: owner-obm-l@sucuri.mat.puc-rio.br Precedence: bulk Reply-To: obm-l@mat.puc-rio.br Essa é brava. Só de curiosidade joguei no maple pra ver o que dava. Eis o resultado: sin(1-1/(x+1))*(x+1)-Si(-1/(x+1))*sin(1)+Ci(-1/(x+1))*cos(1) onde, Si(x)=int(sin(t)/t, t=0..x) Ci(x)=gamma + ln(x) + int((cos(t)-1)/t, t=0..x) Bom agora como chegar nisso é outros quinhentos. Alguém se habilita ? --- hugoiver@ibest.com.br escreveu: > olá, alguém poderia resolver a seguinte integral? > > int( sin(x/(x+1))) dx _______________________________________________________________________ Copa 2002 Yahoo! - Patrocinador oficial da Copa do Mundo da FIFA 2002 http://br.sports.yahoo.com/fifaworldcup/ ========================================================================= Instruções para entrar na lista, sair da lista e usar a lista em http://www.mat.puc-rio.br/~nicolau/olimp/obm-l.html O administrador desta lista é ========================================================================= From owner-obm-l@sucuri.mat.puc-rio.br Sun Jun 16 11:31:16 2002 Return-Path: Received: (from majordom@localhost) by sucuri.mat.puc-rio.br (8.9.3/8.9.3) id LAA13614 for obm-l-list; Sun, 16 Jun 2002 11:30:35 -0300 Received: from gorgo.centroin.com.br (gorgo.centroin.com.br [200.225.63.128]) by sucuri.mat.puc-rio.br (8.9.3/8.9.3) with ESMTP id LAA13610 for ; Sun, 16 Jun 2002 11:30:33 -0300 Received: from centroin.com.br (du97b.rjo.centroin.com.br [200.225.57.97]) (authenticated bits=0) by gorgo.centroin.com.br (8.12.2/8.12.1) with ESMTP id g5GEUgko020148 for ; Sun, 16 Jun 2002 11:30:43 -0300 (BRT) Message-ID: <3D0CA162.7010807@centroin.com.br> Date: Sun, 16 Jun 2002 11:32:02 -0300 From: Augusto =?ISO-8859-1?Q?C=E9sar?= Morgado User-Agent: Mozilla/5.0 (Windows; U; Win98; en-US; rv:0.9.4.1) Gecko/20020508 Netscape6/6.2.3 X-Accept-Language: en-us MIME-Version: 1.0 To: obm-l@mat.puc-rio.br Subject: Re: [obm-l] O de sempre, outra =?ISO-8859-1?Q?d=FAvida=2E=2E?= References: Content-Type: text/plain; charset=ISO-8859-1; format=flowed Content-Transfer-Encoding: 8bit Sender: owner-obm-l@sucuri.mat.puc-rio.br Precedence: bulk Reply-To: obm-l@mat.puc-rio.br Chame x^2 + x de y. Fica y+1 = 156/y y^2 + y -156 = 0 .................. leon-17 wrote: >Obrigado José pela resposta. >Aí vai outra que eu naum consegui... > >A soma dos valores reais de x tais que x²+x+1=156/(x²+x) >é? > >Obrigado antecipado.. > >Thiago Lima > > >__________________________________________________________________________ >Quer ter seu próprio endereço na Internet? >Garanta já o seu e ainda ganhe cinco e-mails personalizados. >DomíniosBOL - http://dominios.bol.com.br > > >========================================================================= >Instruções para entrar na lista, sair da lista e usar a lista em >http://www.mat.puc-rio.br/~nicolau/olimp/obm-l.html >O administrador desta lista é >========================================================================= > > ========================================================================= Instruções para entrar na lista, sair da lista e usar a lista em http://www.mat.puc-rio.br/~nicolau/olimp/obm-l.html O administrador desta lista é ========================================================================= From owner-obm-l@sucuri.mat.puc-rio.br Sun Jun 16 11:36:23 2002 Return-Path: Received: (from majordom@localhost) by sucuri.mat.puc-rio.br (8.9.3/8.9.3) id LAA13683 for obm-l-list; Sun, 16 Jun 2002 11:36:22 -0300 Received: from web13701.mail.yahoo.com (web13701.mail.yahoo.com [216.136.175.134]) by sucuri.mat.puc-rio.br (8.9.3/8.9.3) with SMTP id LAA13679 for ; Sun, 16 Jun 2002 11:36:19 -0300 Message-ID: <20020616143556.43439.qmail@web13701.mail.yahoo.com> Received: from [200.213.88.170] by web13701.mail.yahoo.com via HTTP; Sun, 16 Jun 2002 11:35:56 ART Date: Sun, 16 Jun 2002 11:35:56 -0300 (ART) From: =?iso-8859-1?q?pichurin?= Subject: [obm-l] matrizes e sistemas(corrigido) To: obm-l@mat.puc-rio.br MIME-Version: 1.0 Content-Type: text/plain; charset=iso-8859-1 Content-Transfer-Encoding: 8bit Sender: owner-obm-l@sucuri.mat.puc-rio.br Precedence: bulk Reply-To: obm-l@mat.puc-rio.br Sejam L1 e L2 os valores distintos de L para os quais a equação matricial A*B=B, tal que A é uma matriz quadrada de ordem 2 e B é uma matriz do tipo 2X1, sendo: a11=2 a21=3 a12=3 a22=2 b11=X1 b21=X2 E tem-se que essa equação admite solução, tal que X1 e X2 são diferentes de zero. Então, L1 + L2 vale quanto? _______________________________________________________________________ Copa 2002 Yahoo! - Patrocinador oficial da Copa do Mundo da FIFA 2002 http://br.sports.yahoo.com/fifaworldcup/ ========================================================================= Instruções para entrar na lista, sair da lista e usar a lista em http://www.mat.puc-rio.br/~nicolau/olimp/obm-l.html O administrador desta lista é ========================================================================= From owner-obm-l@sucuri.mat.puc-rio.br Sun Jun 16 11:38:59 2002 Return-Path: Received: (from majordom@localhost) by sucuri.mat.puc-rio.br (8.9.3/8.9.3) id LAA13725 for obm-l-list; Sun, 16 Jun 2002 11:38:54 -0300 Received: from zeus.hotlink.com.br (zeus.hotlink.com.br [200.249.243.250]) by sucuri.mat.puc-rio.br (8.9.3/8.9.3) with SMTP id LAA13717 for ; Sun, 16 Jun 2002 11:38:51 -0300 Received: (qmail 28611 invoked by uid 504); 16 Jun 2002 14:39:48 -0000 Received: from veloz.hotlink.com.br (HELO pureza) (200.249.243.196) by zeus.hotlink.com.br with SMTP; 16 Jun 2002 14:39:47 -0000 Message-ID: <000b01c21543$8a3f6dc0$0e00a8c0@hotlink.com.br> From: "gabriel guedes" To: References: Subject: [obm-l] =?iso-8859-1?Q?Re:_=5Bobm-l=5D_O_de_sempre=2C_outra_d=FAvida..?= Date: Sun, 16 Jun 2002 11:38:54 -0300 MIME-Version: 1.0 Content-Type: text/plain; charset="iso-8859-1" Content-Transfer-Encoding: 8bit X-Priority: 3 X-MSMail-Priority: Normal X-Mailer: Microsoft Outlook Express 5.00.2615.200 X-MimeOLE: Produced By Microsoft MimeOLE V5.00.2615.200 Sender: owner-obm-l@sucuri.mat.puc-rio.br Precedence: bulk Reply-To: obm-l@mat.puc-rio.br Bom , consegui fazer alguma coisa vejam se ta certo. chamando x^2 + x = y tems a nova equação y+1=156/y tal que y^2 + y -156=0 dai y= 12, -13 substituindo na primeira equação x^2 + x -12= 0 x = 3, -4 x^2 + x +13=0 x = [1+sqrt(51)i]/2 ,[1-sqrt(51)i]/2 ----- Original Message ----- From: leon-17 To: Sent: Saturday, June 15, 2002 11:57 PM Subject: [obm-l] O de sempre, outra dúvida.. > Obrigado José pela resposta. > Aí vai outra que eu naum consegui... > > A soma dos valores reais de x tais que x²+x+1=156/(x²+x) > é? > > Obrigado antecipado.. > > Thiago Lima > > > __________________________________________________________________________ > Quer ter seu próprio endereço na Internet? > Garanta já o seu e ainda ganhe cinco e-mails personalizados. > DomíniosBOL - http://dominios.bol.com.br > > > ========================================================================= > Instruções para entrar na lista, sair da lista e usar a lista em > http://www.mat.puc-rio.br/~nicolau/olimp/obm-l.html > O administrador desta lista é > ========================================================================= > ========================================================================= Instruções para entrar na lista, sair da lista e usar a lista em http://www.mat.puc-rio.br/~nicolau/olimp/obm-l.html O administrador desta lista é ========================================================================= From owner-obm-l@sucuri.mat.puc-rio.br Sun Jun 16 11:40:22 2002 Return-Path: Received: (from majordom@localhost) by sucuri.mat.puc-rio.br (8.9.3/8.9.3) id LAA13795 for obm-l-list; Sun, 16 Jun 2002 11:40:20 -0300 Received: from web13703.mail.yahoo.com (web13703.mail.yahoo.com [216.136.175.136]) by sucuri.mat.puc-rio.br (8.9.3/8.9.3) with SMTP id LAA13791 for ; Sun, 16 Jun 2002 11:40:17 -0300 Message-ID: <20020616143954.31943.qmail@web13703.mail.yahoo.com> Received: from [200.213.88.170] by web13703.mail.yahoo.com via HTTP; Sun, 16 Jun 2002 11:39:54 ART Date: Sun, 16 Jun 2002 11:39:54 -0300 (ART) From: =?iso-8859-1?q?pichurin?= Subject: Re: [obm-l] matrizes e sistemas To: obm-l@mat.puc-rio.br In-Reply-To: <3D0BA28C.2030102@centroin.com.br> MIME-Version: 1.0 Content-Type: text/plain; charset=iso-8859-1 Content-Transfer-Encoding: 8bit Sender: owner-obm-l@sucuri.mat.puc-rio.br Precedence: bulk Reply-To: obm-l@mat.puc-rio.br Não. O problema é exatamente esse.A única difderença é que onde está grafado X@ leia-se X2. Peguei esse problema de um livro da Companhia da Escola, que afirma que ele é da Cesgranrio. --- Augusto César Morgado escreveu: > Desconsidere a mensagem anterior. > > Nao seria AX = LX, L sendo numero e tudo que voce > chamou de B nao seria X? > > pichurin wrote: > > >Sejam L1 e L2 os valores distintos de L para os > quais > >a equação matricial A*B=B, tal que A é uma matriz > >quadrada de ordem 2 e B é uma matriz do tipo 2X1, > >sendo: > >a11=2 > >a21=3 > >a12=3 > >a22=2 > >b11=X1 > >b21=X2 > >E tem-se que essa equação admite solução, tal que > X1 e > >X@ são diferentes de zero. > >Então, L1 + L2 vale quanto? > > > > > >_______________________________________________________________________ > >Copa 2002 > >Yahoo! - Patrocinador oficial da Copa do Mundo da > FIFA 2002 > >http://br.sports.yahoo.com/fifaworldcup/ > >========================================================================= > >Instruções para entrar na lista, sair da lista e > usar a lista em > >http://www.mat.puc-rio.br/~nicolau/olimp/obm-l.html > >O administrador desta lista é > > >========================================================================= > > > > > > > ========================================================================= > Instruções para entrar na lista, sair da lista e > usar a lista em > http://www.mat.puc-rio.br/~nicolau/olimp/obm-l.html > O administrador desta lista é > > ========================================================================= _______________________________________________________________________ Copa 2002 Yahoo! - Patrocinador oficial da Copa do Mundo da FIFA 2002 http://br.sports.yahoo.com/fifaworldcup/ ========================================================================= Instruções para entrar na lista, sair da lista e usar a lista em http://www.mat.puc-rio.br/~nicolau/olimp/obm-l.html O administrador desta lista é ========================================================================= From owner-obm-l@sucuri.mat.puc-rio.br Sun Jun 16 11:42:03 2002 Return-Path: Received: (from majordom@localhost) by sucuri.mat.puc-rio.br (8.9.3/8.9.3) id LAA13930 for obm-l-list; Sun, 16 Jun 2002 11:42:00 -0300 Received: from intercon.interconect.com.br (intercon.interconect.com.br [200.241.255.65]) by sucuri.mat.puc-rio.br (8.9.3/8.9.3) with SMTP id LAA13925 for ; Sun, 16 Jun 2002 11:41:57 -0300 Received: (qmail 27687 invoked by uid 105); 16 Jun 2002 14:29:40 -0000 Received: from unknown (HELO t6o7e2) (200.242.250.108) by intercon.interconect.com.br with SMTP; 16 Jun 2002 14:29:39 -0000 Message-ID: <000a01c21544$18e33ca0$6cfaf2c8@t6o7e2> From: "Odelir Maria Casanova dos Santos" To: References: Subject: [obm-l] =?iso-8859-1?Q?Re:_=5Bobm-l=5D_O_de_sempre=2C_outra_d=FAvida..?= Date: Sun, 16 Jun 2002 11:42:51 -0300 MIME-Version: 1.0 Content-Type: text/plain; charset="iso-8859-1" Content-Transfer-Encoding: 8bit X-Priority: 3 X-MSMail-Priority: Normal X-Mailer: Microsoft Outlook Express 6.00.2600.0000 X-MimeOLE: Produced By Microsoft MimeOLE V6.00.2600.0000 X-Virus-Scanned: by AMaViS perl-11 Sender: owner-obm-l@sucuri.mat.puc-rio.br Precedence: bulk Reply-To: obm-l@mat.puc-rio.br Essa não é tão dificil x²+x = y ; assim temos y+1 = 156/y y²+y-156 = 0; daí y = 12 ou y = -13 só que y = x²+x então x²+x-12 = 0 , x²+x+13 = 0 observe que x²+x+13 não apresenta solução para os reais e que em x²+x-12 , x = 3 ou x = -4 então a resposta é igual a -1 Falo cara Dimitri ----- Original Message ----- From: "leon-17" To: Sent: Saturday, June 15, 2002 11:57 PM Subject: [obm-l] O de sempre, outra dúvida.. > Obrigado José pela resposta. > Aí vai outra que eu naum consegui... > > A soma dos valores reais de x tais que x²+x+1=156/(x²+x) > é? > > Obrigado antecipado.. > > Thiago Lima > > > __________________________________________________________________________ > Quer ter seu próprio endereço na Internet? > Garanta já o seu e ainda ganhe cinco e-mails personalizados. > DomíniosBOL - http://dominios.bol.com.br > > > ========================================================================= > Instruções para entrar na lista, sair da lista e usar a lista em > http://www.mat.puc-rio.br/~nicolau/olimp/obm-l.html > O administrador desta lista é > ========================================================================= > ========================================================================= Instruções para entrar na lista, sair da lista e usar a lista em http://www.mat.puc-rio.br/~nicolau/olimp/obm-l.html O administrador desta lista é ========================================================================= From owner-obm-l@sucuri.mat.puc-rio.br Sun Jun 16 14:37:24 2002 Return-Path: Received: (from majordom@localhost) by sucuri.mat.puc-rio.br (8.9.3/8.9.3) id OAA16431 for obm-l-list; Sun, 16 Jun 2002 14:37:08 -0300 Received: from www.zipmail.com.br (smtp.zipmail.com.br [200.187.242.10]) by sucuri.mat.puc-rio.br (8.9.3/8.9.3) with ESMTP id OAA16426 for ; Sun, 16 Jun 2002 14:37:06 -0300 From: luizhenriquerick@zipmail.com.br Received: from [200.165.185.61] by www.zipmail.com.br with HTTP; Sun, 16 Jun 2002 14:36:44 -0300 Message-ID: <3D0BBAFE00002558@www.zipmail.com.br> Date: Sun, 16 Jun 2002 14:36:44 -0300 Subject: [obm-l] =?iso-8859-1?Q?Re=3AAntiga=20fatora=E7=E3o?= To: obm-l@mat.puc-rio.br MIME-Version: 1.0 Content-Type: text/plain; charset="iso-8859-1" Content-Transfer-Encoding: 8bit X-MIME-Autoconverted: from quoted-printable to 8bit by sucuri.mat.puc-rio.br id OAA16427 Sender: owner-obm-l@sucuri.mat.puc-rio.br Precedence: bulk Reply-To: obm-l@mat.puc-rio.br Olá amigos. fiquei um tempo sem verificar o que estava acontecendo na lista , e esses dias eu vi uma questão que mandaram de fatoração , não sei bem quem foi que mandou , mas tentei resolver. se cometi algum equívoco , por favor me de um toque .. Abraço..Ae vai.. X^6 +X^3.Y^3 + Y^6 Sendo X^3 = a e Y^3 = b a² + ab + b² Como (a + b)² = a² + 2ab + b² (a + b)² -2ab = a² + b² (a + b)² - ab (X^3 + Y^3) - X^3.Y^3 SENDO X^3 = a² => a = sqrt(X^3) Y^3 = b² => b = sqrt(Y^3) (a² + b²)² - a².b² (a² + b²)² - (ab)² (a² + b² - ab).(a² + b² + ab) Substituindo X e Y novamente (X^3 + Y^3 - X^3/2).(X^3 + Y^3 + X^3/2) SERÁ QUE ISSO TA CERTO ? ---------------------------------------- |-=Rick-C.R.B.=- | |ICQ 124805654 | |e-mail luizhenriquerick@zipmail.com.br | ---------------------------------------- ------------------------------------------ Use o melhor sistema de busca da Internet Radar UOL - http://www.radaruol.com.br ========================================================================= Instruções para entrar na lista, sair da lista e usar a lista em http://www.mat.puc-rio.br/~nicolau/olimp/obm-l.html O administrador desta lista é ========================================================================= From owner-obm-l@sucuri.mat.puc-rio.br Sun Jun 16 14:57:18 2002 Return-Path: Received: (from majordom@localhost) by sucuri.mat.puc-rio.br (8.9.3/8.9.3) id OAA16872 for obm-l-list; Sun, 16 Jun 2002 14:56:48 -0300 Received: from pina.terra.com.br (pina.terra.com.br [200.176.3.17]) by sucuri.mat.puc-rio.br (8.9.3/8.9.3) with ESMTP id OAA16857 for ; Sun, 16 Jun 2002 14:56:45 -0300 Received: from pavuna.terra.com.br (pavuna.terra.com.br [200.176.3.41]) by pina.terra.com.br (Postfix) with ESMTP id 90FEE52F39 for ; Sun, 16 Jun 2002 14:56:24 -0300 (EST) Received: from terra.com.br (webmail2.terra.com.br [200.176.3.177]) (authenticated user hilhend) by pavuna.terra.com.br (Postfix) with ESMTP id 8A77F680A6 for ; Sun, 16 Jun 2002 14:56:24 -0300 (EST) Date: Sun, 16 Jun 2002 17:56:24 +0000 Message-Id: Subject: [obm-l] =?iso-8859-1?q?Re=3A_=5Bobm=2Dl=5D_Re=3AAntiga_fatora=E7=E3o?= MIME-Version: 1.0 Content-Type: text/plain;charset="iso-8859-1" From: "hilhend" To: "obm-l" X-XaM3-API-Version: 2.4.3.2.9 X-SenderIP: 200.161.14.239 Content-Transfer-Encoding: 8bit X-MIME-Autoconverted: from quoted-printable to 8bit by sucuri.mat.puc-rio.br id OAA16863 Sender: owner-obm-l@sucuri.mat.puc-rio.br Precedence: bulk Reply-To: obm-l@mat.puc-rio.br O problema é que ,me disseram, na fatoração não fica contendo expontes fracionários(X^3/2), como é o caso da solução(abaixo) proposta. > Olá amigos. > fiquei um tempo sem verificar o que estava acontecendo na lista , e esses > dias eu vi uma questão que mandaram de fatoração , não sei bem quem foi > que mandou , mas tentei resolver. > se cometi algum equívoco , por favor me de um toque .. > Abraço..Ae vai.. > > X^6 +X^3.Y^3 + Y^6 > Sendo X^3 = a e Y^3 = b > > a² + ab + b² Como (a + b)² = a² + 2ab + b² > (a + b)² -2ab = a² + b² > (a + b)² - ab > (X^3 + Y^3) - X^3.Y^3 SENDO X^3 = a² => a = sqrt(X^3) > Y^3 = b² => b = sqrt(Y^3) > (a² + b²)² - a².b² > (a² + b²)² - (ab)² > (a² + b² - ab).(a² + b² + ab) > Substituindo X e Y novamente > (X^3 + Y^3 - X^3/2).(X^3 + Y^3 + X^3/2) > > SERÁ QUE ISSO TA CERTO ? > > > > > > > > > > ---------------------------------------- > |-=Rick-C.R.B.=- | > |ICQ 124805654 | > |e-mail luizhenriquerick@zipmail.com.br | > ---------------------------------------- > > > ------------------------------------------ > Use o melhor sistema de busca da Internet > Radar UOL - http://www.radaruol.com.br > > > > ==================================================================== ===== > Instruções para entrar na lista, sair da lista e usar a lista em > http://www.mat.puc-rio.br/~nicolau/olimp/obm-l.html > O administrador desta lista é > ==================================================================== ===== > > ========================================================================= Instruções para entrar na lista, sair da lista e usar a lista em http://www.mat.puc-rio.br/~nicolau/olimp/obm-l.html O administrador desta lista é ========================================================================= From owner-obm-l@sucuri.mat.puc-rio.br Sun Jun 16 17:38:48 2002 Return-Path: Received: (from majordom@localhost) by sucuri.mat.puc-rio.br (8.9.3/8.9.3) id RAA18057 for obm-l-list; Sun, 16 Jun 2002 17:38:35 -0300 Received: from www.zipmail.com.br (smtp.zipmail.com.br [200.187.242.10]) by sucuri.mat.puc-rio.br (8.9.3/8.9.3) with ESMTP id RAA18053 for ; Sun, 16 Jun 2002 17:38:33 -0300 From: luizhenriquerick@zipmail.com.br Received: from [200.165.185.61] by www.zipmail.com.br with HTTP; Sun, 16 Jun 2002 17:38:10 -0300 Message-ID: <3D0CECDA00000278@www.zipmail.com.br> Date: Sun, 16 Jun 2002 17:38:10 -0300 In-Reply-To: Subject: [obm-l] =?iso-8859-1?Q?Re=3A=20=5Bobm=2Dl=5D=20Re=3A=20=5Bobm=2Dl=5D=20Re=3AAntiga=20fatora=E7=E3o?= To: obm-l@mat.puc-rio.br MIME-Version: 1.0 Content-Type: text/plain; charset="iso-8859-1" Content-Transfer-Encoding: 8bit X-MIME-Autoconverted: from quoted-printable to 8bit by sucuri.mat.puc-rio.br id RAA18054 Sender: owner-obm-l@sucuri.mat.puc-rio.br Precedence: bulk Reply-To: obm-l@mat.puc-rio.br Po , essa deve ser braba então. Sera que alguém mais experiente ae não poderia dar uma opinião ? Tipo Morgado , Ponce .. etc ---------------------------------------- |-=Rick-C.R.B.=- | |ICQ 124805654 | |e-mail luizhenriquerick@zipmail.com.br | ---------------------------------------- ------------------------------------------ Use o melhor sistema de busca da Internet Radar UOL - http://www.radaruol.com.br ========================================================================= Instruções para entrar na lista, sair da lista e usar a lista em http://www.mat.puc-rio.br/~nicolau/olimp/obm-l.html O administrador desta lista é ========================================================================= From owner-obm-l@sucuri.mat.puc-rio.br Sun Jun 16 19:29:06 2002 Return-Path: Received: (from majordom@localhost) by sucuri.mat.puc-rio.br (8.9.3/8.9.3) id TAA19149 for obm-l-list; Sun, 16 Jun 2002 19:28:47 -0300 Received: from gorgo.centroin.com.br (gorgo.centroin.com.br [200.225.63.128]) by sucuri.mat.puc-rio.br (8.9.3/8.9.3) with ESMTP id TAA19145 for ; Sun, 16 Jun 2002 19:28:45 -0300 Received: from centroin.com.br (du135c.rjo.centroin.com.br [200.225.58.135]) (authenticated bits=0) by gorgo.centroin.com.br (8.12.2/8.12.1) with ESMTP id g5GMStko029394 for ; Sun, 16 Jun 2002 19:28:56 -0300 (BRT) Message-ID: <3D0D1177.6010203@centroin.com.br> Date: Sun, 16 Jun 2002 19:30:15 -0300 From: Augusto =?ISO-8859-1?Q?C=E9sar?= Morgado User-Agent: Mozilla/5.0 (Windows; U; Win98; en-US; rv:0.9.4.1) Gecko/20020508 Netscape6/6.2.3 X-Accept-Language: en-us MIME-Version: 1.0 To: obm-l@mat.puc-rio.br Subject: Re: [obm-l] matrizes e sistemas References: <20020616143954.31943.qmail@web13703.mail.yahoo.com> Content-Type: multipart/alternative; boundary="------------000304070801090806090606" Sender: owner-obm-l@sucuri.mat.puc-rio.br Precedence: bulk Reply-To: obm-l@mat.puc-rio.br --------------000304070801090806090606 Content-Type: text/plain; charset=ISO-8859-1; format=flowed Content-Transfer-Encoding: 8bit So pode ser: >Sejam L1 e L2 os valores distintos do escalar L para os > quais a equação matricial A*B=LB, tal que A é uma matriz quadrada de ordem 2 e B é uma matriz do tipo 2X1, sendo: a11=2 a21=3 a12=3 a22=2 b11=X1 b21=X2 > >E tem-se que essa equação admite solução, tal que > X1 e X2 são diferentes de zero. Então, L1 + L2 vale quanto? Resposta: 4 _________________ pichurin wrote: >Não. O problema é exatamente esse.A única difderença é >que onde está grafado X@ leia-se X2. >Peguei esse problema de um livro da Companhia da >Escola, que afirma que ele é da Cesgranrio. > > > > > > > > --- Augusto César Morgado >escreveu: > Desconsidere a mensagem anterior. > >>Nao seria AX = LX, L sendo numero e tudo que voce >>chamou de B nao seria X? >> >>pichurin wrote: >> >>>Sejam L1 e L2 os valores distintos de L para os >>> >>quais >> >>>a equação matricial A*B=B, tal que A é uma matriz >>>quadrada de ordem 2 e B é uma matriz do tipo 2X1, >>>sendo: >>>a11=2 >>>a21=3 >>>a12=3 >>>a22=2 >>>b11=X1 >>>b21=X2 >>>E tem-se que essa equação admite solução, tal que >>> >>X1 e >> >>>X@ são diferentes de zero. >>>Então, L1 + L2 vale quanto? >>> >>> >>_______________________________________________________________________ >> >>>Copa 2002 >>>Yahoo! - Patrocinador oficial da Copa do Mundo da >>> >>FIFA 2002 >> >>>http://br.sports.yahoo.com/fifaworldcup/ >>> >>========================================================================= >> >>>Instruções para entrar na lista, sair da lista e >>> >>usar a lista em >> >>>http://www.mat.puc-rio.br/~nicolau/olimp/obm-l.html >>>O administrador desta lista é >>> >> >> >>========================================================================= >> >>> >> >> >========================================================================= > >>Instruções para entrar na lista, sair da lista e >>usar a lista em >>http://www.mat.puc-rio.br/~nicolau/olimp/obm-l.html >>O administrador desta lista é >> >> >========================================================================= > >_______________________________________________________________________ >Copa 2002 >Yahoo! - Patrocinador oficial da Copa do Mundo da FIFA 2002 >http://br.sports.yahoo.com/fifaworldcup/ >========================================================================= >Instruções para entrar na lista, sair da lista e usar a lista em >http://www.mat.puc-rio.br/~nicolau/olimp/obm-l.html >O administrador desta lista é >========================================================================= > > --------------000304070801090806090606 Content-Type: text/html; charset=us-ascii Content-Transfer-Encoding: 7bit So pode ser:
Sejam L1 e L2 os valores distintos do escalar L para os
quais a equação matricial A*B=LB, tal que A é uma matriz
quadrada de ordem 2 e B é uma matriz do tipo 2X1,
sendo:
a11=2
a21=3
a12=3
a22=2
b11=X1
b21=X2

E tem-se que essa equação admite solução, tal que
X1 e X2 são diferentes de zero.
Então, L1 + L2 vale quanto?
Resposta: 4


_________________





pichurin wrote:
Não. O problema é exatamente esse.A única difderença é
que onde está grafado X@ leia-se X2.
Peguei esse problema de um livro da Companhia da
Escola, que afirma que ele é da Cesgranrio.







--- Augusto César Morgado <morgado@centroin.com.br>
escreveu: > Desconsidere a mensagem anterior.
Nao seria AX = LX, L sendo numero e tudo que voce
chamou de B nao seria X?

pichurin wrote:

Sejam L1 e L2 os valores distintos de L para os
quais
a equação matricial A*B=B, tal que A é uma matriz
quadrada de ordem 2 e B é uma matriz do tipo 2X1,
sendo:
a11=2
a21=3
a12=3
a22=2
b11=X1
b21=X2
E tem-se que essa equação admite solução, tal que
X1 e
X@ são diferentes de zero.
Então, L1 + L2 vale quanto?


_______________________________________________________________________
Copa 2002
Yahoo! - Patrocinador oficial da Copa do Mundo da
FIFA 2002
http://br.sports.yahoo.com/fifaworldcup/
=========================================================================
Instruções para entrar na lista, sair da lista e
usar a lista em
http://www.mat.puc-rio.br/~nicolau/olimp/obm-l.html
O administrador desta lista é
<nicolau@mat.puc-rio.br>

=========================================================================



=========================================================================
Instruções para entrar na lista, sair da lista e
usar a lista em
http://www.mat.puc-rio.br/~nicolau/olimp/obm-l.html
O administrador desta lista é
<nicolau@mat.puc-rio.br>

========================================================================= 

_______________________________________________________________________
Copa 2002
Yahoo! - Patrocinador oficial da Copa do Mundo da FIFA 2002
http://br.sports.yahoo.com/fifaworldcup/
=========================================================================
Instruções para entrar na lista, sair da lista e usar a lista em
http://www.mat.puc-rio.br/~nicolau/olimp/obm-l.html
O administrador desta lista é <nicolau@mat.puc-rio.br>
=========================================================================



--------------000304070801090806090606-- ========================================================================= Instruções para entrar na lista, sair da lista e usar a lista em http://www.mat.puc-rio.br/~nicolau/olimp/obm-l.html O administrador desta lista é ========================================================================= From owner-obm-l@sucuri.mat.puc-rio.br Sun Jun 16 19:30:51 2002 Return-Path: Received: (from majordom@localhost) by sucuri.mat.puc-rio.br (8.9.3/8.9.3) id TAA19191 for obm-l-list; Sun, 16 Jun 2002 19:30:50 -0300 Received: from gorgo.centroin.com.br (gorgo.centroin.com.br [200.225.63.128]) by sucuri.mat.puc-rio.br (8.9.3/8.9.3) with ESMTP id TAA19187 for ; Sun, 16 Jun 2002 19:30:48 -0300 Received: from centroin.com.br (du135c.rjo.centroin.com.br [200.225.58.135]) (authenticated bits=0) by gorgo.centroin.com.br (8.12.2/8.12.1) with ESMTP id g5GMUwko029560 for ; Sun, 16 Jun 2002 19:30:59 -0300 (BRT) Message-ID: <3D0D11F2.2060700@centroin.com.br> Date: Sun, 16 Jun 2002 19:32:18 -0300 From: Augusto =?ISO-8859-1?Q?C=E9sar?= Morgado User-Agent: Mozilla/5.0 (Windows; U; Win98; en-US; rv:0.9.4.1) Gecko/20020508 Netscape6/6.2.3 X-Accept-Language: en-us MIME-Version: 1.0 To: obm-l@mat.puc-rio.br Subject: Re: [obm-l] matrizes e sistemas References: <20020616143954.31943.qmail@web13703.mail.yahoo.com> Content-Type: multipart/alternative; boundary="------------000906000503010802010100" Sender: owner-obm-l@sucuri.mat.puc-rio.br Precedence: bulk Reply-To: obm-l@mat.puc-rio.br --------------000906000503010802010100 Content-Type: text/plain; charset=ISO-8859-1; format=flowed Content-Transfer-Encoding: 8bit So pode ser: >Sejam L1 e L2 os valores distintos do escalar L para os > quais a equação matricial (em B) A*B=LB, tal que A é uma matriz quadrada de ordem 2 e B é uma matriz do tipo 2X1, sendo: a11=2 a21=3 a12=3 a22=2 b11=X1 b21=X2 > >admite solução tal que > X1 e X2 não são simultaneamente nulos. Então, L1 + L2 vale quanto? Resposta: 4 _________________ pichurin wrote: >Não. O problema é exatamente esse.A única difderença é >que onde está grafado X@ leia-se X2. >Peguei esse problema de um livro da Companhia da >Escola, que afirma que ele é da Cesgranrio. > > > > > > > > --- Augusto César Morgado >escreveu: > Desconsidere a mensagem anterior. > >>Nao seria AX = LX, L sendo numero e tudo que voce >>chamou de B nao seria X? >> >>pichurin wrote: >> >>>Sejam L1 e L2 os valores distintos de L para os >>> >>quais >> >>>a equação matricial A*B=B, tal que A é uma matriz >>>quadrada de ordem 2 e B é uma matriz do tipo 2X1, >>>sendo: >>>a11=2 >>>a21=3 >>>a12=3 >>>a22=2 >>>b11=X1 >>>b21=X2 >>>E tem-se que essa equação admite solução, tal que >>> >>X1 e >> >>>X@ são diferentes de zero. >>>Então, L1 + L2 vale quanto? >>> >>> >>_______________________________________________________________________ >> >>>Copa 2002 >>>Yahoo! - Patrocinador oficial da Copa do Mundo da >>> >>FIFA 2002 >> >>>http://br.sports.yahoo.com/fifaworldcup/ >>> >>========================================================================= >> >>>Instruções para entrar na lista, sair da lista e >>> >>usar a lista em >> >>>http://www.mat.puc-rio.br/~nicolau/olimp/obm-l.html >>>O administrador desta lista é >>> >> >> >>========================================================================= >> >>> >> >> >========================================================================= > >>Instruções para entrar na lista, sair da lista e >>usar a lista em >>http://www.mat.puc-rio.br/~nicolau/olimp/obm-l.html >>O administrador desta lista é >> >> >========================================================================= > >_______________________________________________________________________ >Copa 2002 >Yahoo! - Patrocinador oficial da Copa do Mundo da FIFA 2002 >http://br.sports.yahoo.com/fifaworldcup/ >========================================================================= >Instruções para entrar na lista, sair da lista e usar a lista em >http://www.mat.puc-rio.br/~nicolau/olimp/obm-l.html >O administrador desta lista é >========================================================================= > > --------------000906000503010802010100 Content-Type: text/html; charset=us-ascii Content-Transfer-Encoding: 7bit So pode ser:
Sejam L1 e L2 os valores distintos do escalar L para os
quais a equação matricial (em B)  A*B=LB, tal que A é uma matriz
quadrada de ordem 2 e B é uma matriz do tipo 2X1,
sendo:
a11=2
a21=3
a12=3
a22=2
b11=X1
b21=X2

admite solução tal que
X1 e X2 não são simultaneamente nulos.
Então, L1 + L2 vale quanto?
Resposta: 4


_________________





pichurin wrote:
Não. O problema é exatamente esse.A única difderença é
que onde está grafado X@ leia-se X2.
Peguei esse problema de um livro da Companhia da
Escola, que afirma que ele é da Cesgranrio.







--- Augusto César Morgado <morgado@centroin.com.br>
escreveu: > Desconsidere a mensagem anterior.
Nao seria AX = LX, L sendo numero e tudo que voce
chamou de B nao seria X?

pichurin wrote:

Sejam L1 e L2 os valores distintos de L para os
quais
a equação matricial A*B=B, tal que A é uma matriz
quadrada de ordem 2 e B é uma matriz do tipo 2X1,
sendo:
a11=2
a21=3
a12=3
a22=2
b11=X1
b21=X2
E tem-se que essa equação admite solução, tal que
X1 e
X@ são diferentes de zero.
Então, L1 + L2 vale quanto?


_______________________________________________________________________
Copa 2002
Yahoo! - Patrocinador oficial da Copa do Mundo da
FIFA 2002
http://br.sports.yahoo.com/fifaworldcup/
=========================================================================
Instruções para entrar na lista, sair da lista e
usar a lista em
http://www.mat.puc-rio.br/~nicolau/olimp/obm-l.html
O administrador desta lista é
<nicolau@mat.puc-rio.br>

=========================================================================



=========================================================================
Instruções para entrar na lista, sair da lista e
usar a lista em
http://www.mat.puc-rio.br/~nicolau/olimp/obm-l.html
O administrador desta lista é
<nicolau@mat.puc-rio.br>

========================================================================= 

_______________________________________________________________________
Copa 2002
Yahoo! - Patrocinador oficial da Copa do Mundo da FIFA 2002
http://br.sports.yahoo.com/fifaworldcup/
=========================================================================
Instruções para entrar na lista, sair da lista e usar a lista em
http://www.mat.puc-rio.br/~nicolau/olimp/obm-l.html
O administrador desta lista é <nicolau@mat.puc-rio.br>
=========================================================================



--------------000906000503010802010100-- ========================================================================= Instruções para entrar na lista, sair da lista e usar a lista em http://www.mat.puc-rio.br/~nicolau/olimp/obm-l.html O administrador desta lista é ========================================================================= From owner-obm-l@sucuri.mat.puc-rio.br Sun Jun 16 20:36:52 2002 Return-Path: Received: (from majordom@localhost) by sucuri.mat.puc-rio.br (8.9.3/8.9.3) id UAA20360 for obm-l-list; Sun, 16 Jun 2002 20:36:34 -0300 Received: from web10107.mail.yahoo.com (web10107.mail.yahoo.com [216.136.130.57]) by sucuri.mat.puc-rio.br (8.9.3/8.9.3) with SMTP id UAA20350 for ; Sun, 16 Jun 2002 20:36:31 -0300 Message-ID: <20020616233610.70196.qmail@web10107.mail.yahoo.com> Received: from [150.161.199.56] by web10107.mail.yahoo.com via HTTP; Sun, 16 Jun 2002 16:36:10 PDT Date: Sun, 16 Jun 2002 16:36:10 -0700 (PDT) From: Rafael WC Subject: [obm-l] circunferencias To: OBM MIME-Version: 1.0 Content-Type: text/plain; charset=us-ascii Sender: owner-obm-l@sucuri.mat.puc-rio.br Precedence: bulk Reply-To: obm-l@mat.puc-rio.br Oi Pessoal! Será que alguém pode me dar uma dica nessa aqui? O que preciso usar? Não estou conseguindo... I e I' são os centros das circunferencias inscritas e ex-inscritas no angulo A de um triangulo ABC. Demonstrar que: AI . AI' = bc Qualquer ajuda é bem-vinda! Rafael. ===== Rafael Werneck Cinoto ICQ# 107011599 rwcinoto@yahoo.com matduvidas@yahoo.com.br http://www.rwcinoto.hpg.com.br/ __________________________________________________ Do You Yahoo!? Yahoo! - Official partner of 2002 FIFA World Cup http://fifaworldcup.yahoo.com ========================================================================= Instruções para entrar na lista, sair da lista e usar a lista em http://www.mat.puc-rio.br/~nicolau/olimp/obm-l.html O administrador desta lista é ========================================================================= From owner-obm-l@sucuri.mat.puc-rio.br Sun Jun 16 21:31:11 2002 Return-Path: Received: (from majordom@localhost) by sucuri.mat.puc-rio.br (8.9.3/8.9.3) id VAA21030 for obm-l-list; Sun, 16 Jun 2002 21:30:55 -0300 Received: from www.zipmail.com.br (smtp.zipmail.com.br [200.187.242.10]) by sucuri.mat.puc-rio.br (8.9.3/8.9.3) with ESMTP id VAA21026 for ; Sun, 16 Jun 2002 21:30:53 -0300 From: yurigomes@zipmail.com.br Received: from [200.151.169.14] by www.zipmail.com.br with HTTP; Sun, 16 Jun 2002 21:30:23 -0300 Message-ID: <3D0C97120000157B@www.zipmail.com.br> Date: Sun, 16 Jun 2002 21:30:23 -0300 In-Reply-To: <20020616233610.70196.qmail@web10107.mail.yahoo.com> Subject: [obm-l] =?iso-8859-1?Q?Re=3A=20=5Bobm=2Dl=5D=20circunferencias?= To: obm-l@mat.puc-rio.br MIME-Version: 1.0 Content-Type: text/plain; charset="iso-8859-1" Content-Transfer-Encoding: 8bit X-MIME-Autoconverted: from quoted-printable to 8bit by sucuri.mat.puc-rio.br id VAA21027 Sender: owner-obm-l@sucuri.mat.puc-rio.br Precedence: bulk Reply-To: obm-l@mat.puc-rio.br Faça lei dos senos nos triângulos ABI' e AIC, donde: i) AI'/sen(90 + B/2)=AB/sen(C/2) AI'= AB.cos(B/2)/sen(C/2) ii) AI/sen(C/2) = AC/sen((A+C)/2) AI = AB.sen(C/2)/cos(B/2) Multiplicando, obtemos AI.AI' = AB.AC -- Mensagem original -- >Oi Pessoal! > >Será que alguém pode me dar uma dica nessa aqui? O que >preciso usar? Não estou conseguindo... > >I e I' são os centros das circunferencias inscritas e >ex-inscritas no angulo A de um triangulo ABC. >Demonstrar que: >AI . AI' = bc > >Qualquer ajuda é bem-vinda! > >Rafael. > >===== >Rafael Werneck Cinoto > ICQ# 107011599 > rwcinoto@yahoo.com > matduvidas@yahoo.com.br >http://www.rwcinoto.hpg.com.br/ > >__________________________________________________ >Do You Yahoo!? >Yahoo! - Official partner of 2002 FIFA World Cup >http://fifaworldcup.yahoo.com >========================================================================= >Instruções para entrar na lista, sair da lista e usar a lista em >http://www.mat.puc-rio.br/~nicolau/olimp/obm-l.html >O administrador desta lista é >========================================================================= > []'s, Yuri ICQ: 64992515 ------------------------------------------ Use o melhor sistema de busca da Internet Radar UOL - http://www.radaruol.com.br ========================================================================= Instruções para entrar na lista, sair da lista e usar a lista em http://www.mat.puc-rio.br/~nicolau/olimp/obm-l.html O administrador desta lista é ========================================================================= From owner-obm-l@sucuri.mat.puc-rio.br Sun Jun 16 22:09:01 2002 Return-Path: Received: (from majordom@localhost) by sucuri.mat.puc-rio.br (8.9.3/8.9.3) id WAA21644 for obm-l-list; Sun, 16 Jun 2002 22:08:17 -0300 Received: from shannon.bol.com.br (shannon.bol.com.br [200.221.24.13]) by sucuri.mat.puc-rio.br (8.9.3/8.9.3) with ESMTP id WAA21639 for ; Sun, 16 Jun 2002 22:08:15 -0300 Received: from bol.com.br (200.221.24.78) by shannon.bol.com.br (5.1.071) id 3D090583000B9A81 for obm-l@mat.puc-rio.br; Sun, 16 Jun 2002 22:07:31 -0300 Date: Sun, 16 Jun 2002 22:07:30 -0300 Message-Id: Subject: [obm-l] =?iso-8859-1?q?F=EDsica?= MIME-Version: 1.0 Content-Type: text/plain;charset="iso-8859-1" From: "rafaelc.l" To: obm-l@mat.puc-rio.br X-XaM3-API-Version: 2.4.3.4.4 X-SenderIP: 172.187.74.186 Content-Transfer-Encoding: 8bit X-MIME-Autoconverted: from quoted-printable to 8bit by sucuri.mat.puc-rio.br id WAA21640 Sender: owner-obm-l@sucuri.mat.puc-rio.br Precedence: bulk Reply-To: obm-l@mat.puc-rio.br Sei que aqui é uma lista de matemática, mas mesmo assim alguém poderia me ajudar na questão 10 da prova de física do IME 2001. É um circuito elétrico com um gráfico. Eu vi uma resolução em que foi ultilizado o cálculo integral. Eu pergunto: qual seria uma outra forma de resolver a questão sem matemática superior?, já que isto não faz parte do programa do IME. __________________________________________________________________________ Quer ter seu próprio endereço na Internet? Garanta já o seu e ainda ganhe cinco e-mails personalizados. DomíniosBOL - http://dominios.bol.com.br ========================================================================= Instruções para entrar na lista, sair da lista e usar a lista em http://www.mat.puc-rio.br/~nicolau/olimp/obm-l.html O administrador desta lista é ========================================================================= From owner-obm-l@sucuri.mat.puc-rio.br Sun Jun 16 23:20:10 2002 Return-Path: Received: (from majordom@localhost) by sucuri.mat.puc-rio.br (8.9.3/8.9.3) id XAA22383 for obm-l-list; Sun, 16 Jun 2002 23:19:45 -0300 Received: from smtp-33.ig.com.br (smtp-33.ig.com.br [200.226.132.183]) by sucuri.mat.puc-rio.br (8.9.3/8.9.3) with SMTP id XAA22379 for ; Sun, 16 Jun 2002 23:19:43 -0300 Received: (qmail 4219 invoked from network); 17 Jun 2002 01:55:28 -0000 Received: from shasta043116.ig.com.br (HELO jat) (200.151.43.116) by smtp-33.ig.com.br with SMTP; 17 Jun 2002 01:55:26 -0000 Message-ID: <002d01c215a2$0d3ecd20$742b97c8@jat> From: "Jose Augusto" To: References: Subject: [obm-l] =?iso-8859-1?Q?Re:_=5Bobm-l=5D_F=EDsica?= Date: Sun, 16 Jun 2002 22:55:25 -0300 MIME-Version: 1.0 Content-Type: text/plain; charset="iso-8859-1" Content-Transfer-Encoding: 8bit X-Priority: 3 X-MSMail-Priority: Normal X-Mailer: Microsoft Outlook Express 5.50.4133.2400 X-MIMEOLE: Produced By Microsoft MimeOLE V5.50.4133.2400 Sender: owner-obm-l@sucuri.mat.puc-rio.br Precedence: bulk Reply-To: obm-l@mat.puc-rio.br ----- Original Message ----- From: "rafaelc.l" To: Sent: Sunday, June 16, 2002 10:07 PM Subject: [obm-l] Física > > Sei que aqui é uma lista de matemática, mas > mesmo assim alguém poderia me ajudar na > questão 10 da prova de física do IME 2001. É > um circuito elétrico com um gráfico. Eu vi > uma resolução em que foi ultilizado o > cálculo integral. Eu pergunto: qual seria > uma outra forma de resolver a questão sem > matemática superior?, já que isto não faz > parte do programa do IME. > http://www.ime.eb.br/~uchoa/Download/IME_Fisica_P_S_2000-2001.pdf C for essa prova ......tem resolucao ai ...... na pagina do ime a resolucao utilizada ta usando integral?????? ========================================================================= Instruções para entrar na lista, sair da lista e usar a lista em http://www.mat.puc-rio.br/~nicolau/olimp/obm-l.html O administrador desta lista é ========================================================================= From owner-obm-l@sucuri.mat.puc-rio.br Sun Jun 16 23:37:41 2002 Return-Path: Received: (from majordom@localhost) by sucuri.mat.puc-rio.br (8.9.3/8.9.3) id XAA22750 for obm-l-list; Sun, 16 Jun 2002 23:37:25 -0300 Received: from smtp-33.ig.com.br (smtp-33.ig.com.br [200.226.132.183]) by sucuri.mat.puc-rio.br (8.9.3/8.9.3) with SMTP id XAA22746 for ; Sun, 16 Jun 2002 23:37:23 -0300 Received: (qmail 6737 invoked from network); 17 Jun 2002 02:27:24 -0000 Received: from 134.61.226.200.in-addr.arpa.ig.com.br (HELO oemcomputer) (200.226.61.134) by smtp-33.ig.com.br with SMTP; 17 Jun 2002 02:27:18 -0000 Message-ID: <009201c215a6$ffb80680$863de2c8@oemcomputer> From: "Daniel" To: References: Subject: [obm-l] =?iso-8859-1?Q?Re:_=5Bobm-l=5D_F=EDsica?= Date: Sun, 16 Jun 2002 23:30:49 -0300 MIME-Version: 1.0 Content-Type: text/plain; charset="iso-8859-1" Content-Transfer-Encoding: 8bit X-Priority: 3 X-MSMail-Priority: Normal X-Mailer: Microsoft Outlook Express 5.00.2615.200 X-MimeOLE: Produced By Microsoft MimeOLE V5.00.2615.200 Sender: owner-obm-l@sucuri.mat.puc-rio.br Precedence: bulk Reply-To: obm-l@mat.puc-rio.br > Sei que aqui é uma lista de matemática, mas > mesmo assim alguém poderia me ajudar na > questão 10 da prova de física do IME 2001. É > um circuito elétrico com um gráfico. Eu vi > uma resolução em que foi ultilizado o > cálculo integral. Eu pergunto: qual seria > uma outra forma de resolver a questão sem > matemática superior?, já que isto não faz > parte do programa do IME. Não sei se ajuda, na verdade se quiser posso achar depois pra vc. É só usar o teorema de Thevenin e transformar o cicruito em um circuito equivalente de Thevenin, assim não é preciso resolver integrais. Daniel ========================================================================= Instruções para entrar na lista, sair da lista e usar a lista em http://www.mat.puc-rio.br/~nicolau/olimp/obm-l.html O administrador desta lista é ========================================================================= From owner-obm-l@sucuri.mat.puc-rio.br Mon Jun 17 01:48:12 2002 Return-Path: Received: (from majordom@localhost) by sucuri.mat.puc-rio.br (8.9.3/8.9.3) id BAA24046 for obm-l-list; Mon, 17 Jun 2002 01:47:53 -0300 Received: from web10104.mail.yahoo.com (web10104.mail.yahoo.com [216.136.130.54]) by sucuri.mat.puc-rio.br (8.9.3/8.9.3) with SMTP id BAA24042 for ; Mon, 17 Jun 2002 01:47:50 -0300 Message-ID: <20020617044730.44549.qmail@web10104.mail.yahoo.com> Received: from [150.161.199.54] by web10104.mail.yahoo.com via HTTP; Sun, 16 Jun 2002 21:47:30 PDT Date: Sun, 16 Jun 2002 21:47:30 -0700 (PDT) From: Rafael WC Subject: [obm-l] 3 circulos! To: OBM MIME-Version: 1.0 Content-Type: text/plain; charset=us-ascii Sender: owner-obm-l@sucuri.mat.puc-rio.br Precedence: bulk Reply-To: obm-l@mat.puc-rio.br Oi Pessoal! Alguém conseguiria me dar uma indica do que usar nesse exercício? Os raios dos circulos inscritos num triangulo retangulo ABC e nos dois triangulos ABH e ACH determinados pela altura relativa à hipotenusa BC são respectivamente r ,r1 e r2. Demonstrar que: r² = r1² + r2² Valeu! Rafael. ===== Rafael Werneck Cinoto ICQ# 107011599 rwcinoto@yahoo.com matduvidas@yahoo.com.br http://www.rwcinoto.hpg.com.br/ __________________________________________________ Do You Yahoo!? Yahoo! - Official partner of 2002 FIFA World Cup http://fifaworldcup.yahoo.com ========================================================================= Instruções para entrar na lista, sair da lista e usar a lista em http://www.mat.puc-rio.br/~nicolau/olimp/obm-l.html O administrador desta lista é ========================================================================= From owner-obm-l@sucuri.mat.puc-rio.br Mon Jun 17 07:00:51 2002 Return-Path: Received: (from majordom@localhost) by sucuri.mat.puc-rio.br (8.9.3/8.9.3) id HAA25841 for obm-l-list; Mon, 17 Jun 2002 07:00:37 -0300 Received: (from nicolau@localhost) by sucuri.mat.puc-rio.br (8.9.3/8.9.3) id HAA25836 for obm-l@mat.puc-rio.br; Mon, 17 Jun 2002 07:00:36 -0300 Date: Mon, 17 Jun 2002 07:00:36 -0300 From: "Nicolau C. Saldanha" To: obm-l@mat.puc-rio.br Subject: [obm-l] Re: =?iso-8859-1?Q?=5Bobm-l=5D_Fatora=E7=E3o-__Rick-C=2ER=2EB?= Message-ID: <20020617070036.A25807@sucuri.mat.puc-rio.br> References: <3D0ABA9600000A51@www.zipmail.com.br> Mime-Version: 1.0 Content-Type: text/plain; charset=iso-8859-1 Content-Disposition: inline Content-Transfer-Encoding: 8bit User-Agent: Mutt/1.2.5i In-Reply-To: <3D0ABA9600000A51@www.zipmail.com.br>; from luizhenriquerick@zipmail.com.br on Sat, Jun 15, 2002 at 10:21:03AM -0300 Sender: owner-obm-l@sucuri.mat.puc-rio.br Precedence: bulk Reply-To: obm-l@mat.puc-rio.br On Sat, Jun 15, 2002 at 10:21:03AM -0300, luizhenriquerick@zipmail.com.br wrote: > Olá amigos.. > Se puderem me dar uma luz nessa questão.. > Obs: > Nicolau , eu sei que vc ja me disse isso mais de mil vezes ..rsrs > Mais qual o tamanho ideal para um arquivo ser mandado para a lista ? > Grato.. O ideal é usar apenas texto. Arquivos *.gif, *.jpg ou similares são ok em alguns casos. O majordom barra automaticamente qq mensagem com >20000 bytes (incluindo tudo). []s, N. ========================================================================= Instruções para entrar na lista, sair da lista e usar a lista em http://www.mat.puc-rio.br/~nicolau/olimp/obm-l.html O administrador desta lista é ========================================================================= From owner-obm-l@sucuri.mat.puc-rio.br Mon Jun 17 13:40:49 2002 Return-Path: Received: (from majordom@localhost) by sucuri.mat.puc-rio.br (8.9.3/8.9.3) id NAA29319 for obm-l-list; Mon, 17 Jun 2002 13:39:32 -0300 Received: from www.zipmail.com.br (smtp.zipmail.com.br [200.187.242.10]) by sucuri.mat.puc-rio.br (8.9.3/8.9.3) with ESMTP id NAA29308 for ; Mon, 17 Jun 2002 13:39:27 -0300 From: peterdirichlet@zipmail.com.br Received: from [200.206.103.3] by www.zipmail.com.br with HTTP; Mon, 17 Jun 2002 13:39:07 -0300 Message-ID: <3D0A2A7100001DE4@www.zipmail.com.br> Date: Mon, 17 Jun 2002 13:39:07 -0300 In-Reply-To: <20020617044730.44549.qmail@web10104.mail.yahoo.com> Subject: [obm-l] =?iso-8859-1?Q?Re=3A=20=5Bobm=2Dl=5D=203=20circulos=21?= To: obm-l@mat.puc-rio.br MIME-Version: 1.0 Content-Type: text/plain; charset="iso-8859-1" Content-Transfer-Encoding: 8bit X-MIME-Autoconverted: from quoted-printable to 8bit by sucuri.mat.puc-rio.br id NAA29316 Sender: owner-obm-l@sucuri.mat.puc-rio.br Precedence: bulk Reply-To: obm-l@mat.puc-rio.br Caro Rafael, agora eu tive uma ideia que pode ajudar.Se voce fizer AB=c AC=b, voce consegue achar AH,BC,BH e HC em funçao de b e c(dica:trigonometria).E,com as ditas medidas voce consegue achar os inraios que voce quer.E e so substituir e comemorar!!!!! Peterdirichlet PS.:estes tipos de exercicio sao bem comuns em olimpiadas! -- Mensagem original -- >Oi Pessoal! > >Alguém conseguiria me dar uma indica do que usar nesse >exercício? > >Os raios dos circulos inscritos num triangulo >retangulo ABC e nos dois triangulos ABH e ACH >determinados pela altura relativa à hipotenusa BC são >respectivamente r ,r1 e r2. Demonstrar que: >r² = r1² + r2² > >Valeu! > >Rafael. > >===== >Rafael Werneck Cinoto > ICQ# 107011599 > rwcinoto@yahoo.com > matduvidas@yahoo.com.br >http://www.rwcinoto.hpg.com.br/ > >__________________________________________________ >Do You Yahoo!? >Yahoo! - Official partner of 2002 FIFA World Cup >http://fifaworldcup.yahoo.com >========================================================================= >Instruções para entrar na lista, sair da lista e usar a lista em >http://www.mat.puc-rio.br/~nicolau/olimp/obm-l.html >O administrador desta lista é >========================================================================= > TRANSIRE SVVM PECTVS MVNDOQUE POTIRE CONGREGATI EX TOTO ORBE MATHEMATICI OB SCRIPTA INSIGNIA TRIBVERE Medalha Fields(John Charles Fields) ------------------------------------------ Use o melhor sistema de busca da Internet Radar UOL - http://www.radaruol.com.br ========================================================================= Instruções para entrar na lista, sair da lista e usar a lista em http://www.mat.puc-rio.br/~nicolau/olimp/obm-l.html O administrador desta lista é ========================================================================= From owner-obm-l@sucuri.mat.puc-rio.br Mon Jun 17 13:46:24 2002 Return-Path: Received: (from majordom@localhost) by sucuri.mat.puc-rio.br (8.9.3/8.9.3) id NAA29474 for obm-l-list; Mon, 17 Jun 2002 13:46:03 -0300 Received: from silva5.uol.com.br (silva5.uol.com.br [200.231.206.218]) by sucuri.mat.puc-rio.br (8.9.3/8.9.3) with ESMTP id NAA29470 for ; Mon, 17 Jun 2002 13:46:00 -0300 Received: from cabru ([200.148.108.99]) by silva5.uol.com.br (8.9.1/8.9.1) with SMTP id NAA14954 for ; Mon, 17 Jun 2002 13:47:06 -0300 (EST) Message-ID: <009501c2161e$8dec51a0$636c94c8@cabru> From: "Bruno" To: References: <3D04E6CB000005CC@www.zipmail.com.br> Subject: Re: [obm-l] Re: [obm-l] Desafio o retorno!! Date: Mon, 17 Jun 2002 13:46:07 -0300 MIME-Version: 1.0 Content-Type: text/plain; charset="iso-8859-1" Content-Transfer-Encoding: 8bit X-Priority: 3 X-MSMail-Priority: Normal X-Mailer: Microsoft Outlook Express 5.50.4133.2400 X-MimeOLE: Produced By Microsoft MimeOLE V5.50.4133.2400 Sender: owner-obm-l@sucuri.mat.puc-rio.br Precedence: bulk Reply-To: obm-l@mat.puc-rio.br obrigado pela resolução, mas eu queria saber se é possivel fatorar apenas em IR..... ----- Original Message ----- From: To: Sent: Monday, June 10, 2002 4:33 PM Subject: [obm-l] Re: [obm-l] Desafio o retorno!! > Vou usar complexos(a paixao de JP): > Seja a expressao f(x)=x^2+x+1.Vamos fatora-la em R+Ri,i^2+1=0. Defina cis > x=sen x+i*cos x=e^(ix). > Entao w=(cis(2*pi/3)) e wbarra=(cis(4*pi)/3) sao zeros de f. > Para as raizes de x^6+x^3+1,ache as raizes cubicas de w e wbarra. > Te mais!!!!!!!!!!!!!!!! > > > -- Mensagem original -- > > >Olá amigos, > >Estou de volta com outro exercício(+difícil na minha opinião) > >Meu professor disse que nem ele consegue fatorar a seguinte expressão em > >IR: > >x^6 + (xy)^3 + y^6 > > > >Abraços, > >Bruno > > > > TRANSIRE SVVM PECTVS MVNDOQUE POTIRE > CONGREGATI EX TOTO ORBE MATHEMATICI OB SCRIPTA INSIGNIA TRIBVERE > Medalha Fields(John Charles Fields) > > > ------------------------------------------ > Use o melhor sistema de busca da Internet > Radar UOL - http://www.radaruol.com.br > > > > ========================================================================= > Instruções para entrar na lista, sair da lista e usar a lista em > http://www.mat.puc-rio.br/~nicolau/olimp/obm-l.html > O administrador desta lista é > ========================================================================= > ========================================================================= Instruções para entrar na lista, sair da lista e usar a lista em http://www.mat.puc-rio.br/~nicolau/olimp/obm-l.html O administrador desta lista é ========================================================================= From owner-obm-l@sucuri.mat.puc-rio.br Mon Jun 17 14:02:20 2002 Return-Path: Received: (from majordom@localhost) by sucuri.mat.puc-rio.br (8.9.3/8.9.3) id OAA29813 for obm-l-list; Mon, 17 Jun 2002 14:01:52 -0300 Received: from www.zipmail.com.br (smtp.zipmail.com.br [200.187.242.10]) by sucuri.mat.puc-rio.br (8.9.3/8.9.3) with ESMTP id OAA29808 for ; Mon, 17 Jun 2002 14:01:46 -0300 From: peterdirichlet@zipmail.com.br Received: from [200.206.103.3] by www.zipmail.com.br with HTTP; Mon, 17 Jun 2002 14:01:26 -0300 Message-ID: <3D0A2A7100001E72@www.zipmail.com.br> Date: Mon, 17 Jun 2002 14:01:26 -0300 In-Reply-To: <5.1.0.14.2.20020614171511.00aa8618@pop.gmx.net> Subject: [obm-l] =?iso-8859-1?Q?Re=3A=20=5Bobm=2Dl=5D=20Mais=20duvidas=20de=20analitica/geo=20plana?= To: obm-l@mat.puc-rio.br MIME-Version: 1.0 Content-Type: text/plain; charset="iso-8859-1" Content-Transfer-Encoding: 8bit X-MIME-Autoconverted: from quoted-printable to 8bit by sucuri.mat.puc-rio.br id OAA29810 Sender: owner-obm-l@sucuri.mat.puc-rio.br Precedence: bulk Reply-To: obm-l@mat.puc-rio.br Meu,pelo que saiba essas coisas de analitica se provam na porrada e sem escrupulos.Basta verificar isso traçando paralelas aos ejes do sistema cartesiano. Quanto ao paralelogramo da para usar complexos.Depois eu jogo essa soluçao. Peterdirichlet -- Mensagem original -- >At 14:00 6/14/2002 -0300, you wrote: > >>Eu, de novo, com meus problemas de analitica. >> >>Tendo dois pontos A(a,b) B(c,d), eu consigo achar a equacao da reta que >>passa pelos dois pontos multiplicando a matriz {a, b | c, d} por {x,y}. >>Como eu posso provar que isso é verdade? >>Outra coisa que eu fiz, mas acho que a resposta nao está conferindo. >>Como eu provo que as diagonais de um paralelogramo se cortam ao meio? >>Pode ser por analitica ou por plana. > >Hum.. vou comentar primeiro o segundo que tenho mais certeza. Temos o >paralelogramo ABCD (segue figura). Seja M o ponto médio do segmento AC, ou > >seja, AM[vetor] = MC[vetor]. Queremos mostrar que M também é ponto médio >do >segumento BC, ou seja, que BM[vetor] = MD[vetor]. >BM[vetor] = BC[vetor] + CM[vetor] = MA[vetor] + AD[vetor] = -DM[vetor] = > >MC[vetor]. >Logo as duas diagonais AC e BD se cortam ao meio. > >O primeiro problema é algo que vem me atormentando há tempos mas eu conheço > >de uma maneira levemente diferente. Sejam os pontos A=(a,b) e B=(c,d) >montando a seguinte matriz e igualando o determinante a 0, tambem chegamos > >na equação geral da reta que passa por esses dois pontos: > | x y 1 | > | a b 1 | = 0 > | c d 1 | matriz [1] >Essa matriz também tem outras propriedades misteriosas... Tomemos um >triangulo em E² com vértices A=(a,b), B=(c,d) e C=(e,f), sendo D o >determinante da matriz abaixo: > | a b 1 | > | c d 1 | > | e f 1 | matriz[2] >A área do triângulo desse triangulo é dada por |D|/2. > >A partir dessa propriedade fica natural 'zerarmos' o determinante que >usamos para achar a equação da reta... já que se os pontos sao colineares > >eles nao podem formar um triangulo e portanto a area deve ser nula. > >Ainda assim.. só disfarçamos um pouco o problema... fica a questão. porque > >cargas d'água ao montarmos a matriz [2] acima, calcularmos o determinante, > >pegarmos seu valor absoluto e dividirmos por dois temos a area do triangulo... > >Quando trabalhamos em E³, quando temos tres vetores u=(x1,y1,z1) >v=(x2,y2,z2) e w=(x3,y3,z3). e calculamos o determinante da matriz formada > >pelos valores de x1,y1,z1...etc. se os tres vetores forem linearmente >independentes, obtemos o volume do paralelepipedo dos quais os tres sao >vertices. Se eles forem linearmente dependentes esse determinante é 0. > >Nao sei até que ponto essas propriedades misteriosas da matrizes se aplicam > >ou podem ser provadas em E². Voltando para a matriz[1] podemos imaginar >tres vetores, v=(x,y,1), u=(c,d,1) e w=(e,f,1). Ao impormos D(matriz[1]) >= >0, queremos que esses tres vetores sejam linearmente dependentes, ou seja > >paralelos ao mesmo plano. Novamente.. ainda é um mistério pra mim como isso > >se relaciona as retas em E². > >Bom, acho que só atrapalhei e inventei mais duvidas do que solucionei seu > >problema de fato... mas como já comentei era algo que vinha me atormentando > >e com sorte outros membros da lista vao jogar uma luz nisso :P > > > > > > >"... a perfect formulation of a problem is already half > its solution." > David Hilbert. >- >[]'s >Fernando Henrique Ferraz Pereira da Rosa >USP, IME, Estatística >http://www.linux.ime.usp.br/~feferraz > TRANSIRE SVVM PECTVS MVNDOQUE POTIRE CONGREGATI EX TOTO ORBE MATHEMATICI OB SCRIPTA INSIGNIA TRIBVERE Medalha Fields(John Charles Fields) ------------------------------------------ Use o melhor sistema de busca da Internet Radar UOL - http://www.radaruol.com.br ========================================================================= Instruções para entrar na lista, sair da lista e usar a lista em http://www.mat.puc-rio.br/~nicolau/olimp/obm-l.html O administrador desta lista é ========================================================================= From owner-obm-l@sucuri.mat.puc-rio.br Mon Jun 17 14:12:07 2002 Return-Path: Received: (from majordom@localhost) by sucuri.mat.puc-rio.br (8.9.3/8.9.3) id OAA30376 for obm-l-list; Mon, 17 Jun 2002 14:11:55 -0300 Received: from www.zipmail.com.br (smtp.zipmail.com.br [200.187.242.10]) by sucuri.mat.puc-rio.br (8.9.3/8.9.3) with ESMTP id OAA30372 for ; Mon, 17 Jun 2002 14:11:53 -0300 From: luizhenriquerick@zipmail.com.br Received: from [200.165.184.76] by www.zipmail.com.br with HTTP; Mon, 17 Jun 2002 14:11:28 -0300 Message-ID: <3D0A2A7300002465@www.zipmail.com.br> Date: Mon, 17 Jun 2002 14:11:28 -0300 In-Reply-To: <20020617070036.A25807@sucuri.mat.puc-rio.br> Subject: [obm-l] =?iso-8859-1?Q?Re=3A=20=5Bobm=2Dl=5D=20Fatora=E7=E3o=2D=20=20Rick=2DC=2ER=2EB?= To: obm-l@mat.puc-rio.br MIME-Version: 1.0 Content-Type: text/plain; charset="iso-8859-1" Content-Transfer-Encoding: 8bit X-MIME-Autoconverted: from quoted-printable to 8bit by sucuri.mat.puc-rio.br id OAA30373 Sender: owner-obm-l@sucuri.mat.puc-rio.br Precedence: bulk Reply-To: obm-l@mat.puc-rio.br Vou ver aqui se consigo enviar caracteres , tipo raiz . ---------------------------------------- |-=Rick-C.R.B.=- | |ICQ 124805654 | |e-mail luizhenriquerick@zipmail.com.br | ---------------------------------------- ------------------------------------------ Use o melhor sistema de busca da Internet Radar UOL - http://www.radaruol.com.br ========================================================================= Instruções para entrar na lista, sair da lista e usar a lista em http://www.mat.puc-rio.br/~nicolau/olimp/obm-l.html O administrador desta lista é ========================================================================= From owner-obm-l@sucuri.mat.puc-rio.br Mon Jun 17 14:39:00 2002 Return-Path: Received: (from majordom@localhost) by sucuri.mat.puc-rio.br (8.9.3/8.9.3) id OAA31400 for obm-l-list; Mon, 17 Jun 2002 14:38:42 -0300 Received: from calhau.terra.com.br (calhau.terra.com.br [200.176.3.20]) by sucuri.mat.puc-rio.br (8.9.3/8.9.3) with ESMTP id OAA31396 for ; Mon, 17 Jun 2002 14:38:38 -0300 Received: from smtp4-poa.terra.com.br (smtp4-poa.terra.com.br [200.176.3.35]) by calhau.terra.com.br (Postfix) with ESMTP id 0C202470BD for ; Mon, 17 Jun 2002 17:38:18 +0000 (GMT) Received: from stabel (dl-nas1-poa-C89A008D.p001.terra.com.br [200.154.0.141]) (authenticated user dudasta) by smtp4-poa.terra.com.br (Postfix) with ESMTP id 7F478AC620 for ; Mon, 17 Jun 2002 14:38:16 -0300 (EST) Message-ID: <001601c21625$c3f43e00$8d009ac8@stabel> From: "Eduardo Casagrande Stabel" To: References: <3D0A2A7100001E72@www.zipmail.com.br> Subject: Re: [obm-l] Re: [obm-l] Mais duvidas de analitica/geo plana Date: Mon, 17 Jun 2002 14:38:14 -0300 MIME-Version: 1.0 Content-Type: text/plain; charset="iso-8859-1" Content-Transfer-Encoding: 8bit X-Priority: 3 X-MSMail-Priority: Normal X-Mailer: Microsoft Outlook Express 6.00.2600.0000 X-MimeOLE: Produced By Microsoft MimeOLE V6.00.2600.0000 Sender: owner-obm-l@sucuri.mat.puc-rio.br Precedence: bulk Reply-To: obm-l@mat.puc-rio.br Caro Peterdirichlet. > Meu,pelo que saiba essas coisas de analitica se provam na porrada e sem > escrupulos.Basta verificar isso traçando paralelas aos ejes do sistema cartesiano. > Quanto ao paralelogramo da para usar complexos.Depois eu jogo essa soluçao. > Peterdirichlet > Não sei o que você quer dizer por "sem escrúpulos", mas pelo que entendi vocês não devem ter recebido uma mensagem que enviei à lista e que mostrava aquela relação a ver com um determinante que nada tem de "misterioso". O modo como eu fiz não é violento, portanto ninguém deve sair ferido... Reenvio abaixo a minha mensagem. **** Ola! Um jeito de esclarever sua dúvida é fazer o seguinte. Sejam (a,b) e (c,d) dois pontos distintos do plano. Prove o seguinte: Se o ponto (x,y) pertence à reta que passa pelos dois pontos então existe um real t tal que t*(a,b) + (1-t)*(c,d) = (x,y) Repare que o grafico da função t -> t*(a,b) + (1-t)*(c,d) é uma reta, e calcule t=0 e t=1 para ver que ela passa pelos pontos (a,b) e (c,d). Depois de provar isso, use as propriedades do determinante. Se o determinante for | x y 1 | | a b 1 | = 0 | c d 1 | então as linhas são linearmente dependentes. Como as duas últimas são linearmente independentes segue que a primeira é combinação das duas últimas (x,y,1) = q*(a,b,1) + p*(c,d,1) Temos q + p = 1, substitui q = t e p = 1 - t (x,y,1) = t*(a,b,1) + (1-t)*(c,d,1) o que implica (x,y) = t*(a,b) + (1-t)*(c,d) e daí (x,y) pertence à reta que passa por (a,b) e (c,d). Reciprocamente, se (x,y) pertence à essa reta, existe t tal que (x,y) = t*(a,b) + (1-t)*(c,d) e daí (x,y,1) = t*(a,b,1) + (1-t)*(c,d,1) o que implica que (x,y,1) é combinação linear de (a,b,1) e (c,d,1) daí o determinante | x y 1 | | a b 1 | = 0 | c d 1 | o que completa a prova. Em relação à primeira observação, a saber, que todos os pontos da reta são obtidos multiplicando-se | a b | | x | | c d |.| y | tenho que dizer que ela não é verdadeira. Por que? Ponha (a,b) = (1,0) e (c,d) = (0,1), todos que estudaram um pouco de matrizes sabem que todos os pontos do plano podem ser obtidos fazendo a multiplicação matricial aí de cima, portanto não se trate de uma reta. Uma possibilidade, em termos de multiplicação seria | a c | | t | | b d |.|1-t| o que é igual à primeira observação do e-mail. Um abraço! Eduardo Casagrande Stabel. Porto Alegre, RS. **** ========================================================================= Instruções para entrar na lista, sair da lista e usar a lista em http://www.mat.puc-rio.br/~nicolau/olimp/obm-l.html O administrador desta lista é ========================================================================= From owner-obm-l@sucuri.mat.puc-rio.br Mon Jun 17 15:50:01 2002 Return-Path: Received: (from majordom@localhost) by sucuri.mat.puc-rio.br (8.9.3/8.9.3) id PAA00545 for obm-l-list; Mon, 17 Jun 2002 15:47:51 -0300 Received: from www.zipmail.com.br (smtp.zipmail.com.br [200.187.242.10]) by sucuri.mat.puc-rio.br (8.9.3/8.9.3) with ESMTP id PAA00541 for ; Mon, 17 Jun 2002 15:47:46 -0300 From: peterdirichlet@zipmail.com.br Received: from [200.206.103.3] by www.zipmail.com.br with HTTP; Mon, 17 Jun 2002 15:47:06 -0300 Message-ID: <3D0E210E00000167@www.zipmail.com.br> Date: Mon, 17 Jun 2002 15:47:06 -0300 In-Reply-To: <009501c2161e$8dec51a0$636c94c8@cabru> Subject: [obm-l] =?iso-8859-1?Q?Re=3A=20=5Bobm=2Dl=5D=20Re=3A=20=5Bobm=2Dl=5D=20Desafio=20o=20retorno=21=21?= To: obm-l@mat.puc-rio.br MIME-Version: 1.0 Content-Type: text/plain; charset="iso-8859-1" Content-Transfer-Encoding: 8bit X-MIME-Autoconverted: from quoted-printable to 8bit by sucuri.mat.puc-rio.br id PAA00542 Sender: owner-obm-l@sucuri.mat.puc-rio.br Precedence: bulk Reply-To: obm-l@mat.puc-rio.br Vou especificar mais: Basta voce achar as raizes de que eu estava falando,e depois agrupar tudo convenientemente(vao aparecer pares de conjugados,que sao legais p/produtar). Te mais!!!!!!!!!!!!!!Peterdirichlet -- Mensagem original -- >obrigado pela resolução, mas eu queria saber se é possivel fatorar apenas >em >IR..... > >----- Original Message ----- >From: >To: >Sent: Monday, June 10, 2002 4:33 PM >Subject: [obm-l] Re: [obm-l] Desafio o retorno!! > > >> Vou usar complexos(a paixao de JP): >> Seja a expressao f(x)=x^2+x+1.Vamos fatora-la em R+Ri,i^2+1=0. Defina cis >> x=sen x+i*cos x=e^(ix). >> Entao w=(cis(2*pi/3)) e wbarra=(cis(4*pi)/3) sao zeros de f. >> Para as raizes de x^6+x^3+1,ache as raizes cubicas de w e wbarra. >> Te mais!!!!!!!!!!!!!!!! >> >> >> -- Mensagem original -- >> >> >Olá amigos, >> >Estou de volta com outro exercício(+difícil na minha opinião) >> >Meu professor disse que nem ele consegue fatorar a seguinte expressão >em >> >IR: >> >x^6 + (xy)^3 + y^6 >> > >> >Abraços, >> >Bruno >> > >> >> TRANSIRE SVVM PECTVS MVNDOQUE POTIRE >> CONGREGATI EX TOTO ORBE MATHEMATICI OB SCRIPTA INSIGNIA TRIBVERE >> Medalha Fields(John Charles Fields) >> >> >> ------------------------------------------ >> Use o melhor sistema de busca da Internet >> Radar UOL - http://www.radaruol.com.br >> >> >> >> ========================================================================= >> Instruções para entrar na lista, sair da lista e usar a lista em >> http://www.mat.puc-rio.br/~nicolau/olimp/obm-l.html >> O administrador desta lista é >> ========================================================================= >> > >========================================================================= >Instruções para entrar na lista, sair da lista e usar a lista em >http://www.mat.puc-rio.br/~nicolau/olimp/obm-l.html >O administrador desta lista é >========================================================================= > TRANSIRE SVVM PECTVS MVNDOQUE POTIRE CONGREGATI EX TOTO ORBE MATHEMATICI OB SCRIPTA INSIGNIA TRIBVERE Medalha Fields(John Charles Fields) ------------------------------------------ Use o melhor sistema de busca da Internet Radar UOL - http://www.radaruol.com.br ========================================================================= Instruções para entrar na lista, sair da lista e usar a lista em http://www.mat.puc-rio.br/~nicolau/olimp/obm-l.html O administrador desta lista é ========================================================================= From owner-obm-l@sucuri.mat.puc-rio.br Mon Jun 17 16:56:33 2002 Return-Path: Received: (from majordom@localhost) by sucuri.mat.puc-rio.br (8.9.3/8.9.3) id QAA02097 for obm-l-list; Mon, 17 Jun 2002 16:55:20 -0300 Received: from cairu.terra.com.br (cairu.terra.com.br [200.176.3.19]) by sucuri.mat.puc-rio.br (8.9.3/8.9.3) with ESMTP id QAA02086 for ; Mon, 17 Jun 2002 16:55:10 -0300 Received: from engenho.terra.com.br (engenho.terra.com.br [200.176.3.42]) by cairu.terra.com.br (Postfix) with ESMTP id 013FF47B17 for ; Mon, 17 Jun 2002 16:54:43 +0000 (GMT) Received: from niski.com (dl-adsl-C8D44CF7.sao.terra.com.br [200.212.76.247]) (authenticated user fniski) by engenho.terra.com.br (Postfix) with ESMTP id AC02F680FA for ; Mon, 17 Jun 2002 16:54:48 -0300 (EST) Message-ID: <3D0E3E89.1060706@niski.com> Date: Mon, 17 Jun 2002 16:54:49 -0300 From: niski User-Agent: Mozilla/5.0 (Windows; U; Windows NT 5.1; en-US; rv:0.9.4.1) Gecko/20020508 Netscape6/6.2.3 X-Accept-Language: en-us MIME-Version: 1.0 To: obm-l@mat.puc-rio.br Subject: Re: [obm-l] Re: [obm-l] Re: [obm-l] Desafio o retorno!! References: <3D0E210E00000167@www.zipmail.com.br> Content-Type: multipart/alternative; boundary="------------020307020809070003070805" Sender: owner-obm-l@sucuri.mat.puc-rio.br Precedence: bulk Reply-To: obm-l@mat.puc-rio.br --------------020307020809070003070805 Content-Type: text/plain; charset=ISO-8859-1; format=flowed Content-Transfer-Encoding: 8bit Você é aluno do Glenn? já tenho a expressao fatorada peterdirichlet@zipmail.com.br wrote: >Vou especificar mais: >Basta voce achar as raizes de que eu estava falando,e depois agrupar tudo >convenientemente(vao aparecer pares de conjugados,que sao legais p/produtar). >Te mais!!!!!!!!!!!!!!Peterdirichlet > >-- Mensagem original -- > >>obrigado pela resolução, mas eu queria saber se é possivel fatorar apenas >>em >>IR..... >> >>----- Original Message ----- >>From: >>To: >>Sent: Monday, June 10, 2002 4:33 PM >>Subject: [obm-l] Re: [obm-l] Desafio o retorno!! >> >> >>>Vou usar complexos(a paixao de JP): >>>Seja a expressao f(x)=x^2+x+1.Vamos fatora-la em R+Ri,i^2+1=0. Defina >>> >cis > >>>x=sen x+i*cos x=e^(ix). >>>Entao w=(cis(2*pi/3)) e wbarra=(cis(4*pi)/3) sao zeros de f. >>>Para as raizes de x^6+x^3+1,ache as raizes cubicas de w e wbarra. >>>Te mais!!!!!!!!!!!!!!!! >>> >>> >>>-- Mensagem original -- >>> >>>>Olá amigos, >>>>Estou de volta com outro exercício(+difícil na minha opinião) >>>>Meu professor disse que nem ele consegue fatorar a seguinte expressão >>>> >>em >> >>>>IR: >>>>x^6 + (xy)^3 + y^6 >>>> >>>>Abraços, >>>>Bruno >>>> >>>TRANSIRE SVVM PECTVS MVNDOQUE POTIRE >>>CONGREGATI EX TOTO ORBE MATHEMATICI OB SCRIPTA INSIGNIA TRIBVERE >>>Medalha Fields(John Charles Fields) >>> >>> >>>------------------------------------------ >>>Use o melhor sistema de busca da Internet >>>Radar UOL - http://www.radaruol.com.br >>> >>> >>> >>>========================================================================= >>>Instruções para entrar na lista, sair da lista e usar a lista em >>>http://www.mat.puc-rio.br/~nicolau/olimp/obm-l.html >>>O administrador desta lista é >>>========================================================================= >>> >>========================================================================= >>Instruções para entrar na lista, sair da lista e usar a lista em >>http://www.mat.puc-rio.br/~nicolau/olimp/obm-l.html >>O administrador desta lista é >>========================================================================= >> > >TRANSIRE SVVM PECTVS MVNDOQUE POTIRE >CONGREGATI EX TOTO ORBE MATHEMATICI OB SCRIPTA INSIGNIA TRIBVERE >Medalha Fields(John Charles Fields) > > >------------------------------------------ >Use o melhor sistema de busca da Internet >Radar UOL - http://www.radaruol.com.br > > > >========================================================================= >Instruções para entrar na lista, sair da lista e usar a lista em >http://www.mat.puc-rio.br/~nicolau/olimp/obm-l.html >O administrador desta lista é >========================================================================= > > --------------020307020809070003070805 Content-Type: text/html; charset=us-ascii Content-Transfer-Encoding: 7bit Você é aluno do Glenn? já tenho a expressao fatorada



peterdirichlet@zipmail.com.br wrote:
Vou especificar mais:
Basta voce achar as raizes de que eu estava falando,e depois agrupar tudo
convenientemente(vao aparecer pares de conjugados,que sao legais p/produtar).
Te mais!!!!!!!!!!!!!!Peterdirichlet

-- Mensagem original --

obrigado pela resolução, mas eu queria saber se é possivel fatorar apenas
em
IR.....

----- Original Message -----
From: <peterdirichlet@zipmail.com.br>
To: <obm-l@mat.puc-rio.br>
Sent: Monday, June 10, 2002 4:33 PM
Subject: [obm-l] Re: [obm-l] Desafio o retorno!!


Vou usar complexos(a paixao de JP):
Seja a expressao f(x)=x^2+x+1.Vamos fatora-la em R+Ri,i^2+1=0. Defina
cis
x=sen x+i*cos x=e^(ix).
Entao w=(cis(2*pi/3)) e wbarra=(cis(4*pi)/3) sao zeros de f.
Para as raizes de x^6+x^3+1,ache as raizes cubicas de w e wbarra.
Te mais!!!!!!!!!!!!!!!!


-- Mensagem original --

Olá amigos,
Estou de volta com outro exercício(+difícil na minha opinião)
Meu professor disse que nem ele consegue fatorar a seguinte expressão
em
IR:
x^6 + (xy)^3 + y^6

Abraços,
Bruno

TRANSIRE SVVM PECTVS MVNDOQUE POTIRE
CONGREGATI EX TOTO ORBE MATHEMATICI OB SCRIPTA INSIGNIA TRIBVERE
Medalha Fields(John Charles Fields)


------------------------------------------
Use o melhor sistema de busca da Internet
Radar UOL - http://www.radaruol.com.br



=========================================================================
Instruções para entrar na lista, sair da lista e usar a lista em
http://www.mat.puc-rio.br/~nicolau/olimp/obm-l.html
O administrador desta lista é <nicolau@mat.puc-rio.br>
=========================================================================

=========================================================================
Instruções para entrar na lista, sair da lista e usar a lista em
http://www.mat.puc-rio.br/~nicolau/olimp/obm-l.html
O administrador desta lista é <nicolau@mat.puc-rio.br>
=========================================================================


TRANSIRE SVVM PECTVS MVNDOQUE POTIRE
CONGREGATI EX TOTO ORBE MATHEMATICI OB SCRIPTA INSIGNIA TRIBVERE
Medalha Fields(John Charles Fields)


------------------------------------------
Use o melhor sistema de busca da Internet
Radar UOL - http://www.radaruol.com.br



=========================================================================
Instruções para entrar na lista, sair da lista e usar a lista em
http://www.mat.puc-rio.br/~nicolau/olimp/obm-l.html
O administrador desta lista é <nicolau@mat.puc-rio.br>
=========================================================================



--------------020307020809070003070805-- ========================================================================= Instruções para entrar na lista, sair da lista e usar a lista em http://www.mat.puc-rio.br/~nicolau/olimp/obm-l.html O administrador desta lista é ========================================================================= From owner-obm-l@sucuri.mat.puc-rio.br Mon Jun 17 17:41:08 2002 Return-Path: Received: (from majordom@localhost) by sucuri.mat.puc-rio.br (8.9.3/8.9.3) id RAA02976 for obm-l-list; Mon, 17 Jun 2002 17:40:23 -0300 Received: from web10105.mail.yahoo.com (web10105.mail.yahoo.com [216.136.130.55]) by sucuri.mat.puc-rio.br (8.9.3/8.9.3) with SMTP id RAA02972 for ; Mon, 17 Jun 2002 17:40:19 -0300 Message-ID: <20020617204000.44151.qmail@web10105.mail.yahoo.com> Received: from [150.161.4.42] by web10105.mail.yahoo.com via HTTP; Mon, 17 Jun 2002 13:39:59 PDT Date: Mon, 17 Jun 2002 13:39:59 -0700 (PDT) From: Rafael WC Subject: Re: [obm-l] 3 circulos! To: obm-l@mat.puc-rio.br In-Reply-To: <3D0A2A7100001DE4@www.zipmail.com.br> MIME-Version: 1.0 Content-Type: multipart/mixed; boundary="0-1584873083-1024346399=:42358" Sender: owner-obm-l@sucuri.mat.puc-rio.br Precedence: bulk Reply-To: obm-l@mat.puc-rio.br --0-1584873083-1024346399=:42358 Content-Type: text/plain; charset=us-ascii Content-Disposition: inline Oi Peter! Valeu pela dica. Vou tentar fazer com isso pra ver se fica melhor porque ficou meio comprida a minha resolução. Veja como acabei resolvendo depois de quase fundir a cuca: >Os raios dos circulos inscritos num triangulo >retangulo ABC e nos dois triangulos ABH e ACH >determinados pela altura relativa à hipotenusa BC são >respectivamente r ,r1 e r2. Demonstrar que: >r² = r1² + r2² Estou mandando a figura (circinsexinscsemel1.gif) que não é muito boa, mas é porque são muitos dados. Fiz o triângulo ABC, retângulo em A, tracei a altura AH e as 3 circunferências em questão: - a de centro O inscrita em ABC e com raio r; - a de centro O1 inscrita em ABH e com raio r1; - a de centro O2 inscrita em ACH e com raio r2. Na cricunferência de centro O, tracei 3 raios nos pontos de tangência com os lados AB, AC e BC, tocando-os nos pontos z. Chamei esses 3 pontos de z, porque você pode ver que no lado AB o segmento Bz é congruente ao segmento Bz do lado BC. Assim como o segmento Cz do lado AC é congruente a Cz do lado BC. Na circunferência O1 tracei dois raios nos pontos de tangência dos lados AB e BC, tocando-os nos pontos x. Chamei os dois de x, porque você pode ver que o segmento Bx do lado AB é congruente ao segmento Bx do lado BC. E por fim, na circunferência O2 tracei dois raios nos pontos de tangência dos lados AC e BC, tocando-os nos pontos y. Chamei os dois de y, porque você pode ver que o segmento Cy do lado AC é congruente ao segmento Cy do lado BC. Agora vamos precisar de 4 semelhanças de triângulos. Primeiro vamos ver que os triângulos AxO1 é semelhante ao triângulo CyO2, pois os dois triângulos são retângulos e o ângulo yCO2 mede C/2, assim como o ângulo xAO1 que é metade do ângulo BAH (AO1 é bissetriz). E o ângulo BAH é congruente ao ângulo C, pois BAH é um triângulo retângulo. Diante dessa semelhança, podemos escrever: xO1/Ax = yO2/Cy r1/(AB - Bx) = r2/Cy E isolando AB: r1/(AB - Bx) = r2/Cy r1.Cy = r2.(AB - Bx) r1.Cy = r2.AB - r2.Bx r1.Cy + r2.Bx = r2.AB AB = (r1.Cy + r2.Bx)/r2 Agora vamos ver a semelhança entre os triângulos AyO2 e BxO1. Você sabe que o ângulo xBO1 mede B/2 e como CHA é retângulo, o ângulo CAH mede B, pois é o complementar de C. Então, sendo AO2 bissetriz do ângulo CAH, yAO2 mede B/2 também. SEndo os dois triângulos retângulos eles são semelhantes. E podemos escrever: yO2/Ay = xO1/Bx r2/(AC - Cy) = r1/Bx E isolando AC: r2/(AC - Cy) = r1/Bx r2.Bx = r1.(AC - Cy) r2.Bx = r1.AC - r1.Cy r2.Bx + r1.Cy = r1.AC AC = (r2.Bx + r1.Cy)/r1 Agora vamos ver a semelhança entre os triângulos CyO2 e CzO. Você pode ver que yO2 e Oz são paralelas então os dois triângulos são semelhantes. Podemos escrever as razões: yO2/Cy = Oz/Cz r2/Cy = r/Cz E isolando Cy: r2/Cy = r/Cz Cy = r2.Cz/r E a última semelhança é entre os triângulos BxO1 e BzO, onde você que xO1 e Oz são paralelas. Então podemos escrever as razões: xO1/Bx = Oz/Bz r1/Bx = r/Bz E isolando Bx: r1/Bx = r/Bz Bx = r1.Bz/r Agora vamos colocar o valor de Cy e Bx nas equações de AB e AC: AB = (r1.Cy + r2.Bx)/r2 AB = (r1.r2.Cz/r + r2.r1.Bz/r)/r2, simplifica r2, AB = r1.Cz/r + r1.Bz/r, coloca r1/r em evidência, AB = (r1/r).(Cz + Bz) AC = (r2.Bx + r1.Cy)/r1 AC = (r2.r1.Bz/r + r1.r2.Cz/r)/r1, simplifica r1, AC = r2.Bz/r + r2.Cz/r, coloca r2/r em evidência, AC = (r2/r).(Bz + Cz) Nas duas fórmulas aparece Bz + Cz, que se você olhar na base BC da figura, verá que a soma desses dois segmentos dá exatamente BC, então AC e AB ficam: AB = (r1/r).(Cz + Bz) AB = (r1/r).BC AC = (r2/r).(Bz + Cz) AC = (r2/r).BC E como o triângulo ABC é retângulo, podemos escrever o teorema de Pitágoras e substituir os valores de AB e AC: AB² + AC² = BC² [(r1/r).BC]² + [(r2/r).BC]² = BC² (r1/r)².BC² + (r2/r)².BC² = BC², simplifica BC², (r1/r)² + (r2/r)² = 1 r1²/r² + r2²/r² = 1, tira o mínimo, r1²/r² + r2²/r² = r²/r² r1² + r2² = r² Como queríamos demonstrar. --- peterdirichlet@zipmail.com.br wrote: > Caro Rafael, > agora eu tive uma ideia que pode ajudar.Se voce > fizer AB=c AC=b, voce consegue > achar AH,BC,BH e HC em funçao de b e > c(dica:trigonometria).E,com as ditas > medidas voce consegue achar os inraios que voce > quer.E e so substituir e > comemorar!!!!! > Peterdirichlet > PS.:estes tipos de exercicio sao bem comuns em > olimpiadas! ===== Rafael Werneck Cinoto ICQ# 107011599 rwcinoto@yahoo.com matduvidas@yahoo.com.br http://www.rwcinoto.hpg.com.br/ __________________________________________________ Do You Yahoo!? Yahoo! - Official partner of 2002 FIFA World Cup http://fifaworldcup.yahoo.com --0-1584873083-1024346399=:42358 Content-Type: image/gif; name="circinsexinscsemel1.gif" Content-Transfer-Encoding: base64 Content-Description: circinsexinscsemel1.gif Content-Disposition: inline; filename="circinsexinscsemel1.gif" R0lGODdhigHrAPcAAAAAAAAAQAAAgAAA/wAgAAAgQAAggAAg/wBAAABAQABA gABA/wBgAABgQABggABg/wCAAACAQACAgACA/wCgAACgQACggACg/wDAAADA QADAgADA/wD/AAD/QAD/gAD//yAAACAAQCAAgCAA/yAgACAgQCAggCAg/yBA ACBAQCBAgCBA/yBgACBgQCBggCBg/yCAACCAQCCAgCCA/yCgACCgQCCggCCg /yDAACDAQCDAgCDA/yD/ACD/QCD/gCD//0AAAEAAQEAAgEAA/0AgAEAgQEAg gEAg/0BAAEBAQEBAgEBA/0BgAEBgQEBggEBg/0CAAECAQECAgECA/0CgAECg QECggECg/0DAAEDAQEDAgEDA/0D/AED/QED/gED//2AAAGAAQGAAgGAA/2Ag AGAgQGAggGAg/2BAAGBAQGBAgGBA/2BgAGBgQGBggGBg/2CAAGCAQGCAgGCA /2CgAGCgQGCggGCg/2DAAGDAQGDAgGDA/2D/AGD/QGD/gGD//4AAAIAAQIAA gIAA/4AgAIAgQIAggIAg/4BAAIBAQIBAgIBA/4BgAIBgQIBggIBg/4CAAICA QICAgICA/4CgAICgQICggICg/4DAAIDAQIDAgIDA/4D/AID/QID/gID//6AA AKAAQKAAgKAA/6AgAKAgQKAggKAg/6BAAKBAQKBAgKBA/6BgAKBgQKBggKBg /6CAAKCAQKCAgKCA/6CgAKCgQKCggKCg/6DAAKDAQKDAgKDA/6D/AKD/QKD/ gKD//8AAAMAAQMAAgMAA/8AgAMAgQMAggMAg/8BAAMBAQMBAgMBA/8BgAMBg QMBggMBg/8CAAMCAQMCAgMCA/8CgAMCgQMCggMCg/8DAAMDAQMDAgMDA/8D/ AMD/QMD/gMD///8AAP8AQP8AgP8A//8gAP8gQP8ggP8g//9AAP9AQP9AgP9A //9gAP9gQP9ggP9g//+AAP+AQP+AgP+A//+gAP+gQP+ggP+g///AAP/AQP/A gP/A////AP//QP//gP///yH5BAAAAAAALAAAAACKAesAAAj3AP8JHEiwoMGD CBMqXMiwocOHECNKnEixosWLGDNq3Mixo8ePIEOKHEmypMmTKFOqXMmypcuX MGPKnBkSAM2bOHPq3MnzoE2bPYMKHUq0aMWfRpMqXcpUJ9KmUKNKndoR6D+r VLNq3cr16kCsXcOKHcsTgNmzYMmqXcsWJdi0bePKnWsRLly6ePPqJWh3r9+/ gAMLHky4sOHDiBMrXsy4sePHkCNLnky5suXLmLvezcy5c8KznkOLNoh08+jT k3+a9Yq6dWWgoFm7nu0Y9lWrpmnrBox7Ne7dwAn/9s03uHG/WFULTH68+Vzm t5cXzO28+tS3y21Pt//OnSv27F9Jdx8vNW3v8OLJq09qnu9vn+vjC+37FXp6 +fhv0pdu/37+/x89pdBdzPXnH4AIZrTaQgRu9x1C1HkVYYL5UbcZdg9CyNCC FHa43YCfHYheiCB6aOKI8JFYXIolanjiiRe2uKKILNL4IoANyojihwy6eOOP 0um444w92ggkhRbWaOSSR5qYpJJMEtnkjRHmqGJDE04pX5UkWulQllqSN2GM skXJY5gIjlkimRCBiWZzWZqmnY8PufkmcHGShtaefM7Z5p3xgdnnhwTuGZGd gKI2Zmw+5gYbo0UmWp2jsTk65JnZcUinpMFdqGmQKlr62adSckpboZuy+OQ7 gF6a6ppdcm54KaajQumqaOaJGumqWLZ362lvcSkrqEIymOGvnCWnJpaz0tpr qcgOZuiw0dX5ZbPOPkv/bLSGCUtqpOAuS9GCiHLb1pOIiltmsYduOyqk5oaF 7kR5Ypvtn+uCG69msn7L7LX2QisRcQDvK2+P5eYrY70KKmymwUxN62Bd+LrL Lr0OBwyxvP4W7LGgHMF778ZidWxtxRnbepG/EpNcsskef9xuVQm7fJ3IA+es scArw2wzxz7r+y/FNAf9c1ZopUztyUSHjPPRmh3LNMrjVnWb0VA39W26GGNU M6ZYZ80emzFPfZRGsIq9lYAqp1p21ytr+LXaNLHtdttLN111iHPTDZPdVwZO 9dmEg9i33ywBfrHFZit999AII84e44LznDflQkNe5OGSl+Rn45hf7vjjla/Z iHlQnw8+euXlusn16qevlPrMloueMMM6cx77Ru/p/fV+OovOtO67xx368MaP LHztQxNfPNyw9+s13ssfD7nzz09NfNhu1yxnn+CbbGD2nis/cKy6Mq9vy73C KzX5NZmfs6eZR6/q7IcaiD38tG4/K6/qWxP+zjek/fEvX84bn+NalT8ODadn IjP/IPmA17D+Ca91c1KW0x52wLclUEkA3Fl9oPM+CCqwg8GTn+qIFULr/c9X IUORuCRoLgZOr3R3WxTrAlQfjfWug2Sr4OKyRSn0cZBqoOESDaMVRCEO8X9P TN3++pKkJf4qVjF8272aeB8rMk5xxfGiqSwVPvbhkHpc9FkJPeKp9MGvjUbj E/XwBrLLifF7T5RcrshVF029bo7Mo6BIjEi6ff1wU6TqG8GgF8AFnjGLfEuk 2MBIJJbxznuAdOQjebehpIFnkko7TyHbxb1SCWuId+xkKQ25OTcmb5XrolT1 RJi8d4kRSJQcYCPtuDfMnbJ+bGzlLankMOVoTm89zF0e/+s4EiwmU22HlBDo epnJiUGohXl0IrRguTFKVnOW6aJfNneJzC/ar5ve/OYyGRkeZ9JukEtK5888 ucIUjvOZ6OGiFhvYqip2jp7TfOcsxWMfGwZUezVS0zBhdDV7UvOYpgOVIB1a TtnUa6EdEhAza7nPLkr0iJuUGxGjF02X2Q13hRPolfyowoFe00alLCnEFKeu lKpURP3R3R+btVOM4gd/v3zoTcFmUVpCNG/iTKFP16NLbNbzS+58jy5tej2+ pXSp45lqKDlKVRmG8ZwdbVSXbIpV68h0ZL9D20qnw02XEqplZ+VlDQOoz7DW 85AADSYkwVpAbolSi5jkpKpG+Ac0tQrWqM4q/yue3HPT21ktRao5T1vvKbS0 gpSYiHWXYx/7Vv9MdpQQtexlndTSA7WOh2y1JFs521W+8lRSa5QZOw3b0GK+ lLZCzazlFDsaGBZup0WzjUHxWdGOzs2dU/It0YCLNkiBsUrHzS050RimiVbN Tttz4FdBGEHRHtSFwATvf4ZLL+xe0rNkU2BgvyveTfLWMuTV3kaP0rG/erS0 RwUt55yappAaq7RWtC+PlKtN/+43v+OlLEFXi2BI1revrPWvax/23se4kll4 HZ3rYsqfUHGwpt88sF3F5FYGb2uqceKmcD0M4KDiV8Qj5g5/dfQg5PrksxJy JoE7CUzzHrbBMgYyIv/vyyQctzOIJ9SWyiKrrE/576lmjbFpPbtkI4exiXkt b6rY5qckS1fBp2LvkIlqSisTtK7nJSqBvTzbEutmvmZb84wKy8kHe67GmqXy XqG8WD6XU6s8tLNJKvVRC7ZXyKCdjY9BAiuMGlOdzXXulHULZkonq824zZSj c5k4SZ8ptl+WMq5CfcOGZvnUCuK07OgJwx1zlaKKbu0lm5whDXpN1avWMwJR ++rWrDe4Nza0PJ+FZpI0ujeexN7hKswW76ZaplJjM1QzhtVf/7jXoYnus20V 7ekGO77NRCWj9zzqUtO3WN22dA/BDU9WSXiossbMsstnSl330bYtge5oRc0z 79qQW68QruT0nuuSFHu734i2cITfbW+vHny71h10/iYcXmxL5oOs8m6hJNmz S1Vby+OO/5+8eZ3aw27a4wXHmJnV7ebDTFFPAD+5xT4evJUvkdlLkeCjDAho q5WU5oOzcoBT08yVQ1YlXN53po/62VRCRudGt2pKkv7wAoPT2uH2N72NTXFl 7ghDEoe31bO+GKffeeosBJvZEe5ngCNGpy2PN69rjV+LQ7rtJC/MsoPmxaiP 9dN1L26iHwl04dx6w1zvO8pP7BYx83jwIZdWHzFI8p6/0kWgXjjkH1/1ayNn XDjO7opuLrDP3RLEt+685/OCXTPDvaBkp3bXocrSuNdp2CfBeb4n7vbrOvzQ KIvR0POZ8HPXbS/M/l1OA895yvl9c7Yvb/JZjxNr66/3vgz3d/9mTNvnw5os F+Y6Wev9bxvTLD3h77338U6VNu7+oepdvSxFPudKw3r9bM/5jXl72vpvPof/ RX9p93/xhn/RNznugXPAFVfOVju4Bz0ltHZlAX6M5RT/9nDmt11zRyOnRxS6 l0V0JhMwxnChIyovdzWExnykRoBLVxZmlIAfSIL6JXYlyHL21IGglzgeSEQh mBOKNG1Clj44+Elhd29INxSZFz4TaHfmg3oCl3s2SIO3d4Soo4JKGBNt5Tq1 YlxK53WNt0Ff6IJZ9mwvuHXft4W7dGF9Z4AqaCwSWH1t2CbgY2zMVTDco4ZF GIY1oXg7kXlFNyhg6Hj7h2lfRENsqHorUyh+PhiHewiIOVh8qVWHu8VoD9iC RaeI+sGIU+eISiaDfqSF4haIQ4h0hv/og2U4NpwYiZD3NFiTflHoZFA4FjHY bHNIWAx4ihVYcS7Ge4iYf0Yxi6xXRsJ4O664OqukWpfYbGmSiqlXSHDmbjIo hd6xjA5Ui4/ogGdIXHfHfvmGcRXSg1fYPvYSTo3mixlXFDaHI95jjXoCM4E1 hvYXjy/hevFijXKkdPAIifpYfSM4U8O4SKrEKJu1ejk3bzbDav8YjoMojeKo FaE3T3CkkEBojtXCMeYWZcA3SIXFjtfkjvR1iH/TgM5RibnXesxYW5fVgxYp d7tBkuM3aHMoMdClkokBjM8Ri1E4eQl5j5cWZXyYk07DkeU2krOnORsHkprW jjzZLQzZW0CEaUsh6FNhI5F6MZVXpDw2GZC4KIsolSixlZUfuZVI44RoskZg GVxiWZD76CFmyRtCqRRqxCkRmI+RZ5NUiTrFiEItcpZKSZcimIF6KT1jeZIF V2yBCR98aUJpuYcsCERFGTFvSYnb6Jh6R5ip1oVvVBuRSUC9uDuJKYZLmYgZ mZm9tZnPSvSZP4KUgSGUfHSY1DgoqOmaguGXsgkjqlmblxFXuKkefribGOmb GQWcbCmcwUmcCdKbxmkcupmck3KbzPmc0Bmd0jmd1FmdGxEQADs= --0-1584873083-1024346399=:42358-- ========================================================================= Instruções para entrar na lista, sair da lista e usar a lista em http://www.mat.puc-rio.br/~nicolau/olimp/obm-l.html O administrador desta lista é ========================================================================= From owner-obm-l@sucuri.mat.puc-rio.br Mon Jun 17 19:52:20 2002 Return-Path: Received: (from majordom@localhost) by sucuri.mat.puc-rio.br (8.9.3/8.9.3) id TAA05149 for obm-l-list; Mon, 17 Jun 2002 19:51:54 -0300 Received: from gorgo.centroin.com.br (gorgo.centroin.com.br [200.225.63.128]) by sucuri.mat.puc-rio.br (8.9.3/8.9.3) with ESMTP id TAA05145 for ; Mon, 17 Jun 2002 19:51:52 -0300 Received: from centroin.com.br (du167c.rjo.centroin.com.br [200.225.58.167]) (authenticated bits=0) by gorgo.centroin.com.br (8.12.2/8.12.1) with ESMTP id g5HMq6ko001679 for ; Mon, 17 Jun 2002 19:52:06 -0300 (BRT) Message-ID: <3D0E6864.7030803@centroin.com.br> Date: Mon, 17 Jun 2002 19:53:24 -0300 From: Augusto =?ISO-8859-1?Q?C=E9sar?= Morgado User-Agent: Mozilla/5.0 (Windows; U; Win98; en-US; rv:0.9.4.1) Gecko/20020508 Netscape6/6.2.3 X-Accept-Language: en-us MIME-Version: 1.0 To: obm-l@mat.puc-rio.br Subject: [Fwd: Re: [obm-l] Re: [obm-l] Re: [obm-l] Re:Antiga =?ISO-8859-1?Q?fatora=E7=E3o=5D?= Content-Type: multipart/alternative; boundary="------------020105010407060106020605" Sender: owner-obm-l@sucuri.mat.puc-rio.br Precedence: bulk Reply-To: obm-l@mat.puc-rio.br --------------020105010407060106020605 Content-Type: text/plain; charset=ISO-8859-1; format=flowed Content-Transfer-Encoding: 8bit -------- Original Message -------- From: - Sun Jun 16 19:22:11 2002 X-Mozilla-Status: 0001 X-Mozilla-Status2: 00000000 Message-ID: <3D0D0F7F.20208@centroin.com.br> Date: Sun, 16 Jun 2002 19:21:51 -0300 From: Augusto César Morgado User-Agent: Mozilla/5.0 (Windows; U; Win98; en-US; rv:0.9.4.1) Gecko/20020508 Netscape6/6.2.3 X-Accept-Language: en-us MIME-Version: 1.0 To: luizhenriquerick@zipmail.com.br Subject: Re: [obm-l] Re: [obm-l] Re: [obm-l] Re:Antiga fatoração References: <3D0CECDA00000278@www.zipmail.com.br> Content-Type: text/plain; charset=ISO-8859-1; format=flowed Content-Transfer-Encoding: 8bit luizhenriquerick@zipmail.com.br wrote: >Po , essa deve ser braba então. >Sera que alguém mais experiente ae não poderia dar uma opinião ? >Tipo Morgado , Ponce .. etc > > ---------------------------------------- > |-=Rick-C.R.B.=- | > |ICQ 124805654 | > |e-mail luizhenriquerick@zipmail.com.br | > ---------------------------------------- > > >------------------------------------------ >Use o melhor sistema de busca da Internet >Radar UOL - http://www.radaruol.com.br > > > >========================================================================= >Instruções para entrar na lista, sair da lista e usar a lista em >http://www.mat.puc-rio.br/~nicolau/olimp/obm-l.html >O administrador desta lista é >========================================================================= > >x^6 +y^6 + x^3.y^3 = (x^9-y^9)/(x^3-y^3) > x^9-y^9 = Produtorio de (x-ycis 2kpi/9) com k variando de 0 a 8 Os fatores correspondentes a k = 0, 3 e 6, multiplicados dao x^3-y^3. Logo, x^6 +y^6 + x^3.y^3 = Produtorio de (x-ycis 2kpi/9) com k= 1,2,4, 5,7, 8 . Estah fatorado como um produto de 6 fatores complexos de primeiro grau. Grupando os fatores 1-8, 2-7 e 4-5, obtem-se uma fatoraçao em tres fatores reais de grau 2. > > --------------020105010407060106020605 Content-Type: text/html; charset=us-ascii Content-Transfer-Encoding: 7bit

-------- Original Message --------
From: - Sun Jun 16 19:22:11 2002
X-Mozilla-Status: 0001
X-Mozilla-Status2: 00000000
Message-ID: <3D0D0F7F.20208@centroin.com.br>
Date: Sun, 16 Jun 2002 19:21:51 -0300
From: Augusto César Morgado <morgado@centroin.com.br>
User-Agent: Mozilla/5.0 (Windows; U; Win98; en-US; rv:0.9.4.1) Gecko/20020508 Netscape6/6.2.3
X-Accept-Language: en-us
MIME-Version: 1.0
To: luizhenriquerick@zipmail.com.br
Subject: Re: [obm-l] Re: [obm-l] Re: [obm-l] Re:Antiga fatoração
References: <3D0CECDA00000278@www.zipmail.com.br>
Content-Type: text/plain; charset=ISO-8859-1; format=flowed
Content-Transfer-Encoding: 8bit


luizhenriquerick@zipmail.com.br wrote:

>Po , essa deve ser braba então.
>Sera que alguém mais experiente ae não poderia dar uma opinião ?
>Tipo Morgado , Ponce .. etc
>
>  ----------------------------------------
> |-=Rick-C.R.B.=-                         |
> |ICQ 124805654                           |
> |e-mail luizhenriquerick@zipmail.com.br  |
>  ----------------------------------------
>
>
>------------------------------------------
>Use o melhor sistema de busca da Internet
>Radar UOL - http://www.radaruol.com.br
>
>
>
>=========================================================================
>Instruções para entrar na lista, sa
ir da lista e usar a lista em
>http://www.mat.puc-rio.br/~nicolau/olimp/obm-l.html
>O administrador desta lista é 
>=========================================================================
>
>x^6 +y^6 + x^3.y^3 = (x^9-y^9)/(x^3-y^3)
>
x^9-y^9 = Produtorio de (x-ycis 2kpi/9) com k variando de 0 a 8
Os fatores correspondentes a  k = 0, 3 e 6, multiplicados dao x^3-y^3.
Logo, 

x^6 +y^6 + x^3.y^3 = Produtorio de (x-ycis 2kpi/9) com k= 1,2,4, 5,7, 8 . Estah fatorado como um produto de 6 fatores complexos de primeiro grau.
Grupando os fatores  1-8, 2-7 e 4-5, obtem-se uma fatoraçao em tres fatores reais de grau 2.


> 
>


--------------020105010407060106020605-- ========================================================================= Instruções para entrar na lista, sair da lista e usar a lista em http://www.mat.puc-rio.br/~nicolau/olimp/obm-l.html O administrador desta lista é ========================================================================= From owner-obm-l@sucuri.mat.puc-rio.br Tue Jun 18 13:33:14 2002 Return-Path: Received: (from majordom@localhost) by sucuri.mat.puc-rio.br (8.9.3/8.9.3) id NAA13970 for obm-l-list; Tue, 18 Jun 2002 13:31:10 -0300 Received: from www.zipmail.com.br (smtp.zipmail.com.br [200.187.242.10]) by sucuri.mat.puc-rio.br (8.9.3/8.9.3) with ESMTP id NAA13966 for ; Tue, 18 Jun 2002 13:31:08 -0300 From: peterdirichlet@zipmail.com.br Received: from [200.206.103.3] by www.zipmail.com.br with HTTP; Tue, 18 Jun 2002 13:30:49 -0300 Message-ID: <3D0E42CF00001215@www.zipmail.com.br> Date: Tue, 18 Jun 2002 13:30:49 -0300 In-Reply-To: <20020617204000.44151.qmail@web10105.mail.yahoo.com> Subject: [obm-l] =?iso-8859-1?Q?De=20novo=20Re=3A=20=5Bobm=2Dl=5D=203=20circulos=21=28gif=29?= To: obm-l@mat.puc-rio.br MIME-Version: 1.0 Content-Type: text/plain; charset="iso-8859-1" Content-Transfer-Encoding: 8bit X-MIME-Autoconverted: from quoted-printable to 8bit by sucuri.mat.puc-rio.br id NAA13967 Sender: owner-obm-l@sucuri.mat.puc-rio.br Precedence: bulk Reply-To: obm-l@mat.puc-rio.br Meu Deus do ceu,geometria cearense na cara de pau!!!!!!!!!!!Por favor meu,o troço ficou bem "soluçao oficial de problema de geometria da IMO".Mas,uma coisa:como voce desenha em gif?Eu nao faço ideia de como usar isso e eu estou precisando urgentemente disso para um artigo da EUREKA que estou fazendo. VALEU!!!!! O jovem de Richelet -- Mensagem original -- >Oi Peter! > >Valeu pela dica. Vou tentar fazer com isso pra ver se >fica melhor porque ficou meio comprida a minha >resolução. Veja como acabei resolvendo depois de quase >fundir a cuca: TRANSIRE SVVM PECTVS MVNDOQUE POTIRE CONGREGATI EX TOTO ORBE MATHEMATICI OB SCRIPTA INSIGNIA TRIBVERE Medalha Fields(John Charles Fields) ------------------------------------------ Use o melhor sistema de busca da Internet Radar UOL - http://www.radaruol.com.br ========================================================================= Instruções para entrar na lista, sair da lista e usar a lista em http://www.mat.puc-rio.br/~nicolau/olimp/obm-l.html O administrador desta lista é ========================================================================= From owner-obm-l@sucuri.mat.puc-rio.br Tue Jun 18 15:31:46 2002 Return-Path: Received: (from majordom@localhost) by sucuri.mat.puc-rio.br (8.9.3/8.9.3) id PAA15882 for obm-l-list; Tue, 18 Jun 2002 15:30:02 -0300 Received: from matinhos.terra.com.br (matinhos.terra.com.br [200.176.3.21]) by sucuri.mat.puc-rio.br (8.9.3/8.9.3) with ESMTP id PAA15868 for ; Tue, 18 Jun 2002 15:29:54 -0300 Received: from taipe.terra.com.br (taipe.terra.com.br [200.176.3.34]) by matinhos.terra.com.br (Postfix) with ESMTP id EC9C547097 for ; Tue, 18 Jun 2002 15:29:09 -0300 (EST) Received: from xt (200-171-248-201.customer.telesp.net.br [200.171.248.201]) (authenticated user macwad) by taipe.terra.com.br (Postfix) with ESMTP id 664371B4080 for ; Tue, 18 Jun 2002 15:29:08 -0300 (EST) Message-ID: <004b01c216f6$214f0d70$0401010a@xt> From: =?iso-8859-1?Q?Vinicius_Jos=E9_Fortuna?= To: References: <3D0E42CF00001215@www.zipmail.com.br> Subject: [obm-l] Axiomas de Peano Date: Tue, 18 Jun 2002 15:29:47 -0300 MIME-Version: 1.0 Content-Type: text/plain; charset="iso-8859-1" Content-Transfer-Encoding: 8bit X-Priority: 3 X-MSMail-Priority: Normal X-Mailer: Microsoft Outlook Express 6.00.2600.0000 X-MimeOLE: Produced By Microsoft MimeOLE V6.00.2600.0000 Sender: owner-obm-l@sucuri.mat.puc-rio.br Precedence: bulk Reply-To: obm-l@mat.puc-rio.br Na Eureka 3, p. 26, há um artigo de Elon Lages Lima chamado "O Princípio da Indução", onde o autor afirma que o conjunto N dos números naturais é caracterizado pelas seguintes propriedades: A) Existe função s: N -> N, que associa a cada n pertencente a N um elemento s(n) pertecente a N, chamado o sucessor de n. B) A função s: N-> N é injetiva. C) Existe um único elemento 1 no conjunto N, tal que 1 != s(n) para todo n pertencente a N. D) Se um subconjunto X contido em N é tal que 1 pertence a N e s(X) está contido em X. As afirmações A, B, C e D são os axiomas de Peano. Agora vem a minha dúvida. Imagine o conjunto de números: V = {0, 1, 2, 3, ...} U {a}, onde o elemento 'a' não pertence a {0, 1, 2, 3, ...} e a função injetiva s: V -> V onde: s(x) = a, se x=a; senão s(x) = x+1 Temos, então, o conjunto V e a função s que satisfazem os axiomas de Peano. Dessa forma, podemos dizer que V é o conjunto dos número naturais, mas não é!!!!! Qual o problema aí??? Alguém pode esclarecer a minha dúvida? Obrigado Vinicius Fortuna ========================================================================= Instruções para entrar na lista, sair da lista e usar a lista em http://www.mat.puc-rio.br/~nicolau/olimp/obm-l.html O administrador desta lista é ========================================================================= From owner-obm-l@sucuri.mat.puc-rio.br Tue Jun 18 16:06:49 2002 Return-Path: Received: (from majordom@localhost) by sucuri.mat.puc-rio.br (8.9.3/8.9.3) id QAA16756 for obm-l-list; Tue, 18 Jun 2002 16:05:43 -0300 Received: from candeias.terra.com.br (candeias.terra.com.br [200.176.3.18]) by sucuri.mat.puc-rio.br (8.9.3/8.9.3) with ESMTP id QAA16752 for ; Tue, 18 Jun 2002 16:05:36 -0300 Received: from engenho.terra.com.br (engenho.terra.com.br [200.176.3.42]) by candeias.terra.com.br (Postfix) with ESMTP id 0BCEB43FAF for ; Tue, 18 Jun 2002 16:05:13 -0300 (EST) Received: from Itautec.terra.com.br (200-158-60-113.dsl.telesp.net.br [200.158.60.113]) (authenticated user bruleite) by engenho.terra.com.br (Postfix) with ESMTP id 5E3FE68204 for ; Tue, 18 Jun 2002 16:05:12 -0300 (EST) Message-Id: <5.1.0.14.0.20020618160028.00a45500@pop.sao.terra.com.br> X-Sender: bruleite@pop.sao.terra.com.br X-Mailer: QUALCOMM Windows Eudora Version 5.1 Date: Tue, 18 Jun 2002 16:03:38 -0300 To: obm-l@mat.puc-rio.br From: "Bruno F. C. Leite" Subject: Re: [obm-l] Axiomas de Peano In-Reply-To: <004b01c216f6$214f0d70$0401010a@xt> References: <3D0E42CF00001215@www.zipmail.com.br> Mime-Version: 1.0 Content-Type: text/plain; charset="iso-8859-1"; format=flowed Content-Transfer-Encoding: 8bit X-MIME-Autoconverted: from quoted-printable to 8bit by sucuri.mat.puc-rio.br id QAA16753 Sender: owner-obm-l@sucuri.mat.puc-rio.br Precedence: bulk Reply-To: obm-l@mat.puc-rio.br At 15:29 18/06/02 -0300, you wrote: >Na Eureka 3, p. 26, há um artigo de Elon Lages Lima chamado "O Princípio da >Indução", onde o autor afirma que o conjunto N dos números naturais é >caracterizado pelas seguintes propriedades: > >A) Existe função s: N -> N, que associa a cada n pertencente a N um elemento >s(n) pertecente a N, chamado o sucessor de n. > >B) A função s: N-> N é injetiva. > >C) Existe um único elemento 1 no conjunto N, tal que 1 != s(n) para todo n >pertencente a N. > >D) Se um subconjunto X contido em N é tal que 1 pertence a N e s(X) está >contido em X. Não me lembro do artigo, mas isto está certo mesmo? Acho que o certo é "se um subconjunto X contido em N é tal que 1 pertence a N e se n está em X implica que s(n) também está, então X=N" (princípio de indução) Com isso o conjunto que você falou (V) não satisfaz a última condição. Bruno Leite http://www.ime.usp.br/~brleite >As afirmações A, B, C e D são os axiomas de Peano. > >Agora vem a minha dúvida. Imagine o conjunto de números: >V = {0, 1, 2, 3, ...} U {a}, onde o elemento 'a' não pertence a {0, 1, 2, 3, >...} >e a função injetiva s: V -> V onde: >s(x) = a, se x=a; senão s(x) = x+1 > >Temos, então, o conjunto V e a função s que satisfazem os axiomas de Peano. >Dessa forma, podemos dizer que V é o conjunto dos número naturais, mas não >é!!!!! >Qual o problema aí??? > >Alguém pode esclarecer a minha dúvida? > >Obrigado > >Vinicius Fortuna > > > > >========================================================================= >Instruções para entrar na lista, sair da lista e usar a lista em >http://www.mat.puc-rio.br/~nicolau/olimp/obm-l.html >O administrador desta lista é >========================================================================= ========================================================================= Instruções para entrar na lista, sair da lista e usar a lista em http://www.mat.puc-rio.br/~nicolau/olimp/obm-l.html O administrador desta lista é ========================================================================= From owner-obm-l@sucuri.mat.puc-rio.br Tue Jun 18 16:27:15 2002 Return-Path: Received: (from majordom@localhost) by sucuri.mat.puc-rio.br (8.9.3/8.9.3) id QAA17203 for obm-l-list; Tue, 18 Jun 2002 16:26:23 -0300 Received: (from nicolau@localhost) by sucuri.mat.puc-rio.br (8.9.3/8.9.3) id QAA17198 for obm-l@mat.puc-rio.br; Tue, 18 Jun 2002 16:26:22 -0300 From: "Nicolau C. Saldanha" Date: Tue, 18 Jun 2002 16:26:22 -0300 To: obm-l@mat.puc-rio.br Subject: Re: [obm-l] Axiomas de Peano Message-ID: <20020618162622.A16977@sucuri.mat.puc-rio.br> References: <3D0E42CF00001215@www.zipmail.com.br> <004b01c216f6$214f0d70$0401010a@xt> Mime-Version: 1.0 Content-Type: text/plain; charset=iso-8859-1 Content-Disposition: inline Content-Transfer-Encoding: 8bit User-Agent: Mutt/1.2.5i F[]rom: "Nicolau C. Saldanha" In-Reply-To: <004b01c216f6$214f0d70$0401010a@xt>; from vinicius.fortuna@ic.unicamp.br on Tue, Jun 18, 2002 at 03:29:47PM -0300 Sender: owner-obm-l@sucuri.mat.puc-rio.br Precedence: bulk Reply-To: obm-l@mat.puc-rio.br On Tue, Jun 18, 2002 at 03:29:47PM -0300, Vinicius José Fortuna wrote: > Na Eureka 3, p. 26, há um artigo de Elon Lages Lima chamado "O Princípio da > Indução", onde o autor afirma que o conjunto N dos números naturais é > caracterizado pelas seguintes propriedades: > > A) Existe função s: N -> N, que associa a cada n pertencente a N um elemento > s(n) pertecente a N, chamado o sucessor de n. > > B) A função s: N-> N é injetiva. > > C) Existe um único elemento 1 no conjunto N, tal que 1 != s(n) para todo n > pertencente a N. > > D) Se um subconjunto X contido em N é tal que 1 pertence a N e s(X) está > contido em X. > > As afirmações A, B, C e D são os axiomas de Peano. A pergunta é o que significa "caracterizado". A resposta é que se A é um conjunto com um elemento especial a1 e f é uma função f: A -> A que satisfazem as seguintes condições: B') f é injetora; C') a imagem de f é A - {a1}; D') para todo subconjunto X de A, se a1 pertence a X e f(X) está contido em X então X = A; então existe uma única função bijetora g: N -> A com g(1) = a1 e g(s(n)) = f(g(n)) para todo N. Podemos pensar em g como uma identificação entre N e A. Assim A não precisa ser o conjunto dos naturais, podemos ter A = Z se definirmos f(n) = -n-1 se n >= 0 = -n se n < 0 > > Agora vem a minha dúvida. Imagine o conjunto de números: > V = {0, 1, 2, 3, ...} U {a}, onde o elemento 'a' não pertence a {0, 1, 2, 3, > ...} > e a função injetiva s: V -> V onde: > s(x) = a, se x=a; senão s(x) = x+1 Este seu exemplo não satisfaz D', tome X = {0,1,2,...}. Temos que 0 pertence a X, s(X) está contido em X mas X != V. > > Temos, então, o conjunto V e a função s que satisfazem os axiomas de Peano. > Dessa forma, podemos dizer que V é o conjunto dos número naturais, mas não > é!!!!! > Qual o problema aí??? > > Alguém pode esclarecer a minha dúvida? Espero que não haja dúvida aqui quanto a se 0 é ou não natural, uma questão puramente de definição/notação. Parece que o próprio Peano em uma versão excluiu 0 (como o Elon fez) e em outra incluiu 0 (como a maioria dos textos modernos de teoria dos conjuntos ou lógica fazem). []s, N. ========================================================================= Instruções para entrar na lista, sair da lista e usar a lista em http://www.mat.puc-rio.br/~nicolau/olimp/obm-l.html O administrador desta lista é ========================================================================= From owner-obm-l@sucuri.mat.puc-rio.br Tue Jun 18 16:38:49 2002 Return-Path: Received: (from majordom@localhost) by sucuri.mat.puc-rio.br (8.9.3/8.9.3) id QAA17667 for obm-l-list; Tue, 18 Jun 2002 16:37:40 -0300 Received: from candeias.terra.com.br (candeias.terra.com.br [200.176.3.18]) by sucuri.mat.puc-rio.br (8.9.3/8.9.3) with ESMTP id QAA17658 for ; Tue, 18 Jun 2002 16:37:37 -0300 Received: from engenho.terra.com.br (engenho.terra.com.br [200.176.3.42]) by candeias.terra.com.br (Postfix) with ESMTP id 531E643F15 for ; Tue, 18 Jun 2002 16:37:22 -0300 (EST) Received: from xt (200-171-248-201.customer.telesp.net.br [200.171.248.201]) (authenticated user macwad) by engenho.terra.com.br (Postfix) with ESMTP id 8CD7B68162 for ; Tue, 18 Jun 2002 16:37:20 -0300 (EST) Message-ID: <008501c216ff$a6f2d980$0401010a@xt> From: =?iso-8859-1?Q?Vinicius_Jos=E9_Fortuna?= To: References: <3D0E42CF00001215@www.zipmail.com.br> <004b01c216f6$214f0d70$0401010a@xt> Subject: Re: [obm-l] Axiomas de Peano Date: Tue, 18 Jun 2002 16:37:56 -0300 MIME-Version: 1.0 Content-Type: text/plain; charset="iso-8859-1" Content-Transfer-Encoding: 8bit X-Priority: 3 X-MSMail-Priority: Normal X-Mailer: Microsoft Outlook Express 6.00.2600.0000 X-MimeOLE: Produced By Microsoft MimeOLE V6.00.2600.0000 Sender: owner-obm-l@sucuri.mat.puc-rio.br Precedence: bulk Reply-To: obm-l@mat.puc-rio.br Ops, faltou uma correção no axioma D. Deveria ser: D) Se um subconjunto X contido em N é tal que 1 pertence a N e s(X) está contido em X então X=N ----- Original Message ----- From: "Vinicius José Fortuna" To: Sent: Tuesday, June 18, 2002 3:29 PM Subject: [obm-l] Axiomas de Peano > Na Eureka 3, p. 26, há um artigo de Elon Lages Lima chamado "O Princípio da > Indução", onde o autor afirma que o conjunto N dos números naturais é > caracterizado pelas seguintes propriedades: > > A) Existe função s: N -> N, que associa a cada n pertencente a N um elemento > s(n) pertecente a N, chamado o sucessor de n. > > B) A função s: N-> N é injetiva. > > C) Existe um único elemento 1 no conjunto N, tal que 1 != s(n) para todo n > pertencente a N. > > D) Se um subconjunto X contido em N é tal que 1 pertence a N e s(X) está > contido em X. > > As afirmações A, B, C e D são os axiomas de Peano. > > Agora vem a minha dúvida. Imagine o conjunto de números: > V = {0, 1, 2, 3, ...} U {a}, onde o elemento 'a' não pertence a {0, 1, 2, 3, > ...} > e a função injetiva s: V -> V onde: > s(x) = a, se x=a; senão s(x) = x+1 > > Temos, então, o conjunto V e a função s que satisfazem os axiomas de Peano. > Dessa forma, podemos dizer que V é o conjunto dos número naturais, mas não > é!!!!! > Qual o problema aí??? > > Alguém pode esclarecer a minha dúvida? > > Obrigado > > Vinicius Fortuna > ========================================================================= Instruções para entrar na lista, sair da lista e usar a lista em http://www.mat.puc-rio.br/~nicolau/olimp/obm-l.html O administrador desta lista é ========================================================================= From owner-obm-l@sucuri.mat.puc-rio.br Tue Jun 18 17:17:27 2002 Return-Path: Received: (from majordom@localhost) by sucuri.mat.puc-rio.br (8.9.3/8.9.3) id RAA18956 for obm-l-list; Tue, 18 Jun 2002 17:17:16 -0300 Received: from www.zipmail.com.br (smtp.zipmail.com.br [200.187.242.10]) by sucuri.mat.puc-rio.br (8.9.3/8.9.3) with ESMTP id RAA18943 for ; Tue, 18 Jun 2002 17:17:13 -0300 From: peterdirichlet@zipmail.com.br Received: from [200.206.103.3] by www.zipmail.com.br with HTTP; Tue, 18 Jun 2002 17:16:54 -0300 Message-ID: <3D0E42CF000018A9@www.zipmail.com.br> Date: Tue, 18 Jun 2002 17:16:54 -0300 In-Reply-To: <004b01c216f6$214f0d70$0401010a@xt> Subject: [obm-l] =?iso-8859-1?Q?Re=3A=20=5Bobm=2Dl=5D=20Axiomas=20de=20Peano?= To: obm-l@mat.puc-rio.br MIME-Version: 1.0 Content-Type: text/plain; charset="iso-8859-1" Content-Transfer-Encoding: 8bit X-MIME-Autoconverted: from quoted-printable to 8bit by sucuri.mat.puc-rio.br id RAA18946 Sender: owner-obm-l@sucuri.mat.puc-rio.br Precedence: bulk Reply-To: obm-l@mat.puc-rio.br Eu to achando que isso e um problema de interpretaçao.Primeiro,veja que a=s(a)=s((a))=... e um pouco estranho(voce nao consegue aplicar PIF)Pior ainda,essa ideia e meio contraditoria. Pode-se provar por induçao que n e s(n) sao diferentes. O caso n=1 e trivial.Prove voce mesmo o passo indutivo. Logo voce ve que s(a)=a e impossivel. ATEEEEEEEEEEEEEEEEEEEE!!!!!!!!!!!!!!!!!!! -- Mensagem original -- >Na Eureka 3, p. 26, há um artigo de Elon Lages Lima chamado "O Princípio >da >Indução", onde o autor afirma que o conjunto N dos números naturais é >caracterizado pelas seguintes propriedades: > >A) Existe função s: N -> N, que associa a cada n pertencente a N um elemento >s(n) pertecente a N, chamado o sucessor de n. >(Cada natural tem um sucessor natural) >B) A função s: N-> N é injetiva. >(sucessores iguais,numeros iguais) >C) Existe um único elemento 1 no conjunto N, tal que 1 != s(n) para todo >n >pertencente a N. >(nenhum natural tem 1 como sucessor) >D) Se um subconjunto X contido em N é tal que 1 pertence a N e s(X) está >contido em X,X=N. >(se X e um conjunto de naturais contem o 1 e contem o sucessor de cada elemento,X=N) >As afirmações A, B, C e D são os axiomas de Peano. > >Agora vem a minha dúvida. Imagine o conjunto de números: >V = {0, 1, 2, 3, ...} U {a}, onde o elemento 'a' não pertence a {0, 1, 2, >3, >...} >e a função injetiva s: V -> V onde: >s(x) = a, se x=a; senão s(x) = x+1 > >Temos, então, o conjunto V e a função s que satisfazem os axiomas de Peano. >Dessa forma, podemos dizer que V é o conjunto dos número naturais, mas não >é!!!!! >Qual o problema aí??? > >Alguém pode esclarecer a minha dúvida? > >Obrigado > >Vinicius Fortuna > > > > >========================================================================= >Instruções para entrar na lista, sair da lista e usar a lista em >http://www.mat.puc-rio.br/~nicolau/olimp/obm-l.html >O administrador desta lista é >========================================================================= > TRANSIRE SVVM PECTVS MVNDOQUE POTIRE CONGREGATI EX TOTO ORBE MATHEMATICI OB SCRIPTA INSIGNIA TRIBVERE Medalha Fields(John Charles Fields) ------------------------------------------ Use o melhor sistema de busca da Internet Radar UOL - http://www.radaruol.com.br ========================================================================= Instruções para entrar na lista, sair da lista e usar a lista em http://www.mat.puc-rio.br/~nicolau/olimp/obm-l.html O administrador desta lista é ========================================================================= From owner-obm-l@sucuri.mat.puc-rio.br Tue Jun 18 17:35:05 2002 Return-Path: Received: (from majordom@localhost) by sucuri.mat.puc-rio.br (8.9.3/8.9.3) id RAA19494 for obm-l-list; Tue, 18 Jun 2002 17:34:43 -0300 Received: from shannon.bol.com.br (shannon.bol.com.br [200.221.24.13]) by sucuri.mat.puc-rio.br (8.9.3/8.9.3) with ESMTP id RAA19490 for ; Tue, 18 Jun 2002 17:34:41 -0300 Received: from bol.com.br (200.221.24.132) by shannon.bol.com.br (5.1.071) id 3D0905830011EB33 for obm-l@mat.puc-rio.br; Tue, 18 Jun 2002 17:33:36 -0300 Date: Tue, 18 Jun 2002 18:33:36 -0200 Message-Id: Subject: [obm-l] =?iso-8859-1?q?Re=3A=5Bobm=2Dl=5D_Re=3A_=5Bobm=2Dl=5D_F=EDsica?= MIME-Version: 1.0 Content-Type: multipart/mixed; boundary="_=__=_XaM3_Boundary.1024432416.2A.502623.42.24240.52.42.101010.1854212783" From: "rafaelc.l" To: obm-l@mat.puc-rio.br X-XaM3-API-Version: 2.4.3.4.4 X-SenderIP: 200.176.166.17 Sender: owner-obm-l@sucuri.mat.puc-rio.br Precedence: bulk Reply-To: obm-l@mat.puc-rio.br --_=__=_XaM3_Boundary.1024432416.2A.502623.42.24240.52.42.101010.1854212783 Content-Type: text/plain;charset="iso-8859-1" Content-Transfer-Encoding: quoted-printable Puts Daniel, teorema de Thevenin? nunca vi isso em livro de ensino m=E9dio e nem fala no programa do IME tbm....mas se vc puder me dizer onde encontro esse teorema, ficarei grato....... Acho que para aquele circuito vc tem que bolar um sistema de logaritmo neperiano e jogar com os dados..... __________________________________________________________________________ AcessoBOL, s=F3 R$ 9,90! O menor pre=E7o do mercado! Assine j=E1! http://www.bol.com.br/acessobol --_=__=_XaM3_Boundary.1024432416.2A.502623.42.24240.52.42.101010.1854212783 Content-Type: text/plain; name="00000H4M" Content-Transfer-Encoding: base64 DQo+ICBTZWkgcXVlIGFxdWkg6SB1bWEgbGlzdGEgZGUgbWF0ZW3hdGljYSwgbWFzDQo+IG1l c21vIGFzc2ltIGFsZ3XpbSBwb2RlcmlhIG1lIGFqdWRhciBuYQ0KPiBxdWVzdONvIDEwIGRh IHByb3ZhIGRlIGbtc2ljYSBkbyBJTUUgMjAwMS4gyQ0KPiB1bSBjaXJjdWl0byBlbOl0cmlj byBjb20gdW0gZ3LhZmljby4gRXUgdmkNCj4gdW1hIHJlc29sdefjbyBlbSBxdWUgZm9pIHVs dGlsaXphZG8gbw0KPiBj4WxjdWxvIGludGVncmFsLiBFdSBwZXJndW50bzogcXVhbCBzZXJp YQ0KPiB1bWEgb3V0cmEgZm9ybWEgZGUgcmVzb2x2ZXIgYSBxdWVzdONvIHNlbQ0KPiBtYXRl beF0aWNhIHN1cGVyaW9yPywgauEgcXVlIGlzdG8gbuNvIGZheg0KPiBwYXJ0ZSBkbyBwcm9n cmFtYSBkbyBJTUUuDQoNCiAgICAgICAgICAgICAgICBO428gc2VpIHNlIGFqdWRhLCBuYSB2 ZXJkYWRlIHNlIHF1aXNlciBwb3NzbyBhY2hhciBkZXBvaXMNCnByYSB2Yy4gySBz8yB1c2Fy IG8gdGVvcmVtYSBkZSBUaGV2ZW5pbiBlIHRyYW5zZm9ybWFyIG8gY2ljcnVpdG8gZW0gdW0N CmNpcmN1aXRvIGVxdWl2YWxlbnRlIGRlIFRoZXZlbmluLCBhc3NpbSBu428g6SBwcmVjaXNv IHJlc29sdmVyIGludGVncmFpcy4NCiAgICAgICAgICAgICAgICAgICAgICAgICAgICBEYW5p ZWwNCg0KDQoNCj09PT09PT09PT09PT09PT09PT09PT09PT09PT09PT09PT09PT09PT09PT09 PT09PT09PT09PT09PT09PT09PT09PT09PT09PT0NCkluc3Rydef1ZXMgcGFyYSBlbnRyYXIg bmEgbGlzdGEsIHNhaXIgZGEgbGlzdGEgZSB1c2FyIGEgbGlzdGEgZW0NCmh0dHA6Ly93d3cu bWF0LnB1Yy1yaW8uYnIvfm5pY29sYXUvb2xpbXAvb2JtLWwuaHRtbA0KTyBhZG1pbmlzdHJh ZG9yIGRlc3RhIGxpc3RhIOkgPG5pY29sYXVAbWF0LnB1Yy1yaW8uYnI+DQo9PT09PT09PT09 PT09PT09PT09PT09PT09PT09PT09PT09PT09PT09PT09PT09PT09PT09PT09PT09PT09PT09 PT09PT09PT09DQo= --_=__=_XaM3_Boundary.1024432416.2A.502623.42.24240.52.42.101010.1854212783-- ========================================================================= Instruções para entrar na lista, sair da lista e usar a lista em http://www.mat.puc-rio.br/~nicolau/olimp/obm-l.html O administrador desta lista é ========================================================================= From owner-obm-l@sucuri.mat.puc-rio.br Tue Jun 18 17:36:21 2002 Return-Path: Received: (from majordom@localhost) by sucuri.mat.puc-rio.br (8.9.3/8.9.3) id RAA19546 for obm-l-list; Tue, 18 Jun 2002 17:36:15 -0300 Received: from shannon.bol.com.br (shannon.bol.com.br [200.221.24.13]) by sucuri.mat.puc-rio.br (8.9.3/8.9.3) with ESMTP id RAA19540 for ; Tue, 18 Jun 2002 17:36:13 -0300 Received: from bol.com.br (200.221.24.132) by shannon.bol.com.br (5.1.071) id 3D0905830011EBF1 for obm-l@mat.puc-rio.br; Tue, 18 Jun 2002 17:35:05 -0300 Date: Tue, 18 Jun 2002 18:35:05 -0200 Message-Id: Subject: [obm-l] =?iso-8859-1?q?Re=3A=5Bobm=2Dl=5D_F=EDsica=2E=2E=2E=2Eengano?= MIME-Version: 1.0 Content-Type: multipart/mixed; boundary="_=__=_XaM3_Boundary.1024432505.2A.934638.42.26100.52.42.101010.742664916" From: "rafaelc.l" To: obm-l@mat.puc-rio.br X-XaM3-API-Version: 2.4.3.4.4 X-SenderIP: 200.176.166.17 Sender: owner-obm-l@sucuri.mat.puc-rio.br Precedence: bulk Reply-To: obm-l@mat.puc-rio.br --_=__=_XaM3_Boundary.1024432505.2A.934638.42.26100.52.42.101010.742664916 Content-Type: text/plain;charset="iso-8859-1" Content-Transfer-Encoding: quoted-printable Desculpe pessoal, =E9 IME 2002 a =FAltima quest=E3o..... Abra=E7o __________________________________________________________________________ AcessoBOL, s=F3 R$ 9,90! O menor pre=E7o do mercado! Assine j=E1! http://www.bol.com.br/acessobol --_=__=_XaM3_Boundary.1024432505.2A.934638.42.26100.52.42.101010.742664916 Content-Type: text/plain; name="00000J83" Content-Transfer-Encoding: base64 IA0KIFNlaSBxdWUgYXF1aSDpIHVtYSBsaXN0YSBkZSBtYXRlbeF0aWNhLCBtYXMgDQptZXNt byBhc3NpbSBhbGd16W0gcG9kZXJpYSBtZSBhanVkYXIgbmEgDQpxdWVzdONvIDEwIGRhIHBy b3ZhIGRlIGbtc2ljYSBkbyBJTUUgMjAwMS4gySANCnVtIGNpcmN1aXRvIGVs6XRyaWNvIGNv bSB1bSBncuFmaWNvLiBFdSB2aSANCnVtYSByZXNvbHXn428gZW0gcXVlIGZvaSB1bHRpbGl6 YWRvIG8gDQpj4WxjdWxvIGludGVncmFsLiBFdSBwZXJndW50bzogcXVhbCBzZXJpYSANCnVt YSBvdXRyYSBmb3JtYSBkZSByZXNvbHZlciBhIHF1ZXN0428gc2VtIA0KbWF0ZW3hdGljYSBz dXBlcmlvcj8sIGrhIHF1ZSBpc3RvIG7jbyBmYXogDQpwYXJ0ZSBkbyBwcm9ncmFtYSBkbyBJ TUUuDQoNCiANCl9fX19fX19fX19fX19fX19fX19fX19fX19fX19fX19fX19fX19fX19fX19f X19fX19fX19fX19fX19fX19fX19fX19fX19fX19fDQpRdWVyIHRlciBzZXUgcHLzcHJpbyBl bmRlcmXnbyBuYSBJbnRlcm5ldD8NCkdhcmFudGEgauEgbyBzZXUgZSBhaW5kYSBnYW5oZSBj aW5jbyBlLW1haWxzIHBlcnNvbmFsaXphZG9zLg0KRG9t7W5pb3NCT0wgLSBodHRwOi8vZG9t aW5pb3MuYm9sLmNvbS5icg0KDQoNCj09PT09PT09PT09PT09PT09PT09PT09PT09PT09PT09 PT09PT09PT09PT09PT09PT09PT09PT09PT09PT09PT09PT09PT09PT0NCkluc3Rydef1ZXMg cGFyYSBlbnRyYXIgbmEgbGlzdGEsIHNhaXIgZGEgbGlzdGEgZSB1c2FyIGEgbGlzdGEgZW0N Cmh0dHA6Ly93d3cubWF0LnB1Yy1yaW8uYnIvfm5pY29sYXUvb2xpbXAvb2JtLWwuaHRtbA0K TyBhZG1pbmlzdHJhZG9yIGRlc3RhIGxpc3RhIOkgPG5pY29sYXVAbWF0LnB1Yy1yaW8uYnI+ DQo9PT09PT09PT09PT09PT09PT09PT09PT09PT09PT09PT09PT09PT09PT09PT09PT09PT09 PT09PT09PT09PT09PT09PT09PT09DQo= --_=__=_XaM3_Boundary.1024432505.2A.934638.42.26100.52.42.101010.742664916-- ========================================================================= Instruções para entrar na lista, sair da lista e usar a lista em http://www.mat.puc-rio.br/~nicolau/olimp/obm-l.html O administrador desta lista é ========================================================================= From owner-obm-l@sucuri.mat.puc-rio.br Tue Jun 18 17:52:12 2002 Return-Path: Received: (from majordom@localhost) by sucuri.mat.puc-rio.br (8.9.3/8.9.3) id RAA20112 for obm-l-list; Tue, 18 Jun 2002 17:51:11 -0300 Received: from hotmail.com (f48.law8.hotmail.com [216.33.241.48]) by sucuri.mat.puc-rio.br (8.9.3/8.9.3) with ESMTP id RAA20095 for ; Tue, 18 Jun 2002 17:51:07 -0300 Received: from mail pickup service by hotmail.com with Microsoft SMTPSVC; Tue, 18 Jun 2002 13:50:52 -0700 Received: from 200.222.105.178 by lw8fd.law8.hotmail.msn.com with HTTP; Tue, 18 Jun 2002 20:50:52 GMT X-Originating-IP: [200.222.105.178] From: "Antonio Neto" To: obm-l@mat.puc-rio.br Subject: Re: [obm-l] 3 circulos! Date: Tue, 18 Jun 2002 20:50:52 +0000 Mime-Version: 1.0 Content-Type: text/plain; format=flowed Message-ID: X-OriginalArrivalTime: 18 Jun 2002 20:50:52.0457 (UTC) FILETIME=[D58A9990:01C21709] Sender: owner-obm-l@sucuri.mat.puc-rio.br Precedence: bulk Reply-To: obm-l@mat.puc-rio.br Aproveita e demonstra que r + r -1 +r_2 = h, onde h é a altura relativa a hipotenusa. Abracos, olavo. >From: Rafael WC >Reply-To: obm-l@mat.puc-rio.br >To: OBM >Subject: [obm-l] 3 circulos! >Date: Sun, 16 Jun 2002 21:47:30 -0700 (PDT) > >Oi Pessoal! > >Alguém conseguiria me dar uma indica do que usar nesse >exercício? > >Os raios dos circulos inscritos num triangulo >retangulo ABC e nos dois triangulos ABH e ACH >determinados pela altura relativa à hipotenusa BC são >respectivamente r ,r1 e r2. Demonstrar que: >r² = r1² + r2² > >Valeu! > >Rafael. > >===== >Rafael Werneck Cinoto > ICQ# 107011599 > rwcinoto@yahoo.com > matduvidas@yahoo.com.br >http://www.rwcinoto.hpg.com.br/ > >__________________________________________________ >Do You Yahoo!? >Yahoo! - Official partner of 2002 FIFA World Cup >http://fifaworldcup.yahoo.com >========================================================================= >Instruções para entrar na lista, sair da lista e usar a lista em >http://www.mat.puc-rio.br/~nicolau/olimp/obm-l.html >O administrador desta lista é >========================================================================= _________________________________________________________________ Join the world’s largest e-mail service with MSN Hotmail. http://www.hotmail.com ========================================================================= Instruções para entrar na lista, sair da lista e usar a lista em http://www.mat.puc-rio.br/~nicolau/olimp/obm-l.html O administrador desta lista é ========================================================================= From owner-obm-l@sucuri.mat.puc-rio.br Tue Jun 18 18:02:30 2002 Return-Path: Received: (from majordom@localhost) by sucuri.mat.puc-rio.br (8.9.3/8.9.3) id SAA20590 for obm-l-list; Tue, 18 Jun 2002 18:01:08 -0300 Received: from shannon.bol.com.br (shannon.bol.com.br [200.221.24.13]) by sucuri.mat.puc-rio.br (8.9.3/8.9.3) with ESMTP id SAA20586 for ; Tue, 18 Jun 2002 18:01:06 -0300 Received: from bol.com.br (200.221.24.132) by shannon.bol.com.br (5.1.071) id 3D0905830011F806 for obm-l@mat.puc-rio.br; Tue, 18 Jun 2002 18:00:01 -0300 Date: Tue, 18 Jun 2002 19:00:02 -0200 Message-Id: Subject: [obm-l] ..... MIME-Version: 1.0 Content-Type: text/plain;charset="iso-8859-1" From: "rafaelc.l" To: obm-l@mat.puc-rio.br X-XaM3-API-Version: 2.4.3.4.4 X-SenderIP: 200.176.166.226 Content-Transfer-Encoding: 8bit X-MIME-Autoconverted: from quoted-printable to 8bit by sucuri.mat.puc-rio.br id SAA20587 Sender: owner-obm-l@sucuri.mat.puc-rio.br Precedence: bulk Reply-To: obm-l@mat.puc-rio.br (IME- 2000) Prove que para qualquer número inteiro k, os números k e k^5 terminam sempre com o mesmo algarismo( algarismo das unidades). __________________________________________________________________________ AcessoBOL, só R$ 9,90! O menor preço do mercado! Assine já! http://www.bol.com.br/acessobol ========================================================================= Instruções para entrar na lista, sair da lista e usar a lista em http://www.mat.puc-rio.br/~nicolau/olimp/obm-l.html O administrador desta lista é ========================================================================= From owner-obm-l@sucuri.mat.puc-rio.br Tue Jun 18 22:29:07 2002 Return-Path: Received: (from majordom@localhost) by sucuri.mat.puc-rio.br (8.9.3/8.9.3) id WAA23520 for obm-l-list; Tue, 18 Jun 2002 22:28:11 -0300 Received: from matinhos.terra.com.br (matinhos.terra.com.br [200.176.3.21]) by sucuri.mat.puc-rio.br (8.9.3/8.9.3) with ESMTP id WAA23516 for ; Tue, 18 Jun 2002 22:28:09 -0300 Received: from pavuna.terra.com.br (pavuna.terra.com.br [200.176.3.41]) by matinhos.terra.com.br (Postfix) with ESMTP id 0EF6547C15 for ; Tue, 18 Jun 2002 22:27:56 -0300 (EST) Received: from stabel (dl-nas1-poa-C89A000E.p001.terra.com.br [200.154.0.14]) (authenticated user dudasta) by pavuna.terra.com.br (Postfix) with ESMTP id 856C968105 for ; Tue, 18 Jun 2002 22:27:54 -0300 (EST) Message-ID: <002f01c21730$8a962540$0e009ac8@stabel> From: "Eduardo Casagrande Stabel" To: References: Subject: Re: [obm-l] ..... Date: Tue, 18 Jun 2002 22:27:54 -0300 MIME-Version: 1.0 Content-Type: text/plain; charset="iso-8859-1" Content-Transfer-Encoding: 8bit X-Priority: 3 X-MSMail-Priority: Normal X-Mailer: Microsoft Outlook Express 6.00.2600.0000 X-MimeOLE: Produced By Microsoft MimeOLE V6.00.2600.0000 Sender: owner-obm-l@sucuri.mat.puc-rio.br Precedence: bulk Reply-To: obm-l@mat.puc-rio.br Oi Rafael, essa questão deve estar vindo para a lista pela terceira vez. Existem várias maneiras de resolvê-la. Primeiro você vê que se quer mostrar que (k^5 - k) é divisível por 10. Depois mostra separadamente que (k^5 - k) é par, pois é uma diferença de números de mesma paridade, e a seguir mostra que (k^5 - k) é divisível por 5. Para ver isso use: i) o pequeno teorema de Fermat que diz que se p é primo (a^p - a) é divisível por p. ii) é preciso que k seja da forma 5q, 5q+-1 ou 5q+-2, e calcula (k^5 - k) expandindo k^5 pelo pelo binômio de Newton e vendo que o resultado é um múltiplo inteiro de 5. Uma maneira mais mecânica e mais acessível aos alunos (médios) de 5a. e 6a. séries é calcular o último algarismo de k^5 para cada algarismo das unidades de k(0,1,2,3,4,5,6,7,8 e 9) e reparar que o resultado de k^5 tem o mesmo algarismo das unidades que k, e daí a diferença é sempre divisível por 10 pois o número termina em zero. Ainda uma quarta maneira é fatorando (k^5 - k) = k(k^4 - 1) = k(k^2 + 1)(k^2 - 1) = k(k^2 + 1)(k + 1)(k - 1) e reparando que se k é das formas 5q, 5q+-1 então na expressão acima vai aparecer um 5q (por causa de k, (k+1) e (k-1) e se k for da forma 5q+-2 então (k^2 + 1) vai ser múltiplo de 5. Enfim, várias maneiras. Muitas questões já propostas na lista (sobre divisibildade) tem uma solução muito parecida com essa, pois envolvem a aritmética dos inteiros (o uso de congruência e outras coisas) discutida, por exemplo, no livro do Nicolau e do Gugu sobre primos de Fermat e outros primos grandes. Um abraço! Eduardo Casagrande Stabel. Porto Alegre, RS. From: "rafaelc.l" > > > (IME- 2000) Prove que para qualquer número inteiro k, os > números k e k^5 terminam sempre com o mesmo algarismo( > algarismo das unidades). > > > __________________________________________________________________________ > AcessoBOL, só R$ 9,90! O menor preço do mercado! > Assine já! http://www.bol.com.br/acessobol > > > ========================================================================= > Instruções para entrar na lista, sair da lista e usar a lista em > http://www.mat.puc-rio.br/~nicolau/olimp/obm-l.html > O administrador desta lista é > ========================================================================= > > ========================================================================= Instruções para entrar na lista, sair da lista e usar a lista em http://www.mat.puc-rio.br/~nicolau/olimp/obm-l.html O administrador desta lista é ========================================================================= From owner-obm-l@sucuri.mat.puc-rio.br Wed Jun 19 13:24:33 2002 Return-Path: Received: (from majordom@localhost) by sucuri.mat.puc-rio.br (8.9.3/8.9.3) id NAA31952 for obm-l-list; Wed, 19 Jun 2002 13:23:35 -0300 Received: from hotmail.com (f44.law4.hotmail.com [216.33.149.44]) by sucuri.mat.puc-rio.br (8.9.3/8.9.3) with ESMTP id NAA31948 for ; Wed, 19 Jun 2002 13:23:32 -0300 Received: from mail pickup service by hotmail.com with Microsoft SMTPSVC; Wed, 19 Jun 2002 09:22:26 -0700 Received: from 200.194.208.216 by lw4fd.law4.hotmail.msn.com with HTTP; Wed, 19 Jun 2002 16:22:26 GMT X-Originating-IP: [200.194.208.216] From: "Laurito Alves" To: obm-l@mat.puc-rio.br Subject: Re: [obm-l] Axiomas de Peano Date: Wed, 19 Jun 2002 16:22:26 +0000 Mime-Version: 1.0 Content-Type: text/plain; format=flowed Message-ID: X-OriginalArrivalTime: 19 Jun 2002 16:22:26.0320 (UTC) FILETIME=[7FF2E100:01C217AD] Sender: owner-obm-l@sucuri.mat.puc-rio.br Precedence: bulk Reply-To: obm-l@mat.puc-rio.br Vinícius, Seja V = {a, 1, 2, 3, 4 ... } e s(x) = a, se x=a; senão s(x) = x+1 Esse conjunto V não satisfaz o último postulado de Peano pois existe o subconjunto X = { 1 , 2, ... } de V tal que s(x) está em X para todo x de X mas X não é V. Logo, V não é N. Laurito M. Alves _________________________________________________________________ Join the world’s largest e-mail service with MSN Hotmail. http://www.hotmail.com ========================================================================= Instruções para entrar na lista, sair da lista e usar a lista em http://www.mat.puc-rio.br/~nicolau/olimp/obm-l.html O administrador desta lista é ========================================================================= From owner-obm-l@sucuri.mat.puc-rio.br Wed Jun 19 13:24:34 2002 Return-Path: Received: (from majordom@localhost) by sucuri.mat.puc-rio.br (8.9.3/8.9.3) id NAA31941 for obm-l-list; Wed, 19 Jun 2002 13:23:04 -0300 Received: from hotmail.com (f17.law4.hotmail.com [216.33.149.17]) by sucuri.mat.puc-rio.br (8.9.3/8.9.3) with ESMTP id NAA31937 for ; Wed, 19 Jun 2002 13:22:59 -0300 Received: from mail pickup service by hotmail.com with Microsoft SMTPSVC; Wed, 19 Jun 2002 09:22:25 -0700 Received: from 200.194.208.216 by lw4fd.law4.hotmail.msn.com with HTTP; Wed, 19 Jun 2002 16:22:25 GMT X-Originating-IP: [200.194.208.216] From: "Laurito Alves" To: obm-l@mat.puc-rio.br Subject: Re: [obm-l] Axiomas de Peano Date: Wed, 19 Jun 2002 16:22:25 +0000 Mime-Version: 1.0 Content-Type: text/plain; format=flowed Message-ID: X-OriginalArrivalTime: 19 Jun 2002 16:22:25.0611 (UTC) FILETIME=[7F86B1B0:01C217AD] Sender: owner-obm-l@sucuri.mat.puc-rio.br Precedence: bulk Reply-To: obm-l@mat.puc-rio.br Vinícius, Seja V = {a, 1, 2, 3, 4 ... } e s(x) = a, se x=a; senão s(x) = x+1 Esse conjunto V não satisfaz o último postulado de Peano pois existe o subconjunto X = { 1 , 2, ... } de V tal que s(x) está em X para todo x de X mas X não é V. Logo, V não é N. Laurito M. Alves _________________________________________________________________ Get your FREE download of MSN Explorer at http://explorer.msn.com/intl.asp. ========================================================================= Instruções para entrar na lista, sair da lista e usar a lista em http://www.mat.puc-rio.br/~nicolau/olimp/obm-l.html O administrador desta lista é ========================================================================= From owner-obm-l@sucuri.mat.puc-rio.br Wed Jun 19 16:30:25 2002 Return-Path: Received: (from majordom@localhost) by sucuri.mat.puc-rio.br (8.9.3/8.9.3) id QAA02853 for obm-l-list; Wed, 19 Jun 2002 16:27:54 -0300 Received: from smtp.ieg.com.br (12.139.226.200.in-addr.arpa.ig.com.br [200.226.139.12] (may be forged)) by sucuri.mat.puc-rio.br (8.9.3/8.9.3) with ESMTP id QAA02848 for ; Wed, 19 Jun 2002 16:27:17 -0300 Received: from igor ([200.222.135.47]) by smtp.ieg.com.br (IeG relay/8.9.3) with SMTP id g5JJSEhh063179 for ; Wed, 19 Jun 2002 16:28:25 -0300 (BRT) Message-ID: <002a01c217c7$35a790e0$2f87dec8@igor> From: "Igor Castro" To: "obm-lista" Subject: [obm-l] =?iso-8859-1?Q?Ajuda_r=E1pida...?= Date: Wed, 19 Jun 2002 15:50:42 -0300 MIME-Version: 1.0 Content-Type: multipart/alternative; boundary="----=_NextPart_000_0005_01C217A9.110A6140" X-Priority: 3 X-MSMail-Priority: Normal X-Mailer: Microsoft Outlook Express 5.00.2615.200 X-MimeOLE: Produced By Microsoft MimeOLE V5.00.2615.200 Sender: owner-obm-l@sucuri.mat.puc-rio.br Precedence: bulk Reply-To: obm-l@mat.puc-rio.br This is a multi-part message in MIME format. ------=_NextPart_000_0005_01C217A9.110A6140 Content-Type: text/plain; charset="iso-8859-1" Content-Transfer-Encoding: quoted-printable Bem companheiros, n=E3o estou conseguindo resolver esse problema que = peguei em um livro de 1 gr=E1u... se puderem dar uma luz... : ) Para quantos valores de n entre 1 e 1990 a fra=E7=E3o impr=F3pria (n^2 = +7)/(n+4) n=E3o =E9 irredut=EDvel? Abra=E7os... Igor.. ------=_NextPart_000_0005_01C217A9.110A6140 Content-Type: text/html; charset="iso-8859-1" Content-Transfer-Encoding: quoted-printable
Bem companheiros, n=E3o estou = conseguindo resolver=20 esse problema que peguei em um livro de 1 gr=E1u... se puderem dar uma = luz... :=20 )
Para quantos valores de n entre 1 e = 1990 a=20 fra=E7=E3o impr=F3pria (n^2 +7)/(n+4) n=E3o =E9 = irredut=EDvel?
Abra=E7os...
Igor..
------=_NextPart_000_0005_01C217A9.110A6140-- ========================================================================= Instruções para entrar na lista, sair da lista e usar a lista em http://www.mat.puc-rio.br/~nicolau/olimp/obm-l.html O administrador desta lista é ========================================================================= From owner-obm-l@sucuri.mat.puc-rio.br Wed Jun 19 16:51:53 2002 Return-Path: Received: (from majordom@localhost) by sucuri.mat.puc-rio.br (8.9.3/8.9.3) id QAA03303 for obm-l-list; Wed, 19 Jun 2002 16:51:07 -0300 Received: (from nicolau@localhost) by sucuri.mat.puc-rio.br (8.9.3/8.9.3) id QAA03298 for obm-l@mat.puc-rio.br; Wed, 19 Jun 2002 16:51:07 -0300 Date: Wed, 19 Jun 2002 16:51:06 -0300 From: "Nicolau C. Saldanha" To: obm-l@mat.puc-rio.br Subject: [obm-l] Re: =?iso-8859-1?Q?=5Bobm-l=5D_Ajuda_r=E1pida=2E=2E=2E?= Message-ID: <20020619165106.B3071@sucuri.mat.puc-rio.br> References: <002a01c217c7$35a790e0$2f87dec8@igor> Mime-Version: 1.0 Content-Type: text/plain; charset=iso-8859-1 Content-Disposition: inline Content-Transfer-Encoding: 8bit User-Agent: Mutt/1.2.5i In-Reply-To: <002a01c217c7$35a790e0$2f87dec8@igor>; from cnaval@ieg.com.br on Wed, Jun 19, 2002 at 03:50:42PM -0300 Sender: owner-obm-l@sucuri.mat.puc-rio.br Precedence: bulk Reply-To: obm-l@mat.puc-rio.br On Wed, Jun 19, 2002 at 03:50:42PM -0300, Igor Castro wrote: > Bem companheiros, não estou conseguindo resolver esse problema que peguei em > um livro de 1 gráu... se puderem dar uma luz... : ) Para quantos valores de n > entre 1 e 1990 a fração imprópria (n^2 +7)/(n+4) não é irredutível? > Abraços... Igor.. mdc(n^2 + 7, n + 4) = mdc(n^2 + 7 - n*(n + 4), n + 4) = mdc(-4n + 7, n + 4) = mdc(-4n + 7 + 4*(n + 4), n + 4) = mdc(23, n + 4) Assim o mdc é 23 se n+4 for múltiplo de 23 e 1 caso contrário. Ou seja, na faixa pedida, a fração é *não* irredutível para n = 19, 19+23, 19+2*23, ..., 19+85*23 = 1974. Assim a resposta é 86. []s, N. ========================================================================= Instruções para entrar na lista, sair da lista e usar a lista em http://www.mat.puc-rio.br/~nicolau/olimp/obm-l.html O administrador desta lista é ========================================================================= From owner-obm-l@sucuri.mat.puc-rio.br Wed Jun 19 17:04:38 2002 Return-Path: Received: (from majordom@localhost) by sucuri.mat.puc-rio.br (8.9.3/8.9.3) id RAA03669 for obm-l-list; Wed, 19 Jun 2002 17:03:49 -0300 Received: from smtp.ieg.com.br (12.139.226.200.in-addr.arpa.ig.com.br [200.226.139.12] (may be forged)) by sucuri.mat.puc-rio.br (8.9.3/8.9.3) with ESMTP id RAA03665 for ; Wed, 19 Jun 2002 17:03:46 -0300 Received: from i29uxvx9muhimu (200-163-082-133-bsace202.dial.telebrasilia.net.br [200.163.82.133]) by smtp.ieg.com.br (IeG relay/8.9.3) with SMTP id g5JK57hf066290 for ; Wed, 19 Jun 2002 17:05:08 -0300 (BRT) Message-ID: <002801c217cc$57d7c180$9865fea9@i29uxvx9muhimu> From: "iver" To: Subject: [obm-l] integral Date: Wed, 19 Jun 2002 17:03:11 -0300 MIME-Version: 1.0 Content-Type: multipart/alternative; boundary="----=_NextPart_000_0025_01C217B3.31436380" X-Priority: 3 X-MSMail-Priority: Normal X-Mailer: Microsoft Outlook Express 6.00.2600.0000 X-MimeOLE: Produced By Microsoft MimeOLE V6.00.2600.0000 Sender: owner-obm-l@sucuri.mat.puc-rio.br Precedence: bulk Reply-To: obm-l@mat.puc-rio.br This is a multi-part message in MIME format. ------=_NextPart_000_0025_01C217B3.31436380 Content-Type: text/plain; charset="iso-8859-1" Content-Transfer-Encoding: quoted-printable algu=E9m pode mostrar como se resovler a seguitne integral int( sin(x/(x+1)) ) dx ------=_NextPart_000_0025_01C217B3.31436380 Content-Type: text/html; charset="iso-8859-1" Content-Transfer-Encoding: quoted-printable
algu=E9m pode mostrar como se resovler = a seguitne=20 integral
 
int( sin(x/(x+1)) ) = dx
------=_NextPart_000_0025_01C217B3.31436380-- ========================================================================= Instruções para entrar na lista, sair da lista e usar a lista em http://www.mat.puc-rio.br/~nicolau/olimp/obm-l.html O administrador desta lista é ========================================================================= From owner-obm-l@sucuri.mat.puc-rio.br Wed Jun 19 20:32:15 2002 Return-Path: Received: (from majordom@localhost) by sucuri.mat.puc-rio.br (8.9.3/8.9.3) id UAA06642 for obm-l-list; Wed, 19 Jun 2002 20:31:43 -0300 Received: from matinhos.terra.com.br (matinhos.terra.com.br [200.176.3.21]) by sucuri.mat.puc-rio.br (8.9.3/8.9.3) with ESMTP id UAA06638 for ; Wed, 19 Jun 2002 20:31:41 -0300 Received: from webmail4.poa.terra.com.br (webmail4.terra.com.br [200.176.3.179]) by matinhos.terra.com.br (Postfix) with ESMTP id 4068F4705E for ; Wed, 19 Jun 2002 20:31:31 -0300 (EST) Received: from terra.com.br (webmail4.poa.terra.com.br [200.176.3.179]) by webmail4.poa.terra.com.br (Postfix) with ESMTP id 3E1B4B97FC for ; Wed, 19 Jun 2002 20:31:31 -0300 (EST) Date: Wed, 19 Jun 2002 23:31:31 +0000 Message-Id: Subject: [obm-l] =?iso-8859-1?q?Fatora=E7=E3o?= MIME-Version: 1.0 Content-Type: text/plain;charset="iso-8859-1" From: "hilhend" To: "obm-l" X-XaM3-API-Version: 2.4.3.2.9 X-SenderIP: 200.161.14.239 Content-Transfer-Encoding: 8bit X-MIME-Autoconverted: from quoted-printable to 8bit by sucuri.mat.puc-rio.br id UAA06639 Sender: owner-obm-l@sucuri.mat.puc-rio.br Precedence: bulk Reply-To: obm-l@mat.puc-rio.br Estou enviando a todos novamente aquele expressao a fatorar em reais X^6 +X^3.Y^3 + Y^6 pois acredito nao tenha sido visto na imensidao de mensagens. Se o problema for inconsistente mostre que e. Um abraço a todos. ========================================================================= Instruções para entrar na lista, sair da lista e usar a lista em http://www.mat.puc-rio.br/~nicolau/olimp/obm-l.html O administrador desta lista é ========================================================================= From owner-obm-l@sucuri.mat.puc-rio.br Wed Jun 19 20:57:22 2002 Return-Path: Received: (from majordom@localhost) by sucuri.mat.puc-rio.br (8.9.3/8.9.3) id UAA07423 for obm-l-list; Wed, 19 Jun 2002 20:57:19 -0300 Received: from traven.uol.com.br (traven.uol.com.br [200.231.206.184]) by sucuri.mat.puc-rio.br (8.9.3/8.9.3) with ESMTP id UAA07419 for ; Wed, 19 Jun 2002 20:57:18 -0300 Received: from cabru ([200.158.71.113]) by traven.uol.com.br (8.9.1/8.9.1) with SMTP id UAA26331 for ; Wed, 19 Jun 2002 20:45:31 -0300 (BRT) Message-ID: <001c01c217ed$3bc8d260$71479ec8@cabru> From: "Bruno" To: References: <3D0E210E00000167@www.zipmail.com.br> <3D0E3E89.1060706@niski.com> Subject: Re: [obm-l] Re: [obm-l] Re: [obm-l] Desafio o retorno!! Date: Wed, 19 Jun 2002 20:58:36 -0300 MIME-Version: 1.0 Content-Type: multipart/alternative; boundary="----=_NextPart_000_0019_01C217D4.14AEC9A0" X-Priority: 3 X-MSMail-Priority: Normal X-Mailer: Microsoft Outlook Express 5.50.4133.2400 X-MimeOLE: Produced By Microsoft MimeOLE V5.50.4133.2400 Sender: owner-obm-l@sucuri.mat.puc-rio.br Precedence: bulk Reply-To: obm-l@mat.puc-rio.br This is a multi-part message in MIME format. ------=_NextPart_000_0019_01C217D4.14AEC9A0 Content-Type: text/plain; charset="iso-8859-1" Content-Transfer-Encoding: quoted-printable Valeu pela ajuda....eu sou mesmo aluno do Glenn... ----- Original Message -----=20 From: niski=20 To: obm-l@mat.puc-rio.br=20 Sent: Monday, June 17, 2002 4:54 PM Subject: Re: [obm-l] Re: [obm-l] Re: [obm-l] Desafio o retorno!! Voc=EA =E9 aluno do Glenn? j=E1 tenho a expressao fatorada peterdirichlet@zipmail.com.br wrote: Vou especificar mais:Basta voce achar as raizes de que eu estava = falando,e depois agrupar tudoconvenientemente(vao aparecer pares de = conjugados,que sao legais p/produtar).Te = mais!!!!!!!!!!!!!!Peterdirichlet -- Mensagem original -- obrigado pela resolu=E7=E3o, mas eu queria saber se =E9 possivel fatorar = apenasemIR.....----- Original Message -----From: = To: Sent: Monday, = June 10, 2002 4:33 PMSubject: [obm-l] Re: [obm-l] Desafio o retorno!! Vou usar complexos(a paixao de JP):Seja a expressao f(x)=3Dx^2+x+1.Vamos = fatora-la em R+Ri,i^2+1=3D0. Defina cis x=3Dsen x+i*cos x=3De^(ix).Entao w=3D(cis(2*pi/3)) e = wbarra=3D(cis(4*pi)/3) sao zeros de f.Para as raizes de x^6+x^3+1,ache = as raizes cubicas de w e wbarra.Te mais!!!!!!!!!!!!!!!!-- Mensagem = original -- Ol=E1 amigos,Estou de volta com outro exerc=EDcio(+dif=EDcil na minha = opini=E3o)Meu professor disse que nem ele consegue fatorar a seguinte = express=E3o em IR:x^6 + (xy)^3 + y^6Abra=E7os,Bruno TRANSIRE SVVM PECTVS MVNDOQUE POTIRECONGREGATI EX TOTO ORBE MATHEMATICI = OB SCRIPTA INSIGNIA TRIBVEREMedalha Fields(John Charles = Fields)------------------------------------------Use o melhor sistema de = busca da InternetRadar UOL - = http://www.radaruol.com.br=3D=3D=3D=3D=3D=3D=3D=3D=3D=3D=3D=3D=3D=3D=3D=3D= =3D=3D=3D=3D=3D=3D=3D=3D=3D=3D=3D=3D=3D=3D=3D=3D=3D=3D=3D=3D=3D=3D=3D=3D=3D= =3D=3D=3D=3D=3D=3D=3D=3D=3D=3D=3D=3D=3D=3D=3D=3D=3D=3D=3D=3D=3D=3D=3D=3D=3D= =3D=3D=3D=3D=3D=3D=3DInstru=E7=F5es para entrar na lista, sair da lista = e usar a lista emhttp://www.mat.puc-rio.br/~nicolau/olimp/obm-l.htmlO = administrador desta lista =E9 = =3D=3D=3D=3D=3D=3D=3D=3D=3D=3D=3D=3D=3D=3D=3D=3D=3D= =3D=3D=3D=3D=3D=3D=3D=3D=3D=3D=3D=3D=3D=3D=3D=3D=3D=3D=3D=3D=3D=3D=3D=3D=3D= =3D=3D=3D=3D=3D=3D=3D=3D=3D=3D=3D=3D=3D=3D=3D=3D=3D=3D=3D=3D=3D=3D=3D=3D=3D= =3D=3D=3D=3D=3D=3D =3D=3D=3D=3D=3D=3D=3D=3D=3D=3D=3D=3D=3D=3D=3D=3D=3D=3D=3D=3D=3D=3D=3D=3D=3D= =3D=3D=3D=3D=3D=3D=3D=3D=3D=3D=3D=3D=3D=3D=3D=3D=3D=3D=3D=3D=3D=3D=3D=3D=3D= =3D=3D=3D=3D=3D=3D=3D=3D=3D=3D=3D=3D=3D=3D=3D=3D=3D=3D=3D=3D=3D=3D=3DInst= ru=E7=F5es para entrar na lista, sair da lista e usar a lista = emhttp://www.mat.puc-rio.br/~nicolau/olimp/obm-l.htmlO administrador = desta lista =E9 = =3D=3D=3D=3D=3D=3D=3D=3D=3D=3D=3D=3D=3D=3D=3D=3D=3D= =3D=3D=3D=3D=3D=3D=3D=3D=3D=3D=3D=3D=3D=3D=3D=3D=3D=3D=3D=3D=3D=3D=3D=3D=3D= =3D=3D=3D=3D=3D=3D=3D=3D=3D=3D=3D=3D=3D=3D=3D=3D=3D=3D=3D=3D=3D=3D=3D=3D=3D= =3D=3D=3D=3D=3D=3D TRANSIRE SVVM PECTVS MVNDOQUE POTIRECONGREGATI EX TOTO ORBE MATHEMATICI = OB SCRIPTA INSIGNIA TRIBVEREMedalha Fields(John Charles = Fields)------------------------------------------Use o melhor sistema de = busca da InternetRadar UOL - = http://www.radaruol.com.br=3D=3D=3D=3D=3D=3D=3D=3D=3D=3D=3D=3D=3D=3D=3D=3D= =3D=3D=3D=3D=3D=3D=3D=3D=3D=3D=3D=3D=3D=3D=3D=3D=3D=3D=3D=3D=3D=3D=3D=3D=3D= =3D=3D=3D=3D=3D=3D=3D=3D=3D=3D=3D=3D=3D=3D=3D=3D=3D=3D=3D=3D=3D=3D=3D=3D=3D= =3D=3D=3D=3D=3D=3D=3DInstru=E7=F5es para entrar na lista, sair da lista = e usar a lista emhttp://www.mat.puc-rio.br/~nicolau/olimp/obm-l.htmlO = administrador desta lista =E9 = =3D=3D=3D=3D=3D=3D=3D=3D=3D=3D=3D=3D=3D=3D=3D=3D=3D= =3D=3D=3D=3D=3D=3D=3D=3D=3D=3D=3D=3D=3D=3D=3D=3D=3D=3D=3D=3D=3D=3D=3D=3D=3D= =3D=3D=3D=3D=3D=3D=3D=3D=3D=3D=3D=3D=3D=3D=3D=3D=3D=3D=3D=3D=3D=3D=3D=3D=3D= =3D=3D=3D=3D=3D=3D ------=_NextPart_000_0019_01C217D4.14AEC9A0 Content-Type: text/html; charset="iso-8859-1" Content-Transfer-Encoding: quoted-printable
Valeu pela ajuda....eu sou mesmo aluno do=20 Glenn...
----- Original Message -----
From:=20 niski =
Sent: Monday, June 17, 2002 = 4:54 PM
Subject: Re: [obm-l] Re: = [obm-l] Re:=20 [obm-l] Desafio o retorno!!

Voc=EA =E9 aluno do Glenn? j=E1 tenho a expressao=20 fatorada



peterdirichlet@zipmail.com.= br=20 wrote:
Vou especificar mais:
Basta voce achar = as raizes de que eu estava falando,e depois agrupar = tudo
convenientemente(vao aparecer pares de conjugados,que sao legais = p/produtar).
Te mais!!!!!!!!!!!!!!Peterdirichlet

-- Mensagem = original --

obrigado pela =
resolu=E7=E3o, mas eu queria saber se =E9 possivel fatorar =
apenas
em
IR.....

----- Original Message -----
From: <peterdirichlet@zipmail.= com.br>
To: <obm-l@mat.puc-rio.br>
= Sent: Monday, June 10, 2002 4:33 PM
Subject: [obm-l] Re: [obm-l] = Desafio o retorno!!


Vou usar complexos(a =
paixao de JP):
Seja a expressao f(x)=3Dx^2+x+1.Vamos fatora-la em = R+Ri,i^2+1=3D0. Defina
cis
x=3Dsen x+i*cos =
x=3De^(ix).
Entao w=3D(cis(2*pi/3)) e wbarra=3D(cis(4*pi)/3) sao = zeros de f.
Para as raizes de x^6+x^3+1,ache as raizes cubicas de w e = wbarra.
Te mais!!!!!!!!!!!!!!!!


-- Mensagem original = --

Ol=E1 amigos,
Estou = de volta com outro exerc=EDcio(+dif=EDcil na minha opini=E3o)
Meu = professor disse que nem ele consegue fatorar a seguinte = express=E3o
em
IR:
x^6 + (xy)^3 + = y^6

Abra=E7os,
Bruno

TRANSIRE SVVM PECTVS MVNDOQUE POTIRE
CONGREGATI EX TOTO = ORBE MATHEMATICI OB SCRIPTA INSIGNIA TRIBVERE
Medalha Fields(John = Charles = Fields)


------------------------------------------
Use o = melhor sistema de busca da Internet
Radar UOL - http://www.radaruol.com.br


=3D=3D=3D=3D=3D=3D=3D=3D=3D=3D=3D=3D=3D=3D=3D=3D=3D=3D=3D=3D=3D=3D= =3D=3D=3D=3D=3D=3D=3D=3D=3D=3D=3D=3D=3D=3D=3D=3D=3D=3D=3D=3D=3D=3D=3D=3D=3D= =3D=3D=3D=3D=3D=3D=3D=3D=3D=3D=3D=3D=3D=3D=3D=3D=3D=3D=3D=3D=3D=3D=3D=3D=3D= =3D
Instru=E7=F5es para entrar na lista, sair da lista e usar a lista = em
http://www.m= at.puc-rio.br/~nicolau/olimp/obm-l.html
O administrador desta = lista =E9 <nicolau@mat.puc-rio.br>=
=3D=3D=3D=3D=3D=3D=3D=3D=3D=3D=3D=3D=3D=3D=3D=3D=3D=3D=3D=3D=3D=3D=3D= =3D=3D=3D=3D=3D=3D=3D=3D=3D=3D=3D=3D=3D=3D=3D=3D=3D=3D=3D=3D=3D=3D=3D=3D=3D= =3D=3D=3D=3D=3D=3D=3D=3D=3D=3D=3D=3D=3D=3D=3D=3D=3D=3D=3D=3D=3D=3D=3D=3D=3D=

=3D=3D=3D=3D=3D=3D=3D=3D=3D=3D=3D=3D=3D=3D=3D=3D=3D=3D=3D=3D=3D=
=3D=3D=3D=3D=3D=3D=3D=3D=3D=3D=3D=3D=3D=3D=3D=3D=3D=3D=3D=3D=3D=3D=3D=3D=3D=
=3D=3D=3D=3D=3D=3D=3D=3D=3D=3D=3D=3D=3D=3D=3D=3D=3D=3D=3D=3D=3D=3D=3D=3D=3D=
=3D=3D
Instru=E7=F5es para entrar na lista, sair da lista e usar a = lista em
http://www.m= at.puc-rio.br/~nicolau/olimp/obm-l.html
O administrador desta = lista =E9 <nicolau@mat.puc-rio.br>=
=3D=3D=3D=3D=3D=3D=3D=3D=3D=3D=3D=3D=3D=3D=3D=3D=3D=3D=3D=3D=3D=3D=3D= =3D=3D=3D=3D=3D=3D=3D=3D=3D=3D=3D=3D=3D=3D=3D=3D=3D=3D=3D=3D=3D=3D=3D=3D=3D= =3D=3D=3D=3D=3D=3D=3D=3D=3D=3D=3D=3D=3D=3D=3D=3D=3D=3D=3D=3D=3D=3D=3D=3D=3D=


TRANSIRE SVVM = PECTVS MVNDOQUE POTIRE
CONGREGATI EX TOTO ORBE MATHEMATICI OB SCRIPTA = INSIGNIA TRIBVERE
Medalha Fields(John Charles = Fields)


------------------------------------------
Use o = melhor sistema de busca da Internet
Radar UOL - http://www.radaruol.com.br


=3D=3D=3D=3D=3D=3D=3D=3D=3D=3D=3D=3D=3D=3D=3D=3D=3D=3D=3D=3D=3D=3D= =3D=3D=3D=3D=3D=3D=3D=3D=3D=3D=3D=3D=3D=3D=3D=3D=3D=3D=3D=3D=3D=3D=3D=3D=3D= =3D=3D=3D=3D=3D=3D=3D=3D=3D=3D=3D=3D=3D=3D=3D=3D=3D=3D=3D=3D=3D=3D=3D=3D=3D= =3D
Instru=E7=F5es para entrar na lista, sair da lista e usar a lista = em
http://www.m= at.puc-rio.br/~nicolau/olimp/obm-l.html
O administrador desta = lista =E9 <nicolau@mat.puc-rio.br>=
=3D=3D=3D=3D=3D=3D=3D=3D=3D=3D=3D=3D=3D=3D=3D=3D=3D=3D=3D=3D=3D=3D=3D= =3D=3D=3D=3D=3D=3D=3D=3D=3D=3D=3D=3D=3D=3D=3D=3D=3D=3D=3D=3D=3D=3D=3D=3D=3D= =3D=3D=3D=3D=3D=3D=3D=3D=3D=3D=3D=3D=3D=3D=3D=3D=3D=3D=3D=3D=3D=3D=3D=3D=3D=



------=_NextPart_000_0019_01C217D4.14AEC9A0-- ========================================================================= Instruções para entrar na lista, sair da lista e usar a lista em http://www.mat.puc-rio.br/~nicolau/olimp/obm-l.html O administrador desta lista é ========================================================================= From owner-obm-l@sucuri.mat.puc-rio.br Wed Jun 19 21:30:01 2002 Return-Path: Received: (from majordom@localhost) by sucuri.mat.puc-rio.br (8.9.3/8.9.3) id VAA08057 for obm-l-list; Wed, 19 Jun 2002 21:29:57 -0300 Received: from smtp.ieg.com.br (11.139.226.200.in-addr.arpa.ig.com.br [200.226.139.11] (may be forged)) by sucuri.mat.puc-rio.br (8.9.3/8.9.3) with ESMTP id VAA08053 for ; Wed, 19 Jun 2002 21:29:55 -0300 Received: from igor ([200.222.144.67]) by smtp.ieg.com.br (IeG relay/8.9.3) with SMTP id g5K0PuBf017550 for ; Wed, 19 Jun 2002 21:26:12 -0300 (BRT) Message-ID: <000701c217f1$893982c0$4390dec8@igor> From: "Igor Castro" To: References: Subject: [obm-l] =?iso-8859-1?Q?Re:_=5Bobm-l=5D_Fatora=E7=E3o?= Date: Wed, 19 Jun 2002 21:29:24 -0300 MIME-Version: 1.0 Content-Type: text/plain; charset="iso-8859-1" Content-Transfer-Encoding: 8bit X-Priority: 3 X-MSMail-Priority: Normal X-Mailer: Microsoft Outlook Express 5.00.2615.200 X-MimeOLE: Produced By Microsoft MimeOLE V5.00.2615.200 Sender: owner-obm-l@sucuri.mat.puc-rio.br Precedence: bulk Reply-To: obm-l@mat.puc-rio.br (x^6 + x^3.y^3 + y^6)(x^3 - y^3) = x^9 - y^9 x^6 + x^3.y^3 + y^6= (x^9 - y^9)/(x^3 - y^3) seria isso? []'s Igor... ----- Original Message ----- From: hilhend To: obm-l Sent: Wednesday, June 19, 2002 8:31 PM Subject: [obm-l] Fatoração > Estou enviando a todos novamente aquele expressao a fatorar em reais > X^6 +X^3.Y^3 + Y^6 > pois acredito nao tenha sido visto na imensidao de mensagens. > Se o problema for inconsistente mostre que e. > Um abraço a todos. > > > ========================================================================= > Instruções para entrar na lista, sair da lista e usar a lista em > http://www.mat.puc-rio.br/~nicolau/olimp/obm-l.html > O administrador desta lista é > ========================================================================= ========================================================================= Instruções para entrar na lista, sair da lista e usar a lista em http://www.mat.puc-rio.br/~nicolau/olimp/obm-l.html O administrador desta lista é ========================================================================= From owner-obm-l@sucuri.mat.puc-rio.br Wed Jun 19 22:33:42 2002 Return-Path: Received: (from majordom@localhost) by sucuri.mat.puc-rio.br (8.9.3/8.9.3) id WAA08984 for obm-l-list; Wed, 19 Jun 2002 22:33:09 -0300 Received: from cairu.terra.com.br (cairu.terra.com.br [200.176.3.19]) by sucuri.mat.puc-rio.br (8.9.3/8.9.3) with ESMTP id WAA08980 for ; Wed, 19 Jun 2002 22:33:07 -0300 Received: from taipe.terra.com.br (taipe.terra.com.br [200.176.3.34]) by cairu.terra.com.br (Postfix) with ESMTP id 650694726C for ; Wed, 19 Jun 2002 22:32:48 +0000 (GMT) Received: from niski.com (dl-adsl-C8D44CF7.sao.terra.com.br [200.212.76.247]) (authenticated user fniski) by taipe.terra.com.br (Postfix) with ESMTP id A460A1B408E for ; Wed, 19 Jun 2002 22:32:55 -0300 (EST) Message-ID: <3D1130C8.5030409@niski.com> Date: Wed, 19 Jun 2002 22:32:56 -0300 From: niski User-Agent: Mozilla/5.0 (Windows; U; Windows NT 5.1; en-US; rv:0.9.4.1) Gecko/20020508 Netscape6/6.2.3 X-Accept-Language: en-us MIME-Version: 1.0 To: obm-l@mat.puc-rio.br Subject: Re: [obm-l] =?ISO-8859-1?Q?Fatora=E7=E3o?= References: Content-Type: text/plain; charset=ISO-8859-1; format=flowed Content-Transfer-Encoding: 8bit Sender: owner-obm-l@sucuri.mat.puc-rio.br Precedence: bulk Reply-To: obm-l@mat.puc-rio.br (x^2 - 2.cos (2pi/9).x.y + y^2)(x^2 - 2 cos (4pi/9).x.y + y^2) (x^2 - 2.cos(8pi/9).x.y +y^2) Eu já fatorei e mostrei pro Gleen. Mas eu usei teoria dos complexos e ele falou que nao quer que fatore assim. hilhend wrote: >Estou enviando a todos novamente aquele expressao a fatorar em reais >X^6 +X^3.Y^3 + Y^6 >pois acredito nao tenha sido visto na imensidao de mensagens. >Se o problema for inconsistente mostre que e. >Um abraço a todos. > > >========================================================================= >Instruções para entrar na lista, sair da lista e usar a lista em >http://www.mat.puc-rio.br/~nicolau/olimp/obm-l.html >O administrador desta lista é >========================================================================= > > ========================================================================= Instruções para entrar na lista, sair da lista e usar a lista em http://www.mat.puc-rio.br/~nicolau/olimp/obm-l.html O administrador desta lista é ========================================================================= From owner-obm-l@sucuri.mat.puc-rio.br Wed Jun 19 22:34:21 2002 Return-Path: Received: (from majordom@localhost) by sucuri.mat.puc-rio.br (8.9.3/8.9.3) id WAA09032 for obm-l-list; Wed, 19 Jun 2002 22:34:20 -0300 Received: from calhau.terra.com.br (calhau.terra.com.br [200.176.3.20]) by sucuri.mat.puc-rio.br (8.9.3/8.9.3) with ESMTP id WAA09022 for ; Wed, 19 Jun 2002 22:34:18 -0300 Received: from srv11-sao.terra.com.br (srv11-sao.terra.com.br [200.176.3.38]) by calhau.terra.com.br (Postfix) with ESMTP id 1425847174 for ; Thu, 20 Jun 2002 01:34:07 +0000 (GMT) Received: from niski.com (dl-adsl-C8D44CF7.sao.terra.com.br [200.212.76.247]) (authenticated user fniski) by srv11-sao.terra.com.br (Postfix) with ESMTP id D1DB9C94C1 for ; Wed, 19 Jun 2002 22:34:05 -0300 (EST) Message-ID: <3D11310B.30405@niski.com> Date: Wed, 19 Jun 2002 22:34:03 -0300 From: niski User-Agent: Mozilla/5.0 (Windows; U; Windows NT 5.1; en-US; rv:0.9.4.1) Gecko/20020508 Netscape6/6.2.3 X-Accept-Language: en-us MIME-Version: 1.0 To: obm-l@mat.puc-rio.br Subject: Re: [obm-l] Re: [obm-l] =?ISO-8859-1?Q?Fatora=E7=E3o?= References: <000701c217f1$893982c0$4390dec8@igor> Content-Type: text/plain; charset=ISO-8859-1; format=flowed Content-Transfer-Encoding: 8bit Sender: owner-obm-l@sucuri.mat.puc-rio.br Precedence: bulk Reply-To: obm-l@mat.puc-rio.br Igor Castro wrote: >(x^6 + x^3.y^3 + y^6)(x^3 - y^3) = x^9 - y^9 > >x^6 + x^3.y^3 + y^6= (x^9 - y^9)/(x^3 - y^3) >seria isso? > Nao. Pq o dominio é Reais. Com a sua fatoracao (onde tem uma divisao) x^3 nao pode ser igual a y^3 o que restringe o dominio. > ========================================================================= Instruções para entrar na lista, sair da lista e usar a lista em http://www.mat.puc-rio.br/~nicolau/olimp/obm-l.html O administrador desta lista é ========================================================================= From owner-obm-l@sucuri.mat.puc-rio.br Wed Jun 19 23:03:15 2002 Return-Path: Received: (from majordom@localhost) by sucuri.mat.puc-rio.br (8.9.3/8.9.3) id XAA09857 for obm-l-list; Wed, 19 Jun 2002 23:03:04 -0300 Received: from smtp016.mail.yahoo.com (smtp016.mail.yahoo.com [216.136.174.113]) by sucuri.mat.puc-rio.br (8.9.3/8.9.3) with SMTP id XAA09853 for ; Wed, 19 Jun 2002 23:03:01 -0300 Received: from 200-204-17-232.dial-up.telesp.net.br (HELO e9i5z1) (caio?voznak@200.204.17.232 with login) by smtp.mail.vip.sc5.yahoo.com with SMTP; 20 Jun 2002 02:02:48 -0000 Message-ID: <000c01c2172d$138a67c0$e811ccc8@e9i5z1> From: "Caio H. Voznak" To: Subject: [obm-l] =?iso-8859-1?Q?Equa=E7=E3o_Trigonom=E9trica?= Date: Tue, 18 Jun 2002 22:02:49 -0300 MIME-Version: 1.0 Content-Type: multipart/alternative; boundary="----=_NextPart_000_0009_01C21713.E2B7A9A0" X-Priority: 3 X-MSMail-Priority: Normal X-Mailer: Microsoft Outlook Express 5.50.4133.2400 X-MimeOLE: Produced By Microsoft MimeOLE V5.50.4133.2400 Sender: owner-obm-l@sucuri.mat.puc-rio.br Precedence: bulk Reply-To: obm-l@mat.puc-rio.br This is a multi-part message in MIME format. ------=_NextPart_000_0009_01C21713.E2B7A9A0 Content-Type: text/plain; charset="iso-8859-1" Content-Transfer-Encoding: quoted-printable No desenvolvimento de um exerc=EDcio cheguei a seguinte equa=E7=E3o: {[sen(a)]/[sen(80=BA-a)]} =3D [sen(80=BA)]/[sen(20=BA)] sendo "a" diferente de 80=B0 ser=E1 que alguem poderia me ajudar a desenvolver essa equa=E7=E3o. Um abra=E7o Caio Voznak=20 --- Outgoing mail is certified Virus Free. Checked by AVG anti-virus system (http://www.grisoft.com). Version: 6.0.345 / Virus Database: 193 - Release Date: 9/4/2002 ------=_NextPart_000_0009_01C21713.E2B7A9A0 Content-Type: text/html; charset="iso-8859-1" Content-Transfer-Encoding: quoted-printable
No desenvolvimento de um exerc=EDcio = cheguei a=20 seguinte equa=E7=E3o:
 
{[sen(a)]/[sen(80=BA-a)]} =3D=20 [sen(80=BA)]/[sen(20=BA)]
 
sendo "a" diferente de = 80=B0
 
ser=E1 que alguem poderia me ajudar a = desenvolver=20 essa equa=E7=E3o.
 
Um abra=E7o
 
Caio Voznak 
 

---
Outgoing mail is certified = Virus=20 Free.
Checked by AVG anti-virus system (http://www.grisoft.com).
Version: = 6.0.345 /=20 Virus Database: 193 - Release Date: 9/4/2002
------=_NextPart_000_0009_01C21713.E2B7A9A0-- _________________________________________________________ Do You Yahoo!? Get your free @yahoo.com address at http://mail.yahoo.com ========================================================================= Instruções para entrar na lista, sair da lista e usar a lista em http://www.mat.puc-rio.br/~nicolau/olimp/obm-l.html O administrador desta lista é ========================================================================= From owner-obm-l@sucuri.mat.puc-rio.br Wed Jun 19 23:10:29 2002 Return-Path: Received: (from majordom@localhost) by sucuri.mat.puc-rio.br (8.9.3/8.9.3) id XAA10054 for obm-l-list; Wed, 19 Jun 2002 23:10:27 -0300 Received: from www.zipmail.com.br (smtp.zipmail.com.br [200.187.242.10]) by sucuri.mat.puc-rio.br (8.9.3/8.9.3) with ESMTP id XAA10050 for ; Wed, 19 Jun 2002 23:10:25 -0300 From: luizhenriquerick@zipmail.com.br Received: from [200.165.184.117] by www.zipmail.com.br with HTTP; Wed, 19 Jun 2002 23:10:14 -0300 Message-ID: <3D0E546200003BB4@www.zipmail.com.br> Date: Wed, 19 Jun 2002 23:10:14 -0300 Subject: [obm-l] =?iso-8859-1?Q?Quadril=E1teros?= To: obm-l@mat.puc-rio.br MIME-Version: 1.0 Content-Type: text/plain; charset="iso-8859-1" Content-Transfer-Encoding: 8bit X-MIME-Autoconverted: from quoted-printable to 8bit by sucuri.mat.puc-rio.br id XAA10051 Sender: owner-obm-l@sucuri.mat.puc-rio.br Precedence: bulk Reply-To: obm-l@mat.puc-rio.br Olá amigos , será que poderiam me ajudar nestas duas questões : 1- Seja um quadrilátero inscritível ABCD cujos lados AB , BC , CD e DA medem respectivamente 1 , 2 , 2 e 3 .Calcule a mediana de Euller do quadrilátero e o raio do círculo circunscrito. 2-Um quadrilátero convexo O tem diagonais respectivamente iguais a 4 e 6 .Qual um possível valor para o seu perímetro . Abraço.. ---------------------------------------- |-=Rick-C.R.B.=- | |ICQ 124805654 | |e-mail luizhenriquerick@zipmail.com.br | ---------------------------------------- ------------------------------------------ Use o melhor sistema de busca da Internet Radar UOL - http://www.radaruol.com.br ========================================================================= Instruções para entrar na lista, sair da lista e usar a lista em http://www.mat.puc-rio.br/~nicolau/olimp/obm-l.html O administrador desta lista é ========================================================================= From owner-obm-l@sucuri.mat.puc-rio.br Thu Jun 20 00:54:27 2002 Return-Path: Received: (from majordom@localhost) by sucuri.mat.puc-rio.br (8.9.3/8.9.3) id AAA11580 for obm-l-list; Thu, 20 Jun 2002 00:54:23 -0300 Received: from www.zipmail.com.br (smtp.zipmail.com.br [200.187.242.10]) by sucuri.mat.puc-rio.br (8.9.3/8.9.3) with ESMTP id AAA11576 for ; Thu, 20 Jun 2002 00:54:21 -0300 From: luizhenriquerick@zipmail.com.br Received: from [200.165.184.117] by www.zipmail.com.br with HTTP; Wed, 19 Jun 2002 23:06:26 -0300 Message-ID: <3D0E546200003BAB@www.zipmail.com.br> Date: Wed, 19 Jun 2002 23:06:26 -0300 In-Reply-To: <000701c217f1$893982c0$4390dec8@igor> Subject: [obm-l] =?iso-8859-1?Q?Re=3A=20=5Bobm=2Dl=5D=20Re=3A=20=5Bobm=2Dl=5D=20Fatora=E7=E3o?= To: obm-l@mat.puc-rio.br MIME-Version: 1.0 Content-Type: text/plain; charset="iso-8859-1" Content-Transfer-Encoding: 8bit X-MIME-Autoconverted: from quoted-printable to 8bit by sucuri.mat.puc-rio.br id AAA11577 Sender: owner-obm-l@sucuri.mat.puc-rio.br Precedence: bulk Reply-To: obm-l@mat.puc-rio.br -- Mensagem original -- >(x^6 + x^3.y^3 + y^6)(x^3 - y^3) = x^9 - y^9 > >x^6 + x^3.y^3 + y^6= (x^9 - y^9)/(x^3 - y^3) >seria isso? >[]'s Igor... > >----- Original Message ----- >From: hilhend >To: obm-l >Sent: Wednesday, June 19, 2002 8:31 PM >Subject: [obm-l] Fatoração > > >> Estou enviando a todos novamente aquele expressao a fatorar em reais >> X^6 +X^3.Y^3 + Y^6 >> pois acredito nao tenha sido visto na imensidao de mensagens. >> Se o problema for inconsistente mostre que e. >> Um abraço a todos. >> >> >> ========================================================================= >> Instruções para entrar na lista, sair da lista e usar a lista em >> http://www.mat.puc-rio.br/~nicolau/olimp/obm-l.html >> O administrador desta lista é >> ========================================================================= > > >========================================================================= >Instruções para entrar na lista, sair da lista e usar a lista em >http://www.mat.puc-rio.br/~nicolau/olimp/obm-l.html >O administrador desta lista é >========================================================================= > -- Mensagem original -- >(x^6 + x^3.y^3 + y^6)(x^3 - y^3) = x^9 - y^9 > >x^6 + x^3.y^3 + y^6= (x^9 - y^9)/(x^3 - y^3) >seria isso? >[]'s Igor... > >----- Original Message ----- >From: hilhend >To: obm-l >Sent: Wednesday, June 19, 2002 8:31 PM >Subject: [obm-l] Fatoração > > >> Estou enviando a todos novamente aquele expressao a fatorar em reais >> X^6 +X^3.Y^3 + Y^6 >> pois acredito nao tenha sido visto na imensidao de mensagens. >> Se o problema for inconsistente mostre que e. >> Um abraço a todos. >> >> >> ========================================================================= >> Instruções para entrar na lista, sair da lista e usar a lista em >> http://www.mat.puc-rio.br/~nicolau/olimp/obm-l.html >> O administrador desta lista é >> ========================================================================= > > >========================================================================= >Instruções para entrar na lista, sair da lista e usar a lista em >http://www.mat.puc-rio.br/~nicolau/olimp/obm-l.html >O administrador desta lista é >========================================================================= > E ae igor , como é que ta ? a resolução dessa questão foi feita pelo Morgado a poucos dias , acho que o hilhend não viu , ae esta denovo. x^6 +y^6 + x^3.y^3 = (x^9-y^9)/(x^3-y^3) > x^9-y^9 = Produtorio de (x-ycis 2kpi/9) com k variando de 0 a 8 Os fatores correspondentes a k = 0, 3 e 6, multiplicados dao x^3-y^3. Logo, x^6 +y^6 + x^3.y^3 = Produtorio de (x-ycis 2kpi/9) com k= 1,2,4, 5,7, 8 . Estah fatorado como um produto de 6 fatores complexos de primeiro grau. Grupando os fatores 1-8, 2-7 e 4-5, obtem-se uma fatoraçao em tres fatores reais de grau 2. ---------------------------------------- |-=Rick-C.R.B.=- | |ICQ 124805654 | |e-mail luizhenriquerick@zipmail.com.br | ---------------------------------------- ------------------------------------------ Use o melhor sistema de busca da Internet Radar UOL - http://www.radaruol.com.br ========================================================================= Instruções para entrar na lista, sair da lista e usar a lista em http://www.mat.puc-rio.br/~nicolau/olimp/obm-l.html O administrador desta lista é ========================================================================= From owner-obm-l@sucuri.mat.puc-rio.br Thu Jun 20 14:12:43 2002 Return-Path: Received: (from majordom@localhost) by sucuri.mat.puc-rio.br (8.9.3/8.9.3) id OAA20799 for obm-l-list; Thu, 20 Jun 2002 14:09:59 -0300 Received: from traven10.uol.com.br (traven10.uol.com.br [200.231.206.211]) by sucuri.mat.puc-rio.br (8.9.3/8.9.3) with ESMTP id OAA20795 for ; Thu, 20 Jun 2002 14:09:50 -0300 Received: from xxx ([200.191.176.239]) by traven10.uol.com.br (8.9.1/8.9.1) with SMTP id OAA09065 for ; Thu, 20 Jun 2002 14:09:33 -0300 (BRT) Message-ID: <002701c21886$b3e1d000$efb0bfc8@xxx> From: "haroldo" To: References: <3D0E546200003BB4@www.zipmail.com.br> Subject: [obm-l] =?iso-8859-1?Q?Re:_=5Bobm-l=5D_Quadril=E1teros?= Date: Thu, 20 Jun 2002 15:17:10 -0300 MIME-Version: 1.0 Content-Type: text/plain; charset="iso-8859-1" Content-Transfer-Encoding: 8bit X-Priority: 3 X-MSMail-Priority: Normal X-Mailer: Microsoft Outlook Express 5.00.2919.6600 X-MimeOLE: Produced By Microsoft MimeOLE V5.00.2919.6600 Sender: owner-obm-l@sucuri.mat.puc-rio.br Precedence: bulk Reply-To: obm-l@mat.puc-rio.br sugestão use a relação de ptolomeu ac+bd= pq e use a relação de Hiparco p/q = ( ab + cd)/ (ad+bc) sendo AB = a BC=b CD=c AD=d LADOS E p e q as diagonais. depois calcule p e q. após isso use a relação de Euler a^2 + b^2 + c^2 + d^2 = p^2 + q^2 + 4 m ^ 2 m= mediana de Euler. ok! ----- Original Message ----- From: To: Sent: Wednesday, June 19, 2002 11:10 PM Subject: [obm-l] Quadriláteros > Olá amigos , será que poderiam me ajudar nestas duas questões : > 1- Seja um quadrilátero inscritível ABCD cujos lados AB , BC , CD e DA medem > respectivamente 1 , 2 , 2 e 3 .Calcule a mediana de Euller do quadrilátero > e o raio do círculo circunscrito. > > > 2-Um quadrilátero convexo O tem diagonais respectivamente iguais a 4 e 6 > .Qual um possível valor para o seu perímetro . > > Abraço.. > > > ---------------------------------------- > |-=Rick-C.R.B.=- | > |ICQ 124805654 | > |e-mail luizhenriquerick@zipmail.com.br | > ---------------------------------------- > > > ------------------------------------------ > Use o melhor sistema de busca da Internet > Radar UOL - http://www.radaruol.com.br > > > > ========================================================================= > Instruções para entrar na lista, sair da lista e usar a lista em > http://www.mat.puc-rio.br/~nicolau/olimp/obm-l.html > O administrador desta lista é > ========================================================================= > ========================================================================= Instruções para entrar na lista, sair da lista e usar a lista em http://www.mat.puc-rio.br/~nicolau/olimp/obm-l.html O administrador desta lista é ========================================================================= From owner-obm-l@sucuri.mat.puc-rio.br Thu Jun 20 15:38:40 2002 Return-Path: Received: (from majordom@localhost) by sucuri.mat.puc-rio.br (8.9.3/8.9.3) id PAA22484 for obm-l-list; Thu, 20 Jun 2002 15:37:30 -0300 Received: from smtp.ieg.com.br (12.139.226.200.in-addr.arpa.ig.com.br [200.226.139.12] (may be forged)) by sucuri.mat.puc-rio.br (8.9.3/8.9.3) with ESMTP id PAA22480 for ; Thu, 20 Jun 2002 15:37:20 -0300 Received: from igor ([200.222.134.158]) by smtp.ieg.com.br (IeG relay/8.9.3) with SMTP id g5KIcW9T098977 for ; Thu, 20 Jun 2002 15:38:40 -0300 (BRT) Message-ID: <003201c21889$7332f220$9e86dec8@igor> From: "Igor Castro" To: References: <3D0E546200003BB4@www.zipmail.com.br> Subject: [obm-l] =?iso-8859-1?Q?Re:_=5Bobm-l=5D_Quadril=E1teros?= Date: Thu, 20 Jun 2002 15:36:49 -0300 MIME-Version: 1.0 Content-Type: text/plain; charset="iso-8859-1" Content-Transfer-Encoding: 8bit X-Priority: 3 X-MSMail-Priority: Normal X-Mailer: Microsoft Outlook Express 5.00.2615.200 X-MimeOLE: Produced By Microsoft MimeOLE V5.00.2615.200 Sender: owner-obm-l@sucuri.mat.puc-rio.br Precedence: bulk Reply-To: obm-l@mat.puc-rio.br Fala ae rick Na primeira ache as duas diagonais(usando hiparco e depois ptolomeu), em seguida aplique a relação da mediana de euller... a para achar o raio, use a formula abc/4R em um triangulo que possua uma diagonal como lado(assim ele estará inscrito)... no segundo analisei apenas com existencia de triangulos e cheguei que o perimetro é maior que 10... mas um valor exato n achei... []'s ----- Original Message ----- From: To: Sent: Wednesday, June 19, 2002 11:10 PM Subject: [obm-l] Quadriláteros > Olá amigos , será que poderiam me ajudar nestas duas questões : > 1- Seja um quadrilátero inscritível ABCD cujos lados AB , BC , CD e DA medem > respectivamente 1 , 2 , 2 e 3 .Calcule a mediana de Euller do quadrilátero > e o raio do círculo circunscrito. > > > 2-Um quadrilátero convexo O tem diagonais respectivamente iguais a 4 e 6 > .Qual um possível valor para o seu perímetro . > > Abraço.. > > > ---------------------------------------- > |-=Rick-C.R.B.=- | > |ICQ 124805654 | > |e-mail luizhenriquerick@zipmail.com.br | > ---------------------------------------- > > > ------------------------------------------ > Use o melhor sistema de busca da Internet > Radar UOL - http://www.radaruol.com.br > > > > ========================================================================= > Instruções para entrar na lista, sair da lista e usar a lista em > http://www.mat.puc-rio.br/~nicolau/olimp/obm-l.html > O administrador desta lista é > ========================================================================= > ========================================================================= Instruções para entrar na lista, sair da lista e usar a lista em http://www.mat.puc-rio.br/~nicolau/olimp/obm-l.html O administrador desta lista é ========================================================================= From owner-obm-l@sucuri.mat.puc-rio.br Thu Jun 20 17:56:40 2002 Return-Path: Received: (from majordom@localhost) by sucuri.mat.puc-rio.br (8.9.3/8.9.3) id RAA24710 for obm-l-list; Thu, 20 Jun 2002 17:54:17 -0300 Received: from smtp-6.ig.com.br (smtp-6.ig.com.br [200.226.132.155]) by sucuri.mat.puc-rio.br (8.9.3/8.9.3) with SMTP id RAA24706 for ; Thu, 20 Jun 2002 17:54:15 -0300 Received: (qmail 27875 invoked from network); 20 Jun 2002 20:53:37 -0000 Received: from shasta030078.ig.com.br (HELO windows9) (200.151.30.78) by smtp-6.ig.com.br with SMTP; 20 Jun 2002 20:53:37 -0000 Message-ID: <006e01c2189c$d383f300$4e1e97c8@windows9> From: "Eric Campos Bastos Guedes" To: References: <20020617044730.44549.qmail@web10104.mail.yahoo.com> Subject: Re: [obm-l] 3 circulos! Date: Thu, 20 Jun 2002 17:50:58 -0300 MIME-Version: 1.0 Content-Type: text/plain; charset="iso-8859-1" Content-Transfer-Encoding: 8bit X-Priority: 3 X-MSMail-Priority: Normal X-Mailer: Microsoft Outlook Express 5.00.2615.200 X-MIMEOLE: Produced By Microsoft MimeOLE V5.00.2615.200 Sender: owner-obm-l@sucuri.mat.puc-rio.br Precedence: bulk Reply-To: obm-l@mat.puc-rio.br > Os raios dos circulos inscritos num triangulo > retangulo ABC e nos dois triangulos ABH e ACH > determinados pela altura relativa a hipotenusa BC sao > respectivamente r ,r1 e r2. Demonstrar que: > r² = r1² + r2² Sejam A, A1, A2 as areas dos circulos inscritos em ABC, ABH, ACH respectivamente. Sejam S, S1, S2 as areas dos triangulos ABC, ABH, ACH respectivamente. Por semelhanca, tem-se A/S = A1/S1 = A2/S2 Como A1/S1 = A2/S2 = A/S entao (A1+A2)/(S1+S2) = A/S, mas S1 + S2 = S, donde (A1+A2)/S = A/S logo A1 + A2 = A ou 2*Pi*r1^2 + 2*Pi*r2^2 = 2*Pi*r^2 cancelando 2*Pi, tem-se r1^2 + r2^2 = r^2 Eric. > Valeu! > ========================================================================= Instruções para entrar na lista, sair da lista e usar a lista em http://www.mat.puc-rio.br/~nicolau/olimp/obm-l.html O administrador desta lista é ========================================================================= From owner-obm-l@sucuri.mat.puc-rio.br Thu Jun 20 21:02:58 2002 Return-Path: Received: (from majordom@localhost) by sucuri.mat.puc-rio.br (8.9.3/8.9.3) id VAA27011 for obm-l-list; Thu, 20 Jun 2002 21:02:44 -0300 Received: from sr1.terra.com.br (sr1.terra.com.br [200.176.3.16]) by sucuri.mat.puc-rio.br (8.9.3/8.9.3) with ESMTP id VAA27004 for ; Thu, 20 Jun 2002 21:02:42 -0300 Received: from una.terra.com.br (una.terra.com.br [200.176.3.184]) by sr1.terra.com.br (Postfix) with ESMTP id A57916EF29 for ; Thu, 20 Jun 2002 21:02:31 -0300 (EST) Received: from terra.com.br (una.terra.com.br [200.176.3.184]) by una.terra.com.br (Postfix) with ESMTP id 7E14C14AC17 for ; Thu, 20 Jun 2002 21:02:31 -0300 (EST) Date: Fri, 21 Jun 2002 00:02:31 +0000 Message-Id: Subject: [obm-l] =?iso-8859-1?q?Re=3A_=5Bobm=2Dl=5D_Re=3A_=5Bobm=2Dl=5D_Re=3A_=5Bobm=2Dl?= =?iso-8859-1?q?=5D_Fatora=E7=E3o?= MIME-Version: 1.0 Content-Type: text/plain;charset="iso-8859-1" From: "hilhend" To: "obm-l" X-XaM3-API-Version: 2.4.3.2.9 X-SenderIP: 200.161.14.239 Content-Transfer-Encoding: 8bit X-MIME-Autoconverted: from quoted-printable to 8bit by sucuri.mat.puc-rio.br id VAA27005 Sender: owner-obm-l@sucuri.mat.puc-rio.br Precedence: bulk Reply-To: obm-l@mat.puc-rio.br Nao pois suponha x=1 e y=1 1^6 + 1^3.1^3 + 1^6 = 3 que e diferente de (1^9 - 1^9)/(1^3 - 1^3) Detalhe eu falei fatoracao em reais e nao em complexos! Muito obrigado pela forca, creio que chegaremos ha algum lugar logo logo. Ate > -- Mensagem original -- > > >(x^6 + x^3.y^3 + y^6)(x^3 - y^3) = x^9 - y^9 > > > >x^6 + x^3.y^3 + y^6= (x^9 - y^9)/(x^3 - y^3) > >seria isso? > >[]'s Igor... > > > >----- Original Message ----- > >From: hilhend > >To: obm-l > >Sent: Wednesday, June 19, 2002 8:31 PM > >Subject: [obm-l] Fatoração > > > > > >> Estou enviando a todos novamente aquele expressao a fatorar em re ais > >> X^6 +X^3.Y^3 + Y^6 > >> pois acredito nao tenha sido visto na imensidao de mensagens. > >> Se o problema for inconsistente mostre que e. > >> Um abraço a todos. > >> > >> > > > E ae igor , como é que ta ? > a resolução dessa questão foi feita pelo Morgado a poucos dias , ach o que > o hilhend não viu , ae esta denovo. > x^6 +y^6 + x^3.y^3 = (x^9-y^9)/(x^3-y^3) > > > x^9-y^9 = Produtorio de (x-ycis 2kpi/9) com k variando de 0 a 8 > Os fatores correspondentes a k = 0, 3 e 6, multiplicados dao x^3- y^3. > Logo, > > x^6 +y^6 + x^3.y^3 = Produtorio de (x- ycis 2kpi/9) com k= 1,2,4, 5,7, 8 > . Estah fatorado como um produto de 6 fatores complexos de primeiro grau. > Grupando os fatores 1-8, 2-7 e 4-5, obtem- se uma fatoraçao em tres fatores > reais de grau 2. > > > > > ---------------------------------------- > |-=Rick-C.R.B.=- | > |ICQ 124805654 | > |e-mail luizhenriquerick@zipmail.com.br | > ---------------------------------------- > > > ------------------------------------------ > Use o melhor sistema de busca da Internet > Radar UOL - http://www.radaruol.com.br > > > > ==================================================================== ===== > Instruções para entrar na lista, sair da lista e usar a lista em > http://www.mat.puc-rio.br/~nicolau/olimp/obm-l.html > O administrador desta lista é > ==================================================================== ===== > > ========================================================================= Instruções para entrar na lista, sair da lista e usar a lista em http://www.mat.puc-rio.br/~nicolau/olimp/obm-l.html O administrador desta lista é ========================================================================= From owner-obm-l@sucuri.mat.puc-rio.br Thu Jun 20 21:40:58 2002 Return-Path: Received: (from majordom@localhost) by sucuri.mat.puc-rio.br (8.9.3/8.9.3) id VAA27998 for obm-l-list; Thu, 20 Jun 2002 21:40:55 -0300 Received: from gorgo.centroin.com.br (gorgo.centroin.com.br [200.225.63.128]) by sucuri.mat.puc-rio.br (8.9.3/8.9.3) with ESMTP id VAA27994 for ; Thu, 20 Jun 2002 21:40:53 -0300 Received: from centroin.com.br (du68c.rjo.centroin.com.br [200.225.58.68]) (authenticated bits=0) by gorgo.centroin.com.br (8.12.2/8.12.1) with ESMTP id g5L0fI6K004763 for ; Thu, 20 Jun 2002 21:41:18 -0300 (BRT) Message-ID: <3D127680.7030300@centroin.com.br> Date: Thu, 20 Jun 2002 21:42:40 -0300 From: Augusto =?ISO-8859-1?Q?C=E9sar?= Morgado User-Agent: Mozilla/5.0 (Windows; U; Win98; en-US; rv:0.9.4.1) Gecko/20020508 Netscape6/6.2.3 X-Accept-Language: en-us MIME-Version: 1.0 To: obm-l@mat.puc-rio.br Subject: Re: [obm-l] =?ISO-8859-1?Q?Equa=E7=E3o=20Trigonom=E9trica?= References: <000c01c2172d$138a67c0$e811ccc8@e9i5z1> Content-Type: multipart/alternative; boundary="------------090201030102040908070200" Sender: owner-obm-l@sucuri.mat.puc-rio.br Precedence: bulk Reply-To: obm-l@mat.puc-rio.br --------------090201030102040908070200 Content-Type: text/plain; charset=ISO-8859-1; format=flowed Content-Transfer-Encoding: 8bit sena sen20 = sen 80 sen(80-a) sena sen20 = sen80 [sen80 cosa - sena cos 80] sena sen 20 = sen80 sen80 cosa - sen80 cos80 sena sena sen20 = sen80 sen80 cosa - (1/2) sen160 sena sena sen20 = sen80 sen80 cosa - (1/2) sen20 sena (3/2) sena sen20 = (1/2) (1-cos160) cosa 3 sena sen20 = (1+cos20) cosa tana = (1/3) [(1+cos20)/sen20] = (1/3) [2 sen10 sen10 / 2 sen10 cos10] = (1/3) tan10 a = arctan [(1/3) tan10] + k180 = 3,36 + k180 aproximadamente Caio H. Voznak wrote: > No desenvolvimento de um exercício cheguei a seguinte equação: > > > > {[sen(a)]/[sen(80º-a)]} = [sen(80º)]/[sen(20º)] > > > > sendo "a" diferente de 80° > > > > será que alguem poderia me ajudar a desenvolver essa equação. > > > > Um abraço > > > > Caio Voznak > > > > > --- > Outgoing mail is certified Virus Free. > Checked by AVG anti-virus system (http://www.grisoft.com ). > Version: 6.0.345 / Virus Database: 193 - Release Date: 9/4/2002 > --------------090201030102040908070200 Content-Type: text/html; charset=us-ascii Content-Transfer-Encoding: 7bit sena sen20 = sen 80 sen(80-a)
sena sen20 = sen80 [sen80 cosa - sena cos 80]  
sena sen 20 = sen80 sen80 cosa - sen80 cos80 sena
sena sen20 = sen80 sen80 cosa - (1/2) sen160 sena
sena sen20 = sen80 sen80 cosa - (1/2) sen20 sena
 (3/2) sena sen20 = (1/2) (1-cos160) cosa
3 sena sen20 = (1+cos20) cosa
tana = (1/3) [(1+cos20)/sen20] = (1/3) [2 sen10 sen10 / 2 sen10 cos10] = (1/3) tan10
a = arctan [(1/3) tan10] + k180 = 3,36 + k180 aproximadamente

Caio H. Voznak wrote:
No desenvolvimento de um exercício cheguei a seguinte equação:
 
{[sen(a)]/[sen(80º-a)]} = [sen(80º)]/[sen(20º)]
 
sendo "a" diferente de 80°
 
será que alguem poderia me ajudar a desenvolver essa equação.
 
Um abraço
 
Caio Voznak 
 

---
Outgoing mail is certified Virus Free.
Checked by AVG anti-virus system (http://www.grisoft.com ).
Version: 6.0.345 / Virus Database: 193 - Release Date: 9/4/2002

--------------090201030102040908070200-- ========================================================================= Instruções para entrar na lista, sair da lista e usar a lista em http://www.mat.puc-rio.br/~nicolau/olimp/obm-l.html O administrador desta lista é ========================================================================= From owner-obm-l@sucuri.mat.puc-rio.br Thu Jun 20 22:24:21 2002 Return-Path: Received: (from majordom@localhost) by sucuri.mat.puc-rio.br (8.9.3/8.9.3) id WAA28716 for obm-l-list; Thu, 20 Jun 2002 22:24:14 -0300 Received: from smtp013.mail.yahoo.com (smtp013.mail.yahoo.com [216.136.173.57]) by sucuri.mat.puc-rio.br (8.9.3/8.9.3) with SMTP id WAA28712 for ; Thu, 20 Jun 2002 22:24:11 -0300 Received: from 200-204-17-250.dial-up.telesp.net.br (HELO e9i5z1) (caio?voznak@200.204.17.250 with login) by smtp.mail.vip.sc5.yahoo.com with SMTP; 21 Jun 2002 01:24:01 -0000 Message-ID: <000901c217f0$d68ab680$fa11ccc8@e9i5z1> From: "Caio H. Voznak" To: References: <000c01c2172d$138a67c0$e811ccc8@e9i5z1> <3D127680.7030300@centroin.com.br> Subject: [obm-l] =?iso-8859-1?Q?Re:_=5Bobm-l=5D_Equa=E7=E3o_Trigonom=E9trica?= Date: Wed, 19 Jun 2002 21:24:20 -0300 MIME-Version: 1.0 Content-Type: multipart/alternative; boundary="----=_NextPart_000_0006_01C217D7.ACE213A0" X-Priority: 3 X-MSMail-Priority: Normal X-Mailer: Microsoft Outlook Express 5.50.4133.2400 X-MimeOLE: Produced By Microsoft MimeOLE V5.50.4133.2400 Sender: owner-obm-l@sucuri.mat.puc-rio.br Precedence: bulk Reply-To: obm-l@mat.puc-rio.br This is a multi-part message in MIME format. ------=_NextPart_000_0006_01C217D7.ACE213A0 Content-Type: text/plain; charset="iso-8859-1" Content-Transfer-Encoding: quoted-printable Caro Augusto Na passagem: sena sen20 =3D sen80 sen80 cosa - (1/2) sen160 sena sena sen20 =3D sen80 sen80 cosa - (1/2) sen20 sena N=E3o seria cos20 no lugar de sen20, o que compromete a = resolu=E7=E3o. ----- Original Message -----=20 From: Augusto C=E9sar Morgado=20 To: obm-l@mat.puc-rio.br=20 Sent: Thursday, June 20, 2002 9:42 PM Subject: Re: [obm-l] Equa=E7=E3o Trigonom=E9trica sena sen20 =3D sen 80 sen(80-a) sena sen20 =3D sen80 [sen80 cosa - sena cos 80] =20 sena sen 20 =3D sen80 sen80 cosa - sen80 cos80 sena sena sen20 =3D sen80 sen80 cosa - (1/2) sen160 sena sena sen20 =3D sen80 sen80 cosa - (1/2) sen20 sena (3/2) sena sen20 =3D (1/2) (1-cos160) cosa 3 sena sen20 =3D (1+cos20) cosa tana =3D (1/3) [(1+cos20)/sen20] =3D (1/3) [2 sen10 sen10 / 2 sen10 = cos10] =3D (1/3) tan10 a =3D arctan [(1/3) tan10] + k180 =3D 3,36 + k180 aproximadamente Caio H. Voznak wrote: No desenvolvimento de um exerc=EDcio cheguei a seguinte equa=E7=E3o: {[sen(a)]/[sen(80=BA-a)]} =3D [sen(80=BA)]/[sen(20=BA)] sendo "a" diferente de 80=B0 ser=E1 que alguem poderia me ajudar a desenvolver essa equa=E7=E3o. Um abra=E7o Caio Voznak=20 --- Outgoing mail is certified Virus Free. Checked by AVG anti-virus system (http://www.grisoft.com ). Version: 6.0.345 / Virus Database: 193 - Release Date: 9/4/2002 --- Outgoing mail is certified Virus Free. Checked by AVG anti-virus system (http://www.grisoft.com). Version: 6.0.345 / Virus Database: 193 - Release Date: 9/4/2002 ------=_NextPart_000_0006_01C217D7.ACE213A0 Content-Type: text/html; charset="iso-8859-1" Content-Transfer-Encoding: quoted-printable
Caro Augusto
 
    Na = passagem:
 
    sena sen20 =3D sen80 sen80 cosa - (1/2) sen160 = sena
   =20 sena sen20 =3D sen80 sen80 cosa - (1/2) sen20 sena
 
     N=E3o seria = cos20 no lugar=20 de sen20, o que compromete a resolu=E7=E3o.
 
 
----- Original Message -----
From:=20 Augusto=20 C=E9sar Morgado
Sent: Thursday, June 20, 2002 = 9:42=20 PM
Subject: Re: [obm-l] = Equa=E7=E3o=20 Trigonom=E9trica

sena sen20 =3D sen 80 sen(80-a)
sena sen20 =3D sen80 = [sen80 cosa=20 - sena cos 80]  
sena sen 20 =3D sen80 sen80 cosa - sen80 = cos80=20 sena
sena sen20 =3D sen80 sen80 cosa - (1/2) sen160 sena
sena = sen20 =3D=20 sen80 sen80 cosa - (1/2) sen20 sena
 (3/2) sena sen20 =3D = (1/2)=20 (1-cos160) cosa
3 sena sen20 =3D (1+cos20) cosa
tana =3D (1/3)=20 [(1+cos20)/sen20] =3D (1/3) [2 sen10 sen10 / 2 sen10 cos10] =3D (1/3) = tan10
a =3D=20 arctan [(1/3) tan10] + k180 =3D 3,36 + k180 = aproximadamente

Caio H.=20 Voznak wrote:
No desenvolvimento de um = exerc=EDcio cheguei a=20 seguinte equa=E7=E3o:
 
{[sen(a)]/[sen(80=BA-a)]} =3D=20 [sen(80=BA)]/[sen(20=BA)]
 
sendo "a" diferente de = 80=B0
 
ser=E1 que alguem poderia me ajudar = a desenvolver=20 essa equa=E7=E3o.
 
Um abra=E7o
 
Caio Voznak 
 

---
Outgoing mail is = certified Virus=20 Free.
Checked by AVG anti-virus system (http://www.grisoft.com = ).
Version:=20 6.0.345 / Virus Database: 193 - Release Date:=20 9/4/2002


---
Outgoing mail is certified Virus = Free.
Checked=20 by AVG anti-virus system (http://www.grisoft.com).
Version: = 6.0.345=20 / Virus Database: 193 - Release Date: = 9/4/2002 ------=_NextPart_000_0006_01C217D7.ACE213A0-- _________________________________________________________ Do You Yahoo!? Get your free @yahoo.com address at http://mail.yahoo.com ========================================================================= Instruções para entrar na lista, sair da lista e usar a lista em http://www.mat.puc-rio.br/~nicolau/olimp/obm-l.html O administrador desta lista é ========================================================================= From owner-obm-l@sucuri.mat.puc-rio.br Thu Jun 20 23:21:41 2002 Return-Path: Received: (from majordom@localhost) by sucuri.mat.puc-rio.br (8.9.3/8.9.3) id XAA29486 for obm-l-list; Thu, 20 Jun 2002 23:21:34 -0300 Received: from gorgo.centroin.com.br (gorgo.centroin.com.br [200.225.63.128]) by sucuri.mat.puc-rio.br (8.9.3/8.9.3) with ESMTP id XAA29482 for ; Thu, 20 Jun 2002 23:21:32 -0300 Received: from centroin.com.br (du23c.rjo.centroin.com.br [200.225.58.23]) (authenticated bits=0) by gorgo.centroin.com.br (8.12.2/8.12.1) with ESMTP id g5L2Lv6K015952 for ; Thu, 20 Jun 2002 23:21:58 -0300 (BRT) Message-ID: <3D128E17.3060804@centroin.com.br> Date: Thu, 20 Jun 2002 23:23:19 -0300 From: Augusto =?ISO-8859-1?Q?C=E9sar?= Morgado User-Agent: Mozilla/5.0 (Windows; U; Win98; en-US; rv:0.9.4.1) Gecko/20020508 Netscape6/6.2.3 X-Accept-Language: en-us MIME-Version: 1.0 To: obm-l@mat.puc-rio.br Subject: [obm-l] problema do ortocentro Content-Type: text/plain; charset=ISO-8859-1; format=flowed Content-Transfer-Encoding: 8bit Sender: owner-obm-l@sucuri.mat.puc-rio.br Precedence: bulk Reply-To: obm-l@mat.puc-rio.br Um problema que me pareceu interessante. Determinar o raio do círculo circunscrito a um triângulo dados: a base AB=12 e a distância de seu ortocentro ao vértice C = 5. ========================================================================= Instruções para entrar na lista, sair da lista e usar a lista em http://www.mat.puc-rio.br/~nicolau/olimp/obm-l.html O administrador desta lista é ========================================================================= From owner-obm-l@sucuri.mat.puc-rio.br Thu Jun 20 23:33:08 2002 Return-Path: Received: (from majordom@localhost) by sucuri.mat.puc-rio.br (8.9.3/8.9.3) id XAA29689 for obm-l-list; Thu, 20 Jun 2002 23:32:45 -0300 Received: from hotmail.com (f82.law9.hotmail.com [64.4.9.82]) by sucuri.mat.puc-rio.br (8.9.3/8.9.3) with ESMTP id XAA29685 for ; Thu, 20 Jun 2002 23:32:42 -0300 Received: from mail pickup service by hotmail.com with Microsoft SMTPSVC; Thu, 20 Jun 2002 19:32:34 -0700 Received: from 200.190.9.208 by lw9fd.law9.hotmail.msn.com with HTTP; Fri, 21 Jun 2002 02:32:34 GMT X-Originating-IP: [200.190.9.208] From: "Rogerio Fajardo" To: obm-l@mat.puc-rio.br Subject: Re: [obm-l] Axiomas de Peano Date: Fri, 21 Jun 2002 02:32:34 +0000 Mime-Version: 1.0 Content-Type: text/plain; format=flowed Message-ID: X-OriginalArrivalTime: 21 Jun 2002 02:32:34.0747 (UTC) FILETIME=[E6AF48B0:01C218CB] Sender: owner-obm-l@sucuri.mat.puc-rio.br Precedence: bulk Reply-To: obm-l@mat.puc-rio.br Aproveitando que o assunto é axiomas de peano, é possível definir aritmética usando só os axiomas de peano (usando lógica de 1ªordem e os símbolos 0 e sucessor de)? Dá pra fazer uma espécie de teorema da recursão nos axiomas de Peano? Ou será necessário usar lógica de segunda ordem, ou esquema de prova? >From: "Bruno F. C. Leite" >Reply-To: obm-l@mat.puc-rio.br >To: obm-l@mat.puc-rio.br >Subject: Re: [obm-l] Axiomas de Peano >Date: Tue, 18 Jun 2002 16:03:38 -0300 > >At 15:29 18/06/02 -0300, you wrote: >>Na Eureka 3, p. 26, há um artigo de Elon Lages Lima chamado "O Princípio >>da >>Indução", onde o autor afirma que o conjunto N dos números naturais é >>caracterizado pelas seguintes propriedades: >> >>A) Existe função s: N -> N, que associa a cada n pertencente a N um >>elemento >>s(n) pertecente a N, chamado o sucessor de n. >> >>B) A função s: N-> N é injetiva. >> >>C) Existe um único elemento 1 no conjunto N, tal que 1 != s(n) para todo n >>pertencente a N. >> >>D) Se um subconjunto X contido em N é tal que 1 pertence a N e s(X) está >>contido em X. > >Não me lembro do artigo, mas isto está certo mesmo? > Acho que o certo é "se um subconjunto X contido em N é tal que 1 pertence >a N e se n está em X implica que s(n) também está, então X=N" (princípio de >indução) > >Com isso o conjunto que você falou (V) não satisfaz a última condição. > >Bruno Leite >http://www.ime.usp.br/~brleite > > > >>As afirmações A, B, C e D são os axiomas de Peano. >> >>Agora vem a minha dúvida. Imagine o conjunto de números: >>V = {0, 1, 2, 3, ...} U {a}, onde o elemento 'a' não pertence a {0, 1, 2, >>3, >>...} >>e a função injetiva s: V -> V onde: >>s(x) = a, se x=a; senão s(x) = x+1 >> >>Temos, então, o conjunto V e a função s que satisfazem os axiomas de >>Peano. >>Dessa forma, podemos dizer que V é o conjunto dos número naturais, mas não >>é!!!!! >>Qual o problema aí??? >> >>Alguém pode esclarecer a minha dúvida? >> >>Obrigado >> >>Vinicius Fortuna >> >> >> >> >>========================================================================= >>Instruções para entrar na lista, sair da lista e usar a lista em >>http://www.mat.puc-rio.br/~nicolau/olimp/obm-l.html >>O administrador desta lista é >>========================================================================= > >========================================================================= >Instruções para entrar na lista, sair da lista e usar a lista em >http://www.mat.puc-rio.br/~nicolau/olimp/obm-l.html >O administrador desta lista é >========================================================================= _________________________________________________________________ Get your FREE download of MSN Explorer at http://explorer.msn.com/intl.asp. ========================================================================= Instruções para entrar na lista, sair da lista e usar a lista em http://www.mat.puc-rio.br/~nicolau/olimp/obm-l.html O administrador desta lista é ========================================================================= From owner-obm-l@sucuri.mat.puc-rio.br Fri Jun 21 00:42:32 2002 Return-Path: Received: (from majordom@localhost) by sucuri.mat.puc-rio.br (8.9.3/8.9.3) id AAA31123 for obm-l-list; Fri, 21 Jun 2002 00:42:25 -0300 Received: from mail.gmx.net (mail.gmx.net [213.165.64.20]) by sucuri.mat.puc-rio.br (8.9.3/8.9.3) with SMTP id AAA31119 for ; Fri, 21 Jun 2002 00:42:22 -0300 Received: (qmail 7843 invoked by uid 0); 21 Jun 2002 03:42:12 -0000 Received: from unknown (HELO gomes) (200.216.104.137) by mail.gmx.net (mp010-rz3) with SMTP; 21 Jun 2002 03:42:12 -0000 Date: Fri, 21 Jun 2002 00:45:08 -0300 From: Igor GomeZZ X-Mailer: The Bat! (v1.60c) Organization: -- X-Priority: 3 (Normal) Message-ID: <7722846962.20020621004508@gmx.net> To: "[OBM]" Subject: [obm-l] =?ISO-8859-1?B?UmU6IFtvYm0tbF0gUXVhZHJpbOF0ZXJvcw==?= In-Reply-To: <003201c21889$7332f220$9e86dec8@igor> References: <3D0E546200003BB4@www.zipmail.com.br> <003201c21889$7332f220$9e86dec8@igor> MIME-Version: 1.0 Content-Type: text/plain; charset=ISO-8859-1 Content-Transfer-Encoding: 8bit Sender: owner-obm-l@sucuri.mat.puc-rio.br Precedence: bulk Reply-To: obm-l@mat.puc-rio.br Em 20/6/2002, 15:36, Igor (cnaval@ieg.com.br) disse: > Na primeira ache as duas diagonais(usando hiparco e depois ptolomeu), em > seguida aplique a relação da mediana de euller... Alguém pode enunciar o teorema de Hiparco pra mim? O de Ptolomeu conheço (Eureka5), mas nunca li sobre esse outro... Fui! ####### Igor GomeZZ ######## UIN: 29249895 Vitória, Espírito Santo, Brasil Criação: 21/6/2002 (00:41) #################################### Pare para pensar: A arte da medicina consiste em distrair enquanto a Natureza cuida da doença. (Voltaire) #################################### ========================================================================= Instruções para entrar na lista, sair da lista e usar a lista em http://www.mat.puc-rio.br/~nicolau/olimp/obm-l.html O administrador desta lista é ========================================================================= From owner-obm-l@sucuri.mat.puc-rio.br Fri Jun 21 00:47:28 2002 Return-Path: Received: (from majordom@localhost) by sucuri.mat.puc-rio.br (8.9.3/8.9.3) id AAA31203 for obm-l-list; Fri, 21 Jun 2002 00:47:25 -0300 Received: from smtp-5.ig.com.br (smtp-5.ig.com.br [200.226.132.154]) by sucuri.mat.puc-rio.br (8.9.3/8.9.3) with SMTP id AAA31198 for ; Fri, 21 Jun 2002 00:47:23 -0300 Received: (qmail 12431 invoked from network); 21 Jun 2002 03:46:59 -0000 Received: from unknown (HELO thomasPIII) (200.201.252.169) by smtp-5.ig.com.br with SMTP; 21 Jun 2002 03:46:59 -0000 Message-ID: <02ca01c218d6$ae201000$6600a8c0@altus.com.br> From: "Thomas de Rossi" To: References: <000701c217f1$893982c0$4390dec8@igor> <3D11310B.30405@niski.com> Subject: [obm-l] =?iso-8859-1?Q?Re:_=5Bobm-l=5D_Re:_=5Bobm-l=5D_Fatora=E7=E3o?= Date: Thu, 20 Jun 2002 21:22:07 -0300 MIME-Version: 1.0 Content-Type: text/plain; charset="iso-8859-1" Content-Transfer-Encoding: 8bit X-Priority: 3 X-MSMail-Priority: Normal X-Mailer: Microsoft Outlook Express 5.00.2919.6600 X-MIMEOLE: Produced By Microsoft MimeOLE V5.00.2919.6600 Sender: owner-obm-l@sucuri.mat.puc-rio.br Precedence: bulk Reply-To: obm-l@mat.puc-rio.br Oi pessoal, Olhei para a fatoração e não entendi a explicação: > Nao. Pq o dominio é Reais. Com a sua fatoracao (onde tem uma divisao) > x^3 nao pode ser igual a y^3 o que restringe o dominio. Poderiam ser mais didáticos na explicação, Sds: Thomas. ----- Original Message ----- From: "niski" To: Sent: Wednesday, June 19, 2002 10:34 PM Subject: Re: [obm-l] Re: [obm-l] Fatoração > > > Igor Castro wrote: > > >(x^6 + x^3.y^3 + y^6)(x^3 - y^3) = x^9 - y^9 > > > >x^6 + x^3.y^3 + y^6= (x^9 - y^9)/(x^3 - y^3) > >seria isso? > > > > Nao. Pq o dominio é Reais. Com a sua fatoracao (onde tem uma divisao) > x^3 nao pode ser igual a y^3 o que restringe o dominio. > > > > > ========================================================================= > Instruções para entrar na lista, sair da lista e usar a lista em > http://www.mat.puc-rio.br/~nicolau/olimp/obm-l.html > O administrador desta lista é > ========================================================================= ========================================================================= Instruções para entrar na lista, sair da lista e usar a lista em http://www.mat.puc-rio.br/~nicolau/olimp/obm-l.html O administrador desta lista é ========================================================================= From owner-obm-l@sucuri.mat.puc-rio.br Fri Jun 21 00:47:29 2002 Return-Path: Received: (from majordom@localhost) by sucuri.mat.puc-rio.br (8.9.3/8.9.3) id AAA31214 for obm-l-list; Fri, 21 Jun 2002 00:47:28 -0300 Received: from smtp-5.ig.com.br (smtp-5.ig.com.br [200.226.132.154]) by sucuri.mat.puc-rio.br (8.9.3/8.9.3) with SMTP id AAA31200 for ; Fri, 21 Jun 2002 00:47:25 -0300 Received: (qmail 12451 invoked from network); 21 Jun 2002 03:47:01 -0000 Received: from unknown (HELO thomasPIII) (200.201.252.169) by smtp-5.ig.com.br with SMTP; 21 Jun 2002 03:47:01 -0000 Message-ID: <02cb01c218d6$af12ad60$6600a8c0@altus.com.br> From: "Thomas de Rossi" To: "Obm-l" References: <002a01c217c7$35a790e0$2f87dec8@igor> <20020619165106.B3071@sucuri.mat.puc-rio.br> Subject: [obm-l] =?iso-8859-1?Q?Re:_=5Bobm-l=5D_Re:_=5Bobm-l=5D_Ajuda_r=E1pida...?= Date: Thu, 20 Jun 2002 21:38:05 -0300 MIME-Version: 1.0 Content-Type: text/plain; charset="iso-8859-1" Content-Transfer-Encoding: 8bit X-Priority: 3 X-MSMail-Priority: Normal X-Mailer: Microsoft Outlook Express 5.00.2919.6600 X-MIMEOLE: Produced By Microsoft MimeOLE V5.00.2919.6600 Sender: owner-obm-l@sucuri.mat.puc-rio.br Precedence: bulk Reply-To: obm-l@mat.puc-rio.br Pegando carona no desconhecido, o que é fração imprópria? Sds: Thomas. ----- Original Message ----- From: "Nicolau C. Saldanha" To: Sent: Wednesday, June 19, 2002 4:51 PM Subject: [obm-l] Re: [obm-l] Ajuda rápida... > On Wed, Jun 19, 2002 at 03:50:42PM -0300, Igor Castro wrote: > > Bem companheiros, não estou conseguindo resolver esse problema que peguei em > > um livro de 1 gráu... se puderem dar uma luz... : ) Para quantos valores de n > > entre 1 e 1990 a fração imprópria (n^2 +7)/(n+4) não é irredutível? > > Abraços... Igor.. > > mdc(n^2 + 7, n + 4) = mdc(n^2 + 7 - n*(n + 4), n + 4) > = mdc(-4n + 7, n + 4) > = mdc(-4n + 7 + 4*(n + 4), n + 4) > = mdc(23, n + 4) > > Assim o mdc é 23 se n+4 for múltiplo de 23 e 1 caso contrário. > Ou seja, na faixa pedida, a fração é *não* irredutível para > n = 19, 19+23, 19+2*23, ..., 19+85*23 = 1974. > Assim a resposta é 86. > > []s, N. > > ========================================================================= > Instruções para entrar na lista, sair da lista e usar a lista em > http://www.mat.puc-rio.br/~nicolau/olimp/obm-l.html > O administrador desta lista é > ========================================================================= ========================================================================= Instruções para entrar na lista, sair da lista e usar a lista em http://www.mat.puc-rio.br/~nicolau/olimp/obm-l.html O administrador desta lista é ========================================================================= From owner-obm-l@sucuri.mat.puc-rio.br Fri Jun 21 01:40:52 2002 Return-Path: Received: (from majordom@localhost) by sucuri.mat.puc-rio.br (8.9.3/8.9.3) id BAA00471 for obm-l-list; Fri, 21 Jun 2002 01:40:40 -0300 Received: from sr1.terra.com.br (sr1.terra.com.br [200.176.3.16]) by sucuri.mat.puc-rio.br (8.9.3/8.9.3) with ESMTP id BAA00467 for ; Fri, 21 Jun 2002 01:40:38 -0300 Received: from pavuna.terra.com.br (pavuna.terra.com.br [200.176.3.41]) by sr1.terra.com.br (Postfix) with ESMTP id 4A1146F63E for ; Fri, 21 Jun 2002 01:40:32 -0300 (EST) Received: from stabel (dl-nas3-poa-C89A060E.p001.terra.com.br [200.154.6.14]) (authenticated user dudasta) by pavuna.terra.com.br (Postfix) with ESMTP id BED8D6809E for ; Fri, 21 Jun 2002 01:40:30 -0300 (EST) Message-ID: <008d01c218dd$c7bd34a0$0e069ac8@stabel> From: "Eduardo Casagrande Stabel" To: References: <002a01c217c7$35a790e0$2f87dec8@igor> <20020619165106.B3071@sucuri.mat.puc-rio.br> <02cb01c218d6$af12ad60$6600a8c0@altus.com.br> Subject: [obm-l] =?iso-8859-1?Q?Re:_=5Bobm-l=5D_Re:_=5Bobm-l=5D_Re:_=5Bobm-l=5D_Ajud?= =?iso-8859-1?Q?a_r=E1pida...?= Date: Fri, 21 Jun 2002 01:40:31 -0300 MIME-Version: 1.0 Content-Type: text/plain; charset="iso-8859-1" Content-Transfer-Encoding: 8bit X-Priority: 3 X-MSMail-Priority: Normal X-Mailer: Microsoft Outlook Express 6.00.2600.0000 X-MimeOLE: Produced By Microsoft MimeOLE V6.00.2600.0000 Sender: owner-obm-l@sucuri.mat.puc-rio.br Precedence: bulk Reply-To: obm-l@mat.puc-rio.br Thomas, eu também não sabia o que era e procurei no AltaVista. Num site destinado a alunos de 5a. série dizia: "Fração Própria: É aquela cujo numerador é menor que o denominador (é sempre menor que 1)". Onde tratava, claramente, de frações positivas. O termo não tem muita importância na resolução daquele exercício, mesmo assim estendendo um pouco mais a definição poderia dizer que r (racional) é fração própria se tivermos -1 < r < 1. Esse termo não é muito usado, pelo menos eu não costumo vê-lo por aí, fora dos livros de 5a. série... Deve ser um daqueles termos pedagógicos usados nas primeiras séries para não fazer confusão na cabeça das crianças. Acho que não fui muito valioso nessa explicação. Um abraço! Eduardo Casagrande Stabel. Porto Alegre, RS. From: "Thomas de Rossi" > Pegando carona no desconhecido, > > o que é fração imprópria? > > Sds: Thomas. > > ----- Original Message ----- > From: "Nicolau C. Saldanha" > To: > Sent: Wednesday, June 19, 2002 4:51 PM > Subject: [obm-l] Re: [obm-l] Ajuda rápida... > > > > On Wed, Jun 19, 2002 at 03:50:42PM -0300, Igor Castro wrote: > > > Bem companheiros, não estou conseguindo resolver esse problema que > peguei em > > > um livro de 1 gráu... se puderem dar uma luz... : ) Para quantos valores > de n > > > entre 1 e 1990 a fração imprópria (n^2 +7)/(n+4) não é irredutível? > > > Abraços... Igor.. > > > > mdc(n^2 + 7, n + 4) = mdc(n^2 + 7 - n*(n + 4), n + 4) > > = mdc(-4n + 7, n + 4) > > = mdc(-4n + 7 + 4*(n + 4), n + 4) > > = mdc(23, n + 4) > > > > Assim o mdc é 23 se n+4 for múltiplo de 23 e 1 caso contrário. > > Ou seja, na faixa pedida, a fração é *não* irredutível para > > n = 19, 19+23, 19+2*23, ..., 19+85*23 = 1974. > > Assim a resposta é 86. > > > > []s, N. > > > > ========================================================================= > > Instruções para entrar na lista, sair da lista e usar a lista em > > http://www.mat.puc-rio.br/~nicolau/olimp/obm-l.html > > O administrador desta lista é > > ========================================================================= > > > ========================================================================= > Instruções para entrar na lista, sair da lista e usar a lista em > http://www.mat.puc-rio.br/~nicolau/olimp/obm-l.html > O administrador desta lista é > ========================================================================= > > ========================================================================= Instruções para entrar na lista, sair da lista e usar a lista em http://www.mat.puc-rio.br/~nicolau/olimp/obm-l.html O administrador desta lista é ========================================================================= From owner-obm-l@sucuri.mat.puc-rio.br Fri Jun 21 02:07:31 2002 Return-Path: Received: (from majordom@localhost) by sucuri.mat.puc-rio.br (8.9.3/8.9.3) id CAA00971 for obm-l-list; Fri, 21 Jun 2002 02:07:14 -0300 Received: from sr1.terra.com.br (sr1.terra.com.br [200.176.3.16]) by sucuri.mat.puc-rio.br (8.9.3/8.9.3) with ESMTP id CAA00967 for ; Fri, 21 Jun 2002 02:07:12 -0300 Received: from pavuna.terra.com.br (pavuna.terra.com.br [200.176.3.41]) by sr1.terra.com.br (Postfix) with ESMTP id 309BC6E83B for ; Fri, 21 Jun 2002 02:07:06 -0300 (EST) Received: from stabel (dl-nas3-poa-C89A060E.p001.terra.com.br [200.154.6.14]) (authenticated user dudasta) by pavuna.terra.com.br (Postfix) with ESMTP id 7BCC1680AD for ; Fri, 21 Jun 2002 02:07:04 -0300 (EST) Message-ID: <009601c218e1$7d9043f0$0e069ac8@stabel> From: "Eduardo Casagrande Stabel" To: References: <000701c217f1$893982c0$4390dec8@igor> <3D11310B.30405@niski.com> <02ca01c218d6$ae201000$6600a8c0@altus.com.br> Subject: [obm-l] =?iso-8859-1?Q?Re:_=5Bobm-l=5D_Re:_=5Bobm-l=5D_Re:_=5Bobm-l=5D_Fatora?= =?iso-8859-1?Q?=E7=E3o?= Date: Fri, 21 Jun 2002 02:07:04 -0300 MIME-Version: 1.0 Content-Type: text/plain; charset="iso-8859-1" Content-Transfer-Encoding: 8bit X-Priority: 3 X-MSMail-Priority: Normal X-Mailer: Microsoft Outlook Express 6.00.2600.0000 X-MimeOLE: Produced By Microsoft MimeOLE V6.00.2600.0000 Sender: owner-obm-l@sucuri.mat.puc-rio.br Precedence: bulk Reply-To: obm-l@mat.puc-rio.br Oi Thomas, acontece o seguinte. Alguém pede uma fatoração da expressão 1 + x + x^2 nos reais, o que quer dizer que querem expressar essa mesma expressão como um produto ou quociente de expressões (possivelmente mais simples) de forma que para todo x real (esse é o domínio explícito: domínio no sentido de considerarmos f(x) = 1 + x + x^2 uma função de domínio real f:R->R) ela se iguala à expressão produto ou quociente. Por exemplo. Temos que (1 + x + x^2)*(x - 1) = (x + x^2 + x^3) - (1 + x + x^2) = (x^3 - 1) ou seja (1 + x + x^2)*(x - 1) = (x^3 - 1) e essa expressão vale para TODO o x real, aí ficamos tentados a escrever (1 + x + x^2) = (x^3 - 1)/(x - 1) o que ainda é verdade, mas só no caso de (x - 1) ser diferente de 0, portanto o domínio onde vale a expressão (x^3 - 1)/(x - 1) é os reais menos o zero, e por isso essa não é uma fatoração válida para todos os reais. Por que surge a restrição na hora do quociente? Se temos a*b = c podemos dizer que vale a = c/b só se b for diferente de zero. Se quiséssemos fatorar a expressão (x^3 - 1)/(x - 1) nos reais sem o zero, aí sim poderíamos dizer (x^3 - 1)/(x - 1) = (1 + x + x^2) pois nesse domínio as duas expressões sempre são iguais. Um outro exemplo é o seguinte. A expressão (x^2 - y^2) pode ser fatorada da seguinte forma (x^2 - y^2) = (x + y)*(x - y) E a expressão da direita vale para todos os reais, por isso é uma fatoração válida. Um outro exemplo ainda é o seguinte. Fatorar, nos reais, a expressão (1 - x^2 + x^4) multiplique-a por (x^2 + 1) (1 - x^2 + x^4)*(x^2 + 1) = (x^2 - x^4 + x^6) + (1 - x^2 + x^4) = (x^6 + 1) o que nos deixa tentados a escrever (1 - x^2 + x^4) = (x^6 + 1)/(x^2 + 1) o que é verdade para todos os reais, então temos uma fatoração válida nos reais. Já nos complexos ela não é válida, pois (x^6 + 1)/(x^2 + 1) não está definida para x=i. Quando queremos fatorar a expressão (x^6 + x^3y^3 + y^6) no reais, estamos interessados em encontrar uma expressão produto ou quociente que valha para todos os pontos do plano (x, y). No caso a expressão (x^9 - y^9)/(x^3 - y^3) = (x^6 + x^3y^3 + y^6) só vale quando (x^3 - y^3) não é zero, ou seja quando x é diferente de y. Portanto essa fatoração funciona funciona em todos os pontos (x, y) do plano, excetuando-se a reta afim, onde vale x=y Espero não ter sido muito simplório e que tenha explicado o que interessa. Um abraço a todos! Eduardo Casagrande Stabel. Porto Alegre, RS. From: "Thomas de Rossi" > Oi pessoal, > > Olhei para a fatoração e não entendi a explicação: > > Nao. Pq o dominio é Reais. Com a sua fatoracao (onde tem uma divisao) > > x^3 nao pode ser igual a y^3 o que restringe o dominio. > > Poderiam ser mais didáticos na explicação, > > Sds: Thomas. > > ----- Original Message ----- > From: "niski" > To: > Sent: Wednesday, June 19, 2002 10:34 PM > Subject: Re: [obm-l] Re: [obm-l] Fatoração > > > > > > > > Igor Castro wrote: > > > > >(x^6 + x^3.y^3 + y^6)(x^3 - y^3) = x^9 - y^9 > > > > > >x^6 + x^3.y^3 + y^6= (x^9 - y^9)/(x^3 - y^3) > > >seria isso? > > > > > > > Nao. Pq o dominio é Reais. Com a sua fatoracao (onde tem uma divisao) > > x^3 nao pode ser igual a y^3 o que restringe o dominio. > > > > > > > > > ========================================================================= > > Instruções para entrar na lista, sair da lista e usar a lista em > > http://www.mat.puc-rio.br/~nicolau/olimp/obm-l.html > > O administrador desta lista é > > ========================================================================= > > ========================================================================= > Instruções para entrar na lista, sair da lista e usar a lista em > http://www.mat.puc-rio.br/~nicolau/olimp/obm-l.html > O administrador desta lista é > ========================================================================= > > ========================================================================= Instruções para entrar na lista, sair da lista e usar a lista em http://www.mat.puc-rio.br/~nicolau/olimp/obm-l.html O administrador desta lista é ========================================================================= From owner-obm-l@sucuri.mat.puc-rio.br Fri Jun 21 03:21:45 2002 Return-Path: Received: (from majordom@localhost) by sucuri.mat.puc-rio.br (8.9.3/8.9.3) id DAA01911 for obm-l-list; Fri, 21 Jun 2002 03:21:28 -0300 Received: from www.zipmail.com.br (smtp.zipmail.com.br [200.187.242.10]) by sucuri.mat.puc-rio.br (8.9.3/8.9.3) with ESMTP id DAA01907 for ; Fri, 21 Jun 2002 03:21:26 -0300 From: luizhenriquerick@zipmail.com.br Received: from [200.165.185.232] by www.zipmail.com.br with HTTP; Fri, 21 Jun 2002 03:21:17 -0300 Message-ID: <3D1265790000048D@www.zipmail.com.br> Date: Fri, 21 Jun 2002 03:21:17 -0300 In-Reply-To: <7722846962.20020621004508@gmx.net> Subject: [obm-l] =?iso-8859-1?Q?Re=3A=20=5Bobm=2Dl=5D=20Re=3A=20=5Bobm=2Dl=5D=20Quadril=E1teros?= To: obm-l@mat.puc-rio.br MIME-Version: 1.0 Content-Type: text/plain; charset="iso-8859-1" Content-Transfer-Encoding: 8bit X-MIME-Autoconverted: from quoted-printable to 8bit by sucuri.mat.puc-rio.br id DAA01908 Sender: owner-obm-l@sucuri.mat.puc-rio.br Precedence: bulk Reply-To: obm-l@mat.puc-rio.br -- Mensagem original -- Considerando um quadrilátero inscrito ABCD , de diagonas , AC=p BD=q , e os lados AB, BC , CD e DA respectivamente a , b , c e d , temos: S(ABC) + S(ADC) = S(ABD) + S(BCD) Então podemos escrever : abp/4R + dcp/4R = adq/4R + bcq/4R (abp + dcp)/4R = (adq + bcq)/4R p(ab + dc) = q(ad + bc) p/q = (ab + dc)/(ad + bc) > >Em 20/6/2002, 15:36, Igor (cnaval@ieg.com.br) disse: > > >> Na primeira ache as duas diagonais(usando hiparco e depois ptolomeu), em >> seguida aplique a relação da mediana de euller... > > Alguém pode enunciar o teorema de Hiparco pra mim? O de Ptolomeu conheço >(Eureka5), mas nunca li sobre esse outro... > >Fui! > > >####### Igor GomeZZ ######## > UIN: 29249895 > Vitória, Espírito Santo, Brasil > Criação: 21/6/2002 (00:41) >#################################### >Pare para pensar: > >A arte da medicina consiste em >distrair enquanto a Natureza cuida >da doença. (Voltaire) > >#################################### > >========================================================================= >Instruções para entrar na lista, sair da lista e usar a lista em >http://www.mat.puc-rio.br/~nicolau/olimp/obm-l.html >O administrador desta lista é >========================================================================= > ---------------------------------------- |-=Rick-C.R.B.=- | |ICQ 124805654 | |e-mail luizhenriquerick@zipmail.com.br | ---------------------------------------- ------------------------------------------ Use o melhor sistema de busca da Internet Radar UOL - http://www.radaruol.com.br ========================================================================= Instruções para entrar na lista, sair da lista e usar a lista em http://www.mat.puc-rio.br/~nicolau/olimp/obm-l.html O administrador desta lista é ========================================================================= From owner-obm-l@sucuri.mat.puc-rio.br Fri Jun 21 11:26:06 2002 Return-Path: Received: (from majordom@localhost) by sucuri.mat.puc-rio.br (8.9.3/8.9.3) id LAA06625 for obm-l-list; Fri, 21 Jun 2002 11:25:48 -0300 Received: from gorgo.centroin.com.br (gorgo.centroin.com.br [200.225.63.128]) by sucuri.mat.puc-rio.br (8.9.3/8.9.3) with ESMTP id LAA06621 for ; Fri, 21 Jun 2002 11:25:46 -0300 Received: from centroin.com.br (du85c.rjo.centroin.com.br [200.225.58.85]) (authenticated bits=0) by gorgo.centroin.com.br (8.12.2/8.12.1) with ESMTP id g5LEQC6K023191 for ; Fri, 21 Jun 2002 11:26:12 -0300 (BRT) Message-ID: <3D1337D6.7090905@centroin.com.br> Date: Fri, 21 Jun 2002 11:27:34 -0300 From: Augusto =?ISO-8859-1?Q?C=E9sar?= Morgado User-Agent: Mozilla/5.0 (Windows; U; Win98; en-US; rv:0.9.4.1) Gecko/20020508 Netscape6/6.2.3 X-Accept-Language: en-us MIME-Version: 1.0 To: obm-l@mat.puc-rio.br Subject: Re: [obm-l] Re: [obm-l] Re: [obm-l] Ajuda =?ISO-8859-1?Q?r=E1pida=2E=2E=2E?= References: <002a01c217c7$35a790e0$2f87dec8@igor> <20020619165106.B3071@sucuri.mat.puc-rio.br> <02cb01c218d6$af12ad60$6600a8c0@altus.com.br> Content-Type: multipart/alternative; boundary="------------040609010107080105040802" Sender: owner-obm-l@sucuri.mat.puc-rio.br Precedence: bulk Reply-To: obm-l@mat.puc-rio.br --------------040609010107080105040802 Content-Type: text/plain; charset=ISO-8859-1; format=flowed Content-Transfer-Encoding: 8bit Aquela cujo numerador eh maior que o denominador. Thomas de Rossi wrote: >Pegando carona no desconhecido, > >o que é fração imprópria? > >Sds: Thomas. > >----- Original Message ----- >From: "Nicolau C. Saldanha" >To: >Sent: Wednesday, June 19, 2002 4:51 PM >Subject: [obm-l] Re: [obm-l] Ajuda rápida... > > >>On Wed, Jun 19, 2002 at 03:50:42PM -0300, Igor Castro wrote: >> >>>Bem companheiros, não estou conseguindo resolver esse problema que >>> >peguei em > >>>um livro de 1 gráu... se puderem dar uma luz... : ) Para quantos valores >>> >de n > >>>entre 1 e 1990 a fração imprópria (n^2 +7)/(n+4) não é irredutível? >>>Abraços... Igor.. >>> >>mdc(n^2 + 7, n + 4) = mdc(n^2 + 7 - n*(n + 4), n + 4) >> = mdc(-4n + 7, n + 4) >> = mdc(-4n + 7 + 4*(n + 4), n + 4) >> = mdc(23, n + 4) >> >>Assim o mdc é 23 se n+4 for múltiplo de 23 e 1 caso contrário. >>Ou seja, na faixa pedida, a fração é *não* irredutível para >>n = 19, 19+23, 19+2*23, ..., 19+85*23 = 1974. >>Assim a resposta é 86. >> >>[]s, N. >> >>========================================================================= >>Instruções para entrar na lista, sair da lista e usar a lista em >>http://www.mat.puc-rio.br/~nicolau/olimp/obm-l.html >>O administrador desta lista é >>========================================================================= >> > > >========================================================================= >Instruções para entrar na lista, sair da lista e usar a lista em >http://www.mat.puc-rio.br/~nicolau/olimp/obm-l.html >O administrador desta lista é >========================================================================= > > --------------040609010107080105040802 Content-Type: text/html; charset=us-ascii Content-Transfer-Encoding: 7bit Aquela cujo numerador eh maior que o denominador.

Thomas de Rossi wrote:
Pegando carona no desconhecido,

o que é fração imprópria?

Sds: Thomas.

----- Original Message -----
From: "Nicolau C. Saldanha" <nicolau@sucuri.mat.puc-rio.br>
To: <obm-l@mat.puc-rio.br>
Sent: Wednesday, June 19, 2002 4:51 PM
Subject: [obm-l] Re: [obm-l] Ajuda rápida...


On Wed, Jun 19, 2002 at 03:50:42PM -0300, Igor Castro wrote:
Bem companheiros, não estou conseguindo resolver esse problema que
peguei em
um livro de 1 gráu... se puderem dar uma luz... : ) Para quantos valores
de n
entre 1 e 1990 a fração imprópria (n^2 +7)/(n+4) não é irredutível?
Abraços... Igor..
mdc(n^2 + 7, n + 4) = mdc(n^2 + 7 - n*(n + 4), n + 4)
= mdc(-4n + 7, n + 4)
= mdc(-4n + 7 + 4*(n + 4), n + 4)
= mdc(23, n + 4)

Assim o mdc é 23 se n+4 for múltiplo de 23 e 1 caso contrário.
Ou seja, na faixa pedida, a fração é *não* irredutível para
n = 19, 19+23, 19+2*23, ..., 19+85*23 = 1974.
Assim a resposta é 86.

[]s, N.

=========================================================================
Instruções para entrar na lista, sair da lista e usar a lista em
http://www.mat.puc-rio.br/~nicolau/olimp/obm-l.html
O administrador desta lista é <nicolau@mat.puc-rio.br>
================== =======================================================


=========================================================================
Instruções para entrar na lista, sair da lista e usar a lista em
http://www.mat.puc-rio.br/~nicolau/olimp/obm-l.html
O administrador desta lista é <nicolau@mat.puc-rio.br>
=========================================================================



--------------040609010107080105040802-- ========================================================================= Instruções para entrar na lista, sair da lista e usar a lista em http://www.mat.puc-rio.br/~nicolau/olimp/obm-l.html O administrador desta lista é ========================================================================= From owner-obm-l@sucuri.mat.puc-rio.br Fri Jun 21 13:28:43 2002 Return-Path: Received: (from majordom@localhost) by sucuri.mat.puc-rio.br (8.9.3/8.9.3) id NAA09054 for obm-l-list; Fri, 21 Jun 2002 13:27:56 -0300 Received: from www.zipmail.com.br (smtp.zipmail.com.br [200.187.242.10]) by sucuri.mat.puc-rio.br (8.9.3/8.9.3) with ESMTP id NAA09050 for ; Fri, 21 Jun 2002 13:27:54 -0300 From: peterdirichlet@zipmail.com.br Received: from [200.206.103.3] by www.zipmail.com.br with HTTP; Fri, 21 Jun 2002 13:27:11 -0300 Message-ID: <3D1264C50000181C@www.zipmail.com.br> Date: Fri, 21 Jun 2002 13:27:11 -0300 In-Reply-To: <3D128E17.3060804@centroin.com.br> Subject: [obm-l] =?iso-8859-1?Q?Re=3A=20=5Bobm=2Dl=5D=20problema=20do=20ortocentro?= To: obm-l@mat.puc-rio.br MIME-Version: 1.0 Content-Type: text/plain; charset="iso-8859-1" Content-Transfer-Encoding: 8bit X-MIME-Autoconverted: from quoted-printable to 8bit by sucuri.mat.puc-rio.br id NAA09051 Sender: owner-obm-l@sucuri.mat.puc-rio.br Precedence: bulk Reply-To: obm-l@mat.puc-rio.br Seja M o ponto medio de AB,e O circuncentro e H ortocentro do triangulo.Sabemos que 2*OM=HC,logo OM=5/2.E OM^2+MB^2=OB^2.Logo OA=6,5.Acertei? -- Mensagem original -- >Um problema que me pareceu interessante. > >Determinar o raio do >círculo circunscrito a um triângulo dados: a base AB=12 e a >distância de seu ortocentro ao vértice C = 5. > > >========================================================================= >Instruções para entrar na lista, sair da lista e usar a lista em >http://www.mat.puc-rio.br/~nicolau/olimp/obm-l.html >O administrador desta lista é >========================================================================= > TRANSIRE SVVM PECTVS MVNDOQUE POTIRE CONGREGATI EX TOTO ORBE MATHEMATICI OB SCRIPTA INSIGNIA TRIBVERE Medalha Fields(John Charles Fields) ------------------------------------------ Use o melhor sistema de busca da Internet Radar UOL - http://www.radaruol.com.br ========================================================================= Instruções para entrar na lista, sair da lista e usar a lista em http://www.mat.puc-rio.br/~nicolau/olimp/obm-l.html O administrador desta lista é ========================================================================= From owner-obm-l@sucuri.mat.puc-rio.br Fri Jun 21 13:41:32 2002 Return-Path: Received: (from majordom@localhost) by sucuri.mat.puc-rio.br (8.9.3/8.9.3) id NAA09263 for obm-l-list; Fri, 21 Jun 2002 13:41:14 -0300 Received: from pina.terra.com.br (pina.terra.com.br [200.176.3.17]) by sucuri.mat.puc-rio.br (8.9.3/8.9.3) with ESMTP id NAA09259 for ; Fri, 21 Jun 2002 13:41:10 -0300 Received: from engenho.terra.com.br (engenho.terra.com.br [200.176.3.42]) by pina.terra.com.br (Postfix) with ESMTP id 8D48553080 for ; Fri, 21 Jun 2002 13:40:23 -0300 (EST) Received: from niski.com (dl-adsl-C8D44CF7.sao.terra.com.br [200.212.76.247]) (authenticated user fniski) by engenho.terra.com.br (Postfix) with ESMTP id 4FE0B68096 for ; Fri, 21 Jun 2002 13:40:22 -0300 (EST) Message-ID: <3D1356EC.8090803@niski.com> Date: Fri, 21 Jun 2002 13:40:12 -0300 From: niski User-Agent: Mozilla/5.0 (Windows; U; Windows NT 5.1; en-US; rv:0.9.4.1) Gecko/20020508 Netscape6/6.2.3 X-Accept-Language: en-us MIME-Version: 1.0 To: obm-l@mat.puc-rio.br Subject: Re: [obm-l] Re: [obm-l] Re: [obm-l] Re: [obm-l] =?ISO-8859-1?Q?Fatora=E7=E3o?= References: <000701c217f1$893982c0$4390dec8@igor> <3D11310B.30405@niski.com> <02ca01c218d6$ae201000$6600a8c0@altus.com.br> <009601c218e1$7d9043f0$0e069ac8@stabel> Content-Type: text/plain; charset=ISO-8859-1; format=flowed Content-Transfer-Encoding: 8bit Sender: owner-obm-l@sucuri.mat.puc-rio.br Precedence: bulk Reply-To: obm-l@mat.puc-rio.br > > > >Eduardo Casagrande Stabel. Porto Alegre, RS. > Esta errado Eduardo. É pedido para fatorar em R voce restringiu o dominio, logo não obedebeu as condicoes do enunciado. Veja o resultado da fatoracao na minha msg. > ========================================================================= Instruções para entrar na lista, sair da lista e usar a lista em http://www.mat.puc-rio.br/~nicolau/olimp/obm-l.html O administrador desta lista é ========================================================================= From owner-obm-l@sucuri.mat.puc-rio.br Fri Jun 21 14:15:54 2002 Return-Path: Received: (from majordom@localhost) by sucuri.mat.puc-rio.br (8.9.3/8.9.3) id OAA10224 for obm-l-list; Fri, 21 Jun 2002 14:15:43 -0300 Received: (from nicolau@localhost) by sucuri.mat.puc-rio.br (8.9.3/8.9.3) id OAA10219 for obm-l@mat.puc-rio.br; Fri, 21 Jun 2002 14:15:43 -0300 Date: Fri, 21 Jun 2002 14:15:43 -0300 From: "Nicolau C. Saldanha" To: obm-l@mat.puc-rio.br Subject: Re: [obm-l] Axiomas de Peano Message-ID: <20020621141543.A9714@sucuri.mat.puc-rio.br> References: Mime-Version: 1.0 Content-Type: text/plain; charset=iso-8859-1 Content-Disposition: inline Content-Transfer-Encoding: 8bit User-Agent: Mutt/1.2.5i In-Reply-To: ; from rogeriofajardo@hotmail.com on Fri, Jun 21, 2002 at 02:32:34AM +0000 Sender: owner-obm-l@sucuri.mat.puc-rio.br Precedence: bulk Reply-To: obm-l@mat.puc-rio.br On Fri, Jun 21, 2002 at 02:32:34AM +0000, Rogerio Fajardo wrote: > Aproveitando que o assunto é axiomas de peano, é possível definir aritmética > usando só os axiomas de peano (usando lógica de 1ªordem e os símbolos 0 e > sucessor de)? Dá pra fazer uma espécie de teorema da recursão nos axiomas de > Peano? Ou será necessário usar lógica de segunda ordem, ou esquema de prova? Boa pergunta. O seguinte é um modelo para os axiomas de Peano apenas com o conceito de sucessor e apenas com lógica de primeira ordem: {0, 1, 2, ...,..., w - 2, w - 1, w, w + 1, w + 2, ...} Mais, valem neste modelo exatamente os mesmos enunciados de lógica de primeira ordem (ressaltando, se a linguagem tiver apenas o conceito de sucessor). A coisa muda totalmente de figura se + e * fizerem parte da linguagem, junto com os axiomas: para todo n, n + 0 = n para todo n e m, n + s(m) = s(n + m) para todo n, n * 0 = 0 para todo n e m, n * s(m) = (n * m) + n Aí fica muito mais difícil (mas, por Gödel e Lowenheim-Skolem, ainda possível) obter modelos estranhos. Para quem não está entendendo, o último axioma de Peano é freqüentemente fraseado assim: Se X é um subconjunto de N com 0 pertencente a X e com s(X) contido em X então X = N. Este axioma pressupõe o conceito de conjunto, ou seja, pressupõe uma teoria de conjuntos. Esta é uma pressuposição bem forte: dada a teoria dos conjuntos podemos *construir* os naturais, então para que os axiomas? Uma solução é usar lógica de 2a ordem: a própria lógica e linguagem incorporam uma versão fraca do conceito de conjunto. A lógica de 2a ordem tem suas dificuldades. A versão mais autocontida (usando lógica de 1a ordem sem teoria dos conjuntos) seria trocar conjuntos por fórmulas. Assim, o último axioma vira uma família de axiomas (um para cada fórmula). Seja p(n) é uma fórmula com uma variável livre n; temos o axioma: Se p(0) e (para todo n, ( p(n) => p(n+1) ) ) então (para todo n, p(n)) []s, N. ========================================================================= Instruções para entrar na lista, sair da lista e usar a lista em http://www.mat.puc-rio.br/~nicolau/olimp/obm-l.html O administrador desta lista é ========================================================================= From owner-obm-l@sucuri.mat.puc-rio.br Fri Jun 21 15:10:29 2002 Return-Path: Received: (from majordom@localhost) by sucuri.mat.puc-rio.br (8.9.3/8.9.3) id PAA11381 for obm-l-list; Fri, 21 Jun 2002 15:08:43 -0300 Received: from fgvrj23.fgv.br (fgvrj23.fgv.br [200.20.164.23]) by sucuri.mat.puc-rio.br (8.9.3/8.9.3) with ESMTP id PAA11377 for ; Fri, 21 Jun 2002 15:08:41 -0300 Received: by FGVRJ23 with Internet Mail Service (5.5.2653.19) id ; Fri, 21 Jun 2002 15:09:09 -0300 Message-ID: <3BE65222F383D611BE1E00D0B7B60A55E0258B@FGVRJ23> From: Ralph Teixeira To: "'obm-l@mat.puc-rio.br'" Subject: [obm-l] =?iso-8859-1?Q?RES=3A_=5Bobm-l=5D_t=2E_dos_n=BAs?= Date: Fri, 21 Jun 2002 15:09:08 -0300 MIME-Version: 1.0 X-Mailer: Internet Mail Service (5.5.2653.19) Content-Type: text/plain; charset="iso-8859-1" Content-Transfer-Encoding: 8bit X-MIME-Autoconverted: from quoted-printable to 8bit by sucuri.mat.puc-rio.br id PAA11378 Sender: owner-obm-l@sucuri.mat.puc-rio.br Precedence: bulk Reply-To: obm-l@mat.puc-rio.br >> mostre q todo quadrado perfeito pode ser representado como soma dos >> quadrados de racionais ,naum inteiros, r e s. Seja n^2 o quadrado perfeito em questão. Como (n^2+1)^2 = (n^2-1)^2 +(2n)^2, temos a^2+b^2=1 onde a=(n^2-1)/(n^2+1) e b=2n/(n^2+1). Note que os denominadores de a e b são primos com n, e ambas as frações são irredutíveis (numerador e denominador primos entre si em ambos os casos). Assim: (an)^2+(bn)^2=n^2 onde r=an e s=bn são racionais não naturais. Abraço, Ralph ========================================================================= Instruções para entrar na lista, sair da lista e usar a lista em http://www.mat.puc-rio.br/~nicolau/olimp/obm-l.html O administrador desta lista é ========================================================================= From owner-obm-l@sucuri.mat.puc-rio.br Fri Jun 21 22:41:37 2002 Return-Path: Received: (from majordom@localhost) by sucuri.mat.puc-rio.br (8.9.3/8.9.3) id WAA17113 for obm-l-list; Fri, 21 Jun 2002 22:41:02 -0300 Received: from sr1.terra.com.br (sr1.terra.com.br [200.176.3.16]) by sucuri.mat.puc-rio.br (8.9.3/8.9.3) with ESMTP id WAA17109 for ; Fri, 21 Jun 2002 22:41:00 -0300 Received: from smtp4-poa.terra.com.br (smtp4-poa.terra.com.br [200.176.3.35]) by sr1.terra.com.br (Postfix) with ESMTP id 0EBC66F53A for ; Fri, 21 Jun 2002 22:40:32 -0300 (EST) Received: from stabel (dl-nas3-poa-C89A0601.p001.terra.com.br [200.154.6.1]) (authenticated user dudasta) by smtp4-poa.terra.com.br (Postfix) with ESMTP id 93FE7AC596 for ; Fri, 21 Jun 2002 22:40:30 -0300 (EST) Message-ID: <003701c2198d$cd2cbea0$01069ac8@stabel> From: "Eduardo Casagrande Stabel" To: References: <000701c217f1$893982c0$4390dec8@igor> <3D11310B.30405@niski.com> <02ca01c218d6$ae201000$6600a8c0@altus.com.br> <009601c218e1$7d9043f0$0e069ac8@stabel> <3D1356EC.8090803@niski.com> Subject: [obm-l] =?iso-8859-1?Q?Re:_=5Bobm-l=5D_Re:_=5Bobm-l=5D_Re:_=5Bobm-l=5D_Re:_=5Bo?= =?iso-8859-1?Q?bm-l=5D_Fatora=E7=E3o?= Date: Fri, 21 Jun 2002 22:39:24 -0300 MIME-Version: 1.0 Content-Type: text/plain; charset="iso-8859-1" Content-Transfer-Encoding: 8bit X-Priority: 3 X-MSMail-Priority: Normal X-Mailer: Microsoft Outlook Express 6.00.2600.0000 X-MimeOLE: Produced By Microsoft MimeOLE V6.00.2600.0000 Sender: owner-obm-l@sucuri.mat.puc-rio.br Precedence: bulk Reply-To: obm-l@mat.puc-rio.br From: "niski" > > > >Eduardo Casagrande Stabel. Porto Alegre, RS. > > > Esta errado Eduardo. É pedido para fatorar em R voce restringiu o > dominio, logo não obedebeu as condicoes do enunciado. > Veja o resultado da fatoracao na minha msg. > Niski, eu disse a seguinte frase: "Quando queremos fatorar a expressão (x^6 + x^3y^3 + y^6) nos reais, estamos interessados em encontrar uma expressão produto ou quociente que valha para todos os pontos do plano (x, y). No caso a expressão (x^9 - y^9)/(x^3 - y^3) = (x^6 + x^3y^3 + y^6) SÓ VALE QUANDO (x^3 - y^3) não é zero, ou seja quando x é diferente de y. Portanto essa fatoração funciona em todos os pontos (x, y) do plano, excetuando-se a reta afim, onde vale x=y" Repare na conclusão: "Portanto essa fatoração funciona em todos os pontos (x, y) do plano, EXCETUANDO-SE a reta afim, onde vale x=y." Se você ler todo o meu e-mail, vai ver que isso quer dizer que essa NÃO É UMA FATORAÇÃO VÁLIDA NOS REAIS, como você diz. Eu estava tentando explicar o que era esse domínio de fatoração, era essa a minha intenção. Um abraço! Eduardo Casagrande Stabel. Porto Alegre, RS. ========================================================================= Instruções para entrar na lista, sair da lista e usar a lista em http://www.mat.puc-rio.br/~nicolau/olimp/obm-l.html O administrador desta lista é ========================================================================= From owner-obm-l@sucuri.mat.puc-rio.br Sat Jun 22 00:09:20 2002 Return-Path: Received: (from majordom@localhost) by sucuri.mat.puc-rio.br (8.9.3/8.9.3) id AAA18036 for obm-l-list; Sat, 22 Jun 2002 00:09:09 -0300 Received: from mail.gmx.net (mail.gmx.net [213.165.64.20]) by sucuri.mat.puc-rio.br (8.9.3/8.9.3) with SMTP id AAA18032 for ; Sat, 22 Jun 2002 00:09:06 -0300 Received: (qmail 9046 invoked by uid 0); 22 Jun 2002 03:08:35 -0000 Received: from unknown (HELO gomes) (200.216.104.16) by mail.gmx.net (mp008-rz3) with SMTP; 22 Jun 2002 03:08:35 -0000 Date: Fri, 21 Jun 2002 14:38:26 -0300 From: Igor GomeZZ X-Mailer: The Bat! (v1.60c) Organization: -- X-Priority: 3 (Normal) Message-ID: <10613348664.20020621143826@gmx.net> To: "luizhenriquerick@zipmail.com.br" Subject: [obm-l] =?ISO-8859-1?B?UmU6IFtvYm0tbF0gUXVhZHJpbOF0ZXJvcw==?= In-Reply-To: <3D1265790000048D@www.zipmail.com.br> References: <3D1265790000048D@www.zipmail.com.br> MIME-Version: 1.0 Content-Type: text/plain; charset=ISO-8859-1 Content-Transfer-Encoding: 8bit Sender: owner-obm-l@sucuri.mat.puc-rio.br Precedence: bulk Reply-To: obm-l@mat.puc-rio.br Em 21/6/2002, 03:21, luizhenriquerick (luizhenriquerick@zipmail.com.br) disse: > Considerando um quadrilátero inscrito ABCD , de diagonas , AC=p BD=q , e > os lados AB, BC , CD e DA respectivamente a , b , c e d , temos: > S(ABC) + S(ADC) = S(ABD) + S(BCD) > Então podemos escrever : > abp/4R + dcp/4R = adq/4R + bcq/4R > (abp + dcp)/4R = (adq + bcq)/4R > p(ab + dc) = q(ad + bc) > p/q = (ab + dc)/(ad + bc) Valeu brow, bem interessante a relação :) Fui! ####### Igor GomeZZ ######## UIN: 29249895 Vitória, Espírito Santo, Brasil Criação: 21/6/2002 (14:37) #################################### Pare para pensar: Quando teu amigo atravessa alguma aflição, não o aborreças perguntando-lhe o que podes fazer por ele. Pensa em algo apropriado e faze-o. (Ed. Howe) #################################### ========================================================================= Instruções para entrar na lista, sair da lista e usar a lista em http://www.mat.puc-rio.br/~nicolau/olimp/obm-l.html O administrador desta lista é ========================================================================= From owner-obm-l@sucuri.mat.puc-rio.br Sat Jun 22 14:53:40 2002 Return-Path: Received: (from majordom@localhost) by sucuri.mat.puc-rio.br (8.9.3/8.9.3) id OAA23142 for obm-l-list; Sat, 22 Jun 2002 14:52:13 -0300 Received: from shen.bol.com.br (shen.bol.com.br [200.221.24.14]) by sucuri.mat.puc-rio.br (8.9.3/8.9.3) with ESMTP id OAA23138 for ; Sat, 22 Jun 2002 14:52:12 -0300 Received: from bol.com.br (200.221.24.110) by shen.bol.com.br (5.1.071) id 3D132D93000868EA for obm-l@mat.puc-rio.br; Sat, 22 Jun 2002 14:51:00 -0300 Date: Sat, 22 Jun 2002 14:50:59 -0300 Message-Id: Subject: [obm-l] combinatoria MIME-Version: 1.0 Content-Type: text/plain;charset="iso-8859-1" From: "adr.scr.m" To: obm-l@mat.puc-rio.br X-XaM3-API-Version: 2.4.3.4.4 X-SenderIP: 200.151.57.234 Content-Transfer-Encoding: 8bit X-MIME-Autoconverted: from quoted-printable to 8bit by sucuri.mat.puc-rio.br id OAA23139 Sender: owner-obm-l@sucuri.mat.puc-rio.br Precedence: bulk Reply-To: obm-l@mat.puc-rio.br estou com duvida neste problema e preciso de ajuda Depois de ter dado um curso,um professor resolve se derpedir de seus 7 alunos oferecendo,durante 7 dias consecutivos,7 jantares para cada 3 alunos cada.De quantos modos ele pode fazer osconvites se ele nao deseja que um mesmo par de alunos compareca a mais de um jantar ? para mim o nºde convites seria o nº de pares distintos ,mas da errado. __________________________________________________________________________ Quer ter seu próprio endereço na Internet? Garanta já o seu e ainda ganhe cinco e-mails personalizados. DomíniosBOL - http://dominios.bol.com.br ========================================================================= Instruções para entrar na lista, sair da lista e usar a lista em http://www.mat.puc-rio.br/~nicolau/olimp/obm-l.html O administrador desta lista é ========================================================================= From owner-obm-l@sucuri.mat.puc-rio.br Sat Jun 22 17:40:13 2002 Return-Path: Received: (from majordom@localhost) by sucuri.mat.puc-rio.br (8.9.3/8.9.3) id RAA24752 for obm-l-list; Sat, 22 Jun 2002 17:39:57 -0300 Received: from ns-3.idc.dglnet.com.br (ns-3.idc.dglnet.com.br [200.218.161.4]) by sucuri.mat.puc-rio.br (8.9.3/8.9.3) with ESMTP id RAA24748 for ; Sat, 22 Jun 2002 17:39:56 -0300 Received: from user (200-204-37-209.dial-up.telesp.net.br [200.204.37.209]) by ns-3.idc.dglnet.com.br (Postfix) with SMTP id C4E6815001A for ; Sat, 22 Jun 2002 17:39:27 -0300 (BRT) Message-ID: <002201c21a2c$eff095c0$d125ccc8@user> From: "Claudio" To: Subject: [obm-l] Batuta! Date: Sat, 22 Jun 2002 17:39:40 -0300 MIME-Version: 1.0 Content-Type: multipart/alternative; boundary="----=_NextPart_000_001F_01C21A13.C95CD400" X-Priority: 3 X-MSMail-Priority: Normal X-Mailer: Microsoft Outlook Express 5.00.2314.1300 X-MimeOLE: Produced By Microsoft MimeOLE V5.00.2314.1300 Sender: owner-obm-l@sucuri.mat.puc-rio.br Precedence: bulk Reply-To: obm-l@mat.puc-rio.br This is a multi-part message in MIME format. ------=_NextPart_000_001F_01C21A13.C95CD400 Content-Type: text/plain; charset="iso-8859-1" Content-Transfer-Encoding: quoted-printable Eis um probleminha de geometria batuta. =C9 da prova francesa de = sele=E7=E3o para a olimp=EDada mundial. Seja ABC um tri=E2ngulo acut=E2ngulo. Sejam A_1 e B_1 os p=E9s das alturas tiradas de A e B, respectivamente. = Seja M o ponto m=E9dio do lado AB. i) Mostre que a reta MA_1 =E9 tangente ao c=EDrculo circunscrito ao = tri=E2ngulo A_1B_1C. ii)Mostre que os c=EDrculos circunscritos aos tri=E2ngulos A_1B_1C, = BMA_1 e AMB_1 t=EAm um ponto em comum. (Sai todinho somente com considera=E7=F5es sobre =E2ngulos.) Saludos! Casemiro. ------=_NextPart_000_001F_01C21A13.C95CD400 Content-Type: text/html; charset="iso-8859-1" Content-Transfer-Encoding: quoted-printable
Eis um probleminha de geometria batuta. = =C9 da prova=20 francesa de sele=E7=E3o para a olimp=EDada mundial.
Seja ABC um tri=E2ngulo = acut=E2ngulo.
Sejam A_1 e B_1 os p=E9s das alturas = tiradas de A e=20 B, respectivamente. Seja M o ponto m=E9dio do lado AB.
 
i) Mostre que a reta MA_1 =E9 tangente = ao c=EDrculo=20 circunscrito ao tri=E2ngulo A_1B_1C.
ii)Mostre que os c=EDrculos = circunscritos aos=20 tri=E2ngulos A_1B_1C, BMA_1 e AMB_1 t=EAm um ponto em = comum.
 
(Sai todinho somente com = considera=E7=F5es sobre=20 =E2ngulos.)
 
Saludos!
Casemiro.
------=_NextPart_000_001F_01C21A13.C95CD400-- ========================================================================= Instruções para entrar na lista, sair da lista e usar a lista em http://www.mat.puc-rio.br/~nicolau/olimp/obm-l.html O administrador desta lista é ========================================================================= From owner-obm-l@sucuri.mat.puc-rio.br Sat Jun 22 21:26:58 2002 Return-Path: Received: (from majordom@localhost) by sucuri.mat.puc-rio.br (8.9.3/8.9.3) id VAA26537 for obm-l-list; Sat, 22 Jun 2002 21:26:00 -0300 Received: from smtp.ieg.com.br (11.139.226.200.in-addr.arpa.ig.com.br [200.226.139.11] (may be forged)) by sucuri.mat.puc-rio.br (8.9.3/8.9.3) with ESMTP id VAA26533 for ; Sat, 22 Jun 2002 21:25:56 -0300 Received: from macacos (200-161-155-60.dsl.telesp.net.br [200.161.155.60]) by smtp.ieg.com.br (IeG relay/8.9.3) with SMTP id g5N0LtXe002056 for ; Sat, 22 Jun 2002 21:21:56 -0300 (BRT) Message-ID: <001701c21a4c$9690bda0$0200a8c0@macacos> From: =?iso-8859-1?Q?Gabriel_P=E9rgola?= To: "Obm-l" Subject: [obm-l] Trigonometria Date: Sat, 22 Jun 2002 21:26:14 -0300 MIME-Version: 1.0 Content-Type: text/plain; charset="iso-8859-1" Content-Transfer-Encoding: 8bit X-Priority: 3 X-MSMail-Priority: Normal X-Mailer: Microsoft Outlook Express 5.50.4807.1700 X-MimeOLE: Produced By Microsoft MimeOLE V5.50.4807.1700 Sender: owner-obm-l@sucuri.mat.puc-rio.br Precedence: bulk Reply-To: obm-l@mat.puc-rio.br Se a +b = 60º, entao: sen a + sen b / cos a + cos b = ? É possivel resolver essa equação trigonometrica sem usar a formula da fatoracao (transformacao de soma em produto) a nível de ensino médio? Tipo deduzindo a formula... Se for possivel, ficaria grato pela demonstracao. Gabriel ========================================================================= Instruções para entrar na lista, sair da lista e usar a lista em http://www.mat.puc-rio.br/~nicolau/olimp/obm-l.html O administrador desta lista é ========================================================================= From owner-obm-l@sucuri.mat.puc-rio.br Sun Jun 23 00:44:22 2002 Return-Path: Received: (from majordom@localhost) by sucuri.mat.puc-rio.br (8.9.3/8.9.3) id AAA27932 for obm-l-list; Sun, 23 Jun 2002 00:40:46 -0300 Received: from web20603.mail.yahoo.com (web20603.mail.yahoo.com [216.136.226.161]) by sucuri.mat.puc-rio.br (8.9.3/8.9.3) with SMTP id AAA27928 for ; Sun, 23 Jun 2002 00:40:43 -0300 Message-ID: <20020623034016.52013.qmail@web20603.mail.yahoo.com> Received: from [200.199.72.187] by web20603.mail.yahoo.com via HTTP; Sun, 23 Jun 2002 00:40:16 ART Date: Sun, 23 Jun 2002 00:40:16 -0300 (ART) From: =?iso-8859-1?q?Murilo=20Andrade?= Subject: Re: [obm-l] Trigonometria To: obm-l@mat.puc-rio.br In-Reply-To: <001701c21a4c$9690bda0$0200a8c0@macacos> MIME-Version: 1.0 Content-Type: text/plain; charset=iso-8859-1 Content-Transfer-Encoding: 8bit Sender: owner-obm-l@sucuri.mat.puc-rio.br Precedence: bulk Reply-To: obm-l@mat.puc-rio.br Eu achei a seguinte maneira de resolver: b = 60 - a (sen a + sen b) / (cos a + cos b) = [sen a + sen (60-a)] / [cos a + cos (60-a) ] Aplicando as formulas do seno da soma e do co-seno da soma (que estao a nivel de ensino medio) dá 1/raiz(3). []'s, Murilo --- Gabriel_Pérgola escreveu: > Se a +b = 60º, entao: sen a + sen b / cos a + cos b > = ? > > É possivel resolver essa equação trigonometrica sem > usar a formula da > fatoracao (transformacao de soma em produto) a nível > de ensino médio? Tipo > deduzindo a formula... > Se for possivel, ficaria grato pela demonstracao. > > Gabriel > > > > ========================================================================= > Instruções para entrar na lista, sair da lista e > usar a lista em > http://www.mat.puc-rio.br/~nicolau/olimp/obm-l.html > O administrador desta lista é > > ========================================================================= _______________________________________________________________________ Copa 2002 Yahoo! - Patrocinador oficial da Copa do Mundo da FIFA 2002 http://br.sports.yahoo.com/fifaworldcup/ ========================================================================= Instruções para entrar na lista, sair da lista e usar a lista em http://www.mat.puc-rio.br/~nicolau/olimp/obm-l.html O administrador desta lista é ========================================================================= From owner-obm-l@sucuri.mat.puc-rio.br Sun Jun 23 01:54:57 2002 Return-Path: Received: (from majordom@localhost) by sucuri.mat.puc-rio.br (8.9.3/8.9.3) id BAA28591 for obm-l-list; Sun, 23 Jun 2002 01:52:07 -0300 Received: from pina.terra.com.br (pina.terra.com.br [200.176.3.17]) by sucuri.mat.puc-rio.br (8.9.3/8.9.3) with ESMTP id BAA28587 for ; Sun, 23 Jun 2002 01:52:05 -0300 Received: from pacuiba.terra.com.br (pacuiba.terra.com.br [200.176.3.40]) by pina.terra.com.br (Postfix) with ESMTP id A3A8252E74 for ; Sun, 23 Jun 2002 01:51:40 -0300 (EST) Received: from stabel (dl-nas3-poa-C89A06DB.p001.terra.com.br [200.154.6.219]) (authenticated user dudasta) by pacuiba.terra.com.br (Postfix) with ESMTP id 4998E7FEC for ; Sun, 23 Jun 2002 01:51:39 -0300 (EST) Message-ID: <001001c21a71$aac29260$db069ac8@stabel> From: "Eduardo Casagrande Stabel" To: References: <001701c21a4c$9690bda0$0200a8c0@macacos> Subject: Re: [obm-l] Trigonometria Date: Sun, 23 Jun 2002 01:51:38 -0300 MIME-Version: 1.0 Content-Type: text/plain; charset="iso-8859-1" Content-Transfer-Encoding: 8bit X-Priority: 3 X-MSMail-Priority: Normal X-Mailer: Microsoft Outlook Express 6.00.2600.0000 X-MimeOLE: Produced By Microsoft MimeOLE V6.00.2600.0000 Sender: owner-obm-l@sucuri.mat.puc-rio.br Precedence: bulk Reply-To: obm-l@mat.puc-rio.br Caro Gabriel Pérgola, se você entrar no site http://mathworld.wolfram.com/TrigonometricAdditionFormulas.html vai encontrar duas demonstrações da fórmula sen(a + b) = sen(a)*cos(b) + sen(b)*cos(a) uma delas é usando a definição geral do seno que engloba os números complexos e utiliza a função exponencial e^z. A outra demonstração usa álgebra simples e as definições básicas de seno e cosseno, e essa deve lhe interessar. Para demonstrar as fórmulas sen(a) + sen(b) = 2*sen( (a+b)/2 )*cos( (a-b)/2 ) cos(a) + cos(b) = 2*cos( (a+b)/2 )*cos( (a-b)/2 ) você só precisa saber expandir sen(a + b) e cos(a + b), o que está devidamente demonstrado no site aí de cima. Se for possível, não perca a oportunidade de ler o conteúdo dessa página. O site MathWorld tem muita coisa interessante, vale a pena navegar por ele para sanar talvez outras dúvidas relacionadas. Um abraço! Eduardo Casagrande Stabel. Porto Alegre, RS. From: "Gabriel Pérgola" > Se a +b = 60º, entao: sen a + sen b / cos a + cos b = ? > > É possivel resolver essa equação trigonometrica sem usar a formula da > fatoracao (transformacao de soma em produto) a nível de ensino médio? Tipo > deduzindo a formula... > Se for possivel, ficaria grato pela demonstracao. > > Gabriel > > > > ========================================================================= > Instruções para entrar na lista, sair da lista e usar a lista em > http://www.mat.puc-rio.br/~nicolau/olimp/obm-l.html > O administrador desta lista é > ========================================================================= > > ========================================================================= Instruções para entrar na lista, sair da lista e usar a lista em http://www.mat.puc-rio.br/~nicolau/olimp/obm-l.html O administrador desta lista é ========================================================================= From owner-obm-l@sucuri.mat.puc-rio.br Sun Jun 23 09:14:58 2002 Return-Path: Received: (from majordom@localhost) by sucuri.mat.puc-rio.br (8.9.3/8.9.3) id JAA01432 for obm-l-list; Sun, 23 Jun 2002 09:09:34 -0300 Received: from web20608.mail.yahoo.com (web20608.mail.yahoo.com [216.136.226.166]) by sucuri.mat.puc-rio.br (8.9.3/8.9.3) with SMTP id JAA01428 for ; Sun, 23 Jun 2002 09:09:31 -0300 Message-ID: <20020623120906.96374.qmail@web20608.mail.yahoo.com> Received: from [200.199.51.17] by web20608.mail.yahoo.com via HTTP; Sun, 23 Jun 2002 09:09:06 ART Date: Sun, 23 Jun 2002 09:09:06 -0300 (ART) From: =?iso-8859-1?q?Murilo=20Andrade?= Subject: Re: [obm-l] Trigonometria To: obm-l@mat.puc-rio.br In-Reply-To: <20020623034016.52013.qmail@web20603.mail.yahoo.com> MIME-Version: 1.0 Content-Type: text/plain; charset=iso-8859-1 Content-Transfer-Encoding: 8bit Sender: owner-obm-l@sucuri.mat.puc-rio.br Precedence: bulk Reply-To: obm-l@mat.puc-rio.br --- Murilo Andrade escreveu: > Eu achei a seguinte maneira de resolver: > > b = 60 - a > > (sen a + sen b) / (cos a + cos b) = [sen a + sen > (60-a)] / [cos a + cos (60-a) ] > > Aplicando as formulas do seno da soma e do co-seno > da > soma (que estao a nivel de ensino medio) dá > 1/raiz(3). Aqui o certo eh: Aplicando as formulas do seno da *diferença* e do co-seno da *diferença* (que estao a nivel de ensino medio) dá 1/raiz(3) > > > > > []'s, > Murilo > > --- Gabriel_Pérgola escreveu: > > > Se a +b = 60º, entao: sen a + sen b / cos a + cos b > > = ? > > > > É possivel resolver essa equação trigonometrica > sem > > usar a formula da > > fatoracao (transformacao de soma em produto) a > nível > > de ensino médio? Tipo > > deduzindo a formula... > > Se for possivel, ficaria grato pela demonstracao. > > > > Gabriel > > > > > > > > > ========================================================================= > > Instruções para entrar na lista, sair da lista e > > usar a lista em > > > http://www.mat.puc-rio.br/~nicolau/olimp/obm-l.html > > O administrador desta lista é > > > > > ========================================================================= > > > _______________________________________________________________________ > Copa 2002 > Yahoo! - Patrocinador oficial da Copa do Mundo da > FIFA 2002 > http://br.sports.yahoo.com/fifaworldcup/ > ========================================================================= > Instruções para entrar na lista, sair da lista e > usar a lista em > http://www.mat.puc-rio.br/~nicolau/olimp/obm-l.html > O administrador desta lista é > > ========================================================================= _______________________________________________________________________ Copa 2002 Yahoo! - Patrocinador oficial da Copa do Mundo da FIFA 2002 http://br.sports.yahoo.com/fifaworldcup/ ========================================================================= Instruções para entrar na lista, sair da lista e usar a lista em http://www.mat.puc-rio.br/~nicolau/olimp/obm-l.html O administrador desta lista é ========================================================================= From owner-obm-l@sucuri.mat.puc-rio.br Sun Jun 23 12:06:17 2002 Return-Path: Received: (from majordom@localhost) by sucuri.mat.puc-rio.br (8.9.3/8.9.3) id MAA02770 for obm-l-list; Sun, 23 Jun 2002 12:00:17 -0300 Received: from smtp-4.ig.com.br (smtp-4.ig.com.br [200.226.132.153]) by sucuri.mat.puc-rio.br (8.9.3/8.9.3) with SMTP id MAA02766 for ; Sun, 23 Jun 2002 12:00:16 -0300 Received: (qmail 6112 invoked from network); 23 Jun 2002 14:59:38 -0000 Received: from shasta013151.ig.com.br (HELO windows9) (200.151.13.151) by smtp-4.ig.com.br with SMTP; 23 Jun 2002 14:59:38 -0000 Message-ID: <003101c21ac6$dbeec3e0$970d97c8@windows9> From: "Eric Campos Bastos Guedes" To: References: <001701c21a4c$9690bda0$0200a8c0@macacos> Subject: Re: [obm-l] Trigonometria Date: Sun, 23 Jun 2002 10:32:23 -0300 MIME-Version: 1.0 Content-Type: text/plain; charset="iso-8859-1" Content-Transfer-Encoding: 8bit X-Priority: 3 X-MSMail-Priority: Normal X-Mailer: Microsoft Outlook Express 5.00.2615.200 X-MIMEOLE: Produced By Microsoft MimeOLE V5.00.2615.200 Sender: owner-obm-l@sucuri.mat.puc-rio.br Precedence: bulk Reply-To: obm-l@mat.puc-rio.br > Se a +b = 60º, entao: sen a + sen b / cos a + cos b = ? > a+b = Pi/3 rd sen a = sen ((a+b)/2 + (a-b)/2) = sen x cos y + sen y cos x onde x = (a+b)/2 e y = (a-b)/2 sen b = ((a+b)/2 - (a-b)/2) = sen x cos y - sen y cos x logo sen a + sen b = 2*sen x cos y cos a = cos((a+b)/2 + (a-b)/2) = cos x cos y - sen x sen y cos b = cos((a+b)/2 - (a-b)/2) = cos x cos y + sen x sen y logo cos a + cos b = 2*cos x cos y (i) suponha y /= Pi/2 + k*Pi entao cos y /= 0 e tem-se (sen a + sen b)/(cos a + cos b) = = (2*sen x cos y)/(2*cos x cos y) = = tan x = tan((a+b)/2) = tan ((Pi/3)/2) = = tan (Pi/6) = raiz(3)/3 (ii) se y = Pi/2 + k*Pi então (a-b)/2 = Pi/2 + k*Pi a-b = Pi + 2k*Pi a = b + Pi + 2k*Pi e portanto cos a + cos b = = cos (b + Pi) + cos b = = -cos b + cos b = 0 e não se pode falar em (sen a + sen b)/(cos a + cos b) Um abrac,o! Eric. ========================================================================= Instruções para entrar na lista, sair da lista e usar a lista em http://www.mat.puc-rio.br/~nicolau/olimp/obm-l.html O administrador desta lista é ========================================================================= From owner-obm-l@sucuri.mat.puc-rio.br Sun Jun 23 13:37:01 2002 Return-Path: Received: (from majordom@localhost) by sucuri.mat.puc-rio.br (8.9.3/8.9.3) id NAA03760 for obm-l-list; Sun, 23 Jun 2002 13:32:41 -0300 Received: from smtp-6.ig.com.br (smtp-6.ig.com.br [200.226.132.155]) by sucuri.mat.puc-rio.br (8.9.3/8.9.3) with SMTP id NAA03756 for ; Sun, 23 Jun 2002 13:32:40 -0300 Received: (qmail 5911 invoked from network); 23 Jun 2002 16:31:58 -0000 Received: from 79.100.226.200.in-addr.arpa.ig.com.br (HELO casa) (200.226.100.79) by smtp-6.ig.com.br with SMTP; 23 Jun 2002 16:31:58 -0000 Message-ID: <002a01c21ad3$8998cf70$4f64e2c8@casa> From: "Alex Vieira" To: Cc: "Alex Vieira" Subject: [obm-l] =?iso-8859-1?Q?C=E1lculo_de_Integral?= Date: Sun, 23 Jun 2002 13:32:12 -0300 MIME-Version: 1.0 Content-Type: multipart/alternative; boundary="----=_NextPart_000_0027_01C21ABA.61880AA0" X-Priority: 3 X-MSMail-Priority: Normal X-Mailer: Microsoft Outlook Express 6.00.2600.0000 X-MimeOLE: Produced By Microsoft MimeOLE V6.00.2600.0000 Sender: owner-obm-l@sucuri.mat.puc-rio.br Precedence: bulk Reply-To: obm-l@mat.puc-rio.br This is a multi-part message in MIME format. ------=_NextPart_000_0027_01C21ABA.61880AA0 Content-Type: text/plain; charset="iso-8859-1" Content-Transfer-Encoding: quoted-printable Caros colegas, Alguem poderia me ajudar? Calcule: INTEGRAL INDEFINIDA NA VARIAVEL x DE ( 1/(RAIZ(x^2-2*x+5)).... Resolvi fazendo um quadrado perfeito no denominador, depois usei algumas formulas trigonometricas, chegando em: ln( | RAIZ(1+((x-1)/2)^2) + (x-1)/2 | ) + C, C real..... que acho que = estah certo... onde ln(algo) eh o logaritmo natural de algo, | algo | eh o m=F3dulo (ou = valor absoluto) de algo e RAIZ eh a raiz quadrada... Tenho por gabarito: ln( | RAIZ(x^2-2*x+5) + x - 1 | ) + C, C real.... que sei que estah certo.... mas nao sei como chegar nessa resposta.... Alguem me ajuda a chegar nesta ultima solucao? Obrigado Alex ------=_NextPart_000_0027_01C21ABA.61880AA0 Content-Type: text/html; charset="iso-8859-1" Content-Transfer-Encoding: quoted-printable
Caros colegas,
 
Alguem poderia me ajudar?
 
Calcule:
INTEGRAL INDEFINIDA NA VARIAVEL x = DE (=20 1/(RAIZ(x^2-2*x+5))....
 
Resolvi fazendo um quadrado perfeito no = denominador, depois usei algumas
formulas trigonometricas, chegando = em:
 
ln( | RAIZ(1+((x-1)/2)^2) + (x-1)/2 | ) = + C, C=20 real..... que acho que estah certo...
 
onde ln(algo) eh o logaritmo = natural de algo,=20 | algo | eh o m=F3dulo (ou valor absoluto) de algo e
RAIZ eh a raiz quadrada...
 
Tenho por gabarito:
 
ln( | RAIZ(x^2-2*x+5) + x - 1 | ) + C, = C=20 real....
 
que sei que estah certo.... mas nao sei = como chegar=20 nessa resposta....
 
Alguem me ajuda a chegar nesta ultima=20 solucao?
 
Obrigado
 
Alex
------=_NextPart_000_0027_01C21ABA.61880AA0-- ========================================================================= Instruções para entrar na lista, sair da lista e usar a lista em http://www.mat.puc-rio.br/~nicolau/olimp/obm-l.html O administrador desta lista é ========================================================================= From owner-obm-l@sucuri.mat.puc-rio.br Sun Jun 23 13:37:13 2002 Return-Path: Received: (from majordom@localhost) by sucuri.mat.puc-rio.br (8.9.3/8.9.3) id NAA03772 for obm-l-list; Sun, 23 Jun 2002 13:33:27 -0300 Received: from hotmail.com (f87.law3.hotmail.com [209.185.241.87]) by sucuri.mat.puc-rio.br (8.9.3/8.9.3) with ESMTP id NAA03768 for ; Sun, 23 Jun 2002 13:33:24 -0300 Received: from mail pickup service by hotmail.com with Microsoft SMTPSVC; Sun, 23 Jun 2002 09:32:59 -0700 Received: from 200.151.56.188 by lw3fd.law3.hotmail.msn.com with HTTP; Sun, 23 Jun 2002 16:32:59 GMT X-Originating-IP: [200.151.56.188] From: "Marcelo Souza" To: obm-l@mat.puc-rio.br Subject: Re: [obm-l] Re: [obm-l] Quadriláteros Date: Sun, 23 Jun 2002 16:32:59 +0000 Mime-Version: 1.0 Content-Type: text/html Message-ID: X-OriginalArrivalTime: 23 Jun 2002 16:32:59.0979 (UTC) FILETIME=[A34A91B0:01C21AD3] Sender: owner-obm-l@sucuri.mat.puc-rio.br Precedence: bulk Reply-To: obm-l@mat.puc-rio.br

Como a prova disto eh geometrica (se necessario dizer, deigualdade triangular) eh possivel mostrar, se naum me engano, que a relacao

p< D+d<2p, a soma daas diagonais eh maior que o semiperimetro e menor que o perimetro.

Logo, p<10<2p, logo 2p>10....aposto que esta questao eh de multiplla escolha, entaum procure por uma opcao que seja maior que 10, que deve se  a resposta, mas achar um valor exato naum dah.

Falow

Marcelo!

>From: "Igor Castro"
>Reply-To: obm-l@mat.puc-rio.br
>To:
>Subject: [obm-l] Re: [obm-l] Quadriláteros
>Date: Thu, 20 Jun 2002 15:36:49 -0300
>
>Fala ae rick
>Na primeira ache as duas diagonais(usando hiparco e depois ptolomeu), em
>seguida aplique a relação da mediana de euller...
>a para achar o raio, use a formula abc/4R em um triangulo que possua uma
>diagonal como lado(assim ele estará inscrito)...
>no segundo analisei apenas com existencia de triangulos e cheguei que o
>perimetro é maior que 10... mas um valor exato n achei...
>[]'s
>
>----- Original Message -----
>From:
>To:
>Sent: Wednesday, June 19, 2002 11:10 PM
>Subject: [obm-l] Quadriláteros
>
>
> > Olá amigos , será que poderiam me ajudar nestas duas questões :
> > 1- Seja um quadrilátero inscritível ABCD cujos lados AB , BC , CD e DA
>medem
> > respectivamente 1 , 2 , 2 e 3 .Calcule a mediana de Euller do quadrilátero
> > e o raio do círculo circunscrito.
> >
> >
> > 2-Um quadrilátero convexo O tem diagonais respectivamente iguais a 4 e 6
> > .Qual um possível valor para o seu perímetro .
> >
> > Abraço..
> >
> >
> > ----------------------------------------
> > |-=Rick-C.R.B.=- |
> > |ICQ 124805654 |
> > |e-mail luizhenriquerick@zipmail.com.br |
> > ----------------------------------------
> >
> >
> > ------------------------------------------
> > Use o melhor sistema de busca da Internet
> > Radar UOL - http://www.radaruol.com.br
> >
> >
> >
> > =========================================================================
> > Instruções para entrar na lista, sair da lista e usar a lista em
> > http://www.mat.puc-rio.br/~nicolau/olimp/obm-l.html
> > O administrador desta lista é
> > =========================================================================
> >
>
>
>=========================================================================
>Instruções para entrar na lista, sair da lista e usar a lista em
>http://www.mat.puc-rio.br/~nicolau/olimp/obm-l.html
>O administrador desta lista é
>=========================================================================


Get your FREE download of MSN Explorer at http://explorer.msn.com.
========================================================================= Instruções para entrar na lista, sair da lista e usar a lista em http://www.mat.puc-rio.br/~nicolau/olimp/obm-l.html O administrador desta lista é ========================================================================= From owner-obm-l@sucuri.mat.puc-rio.br Sun Jun 23 14:46:19 2002 Return-Path: Received: (from majordom@localhost) by sucuri.mat.puc-rio.br (8.9.3/8.9.3) id OAA05015 for obm-l-list; Sun, 23 Jun 2002 14:42:23 -0300 Received: from traven10.uol.com.br (traven10.uol.com.br [200.231.206.211]) by sucuri.mat.puc-rio.br (8.9.3/8.9.3) with ESMTP id OAA05011 for ; Sun, 23 Jun 2002 14:42:21 -0300 Received: from cabru ([200.158.70.217]) by traven10.uol.com.br (8.9.1/8.9.1) with SMTP id OAA07844 for ; Sun, 23 Jun 2002 14:41:57 -0300 (BRT) Message-ID: <003201c21add$96420980$d9469ec8@cabru> From: "Bruno" To: References: <000701c217f1$893982c0$4390dec8@igor> <3D11310B.30405@niski.com> <02ca01c218d6$ae201000$6600a8c0@altus.com.br> <009601c218e1$7d9043f0$0e069ac8@stabel> <3D1356EC.8090803@niski.com> Subject: [obm-l] =?iso-8859-1?Q?Re:_=5Bobm-l=5D_Re:_=5Bobm-l=5D_Re:_=5Bobm-l=5D_Re:_=5Bo?= =?iso-8859-1?Q?bm-l=5D_Fatora=E7=E3o?= Date: Sat, 22 Jun 2002 21:42:56 -0300 MIME-Version: 1.0 Content-Type: text/plain; charset="iso-8859-1" Content-Transfer-Encoding: 8bit X-Priority: 3 X-MSMail-Priority: Normal X-Mailer: Microsoft Outlook Express 5.50.4133.2400 X-MimeOLE: Produced By Microsoft MimeOLE V5.50.4133.2400 Sender: owner-obm-l@sucuri.mat.puc-rio.br Precedence: bulk Reply-To: obm-l@mat.puc-rio.br olá, todos nós em 1º momento pensamos em fatorar usando os complexos... é normal..... abraços... ----- Original Message ----- From: "niski" To: Sent: Friday, June 21, 2002 1:40 PM Subject: Re: [obm-l] Re: [obm-l] Re: [obm-l] Re: [obm-l] Fatoração > > > > > > > >Eduardo Casagrande Stabel. Porto Alegre, RS. > > > Esta errado Eduardo. É pedido para fatorar em R voce restringiu o > dominio, logo não obedebeu as condicoes do enunciado. > Veja o resultado da fatoracao na minha msg. > > > > > > ========================================================================= > Instruções para entrar na lista, sair da lista e usar a lista em > http://www.mat.puc-rio.br/~nicolau/olimp/obm-l.html > O administrador desta lista é > ========================================================================= ========================================================================= Instruções para entrar na lista, sair da lista e usar a lista em http://www.mat.puc-rio.br/~nicolau/olimp/obm-l.html O administrador desta lista é ========================================================================= From owner-obm-l@sucuri.mat.puc-rio.br Sun Jun 23 15:31:44 2002 Return-Path: Received: (from majordom@localhost) by sucuri.mat.puc-rio.br (8.9.3/8.9.3) id PAA05348 for obm-l-list; Sun, 23 Jun 2002 15:27:55 -0300 Received: from saks.bol.com.br (saks.bol.com.br [200.221.24.16]) by sucuri.mat.puc-rio.br (8.9.3/8.9.3) with ESMTP id PAA05344 for ; Sun, 23 Jun 2002 15:27:54 -0300 Received: from bol.com.br (200.221.24.138) by saks.bol.com.br (5.1.071) id 3CDE092B00BEE4A2 for obm-l@mat.puc-rio.br; Sun, 23 Jun 2002 15:26:46 -0300 Date: Sun, 23 Jun 2002 16:26:46 -0200 Message-Id: Subject: [obm-l] =?iso-8859-1?q?Re=3A=5Bobm=2Dl=5D_C=E1lculo_de_Integral?= MIME-Version: 1.0 Content-Type: multipart/mixed; boundary="_=__=_XaM3_Boundary.1024856806.2A.258562.42.1785.52.42.101010.459952558" From: "ozorio_loof" To: obm-l@mat.puc-rio.br X-XaM3-API-Version: 2.4.3.4.4 X-SenderIP: 143.107.130.187 Sender: owner-obm-l@sucuri.mat.puc-rio.br Precedence: bulk Reply-To: obm-l@mat.puc-rio.br --_=__=_XaM3_Boundary.1024856806.2A.258562.42.1785.52.42.101010.459952558 Content-Type: text/plain;charset="iso-8859-1" Content-Transfer-Encoding: quoted-printable Sua resposta est=E1 correta. Basta desenvolver o termo (x-1)^2 e depois usar o fato de que ln(a/b)=3Dln (a)-ln(b). Assim seque que ln[raiz(x^2-2x+5)/2 +(x-1)/2]=3D ln[raiz(x^2-2x+5)+(x-1)]-ln2+C. O termo -ln2+C--> constante. []'s Luiz. __________________________________________________________________________ AcessoBOL, s=F3 R$ 9,90! O menor pre=E7o do mercado! Assine j=E1! http://www.bol.com.br/acessobol --_=__=_XaM3_Boundary.1024856806.2A.258562.42.1785.52.42.101010.459952558 Content-Type: text/plain; name="00000P9N" Content-Transfer-Encoding: base64 Q2Fyb3MgY29sZWdhcywNCg0KQWxndWVtIHBvZGVyaWEgbWUgYWp1ZGFyPw0KDQpDYWxjdWxl Og0KSU5URUdSQUwgSU5ERUZJTklEQSBOQSBWQVJJQVZFTCB4IERFICggMS8oUkFJWih4XjIt Mip4KzUpKS4uLi4NCg0KUmVzb2x2aSBmYXplbmRvIHVtIHF1YWRyYWRvIHBlcmZlaXRvIG5v IGRlbm9taW5hZG9yLCBkZXBvaXMgdXNlaSBhbGd1bWFzDQpmb3JtdWxhcyB0cmlnb25vbWV0 cmljYXMsIGNoZWdhbmRvIGVtOg0KDQpsbiggfCBSQUlaKDErKCh4LTEpLzIpXjIpICsgKHgt MSkvMiB8ICkgKyBDLCBDIHJlYWwuLi4uLiBxdWUgYWNobyBxdWUgZXN0YWggY2VydG8uLi4N Cg0Kb25kZSBsbihhbGdvKSBlaCBvIGxvZ2FyaXRtbyBuYXR1cmFsIGRlIGFsZ28sIHwgYWxn byB8IGVoIG8gbfNkdWxvIChvdSB2YWxvciBhYnNvbHV0bykgZGUgYWxnbyBlDQpSQUlaIGVo IGEgcmFpeiBxdWFkcmFkYS4uLg0KDQpUZW5obyBwb3IgZ2FiYXJpdG86DQoNCmxuKCB8IFJB SVooeF4yLTIqeCs1KSArIHggLSAxIHwgKSArIEMsIEMgcmVhbC4uLi4NCg0KcXVlIHNlaSBx dWUgZXN0YWggY2VydG8uLi4uIG1hcyBuYW8gc2VpIGNvbW8gY2hlZ2FyIG5lc3NhIHJlc3Bv c3RhLi4uLg0KDQpBbGd1ZW0gbWUgYWp1ZGEgYSBjaGVnYXIgbmVzdGEgdWx0aW1hIHNvbHVj YW8/DQoNCk9icmlnYWRvDQoNCkFsZXgNCg== --_=__=_XaM3_Boundary.1024856806.2A.258562.42.1785.52.42.101010.459952558-- ========================================================================= Instruções para entrar na lista, sair da lista e usar a lista em http://www.mat.puc-rio.br/~nicolau/olimp/obm-l.html O administrador desta lista é ========================================================================= From owner-obm-l@sucuri.mat.puc-rio.br Sun Jun 23 15:45:07 2002 Return-Path: Received: (from majordom@localhost) by sucuri.mat.puc-rio.br (8.9.3/8.9.3) id PAA05522 for obm-l-list; Sun, 23 Jun 2002 15:41:17 -0300 Received: from smtp.ieg.com.br (12.139.226.200.in-addr.arpa.ig.com.br [200.226.139.12] (may be forged)) by sucuri.mat.puc-rio.br (8.9.3/8.9.3) with ESMTP id PAA05518 for ; Sun, 23 Jun 2002 15:41:13 -0300 Received: from joaodias (200-157-18-217.intelignet.com.br [200.157.18.217] (may be forged)) by smtp.ieg.com.br (IeG relay/8.9.3) with SMTP id g5NIgL9R077749 for ; Sun, 23 Jun 2002 15:42:24 -0300 (BRT) Message-ID: <000301c21ae5$6a5b5a80$d9129dc8@joaodias> From: "Fabio Dias" To: References: <002a01c21ad3$8998cf70$4f64e2c8@casa> Subject: [obm-l] =?iso-8859-1?Q?Re:_=5Bobm-l=5D_C=E1lculo_de_Integral?= Date: Sun, 23 Jun 2002 14:45:12 -0300 MIME-Version: 1.0 Content-Type: text/plain; charset="iso-8859-1" Content-Transfer-Encoding: 8bit X-Priority: 3 X-MSMail-Priority: Normal X-Mailer: Microsoft Outlook Express 6.00.2600.0000 X-MimeOLE: Produced By Microsoft MimeOLE V6.00.2600.0000 Sender: owner-obm-l@sucuri.mat.puc-rio.br Precedence: bulk Reply-To: obm-l@mat.puc-rio.br ----- Original Message ----- From: Alex Vieira To: obm-l@mat.puc-rio.br Cc: Alex Vieira Sent: Sunday, June 23, 2002 1:32 PM Subject: [obm-l] Cálculo de Integral >[...] > >ln( | RAIZ(1+((x-1)/2)^2) + (x-1)/2 | ) + C, C real..... que acho que estah certo... > >[...] > >Tenho por gabarito: > >ln( | RAIZ(x^2-2*x+5) + x - 1 | ) + C, C real.... > >que sei que estah certo.... mas nao sei como chegar nessa resposta.... > ln(|(x^2-2x+5)^(1/2) + x - 1|) + C = ln((1/2)*|(x^2-2x+5)^(1/2)+ x - 1|) + ln(2) + C = ln(|((x^2-2x+1)/4 + 1)^(1/2) + (x-1)/2|) + ln(2) + C = ln(|(1+((x-1)/2)^2) + (x-1)/2|) + C'; C' = C + ln(2) []s, Fábio Dias ========================================================================= Instruções para entrar na lista, sair da lista e usar a lista em http://www.mat.puc-rio.br/~nicolau/olimp/obm-l.html O administrador desta lista é ========================================================================= From owner-obm-l@sucuri.mat.puc-rio.br Sun Jun 23 16:49:58 2002 Return-Path: Received: (from majordom@localhost) by sucuri.mat.puc-rio.br (8.9.3/8.9.3) id QAA07059 for obm-l-list; Sun, 23 Jun 2002 16:45:57 -0300 Received: from smtp012.mail.yahoo.com (smtp012.mail.yahoo.com [216.136.173.32]) by sucuri.mat.puc-rio.br (8.9.3/8.9.3) with SMTP id QAA07055 for ; Sun, 23 Jun 2002 16:45:54 -0300 Received: from 200-204-17-170.dial-up.telesp.net.br (HELO e9i5z1) (caio?voznak@200.204.17.170 with login) by smtp.mail.vip.sc5.yahoo.com with SMTP; 23 Jun 2002 19:45:27 -0000 Message-ID: <00be01c21a1d$122fee20$c511ccc8@e9i5z1> From: "Caio H. Voznak" To: Subject: [obm-l] Geometria interssante Date: Sat, 22 Jun 2002 15:45:07 -0300 MIME-Version: 1.0 Content-Type: multipart/mixed; boundary="----=_NextPart_000_00BA_01C21A03.C8CE9100" X-Priority: 3 X-MSMail-Priority: Normal X-Mailer: Microsoft Outlook Express 5.50.4133.2400 X-MimeOLE: Produced By Microsoft MimeOLE V5.50.4133.2400 Sender: owner-obm-l@sucuri.mat.puc-rio.br Precedence: bulk Reply-To: obm-l@mat.puc-rio.br This is a multi-part message in MIME format. ------=_NextPart_000_00BA_01C21A03.C8CE9100 Content-Type: multipart/alternative; boundary="----=_NextPart_001_00BB_01C21A03.C8CE9100" ------=_NextPart_001_00BB_01C21A03.C8CE9100 Content-Type: text/plain; charset="iso-8859-1" Content-Transfer-Encoding: quoted-printable Por favor ser=E1 que alguem conhece um solu=E7=E3o para a seguinte = quest=E3o: S=E3o dadas duas retas convergentes em um ponto O que formam um angulo = agudo teta entre si, tamb=E9m =E9 dado um ponto P localizado abaixo das = retas, ambos fixos, e uma medida d. =C9 pedido uma semireta com in=EDcio = em P e que corte ambas as retas convergentes obtendo a medida d entre as = retas. Segue um esbo=E7o em anexo. =20 Um abra=E7o, Caio Voznak --- Outgoing mail is certified Virus Free. Checked by AVG anti-virus system (http://www.grisoft.com). Version: 6.0.345 / Virus Database: 193 - Release Date: 9/4/2002 ------=_NextPart_001_00BB_01C21A03.C8CE9100 Content-Type: text/html; charset="iso-8859-1" Content-Transfer-Encoding: quoted-printable
Por favor ser=E1 que alguem conhece um = solu=E7=E3o para a=20 seguinte quest=E3o:
 
S=E3o dadas duas retas convergentes em = um ponto O que=20 formam um angulo agudo = teta entre si, tamb=E9m=20 =E9 dado um ponto P localizado abaixo das retas, ambos fixos, e uma = medida d.=20 =C9 pedido uma semireta com in=EDcio em P e que corte ambas as retas=20 convergentes obtendo a medida d entre as retas. Segue um = esbo=E7o em=20 anexo.   
 
Um abra=E7o,
Caio Voznak
 

---
Outgoing mail is certified = Virus=20 Free.
Checked by AVG anti-virus system (http://www.grisoft.com).
Version: = 6.0.345 /=20 Virus Database: 193 - Release Date: 9/4/2002
------=_NextPart_001_00BB_01C21A03.C8CE9100-- ------=_NextPart_000_00BA_01C21A03.C8CE9100 Content-Type: image/jpeg; name="=?iso-8859-1?Q?esbo=E7o.jpg?=" Content-Transfer-Encoding: base64 Content-Disposition: attachment; filename="=?iso-8859-1?Q?esbo=E7o.jpg?=" /9j/4AAQSkZJRgABAQEBLAEsAAD/2wBDAAgGBgcGBQgHBwcJCQgKDBQNDAsLDBkSEw8UHRofHh0a HBwgJC4nICIsIxwcKDcpLDAxNDQ0Hyc5PTgyPC4zNDL/2wBDAQkJCQwLDBgNDRgyIRwhMjIyMjIy MjIyMjIyMjIyMjIyMjIyMjIyMjIyMjIyMjIyMjIyMjIyMjIyMjIyMjIyMjL/wAARCAEsAZADASIA AhEBAxEB/8QAHwAAAQUBAQEBAQEAAAAAAAAAAAECAwQFBgcICQoL/8QAtRAAAgEDAwIEAwUFBAQA AAF9AQIDAAQRBRIhMUEGE1FhByJxFDKBkaEII0KxwRVS0fAkM2JyggkKFhcYGRolJicoKSo0NTY3 ODk6Q0RFRkdISUpTVFVWV1hZWmNkZWZnaGlqc3R1dnd4eXqDhIWGh4iJipKTlJWWl5iZmqKjpKWm p6ipqrKztLW2t7i5usLDxMXGx8jJytLT1NXW19jZ2uHi4+Tl5ufo6erx8vP09fb3+Pn6/8QAHwEA AwEBAQEBAQEBAQAAAAAAAAECAwQFBgcICQoL/8QAtREAAgECBAQDBAcFBAQAAQJ3AAECAxEEBSEx BhJBUQdhcRMiMoEIFEKRobHBCSMzUvAVYnLRChYkNOEl8RcYGRomJygpKjU2Nzg5OkNERUZHSElK U1RVVldYWVpjZGVmZ2hpanN0dXZ3eHl6goOEhYaHiImKkpOUlZaXmJmaoqOkpaanqKmqsrO0tba3 uLm6wsPExcbHyMnK0tPU1dbX2Nna4uPk5ebn6Onq8vP09fb3+Pn6/9oADAMBAAIRAxEAPwD3+iii gAooooAKKKKACiiigAooooAKKKKACiiigAooooAKKKKACiiigAooooAKKKKACiiigAooooAKKKKA CiiigAooooAKKKKACiiigAooooAKKKKACiiigAooooAKKKKACiiigAooooAKKKKACiiigAooooAK KKKACiiigAooooAKKKKACiiigAooooAKKKKACiiigAooooAKKKKACiiigAooooAKKKKACiiigAoo ooAKKKKACiiigAooooAKKKKACiiigAooooAKKKKACiiigAooooAKKKKACiiigAooooAKKKKACiii gAooooAKKKKACiiigAooqOeeG1t5bi4ljhgiQvJJIwVUUDJJJ4AA5zQBJRWXpuuQ6tcMLK1u5LMI St+0YSCQ5HCbiGcEEMHVTGR0YnitSgAooooAKKKKACiiigAooooAKKKKACiiigAooooAKKz9T1G6 sPK+zaNfalvzu+yPAvl4xjPmyJ1z2z0OccZp+HvEUniG3gu00TUrOzuLdbiG5umg2yKwBUAJKzAk HPIHQ5weKANyiiigAooooAKKKKACiiigAooooAKKKKACiiigAooooAKKKKACiiigAooooAKKKKAC iiigAoorn/8AhL7C740OKfXmH3jphjeNB3zM7LFuHHyb9+GB245oA6CsvUvEekaTcLa3d9GLx0Dx 2cQMtxIuSMpCgLsODkhTgKT0Bqn/AGZruqfNqeqf2dCeDZaUQcr0YPO67zkDgxrEy7iMkgMNTTdK sdIt2gsLaOBHcySFeWlcgAu7Hl3OBlmJJ7k0AZf2jxFq/wAkFl/YVv0ea7Mc1yfXy0jZo14OQ7M2 CpBjI5qSDwppouIrzURJq9/E4eO61HbI0TA8GNQAkR4XJjVc7QTkjNblFABRRRQAUUUUAFFFFABR RRQAUUUUAFFFFABRRRQAUUUUAFc/4E/5J54a/wCwVa/+ilroK5/wJ/yTzw1/2CrX/wBFLQB0FFFF ABRRRQAUUUUAFFFFABRRRQAUUUUAFFFFABRRRQAUUUUAFFFFABRRRQAUVj33ifS7O8ksEn+2ammM 6fZ/vZwSAV3KP9WpyvzvtQblywzVfZ4l1XiR4NCtTyDAy3N2wPIBLL5UTDADDEwOSARgMQDU1LVt N0a3W41TULSxgZwiyXUyxKWwTgFiBnAJx7Gsv+2NY1P/AJA2k+Vbnj7Zqu+368bkg2+Y205yr+Vn A2sQdwuaboFjptw14FkudQdCkl9dN5k7KSCVDH7iFhu8tAqAk4UVqUAc/wD8Isl9z4gvp9YU8m1n VUtFJ6gQqAHXOColMhXaCDnJPQUUUAFFFFABRRRQAUUUUAFFFFABRRRQAUUUUAFFFFABRRRQAUUU UAFFFFABXP8AgT/knnhr/sFWv/opa6Cuf8Cf8k88Nf8AYKtf/RS0AdBWfDqfm+Ib3SfJx9mtILnz d33vNeZduMcY8nOc87u2OcvxbG15LoGmNNJHaX+piK7RAv7+JIJpTE2QfkZolDDupYdCar6DpVjo /jrW7bTraO1tzpli4gi+WNCZbsHYnRAcZIUAEknqSSAdZRRRQAUUUUAFFFFABRRRQAUUUUAFFFFA BRRRQAUUVn6nrml6N5Q1G/gt5JsiCFn/AHk5GMrGg+aRuQNqgkkgAZIoA0KjnnhtbeW4uJY4YIkL ySSMFVFAySSeAAOc1h/2h4g1PnTNOg0+1b7t1qZYyY6hhbJglSMDDyRuCTlPlw0kHhWxFxFdahNd 6tdxuJFlv5d6q6n5XWEYiRxjAZEU9ecliQCP/hJ/7R+Tw3Z/2t/09+b5VmP+22G39GH7pZMMMNt6 0f2LqmpfPrGszxRt96x0xvIjA6gGYDzmYH+NWjDADKAFgegooAr2NhZ6ZZx2dhaQWlrHnZDBGI0X JJOFHAyST+NWKKKACiiigAooooAKKKKACiiigAooooAKKKKACiiigAooooAKKKKACiiigAooooAK KKKACuf8Cf8AJPPDX/YKtf8A0UtdBXP+BP8Aknnhr/sFWv8A6KWgDYvrG31GzktbqPzIXwSAxUgg gqysMFWBAIYEEEAggiqeleHtM0W4uLmygkW4uURJ55Z5JZJghYrvd2JYjeQCTnGB0UAalFABRRRQ AUUUUAFFFFABRRRQAUUUUAc/qBfWPEJ0MTzwWdvaLdXZgmaGSQyOywqsiEMFHlSlsFTkRjJUupNP L6P4hGhmeeezuLRrq0M8zTSRmN1WZWkclip82IrkscmQZChFBqAfR/EJ1wQTz2dxaLa3YghaaSMx uzQssaAsVPmyhsBjkxnAUOwNPD6x4hGuGCeCzt7RrW0E8LQySGR1aZmjcBgo8qILkKciQ4KlGIB0 FYc/iqxFxLa6fDd6tdxuY2isIt6q6n5kaY4iRxjJV3U9OMlQbms6LZa/pz2GorO9q+Q6Q3MkO8EE FWMbKWUgnKng+lZcHhe8063it9L8S6lb29sgS1tJIbeSCNFGEjP7oSMgAA/1gYj+LPNAEn2LxBq3 y6ndQaXa9Gt9MlaWSUd8zuilFIJGEQOCARIOlaGmaJp2j+a1lb7ZpsedcSO0s02M7fMlcl3wCQNx OBwMCs/Z4wtfn8/Q9Tzx5PkzWO3/AGvM3TZ9Nu0dc7hjBP7X8RWv/H74X+0bvuf2VfxzbfXf54hx 2xt3d844yAdBRXP/APCXWsXyXul65a3A+/D/AGVPcbfT95ArxnjB+VjjODgggSQeNPCt1cRW9v4l 0aaeVwkccd/EzOxOAAA2SSeMUAblFFFABRRRQAUUUUAFFFFABRRRQAUUUUAFFFFABRRRQAUUUUAF FFFABRRRQAUUUUAFFFFABRRRQAVz/gT/AJJ54a/7BVr/AOilrQ1PRrXV/K+0y30flZ2/ZL+e2znG c+U67unfOOcdTVPSvCWlaK9u1g2pIlsmyKF9UuZIlXbtA8t5CmAOgxxxjoKANyiiigAooooAKKKK ACiiigAooooAKKKKACiiigAooooAKKKKACo54Ibq3lt7iKOaCVCkkcihldSMEEHggjjFSUUAc/8A 8IP4aj5s9Ig06ToZtMLWUjD+6XhKsV6HaTjIBxkCj/hFfs//ACDde1yx3f6z/TPte/0/4+RLtxz9 3bnPOcDHQUUAc/8A2d4pg/d23iKxlhH3Xv8ASzJMfXc0UsSHnphF4xnJySfbvFkfzy6BpTxryy2+ ru0jDuED26qW9AzKM9SBzXQUUAc//wAJRLB82o+HNcsoTwJPIS6y3pttnlccZ5KheMZyQCf8Jx4a j4vNXg06TqIdTDWUjD+8EmCsV6jcBjIIzkGugooAr2N/Z6nZx3lhdwXdrJnZNBIJEbBIOGHBwQR+ FWKx77wn4b1O8kvL/wAP6Vd3UmN809lHI7YAAyxGTgAD8Kr/APCI2sXz2Wqa5a3A+5N/as9xt9f3 c7PGeMj5lOM5GCAQAdBRXP8A9ja9bfPZ+Kp55DwV1OyhljA9QIVhbd05LEYzwTgg3+MLX5PI0PU8 8+d501jt/wBny9s2fXduHXG0YyQDoKK5/wDtzW4P3dz4SvpZh957C7tpIT6bWlkic8dcovOcZGCT /hNNET5rl76yhH3ri/025tYU9N0ssaouTwMkZJAHJFAHQUVl6b4l0HWbhrfS9b02+nVC7R2t0krB cgZIUk4yQM+4rUoAKKKKACiiigAooooAKKKKACiiigAooooAKKKKACiiigAooooAKKKKACiiigAo oooAKKKKACiiigAooooAKKKKACiiigAooooAKKKKACiiigAooooAKKKKACiiigAooooAp6lpOm6z brb6pp9pfQK4dY7qFZVDYIyAwIzgkZ9zWX/whWgx8WdrPp0fUw6ZeTWUbH+8UhdVLdBuIzgAZwBX QUUAZem6RPp1wzHW9Su7fYUjtroxMsYyMYcIJGIAxl3YnJJyea1KKKACiiigAooooAKKKKACiiig AooooAKKKKACiiigAooooAKKKKACiiigAooooAKKKKACiiigAooooAKKKKACiiigAooooAKKKKAC iiigAooooAKKKKACiiigAooooAKKKKACiiigAooooAKKKKACiiigAooooAKKKKACiiigAooooAKK KKACiiigAooooAKKKKACiiigAooooAKKKKACiiigAooooAKKKKACiiigAry/SLfwrJHqBuvDM95q 51XUMXNvo8vmM/2uXZsuggVGHygP5i7COWXbkeoVj+GrG40/Sp4bqPy5G1C9mA3A5SS6lkQ8eqsp 9s880AXNJjvodGsYtUmjn1BLeNbqWMYV5Qo3sOBwWyeg+gqO7tNRm1jTrm21T7PYweb9rs/s6v8A aty4T5zym088dehrQooAKKKKACiiigAooooAKKKKACiiigAooooAKKKKACiiigAooooAKKKKACii igAooooAKKKKACiiigAooooAKKKKACiiigAooooAKKKKACiiigAooooAKKKKACiiigAooooAKKKK ACiiigAooooAKKKKACiiigAooooAKKKKACiiigAooooAKKKKACiiigAooooAKKKKACiiigAooooA KKKKACiiigAooooAKKKKACiiigAooooAKKKKACiiigAooooAKKKKACiiigAooooAKKKKACiiigAo oooAKKKKACiiigAooooAKKKKACiiigAooooAKKKKACiiigAooooAKKKKACiiigAooooAKKKKACii igAooooAKKKKACiiigAooooAKKKKACiiigAooooAKKKKACiiigAooooAKKKKACiiigAooooAKKKK ACiiigAooooAKKKKACiiigAooooAKKKKACiiigAooooAKKKKACiiigAooooAKKKKACiiigAooooA KKKKACiiigAooooAKKKKACiiigAooooAKKKKACiiigAooooAKKKKACiiigAooooAKKKKACiiigAo oooAKKKKACiiigAooooAKKKKACiiigAooooAKKKKACiiigAooooAKKKKACiiigAooooAKKKKACii igAooooAKKKKACiiigAooooAKKKKACiiigAooooAKKKKACiiigAooooAKKKKACiiigAooooAKKKK ACiiigAooooAKKKKACiiigAooooAKKKKACiiigAooooAKKKKACiiigAooooAKKKKACiiigAooooA KKKKACiiigAooooAKKKKACiiigAooooAKKKKACiiigAooooAKKKKACiiigAooooAKKKKACiiigAo oooAKKKKACiiigAooooAKKKKACiiigAooooAKKKKACiiigAooooAKKKKACiiigAooooAKKKKACii igAooooAKKKKACiiigAooooAKKKKACiiigAooooAKKKKACiiigAooooAKKKKACiiigAooooAKKKK ACiiigAooooAKKKKACiiigAooooAKKKKACiiigAooooAKKKKACiiigAooooAKKKKACiiigAooooA KKKKACiiigAooooAKKKKACiiigAooooA/9k= ------=_NextPart_000_00BA_01C21A03.C8CE9100-- _________________________________________________________ Do You Yahoo!? Get your free @yahoo.com address at http://mail.yahoo.com ========================================================================= Instruções para entrar na lista, sair da lista e usar a lista em http://www.mat.puc-rio.br/~nicolau/olimp/obm-l.html O administrador desta lista é ========================================================================= From owner-obm-l@sucuri.mat.puc-rio.br Sun Jun 23 18:57:19 2002 Return-Path: Received: (from majordom@localhost) by sucuri.mat.puc-rio.br (8.9.3/8.9.3) id SAA08490 for obm-l-list; Sun, 23 Jun 2002 18:54:10 -0300 Received: from pina.terra.com.br (pina.terra.com.br [200.176.3.17]) by sucuri.mat.puc-rio.br (8.9.3/8.9.3) with ESMTP id SAA08486 for ; Sun, 23 Jun 2002 18:54:08 -0300 Received: from engenho.terra.com.br (engenho.terra.com.br [200.176.3.42]) by pina.terra.com.br (Postfix) with ESMTP id 5CA4E52F5A for ; Sun, 23 Jun 2002 18:53:45 -0300 (EST) Received: from terra.com.br (200-206-243-51.dsl.telesp.net.br [200.206.243.51]) (authenticated user lponce) by engenho.terra.com.br (Postfix) with ESMTP id D050C680F4 for ; Sun, 23 Jun 2002 18:53:44 -0300 (EST) Message-ID: <3D164336.E323E657@terra.com.br> Date: Sun, 23 Jun 2002 18:52:55 -0300 From: Luiz Antonio Ponce Alonso X-Mailer: Mozilla 4.79 [en] (Windows NT 5.0; U) X-Accept-Language: en MIME-Version: 1.0 To: obm-l@mat.puc-rio.br Subject: Re: [obm-l] Batuta! References: <002201c21a2c$eff095c0$d125ccc8@user> Content-Type: multipart/alternative; boundary="------------90E8CBF07B34B466EF9DD6B3" Sender: owner-obm-l@sucuri.mat.puc-rio.br Precedence: bulk Reply-To: obm-l@mat.puc-rio.br --------------90E8CBF07B34B466EF9DD6B3 Content-Type: text/plain; charset=iso-8859-1 Content-Transfer-Encoding: 8bit Olá amigo Arconcher, desculpe-me por não ter entrado em contato. Estou cansado e com vários problemas que depois conto a você. O problema que o Caio mandou para a lista da obm de construção geométrica, fui eu que pedi ajuda a ele. Caso você tenha alguma ideia, gostaria de recebe-la. Um abraço PONCE Claudio wrote: > Eis um probleminha de geometria batuta. É da prova francesa de seleção > para a olimpíada mundial.Seja ABC um triângulo acutângulo.Sejam A_1 e > B_1 os pés das alturas tiradas de A e B, respectivamente. Seja M o > ponto médio do lado AB. i) Mostre que a reta MA_1 é tangente ao > círculo circunscrito ao triângulo A_1B_1C.ii)Mostre que os círculos > circunscritos aos triângulos A_1B_1C, BMA_1 e AMB_1 têm um ponto em > comum. (Sai todinho somente com considerações sobre > ângulos.) Saludos!Casemiro. --------------90E8CBF07B34B466EF9DD6B3 Content-Type: text/html; charset=us-ascii Content-Transfer-Encoding: 7bit Olá amigo Arconcher,
desculpe-me por não ter entrado em contato.
Estou cansado e com vários problemas que depois conto a você.
O problema que o Caio mandou para a lista da obm de construção geométrica,
fui eu que pedi ajuda a ele. Caso você tenha alguma ideia, gostaria de recebe-la.
Um abraço
PONCE

Claudio wrote:

Eis um probleminha de geometria batuta. É da prova francesa de seleção para a olimpíada mundial.Seja ABC um triângulo acutângulo.Sejam A_1 e B_1 os pés das alturas tiradas de A e B, respectivamente. Seja M o ponto médio do lado AB. i) Mostre que a reta MA_1 é tangente ao círculo circunscrito ao triângulo A_1B_1C.ii)Mostre que os círculos circunscritos aos triângulos A_1B_1C, BMA_1 e AMB_1 têm um ponto em comum. (Sai todinho somente com considerações sobre ângulos.) Saludos!Casemiro.
--------------90E8CBF07B34B466EF9DD6B3-- ========================================================================= Instruções para entrar na lista, sair da lista e usar a lista em http://www.mat.puc-rio.br/~nicolau/olimp/obm-l.html O administrador desta lista é ========================================================================= From owner-obm-l@sucuri.mat.puc-rio.br Sun Jun 23 20:58:31 2002 Return-Path: Received: (from majordom@localhost) by sucuri.mat.puc-rio.br (8.9.3/8.9.3) id UAA09677 for obm-l-list; Sun, 23 Jun 2002 20:55:49 -0300 Received: from smtp.ieg.com.br (12.139.226.200.in-addr.arpa.ig.com.br [200.226.139.12] (may be forged)) by sucuri.mat.puc-rio.br (8.9.3/8.9.3) with ESMTP id UAA09673 for ; Sun, 23 Jun 2002 20:55:47 -0300 Received: from macacos (200-161-154-224.dsl.telesp.net.br [200.161.154.224]) by smtp.ieg.com.br (IeG relay/8.9.3) with SMTP id g5NNuu9R005808 for ; Sun, 23 Jun 2002 20:56:57 -0300 (BRT) Message-ID: <002701c21b11$8c958b80$0200a8c0@macacos> From: =?iso-8859-1?Q?Gabriel_P=E9rgola?= To: References: <001701c21a4c$9690bda0$0200a8c0@macacos> <003101c21ac6$dbeec3e0$970d97c8@windows9> Subject: Re: [obm-l] Trigonometria Date: Sun, 23 Jun 2002 20:56:08 -0300 MIME-Version: 1.0 Content-Type: text/plain; charset="iso-8859-1" Content-Transfer-Encoding: 8bit X-Priority: 3 X-MSMail-Priority: Normal X-Mailer: Microsoft Outlook Express 5.50.4807.1700 X-MimeOLE: Produced By Microsoft MimeOLE V5.50.4807.1700 Sender: owner-obm-l@sucuri.mat.puc-rio.br Precedence: bulk Reply-To: obm-l@mat.puc-rio.br Você deve ter errado em algum passo de suas contas. A resposta é (raiz de 3)/3. Gabriel > > Se a +b = 60º, entao: sen a + sen b / cos a + cos b = ? > > > a+b = Pi/3 rd > > sen a = sen ((a+b)/2 + (a-b)/2) = > sen x cos y + sen y cos x > > onde x = (a+b)/2 e y = (a-b)/2 > > sen b = ((a+b)/2 - (a-b)/2) = > sen x cos y - sen y cos x > > logo sen a + sen b = 2*sen x cos y > > cos a = cos((a+b)/2 + (a-b)/2) = > cos x cos y - sen x sen y > > cos b = cos((a+b)/2 - (a-b)/2) = > cos x cos y + sen x sen y > > logo cos a + cos b = 2*cos x cos y > > (i) suponha y /= Pi/2 + k*Pi > entao cos y /= 0 e tem-se > > (sen a + sen b)/(cos a + cos b) = > = (2*sen x cos y)/(2*cos x cos y) = > = tan x = tan((a+b)/2) = tan ((Pi/3)/2) = > = tan (Pi/6) = raiz(3)/3 > > (ii) se y = Pi/2 + k*Pi então > > (a-b)/2 = Pi/2 + k*Pi > a-b = Pi + 2k*Pi > a = b + Pi + 2k*Pi > > e portanto > > cos a + cos b = > = cos (b + Pi) + cos b = > = -cos b + cos b = 0 > > e não se pode falar em > (sen a + sen b)/(cos a + cos b) > > Um abrac,o! > > Eric. ========================================================================= Instruções para entrar na lista, sair da lista e usar a lista em http://www.mat.puc-rio.br/~nicolau/olimp/obm-l.html O administrador desta lista é ========================================================================= From owner-obm-l@sucuri.mat.puc-rio.br Sun Jun 23 20:58:31 2002 Return-Path: Received: (from majordom@localhost) by sucuri.mat.puc-rio.br (8.9.3/8.9.3) id UAA09688 for obm-l-list; Sun, 23 Jun 2002 20:55:56 -0300 Received: from smtp.ieg.com.br (12.139.226.200.in-addr.arpa.ig.com.br [200.226.139.12] (may be forged)) by sucuri.mat.puc-rio.br (8.9.3/8.9.3) with ESMTP id UAA09684 for ; Sun, 23 Jun 2002 20:55:55 -0300 Received: from macacos (200-161-154-224.dsl.telesp.net.br [200.161.154.224]) by smtp.ieg.com.br (IeG relay/8.9.3) with SMTP id g5NNv99R005825 for ; Sun, 23 Jun 2002 20:57:10 -0300 (BRT) Message-ID: <003001c21b11$942d83c0$0200a8c0@macacos> From: =?iso-8859-1?Q?Gabriel_P=E9rgola?= To: "Obm-l" References: <001701c21a4c$9690bda0$0200a8c0@macacos> <003101c21ac6$dbeec3e0$970d97c8@windows9> Subject: Re: [obm-l] Trigonometria Date: Sun, 23 Jun 2002 20:56:21 -0300 MIME-Version: 1.0 Content-Type: text/plain; charset="iso-8859-1" Content-Transfer-Encoding: 8bit X-Priority: 3 X-MSMail-Priority: Normal X-Mailer: Microsoft Outlook Express 5.50.4807.1700 X-MimeOLE: Produced By Microsoft MimeOLE V5.50.4807.1700 Sender: owner-obm-l@sucuri.mat.puc-rio.br Precedence: bulk Reply-To: obm-l@mat.puc-rio.br Você deve ter errado em algum passo de suas contas. A resposta é (raiz de 3)/3. Gabriel > > Se a +b = 60º, entao: sen a + sen b / cos a + cos b = ? > > > a+b = Pi/3 rd > > sen a = sen ((a+b)/2 + (a-b)/2) = > sen x cos y + sen y cos x > > onde x = (a+b)/2 e y = (a-b)/2 > > sen b = ((a+b)/2 - (a-b)/2) = > sen x cos y - sen y cos x > > logo sen a + sen b = 2*sen x cos y > > cos a = cos((a+b)/2 + (a-b)/2) = > cos x cos y - sen x sen y > > cos b = cos((a+b)/2 - (a-b)/2) = > cos x cos y + sen x sen y > > logo cos a + cos b = 2*cos x cos y > > (i) suponha y /= Pi/2 + k*Pi > entao cos y /= 0 e tem-se > > (sen a + sen b)/(cos a + cos b) = > = (2*sen x cos y)/(2*cos x cos y) = > = tan x = tan((a+b)/2) = tan ((Pi/3)/2) = > = tan (Pi/6) = raiz(3)/3 > > (ii) se y = Pi/2 + k*Pi então > > (a-b)/2 = Pi/2 + k*Pi > a-b = Pi + 2k*Pi > a = b + Pi + 2k*Pi > > e portanto > > cos a + cos b = > = cos (b + Pi) + cos b = > = -cos b + cos b = 0 > > e não se pode falar em > (sen a + sen b)/(cos a + cos b) > > Um abrac,o! > > Eric. ========================================================================= Instruções para entrar na lista, sair da lista e usar a lista em http://www.mat.puc-rio.br/~nicolau/olimp/obm-l.html O administrador desta lista é ========================================================================= From owner-obm-l@sucuri.mat.puc-rio.br Sun Jun 23 21:15:13 2002 Return-Path: Received: (from majordom@localhost) by sucuri.mat.puc-rio.br (8.9.3/8.9.3) id VAA10030 for obm-l-list; Sun, 23 Jun 2002 21:12:34 -0300 Received: from imo-r04.mx.aol.com (imo-r04.mx.aol.com [152.163.225.100]) by sucuri.mat.puc-rio.br (8.9.3/8.9.3) with ESMTP id VAA10022 for ; Sun, 23 Jun 2002 21:12:31 -0300 From: SSayajinGoten@aol.com Received: from SSayajinGoten@aol.com by imo-r04.mx.aol.com (mail_out_v32.21.) id z.156.fcf5483 (16336) for ; Sun, 23 Jun 2002 20:12:01 -0400 (EDT) Message-ID: <156.fcf5483.2a47bdd1@aol.com> Date: Sun, 23 Jun 2002 20:12:01 EDT Subject: [obm-l] Helps! To: obm-l@mat.puc-rio.br MIME-Version: 1.0 Content-Type: text/plain; charset="ISO-8859-1" X-Mailer: AOL 4.0 for Windows sub 108 Content-Transfer-Encoding: 8bit X-MIME-Autoconverted: from quoted-printable to 8bit by sucuri.mat.puc-rio.br id VAA10023 Sender: owner-obm-l@sucuri.mat.puc-rio.br Precedence: bulk Reply-To: obm-l@mat.puc-rio.br Wells, não estou conseguindo fazer uma questão que caio no simulado do meu curso, será que alguem poderia me ajudar? Dois colecionadores de selos têm juntos 500 selos. Cada colecionador comprou um álbum para colocar seus selos. Os dois álbuns eram idênticos, tendo o mesmo número de páginas. Se o primeiro colecionador colocar exatamente 21 selos em cada página, ele vai conseguir colocar todos os selos e usar todas as páginas do álbum. Se o segundo colecionador colocar 20 selos em cada página do álbum ,sobrarão alguns selos, caso ele coloque 23 selos em cada página, sobra pelo menos uma página,totalmente vazia,podendo haver ainda uma outra página com menos de 23 selos. o numero de páginas que há no álbum possui: a)2 divisores naturais b)4 divisores naturais c)6 divisores naturais d)8 divisores naturais e)10 divisores naturais ========================================================================= Instruções para entrar na lista, sair da lista e usar a lista em http://www.mat.puc-rio.br/~nicolau/olimp/obm-l.html O administrador desta lista é ========================================================================= From owner-obm-l@sucuri.mat.puc-rio.br Sun Jun 23 21:41:14 2002 Return-Path: Received: (from majordom@localhost) by sucuri.mat.puc-rio.br (8.9.3/8.9.3) id VAA10864 for obm-l-list; Sun, 23 Jun 2002 21:38:28 -0300 Received: from smtp-4.ig.com.br (smtp-4.ig.com.br [200.226.132.153]) by sucuri.mat.puc-rio.br (8.9.3/8.9.3) with SMTP id VAA10860 for ; Sun, 23 Jun 2002 21:38:27 -0300 Received: (qmail 25898 invoked from network); 24 Jun 2002 00:37:47 -0000 Received: from 83.225.226.200.in-addr.arpa.ig.com.br (HELO oemcomputer) (200.226.225.83) by smtp-4.ig.com.br with SMTP; 24 Jun 2002 00:37:47 -0000 Message-ID: <000501c21b17$f77c2020$53e1e2c8@oemcomputer> From: "Daniel" To: "Lista OBM" Subject: [obm-l] =?iso-8859-1?B?U+1tYm9sb3M=?= Date: Sun, 23 Jun 2002 19:15:52 -0300 MIME-Version: 1.0 Content-Type: multipart/alternative; boundary="----=_NextPart_000_0007_01C21AEA.6407D6E0" X-Priority: 3 X-MSMail-Priority: Normal X-Mailer: Microsoft Outlook Express 5.00.2615.200 X-MIMEOLE: Produced By Microsoft MimeOLE V5.00.2615.200 Sender: owner-obm-l@sucuri.mat.puc-rio.br Precedence: bulk Reply-To: obm-l@mat.puc-rio.br This is a multi-part message in MIME format. ------=_NextPart_000_0007_01C21AEA.6407D6E0 Content-Type: text/plain; charset="iso-8859-1" Content-Transfer-Encoding: quoted-printable Ol=E1 a todos, gostaria de saber se existe algum livro = ou site, onde pudesse encontrar todos os s=EDmbolos usados em = Matem=E1tica, seu significado e sua aplica=E7=E3o. Daniel, S=E3o Jos=E9 dos Campos - SP ------=_NextPart_000_0007_01C21AEA.6407D6E0 Content-Type: text/html; charset="iso-8859-1" Content-Transfer-Encoding: quoted-printable
       =20         Ol=E1 a todos, gostaria de saber = se existe=20 algum livro ou site, onde pudesse encontrar todos os s=EDmbolos usados = em=20 Matem=E1tica, seu significado e sua aplica=E7=E3o.
 
       =20     Daniel,
       =20             =    =20     S=E3o Jos=E9 dos Campos - = SP
------=_NextPart_000_0007_01C21AEA.6407D6E0-- ========================================================================= Instruções para entrar na lista, sair da lista e usar a lista em http://www.mat.puc-rio.br/~nicolau/olimp/obm-l.html O administrador desta lista é ========================================================================= From owner-obm-l@sucuri.mat.puc-rio.br Sun Jun 23 22:59:54 2002 Return-Path: Received: (from majordom@localhost) by sucuri.mat.puc-rio.br (8.9.3/8.9.3) id WAA12139 for obm-l-list; Sun, 23 Jun 2002 22:56:51 -0300 Received: from Euler.impa.br (euler.impa.br [147.65.1.3]) by sucuri.mat.puc-rio.br (8.9.3/8.9.3) with ESMTP id WAA12135 for ; Sun, 23 Jun 2002 22:56:49 -0300 Received: from [147.65.11.11] (dial11.impa.br [147.65.11.11]) by Euler.impa.br (8.11.6/8.11.6) with ESMTP id g5O1uPf18858 for ; Sun, 23 Jun 2002 22:56:25 -0300 (EST) Message-Id: <200206240156.g5O1uPf18858@Euler.impa.br> X-Mailer: Microsoft Outlook Express Macintosh Edition - 4.5 (0410) Date: Tue, 25 Jun 2002 22:54:27 -0300 Subject: Re: [obm-l] Geometria interssante From: "Eduardo Wagner" To: obm-l@mat.puc-rio.br Mime-version: 1.0 X-Priority: 3 Content-type: multipart/alternative; boundary="MS_Mac_OE_3107890467_143835_MIME_Part" Sender: owner-obm-l@sucuri.mat.puc-rio.br Precedence: bulk Reply-To: obm-l@mat.puc-rio.br > THIS MESSAGE IS IN MIME FORMAT. Since your mail reader does not understand this format, some or all of this message may not be legible. --MS_Mac_OE_3107890467_143835_MIME_Part Content-type: text/plain; charset="ISO-8859-1" Content-transfer-encoding: quoted-printable Caio: Seu problema nao tem solucao com regua e compasso. Ele envolve uma curva chamada "conchoide de Nicomedes". Agora, um problema muito interessante e que tem solucao com regua e compasso eh o seguinte. "Determinar a semi-reta de origem P que, ao cortar os lados do angulo dado, forme um triangulo de perimetro d (dado). Abraco, Wagner. ---------- From: "Caio H. Voznak" To: Subject: [obm-l] Geometria interssante Date: Sat, Jun 22, 2002, 3:45 PM Por favor ser=E1 que alguem conhece um solu=E7=E3o para a seguinte quest=E3o: S=E3o dadas duas retas convergentes em um ponto O que formam um angulo agudo teta entre si, tamb=E9m =E9 dado um ponto P localizado abaixo das retas, ambos fixos, e uma medida d. =C9 pedido uma semireta com in=EDcio em P e que corte ambas as retas convergentes obtendo a medida d entre as retas. Segue um esbo=E7o em anexo. Um abra=E7o, Caio Voznak --- Outgoing mail is certified Virus Free. Checked by AVG anti-virus system (http://www.grisoft.com). Version: 6.0.345 / Virus Database: 193 - Release Date: 9/4/2002 --MS_Mac_OE_3107890467_143835_MIME_Part Content-type: text/html; charset="ISO-8859-1" Content-transfer-encoding: quoted-printable Re: [obm-l] Geometria interssante Caio:

Seu problema nao tem solucao com regua e compasso. Ele envolve
uma curva chamada "conchoide de Nicomedes".
Agora, um problema muito interessante e que tem solucao com
regua e compasso eh o seguinte.
"Determinar a semi-reta de origem P que, ao cortar os lados
do angulo dado, forme um triangulo de perimetro d (dado).

Abraco,

Wagner.

----------
From: "Caio H. Voznak" <caio_voznak@yahoo.com.br>
To: <obm-l@mat.puc-rio.br>
Subject: [obm-l] Geometria interssante
Date: Sat, Jun 22, 2002, 3:45 PM


Por favor ser=E1 que alguem con= hece um solu=E7=E3o para a seguinte quest=E3o:
 
S=E3o dadas duas retas convergentes em um p= onto O que formam um angulo agudo teta entre si, tamb=E9m =E9 dado um ponto P lo= calizado abaixo das retas, ambos fixos, e uma medida d. =C9 pedido uma semiret= a com in=EDcio em P e que corte ambas as retas convergentes obtendo a medida d= entre as retas. Segue um esbo=E7o em anexo.   
 
Um abra=E7o,
Caio Voznak
 

---
Outgoing mail is certified Virus Free.
Checked by AVG anti-virus system (http://www.griso= ft.com).
Version: 6.0.345 / Virus Database: 193 - Release Date: 9/4/2002

--MS_Mac_OE_3107890467_143835_MIME_Part-- ========================================================================= Instruções para entrar na lista, sair da lista e usar a lista em http://www.mat.puc-rio.br/~nicolau/olimp/obm-l.html O administrador desta lista é ========================================================================= From owner-obm-l@sucuri.mat.puc-rio.br Sun Jun 23 23:57:04 2002 Return-Path: Received: (from majordom@localhost) by sucuri.mat.puc-rio.br (8.9.3/8.9.3) id XAA12971 for obm-l-list; Sun, 23 Jun 2002 23:55:17 -0300 Received: from web10108.mail.yahoo.com (web10108.mail.yahoo.com [216.136.130.58]) by sucuri.mat.puc-rio.br (8.9.3/8.9.3) with SMTP id XAA12967 for ; Sun, 23 Jun 2002 23:55:14 -0300 Message-ID: <20020624025451.38433.qmail@web10108.mail.yahoo.com> Received: from [200.189.87.112] by web10108.mail.yahoo.com via HTTP; Sun, 23 Jun 2002 19:54:51 PDT Date: Sun, 23 Jun 2002 19:54:51 -0700 (PDT) From: Rafael WC Subject: [obm-l] permutação caótica com repetição To: OBM MIME-Version: 1.0 Content-Type: text/plain; charset=us-ascii Sender: owner-obm-l@sucuri.mat.puc-rio.br Precedence: bulk Reply-To: obm-l@mat.puc-rio.br Alguém pode me ajudar com essa? 17 - Quantos são os anagramas da palavra MATEMÁTICA, em que nenhuma das letras ocupa a posição ocupada inicialmente em cada palavra? (Não considerar o acento na palavra matemática) Se não houvesse repetição das letras seria uma simples permutação caótica, mas nunca tentei fazer com repetição. Se alguém conseguir qualquer coisa... Abraços, Rafael. ===== Rafael Werneck Cinoto ICQ# 107011599 rwcinoto@yahoo.com matduvidas@yahoo.com.br http://www.rwcinoto.hpg.com.br/ __________________________________________________ Do You Yahoo!? Yahoo! - Official partner of 2002 FIFA World Cup http://fifaworldcup.yahoo.com ========================================================================= Instruções para entrar na lista, sair da lista e usar a lista em http://www.mat.puc-rio.br/~nicolau/olimp/obm-l.html O administrador desta lista é ========================================================================= From owner-obm-l@sucuri.mat.puc-rio.br Mon Jun 24 14:02:00 2002 Return-Path: Received: (from majordom@localhost) by sucuri.mat.puc-rio.br (8.9.3/8.9.3) id NAA22112 for obm-l-list; Mon, 24 Jun 2002 13:57:59 -0300 Received: from salvatore.bol.com.br (salvatore.bol.com.br [200.221.24.47]) by sucuri.mat.puc-rio.br (8.9.3/8.9.3) with ESMTP id NAA22108 for ; Mon, 24 Jun 2002 13:57:56 -0300 Received: from bol.com.br (200.221.24.119) by salvatore.bol.com.br (5.1.071) id 3D0E93990018BF19 for obm-l@mat.puc-rio.br; Mon, 24 Jun 2002 13:57:10 -0300 Date: Mon, 24 Jun 2002 14:57:10 -0200 Message-Id: Subject: [obm-l] =?iso-8859-1?q?Re=3A=5Bobm=2Dl=5D_permuta=E7=E3o_ca=F3tica_com_repeti=E7?= =?iso-8859-1?q?=E3o?= MIME-Version: 1.0 Content-Type: multipart/mixed; boundary="_=__=_XaM3_Boundary.1024937830.2A.937278.42.14931.52.42.101010.712862794" From: "Peterdirichlet" To: obm-l@mat.puc-rio.br X-XaM3-API-Version: 2.4.3.4.4 X-SenderIP: 200.206.103.3 Sender: owner-obm-l@sucuri.mat.puc-rio.br Precedence: bulk Reply-To: obm-l@mat.puc-rio.br --_=__=_XaM3_Boundary.1024937830.2A.937278.42.14931.52.42.101010.712862794 Content-Type: text/plain;charset="iso-8859-1" Content-Transfer-Encoding: quoted-printable Cara,essas coisas de repeti=E7ao eu geralmente jogo assim:ponho indices em cada letra repetida(m(a1)tem(a2) tic(a3),no seu caso),trabalhar diferenciado(os indices diferenciam as letras)e depois considere as repeti=E7oes propriamente (des)ditas entre os respectivos indices. Tente ai em casa que eu tento aqui mesmo. Te mais!Dirichlet __________________________________________________________________________ AcessoBOL, s=F3 R$ 9,90! O menor pre=E7o do mercado! Assine j=E1! http://www.bol.com.br/acessobol --_=__=_XaM3_Boundary.1024937830.2A.937278.42.14931.52.42.101010.712862794 Content-Type: text/plain; name="000003BW" Content-Transfer-Encoding: base64 QWxndeltIHBvZGUgbWUgYWp1ZGFyIGNvbSBlc3NhPw0KDQoxNyAtIFF1YW50b3Mgc+NvIG9z IGFuYWdyYW1hcyBkYSBwYWxhdnJhIE1BVEVNwVRJQ0EsDQplbSBxdWUgbmVuaHVtYSBkYXMg bGV0cmFzIG9jdXBhIGEgcG9zaefjbyBvY3VwYWRhDQppbmljaWFsbWVudGUgZW0gY2FkYSBw YWxhdnJhPw0KKE7jbyBjb25zaWRlcmFyIG8gYWNlbnRvIG5hIHBhbGF2cmEgbWF0ZW3hdGlj YSkNCg0KU2UgbuNvIGhvdXZlc3NlIHJlcGV0aefjbyBkYXMgbGV0cmFzIHNlcmlhIHVtYSBz aW1wbGVzDQpwZXJtdXRh5+NvIGNh83RpY2EsIG1hcyBudW5jYSB0ZW50ZWkgZmF6ZXIgY29t DQpyZXBldGnn428uDQoNClNlIGFsZ3XpbSBjb25zZWd1aXIgcXVhbHF1ZXIgY29pc2EuLi4N Cg0KQWJyYedvcywNCg0KUmFmYWVsLg0KDQo9PT09PQ0KUmFmYWVsIFdlcm5lY2sgQ2lub3Rv DQogICAgICAgSUNRIyAxMDcwMTE1OTkNCiAgICAgcndjaW5vdG9AeWFob28uY29tDQogICBt YXRkdXZpZGFzQHlhaG9vLmNvbS5icg0KaHR0cDovL3d3dy5yd2Npbm90by5ocGcuY29tLmJy Lw0KDQpfX19fX19fX19fX19fX19fX19fX19fX19fX19fX19fX19fX19fX19fX19fX19fX19f Xw0KRG8gWW91IFlhaG9vIT8NCllhaG9vISAtIE9mZmljaWFsIHBhcnRuZXIgb2YgMjAwMiBG SUZBIFdvcmxkIEN1cA0KaHR0cDovL2ZpZmF3b3JsZGN1cC55YWhvby5jb20NCj09PT09PT09 PT09PT09PT09PT09PT09PT09PT09PT09PT09PT09PT09PT09PT09PT09PT09PT09PT09PT09 PT09PT09PT09PT0NCkluc3Rydef1ZXMgcGFyYSBlbnRyYXIgbmEgbGlzdGEsIHNhaXIgZGEg bGlzdGEgZSB1c2FyIGEgbGlzdGEgZW0NCmh0dHA6Ly93d3cubWF0LnB1Yy1yaW8uYnIvfm5p Y29sYXUvb2xpbXAvb2JtLWwuaHRtbA0KTyBhZG1pbmlzdHJhZG9yIGRlc3RhIGxpc3RhIOkg PG5pY29sYXVAbWF0LnB1Yy1yaW8uYnI+DQo9PT09PT09PT09PT09PT09PT09PT09PT09PT09 PT09PT09PT09PT09PT09PT09PT09PT09PT09PT09PT09PT09PT09PT09PT09DQo= --_=__=_XaM3_Boundary.1024937830.2A.937278.42.14931.52.42.101010.712862794-- ========================================================================= Instruções para entrar na lista, sair da lista e usar a lista em http://www.mat.puc-rio.br/~nicolau/olimp/obm-l.html O administrador desta lista é ========================================================================= From owner-obm-l@sucuri.mat.puc-rio.br Mon Jun 24 14:33:18 2002 Return-Path: Received: (from majordom@localhost) by sucuri.mat.puc-rio.br (8.9.3/8.9.3) id OAA22745 for obm-l-list; Mon, 24 Jun 2002 14:33:00 -0300 Received: from gorgo.centroin.com.br (gorgo.centroin.com.br [200.225.63.128]) by sucuri.mat.puc-rio.br (8.9.3/8.9.3) with ESMTP id OAA22735 for ; Mon, 24 Jun 2002 14:32:55 -0300 Received: from centroin.com.br (du140c.rjo.centroin.com.br [200.225.58.140]) (authenticated bits=0) by gorgo.centroin.com.br (8.12.2/8.12.1) with ESMTP id g5OHWxwk024594 for ; Mon, 24 Jun 2002 14:32:59 -0300 (BRT) Message-ID: <3D17581F.9040303@centroin.com.br> Date: Mon, 24 Jun 2002 14:34:23 -0300 From: Augusto =?ISO-8859-1?Q?C=E9sar?= Morgado User-Agent: Mozilla/5.0 (Windows; U; Win98; en-US; rv:0.9.4.1) Gecko/20020508 Netscape6/6.2.3 X-Accept-Language: en-us MIME-Version: 1.0 To: obm-l@mat.puc-rio.br Subject: Re: [obm-l] combinatoria References: Content-Type: multipart/alternative; boundary="------------070806080500050806020200" Sender: owner-obm-l@sucuri.mat.puc-rio.br Precedence: bulk Reply-To: obm-l@mat.puc-rio.br --------------070806080500050806020200 Content-Type: text/plain; charset=ISO-8859-1; format=flowed Content-Transfer-Encoding: 8bit Perceba inicialmente que nenhum aluno pode ir a mais de tres jantares. Se o aluno 1 vai a um jantar com 2 e 3, a outro com 4 e 5 e a um outro com 6 e 7, ele não tem com quem ir a um quarto jantar. Como o professor faz 21 convites (3 por dia durante sete dias), e nenhum aluno pode receber mais de 3 convites, cada aluno recebe exatamente 3 convites.Perceba, então, que cada aluno encontrará cada um dos outros alunos em 1 jantar. Vamos montar uma tabela (dias a esquerda e convidados a direita): como o aluno 1 vai a 3 jantares e em um deles tem a companhia do 2, 1) 1, 2 2) 1 3) 1 4) 5) 6) 7) O terceiro do jantar numero 1 pode ser escolhido de 5 modos. Digamos que tenha sido o 3. O aluno 4 vai a um jantar com o 1. 1) 1, 2,3 2) 1,4 3) 1 4) 5) 6) 7) O terceiro do jantar numero 2 pode ser escolhido de 3 modos. Digamos que tenha sido o 5. Por exclusão, o jantar numero 3 está determinado. 1) 1, 2,3 2) 1,4,5 3) 1,6,7 4) 5) 6) 7) O aluno dois vai a mais dois jantares, um dos quais em companhia do 4. Digamos 1) 1, 2,3 2) 1,4,5 3) 1,6,7 4) 2,4 5) 2 6) 7) O terceiro do jantar 4 pode serescolhido de 2 modos (6 ou 7). Digamos 1) 1, 2,3 2) 1,4,5 3) 1,6,7 4) 2,4, 6 5) 2 6) 7) Agora o final eh forçado: 1) 1, 2,3 2) 1,4,5 3) 1,6,7 4) 2,4,6 5) 2,5,7 6) 3 7) 3 1) 1, 2,3 2) 1,4,5 3) 1,6,7 4) 2,4,6 5) 2,5,7 6) 3,4 7) 3, 6 1) 1, 2,3 2) 1,4,5 3) 1,6,7 4) 2,4,6 5) 2,5,7 6) 3,4,7 7)3,6,5 Uma resposta possivel eh 6x5x2=30 ( se so ligamos para os grupos de convidados e não para os dias da semana) A outra resposta possivel eh 30x7! adr.scr.m wrote: >estou com duvida neste problema e preciso de >ajuda >Depois de ter dado um curso,um professor >resolve se derpedir de seus 7 alunos >oferecendo,durante 7 dias consecutivos,7 >jantares para cada 3 alunos cada.De quantos >modos ele pode fazer osconvites se ele nao >deseja que um mesmo par de alunos compareca >a mais de um jantar ? > > para mim o nºde convites seria o nº de >pares distintos ,mas da errado. > > > >__________________________________________________________________________ >Quer ter seu próprio endereço na Internet? >Garanta já o seu e ainda ganhe cinco e-mails personalizados. >DomíniosBOL - http://dominios.bol.com.br > > >========================================================================= >Instruções para entrar na lista, sair da lista e usar a lista em >http://www.mat.puc-rio.br/~nicolau/olimp/obm-l.html >O administrador desta lista é >========================================================================= > > --------------070806080500050806020200 Content-Type: text/html; charset=us-ascii Content-Transfer-Encoding: 7bit Perceba inicialmente que nenhum aluno pode ir a mais de tres jantares. Se o aluno 1 vai a um jantar com 2 e 3, a outro com 4 e 5 e a um outro com 6 e 7, ele não tem com quem ir a um quarto jantar.
Como o professor faz 21 convites (3 por dia durante sete dias), e nenhum aluno pode receber mais de 3 convites, cada aluno recebe exatamente 3 convites.Perceba, então, que cada aluno encontrará cada um dos outros alunos em 1 jantar.
Vamos montar uma tabela (dias a esquerda e convidados a direita): como o aluno 1 vai a 3 jantares e em um  deles tem a companhia do 2,
1) 1, 2           2) 1        3) 1          4)            5)                  6)           7)
O terceiro do jantar numero 1 pode ser escolhido de 5 modos. Digamos que tenha sido o 3. O aluno 4 vai a um jantar com o 1.
1) 1, 2,3           2) 1,4        3) 1          4)            5)                  6)           7)
O terceiro do jantar numero 2  pode ser escolhido de 3 modos. Digamos que tenha sido o 5. Por exclusão, o jantar numero 3 está determinado.
1) 1, 2,3           2) 1,4,5        3) 1,6,7          4)            5)                  6)           7)
O aluno dois vai a mais dois jantares, um dos quais em companhia do 4. Digamos
1) 1, 2,3           2) 1,4,5        3) 1,6,7          4) 2,4            5) 2                  6)           7)
O terceiro do jantar 4 pode serescolhido de 2 modos (6 ou 7). Digamos
1) 1, 2,3           2) 1,4,5        3) 1,6,7          4) 2,4, 6            5) 2                  6)           7)
Agora o final eh forçado:
1) 1, 2,3           2) 1,4,5        3) 1,6,7          4) 2,4,6            5) 2,5,7          6) 3            7) 3
1) 1, 2,3           2) 1,4,5        3) 1,6,7          4) 2,4,6            5) 2,5,7          6)  3,4          7) 3, 6
1) 1, 2,3           2) 1,4,5        3) 1,6,7          4) 2,4,6            5) 2,5,7          6) 3,4,7           7)3,6,5
Uma resposta possivel eh 6x5x2=30 ( se so ligamos para os grupos de convidados e não para os dias da semana)
A outra resposta possivel eh  30x7!







 

adr.scr.m wrote:
estou com duvida neste problema e preciso de 
ajuda
Depois de ter dado um curso,um professor
resolve se derpedir de seus 7 alunos
oferecendo,durante 7 dias consecutivos,7
jantares para cada 3 alunos cada.De quantos
modos ele pode fazer osconvites se ele nao
deseja que um mesmo par de alunos compareca
a mais de um jantar ?

para mim o nºde convites seria o nº de
pares distintos ,mas da errado.



__________________________________________________________________________
Quer ter seu próprio endereço na Internet?
Garanta já o seu e ainda ganhe cinco e-mails personalizados.
DomíniosBOL - http://dominios.bol.com.br


=========================================================================
Instruções para entrar na lista, sair da lista e usar a lista em
http://www.mat.puc-rio.br/~nicolau/olimp/obm-l.html
O administrador desta lista é <nicolau@mat.puc-rio.br>
=========================================================================



--------------070806080500050806020200-- ========================================================================= Instruções para entrar na lista, sair da lista e usar a lista em http://www.mat.puc-rio.br/~nicolau/olimp/obm-l.html O administrador desta lista é ========================================================================= From owner-obm-l@sucuri.mat.puc-rio.br Mon Jun 24 19:22:02 2002 Return-Path: Received: (from majordom@localhost) by sucuri.mat.puc-rio.br (8.9.3/8.9.3) id TAA28271 for obm-l-list; Mon, 24 Jun 2002 19:20:32 -0300 Received: from candeias.terra.com.br (candeias.terra.com.br [200.176.3.18]) by sucuri.mat.puc-rio.br (8.9.3/8.9.3) with ESMTP id TAA28267 for ; Mon, 24 Jun 2002 19:20:30 -0300 Received: from srv11-sao.terra.com.br (srv11-sao.terra.com.br [200.176.3.38]) by candeias.terra.com.br (Postfix) with ESMTP id AA88A4410A for ; Mon, 24 Jun 2002 19:20:10 -0300 (EST) Received: from emilio (200-158-164-220.dsl.telesp.net.br [200.158.164.220]) (authenticated user rodrigozerati) by srv11-sao.terra.com.br (Postfix) with ESMTP id 71575C94CB for ; Mon, 24 Jun 2002 19:20:09 -0300 (EST) Message-ID: <002201c21bcd$34e58d60$4f31fea9@emilio> From: "Rodrigo Zerati" To: References: <3D17581F.9040303@centroin.com.br> Subject: Re: [obm-l] combinatoria Date: Mon, 24 Jun 2002 19:19:27 -0300 MIME-Version: 1.0 Content-Type: multipart/alternative; boundary="----=_NextPart_000_001F_01C21BB4.0EBFA8A0" X-Priority: 3 X-MSMail-Priority: Normal X-Mailer: Microsoft Outlook Express 5.00.2615.200 X-MimeOLE: Produced By Microsoft MimeOLE V5.00.2615.200 Sender: owner-obm-l@sucuri.mat.puc-rio.br Precedence: bulk Reply-To: obm-l@mat.puc-rio.br This is a multi-part message in MIME format. ------=_NextPart_000_001F_01C21BB4.0EBFA8A0 Content-Type: text/plain; charset="iso-8859-1" Content-Transfer-Encoding: quoted-printable Apenas uma pequena corre=E7=E3o neste problema, =E9 que no final, a = conta esta 6x5x2=3D30, mas na verdade =E9 5x3x2=3D30 (1-5 = modos;2-3modos;3-2 modos) Abra=E7o=20 Rodrigo Zerati=20 ----- Original Message -----=20 From: Augusto C=E9sar Morgado=20 To: obm-l@mat.puc-rio.br=20 Sent: Monday, June 24, 2002 2:34 PM Subject: Re: [obm-l] combinatoria Perceba inicialmente que nenhum aluno pode ir a mais de tres jantares. = Se o aluno 1 vai a um jantar com 2 e 3, a outro com 4 e 5 e a um outro = com 6 e 7, ele n=E3o tem com quem ir a um quarto jantar.=20 Como o professor faz 21 convites (3 por dia durante sete dias), e = nenhum aluno pode receber mais de 3 convites, cada aluno recebe = exatamente 3 convites.Perceba, ent=E3o, que cada aluno encontrar=E1 cada = um dos outros alunos em 1 jantar. Vamos montar uma tabela (dias a esquerda e convidados a direita): como = o aluno 1 vai a 3 jantares e em um deles tem a companhia do 2,=20 1) 1, 2 2) 1 3) 1 4) 5) = 6) 7) O terceiro do jantar numero 1 pode ser escolhido de 5 modos. Digamos = que tenha sido o 3. O aluno 4 vai a um jantar com o 1. 1) 1, 2,3 2) 1,4 3) 1 4) 5) = 6) 7) O terceiro do jantar numero 2 pode ser escolhido de 3 modos. Digamos = que tenha sido o 5. Por exclus=E3o, o jantar numero 3 est=E1 = determinado. 1) 1, 2,3 2) 1,4,5 3) 1,6,7 4) 5) = 6) 7) O aluno dois vai a mais dois jantares, um dos quais em companhia do 4. = Digamos 1) 1, 2,3 2) 1,4,5 3) 1,6,7 4) 2,4 = 5) 2 6) 7) O terceiro do jantar 4 pode serescolhido de 2 modos (6 ou 7). Digamos 1) 1, 2,3 2) 1,4,5 3) 1,6,7 4) 2,4, 6 = 5) 2 6) 7) Agora o final eh for=E7ado: 1) 1, 2,3 2) 1,4,5 3) 1,6,7 4) 2,4,6 = 5) 2,5,7 6) 3 7) 3 1) 1, 2,3 2) 1,4,5 3) 1,6,7 4) 2,4,6 = 5) 2,5,7 6) 3,4 7) 3, 6 1) 1, 2,3 2) 1,4,5 3) 1,6,7 4) 2,4,6 = 5) 2,5,7 6) 3,4,7 7)3,6,5 Uma resposta possivel eh 6x5x2=3D30 ( se so ligamos para os grupos de = convidados e n=E3o para os dias da semana) A outra resposta possivel eh 30x7! =20 adr.scr.m wrote: estou com duvida neste problema e preciso de ajudaDepois de ter dado um = curso,um professor resolve se derpedir de seus 7 alunos = oferecendo,durante 7 dias consecutivos,7 jantares para cada 3 alunos = cada.De quantos modos ele pode fazer osconvites se ele nao deseja que um = mesmo par de alunos compareca a mais de um jantar ? para mim o n=BAde = convites seria o n=BA de pares distintos ,mas da errado. = _________________________________________________________________________= _Quer ter seu pr=F3prio endere=E7o na Internet?Garanta j=E1 o seu e = ainda ganhe cinco e-mails personalizados.Dom=EDniosBOL - = http://dominios.bol.com.br=3D=3D=3D=3D=3D=3D=3D=3D=3D=3D=3D=3D=3D=3D=3D=3D= =3D=3D=3D=3D=3D=3D=3D=3D=3D=3D=3D=3D=3D=3D=3D=3D=3D=3D=3D=3D=3D=3D=3D=3D=3D= =3D=3D=3D=3D=3D=3D=3D=3D=3D=3D=3D=3D=3D=3D=3D=3D=3D=3D=3D=3D=3D=3D=3D=3D=3D= =3D=3D=3D=3D=3D=3D=3DInstru=E7=F5es para entrar na lista, sair da lista = e usar a lista emhttp://www.mat.puc-rio.br/~nicolau/olimp/obm-l.htmlO = administrador desta lista =E9 = =3D=3D=3D=3D=3D=3D=3D=3D=3D=3D=3D=3D=3D=3D=3D=3D=3D= =3D=3D=3D=3D=3D=3D=3D=3D=3D=3D=3D=3D=3D=3D=3D=3D=3D=3D=3D=3D=3D=3D=3D=3D=3D= =3D=3D=3D=3D=3D=3D=3D=3D=3D=3D=3D=3D=3D=3D=3D=3D=3D=3D=3D=3D=3D=3D=3D=3D=3D= =3D=3D=3D=3D=3D=3D ------=_NextPart_000_001F_01C21BB4.0EBFA8A0 Content-Type: text/html; charset="iso-8859-1" Content-Transfer-Encoding: quoted-printable
Apenas uma pequena corre=E7=E3o neste = problema, =E9 que=20 no final, a conta esta 6x5x2=3D30, mas na verdade =E9  5x3x2=3D30 = (1-5=20 modos;2-3modos;3-2 modos)
Abra=E7o
Rodrigo Zerati 
----- Original Message -----
From:=20 Augusto=20 C=E9sar Morgado
Sent: Monday, June 24, 2002 = 2:34 PM
Subject: Re: [obm-l] = combinatoria

Perceba inicialmente que nenhum aluno pode ir a mais de = tres=20 jantares. Se o aluno 1 vai a um jantar com 2 e 3, a outro com 4 e 5 e = a um=20 outro com 6 e 7, ele n=E3o tem com quem ir a um quarto jantar. =
Como o=20 professor faz 21 convites (3 por dia durante sete dias), e nenhum = aluno pode=20 receber mais de 3 convites, cada aluno recebe exatamente 3 = convites.Perceba,=20 ent=E3o, que cada aluno encontrar=E1 cada um dos outros alunos em 1=20 jantar.
Vamos montar uma tabela (dias a esquerda e convidados a = direita):=20 como o aluno 1 vai a 3 jantares e em um  deles tem a companhia do = 2,=20
1) 1, 2           2) 1     =  =20  3) 1          4)       =  =20    5)               =  =20  6)           7)
O terceiro do jantar = numero=20 1 pode ser escolhido de 5 modos. Digamos que tenha sido o 3. O = aluno 4=20 vai a um jantar com o 1.
1) 1, 2,3         =   2)=20 1,4        3) 1         =  4)=20            5)       =    =20        6)           = 7)
O=20 terceiro do jantar numero 2  pode ser escolhido de 3 = modos.=20 Digamos que tenha sido o 5. Por exclus=E3o, o jantar numero 3 est=E1=20 determinado.
1) 1, 2,3           2) 1,4,5 =  =20      3) 1,6,7          4) =  =20          5)         =    =20      6)           7)
O = aluno dois=20 vai a mais dois jantares, um dos quais em companhia do 4. = Digamos
1) 1, 2,3=20           2) 1,4,5       =  3)=20 1,6,7          4) 2,4       =  =20    5) 2               =  =20  6)           7)
O terceiro do jantar = 4 pode=20 serescolhido de 2 modos (6 ou 7). Digamos
1) 1, 2,3   =  =20       2) 1,4,5        3) 1,6,7 =  =20        4) 2,4, 6         =  =20  5) 2                 =  6)=20           7)
Agora o final eh = for=E7ado:
1) 1,=20 2,3           2) 1,4,5       =  3)=20 1,6,7          4) 2,4,6       =  =20    5) 2,5,7          6) = 3   =20         7) 3
1) 1, 2,3       =  =20   2) 1,4,5        3) 1,6,7     =  =20    4) 2,4,6            5) = 2,5,7  =20        6)  3,4      =    =20 7) 3, 6
1) 1, 2,3           2) 1,4,5 =    =20    3) 1,6,7          4) 2,4,6 =    =20        5) 2,5,7         =  6)=20 3,4,7           7)3,6,5
Uma resposta = possivel eh=20 6x5x2=3D30 ( se so ligamos para os grupos de convidados e n=E3o para = os dias da=20 semana)
A outra resposta possivel eh=20  30x7!







 

adr.scr.m = wrote:
estou com duvida neste problema e preciso =
de 
ajuda
Depois de ter dado um curso,um professor
resolve se = derpedir de seus 7 alunos
oferecendo,durante 7 dias consecutivos,7 =
jantares para cada 3 alunos cada.De quantos
modos ele pode fazer = osconvites se ele nao
deseja que um mesmo par de alunos compareca =
a mais de um jantar ?

para mim o n=BAde convites seria o = n=BA de
pares distintos ,mas da errado.


=
_____________________________________________________________________= _____
Quer ter seu pr=F3prio endere=E7o na Internet?
Garanta j=E1 = o seu e ainda ganhe cinco e-mails personalizados.
Dom=EDniosBOL - http://dominios.bol.com.br

=3D=3D=3D=3D=3D=3D=3D=3D=3D=3D=3D=3D=3D=3D=3D=3D=3D=3D=3D=3D=3D=3D=3D= =3D=3D=3D=3D=3D=3D=3D=3D=3D=3D=3D=3D=3D=3D=3D=3D=3D=3D=3D=3D=3D=3D=3D=3D=3D= =3D=3D=3D=3D=3D=3D=3D=3D=3D=3D=3D=3D=3D=3D=3D=3D=3D=3D=3D=3D=3D=3D=3D=3D=3D=
Instru=E7=F5es para entrar na lista, sair da lista e usar a lista = em
http://www.m= at.puc-rio.br/~nicolau/olimp/obm-l.html
O administrador desta = lista =E9 <nicolau@mat.puc-rio.br>=
=3D=3D=3D=3D=3D=3D=3D=3D=3D=3D=3D=3D=3D=3D=3D=3D=3D=3D=3D=3D=3D=3D=3D= =3D=3D=3D=3D=3D=3D=3D=3D=3D=3D=3D=3D=3D=3D=3D=3D=3D=3D=3D=3D=3D=3D=3D=3D=3D= =3D=3D=3D=3D=3D=3D=3D=3D=3D=3D=3D=3D=3D=3D=3D=3D=3D=3D=3D=3D=3D=3D=3D=3D=3D=



------=_NextPart_000_001F_01C21BB4.0EBFA8A0-- ========================================================================= Instruções para entrar na lista, sair da lista e usar a lista em http://www.mat.puc-rio.br/~nicolau/olimp/obm-l.html O administrador desta lista é ========================================================================= From owner-obm-l@sucuri.mat.puc-rio.br Mon Jun 24 20:56:07 2002 Return-Path: Received: (from majordom@localhost) by sucuri.mat.puc-rio.br (8.9.3/8.9.3) id UAA29794 for obm-l-list; Mon, 24 Jun 2002 20:55:55 -0300 Received: from gorgo.centroin.com.br (gorgo.centroin.com.br [200.225.63.128]) by sucuri.mat.puc-rio.br (8.9.3/8.9.3) with ESMTP id UAA29790 for ; Mon, 24 Jun 2002 20:55:53 -0300 Received: from centroin.com.br (du1b.nit.centroin.com.br [200.225.56.1]) (authenticated bits=0) by gorgo.centroin.com.br (8.12.2/8.12.1) with ESMTP id g5ONu1L7007817 for ; Mon, 24 Jun 2002 20:56:01 -0300 (BRT) Message-ID: <3D17B1E3.70503@centroin.com.br> Date: Mon, 24 Jun 2002 20:57:23 -0300 From: Augusto =?ISO-8859-1?Q?C=E9sar?= Morgado User-Agent: Mozilla/5.0 (Windows; U; Win98; en-US; rv:0.9.4.1) Gecko/20020508 Netscape6/6.2.3 X-Accept-Language: en-us MIME-Version: 1.0 To: obm-l@mat.puc-rio.br Subject: Re: [obm-l] combinatoria References: <3D17581F.9040303@centroin.com.br> <002201c21bcd$34e58d60$4f31fea9@emilio> Content-Type: multipart/alternative; boundary="------------000504000803090309070006" Sender: owner-obm-l@sucuri.mat.puc-rio.br Precedence: bulk Reply-To: obm-l@mat.puc-rio.br --------------000504000803090309070006 Content-Type: text/plain; charset=ISO-8859-1; format=flowed Content-Transfer-Encoding: 8bit Obrigado Rodrigo Zerati wrote: > Apenas uma pequena correção neste problema, é que no final, a conta > esta 6x5x2=30, mas na verdade é 5x3x2=30 (1-5 modos;2-3modos;3-2 modos) > > Abraço > > Rodrigo Zerati > > ----- Original Message ----- > > From: Augusto César Morgado > > To: obm-l@mat.puc-rio.br > > Sent: Monday, June 24, 2002 2:34 PM > > Subject: Re: [obm-l] combinatoria > > > Perceba inicialmente que nenhum aluno pode ir a mais de tres > jantares. Se o aluno 1 vai a um jantar com 2 e 3, a outro com 4 e > 5 e a um outro com 6 e 7, ele não tem com quem ir a um quarto jantar. > Como o professor faz 21 convites (3 por dia durante sete dias), e > nenhum aluno pode receber mais de 3 convites, cada aluno recebe > exatamente 3 convites.Perceba, então, que cada aluno encontrará > cada um dos outros alunos em 1 jantar. > Vamos montar uma tabela (dias a esquerda e convidados a direita): > como o aluno 1 vai a 3 jantares e em um deles tem a companhia do 2, > 1) 1, 2 2) 1 3) 1 4) 5) > 6) 7) > O terceiro do jantar numero 1 pode ser escolhido de 5 modos. > Digamos que tenha sido o 3. O aluno 4 vai a um jantar com o 1. > 1) 1, 2,3 2) 1,4 3) 1 4) 5) > 6) 7) > O terceiro do jantar numero 2 pode ser escolhido de 3 modos. > Digamos que tenha sido o 5. Por exclusão, o jantar numero 3 está > determinado. > 1) 1, 2,3 2) 1,4,5 3) 1,6,7 4) > 5) 6) 7) > O aluno dois vai a mais dois jantares, um dos quais em companhia > do 4. Digamos > 1) 1, 2,3 2) 1,4,5 3) 1,6,7 4) 2,4 > 5) 2 6) 7) > O terceiro do jantar 4 pode serescolhido de 2 modos (6 ou 7). Digamos > 1) 1, 2,3 2) 1,4,5 3) 1,6,7 4) 2,4, 6 > 5) 2 6) 7) > Agora o final eh forçado: > 1) 1, 2,3 2) 1,4,5 3) 1,6,7 4) 2,4,6 > 5) 2,5,7 6) 3 7) 3 > 1) 1, 2,3 2) 1,4,5 3) 1,6,7 4) 2,4,6 > 5) 2,5,7 6) 3,4 7) 3, 6 > 1) 1, 2,3 2) 1,4,5 3) 1,6,7 4) 2,4,6 > 5) 2,5,7 6) 3,4,7 7)3,6,5 > Uma resposta possivel eh 6x5x2=30 ( se so ligamos para os grupos > de convidados e não para os dias da semana) > A outra resposta possivel eh 30x7! > > > > > > > > > > adr.scr.m wrote: > >>estou com duvida neste problema e preciso de >>ajuda >>Depois de ter dado um curso,um professor >>resolve se derpedir de seus 7 alunos >>oferecendo,durante 7 dias consecutivos,7 >>jantares para cada 3 alunos cada.De quantos >>modos ele pode fazer osconvites se ele nao >>deseja que um mesmo par de alunos compareca >>a mais de um jantar ? >> >> para mim o nºde convites seria o nº de >>pares distintos ,mas da errado. >> >> >> >>__________________________________________________________________________ >>Quer ter seu próprio endereço na Internet? >>Garanta já o seu e ainda ganhe cinco e-mails personalizados. >>DomíniosBOL - http://dominios.bol.com.br >> >> >>========================================================================= >>Instruções para entrar na lista, sair da lista e usar a lista em >>http://www.mat.puc-rio.br/~nicolau/olimp/obm-l.html >>O administrador desta lista é >>========================================================================= >> >> > --------------000504000803090309070006 Content-Type: text/html; charset=us-ascii Content-Transfer-Encoding: 7bit Obrigado

Rodrigo Zerati wrote:
Apenas uma pequena correção neste problema, é que no final, a conta esta 6x5x2=30, mas na verdade é  5x3x2=30 (1-5 modos;2-3modos;3-2 modos)
Abraço
Rodrigo Zerati 
----- Original Message -----
Sent: Monday, June 24, 2002 2:34 PM
Subject: Re: [obm-l] combinatoria

Perceba inicialmente que nenhum aluno pode ir a mais de tres jantares. Se o aluno 1 vai a um jantar com 2 e 3, a outro com 4 e 5 e a um outro com 6 e 7, ele não tem com quem ir a um quarto jantar.
Como o professor faz 21 convites (3 por dia durante sete dias), e nenhum aluno pode receber mais de 3 convites, cada aluno recebe exatamente 3 convites.Perceba, então, que cada aluno encontrará cada um dos outros alunos em 1 jantar.
Vamos montar uma tabela (dias a esquerda e convidados a direita): como o aluno 1 vai a 3 jantares e em um  deles tem a companhia do 2,
1) 1, 2           2) 1        3) 1          4)            5)                  6)           7)
O terceiro do jantar numero 1 pode ser escolhido de 5 modos. Digamos que tenha sido o 3. O aluno 4 vai a um jantar com o 1.
1) 1, 2,3           2) 1,4        3) 1          4)            5)                  6)           7)
O terceiro do jantar numero 2  pode ser escolhido de 3 modos. Digamos que tenha sido o 5. Por exclusão, o jantar numero 3 está determinado.
1) 1, 2,3           2) 1,4,5        3) 1,6,7          4)            5)                  6)           7)
O aluno dois vai a mais dois jantares, um dos quais em companhia do 4. Digamos
1) 1, 2,3           2) 1,4,5        3) 1,6,7          4) 2,4            5) 2                  6)           7)
O terceiro do jantar 4 pode serescolhido de 2 modos (6 ou 7). Digamos
1) 1, 2,3           2) 1,4,5        3) 1,6,7          4) 2,4, 6            5) 2                  6)           7)
Agora o final eh forçado:
1) 1, 2,3           2) 1,4,5        3) 1,6,7          4) 2,4,6            5) 2,5,7          6) 3            7) 3
1) 1, 2,3           2) 1,4,5        3) 1,6,7          4) 2,4,6            5) 2,5,7          6)  3,4          7) 3, 6
1) 1, 2,3           2) 1,4,5        3) 1,6,7          4) 2,4,6            5) 2,5,7          6) 3,4,7           7)3,6,5
Uma resposta possivel eh 6x5x2=30 ( se so ligamos para os grupos de convidados e não para os dias da semana)
A outra resposta possivel eh  30x7!







 

adr.scr.m wrote:
estou com duvida neste problema e preciso de 
ajuda
Depois de ter dado um curso,um professor
resolve se derpedir de seus 7 alunos
oferecendo,durante 7 dias consecutivos,7
jantares para cada 3 alunos cada.De quantos
modos ele pode fazer osconvites se ele nao
deseja que um mesmo par de alunos compareca
a mais de um jantar ?

para mim o nºde convites seria o nº de
pares distintos ,mas da errado.



__________________________________________________________________________
Quer ter seu próprio endereço na Internet?
Garanta já o seu e ainda ganhe cinco e-mails personalizados.
DomíniosBOL - http://dominios.bol.com.br


=========================================================================
Instruções para entrar na lista, sair da lista e usar a lista em
http://www.mat.puc-rio.br/~nicolau/olimp/obm-l.html
O administrador desta lista é <nicolau@mat.puc-rio.br>
=========================================================================




--------------000504000803090309070006-- ========================================================================= Instruções para entrar na lista, sair da lista e usar a lista em http://www.mat.puc-rio.br/~nicolau/olimp/obm-l.html O administrador desta lista é ========================================================================= From owner-obm-l@sucuri.mat.puc-rio.br Tue Jun 25 12:55:13 2002 Return-Path: Received: (from majordom@localhost) by sucuri.mat.puc-rio.br (8.9.3/8.9.3) id MAA06585 for obm-l-list; Tue, 25 Jun 2002 12:51:43 -0300 Received: from salvatore.bol.com.br (salvatore.bol.com.br [200.221.24.47]) by sucuri.mat.puc-rio.br (8.9.3/8.9.3) with ESMTP id MAA06581 for ; Tue, 25 Jun 2002 12:51:41 -0300 Received: from bol.com.br (200.221.24.119) by salvatore.bol.com.br (5.1.071) id 3D0E9399001D19A2 for obm-l@mat.puc-rio.br; Tue, 25 Jun 2002 12:50:58 -0300 Date: Tue, 25 Jun 2002 13:50:58 -0200 Message-Id: Subject: [obm-l] Desigualdades MIME-Version: 1.0 Content-Type: text/plain;charset="iso-8859-1" From: "Peterdirichlet" To: obm-l@mat.puc-rio.br X-XaM3-API-Version: 2.4.3.4.4 X-SenderIP: 200.206.103.3 Content-Transfer-Encoding: 8bit X-MIME-Autoconverted: from quoted-printable to 8bit by sucuri.mat.puc-rio.br id MAA06582 Sender: owner-obm-l@sucuri.mat.puc-rio.br Precedence: bulk Reply-To: obm-l@mat.puc-rio.br Turma,alguem conhece a demonstraçao da desigualdade de Schur e as desigualdades de Bernoulli,Hölder,Young e Erdös-Mordell? Ass.:Dirichlet __________________________________________________________________________ AcessoBOL, só R$ 9,90! O menor preço do mercado! Assine já! http://www.bol.com.br/acessobol ========================================================================= Instruções para entrar na lista, sair da lista e usar a lista em http://www.mat.puc-rio.br/~nicolau/olimp/obm-l.html O administrador desta lista é ========================================================================= From owner-obm-l@sucuri.mat.puc-rio.br Tue Jun 25 13:42:47 2002 Return-Path: Received: (from majordom@localhost) by sucuri.mat.puc-rio.br (8.9.3/8.9.3) id NAA07626 for obm-l-list; Tue, 25 Jun 2002 13:42:40 -0300 Received: from www.zipmail.com.br (smtp.zipmail.com.br [200.187.242.10]) by sucuri.mat.puc-rio.br (8.9.3/8.9.3) with ESMTP id NAA07617 for ; Tue, 25 Jun 2002 13:42:34 -0300 From: luizhenriquerick@zipmail.com.br Received: from [200.165.185.31] by www.zipmail.com.br with HTTP; Tue, 25 Jun 2002 13:42:08 -0300 Message-ID: <3D188FA000000087@www.zipmail.com.br> Date: Tue, 25 Jun 2002 13:42:08 -0300 Subject: [obm-l] =?iso-8859-1?Q?FATORA=C7=C3O?= To: obm-l@mat.puc-rio.br MIME-Version: 1.0 Content-Type: text/plain; charset="iso-8859-1" Content-Transfer-Encoding: 8bit X-MIME-Autoconverted: from quoted-printable to 8bit by sucuri.mat.puc-rio.br id NAA07623 Sender: owner-obm-l@sucuri.mat.puc-rio.br Precedence: bulk Reply-To: obm-l@mat.puc-rio.br Olá amigos , tive alguns problemas para fatorar esses exercícios abaixo ,principalmente a número 3 que é bem estranha , se puderem me dar uma ajudinha ; 1- Se P = ( 3^2^0 + 1 )( 3^2^1 + 1 )( 3^2^2 + 1 ) . . . ( 3^2^n + 1 ) então P é igual a: 2- A expressão [68 + 48 . (2)^1/2]^1/4 - [ 25 + 22 . (2)^1/2]^1/3 vale: 3- (bc - a²)^-1 + (ca - b²)^-1 + (ab - c²)^-1 = 0 então a ( bc - a²)^-2 + b(ac - b²)^-2 + c (ab - c²)^-2 é igual : Abraço. ---------------------------------------- |-=Rick-C.R.B.=- | |ICQ 124805654 | |e-mail luizhenriquerick@zipmail.com.br | ---------------------------------------- ------------------------------------------ Use o melhor sistema de busca da Internet Radar UOL - http://www.radaruol.com.br ========================================================================= Instruções para entrar na lista, sair da lista e usar a lista em http://www.mat.puc-rio.br/~nicolau/olimp/obm-l.html O administrador desta lista é ========================================================================= From owner-obm-l@sucuri.mat.puc-rio.br Tue Jun 25 13:45:53 2002 Return-Path: Received: (from majordom@localhost) by sucuri.mat.puc-rio.br (8.9.3/8.9.3) id NAA07729 for obm-l-list; Tue, 25 Jun 2002 13:45:46 -0300 Received: (from nicolau@localhost) by sucuri.mat.puc-rio.br (8.9.3/8.9.3) id NAA07724 for obm-l@mat.puc-rio.br; Tue, 25 Jun 2002 13:45:46 -0300 Date: Tue, 25 Jun 2002 13:45:46 -0300 From: "Nicolau C. Saldanha" To: obm-l@mat.puc-rio.br Subject: [obm-l] Cone Sul Message-ID: <20020625134546.A7573@sucuri.mat.puc-rio.br> Mime-Version: 1.0 Content-Type: text/plain; charset=us-ascii Content-Disposition: inline User-Agent: Mutt/1.2.5i Sender: owner-obm-l@sucuri.mat.puc-rio.br Precedence: bulk Reply-To: obm-l@mat.puc-rio.br Est'a em andamento a Olimpiada do Cone Sul. Hoje foi o primeiro dia de prova. A prova deve entrar no ar a qualquer momento. []s, N. ========================================================================= Instruções para entrar na lista, sair da lista e usar a lista em http://www.mat.puc-rio.br/~nicolau/olimp/obm-l.html O administrador desta lista é ========================================================================= From owner-obm-l@sucuri.mat.puc-rio.br Tue Jun 25 17:24:07 2002 Return-Path: Received: (from majordom@localhost) by sucuri.mat.puc-rio.br (8.9.3/8.9.3) id RAA12384 for obm-l-list; Tue, 25 Jun 2002 17:22:01 -0300 Received: (from nicolau@localhost) by sucuri.mat.puc-rio.br (8.9.3/8.9.3) id RAA12379 for obm-l@mat.puc-rio.br; Tue, 25 Jun 2002 17:22:00 -0300 Date: Tue, 25 Jun 2002 17:22:00 -0300 From: "Nicolau C. Saldanha" To: obm-l@mat.puc-rio.br Subject: Re: [obm-l] Cone Sul Message-ID: <20020625172200.A12363@sucuri.mat.puc-rio.br> References: <20020625134546.A7573@sucuri.mat.puc-rio.br> Mime-Version: 1.0 Content-Type: text/plain; charset=iso-8859-1 Content-Disposition: inline Content-Transfer-Encoding: 8bit User-Agent: Mutt/1.2.5i In-Reply-To: <20020625134546.A7573@sucuri.mat.puc-rio.br>; from nicolau@sucuri.mat.puc-rio.br on Tue, Jun 25, 2002 at 01:45:46PM -0300 Sender: owner-obm-l@sucuri.mat.puc-rio.br Precedence: bulk Reply-To: obm-l@mat.puc-rio.br On Tue, Jun 25, 2002 at 01:45:46PM -0300, Nicolau C. Saldanha wrote: > Est'a em andamento a Olimpiada do Cone Sul. > Hoje foi o primeiro dia de prova. > A prova deve entrar no ar a qualquer momento. A prova já está disponível em http://www.teorema.mat.br/cone.htm []s, N. ========================================================================= Instruções para entrar na lista, sair da lista e usar a lista em http://www.mat.puc-rio.br/~nicolau/olimp/obm-l.html O administrador desta lista é ========================================================================= From owner-obm-l@sucuri.mat.puc-rio.br Tue Jun 25 19:15:07 2002 Return-Path: Received: (from majordom@localhost) by sucuri.mat.puc-rio.br (8.9.3/8.9.3) id TAA14222 for obm-l-list; Tue, 25 Jun 2002 19:14:36 -0300 Received: from smtp-4.ig.com.br (smtp-4.ig.com.br [200.226.132.153]) by sucuri.mat.puc-rio.br (8.9.3/8.9.3) with SMTP id TAA14218 for ; Tue, 25 Jun 2002 19:14:34 -0300 Received: (qmail 16839 invoked from network); 25 Jun 2002 22:14:02 -0000 Received: from shasta029099.ig.com.br (HELO windows9) (200.151.29.99) by smtp-4.ig.com.br with SMTP; 25 Jun 2002 22:14:02 -0000 Message-ID: <003801c21c95$e35e2d20$631d97c8@windows9> From: "Eric Campos Bastos Guedes" To: References: <001701c21a4c$9690bda0$0200a8c0@macacos><003101c21ac6$dbeec3e0$970d97c8@windows9> <003001c21b11$942d83c0$0200a8c0@macacos> Subject: Re: [obm-l] Trigonometria Date: Tue, 25 Jun 2002 19:13:32 -0300 MIME-Version: 1.0 Content-Type: text/plain; charset="iso-8859-1" Content-Transfer-Encoding: 8bit X-Priority: 3 X-MSMail-Priority: Normal X-Mailer: Microsoft Outlook Express 5.00.2615.200 X-MIMEOLE: Produced By Microsoft MimeOLE V5.00.2615.200 Sender: owner-obm-l@sucuri.mat.puc-rio.br Precedence: bulk Reply-To: obm-l@mat.puc-rio.br > Você deve ter errado em algum passo de suas contas. > A resposta é (raiz de 3)/3. > > Gabriel Foi exatamente essa a resposta que obtive. Eric. > > > > Se a +b = 60º, entao: sen a + sen b / cos a + cos b = ? > > > > > a+b = Pi/3 rd > > > > sen a = sen ((a+b)/2 + (a-b)/2) = > > sen x cos y + sen y cos x > > > > onde x = (a+b)/2 e y = (a-b)/2 > > > > sen b = ((a+b)/2 - (a-b)/2) = > > sen x cos y - sen y cos x > > > > logo sen a + sen b = 2*sen x cos y > > > > cos a = cos((a+b)/2 + (a-b)/2) = > > cos x cos y - sen x sen y > > > > cos b = cos((a+b)/2 - (a-b)/2) = > > cos x cos y + sen x sen y > > > > logo cos a + cos b = 2*cos x cos y > > > > (i) suponha y /= Pi/2 + k*Pi > > entao cos y /= 0 e tem-se > > > > (sen a + sen b)/(cos a + cos b) = > > = (2*sen x cos y)/(2*cos x cos y) = > > = tan x = tan((a+b)/2) = tan ((Pi/3)/2) = > > = tan (Pi/6) = raiz(3)/3 > > > > (ii) se y = Pi/2 + k*Pi então > > > > (a-b)/2 = Pi/2 + k*Pi > > a-b = Pi + 2k*Pi > > a = b + Pi + 2k*Pi > > > > e portanto > > > > cos a + cos b = > > = cos (b + Pi) + cos b = > > = -cos b + cos b = 0 > > > > e não se pode falar em > > (sen a + sen b)/(cos a + cos b) > > > > Um abrac,o! > > > > Eric. > > > > ========================================================================= > Instruções para entrar na lista, sair da lista e usar a lista em > http://www.mat.puc-rio.br/~nicolau/olimp/obm-l.html > O administrador desta lista é > ========================================================================= ========================================================================= Instruções para entrar na lista, sair da lista e usar a lista em http://www.mat.puc-rio.br/~nicolau/olimp/obm-l.html O administrador desta lista é ========================================================================= From owner-obm-l@sucuri.mat.puc-rio.br Wed Jun 26 02:10:14 2002 Return-Path: Received: (from majordom@localhost) by sucuri.mat.puc-rio.br (8.9.3/8.9.3) id CAA18155 for obm-l-list; Wed, 26 Jun 2002 02:09:47 -0300 Received: from web21308.mail.yahoo.com (web21308.mail.yahoo.com [216.136.128.174]) by sucuri.mat.puc-rio.br (8.9.3/8.9.3) with SMTP id CAA18151 for ; Wed, 26 Jun 2002 02:09:44 -0300 Message-ID: <20020626050918.75751.qmail@web21308.mail.yahoo.com> Received: from [200.227.209.97] by web21308.mail.yahoo.com via HTTP; Wed, 26 Jun 2002 02:09:18 ART Date: Wed, 26 Jun 2002 02:09:18 -0300 (ART) From: =?iso-8859-1?q?Marcos=20Reynaldo?= Subject: [obm-l] duvida em limite To: obm-l@mat.puc-rio.br In-Reply-To: <200206240156.g5O1uPf18858@Euler.impa.br> MIME-Version: 1.0 Content-Type: text/plain; charset=iso-8859-1 Content-Transfer-Encoding: 8bit Sender: owner-obm-l@sucuri.mat.puc-rio.br Precedence: bulk Reply-To: obm-l@mat.puc-rio.br Caros colegas, talvez voces possam me ajudar em numa duvida. Resolvendo uns problemas de Cálculo do livro Calculo A da Diva Marilia e Miriam Buss, me deparei com o limite de raiz quadrada de x quando x tende a zero. Pelo que eu lembro, esse limite não existe. Mas as autoras do livro do Cálculo A, resolvem um exercicio que envolvem a soma de três funções dentre elas raiz de x e 1/x^2 (a outra não lembro, mas é tipo x, vamos dizer), da seguinte forma lim (x + raiz x + 1/x^2) quando x tende a zero = 0 + 0 + infinito = + infinito. Ora, mais ai ela considera que lim de raz x quando x tende a zero é 0. Olhei um exercicio do Guidorizzi (lim raiz de x quando x tende a zero) e ele dá como resposta 0. Não sei se não aprendi direito, mas como pode ser zero? Pela direita tudo bem , mas pela esquerda temos números complexos e esse conjunto não eh ordenado para falar que tende a zero. Gostaria de saber dos colegas quem estah certo eu ou os autores. _______________________________________________________________________ Copa 2002 Yahoo! - Patrocinador oficial da Copa do Mundo da FIFA 2002 http://br.sports.yahoo.com/fifaworldcup/ ========================================================================= Instruções para entrar na lista, sair da lista e usar a lista em http://www.mat.puc-rio.br/~nicolau/olimp/obm-l.html O administrador desta lista é ========================================================================= From owner-obm-l@sucuri.mat.puc-rio.br Wed Jun 26 02:15:51 2002 Return-Path: Received: (from majordom@localhost) by sucuri.mat.puc-rio.br (8.9.3/8.9.3) id CAA18272 for obm-l-list; Wed, 26 Jun 2002 02:15:50 -0300 Received: from web21302.mail.yahoo.com (web21302.mail.yahoo.com [216.136.173.210]) by sucuri.mat.puc-rio.br (8.9.3/8.9.3) with SMTP id CAA18268 for ; Wed, 26 Jun 2002 02:15:47 -0300 Message-ID: <20020626051530.52689.qmail@web21302.mail.yahoo.com> Received: from [200.227.209.97] by web21302.mail.yahoo.com via HTTP; Wed, 26 Jun 2002 02:15:30 ART Date: Wed, 26 Jun 2002 02:15:30 -0300 (ART) From: =?iso-8859-1?q?Marcos=20Reynaldo?= Subject: [obm-l] outra duvida To: obm-l@mat.puc-rio.br In-Reply-To: <200206240156.g5O1uPf18858@Euler.impa.br> MIME-Version: 1.0 Content-Type: text/plain; charset=iso-8859-1 Content-Transfer-Encoding: 8bit Sender: owner-obm-l@sucuri.mat.puc-rio.br Precedence: bulk Reply-To: obm-l@mat.puc-rio.br Outra duvida, a maioria dos livros de calculo define que uma funcao eh continua num ponto x=a quando 1) f(a) existe 2) lim f(x) quando x tende a a existe 3) lim f(x) quando x tende a a = f(a) Ora mas a primeira condicão não tem sentido nenhum. Pra analisar se uma função é continua tem que analisar nos pontos do dominio da função e não fora. Portanto f(a) sempre existe. Dessa definição poderia concluir então que se f(a) não existe a função é descontinua. Mas não se pode falar nada pois ela nem é definida. É a mesma coisa que perguntar qual a cor dos olhos da mula sem cabeça. Se não tem cabeça como posso dizer que isso ou aquilo. Tô errado na minha consideração ? []'s Marcos _______________________________________________________________________ Copa 2002 Yahoo! - Patrocinador oficial da Copa do Mundo da FIFA 2002 http://br.sports.yahoo.com/fifaworldcup/ ========================================================================= Instruções para entrar na lista, sair da lista e usar a lista em http://www.mat.puc-rio.br/~nicolau/olimp/obm-l.html O administrador desta lista é ========================================================================= From owner-obm-l@sucuri.mat.puc-rio.br Wed Jun 26 03:01:20 2002 Return-Path: Received: (from majordom@localhost) by sucuri.mat.puc-rio.br (8.9.3/8.9.3) id DAA19363 for obm-l-list; Wed, 26 Jun 2002 03:01:16 -0300 Received: from candeias.terra.com.br (candeias.terra.com.br [200.176.3.18]) by sucuri.mat.puc-rio.br (8.9.3/8.9.3) with ESMTP id DAA19355 for ; Wed, 26 Jun 2002 03:01:14 -0300 Received: from smtp4-poa.terra.com.br (smtp4-poa.terra.com.br [200.176.3.35]) by candeias.terra.com.br (Postfix) with ESMTP id 2287843D9A for ; Wed, 26 Jun 2002 03:00:59 -0300 (EST) Received: from stabel (dl-nas3-poa-C89A0646.p001.terra.com.br [200.154.6.70]) (authenticated user dudasta) by smtp4-poa.terra.com.br (Postfix) with ESMTP id C3C81AC59C for ; Wed, 26 Jun 2002 03:00:57 -0300 (EST) Message-ID: <000701c21cd6$d77b0d20$46069ac8@stabel> From: "Eduardo Casagrande Stabel" To: Subject: [obm-l] Matriz de Vandermonde Date: Wed, 26 Jun 2002 03:00:55 -0300 MIME-Version: 1.0 Content-Type: text/plain; charset="iso-8859-1" Content-Transfer-Encoding: 8bit X-Priority: 3 X-MSMail-Priority: Normal X-Mailer: Microsoft Outlook Express 6.00.2600.0000 X-MIMEOLE: Produced By Microsoft MimeOLE V6.00.2600.0000 Sender: owner-obm-l@sucuri.mat.puc-rio.br Precedence: bulk Reply-To: obm-l@mat.puc-rio.br Ola pessoal da lista! Uma matriz de Vandermonde é uma matriz P da forma P_(i,j) = [t_(i-1)]^j onde i e j estão entre 0 e n um jeito mais explicito é o seguinte P = [ 1 t_0 (t_0)^2 (t_0)^3 ... (t_0)^n ] [ 1 t_1 (t_1)^2 (t_1)^3 ... (t_1)^n ] [ ... ] [ 1 t_n (t_n)^2 (t_n)^3 ... (t_n)^n ] Eu não estou conseguindo demonstrar que se os t_i's são todos distintos então a matriz P é inversível. Alguém demonstra? Obrigado pela futura ajuda Eduardo Casagrande Stabel. Porto Alegre, RS. ========================================================================= Instruções para entrar na lista, sair da lista e usar a lista em http://www.mat.puc-rio.br/~nicolau/olimp/obm-l.html O administrador desta lista é ========================================================================= From owner-obm-l@sucuri.mat.puc-rio.br Wed Jun 26 03:50:29 2002 Return-Path: Received: (from majordom@localhost) by sucuri.mat.puc-rio.br (8.9.3/8.9.3) id DAA20204 for obm-l-list; Wed, 26 Jun 2002 03:49:27 -0300 Received: from pina.terra.com.br (pina.terra.com.br [200.176.3.17]) by sucuri.mat.puc-rio.br (8.9.3/8.9.3) with ESMTP id DAA20191 for ; Wed, 26 Jun 2002 03:49:24 -0300 Received: from smtp4-poa.terra.com.br (smtp4-poa.terra.com.br [200.176.3.35]) by pina.terra.com.br (Postfix) with ESMTP id B49B252FE3 for ; Wed, 26 Jun 2002 03:49:09 -0300 (EST) Received: from stabel (dl-nas3-poa-C89A0646.p001.terra.com.br [200.154.6.70]) (authenticated user dudasta) by smtp4-poa.terra.com.br (Postfix) with ESMTP id EAF42AC5A6 for ; Wed, 26 Jun 2002 03:49:07 -0300 (EST) Message-ID: <001401c21cdd$91f974b0$46069ac8@stabel> From: "Eduardo Casagrande Stabel" To: References: <20020626050918.75751.qmail@web21308.mail.yahoo.com> Subject: Re: [obm-l] duvida em limite Date: Wed, 26 Jun 2002 03:49:05 -0300 MIME-Version: 1.0 Content-Type: text/plain; charset="iso-8859-1" Content-Transfer-Encoding: 8bit X-Priority: 3 X-MSMail-Priority: Normal X-Mailer: Microsoft Outlook Express 6.00.2600.0000 X-MIMEOLE: Produced By Microsoft MimeOLE V6.00.2600.0000 Sender: owner-obm-l@sucuri.mat.puc-rio.br Precedence: bulk Reply-To: obm-l@mat.puc-rio.br From: "Marcos Reynaldo" [Primeiro e-mail] > Caros colegas, talvez voces possam me ajudar em numa > duvida. > Resolvendo uns problemas de Cálculo do livro Calculo A > da Diva Marilia e Miriam Buss, me deparei com o limite > de raiz quadrada de x quando x tende a zero. Pelo que > eu lembro, esse limite não existe. Mas as autoras do > livro do Cálculo A, resolvem um exercicio que envolvem > a soma de três funções dentre elas raiz de x e 1/x^2 > (a outra não lembro, mas é tipo x, vamos dizer), da > seguinte forma lim (x + raiz x + 1/x^2) quando x tende > a zero = 0 + 0 + infinito = + infinito. Ora, mais ai > ela considera que lim de raz x quando x tende a zero é > 0. Olhei um exercicio do Guidorizzi (lim raiz de x > quando x tende a zero) e ele dá como resposta 0. > Não sei se não aprendi direito, mas como pode ser > zero? Pela direita tudo bem , mas pela esquerda temos > números complexos e esse conjunto não eh ordenado para > falar que tende a zero. > Gostaria de saber dos colegas quem estah certo eu ou > os autores. Marcos, depende do domínio que está sendo considerado. Na função raiz : R^(+) -> R raiz(x) = x^(1/2) certamente temos lim( raiz(x) , x->0 ) = 0, pelo que você mesmo disse, pois pela direita o limite é zero e como ela não está definida à esquerda de zero, não há mais o que se considerar. No caso complexo, podemos definir uma função raiz (contínua) que satisfaz (raiz(z))^2 = z somente em um pedaço do plano complexo, por exemplo para z = r * e^(i*a) onde r > 0 e -pi < a < pi definimos raiz(z) = raiz(r) * e^(i*(a/2)) e raiz(0) = 0 nesse domínio (que é os complexos tirando fora os números reais negativos) temos lim( raiz(z) , z->0 ) = 0, mas aqui o limite é num sentido um pouco diferente do limite que você viu para os números reais. Para se definir limite nos complexos você procede assim: numa sequência de complexos z_n dizemos que ela é convergente ao limite z, se for satisfeita a seguinte condição: para todo e>0 existe um N tal que n > N implica |z - z_n| < e. Ou seja, a partir de um certo n os z_n ficam muito próximos (no plano complexo) de z. Assim, você não precisa de uma ordem para definir o limite, só precisa de uma função (a chamada métrica) que calcula a distância de elementos. (existem ainda outras formas de definir limite sem utilizar métricas, só para constar) A idéia de limite é, na verdade, muito mais geral que o caso real, portanto. [Segundo e-mail] > Outra duvida, a maioria dos livros de calculo define > que uma funcao eh continua num ponto x=a quando > 1) f(a) existe > 2) lim f(x) quando x tende a a existe > 3) lim f(x) quando x tende a a = f(a) > > Ora mas a primeira condicão não tem sentido nenhum. > Pra analisar se uma função é continua tem que analisar > nos pontos do dominio da função e não fora. Portanto > f(a) sempre existe. Dessa definição poderia concluir > então que se f(a) não existe a função é descontinua. > Mas não se pode falar nada pois ela nem é definida. É > a mesma coisa que perguntar qual a cor dos olhos da > mula sem cabeça. Se não tem cabeça como posso dizer > que isso ou aquilo. > Tô errado na minha consideração ? > > []'s Marcos A primeira condição tem sentido sim, e quer dizer que "a" pertence ao domínio da f. O fato de f(a) existir é puramente uma questão de como foi definida a função f. Por exemplo, a função f: R-{0} -> R f(x) = 1, pra todo x do domínio é contínua em todos os pontos de seu domínio (os reais não nulos), mas não faz sentido perguntar se a f é contínua no zero, afinal não podemos comparar o valor lim( f(x), x-> 0) (que nesse caso existe e é igual a 1) com f(0), pois nem definimos quanto é f no zero. (e vale a sua comparação, é como perguntar a cor dos olhos da mula sem cabeça). É claro que nós podemos estender (aumentar o domínio da função) a f para todos os reais definindo f(0)=1 e aí a nova f vai ser contínua em zero. Vou me repetir mais uma vez, o que a definição diz é que só podemos perguntar se a f é contínua num determinado ponto se ela estiver definida naquele ponto. Acho que esclareci um pouco. Um abraço! Eduardo Casagrande Stabel. Porto Alegre, RS. ========================================================================= Instruções para entrar na lista, sair da lista e usar a lista em http://www.mat.puc-rio.br/~nicolau/olimp/obm-l.html O administrador desta lista é ========================================================================= From owner-obm-l@sucuri.mat.puc-rio.br Wed Jun 26 08:36:32 2002 Return-Path: Received: (from majordom@localhost) by sucuri.mat.puc-rio.br (8.9.3/8.9.3) id IAA22373 for obm-l-list; Wed, 26 Jun 2002 08:36:21 -0300 Received: from web20421.mail.yahoo.com (web20409.mail.yahoo.com [66.163.169.97]) by sucuri.mat.puc-rio.br (8.9.3/8.9.3) with SMTP id IAA22369 for ; Wed, 26 Jun 2002 08:36:16 -0300 Message-ID: <20020626113600.48079.qmail@web20421.mail.yahoo.com> Received: from [200.206.103.3] by web20409.mail.yahoo.com via HTTP; Wed, 26 Jun 2002 08:36:00 ART Date: Wed, 26 Jun 2002 08:36:00 -0300 (ART) From: =?iso-8859-1?q?Ricardo=20Dirichlet?= Subject: Re: [obm-l] duvida em limite To: obm-l@mat.puc-rio.br In-Reply-To: <20020626050918.75751.qmail@web21308.mail.yahoo.com> MIME-Version: 1.0 Content-Type: text/plain; charset=iso-8859-1 Content-Transfer-Encoding: 8bit Sender: owner-obm-l@sucuri.mat.puc-rio.br Precedence: bulk Reply-To: obm-l@mat.puc-rio.br --- Marcos Reynaldo escreveu: > Caros colegas, talvez voces possam me ajudar em numa > duvida. > Resolvendo uns problemas de Cálculo do livro Calculo > A > da Diva Marilia e Miriam Buss, me deparei com o > limite > de raiz quadrada de x quando x tende a zero. Pelo > que > eu lembro, esse limite não existe. Mas as autoras do > livro do Cálculo A, resolvem um exercicio que > envolvem > a soma de três funções dentre elas raiz de x e 1/x^2 > (a outra não lembro, mas é tipo x, vamos dizer), da > seguinte forma lim (x + raiz x + 1/x^2) quando x > tende > a zero = 0 + 0 + infinito = + infinito. Ora, mais ai > ela considera que lim de raz x quando x tende a zero > é > 0. Olhei um exercicio do Guidorizzi (lim raiz de x > quando x tende a zero) e ele dá como resposta 0. > Não sei se não aprendi direito, mas como pode ser > zero? Pela direita tudo bem , mas pela esquerda > temos > números complexos e esse conjunto não eh ordenado > para > falar que tende a zero. > Gostaria de saber dos colegas quem estah certo eu ou > os autores. > > Reynaldo,pode-se definir limites nos complexos tambem,sabia?A definiçao e analoga,mas voce usa a noçao de modulo em complexos para desenvolver a teoria(por isso e analoga).Eu tenho um livro,"Variaveis Compexas",da Coleçao Schaum,Ed.McGrawHill entre alguns livros de meu arsenal.La,o Murray Spiegel(eu acho)define numeros complexos,mostra algumas aplicaçoes(soma de senos em PA,raizes de equaçoes legais,geometria com vetores,e por ai vai)e as definiçoes de limite em complexos.Nao sei se voce consegue achar esse livro por ai,ja que ele e bem velho(achei num porao de uma casa de um amigo da familia).De todo modo,divirta-se! Ass.:Peterdirichlet_______________________________________________________________________ > Copa 2002 > Yahoo! - Patrocinador oficial da Copa do Mundo da > FIFA 2002 > http://br.sports.yahoo.com/fifaworldcup/ > ========================================================================= > Instruções para entrar na lista, sair da lista e > usar a lista em > http://www.mat.puc-rio.br/~nicolau/olimp/obm-l.html > O administrador desta lista é > > ========================================================================= _______________________________________________________________________ Copa 2002 Yahoo! - Patrocinador oficial da Copa do Mundo da FIFA 2002 http://br.sports.yahoo.com/fifaworldcup/ ========================================================================= Instruções para entrar na lista, sair da lista e usar a lista em http://www.mat.puc-rio.br/~nicolau/olimp/obm-l.html O administrador desta lista é ========================================================================= From owner-obm-l@sucuri.mat.puc-rio.br Wed Jun 26 09:22:49 2002 Return-Path: Received: (from majordom@localhost) by sucuri.mat.puc-rio.br (8.9.3/8.9.3) id JAA23048 for obm-l-list; Wed, 26 Jun 2002 09:22:45 -0300 Received: from web20421.mail.yahoo.com (web20409.mail.yahoo.com [66.163.169.97]) by sucuri.mat.puc-rio.br (8.9.3/8.9.3) with SMTP id JAA23044 for ; Wed, 26 Jun 2002 09:22:42 -0300 Message-ID: <20020626122227.56458.qmail@web20421.mail.yahoo.com> Received: from [200.206.103.3] by web20409.mail.yahoo.com via HTTP; Wed, 26 Jun 2002 09:22:27 ART Date: Wed, 26 Jun 2002 09:22:27 -0300 (ART) From: =?iso-8859-1?q?Ricardo=20Dirichlet?= Subject: Re: [obm-l] outra duvida To: obm-l@mat.puc-rio.br In-Reply-To: <20020626051530.52689.qmail@web21302.mail.yahoo.com> MIME-Version: 1.0 Content-Type: text/plain; charset=iso-8859-1 Content-Transfer-Encoding: 8bit Sender: owner-obm-l@sucuri.mat.puc-rio.br Precedence: bulk Reply-To: obm-l@mat.puc-rio.br Tem um pequeno detalhe:voce pode definir funçoes nas quais nao exista f(a) mas exista lim f(x) com x cada vez mais "perto de a"(e o mesmo valendo a,acredita?!?!???!!??!?!)!!!!. Para sacanear,um exemplo simples e f(x)=(x-a)^2/(x-a).Como nao existe divisao por 0 em reais(e complexos),nao existe f(a).Mas existe o limite de f(x) quando x tende a a,e o mesmo e o ponto a. Se eu nao me engano isso se chama DESCONTINUIDADE EVITAVEL(poderiamos definir f(a)=a e evitar a descontinuidade).E como voce tentar(nesse caso com sucesso)por uma cabeça na mula e depois dizer a cor dos olhos da (des)dita. Ah,a descontinuidade evitanel ocorre tambem se f(a) nao for igual ao limite(por exemplo,f(1)=1 mol e f(x)=x para x diferente de 1). Qualquer duvida depois eu esclareço. Peterdirichlet --- Marcos Reynaldo escreveu: > Outra duvida, a maioria dos livros de calculo define > que uma funcao eh continua num ponto x=a quando > 1) f(a) existe > 2) lim f(x) quando x tende a a existe > 3) lim f(x) quando x tende a a = f(a) > > Ora mas a primeira condicão não tem sentido nenhum. > Pra analisar se uma função é continua tem que > analisar > nos pontos do dominio da função e não fora. Portanto > f(a) sempre existe. Dessa definição poderia concluir > então que se f(a) não existe a função é descontinua. > Mas não se pode falar nada pois ela nem é definida. > É > a mesma coisa que perguntar qual a cor dos olhos da > mula sem cabeça. Se não tem cabeça como posso dizer > que isso ou aquilo. > Tô errado na minha consideração ? > > []'s Marcos > > _______________________________________________________________________ > Copa 2002 > Yahoo! - Patrocinador oficial da Copa do Mundo da > FIFA 2002 > http://br.sports.yahoo.com/fifaworldcup/ > ========================================================================= > Instruções para entrar na lista, sair da lista e > usar a lista em > http://www.mat.puc-rio.br/~nicolau/olimp/obm-l.html > O administrador desta lista é > > ========================================================================= _______________________________________________________________________ Copa 2002 Yahoo! - Patrocinador oficial da Copa do Mundo da FIFA 2002 http://br.sports.yahoo.com/fifaworldcup/ ========================================================================= Instruções para entrar na lista, sair da lista e usar a lista em http://www.mat.puc-rio.br/~nicolau/olimp/obm-l.html O administrador desta lista é ========================================================================= From owner-obm-l@sucuri.mat.puc-rio.br Wed Jun 26 09:56:52 2002 Return-Path: Received: (from majordom@localhost) by sucuri.mat.puc-rio.br (8.9.3/8.9.3) id JAA23681 for obm-l-list; Wed, 26 Jun 2002 09:56:16 -0300 Received: from web20411.mail.yahoo.com (web20411.mail.yahoo.com [216.136.227.9]) by sucuri.mat.puc-rio.br (8.9.3/8.9.3) with SMTP id JAA23677 for ; Wed, 26 Jun 2002 09:56:13 -0300 Message-ID: <20020626125557.38786.qmail@web20411.mail.yahoo.com> Received: from [200.206.103.3] by web20411.mail.yahoo.com via HTTP; Wed, 26 Jun 2002 09:55:57 ART Date: Wed, 26 Jun 2002 09:55:57 -0300 (ART) From: =?iso-8859-1?q?Johann=20Dirichlet?= Subject: Re: [obm-l] Matriz de Vandermonde To: obm-l@mat.puc-rio.br In-Reply-To: <000701c21cd6$d77b0d20$46069ac8@stabel> MIME-Version: 1.0 Content-Type: text/plain; charset=iso-8859-1 Content-Transfer-Encoding: 8bit Sender: owner-obm-l@sucuri.mat.puc-rio.br Precedence: bulk Reply-To: obm-l@mat.puc-rio.br Duda,eu me lembro de que uma matriz e nao inversivel se e so se for singular,ou seja, seu determinante for 0.Entao o que voce quer provar e que se os t's forem diferentes o determinante nao e zero.Se eu nao me engano ha uma formula para a matriz de Vandermonde que so usa as diferenças entre os t's.Se voce conseguir acha-la(deve ter em qualquer livro sobre isso),COMEMORE!!!!!!!! Peterdirichlet --- Eduardo Casagrande Stabel escreveu: > Ola pessoal da lista! > > Uma matriz de Vandermonde é uma matriz P da forma > P_(i,j) = [t_(i-1)]^j onde i e j estão entre 0 e n > um jeito mais explicito é o seguinte > P = > [ 1 t_0 (t_0)^2 (t_0)^3 ... (t_0)^n ] > [ 1 t_1 (t_1)^2 (t_1)^3 ... (t_1)^n ] > [ ... ] > [ 1 t_n (t_n)^2 (t_n)^3 ... (t_n)^n ] > > Eu não estou conseguindo demonstrar que se os t_i's > são todos distintos > então a matriz P é inversível. > > Alguém demonstra? > > Obrigado pela futura ajuda > > Eduardo Casagrande Stabel. Porto Alegre, RS. > > ========================================================================= > Instruções para entrar na lista, sair da lista e > usar a lista em > http://www.mat.puc-rio.br/~nicolau/olimp/obm-l.html > O administrador desta lista é > > ========================================================================= _______________________________________________________________________ Copa 2002 Yahoo! - Patrocinador oficial da Copa do Mundo da FIFA 2002 http://br.sports.yahoo.com/fifaworldcup/ ========================================================================= Instruções para entrar na lista, sair da lista e usar a lista em http://www.mat.puc-rio.br/~nicolau/olimp/obm-l.html O administrador desta lista é ========================================================================= From owner-obm-l@sucuri.mat.puc-rio.br Wed Jun 26 11:40:05 2002 Return-Path: Received: (from majordom@localhost) by sucuri.mat.puc-rio.br (8.9.3/8.9.3) id LAA25422 for obm-l-list; Wed, 26 Jun 2002 11:39:55 -0300 Received: from web10205.mail.yahoo.com (web10205.mail.yahoo.com [216.136.130.69]) by sucuri.mat.puc-rio.br (8.9.3/8.9.3) with SMTP id LAA25418 for ; Wed, 26 Jun 2002 11:39:52 -0300 Message-ID: <20020626143937.6986.qmail@web10205.mail.yahoo.com> Received: from [161.24.73.175] by web10205.mail.yahoo.com via HTTP; Wed, 26 Jun 2002 11:39:37 ART Date: Wed, 26 Jun 2002 11:39:37 -0300 (ART) From: =?iso-8859-1?q?Humberto=20Naves?= Subject: Re: [obm-l] Matriz de Vandermonde To: obm-l@mat.puc-rio.br In-Reply-To: <000701c21cd6$d77b0d20$46069ac8@stabel> MIME-Version: 1.0 Content-Type: text/plain; charset=iso-8859-1 Content-Transfer-Encoding: 8bit Sender: owner-obm-l@sucuri.mat.puc-rio.br Precedence: bulk Reply-To: obm-l@mat.puc-rio.br Oi, É possível demonstrar que o determinante de Vandermonde é Produtório (0 <= i < j <= n) de ((t_i) - (t_j)). Para ver isso, basta encarar o determinante como um polinômio em t_i, e ver que quando t_i = t_j, o polinômio se anula. Logo se os t_i's forem distintos, o determinante é diferente de 0. Falow, Humberto --- Eduardo Casagrande Stabel escreveu: > Ola pessoal da lista! > > Uma matriz de Vandermonde é uma matriz P da forma > P_(i,j) = [t_(i-1)]^j onde i e j estão entre 0 e n > um jeito mais explicito é o seguinte > P = > [ 1 t_0 (t_0)^2 (t_0)^3 ... (t_0)^n ] > [ 1 t_1 (t_1)^2 (t_1)^3 ... (t_1)^n ] > [ ... ] > [ 1 t_n (t_n)^2 (t_n)^3 ... (t_n)^n ] > > Eu não estou conseguindo demonstrar que se os t_i's são todos distintos > então a matriz P é inversível. > > Alguém demonstra? > > Obrigado pela futura ajuda > > Eduardo Casagrande Stabel. Porto Alegre, RS. > > ========================================================================= > Instruções para entrar na lista, sair da lista e usar a lista em > http://www.mat.puc-rio.br/~nicolau/olimp/obm-l.html > O administrador desta lista é > ========================================================================= _______________________________________________________________________ Copa 2002 Yahoo! - Patrocinador oficial da Copa do Mundo da FIFA 2002 http://br.sports.yahoo.com/fifaworldcup/ ========================================================================= Instruções para entrar na lista, sair da lista e usar a lista em http://www.mat.puc-rio.br/~nicolau/olimp/obm-l.html O administrador desta lista é ========================================================================= From owner-obm-l@sucuri.mat.puc-rio.br Wed Jun 26 12:03:49 2002 Return-Path: Received: (from majordom@localhost) by sucuri.mat.puc-rio.br (8.9.3/8.9.3) id MAA25980 for obm-l-list; Wed, 26 Jun 2002 12:03:42 -0300 Received: from web20416.mail.yahoo.com (web20404.mail.yahoo.com [66.163.169.92]) by sucuri.mat.puc-rio.br (8.9.3/8.9.3) with SMTP id MAA25976 for ; Wed, 26 Jun 2002 12:03:40 -0300 Message-ID: <20020626150324.83761.qmail@web20416.mail.yahoo.com> Received: from [200.206.103.3] by web20404.mail.yahoo.com via HTTP; Wed, 26 Jun 2002 12:03:24 ART Date: Wed, 26 Jun 2002 12:03:24 -0300 (ART) From: =?iso-8859-1?q?Johann=20Dirichlet?= Subject: [obm-l] duvida To: obm-l@mat.puc-rio.br MIME-Version: 1.0 Content-Type: text/plain; charset=iso-8859-1 Content-Transfer-Encoding: 8bit Sender: owner-obm-l@sucuri.mat.puc-rio.br Precedence: bulk Reply-To: obm-l@mat.puc-rio.br E ai turma,alguem consegue me dizer como faço para resolver isso? Sejam n,p,q inteiros positivos com n>p+q;sejam x0,x1,...,xn inteiros tais que :1)x0=xn=0;2)para cada inteiro i com 1<=I<=n,xi-x(i-1)=p ou -q. Prove que existe um par (i,j)com xi=xj e i ========================================================================= From owner-obm-l@sucuri.mat.puc-rio.br Wed Jun 26 16:23:38 2002 Return-Path: Received: (from majordom@localhost) by sucuri.mat.puc-rio.br (8.9.3/8.9.3) id QAA30354 for obm-l-list; Wed, 26 Jun 2002 16:21:32 -0300 Received: from traven9.uol.com.br (traven9.uol.com.br [200.231.206.210]) by sucuri.mat.puc-rio.br (8.9.3/8.9.3) with ESMTP id QAA30350 for ; Wed, 26 Jun 2002 16:21:29 -0300 Received: from oemcomputer ([200.227.68.195]) by traven9.uol.com.br (8.9.1/8.9.1) with SMTP id QAA25008; Wed, 26 Jun 2002 16:22:25 -0300 (BRT) Message-ID: <000901c21d46$bd9a08a0$c344e3c8@oemcomputer> From: "Paulo Rodrigues" To: , , "Olimpiada Brasileira de Matematica" , Subject: [obm-l] Prova do 2o dia - Cone Sul Date: Wed, 26 Jun 2002 16:19:59 -0300 MIME-Version: 1.0 Content-Type: text/plain; charset="iso-8859-1" Content-Transfer-Encoding: 8bit X-Priority: 3 X-MSMail-Priority: Normal X-Mailer: Microsoft Outlook Express 5.00.2615.200 X-MimeOLE: Produced By Microsoft MimeOLE V5.00.2615.200 Sender: owner-obm-l@sucuri.mat.puc-rio.br Precedence: bulk Reply-To: obm-l@mat.puc-rio.br A prova do segundo dia da XIII Olimpíada do Cone Sul está disponível em http://www.olimpiada.mat.br Paulo --- esta mensagem não contém vírus! Checked by AVG anti-virus system (http://www.grisoft.com). Version: 6.0.363 / Virus Database: 201 - Release Date: 21/05/2002 ========================================================================= Instruções para entrar na lista, sair da lista e usar a lista em http://www.mat.puc-rio.br/~nicolau/olimp/obm-l.html O administrador desta lista é ========================================================================= From owner-obm-l@sucuri.mat.puc-rio.br Wed Jun 26 19:45:09 2002 Return-Path: Received: (from majordom@localhost) by sucuri.mat.puc-rio.br (8.9.3/8.9.3) id TAA00894 for obm-l-list; Wed, 26 Jun 2002 19:44:38 -0300 Received: from hotmail.com (f71.law4.hotmail.com [216.33.149.71]) by sucuri.mat.puc-rio.br (8.9.3/8.9.3) with ESMTP id TAA00890 for ; Wed, 26 Jun 2002 19:44:35 -0300 Received: from mail pickup service by hotmail.com with Microsoft SMTPSVC; Wed, 26 Jun 2002 15:43:42 -0700 Received: from 200.194.208.216 by lw4fd.law4.hotmail.msn.com with HTTP; Wed, 26 Jun 2002 22:43:41 GMT X-Originating-IP: [200.194.208.216] From: "Laurito Alves" To: obm-l@mat.puc-rio.br Subject: Re: [obm-l] outra duvida Date: Wed, 26 Jun 2002 22:43:41 +0000 Mime-Version: 1.0 Content-Type: text/plain; format=flowed Message-ID: X-OriginalArrivalTime: 26 Jun 2002 22:43:42.0090 (UTC) FILETIME=[EBDC9EA0:01C21D62] Sender: owner-obm-l@sucuri.mat.puc-rio.br Precedence: bulk Reply-To: obm-l@mat.puc-rio.br Ricardo, Com sua observação posso inferir que a função f: R-{a} --> R dada por f(x) = (x-a)^2/(x-a) é descontínua em x=a pois existe o limite de f(x) quando x tende a a, correto ? Essa mesma função é contínua ou descontínua em x = 1+i ? Laurito >From: Ricardo Dirichlet >Reply-To: obm-l@mat.puc-rio.br >To: obm-l@mat.puc-rio.br >Subject: Re: [obm-l] outra duvida >Date: Wed, 26 Jun 2002 09:22:27 -0300 (ART) > >Tem um pequeno detalhe:voce pode definir funçoes nas >quais nao exista f(a) mas exista lim f(x) com x cada >vez mais "perto de a"(e o mesmo valendo >a,acredita?!?!???!!??!?!)!!!!. >Para sacanear,um exemplo simples e >f(x)=(x-a)^2/(x-a).Como nao existe divisao por 0 em >reais(e complexos),nao existe f(a).Mas existe o limite >de f(x) quando x tende a a,e o mesmo e o ponto a. >Se eu nao me engano isso se chama DESCONTINUIDADE >EVITAVEL(poderiamos definir f(a)=a e evitar a >descontinuidade).E como voce tentar(nesse caso com >sucesso)por uma cabeça na mula e depois dizer a cor >dos olhos da (des)dita. >Ah,a descontinuidade evitanel ocorre tambem se f(a) >nao for igual ao limite(por exemplo,f(1)=1 mol e >f(x)=x para x diferente de 1). >Qualquer duvida depois eu esclareço. >Peterdirichlet _________________________________________________________________ Send and receive Hotmail on your mobile device: http://mobile.msn.com ========================================================================= Instruções para entrar na lista, sair da lista e usar a lista em http://www.mat.puc-rio.br/~nicolau/olimp/obm-l.html O administrador desta lista é ========================================================================= From owner-obm-l@sucuri.mat.puc-rio.br Wed Jun 26 19:50:18 2002 Return-Path: Received: (from majordom@localhost) by sucuri.mat.puc-rio.br (8.9.3/8.9.3) id TAA01005 for obm-l-list; Wed, 26 Jun 2002 19:50:13 -0300 Received: from gorgo.centroin.com.br (gorgo.centroin.com.br [200.225.63.128]) by sucuri.mat.puc-rio.br (8.9.3/8.9.3) with ESMTP id TAA00992 for ; Wed, 26 Jun 2002 19:50:09 -0300 Received: from centroin.com.br (du168c.rjo.centroin.com.br [200.225.58.168]) (authenticated bits=0) by gorgo.centroin.com.br (8.12.2/8.12.1) with ESMTP id g5QMoCJF010638 for ; Wed, 26 Jun 2002 19:50:13 -0300 (BRT) Message-ID: <3D1A457A.1050507@centroin.com.br> Date: Wed, 26 Jun 2002 19:51:38 -0300 From: Augusto =?ISO-8859-1?Q?C=E9sar?= Morgado User-Agent: Mozilla/5.0 (Windows; U; Win98; en-US; rv:0.9.4.1) Gecko/20020508 Netscape6/6.2.3 X-Accept-Language: en-us MIME-Version: 1.0 To: obm-l@mat.puc-rio.br Subject: Re: [obm-l] =?ISO-8859-1?Q?FATORA=C7=C3O?= References: <3D188FA000000087@www.zipmail.com.br> Content-Type: text/plain; charset=ISO-8859-1; format=flowed Content-Transfer-Encoding: 8bit Sender: owner-obm-l@sucuri.mat.puc-rio.br Precedence: bulk Reply-To: obm-l@mat.puc-rio.br O 3 ja apareceu na lista ha pouco tempo. No 1, multiplique por ( 3^2^0 - 1 ) luizhenriquerick@zipmail.com.br wrote: >Olá amigos , tive alguns problemas para fatorar esses exercícios abaixo >,principalmente a número 3 que é bem estranha , se puderem me dar uma ajudinha >; >1- >Se P = ( 3^2^0 + 1 )( 3^2^1 + 1 )( 3^2^2 + 1 ) . . . ( 3^2^n + 1 ) então >P é igual a: > > >2- > A expressão [68 + 48 . (2)^1/2]^1/4 - [ 25 + 22 . (2)^1/2]^1/3 > vale: > >3- > >(bc - a²)^-1 + (ca - b²)^-1 + (ab - c²)^-1 = 0 então a ( bc - a²)^-2 > + b(ac - b²)^-2 + c (ab - c²)^-2 é igual : > > >Abraço. > > ---------------------------------------- > |-=Rick-C.R.B.=- | > |ICQ 124805654 | > |e-mail luizhenriquerick@zipmail.com.br | > ---------------------------------------- > > >------------------------------------------ >Use o melhor sistema de busca da Internet >Radar UOL - http://www.radaruol.com.br > > > >========================================================================= >Instruções para entrar na lista, sair da lista e usar a lista em >http://www.mat.puc-rio.br/~nicolau/olimp/obm-l.html >O administrador desta lista é >========================================================================= > > ========================================================================= Instruções para entrar na lista, sair da lista e usar a lista em http://www.mat.puc-rio.br/~nicolau/olimp/obm-l.html O administrador desta lista é ========================================================================= From owner-obm-l@sucuri.mat.puc-rio.br Wed Jun 26 20:16:17 2002 Return-Path: Received: (from majordom@localhost) by sucuri.mat.puc-rio.br (8.9.3/8.9.3) id UAA01925 for obm-l-list; Wed, 26 Jun 2002 20:16:09 -0300 Received: from gorgo.centroin.com.br (gorgo.centroin.com.br [200.225.63.128]) by sucuri.mat.puc-rio.br (8.9.3/8.9.3) with ESMTP id UAA01921 for ; Wed, 26 Jun 2002 20:16:07 -0300 Received: from centroin.com.br (du6c.rjo.centroin.com.br [200.225.58.6]) (authenticated bits=0) by gorgo.centroin.com.br (8.12.2/8.12.1) with ESMTP id g5QNGAJF012117 for ; Wed, 26 Jun 2002 20:16:10 -0300 (BRT) Message-ID: <3D1A4B90.4030200@centroin.com.br> Date: Wed, 26 Jun 2002 20:17:36 -0300 From: Augusto =?ISO-8859-1?Q?C=E9sar?= Morgado User-Agent: Mozilla/5.0 (Windows; U; Win98; en-US; rv:0.9.4.1) Gecko/20020508 Netscape6/6.2.3 X-Accept-Language: en-us MIME-Version: 1.0 To: obm-l@mat.puc-rio.br Subject: Re: [obm-l] =?ISO-8859-1?Q?FATORA=C7=C3O?= References: <3D188FA000000087@www.zipmail.com.br> Content-Type: text/plain; charset=ISO-8859-1; format=flowed Content-Transfer-Encoding: 8bit Sender: owner-obm-l@sucuri.mat.puc-rio.br Precedence: bulk Reply-To: obm-l@mat.puc-rio.br No 2, [68 + 48 . (2)^1/2] = 4 [ 3 + 2 raiz de 2] ^2 [ 25 + 22 . (2)^1/2] = [1+2raiz de 2] ^3 luizhenriquerick@zipmail.com.br wrote: >Olá amigos , tive alguns problemas para fatorar esses exercícios abaixo >,principalmente a número 3 que é bem estranha , se puderem me dar uma ajudinha >; >1- >Se P = ( 3^2^0 + 1 )( 3^2^1 + 1 )( 3^2^2 + 1 ) . . . ( 3^2^n + 1 ) então >P é igual a: > > >2- > A expressão [68 + 48 . (2)^1/2]^1/4 - [ 25 + 22 . (2)^1/2]^1/3 > vale: > >3- > >(bc - a²)^-1 + (ca - b²)^-1 + (ab - c²)^-1 = 0 então a ( bc - a²)^-2 > + b(ac - b²)^-2 + c (ab - c²)^-2 é igual : > > >Abraço. > > ---------------------------------------- > |-=Rick-C.R.B.=- | > |ICQ 124805654 | > |e-mail luizhenriquerick@zipmail.com.br | > ---------------------------------------- > > >------------------------------------------ >Use o melhor sistema de busca da Internet >Radar UOL - http://www.radaruol.com.br > > > >========================================================================= >Instruções para entrar na lista, sair da lista e usar a lista em >http://www.mat.puc-rio.br/~nicolau/olimp/obm-l.html >O administrador desta lista é >========================================================================= > > ========================================================================= Instruções para entrar na lista, sair da lista e usar a lista em http://www.mat.puc-rio.br/~nicolau/olimp/obm-l.html O administrador desta lista é ========================================================================= From owner-obm-l@sucuri.mat.puc-rio.br Wed Jun 26 20:58:09 2002 Return-Path: Received: (from majordom@localhost) by sucuri.mat.puc-rio.br (8.9.3/8.9.3) id UAA02863 for obm-l-list; Wed, 26 Jun 2002 20:57:10 -0300 Received: from imo-m01.mx.aol.com (imo-m01.mx.aol.com [64.12.136.4]) by sucuri.mat.puc-rio.br (8.9.3/8.9.3) with ESMTP id UAA02859 for ; Wed, 26 Jun 2002 20:57:08 -0300 From: Euraul@aol.com Received: from Euraul@aol.com by imo-m01.mx.aol.com (mail_out_v32.21.) id z.47.1f083ca7 (4363) for ; Wed, 26 Jun 2002 19:56:34 -0400 (EDT) Message-ID: <47.1f083ca7.2a4baeb1@aol.com> Date: Wed, 26 Jun 2002 19:56:33 EDT Subject: [obm-l] Tartaglia To: obm-l@mat.puc-rio.br MIME-Version: 1.0 Content-Type: multipart/alternative; boundary="part1_47.1f083ca7.2a4baeb1_boundary" X-Mailer: AOL 6.0 for Windows BR sub 10516 Sender: owner-obm-l@sucuri.mat.puc-rio.br Precedence: bulk Reply-To: obm-l@mat.puc-rio.br --part1_47.1f083ca7.2a4baeb1_boundary Content-Type: text/plain; charset="ISO-8859-1" Content-Transfer-Encoding: quoted-printable Sauda=E7=F5es a todos, estou lendo algo sobre Tartaglia, Cardan e Del Fiore onde encontrei=20 problemas que gostaria de suas resolu=E7=F5es: 1-) Cortar uma reta de comprimento dado em 3 segmentos com os quais seja=20 poss=EDvel construir um tri=E2ngulo ret=E2ngulo. 2-) Um tonel est=E1 cheio de vinho puro. Cada dia s=E3o tirados dele dois ba= ldes,=20 substitu=EDdos por dois baldes de =E1gua. Ao cabo de seis dias h=E1 metade d= e vinho=20 e metade de =E1gua. Qual a capacidade do tonel ? P.S. : Partindo de um tonel cheio de =E1gua e acrescendo vinho, a resposta=20 seria a mesma ? Obrigado antecipadamente pelos esclarecimentos, Raul --part1_47.1f083ca7.2a4baeb1_boundary Content-Type: text/html; charset="ISO-8859-1" Content-Transfer-Encoding: quoted-printable     = ; Sauda=E7=F5es a todos,
      estou lendo algo sobre Tartaglia, C= ardan e Del Fiore onde encontrei problemas que gostaria de suas resolu=E7= =F5es:
1-) Cortar uma reta de comprimento dado em 3 segmentos com os quais seja= poss=EDvel construir um tri=E2ngulo ret=E2ngulo.
2-) Um tonel est=E1 cheio de vinho puro. Cada dia s=E3o tirados dele doi= s baldes, substitu=EDdos por dois baldes de =E1gua. Ao cabo de seis dias h= =E1 metade de vinho e metade de =E1gua. Qual a capacidade do tonel ?
P.S. : Partindo de um tonel cheio de =E1gua e acrescendo vinho, a respos= ta seria a mesma ?
      Obrigado antecipadamente pelos escl= arecimentos,
            = ;Raul
--part1_47.1f083ca7.2a4baeb1_boundary-- ========================================================================= Instruções para entrar na lista, sair da lista e usar a lista em http://www.mat.puc-rio.br/~nicolau/olimp/obm-l.html O administrador desta lista é ========================================================================= From owner-obm-l@sucuri.mat.puc-rio.br Wed Jun 26 22:28:35 2002 Return-Path: Received: (from majordom@localhost) by sucuri.mat.puc-rio.br (8.9.3/8.9.3) id WAA04196 for obm-l-list; Wed, 26 Jun 2002 22:28:28 -0300 Received: from mail.gmx.net (mail.gmx.net [213.165.64.20]) by sucuri.mat.puc-rio.br (8.9.3/8.9.3) with SMTP id WAA04192 for ; Wed, 26 Jun 2002 22:28:25 -0300 Received: (qmail 4436 invoked by uid 0); 27 Jun 2002 01:27:47 -0000 Received: from 200-206-211-169.dsl.telesp.net.br (HELO mentecapto.gmx.de) (200.206.211.169) by mail.gmx.net (mp001-rz3) with SMTP; 27 Jun 2002 01:27:47 -0000 Message-Id: <5.1.0.14.2.20020626220557.03b14380@pop.gmx.net> X-Sender: mentus@gmx.de@pop.gmx.net X-Mailer: QUALCOMM Windows Eudora Version 5.1 Date: Wed, 26 Jun 2002 22:25:58 -0300 To: obm-l@sucuri.mat.puc-rio.br From: "Fernando Henrique Ferraz P. da Rosa" Subject: [obm-l] Ajuda - Limite.... Mime-Version: 1.0 Content-Type: multipart/mixed; x-avg-checked=avg-ok-6EBF775D; boundary="=======69D75849=======" Sender: owner-obm-l@sucuri.mat.puc-rio.br Precedence: bulk Reply-To: obm-l@mat.puc-rio.br --=======69D75849======= Content-Type: text/plain; x-avg-checked=avg-ok-6EBF775D; charset=iso-8859-1; format=flowed Content-Transfer-Encoding: quoted-printable Estou tentando resolver esse limite faz tempo mas n=E3o est=E1= saindo=20 de jeito algum.. =C9 o seguinte: lim [x -> 0+] x^(tan(x=B2)). Meus esbo=E7os: x -> 0... tan(x=B2) -> 0.... temos 0^0... Colocando na forma exponencial: (exp(y) =3D e^(y)): x^tan(x=B2) =3D exp(ln(x^tan(x=B2)) =3D exp(tan(x=B2).ln(x)). Ficamos ent=E3o com o seguinte limite: lim [x-> 0+] tan(x=B2).ln(x). tan(x=B2) -> 0 ln(x) -> -infinito Temos entao 0.-infinito.. indetermina=E7=E3o... 'Transformando' isso numa fra=E7=E3o para poder usarmos= L'Hospital: a) Fazendo tan(x=B2).ln(x) =3D ln(x)/(1/tan(x=B2)) lim [x-> 0+] ln(x)/(1/tan(x=B2)) ln(x) -> -infinito 1/tan(x=B2)) =3D cotg(x=B2) -> infinito infinito/infinito outra indeterminacao.. aplicando= L'Hospital: lim [x-> 0+] ln(x)/cotg(x=B2) =3D lim [x->0+]=20 (1/x)/-2x.cossec=B2(x=B2) =3D lim [x-> 0+] 1/(-2x=B2cossec=B2(x=B2)) Agora temos -2x=B2 -> 0 e cossec=B2(x=B2) -> infinito... 0.infinito.. mais uma indeterminacao.... 1/0.infinito.. Nao podemos mais aplicar L'Hospital e sei la= =20 como sair daqui... b) Outra opcao serial fazer tan(x=B2).ln(x) =3D tan(x=B2)/(1/ln(x)), dai: lim [x->0+] tan(x=B2)/(1/ln(x)).. dai temos tan(x=B2) -> 0 1/ln(x) -> 0 0/0, indetermina=E7=E3o, aplicamos L'Hospital: lim [x->0+] tan(x=B2)/(1/ln(x)) =3D lim [x->0+]=20 2x.sec=B2(x=B2)/(-1/ln=B2(x).x) =3D lim [x->0+] 2x=B2.sec=B2(x=B2).ln=B2(x).x 2x=B2 -> 0 sec=B2x=B2 -> 1 ln=B2(x) -> infinito x -> 0... 0.1.0.infinito.. epa.. outra indetermina=E7=E3o... c)... j=E1 esgotei todas as id=E9ias que me vieram e ainda n=E3o consegui= sair=20 disso.. alguem tem alguma luz? BTW... a resposta =E9 1.. Ent=E3o esse limite (lim [x-> 0+] tan(x=B2).ln(x))= tem=20 que dar 0. "As long as a branch of science offers an abundance of problems, so long it is alive." David Hilbert. - []'s Fernando Henrique Ferraz Pereira da Rosa mentus@gmx.de Estat=EDstica USP [ http://www.linux.ime.usp.br/~feferraz ] --=======69D75849======= Content-Type: text/plain; charset=us-ascii; x-avg=cert; x-avg-checked=avg-ok-6EBF775D Content-Disposition: inline --- Outgoing mail is certified Virus Free. Checked by AVG anti-virus system (http://www.grisoft.com). Version: 6.0.371 / Virus Database: 206 - Release Date: 6/13/2002 --=======69D75849=======-- ========================================================================= Instruções para entrar na lista, sair da lista e usar a lista em http://www.mat.puc-rio.br/~nicolau/olimp/obm-l.html O administrador desta lista é ========================================================================= From owner-obm-l@sucuri.mat.puc-rio.br Wed Jun 26 23:26:10 2002 Return-Path: Received: (from majordom@localhost) by sucuri.mat.puc-rio.br (8.9.3/8.9.3) id XAA05180 for obm-l-list; Wed, 26 Jun 2002 23:25:56 -0300 Received: from smtp011.mail.yahoo.com (smtp011.mail.yahoo.com [216.136.173.31]) by sucuri.mat.puc-rio.br (8.9.3/8.9.3) with SMTP id XAA05175 for ; Wed, 26 Jun 2002 23:25:53 -0300 Received: from 200-204-17-219.dial-up.telesp.net.br (HELO e9i5z1) (caio?voznak@200.204.17.219 with login) by smtp.mail.vip.sc5.yahoo.com with SMTP; 27 Jun 2002 02:25:21 -0000 Message-ID: <001601c21cb0$7a058880$db11ccc8@e9i5z1> From: "Caio H. Voznak" To: References: <200206240156.g5O1uPf18858@Euler.impa.br> Subject: Re: [obm-l] Geometria interssante Date: Tue, 25 Jun 2002 22:26:15 -0300 MIME-Version: 1.0 Content-Type: multipart/alternative; boundary="----=_NextPart_000_0013_01C21C97.51A36F60" X-Priority: 3 X-MSMail-Priority: Normal X-Mailer: Microsoft Outlook Express 5.50.4133.2400 X-MIMEOLE: Produced By Microsoft MimeOLE V5.50.4133.2400 Sender: owner-obm-l@sucuri.mat.puc-rio.br Precedence: bulk Reply-To: obm-l@mat.puc-rio.br This is a multi-part message in MIME format. ------=_NextPart_000_0013_01C21C97.51A36F60 Content-Type: text/plain; charset="iso-8859-1" Content-Transfer-Encoding: quoted-printable Re: [obm-l] Geometria interssanteEduardo Wagner, Infelizmente tenetei resolver o problema indicado, mas n=E3o estou = chegando a solu=E7=E3o utilizando regua e compasso. Voc=EA poderia me = indicar materiais de referencia no uso desta curva, pois procurei mas = s=F3 achei esbo=E7os de sua forma. Abra=E7o, Caio.=20 ----- Original Message -----=20 From: Eduardo Wagner=20 To: obm-l@mat.puc-rio.br=20 Sent: Tuesday, June 25, 2002 10:54 PM Subject: Re: [obm-l] Geometria interssante Caio: Seu problema nao tem solucao com regua e compasso. Ele envolve uma curva chamada "conchoide de Nicomedes". Agora, um problema muito interessante e que tem solucao com regua e compasso eh o seguinte. "Determinar a semi-reta de origem P que, ao cortar os lados do angulo dado, forme um triangulo de perimetro d (dado). Abraco, Wagner. ---------- From: "Caio H. Voznak" To: Subject: [obm-l] Geometria interssante Date: Sat, Jun 22, 2002, 3:45 PM Por favor ser=E1 que alguem conhece um solu=E7=E3o para a seguinte = quest=E3o: =20 S=E3o dadas duas retas convergentes em um ponto O que formam um = angulo agudo teta entre si, tamb=E9m =E9 dado um ponto P localizado = abaixo das retas, ambos fixos, e uma medida d. =C9 pedido uma semireta = com in=EDcio em P e que corte ambas as retas convergentes obtendo a = medida d entre as retas. Segue um esbo=E7o em anexo. =20 =20 Um abra=E7o, Caio Voznak =20 --- Outgoing mail is certified Virus Free. Checked by AVG anti-virus system (http://www.grisoft.com). Version: 6.0.345 / Virus Database: 193 - Release Date: 9/4/2002 ------=_NextPart_000_0013_01C21C97.51A36F60 Content-Type: text/html; charset="iso-8859-1" Content-Transfer-Encoding: quoted-printable Re: [obm-l] Geometria interssante
Eduardo Wagner,
 
Infelizmente tenetei resolver o = problema indicado,=20 mas n=E3o estou chegando a solu=E7=E3o utilizando regua e compasso. = Voc=EA poderia me=20 indicar materiais de referencia no uso desta curva, pois procurei = mas s=F3=20 achei esbo=E7os de sua forma.
 
Abra=E7o,
 
Caio. 
----- Original Message -----
From:=20 Eduardo = Wagner
Sent: Tuesday, June 25, 2002 = 10:54=20 PM
Subject: Re: [obm-l] Geometria=20 interssante

Caio:

Seu problema nao tem solucao com regua e = compasso.=20 Ele envolve
uma curva chamada "conchoide de Nicomedes".
Agora, = um=20 problema muito interessante e que tem solucao com
regua e compasso = eh o=20 seguinte.
"Determinar a semi-reta de origem P que, ao cortar os = lados
do=20 angulo dado, forme um triangulo de perimetro d=20 (dado).

Abraco,

Wagner.

----------
From: "Caio = H.=20 Voznak" <caio_voznak@yahoo.com.br>
To:=20 <obm-l@mat.puc-rio.br>
Subject: [obm-l] Geometria=20 interssante
Date: Sat, Jun 22, 2002, 3:45 PM


Por favor ser=E1 que = alguem conhece=20 um solu=E7=E3o para a seguinte = quest=E3o:
 
S=E3o dadas duas retas convergentes em = um ponto O que=20 formam um angulo agudo teta entre si, tamb=E9m =E9 dado um ponto P = localizado=20 abaixo das retas, ambos fixos, e uma medida d. =C9 pedido uma = semireta com=20 in=EDcio em P e que corte ambas as retas convergentes obtendo a = medida d entre=20 as retas. Segue um esbo=E7o em anexo.=20   
 
Um=20 abra=E7o,
Caio Voznak
 

---
Outgoing mail is certified Virus = Free.
Checked by=20 AVG anti-virus system (http://www.grisoft.com
).
Version: = 6.0.345 /=20 Virus Database: 193 - Release Date:=20 9/4/2002

------=_NextPart_000_0013_01C21C97.51A36F60-- _________________________________________________________ Do You Yahoo!? Get your free @yahoo.com address at http://mail.yahoo.com ========================================================================= Instruções para entrar na lista, sair da lista e usar a lista em http://www.mat.puc-rio.br/~nicolau/olimp/obm-l.html O administrador desta lista é ========================================================================= From owner-obm-l@sucuri.mat.puc-rio.br Thu Jun 27 09:19:29 2002 Return-Path: Received: (from majordom@localhost) by sucuri.mat.puc-rio.br (8.9.3/8.9.3) id JAA09796 for obm-l-list; Thu, 27 Jun 2002 09:18:27 -0300 Received: from Euler.impa.br (euler.impa.br [147.65.1.3]) by sucuri.mat.puc-rio.br (8.9.3/8.9.3) with ESMTP id JAA09792 for ; Thu, 27 Jun 2002 09:18:25 -0300 Received: from obm-01 (obm-01.impa.br [147.65.2.170]) by Euler.impa.br (8.11.6/8.11.6) with SMTP id g5RCHvf25479 for ; Thu, 27 Jun 2002 09:17:57 -0300 (EST) Message-Id: <3.0.5.32.20020627091943.00808100@pop.impa.br> X-Sender: obm@pop.impa.br X-Mailer: QUALCOMM Windows Eudora Light Version 3.0.5 (32) Date: Thu, 27 Jun 2002 09:19:43 -0300 To: obm-l@mat.puc-rio.br From: Olimpiada Brasileira de Matematica Subject: [obm-l] Novidades. Mime-Version: 1.0 Content-Type: text/plain; charset="us-ascii" Sender: owner-obm-l@sucuri.mat.puc-rio.br Precedence: bulk Reply-To: obm-l@mat.puc-rio.br Caros(as) amigos(as) da lista: Ja estao no ar as versoes .doc e .htm da prova da XIII Olimpiada de Matematica do Cone Sul, tambem estao as provas de selecao e as listas de preparacao da competicao. Confiram no arquivo de provas da nossa pagina: http://www.obm.org.br/provas.htm Abracos, Nelly. ========================================================================= Instruções para entrar na lista, sair da lista e usar a lista em http://www.mat.puc-rio.br/~nicolau/olimp/obm-l.html O administrador desta lista é ========================================================================= From owner-obm-l@sucuri.mat.puc-rio.br Thu Jun 27 10:38:40 2002 Return-Path: Received: (from majordom@localhost) by sucuri.mat.puc-rio.br (8.9.3/8.9.3) id KAA11198 for obm-l-list; Thu, 27 Jun 2002 10:38:20 -0300 Received: from hotmail.com (law2-f105.hotmail.com [216.32.181.105]) by sucuri.mat.puc-rio.br (8.9.3/8.9.3) with ESMTP id KAA11186 for ; Thu, 27 Jun 2002 10:38:16 -0300 Received: from mail pickup service by hotmail.com with Microsoft SMTPSVC; Thu, 27 Jun 2002 06:37:46 -0700 Received: from 32.94.119.253 by lw2fd.hotmail.msn.com with HTTP; Thu, 27 Jun 2002 13:37:46 GMT X-Originating-IP: [32.94.119.253] From: "Paulo Santa Rita" To: obm-l@mat.puc-rio.br Subject: Re: [obm-l] Ajuda - Limite.... Date: Thu, 27 Jun 2002 13:37:46 +0000 Mime-Version: 1.0 Content-Type: text/plain; charset=iso-8859-1; format=flowed Message-ID: X-OriginalArrivalTime: 27 Jun 2002 13:37:46.0867 (UTC) FILETIME=[D2A3FC30:01C21DDF] Sender: owner-obm-l@sucuri.mat.puc-rio.br Precedence: bulk Reply-To: obm-l@mat.puc-rio.br Oi Fernando e demais colegas desta lista, Voce ja passou pela solucao diversas vezes, apenas nao percebeu isso. Os limites abaixo sao para X tendendo a zero pela direita : y=x^(tg(x^2)) => Ln(y)=tg(x^2)*ln(x) LIM Ln(y)=LIM [tg(x^2)*Ln(x)]=LIM[ Ln(x)/(1/tg(x^2)) ]= LIM Ln(y) = LIM[Ln(x)/cotg(x^2)] indeterminacao da forma INF/INF. Aplicando L'Hopital : LIM Ln(y)=LIM[ (1/x)/(-2x*cosec^2(x^2)) ] LIM Ln(y)=-LIM [(sen^2(x^2))/(2*(x^2)) ] LIM Ln(y)=-(1/2)LIM[(sen^2(x^2))/(x^2) ] indeterminacao da forma 0/0. Aplicando L'Hopital : LIM Ln(y)=-(1/2)LIM[(2*sen(x^2)*cos(x^2).2x)/(2x)] LIM Ln(y)=-LIM[sen(x^2)*cos(x^2)]=-LIM(sen(x^2))*LIM(cos(x^2)) LIM Ln(y) = -0*1 = 0 Ln LIM(y)=0 => LIM(y)= e^0 => LIM(y)=1 Ja que voce gosta de limites, fica a questao : Calcule : LIM [(arcsen(x)/x)^(1/x^2)] quando x -> 0 Nota : a resposta nao e raiz quadrada de "e". Um abraco Paulo Santa Rita 5,1035,270602 >From: "Fernando Henrique Ferraz P. da Rosa" >Reply-To: obm-l@mat.puc-rio.br >To: obm-l@sucuri.mat.puc-rio.br >Subject: [obm-l] Ajuda - Limite.... >Date: Wed, 26 Jun 2002 22:25:58 -0300 > > Estou tentando resolver esse limite faz tempo mas não está saindo >de jeito algum.. É o seguinte: > lim [x -> 0+] x^(tan(x²)). > >Meus esboços: > x -> 0... tan(x²) -> 0.... temos 0^0... > Colocando na forma exponencial: (exp(y) = e^(y)): > x^tan(x²) = exp(ln(x^tan(x²)) = exp(tan(x²).ln(x)). > Ficamos então com o seguinte limite: > lim [x-> 0+] tan(x²).ln(x). > tan(x²) -> 0 > ln(x) -> -infinito > Temos entao 0.-infinito.. indeterminação... > 'Transformando' isso numa fração para poder usarmos >L'Hospital: >a) Fazendo tan(x²).ln(x) = ln(x)/(1/tan(x²)) > lim [x-> 0+] ln(x)/(1/tan(x²)) > ln(x) -> -infinito > 1/tan(x²)) = cotg(x²) -> infinito > infinito/infinito outra indeterminacao.. aplicando >L'Hospital: > lim [x-> 0+] ln(x)/cotg(x²) = lim [x->0+] >(1/x)/-2x.cossec²(x²) = > lim [x-> 0+] 1/(-2x²cossec²(x²)) > Agora temos -2x² -> 0 > e cossec²(x²) -> infinito... > 0.infinito.. mais uma indeterminacao.... > 1/0.infinito.. Nao podemos mais aplicar L'Hospital e sei la >como sair daqui... > >b) Outra opcao serial fazer tan(x²).ln(x) = tan(x²)/(1/ln(x)), dai: > lim [x->0+] tan(x²)/(1/ln(x)).. > dai temos tan(x²) -> 0 > 1/ln(x) -> 0 > 0/0, indeterminação, aplicamos L'Hospital: > lim [x->0+] tan(x²)/(1/ln(x)) = lim [x->0+] >2x.sec²(x²)/(-1/ln²(x).x) = > lim [x->0+] 2x².sec²(x²).ln²(x).x > 2x² -> 0 > sec²x² -> 1 > ln²(x) -> infinito > x -> 0... > 0.1.0.infinito.. epa.. outra indeterminação... > >c)... já esgotei todas as idéias que me vieram e ainda não consegui sair >disso.. alguem tem alguma luz? > >BTW... a resposta é 1.. Então esse limite (lim [x-> 0+] tan(x²).ln(x)) tem >que dar 0. > > > >"As long as a branch of science offers an abundance of problems, > so long it is alive." > David Hilbert. > >- >[]'s >Fernando Henrique Ferraz Pereira da Rosa >mentus@gmx.de >Estatística USP [ http://www.linux.ime.usp.br/~feferraz ] > >--- >Outgoing mail is certified Virus Free. >Checked by AVG anti-virus system (http://www.grisoft.com). >Version: 6.0.371 / Virus Database: 206 - Release Date: 6/13/2002 _________________________________________________________________ Una-se ao maior serviço de email do mundo: o MSN Hotmail. http://www.hotmail.com ========================================================================= Instruções para entrar na lista, sair da lista e usar a lista em http://www.mat.puc-rio.br/~nicolau/olimp/obm-l.html O administrador desta lista é ========================================================================= From owner-obm-l@sucuri.mat.puc-rio.br Thu Jun 27 10:38:43 2002 Return-Path: Received: (from majordom@localhost) by sucuri.mat.puc-rio.br (8.9.3/8.9.3) id KAA11239 for obm-l-list; Thu, 27 Jun 2002 10:38:40 -0300 Received: from mat.puc-rio.br (IDENT:root@perere.mat.puc-rio.br [139.82.27.60]) by sucuri.mat.puc-rio.br (8.9.3/8.9.3) with ESMTP id KAA11222 for ; Thu, 27 Jun 2002 10:38:34 -0300 Received: from localhost (fredpalm@localhost) by mat.puc-rio.br (8.9.3/8.9.3) with ESMTP id CAA06199; Thu, 27 Jun 2002 02:49:54 -0300 Date: Thu, 27 Jun 2002 02:49:54 -0300 (BRT) From: Carlos Frederico Borges Palmeira To: obm-l@mat.puc-rio.br cc: obm-l@sucuri.mat.puc-rio.br Subject: Re: [obm-l] Ajuda - Limite.... In-Reply-To: <5.1.0.14.2.20020626220557.03b14380@pop.gmx.net> Message-ID: MIME-Version: 1.0 Content-Type: TEXT/PLAIN; charset=X-UNKNOWN Content-Transfer-Encoding: 8bit X-MIME-Autoconverted: from QUOTED-PRINTABLE to 8bit by sucuri.mat.puc-rio.br id KAA11223 Sender: owner-obm-l@sucuri.mat.puc-rio.br Precedence: bulk Reply-To: obm-l@mat.puc-rio.br oi fernando , veja ai em baixo uma maneira de fazer. Fred palmeira On Wed, 26 Jun 2002, Fernando Henrique Ferraz P. da Rosa wrote: > Estou tentando resolver esse limite faz tempo mas não está saindo > de jeito algum.. É o seguinte: > lim [x -> 0+] x^(tan(x²)). > > Meus esboços: > x -> 0... tan(x²) -> 0.... temos 0^0... > Colocando na forma exponencial: (exp(y) = e^(y)): > x^tan(x²) = exp(ln(x^tan(x²)) = exp(tan(x²).ln(x)). > Ficamos então com o seguinte limite: > lim [x-> 0+] tan(x²).ln(x). > tan(x²) -> 0 > ln(x) -> -infinito > Temos entao 0.-infinito.. indeterminação... > 'Transformando' isso numa fração para poder usarmos L'Hospital: > a) Fazendo tan(x²).ln(x) = ln(x)/(1/tan(x²)) calcule o limite de (x^2)ln(x) usando l'hopital na fracao lnx/(1/x^2). e' facil ver que da' zero. como tanx/x tem limite 1, tan(x^2).ln(x) tambem tem limite zero. (ha' detalhes a preencher) > lim [x-> 0+] ln(x)/(1/tan(x²)) > ln(x) -> -infinito > 1/tan(x²)) = cotg(x²) -> infinito > infinito/infinito outra indeterminacao.. aplicando L'Hospital: > lim [x-> 0+] ln(x)/cotg(x²) = lim [x->0+] > (1/x)/-2x.cossec²(x²) = > lim [x-> 0+] 1/(-2x²cossec²(x²)) > Agora temos -2x² -> 0 > e cossec²(x²) -> infinito... > 0.infinito.. mais uma indeterminacao.... > 1/0.infinito.. Nao podemos mais aplicar L'Hospital e sei la > como sair daqui... > > b) Outra opcao serial fazer tan(x²).ln(x) = tan(x²)/(1/ln(x)), dai: > lim [x->0+] tan(x²)/(1/ln(x)).. > dai temos tan(x²) -> 0 > 1/ln(x) -> 0 > 0/0, indeterminação, aplicamos L'Hospital: > lim [x->0+] tan(x²)/(1/ln(x)) = lim [x->0+] > 2x.sec²(x²)/(-1/ln²(x).x) = > lim [x->0+] 2x².sec²(x²).ln²(x).x > 2x² -> 0 > sec²x² -> 1 > ln²(x) -> infinito > x -> 0... > 0.1.0.infinito.. epa.. outra indeterminação... > > c)... já esgotei todas as idéias que me vieram e ainda não consegui sair > disso.. alguem tem alguma luz? > > BTW... a resposta é 1.. Então esse limite (lim [x-> 0+] tan(x²).ln(x)) tem > que dar 0. > > > > "As long as a branch of science offers an abundance of problems, > so long it is alive." > David Hilbert. > > - > []'s > Fernando Henrique Ferraz Pereira da Rosa > mentus@gmx.de > Estatística USP [ http://www.linux.ime.usp.br/~feferraz ] > ========================================================================= Instruções para entrar na lista, sair da lista e usar a lista em http://www.mat.puc-rio.br/~nicolau/olimp/obm-l.html O administrador desta lista é ========================================================================= From owner-obm-l@sucuri.mat.puc-rio.br Thu Jun 27 10:38:47 2002 Return-Path: Received: (from majordom@localhost) by sucuri.mat.puc-rio.br (8.9.3/8.9.3) id KAA11226 for obm-l-list; Thu, 27 Jun 2002 10:38:38 -0300 Received: from mat.puc-rio.br (IDENT:root@perere.mat.puc-rio.br [139.82.27.60]) by sucuri.mat.puc-rio.br (8.9.3/8.9.3) with ESMTP id KAA11219 for ; Thu, 27 Jun 2002 10:38:34 -0300 Received: from localhost (fredpalm@localhost) by mat.puc-rio.br (8.9.3/8.9.3) with ESMTP id CAA06199; Thu, 27 Jun 2002 02:49:54 -0300 Date: Thu, 27 Jun 2002 02:49:54 -0300 (BRT) From: Carlos Frederico Borges Palmeira To: obm-l@mat.puc-rio.br cc: obm-l@sucuri.mat.puc-rio.br Subject: Re: [obm-l] Ajuda - Limite.... In-Reply-To: <5.1.0.14.2.20020626220557.03b14380@pop.gmx.net> Message-ID: MIME-Version: 1.0 Content-Type: TEXT/PLAIN; charset=X-UNKNOWN Content-Transfer-Encoding: 8bit X-MIME-Autoconverted: from QUOTED-PRINTABLE to 8bit by sucuri.mat.puc-rio.br id KAA11221 Sender: owner-obm-l@sucuri.mat.puc-rio.br Precedence: bulk Reply-To: obm-l@mat.puc-rio.br oi fernando , veja ai em baixo uma maneira de fazer. Fred palmeira On Wed, 26 Jun 2002, Fernando Henrique Ferraz P. da Rosa wrote: > Estou tentando resolver esse limite faz tempo mas não está saindo > de jeito algum.. É o seguinte: > lim [x -> 0+] x^(tan(x²)). > > Meus esboços: > x -> 0... tan(x²) -> 0.... temos 0^0... > Colocando na forma exponencial: (exp(y) = e^(y)): > x^tan(x²) = exp(ln(x^tan(x²)) = exp(tan(x²).ln(x)). > Ficamos então com o seguinte limite: > lim [x-> 0+] tan(x²).ln(x). > tan(x²) -> 0 > ln(x) -> -infinito > Temos entao 0.-infinito.. indeterminação... > 'Transformando' isso numa fração para poder usarmos L'Hospital: > a) Fazendo tan(x²).ln(x) = ln(x)/(1/tan(x²)) calcule o limite de (x^2)ln(x) usando l'hopital na fracao lnx/(1/x^2). e' facil ver que da' zero. como tanx/x tem limite 1, tan(x^2).ln(x) tambem tem limite zero. (ha' detalhes a preencher) > lim [x-> 0+] ln(x)/(1/tan(x²)) > ln(x) -> -infinito > 1/tan(x²)) = cotg(x²) -> infinito > infinito/infinito outra indeterminacao.. aplicando L'Hospital: > lim [x-> 0+] ln(x)/cotg(x²) = lim [x->0+] > (1/x)/-2x.cossec²(x²) = > lim [x-> 0+] 1/(-2x²cossec²(x²)) > Agora temos -2x² -> 0 > e cossec²(x²) -> infinito... > 0.infinito.. mais uma indeterminacao.... > 1/0.infinito.. Nao podemos mais aplicar L'Hospital e sei la > como sair daqui... > > b) Outra opcao serial fazer tan(x²).ln(x) = tan(x²)/(1/ln(x)), dai: > lim [x->0+] tan(x²)/(1/ln(x)).. > dai temos tan(x²) -> 0 > 1/ln(x) -> 0 > 0/0, indeterminação, aplicamos L'Hospital: > lim [x->0+] tan(x²)/(1/ln(x)) = lim [x->0+] > 2x.sec²(x²)/(-1/ln²(x).x) = > lim [x->0+] 2x².sec²(x²).ln²(x).x > 2x² -> 0 > sec²x² -> 1 > ln²(x) -> infinito > x -> 0... > 0.1.0.infinito.. epa.. outra indeterminação... > > c)... já esgotei todas as idéias que me vieram e ainda não consegui sair > disso.. alguem tem alguma luz? > > BTW... a resposta é 1.. Então esse limite (lim [x-> 0+] tan(x²).ln(x)) tem > que dar 0. > > > > "As long as a branch of science offers an abundance of problems, > so long it is alive." > David Hilbert. > > - > []'s > Fernando Henrique Ferraz Pereira da Rosa > mentus@gmx.de > Estatística USP [ http://www.linux.ime.usp.br/~feferraz ] > ========================================================================= Instruções para entrar na lista, sair da lista e usar a lista em http://www.mat.puc-rio.br/~nicolau/olimp/obm-l.html O administrador desta lista é ========================================================================= From owner-obm-l@sucuri.mat.puc-rio.br Thu Jun 27 10:53:22 2002 Return-Path: Received: (from majordom@localhost) by sucuri.mat.puc-rio.br (8.9.3/8.9.3) id KAA11689 for obm-l-list; Thu, 27 Jun 2002 10:53:14 -0300 Received: from mat.puc-rio.br (IDENT:root@perere.mat.puc-rio.br [139.82.27.60]) by sucuri.mat.puc-rio.br (8.9.3/8.9.3) with ESMTP id KAA11685 for ; Thu, 27 Jun 2002 10:53:12 -0300 Received: from localhost (fredpalm@localhost) by mat.puc-rio.br (8.9.3/8.9.3) with ESMTP id DAA06211 for ; Thu, 27 Jun 2002 03:04:33 -0300 Date: Thu, 27 Jun 2002 03:04:32 -0300 (BRT) From: Carlos Frederico Borges Palmeira To: obm-l@mat.puc-rio.br Subject: Re: [obm-l] Tartaglia In-Reply-To: <47.1f083ca7.2a4baeb1@aol.com> Message-ID: MIME-Version: 1.0 Content-Type: TEXT/PLAIN; charset=X-UNKNOWN Content-Transfer-Encoding: 8bit X-MIME-Autoconverted: from QUOTED-PRINTABLE to 8bit by sucuri.mat.puc-rio.br id KAA11686 Sender: owner-obm-l@sucuri.mat.puc-rio.br Precedence: bulk Reply-To: obm-l@mat.puc-rio.br uma solucao geometrica para o problema 1: divida o segmento dado em segmentos proporcionais a 3 ,4, 5, ou qualquer outros 3 valores que sejam lados de um triangulo retangulo. Fred palmeira On Wed, 26 Jun 2002 Euraul@aol.com wrote: > Saudações a todos, > estou lendo algo sobre Tartaglia, Cardan e Del Fiore onde encontrei > problemas que gostaria de suas resoluções: > 1-) Cortar uma reta de comprimento dado em 3 segmentos com os quais seja > possível construir um triângulo retângulo. > 2-) Um tonel está cheio de vinho puro. Cada dia são tirados dele dois baldes, > substituídos por dois baldes de água. Ao cabo de seis dias há metade de vinho > e metade de água. Qual a capacidade do tonel ? > P.S. : Partindo de um tonel cheio de água e acrescendo vinho, a resposta > seria a mesma ? > Obrigado antecipadamente pelos esclarecimentos, > Raul > ========================================================================= Instruções para entrar na lista, sair da lista e usar a lista em http://www.mat.puc-rio.br/~nicolau/olimp/obm-l.html O administrador desta lista é ========================================================================= From owner-obm-l@sucuri.mat.puc-rio.br Thu Jun 27 14:55:27 2002 Return-Path: Received: (from majordom@localhost) by sucuri.mat.puc-rio.br (8.9.3/8.9.3) id OAA16978 for obm-l-list; Thu, 27 Jun 2002 14:55:15 -0300 Received: from fgvrj23.fgv.br (fgvrj23.fgv.br [200.20.164.23]) by sucuri.mat.puc-rio.br (8.9.3/8.9.3) with ESMTP id OAA16972 for ; Thu, 27 Jun 2002 14:55:12 -0300 Received: by FGVRJ23 with Internet Mail Service (5.5.2655.55) id ; Thu, 27 Jun 2002 14:55:42 -0300 Message-ID: <3BE65222F383D611BE1E00D0B7B60A55E02590@FGVRJ23> From: Ralph Teixeira To: "'obm-l@mat.puc-rio.br'" Subject: RES: [obm-l] Helps! Date: Thu, 27 Jun 2002 14:55:42 -0300 MIME-Version: 1.0 X-Mailer: Internet Mail Service (5.5.2655.55) Content-Type: text/plain; charset="iso-8859-1" Content-Transfer-Encoding: 8bit X-MIME-Autoconverted: from quoted-printable to 8bit by sucuri.mat.puc-rio.br id OAA16973 Sender: owner-obm-l@sucuri.mat.puc-rio.br Precedence: bulk Reply-To: obm-l@mat.puc-rio.br Hmmm... Vejamos. Seja p o numero de paginas no album. Entao o primeiro colecionador tem 21p selos, e o segundo tem b=500-21p selos. Colocando 20 por pagina, o segundo colecionador poe 20p selos, mas sobram selos. Entao b>=20p. Usando (p-1) paginas com 23 por pagina, ele colocaria 23(p-1) selos, que eh mais do que ele tem. Entao b<=23(p-1) Em suma: 20p <= 500-21p <= 23(p-1) A primeira dah 41p <= 500, isto eh, p <= 12. A segunda dah 44p >= 523, isto eh, p >= 12. Concluimos que p=12, que tem 1,2,3,4,6 e 12 como divisores (6 divisores naturais). A proposito, o primeiro colecionador tem 21p=252 selos e o segundo tem 500-252 = 248 selos. Abraco, Ralph -----Mensagem original----- De: SSayajinGoten@aol.com [mailto:SSayajinGoten@aol.com] Enviada em: domingo, 23 de junho de 2002 21:12 Para: obm-l@mat.puc-rio.br Assunto: [obm-l] Helps! Wells, não estou conseguindo fazer uma questão que caio no simulado do meu curso, será que alguem poderia me ajudar? Dois colecionadores de selos têm juntos 500 selos. Cada colecionador comprou um álbum para colocar seus selos. Os dois álbuns eram idênticos, tendo o mesmo número de páginas. Se o primeiro colecionador colocar exatamente 21 selos em cada página, ele vai conseguir colocar todos os selos e usar todas as páginas do álbum. Se o segundo colecionador colocar 20 selos em cada página do álbum ,sobrarão alguns selos, caso ele coloque 23 selos em cada página, sobra pelo menos uma página,totalmente vazia,podendo haver ainda uma outra página com menos de 23 selos. o numero de páginas que há no álbum possui: a)2 divisores naturais b)4 divisores naturais c)6 divisores naturais d)8 divisores naturais e)10 divisores naturais ========================================================================= Instruções para entrar na lista, sair da lista e usar a lista em http://www.mat.puc-rio.br/~nicolau/olimp/obm-l.html O administrador desta lista é ========================================================================= From owner-obm-l@sucuri.mat.puc-rio.br Thu Jun 27 15:02:59 2002 Return-Path: Received: (from majordom@localhost) by sucuri.mat.puc-rio.br (8.9.3/8.9.3) id PAA17240 for obm-l-list; Thu, 27 Jun 2002 15:02:57 -0300 Received: from fgvrj23.fgv.br (fgvrj23.fgv.br [200.20.164.23]) by sucuri.mat.puc-rio.br (8.9.3/8.9.3) with ESMTP id PAA17236 for ; Thu, 27 Jun 2002 15:02:55 -0300 Received: by FGVRJ23 with Internet Mail Service (5.5.2655.55) id ; Thu, 27 Jun 2002 15:03:23 -0300 Message-ID: <3BE65222F383D611BE1E00D0B7B60A55E02591@FGVRJ23> From: Ralph Teixeira To: "'obm-l@mat.puc-rio.br'" Subject: RES: [obm-l] Ajuda - Limite.... Date: Thu, 27 Jun 2002 15:03:20 -0300 MIME-Version: 1.0 X-Mailer: Internet Mail Service (5.5.2655.55) Content-Type: text/plain; charset="iso-8859-1" Content-Transfer-Encoding: 8bit X-MIME-Autoconverted: from quoted-printable to 8bit by sucuri.mat.puc-rio.br id PAA17237 Sender: owner-obm-l@sucuri.mat.puc-rio.br Precedence: bulk Reply-To: obm-l@mat.puc-rio.br Use a primeira opção. Quando chegar à cossecante, lembre-se que cossec(x^2) = 1/sin(x^2). Substitua lá, veja a indeterminação do tipo 0/0 e continue. Vai dar certo. Abraço, Ralph -----Mensagem original----- De: Fernando Henrique Ferraz P. da Rosa [mailto:mentus@gmx.de] Enviada em: quarta-feira, 26 de junho de 2002 22:26 Para: obm-l@sucuri.mat.puc-rio.br Assunto: [obm-l] Ajuda - Limite.... Estou tentando resolver esse limite faz tempo mas não está saindo de jeito algum.. É o seguinte: lim [x -> 0+] x^(tan(x²)). Meus esboços: x -> 0... tan(x²) -> 0.... temos 0^0... Colocando na forma exponencial: (exp(y) = e^(y)): x^tan(x²) = exp(ln(x^tan(x²)) = exp(tan(x²).ln(x)). Ficamos então com o seguinte limite: lim [x-> 0+] tan(x²).ln(x). tan(x²) -> 0 ln(x) -> -infinito Temos entao 0.-infinito.. indeterminação... 'Transformando' isso numa fração para poder usarmos L'Hospital: a) Fazendo tan(x²).ln(x) = ln(x)/(1/tan(x²)) lim [x-> 0+] ln(x)/(1/tan(x²)) ln(x) -> -infinito 1/tan(x²)) = cotg(x²) -> infinito infinito/infinito outra indeterminacao.. aplicando L'Hospital: lim [x-> 0+] ln(x)/cotg(x²) = lim [x->0+] (1/x)/-2x.cossec²(x²) = lim [x-> 0+] 1/(-2x²cossec²(x²)) Agora temos -2x² -> 0 e cossec²(x²) -> infinito... 0.infinito.. mais uma indeterminacao.... 1/0.infinito.. Nao podemos mais aplicar L'Hospital e sei la como sair daqui... b) Outra opcao serial fazer tan(x²).ln(x) = tan(x²)/(1/ln(x)), dai: lim [x->0+] tan(x²)/(1/ln(x)).. dai temos tan(x²) -> 0 1/ln(x) -> 0 0/0, indeterminação, aplicamos L'Hospital: lim [x->0+] tan(x²)/(1/ln(x)) = lim [x->0+] 2x.sec²(x²)/(-1/ln²(x).x) = lim [x->0+] 2x².sec²(x²).ln²(x).x 2x² -> 0 sec²x² -> 1 ln²(x) -> infinito x -> 0... 0.1.0.infinito.. epa.. outra indeterminação... c)... já esgotei todas as idéias que me vieram e ainda não consegui sair disso.. alguem tem alguma luz? BTW... a resposta é 1.. Então esse limite (lim [x-> 0+] tan(x²).ln(x)) tem que dar 0. "As long as a branch of science offers an abundance of problems, so long it is alive." David Hilbert. - []'s Fernando Henrique Ferraz Pereira da Rosa mentus@gmx.de Estatística USP [ http://www.linux.ime.usp.br/~feferraz ] ========================================================================= Instruções para entrar na lista, sair da lista e usar a lista em http://www.mat.puc-rio.br/~nicolau/olimp/obm-l.html O administrador desta lista é ========================================================================= From owner-obm-l@sucuri.mat.puc-rio.br Thu Jun 27 23:45:09 2002 Return-Path: Received: (from majordom@localhost) by sucuri.mat.puc-rio.br (8.9.3/8.9.3) id XAA24555 for obm-l-list; Thu, 27 Jun 2002 23:44:24 -0300 Received: from Euler.impa.br (euler.impa.br [147.65.1.3]) by sucuri.mat.puc-rio.br (8.9.3/8.9.3) with ESMTP id XAA24551 for ; Thu, 27 Jun 2002 23:44:21 -0300 Received: from [147.65.11.6] (dial06.impa.br [147.65.11.6]) by Euler.impa.br (8.11.6/8.11.6) with ESMTP id g5S2hpf25586 for ; Thu, 27 Jun 2002 23:43:52 -0300 (EST) Message-Id: <200206280243.g5S2hpf25586@Euler.impa.br> X-Mailer: Microsoft Outlook Express Macintosh Edition - 4.5 (0410) Date: Sat, 29 Jun 2002 23:41:53 -0300 Subject: Re: [obm-l] Geometria interssante From: "Eduardo Wagner" To: obm-l@mat.puc-rio.br Mime-version: 1.0 X-Priority: 3 Content-type: multipart/alternative; boundary="MS_Mac_OE_3108238913_49487_MIME_Part" Sender: owner-obm-l@sucuri.mat.puc-rio.br Precedence: bulk Reply-To: obm-l@mat.puc-rio.br > THIS MESSAGE IS IN MIME FORMAT. Since your mail reader does not understand this format, some or all of this message may not be legible. --MS_Mac_OE_3108238913_49487_MIME_Part Content-type: text/plain; charset="ISO-8859-1" Content-transfer-encoding: quoted-printable Caro Caio: O problema que sugeri nao necessita de nenhuma curva. Ele se resolve com a seguinte propriedade: Em um tri=E2ngulo ABC, a circunfer=EAncia exinscrita relativa ao lado BC tangencia a reta AB em D. Ent=E3o AD =E9 igual ao semiper=EDmetro do tri=E2ngulo ABC. Abraco, W. ---------- From: "Caio H. Voznak" To: Subject: Re: [obm-l] Geometria interssante Date: Tue, Jun 25, 2002, 10:26 PM Eduardo Wagner, Infelizmente tenetei resolver o problema indicado, mas n=E3o estou chegando a solu=E7=E3o utilizando regua e compasso. Voc=EA poderia me indicar materiais de referencia no uso desta curva, pois procurei mas s=F3 achei esbo=E7os de sua forma. Abra=E7o, Caio. ----- Original Message ----- From: Eduardo Wagner To: obm-l@mat.puc-rio.br Sent: Tuesday, June 25, 2002 10:54 PM Subject: Re: [obm-l] Geometria interssante Caio: Seu problema nao tem solucao com regua e compasso. Ele envolve uma curva chamada "conchoide de Nicomedes". Agora, um problema muito interessante e que tem solucao com regua e compasso eh o seguinte. "Determinar a semi-reta de origem P que, ao cortar os lados do angulo dado, forme um triangulo de perimetro d (dado). Abraco, Wagner. ---------- From: "Caio H. Voznak" To: Subject: [obm-l] Geometria interssante Date: Sat, Jun 22, 2002, 3:45 PM Por favor ser=E1 que alguem conhece um solu=E7=E3o para a seguinte quest=E3o: S=E3o dadas duas retas convergentes em um ponto O que formam um angulo agudo teta entre si, tamb=E9m =E9 dado um ponto P localizado abaixo das retas, ambos fixos, e uma medida d. =C9 pedido uma semireta com in=EDcio em P e que corte ambas as retas convergentes obtendo a medida d entre as retas. Segue um esbo=E7o em anexo. Um abra=E7o, Caio Voznak --- Outgoing mail is certified Virus Free. Checked by AVG anti-virus system (http://www.grisoft.com). Version: 6.0.345 / Virus Database: 193 - Release Date: 9/4/2002 --MS_Mac_OE_3108238913_49487_MIME_Part Content-type: text/html; charset="ISO-8859-1" Content-transfer-encoding: quoted-printable Re: [obm-l] Geometria interssante Caro Caio:

O problema que sugeri nao necessita de nenhuma curva.
Ele se resolve com a seguinte propriedade:
Em um tri=E2ngulo ABC, a circunfer=EAncia exinscrita relativa
ao lado BC tangencia a reta AB em D. Ent=E3o AD =E9 igual
ao semiper=EDmetro do tri=E2ngulo ABC.

Abraco,

W.

----------
From: "Caio H. Voznak" <caio_voznak@yahoo.com.br>
To: <obm-l@mat.puc-rio.br>
Subject: Re: [obm-l] Geometria interssante
Date: Tue, Jun 25, 2002, 10:26 PM


Eduardo Wagner,
 
Infelizmente tenetei resolver o problema = indicado, mas n=E3o estou chegando a solu=E7=E3o utilizando regua e compasso. Voc=EA= poderia me indicar materiais de referencia no uso desta curva, pois procure= i mas s=F3 achei esbo=E7os de sua forma.
 
Abra=E7o,
 
Caio.
----- Original Message -----
From: Eduardo Wagner <mailto:wagner@impa= .br>  
To: obm-l@mat.puc-rio.br <mailto:obm-l@mat.puc-rio.br>  
Sent: Tuesday, June 25, 2002 10:54 PM
Subject: Re: [obm-l] Geometria interssante

Caio:

Seu problema nao tem solucao com regua e compasso. Ele envolve
uma curva chamada "conchoide de Nicomedes".
Agora, um problema muito interessante e que tem solucao com
regua e compasso eh o seguinte.
"Determinar a semi-reta de origem P que, ao cortar os lados
do angulo dado, forme um triangulo de perimetro d (dado).

Abraco,

Wagner.

----------
From: "Caio H. Voznak" <caio_voznak@y= ahoo.com.br>
To: <obm-l@mat.puc-rio.br>
Subject: [obm-l] Geometria interssante
Date: Sat, Jun 22, 2002, 3:45 PM


Por favor ser=E1 que alguem con= hece um solu=E7=E3o para a seguinte quest=E3o:
 
S=E3o dadas duas retas convergentes em um p= onto O que formam um angulo agudo teta entre si, tamb=E9m =E9 dado um ponto P lo= calizado abaixo das retas, ambos fixos, e uma medida d. =C9 pedido uma semiret= a com in=EDcio em P e que corte ambas as retas convergentes obtendo a medida d= entre as retas. Segue um esbo=E7o em anexo.   
 
Um abra=E7o,
Caio Voznak
 

---
Outgoing mail is certified Virus Free.
Checked by AVG anti-virus system (http://www.griso= ft.com).
Version: 6.0.345 / Virus Database: 193 - Release Date: 9/4/2002


--MS_Mac_OE_3108238913_49487_MIME_Part-- ========================================================================= Instruções para entrar na lista, sair da lista e usar a lista em http://www.mat.puc-rio.br/~nicolau/olimp/obm-l.html O administrador desta lista é ========================================================================= From owner-obm-l@sucuri.mat.puc-rio.br Fri Jun 28 09:54:12 2002 Return-Path: Received: (from majordom@localhost) by sucuri.mat.puc-rio.br (8.9.3/8.9.3) id JAA29600 for obm-l-list; Fri, 28 Jun 2002 09:53:49 -0300 Received: from Euler.impa.br (euler.impa.br [147.65.1.3]) by sucuri.mat.puc-rio.br (8.9.3/8.9.3) with ESMTP id JAA29595 for ; Fri, 28 Jun 2002 09:53:47 -0300 Received: from obm-01 (obm-01.impa.br [147.65.2.170]) by Euler.impa.br (8.11.6/8.11.6) with SMTP id g5SCrLf04426 for ; Fri, 28 Jun 2002 09:53:22 -0300 (EST) Message-Id: <3.0.5.32.20020628095510.00844480@pop.impa.br> X-Sender: obm@pop.impa.br X-Mailer: QUALCOMM Windows Eudora Light Version 3.0.5 (32) Date: Fri, 28 Jun 2002 09:55:10 -0300 To: obm-l@mat.puc-rio.br From: Olimpiada Brasileira de Matematica Subject: [obm-l] Selecionados VIII Olimpiada de Maio. Mime-Version: 1.0 Content-Type: text/plain; charset="us-ascii" Sender: owner-obm-l@sucuri.mat.puc-rio.br Precedence: bulk Reply-To: obm-l@mat.puc-rio.br Caros(as) amigos(as) da lista: Envio a relacao de selecionados para VIII Olimpiada de Maio. (as provas destes alunos foram encaminhadas para a Comissao Organizadora - FICOM - na Argentina, onde serao novamente avaliadas para o resultado final. Abracos, Nelly. Nivel 1 ******** Eduardo Fischer Encantado-RS Pedro Nogueira Machado Rio de Janeiro-RJ Andre Marcio de Lima Curvello Goiania-GO Katja Stephanie Ried Valinhos-SP Enzo Haruo Hiraoka Moriyama Sao Paulo-SP Anderson Gleryston Silva Sousa Campina Grande-PB Mariana Rasser Brolezzi Santo Andre-SP Arthur R. de Oliveira Sobral SJ. dos Campos-SP Cassio Kendi Takamori SJ. dos Campos-SP Diogo B. Moraes Morant de Holanda Rio de Janeiro-RJ **************************************************** Nivel 2 ******* Fabio Dias Moreira Rio de Janeiro-RJ Guilherme Rodrigues Salermo Goiania-GO Telmo Luis Correa Junior Santo Andre-SP Thiago Costa Leite Santos Sao Paulo-SP Andre Rodrigues Salermo Goiania-GO Henry Wei Cheng Hsu Sao Paulo-SP Rodrigo Aguiar Pinheiro Fortaleza-CE Rafael Marini Silva Vila Velha-ES Larissa Rodrigues Ribeiro Fortaleza-CE Douglas Bokliang Ang Cunha SJ. dos Campos-SP **************************************************** ========================================================================= Instruções para entrar na lista, sair da lista e usar a lista em http://www.mat.puc-rio.br/~nicolau/olimp/obm-l.html O administrador desta lista é ========================================================================= From owner-obm-l@sucuri.mat.puc-rio.br Fri Jun 28 11:07:42 2002 Return-Path: Received: (from majordom@localhost) by sucuri.mat.puc-rio.br (8.9.3/8.9.3) id LAA30993 for obm-l-list; Fri, 28 Jun 2002 11:07:18 -0300 Received: from Euler.impa.br (euler.impa.br [147.65.1.3]) by sucuri.mat.puc-rio.br (8.9.3/8.9.3) with ESMTP id LAA30989 for ; Fri, 28 Jun 2002 11:07:11 -0300 Received: from obm-01 (obm-01.impa.br [147.65.2.170]) by Euler.impa.br (8.11.6/8.11.6) with SMTP id g5SE6hf09817 for ; Fri, 28 Jun 2002 11:06:43 -0300 (EST) Message-Id: <3.0.5.32.20020628110831.0083c4b0@pop.impa.br> X-Sender: obm@pop.impa.br X-Mailer: QUALCOMM Windows Eudora Light Version 3.0.5 (32) Date: Fri, 28 Jun 2002 11:08:31 -0300 To: obm-l@mat.puc-rio.br From: Olimpiada Brasileira de Matematica Subject: [obm-l] Resultado XIII Olimpiada do Cone Sul. Mime-Version: 1.0 Content-Type: text/plain; charset="us-ascii" Sender: owner-obm-l@sucuri.mat.puc-rio.br Precedence: bulk Reply-To: obm-l@mat.puc-rio.br Caros(as) amigos(as) da lista: Excelente resultado brasileiro na XIII Olimpiada do Cone Sul. BRA1: Alex Correa Abreu (Niteroi-RJ) - Medalha de Ouro BRA2: Israel Dourado Carrah (Fortaleza-CE) - Medalha de Bronze BRA3: Larissa C. Queiroz de Lima (Fortaleza-CE) - Medalha de Ouro BRA4: Rafael Daigo Hirama (Campinas - SP) - Medalha de Ouro Abracos, Nelly. ========================================================================= Instruções para entrar na lista, sair da lista e usar a lista em http://www.mat.puc-rio.br/~nicolau/olimp/obm-l.html O administrador desta lista é ========================================================================= From owner-obm-l@sucuri.mat.puc-rio.br Fri Jun 28 12:37:29 2002 Return-Path: Received: (from majordom@localhost) by sucuri.mat.puc-rio.br (8.9.3/8.9.3) id MAA00336 for obm-l-list; Fri, 28 Jun 2002 12:36:48 -0300 Received: from web20416.mail.yahoo.com (web20404.mail.yahoo.com [66.163.169.92]) by sucuri.mat.puc-rio.br (8.9.3/8.9.3) with SMTP id MAA00332 for ; Fri, 28 Jun 2002 12:36:45 -0300 Message-ID: <20020628153619.81715.qmail@web20416.mail.yahoo.com> Received: from [200.206.103.3] by web20404.mail.yahoo.com via HTTP; Fri, 28 Jun 2002 12:36:19 ART Date: Fri, 28 Jun 2002 12:36:19 -0300 (ART) From: =?iso-8859-1?q?Johann=20Dirichlet?= Subject: [obm-l] Mais dúvidas em Teorema Chinês dos Restos e outros To: obm-l@mat.puc-rio.br MIME-Version: 1.0 Content-Type: text/plain; charset=iso-8859-1 Content-Transfer-Encoding: 8bit Sender: owner-obm-l@sucuri.mat.puc-rio.br Precedence: bulk Reply-To: obm-l@mat.puc-rio.br Oi turma!!!!To com um problema quase milenar aqui. "Prove que existe pelo menos um inteiro k tal que 1+k*(2^n) nunca seja primo para qualquer n" Pelo que eu saiba foi o Wroclav Sierpinski o primeiro a provar isso,usando o fato de que Fermat estava errado.E tambem se eu nao me engano deve ter outro k para o qual 1-k*(2^n) sempre e composto.Mas ate agora eu nao tive a menor ideia de como se faz isso(alias nunca vi um problema de teorema dos restos na minha vida). Qualquer ajuda eu agradeço. Peterdirichlet _______________________________________________________________________ Copa 2002 Yahoo! - Patrocinador oficial da Copa do Mundo da FIFA 2002 http://br.sports.yahoo.com/fifaworldcup/ ========================================================================= Instruções para entrar na lista, sair da lista e usar a lista em http://www.mat.puc-rio.br/~nicolau/olimp/obm-l.html O administrador desta lista é ========================================================================= From owner-obm-l@sucuri.mat.puc-rio.br Fri Jun 28 21:56:14 2002 Return-Path: Received: (from majordom@localhost) by sucuri.mat.puc-rio.br (8.9.3/8.9.3) id VAA08007 for obm-l-list; Fri, 28 Jun 2002 21:56:02 -0300 Received: from smtp.ieg.com.br (11.139.226.200.in-addr.arpa.ig.com.br [200.226.139.11] (may be forged)) by sucuri.mat.puc-rio.br (8.9.3/8.9.3) with ESMTP id VAA08003 for ; Fri, 28 Jun 2002 21:56:01 -0300 Received: from igor ([200.222.226.217]) by smtp.ieg.com.br (IeG relay/8.9.3) with SMTP id g5T0phqt069954 for ; Fri, 28 Jun 2002 21:51:57 -0300 (BRT) Message-ID: <001601c21f07$a095d000$d9e2dec8@igor> From: "Igor Castro" To: "obm-lista" Subject: [obm-l] biquadrada... Date: Fri, 28 Jun 2002 21:55:11 -0300 MIME-Version: 1.0 Content-Type: multipart/alternative; boundary="----=_NextPart_000_0013_01C21EEE.79C81280" X-Priority: 3 X-MSMail-Priority: Normal X-Mailer: Microsoft Outlook Express 5.00.2615.200 X-MimeOLE: Produced By Microsoft MimeOLE V5.00.2615.200 Sender: owner-obm-l@sucuri.mat.puc-rio.br Precedence: bulk Reply-To: obm-l@mat.puc-rio.br This is a multi-part message in MIME format. ------=_NextPart_000_0013_01C21EEE.79C81280 Content-Type: text/plain; charset="iso-8859-1" Content-Transfer-Encoding: quoted-printable Ol=E1 colegas da lista, podem me dar uma ajuda nesse problema?=20 Uma equa=E7=E3o biquadrada de coeficientes inteiros, cuja soma desses = coeficientes =E9 zero, tem como uma de suas ra=EDzes sqrt(3). O produto = das ra=EDzes dessa equa=E7=E3o =E9: eu achei 3... mas um amigo n=E3o confirmou o resultado... agrade=E7o = desde j=E1.. []'s Igor.. ------=_NextPart_000_0013_01C21EEE.79C81280 Content-Type: text/html; charset="iso-8859-1" Content-Transfer-Encoding: quoted-printable
Ol=E1 colegas da lista,
podem me dar uma ajuda nesse = problema? 

Uma=20 equa=E7=E3o biquadrada de coeficientes inteiros, cuja soma desses = coeficientes =E9=20 zero, tem como uma de suas ra=EDzes sqrt(3). O produto das ra=EDzes = dessa equa=E7=E3o=20 =E9:

eu=20 achei 3... mas um amigo n=E3o confirmou o resultado... agrade=E7o desde=20 j=E1..

[]'s=20 Igor..

------=_NextPart_000_0013_01C21EEE.79C81280-- ========================================================================= Instruções para entrar na lista, sair da lista e usar a lista em http://www.mat.puc-rio.br/~nicolau/olimp/obm-l.html O administrador desta lista é ========================================================================= From owner-obm-l@sucuri.mat.puc-rio.br Sat Jun 29 00:39:03 2002 Return-Path: Received: (from majordom@localhost) by sucuri.mat.puc-rio.br (8.9.3/8.9.3) id AAA09752 for obm-l-list; Sat, 29 Jun 2002 00:36:48 -0300 Received: from smtp018.mail.yahoo.com (smtp018.mail.yahoo.com [216.136.174.115]) by sucuri.mat.puc-rio.br (8.9.3/8.9.3) with SMTP id AAA09748 for ; Sat, 29 Jun 2002 00:36:45 -0300 Received: from 200-204-18-219.dial-up.telesp.net.br (HELO e9i5z1) (caio?voznak@200.204.18.219 with login) by smtp.mail.vip.sc5.yahoo.com with SMTP; 29 Jun 2002 03:36:20 -0000 Message-ID: <002c01c21e4c$c04cc680$db12ccc8@e9i5z1> From: "Caio H. Voznak" To: References: <001601c21f07$a095d000$d9e2dec8@igor> Subject: Re: [obm-l] biquadrada... Date: Thu, 27 Jun 2002 23:36:17 -0300 MIME-Version: 1.0 Content-Type: multipart/alternative; boundary="----=_NextPart_000_0029_01C21E33.6EF7E3A0" X-Priority: 3 X-MSMail-Priority: Normal X-Mailer: Microsoft Outlook Express 5.50.4133.2400 X-MimeOLE: Produced By Microsoft MimeOLE V5.50.4133.2400 Sender: owner-obm-l@sucuri.mat.puc-rio.br Precedence: bulk Reply-To: obm-l@mat.puc-rio.br This is a multi-part message in MIME format. ------=_NextPart_000_0029_01C21E33.6EF7E3A0 Content-Type: text/plain; charset="iso-8859-1" Content-Transfer-Encoding: quoted-printable Igor, Tb achei 3. Sendo a equa=E7=E3o compativel x^4 - 4x^2 + 3 =3D0 Abra=E7os Caio Voznak ----- Original Message -----=20 From: Igor Castro=20 To: obm-lista=20 Sent: Friday, June 28, 2002 9:55 PM Subject: [obm-l] biquadrada... Ol=E1 colegas da lista, podem me dar uma ajuda nesse problema?=20 Uma equa=E7=E3o biquadrada de coeficientes inteiros, cuja soma desses = coeficientes =E9 zero, tem como uma de suas ra=EDzes sqrt(3). O produto = das ra=EDzes dessa equa=E7=E3o =E9: eu achei 3... mas um amigo n=E3o confirmou o resultado... agrade=E7o = desde j=E1.. []'s Igor.. --- Outgoing mail is certified Virus Free. Checked by AVG anti-virus system (http://www.grisoft.com). Version: 6.0.345 / Virus Database: 193 - Release Date: 9/4/2002 ------=_NextPart_000_0029_01C21E33.6EF7E3A0 Content-Type: text/html; charset="iso-8859-1" Content-Transfer-Encoding: quoted-printable
Igor,
 
 Tb achei 3. Sendo a equa=E7=E3o = compativel x^4 -=20 4x^2 + 3 =3D0
 
Abra=E7os
Caio Voznak
----- Original Message -----
From:=20 Igor = Castro=20
Sent: Friday, June 28, 2002 = 9:55 PM
Subject: [obm-l] = biquadrada...

Ol=E1 colegas da lista,
podem me dar uma ajuda nesse=20 problema? 

Uma=20 equa=E7=E3o biquadrada de coeficientes inteiros, cuja soma desses = coeficientes =E9=20 zero, tem como uma de suas ra=EDzes sqrt(3). O produto das ra=EDzes = dessa equa=E7=E3o=20 =E9:

eu=20 achei 3... mas um amigo n=E3o confirmou o resultado... agrade=E7o = desde=20 j=E1..

[]'s=20 Igor..


---
Outgoing=20 mail is certified Virus Free.
Checked by AVG anti-virus system (http://www.grisoft.com).
Version: = 6.0.345=20 / Virus Database: 193 - Release Date:=20 9/4/2002

------=_NextPart_000_0029_01C21E33.6EF7E3A0-- _________________________________________________________ Do You Yahoo!? Get your free @yahoo.com address at http://mail.yahoo.com ========================================================================= Instruções para entrar na lista, sair da lista e usar a lista em http://www.mat.puc-rio.br/~nicolau/olimp/obm-l.html O administrador desta lista é ========================================================================= From owner-obm-l@sucuri.mat.puc-rio.br Sat Jun 29 11:30:22 2002 Return-Path: Received: (from majordom@localhost) by sucuri.mat.puc-rio.br (8.9.3/8.9.3) id LAA13524 for obm-l-list; Sat, 29 Jun 2002 11:30:12 -0300 Received: from pina.terra.com.br (pina.terra.com.br [200.176.3.17]) by sucuri.mat.puc-rio.br (8.9.3/8.9.3) with ESMTP id LAA13520 for ; Sat, 29 Jun 2002 11:30:10 -0300 Received: from pacuiba.terra.com.br (pacuiba.terra.com.br [200.176.3.40]) by pina.terra.com.br (Postfix) with ESMTP id DA24852FB8 for ; Sat, 29 Jun 2002 11:29:48 -0300 (EST) Received: from stabel (dl-nas1-poa-C89A0083.p001.terra.com.br [200.154.0.131]) (authenticated user dudasta) by pacuiba.terra.com.br (Postfix) with ESMTP id 5AE23800F for ; Sat, 29 Jun 2002 11:29:47 -0300 (EST) Message-ID: <001e01c21f79$6d2d1490$83009ac8@stabel> From: "Eduardo Casagrande Stabel" To: References: <001601c21f07$a095d000$d9e2dec8@igor> Subject: Re: [obm-l] biquadrada... Date: Sat, 29 Jun 2002 11:29:47 -0300 MIME-Version: 1.0 Content-Type: text/plain; charset="iso-8859-1" Content-Transfer-Encoding: 8bit X-Priority: 3 X-MSMail-Priority: Normal X-Mailer: Microsoft Outlook Express 6.00.2600.0000 X-MimeOLE: Produced By Microsoft MimeOLE V6.00.2600.0000 Sender: owner-obm-l@sucuri.mat.puc-rio.br Precedence: bulk Reply-To: obm-l@mat.puc-rio.br Oi Igor, numa equação do tipo p(X) = aX^4 + bX^2 + c = 0, temos X solução real se e somente se -X também é solução real (pois X^2 = (-X)^2 se X é real). Pelos dados da questão vemos que p( raiz(3) ) = 0 p( 1 ) = a + b + c = 0 Portanto também são raízes - raiz(3) e - 1. Daí p(X) = (X - raiz(3))*(X + raiz(3))*(X - 1)*(X + 1) = = (X^2 - 3)*(X^2 - 1) = X^4 - 4X^2 + 3 o que confirma a resposta do Caio H. Voznak, a resposta é 3. Um abraço! Eduardo Casagrande Stabel. Porto Alegre, RS. From: Igor Castro Olá colegas da lista, podem me dar uma ajuda nesse problema? Uma equação biquadrada de coeficientes inteiros, cuja soma desses coeficientes é zero, tem como uma de suas raízes sqrt(3). O produto das raízes dessa equação é: eu achei 3... mas um amigo não confirmou o resultado... agradeço desde já.. []'s Igor.. ========================================================================= Instruções para entrar na lista, sair da lista e usar a lista em http://www.mat.puc-rio.br/~nicolau/olimp/obm-l.html O administrador desta lista é ========================================================================= From owner-obm-l@sucuri.mat.puc-rio.br Sat Jun 29 11:30:22 2002 Return-Path: Received: (from majordom@localhost) by sucuri.mat.puc-rio.br (8.9.3/8.9.3) id LAA13538 for obm-l-list; Sat, 29 Jun 2002 11:30:19 -0300 Received: from sr1.terra.com.br (sr1.terra.com.br [200.176.3.16]) by sucuri.mat.puc-rio.br (8.9.3/8.9.3) with ESMTP id LAA13534 for ; Sat, 29 Jun 2002 11:30:17 -0300 Received: from pavuna.terra.com.br (pavuna.terra.com.br [200.176.3.41]) by sr1.terra.com.br (Postfix) with ESMTP id 6F0526EA83 for ; Sat, 29 Jun 2002 11:29:56 -0300 (EST) Received: from stabel (dl-nas1-poa-C89A0083.p001.terra.com.br [200.154.0.131]) (authenticated user dudasta) by pavuna.terra.com.br (Postfix) with ESMTP id 0FCC66809D for ; Sat, 29 Jun 2002 11:29:55 -0300 (EST) Message-ID: <001f01c21f79$71c73620$83009ac8@stabel> From: "Eduardo Casagrande Stabel" To: References: <001601c21f07$a095d000$d9e2dec8@igor> Subject: Re: [obm-l] biquadrada... Date: Sat, 29 Jun 2002 11:29:55 -0300 MIME-Version: 1.0 Content-Type: text/plain; charset="iso-8859-1" Content-Transfer-Encoding: 8bit X-Priority: 3 X-MSMail-Priority: Normal X-Mailer: Microsoft Outlook Express 6.00.2600.0000 X-MimeOLE: Produced By Microsoft MimeOLE V6.00.2600.0000 Sender: owner-obm-l@sucuri.mat.puc-rio.br Precedence: bulk Reply-To: obm-l@mat.puc-rio.br Oi Igor, numa equação do tipo p(X) = aX^4 + bX^2 + c = 0, temos X solução real se e somente se -X também é solução real (pois X^2 = (-X)^2 se X é real). Pelos dados da questão vemos que p( raiz(3) ) = 0 p( 1 ) = a + b + c = 0 Portanto também são raízes - raiz(3) e - 1. Daí p(X) = (X - raiz(3))*(X + raiz(3))*(X - 1)*(X + 1) = = (X^2 - 3)*(X^2 - 1) = X^4 - 4X^2 + 3 o que confirma a resposta do Caio H. Voznak, a resposta é 3. Um abraço! Eduardo Casagrande Stabel. Porto Alegre, RS. From: Igor Castro Olá colegas da lista, podem me dar uma ajuda nesse problema? Uma equação biquadrada de coeficientes inteiros, cuja soma desses coeficientes é zero, tem como uma de suas raízes sqrt(3). O produto das raízes dessa equação é: eu achei 3... mas um amigo não confirmou o resultado... agradeço desde já.. []'s Igor.. ========================================================================= Instruções para entrar na lista, sair da lista e usar a lista em http://www.mat.puc-rio.br/~nicolau/olimp/obm-l.html O administrador desta lista é ========================================================================= From owner-obm-l@sucuri.mat.puc-rio.br Sat Jun 29 11:30:48 2002 Return-Path: Received: (from majordom@localhost) by sucuri.mat.puc-rio.br (8.9.3/8.9.3) id LAA13575 for obm-l-list; Sat, 29 Jun 2002 11:30:47 -0300 Received: from sr1.terra.com.br (sr1.terra.com.br [200.176.3.16]) by sucuri.mat.puc-rio.br (8.9.3/8.9.3) with ESMTP id LAA13571 for ; Sat, 29 Jun 2002 11:30:45 -0300 Received: from smtp4-poa.terra.com.br (smtp4-poa.terra.com.br [200.176.3.35]) by sr1.terra.com.br (Postfix) with ESMTP id 6FADE6EB13 for ; Sat, 29 Jun 2002 11:30:24 -0300 (EST) Received: from stabel (dl-nas1-poa-C89A0083.p001.terra.com.br [200.154.0.131]) (authenticated user dudasta) by smtp4-poa.terra.com.br (Postfix) with ESMTP id E7E8BAC5B3 for ; Sat, 29 Jun 2002 11:30:22 -0300 (EST) Message-ID: <002901c21f79$825f4220$83009ac8@stabel> From: "Eduardo Casagrande Stabel" To: References: <001601c21f07$a095d000$d9e2dec8@igor> Subject: Re: [obm-l] biquadrada... Date: Sat, 29 Jun 2002 11:30:23 -0300 MIME-Version: 1.0 Content-Type: text/plain; charset="iso-8859-1" Content-Transfer-Encoding: 8bit X-Priority: 3 X-MSMail-Priority: Normal X-Mailer: Microsoft Outlook Express 6.00.2600.0000 X-MimeOLE: Produced By Microsoft MimeOLE V6.00.2600.0000 Sender: owner-obm-l@sucuri.mat.puc-rio.br Precedence: bulk Reply-To: obm-l@mat.puc-rio.br Oi Igor, numa equação do tipo p(X) = aX^4 + bX^2 + c = 0, temos X solução real se e somente se -X também é solução real (pois X^2 = (-X)^2 se X é real). Pelos dados da questão vemos que p( raiz(3) ) = 0 p( 1 ) = a + b + c = 0 Portanto também são raízes - raiz(3) e - 1. Daí p(X) = (X - raiz(3))*(X + raiz(3))*(X - 1)*(X + 1) = = (X^2 - 3)*(X^2 - 1) = X^4 - 4X^2 + 3 o que confirma a resposta do Caio H. Voznak, a resposta é 3. Um abraço! Eduardo Casagrande Stabel. Porto Alegre, RS. From: Igor Castro From: Igor Castro Olá colegas da lista, podem me dar uma ajuda nesse problema? Uma equação biquadrada de coeficientes inteiros, cuja soma desses coeficientes é zero, tem como uma de suas raízes sqrt(3). O produto das raízes dessa equação é: eu achei 3... mas um amigo não confirmou o resultado... agradeço desde já.. []'s Igor.. ========================================================================= Instruções para entrar na lista, sair da lista e usar a lista em http://www.mat.puc-rio.br/~nicolau/olimp/obm-l.html O administrador desta lista é ========================================================================= From owner-obm-l@sucuri.mat.puc-rio.br Sat Jun 29 14:26:01 2002 Return-Path: Received: (from majordom@localhost) by sucuri.mat.puc-rio.br (8.9.3/8.9.3) id OAA15992 for obm-l-list; Sat, 29 Jun 2002 14:24:17 -0300 Received: from shen.bol.com.br (shen.bol.com.br [200.221.24.14]) by sucuri.mat.puc-rio.br (8.9.3/8.9.3) with ESMTP id OAA15988 for ; Sat, 29 Jun 2002 14:24:15 -0300 Received: from bol.com.br (200.221.24.129) by shen.bol.com.br (5.1.071) id 3D175B720020F76D for obm-l@mat.puc-rio.br; Sat, 29 Jun 2002 14:23:29 -0300 Date: Sat, 29 Jun 2002 15:23:30 -0200 Message-Id: Subject: [obm-l] infinito MIME-Version: 1.0 Content-Type: text/plain;charset="iso-8859-1" From: "adr.scr.m" To: obm-l@mat.puc-rio.br X-XaM3-API-Version: 2.4.3.4.4 X-SenderIP: 200.151.29.5 Content-Transfer-Encoding: 8bit X-MIME-Autoconverted: from quoted-printable to 8bit by sucuri.mat.puc-rio.br id OAA15989 Sender: owner-obm-l@sucuri.mat.puc-rio.br Precedence: bulk Reply-To: obm-l@mat.puc-rio.br Outro dia meu professor (de FISICA),fez uma representacao dos numeros de 0 ate + ou - infinito e eu nao entendi muito bem.Ele os colocou num circulo,e disse que nao existem os numeros + ou - infinito,e` somente um numero,e que o sinal dependeria por que lado voce chegaria ao infinito,igual a ideia do +ou- 0.Queria saber se esta certa e porque?E porque nao seria uma reta? Obrigado. Adriano. __________________________________________________________________________ AcessoBOL, só R$ 9,90! O menor preço do mercado! Assine já! http://www.bol.com.br/acessobol ========================================================================= Instruções para entrar na lista, sair da lista e usar a lista em http://www.mat.puc-rio.br/~nicolau/olimp/obm-l.html O administrador desta lista é ========================================================================= From owner-obm-l@sucuri.mat.puc-rio.br Sat Jun 29 21:14:58 2002 Return-Path: Received: (from majordom@localhost) by sucuri.mat.puc-rio.br (8.9.3/8.9.3) id VAA18797 for obm-l-list; Sat, 29 Jun 2002 21:13:25 -0300 Received: from imo-d08.mx.aol.com (imo-d08.mx.aol.com [205.188.157.40]) by sucuri.mat.puc-rio.br (8.9.3/8.9.3) with ESMTP id VAA18793 for ; Sat, 29 Jun 2002 21:13:23 -0300 From: Euraul@aol.com Received: from Euraul@aol.com by imo-d08.mx.aol.com (mail_out_v32.21.) id z.27.29b4548b (15700) for ; Sat, 29 Jun 2002 20:12:56 -0400 (EDT) Received: from web41.aolmail.aol.com (web41.aolmail.aol.com [205.188.161.2]) by air-id05.mx.aol.com (v86_r1.13) with ESMTP id MAILINID52-0629201256; Sat, 29 Jun 2002 20:12:56 -0400 Date: Sat, 29 Jun 2002 20:12:56 EDT Subject: [obm-l] Joguinho... To: Content-Language: br Mime-Version: 1.0 Content-Type: text/plain; charset=ISO-8859-1 X-Mailer: Unknown (No Version) Message-ID: <27.29b4548b.2a4fa708@aol.com> Content-Transfer-Encoding: 8bit X-MIME-Autoconverted: from quoted-printable to 8bit by sucuri.mat.puc-rio.br id VAA18794 Sender: owner-obm-l@sucuri.mat.puc-rio.br Precedence: bulk Reply-To: obm-l@mat.puc-rio.br Como obter o número 21 usando uma única vez os algarismos 1,5,6 e 7 com apenas as quatro operações(+,-,: e .). PS : pode formar números de dois ou mais algarismos. ========================================================================= Instruções para entrar na lista, sair da lista e usar a lista em http://www.mat.puc-rio.br/~nicolau/olimp/obm-l.html O administrador desta lista é ========================================================================= From owner-obm-l@sucuri.mat.puc-rio.br Sat Jun 29 22:20:11 2002 Return-Path: Received: (from majordom@localhost) by sucuri.mat.puc-rio.br (8.9.3/8.9.3) id WAA19545 for obm-l-list; Sat, 29 Jun 2002 22:18:43 -0300 Received: from hotmail.com (f189.sea1.hotmail.com [207.68.163.189]) by sucuri.mat.puc-rio.br (8.9.3/8.9.3) with ESMTP id WAA19541 for ; Sat, 29 Jun 2002 22:18:40 -0300 Received: from mail pickup service by hotmail.com with Microsoft SMTPSVC; Sat, 29 Jun 2002 18:18:18 -0700 Received: from 198.81.8.146 by sea1fd.sea1.hotmail.msn.com with HTTP; Sun, 30 Jun 2002 01:18:18 GMT X-Originating-IP: [198.81.8.146] From: "Frederico Reis Marques de Brito" To: obm-l@mat.puc-rio.br Subject: Re: [obm-l] infinito Date: Sat, 29 Jun 2002 22:18:18 -0300 Mime-Version: 1.0 Content-Type: text/plain; charset=iso-8859-1; format=flowed Message-ID: X-OriginalArrivalTime: 30 Jun 2002 01:18:18.0757 (UTC) FILETIME=[046CEB50:01C21FD4] Sender: owner-obm-l@sucuri.mat.puc-rio.br Precedence: bulk Reply-To: obm-l@mat.puc-rio.br Imagino que ele tenha se referido a compactificação de IR. O conjunto dos números reais pode ser compactificado com a adjunção de infinito. É como se o infinito fosse considerado como um "número". Imagine o conjunto dos números reais como uma reta. Agora imagine um círculo. Tire um ponto do círculo, o que sobra pode ser "desenrolado" numa reta. Este ponto que extraimos é o "infinito" . A construção, do ponto de vista intuitivo, é o inverso desta. Isto é, tomamos a reta e acrescentamos oum ponto fora dela, o infinito, o resultado é uma circunferência. Daí, + e - infinito, as "pontas da reta" são unidas e, portanto, coincidem na compactificação. Espero ter ajudado. Fred. >From: "adr.scr.m" >Reply-To: obm-l@mat.puc-rio.br >To: obm-l@mat.puc-rio.br >Subject: [obm-l] infinito >Date: Sat, 29 Jun 2002 15:23:30 -0200 > >Outro dia meu professor (de FISICA),fez uma >representacao dos numeros de 0 ate + ou - >infinito e eu nao entendi muito bem.Ele os >colocou num circulo,e disse que nao existem >os numeros + ou - infinito,e` somente um >numero,e que o sinal dependeria por que lado >voce chegaria ao infinito,igual a ideia do >+ou- 0.Queria saber se esta certa e porque?E >porque nao seria uma reta? >Obrigado. >Adriano. > > >__________________________________________________________________________ >AcessoBOL, só R$ 9,90! O menor preço do mercado! >Assine já! http://www.bol.com.br/acessobol > > >========================================================================= >Instruções para entrar na lista, sair da lista e usar a lista em >http://www.mat.puc-rio.br/~nicolau/olimp/obm-l.html >O administrador desta lista é >========================================================================= ========================================================================= Instruções para entrar na lista, sair da lista e usar a lista em http://www.mat.puc-rio.br/~nicolau/olimp/obm-l.html O administrador desta lista é ========================================================================= From owner-obm-l@sucuri.mat.puc-rio.br Sat Jun 29 22:35:36 2002 Return-Path: Received: (from majordom@localhost) by sucuri.mat.puc-rio.br (8.9.3/8.9.3) id WAA19873 for obm-l-list; Sat, 29 Jun 2002 22:34:14 -0300 Received: from hotmail.com (f108.law3.hotmail.com [209.185.241.108]) by sucuri.mat.puc-rio.br (8.9.3/8.9.3) with ESMTP id WAA19868 for ; Sat, 29 Jun 2002 22:34:11 -0300 Received: from mail pickup service by hotmail.com with Microsoft SMTPSVC; Sat, 29 Jun 2002 18:33:50 -0700 Received: from 200.151.11.172 by lw3fd.law3.hotmail.msn.com with HTTP; Sun, 30 Jun 2002 01:33:50 GMT X-Originating-IP: [200.151.11.172] From: "Marcelo Souza" To: obm-l@mat.puc-rio.br Subject: Re: [obm-l] infinito Date: Sun, 30 Jun 2002 01:33:50 +0000 Mime-Version: 1.0 Content-Type: text/html Message-ID: X-OriginalArrivalTime: 30 Jun 2002 01:33:50.0586 (UTC) FILETIME=[2FD6BDA0:01C21FD6] Sender: owner-obm-l@sucuri.mat.puc-rio.br Precedence: bulk Reply-To: obm-l@mat.puc-rio.br

A representação por uma reta (em Matemática) é apenas teórica (reta infinita...)....É que na realidade (fisicamente) uma reta eh considerada como um circunferêcia de raio infinito...fica fácil se vc pensar no chão que vc pisa (visto que a terra eh "redonda")...Agora veja....se vc desenhar o ciclo trigonométrico e traçar a reta das tangentes, terá uma explicaçao para este + ou - infinito...Creio eu que ele esteja tomando as noções de cálculo. Enfim, desenhando as tangentes, temos que a tangente de 90 eh naum definida, ou, se definida, tendendo ao infinito....se vc percorrer o ciclo trigonometrico de outra maneira,,,, (de 180 ate 90), vera que os arcos tenderao para infinito pelo lado negativo....ou seja, os infinitos sao os mesmos.....Espero ter sido claro, pois isto eh  meio chato de se escrever...queria conseguir ser mais formal.

falow

abracos

Marcelo

>From: "adr.scr.m"
>Reply-To: obm-l@mat.puc-rio.br
>To: obm-l@mat.puc-rio.br
>Subject: [obm-l] infinito
>Date: Sat, 29 Jun 2002 15:23:30 -0200
>
>Outro dia meu professor (de FISICA),fez uma
>representacao dos numeros de 0 ate + ou -
>infinito e eu nao entendi muito bem.Ele os
>colocou num circulo,e disse que nao existem
>os numeros + ou - infinito,e` somente um
>numero,e que o sinal dependeria por que lado
>voce chegaria ao infinito,igual a ideia do
>+ou- 0.Queria saber se esta certa e porque?E
>porque nao seria uma reta?
>Obrigado.
>Adriano.
>
>
>__________________________________________________________________________
>AcessoBOL, só R$ 9,90! O menor preço do mercado!
>Assine já! http://www.bol.com.br/acessobol
>
>
>=========================================================================
>Instruções para entrar na lista, sair da lista e usar a lista em
>http://www.mat.puc-rio.br/~nicolau/olimp/obm-l.html
>O administrador desta lista é
>=========================================================================


Chat with friends online, try MSN Messenger: Click Here
========================================================================= Instruções para entrar na lista, sair da lista e usar a lista em http://www.mat.puc-rio.br/~nicolau/olimp/obm-l.html O administrador desta lista é ========================================================================= From owner-obm-l@sucuri.mat.puc-rio.br Sat Jun 29 22:38:35 2002 Return-Path: Received: (from majordom@localhost) by sucuri.mat.puc-rio.br (8.9.3/8.9.3) id WAA19968 for obm-l-list; Sat, 29 Jun 2002 22:37:19 -0300 Received: from hotmail.com (f109.law3.hotmail.com [209.185.241.109]) by sucuri.mat.puc-rio.br (8.9.3/8.9.3) with ESMTP id WAA19964 for ; Sat, 29 Jun 2002 22:37:17 -0300 Received: from mail pickup service by hotmail.com with Microsoft SMTPSVC; Sat, 29 Jun 2002 18:36:56 -0700 Received: from 200.151.11.172 by lw3fd.law3.hotmail.msn.com with HTTP; Sun, 30 Jun 2002 01:36:56 GMT X-Originating-IP: [200.151.11.172] From: "Marcelo Souza" To: obm-l@mat.puc-rio.br Subject: Re: [obm-l] biquadrada... Date: Sun, 30 Jun 2002 01:36:56 +0000 Mime-Version: 1.0 Content-Type: text/plain; format=flowed Message-ID: X-OriginalArrivalTime: 30 Jun 2002 01:36:56.0560 (UTC) FILETIME=[9EB01F00:01C21FD6] Sender: owner-obm-l@sucuri.mat.puc-rio.br Precedence: bulk Reply-To: obm-l@mat.puc-rio.br Fala igor! Trankilin? Seguinte, as raizes de um equacao briqudradas sao simetricas, naum eh? se sqrt3 eh raix entao -sqrt3 tb eh raiz....tenta isso que deve ser assim que sai! (e obviamente a soma destas raizes (as 4) sera zero( valeu abracos marcelo! >From: "Igor Castro" >Reply-To: obm-l@mat.puc-rio.br >To: "obm-lista" >Subject: [obm-l] biquadrada... >Date: Fri, 28 Jun 2002 21:55:11 -0300 > >Olá colegas da lista, >podem me dar uma ajuda nesse problema? >Uma equação biquadrada de coeficientes inteiros, cuja soma desses >coeficientes é zero, tem como uma de suas raízes sqrt(3). O produto das >raízes dessa equação é: > >eu achei 3... mas um amigo não confirmou o resultado... agradeço desde já.. > >[]'s Igor.. > _________________________________________________________________ Chat with friends online, try MSN Messenger: http://messenger.msn.com ========================================================================= Instruções para entrar na lista, sair da lista e usar a lista em http://www.mat.puc-rio.br/~nicolau/olimp/obm-l.html O administrador desta lista é ========================================================================= From owner-obm-l@sucuri.mat.puc-rio.br Sun Jun 30 01:18:28 2002 Return-Path: Received: (from majordom@localhost) by sucuri.mat.puc-rio.br (8.9.3/8.9.3) id BAA21483 for obm-l-list; Sun, 30 Jun 2002 01:16:40 -0300 Received: from web13707.mail.yahoo.com (web13707.mail.yahoo.com [216.136.175.140]) by sucuri.mat.puc-rio.br (8.9.3/8.9.3) with SMTP id BAA21479 for ; Sun, 30 Jun 2002 01:16:37 -0300 Message-ID: <20020630041616.93959.qmail@web13707.mail.yahoo.com> Received: from [200.213.88.157] by web13707.mail.yahoo.com via HTTP; Sun, 30 Jun 2002 01:16:16 ART Date: Sun, 30 Jun 2002 01:16:16 -0300 (ART) From: =?iso-8859-1?q?pichurin?= Subject: [obm-l] espacial To: obm-l@mat.puc-rio.br MIME-Version: 1.0 Content-Type: text/plain; charset=iso-8859-1 Content-Transfer-Encoding: 8bit Sender: owner-obm-l@sucuri.mat.puc-rio.br Precedence: bulk Reply-To: obm-l@mat.puc-rio.br qual é a resolução(detalhada) desses exercícios? -Quatro pontos coplanares determinam um único plano? -Considerando duas retas r e s pararlelas a um plano A, pode existir uma reta contida em A que seja concorrente com r ou s? -Se dois planos são secantes, então existe uma reta de um deles reversa a uma reta do outro? -Se dois planos distintos são paralelos, então toda reta paralela a um deles é paralela ao outro? -Considere duas circunferências de raios congruentes, centros comuns e contidas em planso diferentes.Qual é a intersecção dessas duas circunferências? -Uma reta num plano pode ser considerada paralela ao plano? _______________________________________________________________________ Yahoo! Encontros O lugar certo para encontrar a sua alma gêmea. http://br.encontros.yahoo.com/ ========================================================================= Instruções para entrar na lista, sair da lista e usar a lista em http://www.mat.puc-rio.br/~nicolau/olimp/obm-l.html O administrador desta lista é ========================================================================= From owner-obm-l@sucuri.mat.puc-rio.br Sun Jun 30 11:52:51 2002 Return-Path: Received: (from majordom@localhost) by sucuri.mat.puc-rio.br (8.9.3/8.9.3) id LAA27936 for obm-l-list; Sun, 30 Jun 2002 11:51:23 -0300 Received: from hotmail.com (f57.law3.hotmail.com [209.185.241.57]) by sucuri.mat.puc-rio.br (8.9.3/8.9.3) with ESMTP id LAA27932 for ; Sun, 30 Jun 2002 11:51:20 -0300 Received: from mail pickup service by hotmail.com with Microsoft SMTPSVC; Sun, 30 Jun 2002 07:51:01 -0700 Received: from 200.151.28.185 by lw3fd.law3.hotmail.msn.com with HTTP; Sun, 30 Jun 2002 14:51:01 GMT X-Originating-IP: [200.151.28.185] From: "Marcelo Souza" To: obm-l@mat.puc-rio.br Subject: Re: [obm-l] espacial Date: Sun, 30 Jun 2002 14:51:01 +0000 Mime-Version: 1.0 Content-Type: text/plain; format=flowed Message-ID: X-OriginalArrivalTime: 30 Jun 2002 14:51:01.0898 (UTC) FILETIME=[8D85BEA0:01C22045] Sender: owner-obm-l@sucuri.mat.puc-rio.br Precedence: bulk Reply-To: obm-l@mat.puc-rio.br > >qual é a resolução(detalhada) desses exercícios? > > >-Quatro pontos coplanares determinam um único plano? >-Considerando duas retas r e s pararlelas a um plano >A, pode existir uma reta contida em A que seja >concorrente com r ou s? >-Se dois planos são secantes, então existe uma reta de >um deles reversa a uma reta do outro? >-Se dois planos distintos são paralelos, então toda >reta paralela a um deles é paralela ao outro? >-Considere duas circunferências de raios congruentes, >centros comuns e contidas em planso diferentes.Qual é >a intersecção dessas duas circunferências? >-Uma reta num plano pode ser considerada paralela ao plano? Oi... Naum sei se tem uma resolução detalhada (talbez vc queira formalizar)...mas vc formaliza utilizando os postulados da geometria. -A primeira, naum sei se estou errado, mas a definicao de coplanares eh estarem em um mesmo plano, logo se sao 4 ptos coplanares estao num unico plano.(se foi bem isso que entendi) -A intersecçao das duas circunferencias eh uma resta (ja que as duas determinam dois planos que se intersectam) Tem algumas que sei responder, mas pode haver alguma exceçAo que pode me falhar na memoria, entao prefiro deixar aí. falow Marcelo _________________________________________________________________ Chat with friends online, try MSN Messenger: http://messenger.msn.com ========================================================================= Instruções para entrar na lista, sair da lista e usar a lista em http://www.mat.puc-rio.br/~nicolau/olimp/obm-l.html O administrador desta lista é ========================================================================= From owner-obm-l@sucuri.mat.puc-rio.br Sun Jun 30 14:41:28 2002 Return-Path: Received: (from majordom@localhost) by sucuri.mat.puc-rio.br (8.9.3/8.9.3) id OAA29795 for obm-l-list; Sun, 30 Jun 2002 14:39:56 -0300 Received: (from nicolau@localhost) by sucuri.mat.puc-rio.br (8.9.3/8.9.3) id OAA29790 for obm-l@mat.puc-rio.br; Sun, 30 Jun 2002 14:39:56 -0300 Date: Sun, 30 Jun 2002 14:39:56 -0300 From: "Nicolau C. Saldanha" To: obm-l@mat.puc-rio.br Subject: Re: [obm-l] duvida em limite Message-ID: <20020630143956.A29602@sucuri.mat.puc-rio.br> References: <200206240156.g5O1uPf18858@Euler.impa.br> <20020626050918.75751.qmail@web21308.mail.yahoo.com> Mime-Version: 1.0 Content-Type: text/plain; charset=iso-8859-1 Content-Disposition: inline Content-Transfer-Encoding: 8bit User-Agent: Mutt/1.2.5i In-Reply-To: <20020626050918.75751.qmail@web21308.mail.yahoo.com>; from marc_reybr@yahoo.com.br on Wed, Jun 26, 2002 at 02:09:18AM -0300 Sender: owner-obm-l@sucuri.mat.puc-rio.br Precedence: bulk Reply-To: obm-l@mat.puc-rio.br On Wed, Jun 26, 2002 at 02:09:18AM -0300, Marcos Reynaldo wrote: > Caros colegas, talvez voces possam me ajudar em numa > duvida. > Resolvendo uns problemas de Cálculo do livro Calculo A > da Diva Marilia e Miriam Buss, me deparei com o limite > de raiz quadrada de x quando x tende a zero. Pelo que > eu lembro, esse limite não existe. Mas as autoras do > livro do Cálculo A, resolvem um exercicio que envolvem > a soma de três funções dentre elas raiz de x e 1/x^2 > (a outra não lembro, mas é tipo x, vamos dizer), da > seguinte forma lim (x + raiz x + 1/x^2) quando x tende > a zero = 0 + 0 + infinito = + infinito. Ora, mais ai > ela considera que lim de raz x quando x tende a zero é > 0. Olhei um exercicio do Guidorizzi (lim raiz de x > quando x tende a zero) e ele dá como resposta 0. > Não sei se não aprendi direito, mas como pode ser > zero? Pela direita tudo bem , mas pela esquerda temos > números complexos e esse conjunto não eh ordenado para > falar que tende a zero. > Gostaria de saber dos colegas quem estah certo eu ou > os autores. Antes de mais nada é bom observar que os livros de cálculo não costumam ser muito cuidadosos com as definições. Às vezes não são nem consistentes. Se você deseja ver definições cuidadosas de limite, continuidade, ou qualquer outro conceito visto nos cursos de cálculo você deve consultar um bom livro de análise. O livro do Elon (curso de análise, vol 1, projeto Euclides) é um exemplo de um bom livro de análise. Mas respondendo sua pergunta, uma definição usual de limite (escrita mais cuidadosamente do que em alguns livros de cálculo) é a seguinte: Seja f : A -> R uma função, A um subconjunto de R. Seja x0 um ponto de R que pode estar em A ou não mas deve ser ponto de acumulação de A. Então lim_{x -> x0} f(x) = L se e somente se para todo número real epsilon > 0 existe um número real delta > 0 tal que se 0 < |x - x0| < delta, x em A então |f(x) - L| < epsilon. Dado um conjunto A de números reais e x0 um número real dizemos que x0 é ponto de acumulação de A se para todo epsilon > 0 existir x em A com 0 < |x - x0| < epsilon. Se x0 não for ponto de acumulação de A a definição fica meio sem pé nem cabeça, qualquer número é limite (por vacuidade). De acordo com esta definição é correto dizer que o limite de sqrt(x) quando x -> 0 é igual a 0. Note que o domínio da função sqrt é o intervalo [0, +infinito). Note também que o limite da soma é a soma dos limites para funções com o mesmo domínio (ou pelo menos com um domínio comum grande). Se definirmos f: (0,1) -> R, f(x) = x e g: (-1,0) -> R, g(x) = x então temos lim_{x -> 0} f(x) = 0 e lim_{x -> 0} g(x) = 0 mas lim_{x -> 0} (f(x) + g(x)) não faz o menor sentido pois f(x) + g(x) não está definida nunca. []s, N. ========================================================================= Instruções para entrar na lista, sair da lista e usar a lista em http://www.mat.puc-rio.br/~nicolau/olimp/obm-l.html O administrador desta lista é ========================================================================= From owner-obm-l@sucuri.mat.puc-rio.br Sun Jun 30 14:42:52 2002 Return-Path: Received: (from majordom@localhost) by sucuri.mat.puc-rio.br (8.9.3/8.9.3) id OAA29828 for obm-l-list; Sun, 30 Jun 2002 14:41:35 -0300 Received: (from nicolau@localhost) by sucuri.mat.puc-rio.br (8.9.3/8.9.3) id OAA29823 for obm-l@mat.puc-rio.br; Sun, 30 Jun 2002 14:41:34 -0300 Date: Sun, 30 Jun 2002 14:41:34 -0300 From: "Nicolau C. Saldanha" To: obm-l@mat.puc-rio.br Subject: Re: [obm-l] outra duvida Message-ID: <20020630144134.B29602@sucuri.mat.puc-rio.br> References: <200206240156.g5O1uPf18858@Euler.impa.br> <20020626051530.52689.qmail@web21302.mail.yahoo.com> Mime-Version: 1.0 Content-Type: text/plain; charset=iso-8859-1 Content-Disposition: inline Content-Transfer-Encoding: 8bit User-Agent: Mutt/1.2.5i In-Reply-To: <20020626051530.52689.qmail@web21302.mail.yahoo.com>; from marc_reybr@yahoo.com.br on Wed, Jun 26, 2002 at 02:15:30AM -0300 Sender: owner-obm-l@sucuri.mat.puc-rio.br Precedence: bulk Reply-To: obm-l@mat.puc-rio.br On Wed, Jun 26, 2002 at 02:15:30AM -0300, Marcos Reynaldo wrote: > Outra duvida, a maioria dos livros de calculo define > que uma funcao eh continua num ponto x=a quando > 1) f(a) existe > 2) lim f(x) quando x tende a a existe > 3) lim f(x) quando x tende a a = f(a) > > Ora mas a primeira condicão não tem sentido nenhum. > Pra analisar se uma função é continua tem que analisar > nos pontos do dominio da função e não fora. Portanto > f(a) sempre existe. Dessa definição poderia concluir > então que se f(a) não existe a função é descontinua. > Mas não se pode falar nada pois ela nem é definida. É > a mesma coisa que perguntar qual a cor dos olhos da > mula sem cabeça. Se não tem cabeça como posso dizer > que isso ou aquilo. > Tô errado na minha consideração ? Você está basicamente certo. A frase 'f(a) existe' é uma forma (menos cuidadosa) de dizer que a está no domínio de f. []s, N. ========================================================================= Instruções para entrar na lista, sair da lista e usar a lista em http://www.mat.puc-rio.br/~nicolau/olimp/obm-l.html O administrador desta lista é ========================================================================= From owner-obm-l@sucuri.mat.puc-rio.br Sun Jun 30 14:44:47 2002 Return-Path: Received: (from majordom@localhost) by sucuri.mat.puc-rio.br (8.9.3/8.9.3) id OAA29884 for obm-l-list; Sun, 30 Jun 2002 14:43:30 -0300 Received: (from nicolau@localhost) by sucuri.mat.puc-rio.br (8.9.3/8.9.3) id OAA29879 for obm-l@mat.puc-rio.br; Sun, 30 Jun 2002 14:43:30 -0300 Date: Sun, 30 Jun 2002 14:43:30 -0300 From: "Nicolau C. Saldanha" To: obm-l@mat.puc-rio.br Subject: Re: [obm-l] Matriz de Vandermonde Message-ID: <20020630144330.C29602@sucuri.mat.puc-rio.br> References: <000701c21cd6$d77b0d20$46069ac8@stabel> <20020626143937.6986.qmail@web10205.mail.yahoo.com> Mime-Version: 1.0 Content-Type: text/plain; charset=iso-8859-1 Content-Disposition: inline Content-Transfer-Encoding: 8bit User-Agent: Mutt/1.2.5i In-Reply-To: <20020626143937.6986.qmail@web10205.mail.yahoo.com>; from hnaves@yahoo.com on Wed, Jun 26, 2002 at 11:39:37AM -0300 Sender: owner-obm-l@sucuri.mat.puc-rio.br Precedence: bulk Reply-To: obm-l@mat.puc-rio.br On Wed, Jun 26, 2002 at 11:39:37AM -0300, Humberto Naves wrote: > Oi, > > É possível demonstrar que o determinante de Vandermonde é > > Produtório (0 <= i < j <= n) de ((t_i) - (t_j)). > > Para ver isso, basta encarar o determinante como um polinômio em t_i, e ver > que quando t_i = t_j, o polinômio se anula. Logo se os t_i's forem distintos, o > determinante é diferente de 0. Só completando, é preciso ver que o grau deste polinômio é (n-1) logo já encontramos todas as raízes pelo argumento do Humberto. []s, N. > > Falow, Humberto > > --- Eduardo Casagrande Stabel escreveu: > Ola pessoal > da lista! > > > > Uma matriz de Vandermonde é uma matriz P da forma > > P_(i,j) = [t_(i-1)]^j onde i e j estão entre 0 e n > > um jeito mais explicito é o seguinte > > P = > > [ 1 t_0 (t_0)^2 (t_0)^3 ... (t_0)^n ] > > [ 1 t_1 (t_1)^2 (t_1)^3 ... (t_1)^n ] > > [ ... ] > > [ 1 t_n (t_n)^2 (t_n)^3 ... (t_n)^n ] > > > > Eu não estou conseguindo demonstrar que se os t_i's são todos distintos > > então a matriz P é inversível. > > > > Alguém demonstra? > > > > Obrigado pela futura ajuda > > > > Eduardo Casagrande Stabel. Porto Alegre, RS. > > > > ========================================================================= > > Instruções para entrar na lista, sair da lista e usar a lista em > > http://www.mat.puc-rio.br/~nicolau/olimp/obm-l.html > > O administrador desta lista é > > ========================================================================= > > _______________________________________________________________________ > Copa 2002 > Yahoo! - Patrocinador oficial da Copa do Mundo da FIFA 2002 > http://br.sports.yahoo.com/fifaworldcup/ > ========================================================================= > Instruções para entrar na lista, sair da lista e usar a lista em > http://www.mat.puc-rio.br/~nicolau/olimp/obm-l.html > O administrador desta lista é > ========================================================================= ========================================================================= Instruções para entrar na lista, sair da lista e usar a lista em http://www.mat.puc-rio.br/~nicolau/olimp/obm-l.html O administrador desta lista é ========================================================================= From owner-obm-l@sucuri.mat.puc-rio.br Sun Jun 30 14:55:40 2002 Return-Path: Received: (from majordom@localhost) by sucuri.mat.puc-rio.br (8.9.3/8.9.3) id OAA30161 for obm-l-list; Sun, 30 Jun 2002 14:54:21 -0300 Received: (from nicolau@localhost) by sucuri.mat.puc-rio.br (8.9.3/8.9.3) id OAA30156 for obm-l@mat.puc-rio.br; Sun, 30 Jun 2002 14:54:21 -0300 Date: Sun, 30 Jun 2002 14:54:21 -0300 From: "Nicolau C. Saldanha" To: obm-l@mat.puc-rio.br Subject: Re: [obm-l] infinito Message-ID: <20020630145421.D29602@sucuri.mat.puc-rio.br> References: Mime-Version: 1.0 Content-Type: text/plain; charset=iso-8859-1 Content-Disposition: inline Content-Transfer-Encoding: 8bit User-Agent: Mutt/1.2.5i In-Reply-To: ; from adr.scr.m@bol.com.br on Sat, Jun 29, 2002 at 03:23:30PM -0200 Sender: owner-obm-l@sucuri.mat.puc-rio.br Precedence: bulk Reply-To: obm-l@mat.puc-rio.br On Sat, Jun 29, 2002 at 03:23:30PM -0200, adr.scr.m wrote: > Outro dia meu professor (de FISICA),fez uma > representacao dos numeros de 0 ate + ou - > infinito e eu nao entendi muito bem.Ele os > colocou num circulo,e disse que nao existem > os numeros + ou - infinito,e` somente um > numero,e que o sinal dependeria por que lado > voce chegaria ao infinito,igual a ideia do > +ou- 0.Queria saber se esta certa e porque?E > porque nao seria uma reta? > Obrigado. > Adriano. Existem muitas maneiras de aumentar R para incluir um ou mais elementos infinitos. Não existe uma maneira mais certa do que a outra, existem maneiras diferentes que são úteis ou importantes em situações diferentes. Uma construção comum é acrescentar (+infinito) de um lado e (-infinito) do outro. Outra construção comum é esta que seu professor de física mostrou, acrescentar um elemento chamado (infinito) para formar um círculo. Seu professor talvez esteja sendo um pouco dogmático ao dizer que "não existem os números + ou - infinito", interprete isto como "no meu curso a construção mais útil, mais interessante, será a de acrescentar *um* elemento chamado (infinito) para formar um círculo". []s, N. ========================================================================= Instruções para entrar na lista, sair da lista e usar a lista em http://www.mat.puc-rio.br/~nicolau/olimp/obm-l.html O administrador desta lista é ========================================================================= From owner-obm-l@sucuri.mat.puc-rio.br Sun Jun 30 15:00:00 2002 Return-Path: Received: (from majordom@localhost) by sucuri.mat.puc-rio.br (8.9.3/8.9.3) id OAA30545 for obm-l-list; Sun, 30 Jun 2002 14:58:39 -0300 Received: (from nicolau@localhost) by sucuri.mat.puc-rio.br (8.9.3/8.9.3) id OAA30540 for obm-l@mat.puc-rio.br; Sun, 30 Jun 2002 14:58:39 -0300 Date: Sun, 30 Jun 2002 14:58:38 -0300 From: "Nicolau C. Saldanha" To: obm-l@mat.puc-rio.br Subject: Re: [obm-l] espacial Message-ID: <20020630145838.E29602@sucuri.mat.puc-rio.br> References: <20020630041616.93959.qmail@web13707.mail.yahoo.com> Mime-Version: 1.0 Content-Type: text/plain; charset=iso-8859-1 Content-Disposition: inline Content-Transfer-Encoding: 8bit User-Agent: Mutt/1.2.5i In-Reply-To: <20020630041616.93959.qmail@web13707.mail.yahoo.com>; from pichurinbr@yahoo.com.br on Sun, Jun 30, 2002 at 01:16:16AM -0300 Sender: owner-obm-l@sucuri.mat.puc-rio.br Precedence: bulk Reply-To: obm-l@mat.puc-rio.br On Sun, Jun 30, 2002 at 01:16:16AM -0300, pichurin wrote: > qual é a resolução(detalhada) desses exercícios? > > > -Quatro pontos coplanares determinam um único plano? Se eles forem colineares não. > -Considerando duas retas r e s pararlelas a um plano > A, pode existir uma reta contida em A que seja > concorrente com r ou s? Só se r ou s estiver contida em A, o que é um caso particular de paralelismo. > -Se dois planos são secantes, então existe uma reta de > um deles reversa a uma reta do outro? Sim. Faça um desenho. > -Se dois planos distintos são paralelos, então toda > reta paralela a um deles é paralela ao outro? Sim. Lembre-se que uma reta contida em um plano é paralela ao plano. > -Considere duas circunferências de raios congruentes, > centros comuns e contidas em planso diferentes.Qual é > a intersecção dessas duas circunferências? Dois pontos. Faça um desenho. > -Uma reta num plano pode ser considerada paralela ao plano? Deve ser considerada paralela ao plano. Uma reta também deve ser considerada paralela a si própria e um plano deve ser considerado paralelo a si próprio. []s, N. ========================================================================= Instruções para entrar na lista, sair da lista e usar a lista em http://www.mat.puc-rio.br/~nicolau/olimp/obm-l.html O administrador desta lista é ========================================================================= From owner-obm-l@sucuri.mat.puc-rio.br Sun Jun 30 15:28:31 2002 Return-Path: Received: (from majordom@localhost) by sucuri.mat.puc-rio.br (8.9.3/8.9.3) id PAA31984 for obm-l-list; Sun, 30 Jun 2002 15:27:04 -0300 Received: from www.zipmail.com.br (smtp.zipmail.com.br [200.187.242.10]) by sucuri.mat.puc-rio.br (8.9.3/8.9.3) with ESMTP id PAA31980 for ; Sun, 30 Jun 2002 15:27:02 -0300 From: luizhenriquerick@zipmail.com.br Received: from [200.165.185.194] by www.zipmail.com.br with HTTP; Sun, 30 Jun 2002 15:26:43 -0300 Message-ID: <3D1CE18A00003095@www.zipmail.com.br> Date: Sun, 30 Jun 2002 15:26:43 -0300 Subject: [obm-l] =?iso-8859-1?Q?Dica=21?= To: obm-l@mat.puc-rio.br MIME-Version: 1.0 Content-Type: text/plain; charset="iso-8859-1" Content-Transfer-Encoding: 8bit X-MIME-Autoconverted: from quoted-printable to 8bit by sucuri.mat.puc-rio.br id PAA31981 Sender: owner-obm-l@sucuri.mat.puc-rio.br Precedence: bulk Reply-To: obm-l@mat.puc-rio.br Será que alguém poderia me dizer , como faço para ler arquivos no formato "PS"? ---------------------------------------- |-=Rick-C.R.B.=- | |ICQ 124805654 | |e-mail luizhenriquerick@zipmail.com.br | ---------------------------------------- ------------------------------------------ Use o melhor sistema de busca da Internet Radar UOL - http://www.radaruol.com.br ========================================================================= Instruções para entrar na lista, sair da lista e usar a lista em http://www.mat.puc-rio.br/~nicolau/olimp/obm-l.html O administrador desta lista é ========================================================================= From owner-obm-l@sucuri.mat.puc-rio.br Sun Jun 30 16:07:23 2002 Return-Path: Received: (from majordom@localhost) by sucuri.mat.puc-rio.br (8.9.3/8.9.3) id QAA00313 for obm-l-list; Sun, 30 Jun 2002 16:05:57 -0300 Received: from sr1.terra.com.br (sr1.terra.com.br [200.176.3.16]) by sucuri.mat.puc-rio.br (8.9.3/8.9.3) with ESMTP id QAA00301 for ; Sun, 30 Jun 2002 16:05:54 -0300 Received: from srv9-sao.terra.com.br (srv9-sao.terra.com.br [200.176.3.37]) by sr1.terra.com.br (Postfix) with ESMTP id 53FA06EFCC for ; Sun, 30 Jun 2002 16:05:37 -0300 (EST) Received: from emilio (200-158-164-218.dsl.telesp.net.br [200.158.164.218]) (authenticated user rodrigozerati) by srv9-sao.terra.com.br (Postfix) with ESMTP id B8F47C8796 for ; Sun, 30 Jun 2002 16:05:36 -0300 (EST) Message-ID: <004901c22069$2678c8a0$4f31fea9@emilio> From: "Rodrigo Zerati" To: "OBM" Subject: [obm-l] livro... Date: Sun, 30 Jun 2002 16:05:49 -0300 MIME-Version: 1.0 Content-Type: multipart/alternative; boundary="----=_NextPart_000_0046_01C22050.005A8500" X-Priority: 3 X-MSMail-Priority: Normal X-Mailer: Microsoft Outlook Express 5.00.2615.200 X-MIMEOLE: Produced By Microsoft MimeOLE V5.00.2615.200 Sender: owner-obm-l@sucuri.mat.puc-rio.br Precedence: bulk Reply-To: obm-l@mat.puc-rio.br This is a multi-part message in MIME format. ------=_NextPart_000_0046_01C22050.005A8500 Content-Type: text/plain; charset="iso-8859-1" Content-Transfer-Encoding: quoted-printable Ol=E1, gostaria de saber se algu=E9m sabe o nome do livro que =E9 russo, = e que o cara transforma tudo em triangulos, ele resolve tudo pela = algebra..... Se alguem souber agrade=E7o desde ja abra=E7os, Rodrigo Zerati ------=_NextPart_000_0046_01C22050.005A8500 Content-Type: text/html; charset="iso-8859-1" Content-Transfer-Encoding: quoted-printable
Ol=E1, gostaria de saber se algu=E9m = sabe o nome do=20 livro que =E9 russo, e que o cara transforma tudo em triangulos, ele = resolve tudo=20 pela algebra.....
Se alguem souber agrade=E7o desde = ja
 
abra=E7os,
 
Rodrigo = Zerati
------=_NextPart_000_0046_01C22050.005A8500-- ========================================================================= Instruções para entrar na lista, sair da lista e usar a lista em http://www.mat.puc-rio.br/~nicolau/olimp/obm-l.html O administrador desta lista é ========================================================================= From owner-obm-l@sucuri.mat.puc-rio.br Sun Jun 30 18:23:10 2002 Return-Path: Received: (from majordom@localhost) by sucuri.mat.puc-rio.br (8.9.3/8.9.3) id SAA01448 for obm-l-list; Sun, 30 Jun 2002 18:22:48 -0300 Received: from shen.bol.com.br (shen.bol.com.br [200.221.24.14]) by sucuri.mat.puc-rio.br (8.9.3/8.9.3) with ESMTP id SAA01444 for ; Sun, 30 Jun 2002 18:22:47 -0300 Received: from bol.com.br (200.221.24.134) by shen.bol.com.br (5.1.071) id 3D175B72002615A1 for obm-l@mat.puc-rio.br; Sun, 30 Jun 2002 18:22:04 -0300 Date: Sun, 30 Jun 2002 18:22:05 -0300 Message-Id: Subject: Re: [obm-l] infinito MIME-Version: 1.0 Content-Type: text/plain;charset="iso-8859-1" From: "adr.scr.m" To: obm-l@mat.puc-rio.br X-XaM3-API-Version: 2.4.3.4.4 X-SenderIP: 200.151.95.4 Content-Transfer-Encoding: 8bit X-MIME-Autoconverted: from quoted-printable to 8bit by sucuri.mat.puc-rio.br id SAA01445 Sender: owner-obm-l@sucuri.mat.puc-rio.br Precedence: bulk Reply-To: obm-l@mat.puc-rio.br entendi,mas ele falou que tal situacao ocorre tambem no grafico da tan(x),que ela cresce ate o + infinito e volta pelo - infinito. Esta certo tambem isto ? Obrigado. Adriano. __________________________________________________________________________ AcessoBOL, só R$ 9,90! O menor preço do mercado! Assine já! http://www.bol.com.br/acessobol ========================================================================= Instruções para entrar na lista, sair da lista e usar a lista em http://www.mat.puc-rio.br/~nicolau/olimp/obm-l.html O administrador desta lista é ========================================================================= From owner-obm-l@sucuri.mat.puc-rio.br Sun Jun 30 19:16:05 2002 Return-Path: Received: (from majordom@localhost) by sucuri.mat.puc-rio.br (8.9.3/8.9.3) id TAA02066 for obm-l-list; Sun, 30 Jun 2002 19:15:14 -0300 Received: from pina.terra.com.br (pina.terra.com.br [200.176.3.17]) by sucuri.mat.puc-rio.br (8.9.3/8.9.3) with ESMTP id TAA02062 for ; Sun, 30 Jun 2002 19:15:13 -0300 Received: from engenho.terra.com.br (engenho.terra.com.br [200.176.3.42]) by pina.terra.com.br (Postfix) with ESMTP id CFBC253031 for ; Sun, 30 Jun 2002 19:14:55 -0300 (EST) Received: from GOOFY (200-206-135-114.dsl.telesp.net.br [200.206.135.114]) (authenticated user ojrmat) by engenho.terra.com.br (Postfix) with ESMTP id 9E76B68182 for ; Sun, 30 Jun 2002 19:14:50 -0300 (EST) From: "Osni Jose Rapelli" To: Subject: RE: [obm-l] biquadrada... Date: Sun, 30 Jun 2002 19:14:20 -0300 Message-ID: <000001c22083$8691c560$7287cec8@GOOFY> MIME-Version: 1.0 Content-Type: text/plain; charset="iso-8859-1" X-Priority: 3 (Normal) X-MSMail-Priority: Normal X-Mailer: Microsoft Outlook, Build 10.0.2616 Importance: Normal In-Reply-To: <002c01c21e4c$c04cc680$db12ccc8@e9i5z1> X-MimeOLE: Produced By Microsoft MimeOLE V6.00.2600.0000 Content-Transfer-Encoding: 8bit X-MIME-Autoconverted: from quoted-printable to 8bit by sucuri.mat.puc-rio.br id TAA02063 Sender: owner-obm-l@sucuri.mat.puc-rio.br Precedence: bulk Reply-To: obm-l@mat.puc-rio.br Igor toda equação do tipo ax^4-2ax^2-3a=0 admite como solução sqrt(3). Pela teoria polinomial sabemos que as raízes irracionais sempre aparecem em pares, para ser mais exato Sempre aparecem com seu conjugado portando -sqrt(3) também é raiz Fazendo a substituição y=x^2 Obtemos uma equação do 2o. Grau ay^2-2ay-3a=0 Cujo delta=4a^2 e portanto sqrt(delta)=2a Desta forma temo como raízes desta equação y1=3 e y2=-1 Voltando a troca de variáveis vemos que x1=sqrt(3) x2=-sqrt(3) x3=i x4=-i Onde i^2=-1 Desta forma x1.x2.x3.x4=sqrt(3).-sqrt(3).i.-i=-3 O mesmo resultado ocorre se considerarmos apenas as respostas reias Portanto sempre teremos nas condições propostas a multiplicação das raízes como sendo -3 OBS: O a deve ser sempre diferente de zero. Uma outra forma de mostrar isso é através das relações de Girardi X1.x2.x3.x4=a4/a0 Como na equação temos a4=-3a e o ao=a Chegamos que x1.x2.x3.x4=-3 Um forte abraço OSNI JOSE RAPELLI ========================================================================= Instruções para entrar na lista, sair da lista e usar a lista em http://www.mat.puc-rio.br/~nicolau/olimp/obm-l.html O administrador desta lista é ========================================================================= From owner-obm-l@sucuri.mat.puc-rio.br Sun Jun 30 19:22:38 2002 Return-Path: Received: (from majordom@localhost) by sucuri.mat.puc-rio.br (8.9.3/8.9.3) id TAA02171 for obm-l-list; Sun, 30 Jun 2002 19:21:20 -0300 Received: from pina.terra.com.br (pina.terra.com.br [200.176.3.17]) by sucuri.mat.puc-rio.br (8.9.3/8.9.3) with ESMTP id TAA02166 for ; Sun, 30 Jun 2002 19:21:18 -0300 Received: from engenho.terra.com.br (engenho.terra.com.br [200.176.3.42]) by pina.terra.com.br (Postfix) with ESMTP id 13B5252FA5 for ; Sun, 30 Jun 2002 19:21:01 -0300 (EST) Received: from GOOFY (200-206-135-114.dsl.telesp.net.br [200.206.135.114]) (authenticated user ojrmat) by engenho.terra.com.br (Postfix) with ESMTP id EBB5268245 for ; Sun, 30 Jun 2002 19:20:52 -0300 (EST) From: "Osni Jose Rapelli" To: Subject: RE: [obm-l] Dica! Date: Sun, 30 Jun 2002 19:20:26 -0300 Message-ID: <000101c22084$61c22300$7287cec8@GOOFY> MIME-Version: 1.0 Content-Type: text/plain; charset="iso-8859-1" X-Priority: 3 (Normal) X-MSMail-Priority: Normal X-Mailer: Microsoft Outlook, Build 10.0.2616 Importance: Normal In-Reply-To: <3D1CE18A00003095@www.zipmail.com.br> X-MimeOLE: Produced By Microsoft MimeOLE V6.00.2600.0000 Content-Transfer-Encoding: 8bit X-MIME-Autoconverted: from quoted-printable to 8bit by sucuri.mat.puc-rio.br id TAA02167 Sender: owner-obm-l@sucuri.mat.puc-rio.br Precedence: bulk Reply-To: obm-l@mat.puc-rio.br De onde são gerados estes arquivos comn a extensão ps Se for postscript são arquivos de impressora... ======================================================================== = Instruções para entrar na lista, sair da lista e usar a lista em http://www.mat.puc-rio.br/~nicolau/olimp/obm-l.html O administrador desta lista é ======================================================================== = ========================================================================= Instruções para entrar na lista, sair da lista e usar a lista em http://www.mat.puc-rio.br/~nicolau/olimp/obm-l.html O administrador desta lista é ========================================================================= From owner-obm-l@sucuri.mat.puc-rio.br Sun Jun 30 19:22:54 2002 Return-Path: Received: (from majordom@localhost) by sucuri.mat.puc-rio.br (8.9.3/8.9.3) id TAA02202 for obm-l-list; Sun, 30 Jun 2002 19:21:51 -0300 Received: from candeias.terra.com.br (candeias.terra.com.br [200.176.3.18]) by sucuri.mat.puc-rio.br (8.9.3/8.9.3) with ESMTP id TAA02198 for ; Sun, 30 Jun 2002 19:21:49 -0300 Received: from engenho.terra.com.br (engenho.terra.com.br [200.176.3.42]) by candeias.terra.com.br (Postfix) with ESMTP id 14BBD43D2A for ; Sun, 30 Jun 2002 19:21:32 -0300 (EST) Received: from GOOFY (200-206-135-114.dsl.telesp.net.br [200.206.135.114]) (authenticated user ojrmat) by engenho.terra.com.br (Postfix) with ESMTP id 60AFE68245 for ; Sun, 30 Jun 2002 19:21:27 -0300 (EST) From: "Osni Jose Rapelli" To: Subject: RE: [obm-l] livro... Date: Sun, 30 Jun 2002 19:21:02 -0300 Message-ID: <000201c22084$774556c0$7287cec8@GOOFY> MIME-Version: 1.0 Content-Type: multipart/alternative; boundary="----=_NextPart_000_0003_01C2206B.51F81EC0" X-Priority: 3 (Normal) X-MSMail-Priority: Normal X-Mailer: Microsoft Outlook, Build 10.0.2616 Importance: Normal In-Reply-To: <004901c22069$2678c8a0$4f31fea9@emilio> X-MimeOLE: Produced By Microsoft MimeOLE V6.00.2600.0000 Sender: owner-obm-l@sucuri.mat.puc-rio.br Precedence: bulk Reply-To: obm-l@mat.puc-rio.br This is a multi-part message in MIME format. ------=_NextPart_000_0003_01C2206B.51F81EC0 Content-Type: text/plain; charset="iso-8859-1" Content-Transfer-Encoding: quoted-printable =20 =20 Tamb=E9m n=E3o sei mas me interessei muito pela id=E9ia vou procurar = este livro ------=_NextPart_000_0003_01C2206B.51F81EC0 Content-Type: text/html; charset="iso-8859-1" Content-Transfer-Encoding: quoted-printable

 

 

Tamb=E9m n=E3o sei mas me interessei muito pela id=E9ia vou procurar este = livro

------=_NextPart_000_0003_01C2206B.51F81EC0-- ========================================================================= Instruções para entrar na lista, sair da lista e usar a lista em http://www.mat.puc-rio.br/~nicolau/olimp/obm-l.html O administrador desta lista é =========================================================================